100% found this document useful (9 votes)
24K views504 pages

Ssce Hidden Facts in Further Maths

The document is a comprehensive textbook titled 'Hidden Facts in Further Mathematics' by M. A. Otumudia, aimed at improving understanding of Further Mathematics concepts and addressing low grades in the subject. It includes a detailed syllabus covering various mathematical topics, past examination questions, and exercises with answers provided at the end. The book has undergone multiple revisions to enhance its content and examples, particularly in areas such as matrices and coordinate geometry.

Uploaded by

davidlanny10
Copyright
© © All Rights Reserved
We take content rights seriously. If you suspect this is your content, claim it here.
Available Formats
Download as PDF, TXT or read online on Scribd
100% found this document useful (9 votes)
24K views504 pages

Ssce Hidden Facts in Further Maths

The document is a comprehensive textbook titled 'Hidden Facts in Further Mathematics' by M. A. Otumudia, aimed at improving understanding of Further Mathematics concepts and addressing low grades in the subject. It includes a detailed syllabus covering various mathematical topics, past examination questions, and exercises with answers provided at the end. The book has undergone multiple revisions to enhance its content and examples, particularly in areas such as matrices and coordinate geometry.

Uploaded by

davidlanny10
Copyright
© © All Rights Reserved
We take content rights seriously. If you suspect this is your content, claim it here.
Available Formats
Download as PDF, TXT or read online on Scribd
You are on page 1/ 504

HIDDEN FACTS

IN
FURTHER
MATHEMATICS

BY:
M. A. OTUMUDIA BSC MATHEMATICS EDUCATION UNI JOS

UGHELLI

I
© OTUMUDIA PUBLISHERS LIMITED 2017
ISBN:978-978-912-793-2
All right reserved. No part of this book may be reproduced, Stored in a retrieval system, or
transmitted in any form or by any means, recording, photocopying, or otherwise, without the prior
written permission of the publisher.

We are grateful to WAEC & NECO for permission to reproduce questions form past General
Mathematics School Certificate examination papers .

We are also grateful to all those who contributed to the success of this book.

PUBLISHED BY:
OTUMUDIA PUBLISHERS LIMITED
NO 3 ADONOVWE STREET, OFF CENTRAL MOTOR PARK.
P. O BOX 540, UGHELLI NORTH,
DELTA STATE.
08038633394,
08022264074, 08022264072

II
LAGOS STATE OFFICE
28 AWOFESO STREET, BY MR. BIGGS, OFF SHIPELU STREET
PALM GROOVE
SHOMOLU

OSUN STATE OFFICE


REALITY HIGH SCHOOL
(OLUSESI MATHEMATICAL CENTRE)
KAYAFANDA AREA BY AYESO POLICE STATION
ILESHA.

JOS OFFICE:
76/9 DODO STREET
FORMER QUEENS BREAD OFFICE
BEHIND KWARARAFA
JOS.

CONTACT HEAD OFFICE FOR INFORMATION ON LOCAL DISTRIBUTOR


AROUND YOU NOT MENTIONED ABOVE.

III
PREFACE
To boost the numerous efforts by Mathematicians in addressing the perennial problems
of poor understanding and its attendant low grades in the subject, this book covers the
FURTHER MATHEMATICS syllabus directly, including oversight areas

A pool of past questions over the years from FURTHER MATHEMATICS


were painstakingly collected, part of which were used as additional examples to
illustrate elaborately the required concepts on each topic while similar ones were
left as exercises.

Answers and mathematical tables are provided at the end of the text.

PREFACE to 2nd edition 2014


A careful revision was carried out, more examples and exercises were added
in some topics such as matrix, areas under and between curves.

PREFACE to 3rd edition 2017


Coordinate geometry was expanded along with conic section

IV
CONTENTS
CHAPTER ONE CHAPTER SIX
Indices and logarithm 1 Mapping 58
Law of indices 1 Types of mapping 58
Ordinary indices problems 1 Position of mapping 59
Indices in algebraic terms and powers 3 Value of a function 61
Indices involving equations 4 Composition of function 63
Theory of logarithm 9 Inverse of a function 65
Laws of logarithm 9 Joint cases 67
Simplification in logarithm 9
Substitution in logarithm 12 CHAPTER SEVEN
Change of base 13 Binary Operation 69
Logarithmic equation 16 properties of binary operation 69
Common logarithm 20 non tabular binary operation 69
Problems on common logarithm 21 Tabular binary operation 71
Defined binary operation problems 76
CHAPTER TWO
Simultaneous linear, quadratic & cubic equations - CHAPTER EIGHT
and roots symmetric functions 23 AP, GP and sequence 79
Simultaneous linear equation 23 Arithmetic progression (AP) 79
Simultaneous linear equation with fraction 24 Unknown a and d resulting to simultaneous -
Quadratic equations 25 linear equations 81
Factorization method 25 Missing terms 82
Completing the squares and general formula 27 Terms formula for a particular AP 82
Making a given incomplete quadratic equation Sum of a AP 83
A perfect square 28 Tn & Sn cases 85
Radical equations 29 geometric progression (GP) 86
Simultaneous linear and quadratic equations 30 unknown a and r leading to equation 87
Cubic equations 32 sum of ‘n’ terms of a GP 88
Deriving of quadratic equation with known roots 33 sum to infinity 90
Symmetric functions of roots 33 problems on sequence 92
Equation from defined roots 35
Graphical method of solving quadratic equation 37 CHAPTER NINE
Sets 95
CHAPTER THREE Types of sets 95
Inequalities 39 Relationship between sets 96
Linear inequalities in one variable involving fractions 40 Laws of set 97
Inequalities involving quadratic expression 42 Number set 97
Inequalities combination, splitting and range 43 Venn diagram problems 100
Single inequalities number line 44 Venn shading 102
Double inequalities number line 44 Two subsets problems 103

Graphical representation of Linear inequalities - Three subsets problems 105


in two variables 45
System of inequalities 47 CHAPTER TEN
Trignometry 108
CHAPTER FOUR Special angles trig ratios 108
Undefined expressions, expression equals - Derived trig ratios problems 109
zero and limits 49 Trig ratio’s on angles between 0o and 360o 112
Undefined expressions 49 Trig of negatives angles 114
Expression equals zero 50 Identities 114
Limit of a Function 50 Problems of identities 115
Compound & multiple angles 117
CHAPTER FIVE Problems on identities II 118
Remainder and factor theorems & surd 52 Identities and equations 120
Remainder and factor theorems 52 Half angles 122
Remainder case 54 Trignometric graph 123
Surd 55 Graph plotting in trignometry 125
Rationalization of surds 55
Surd and conjugate 56

V
CHAPTER ELEVEN CHAPTER SIXTEEN
Pascal triangle and Binomial theorem 127 Latitude and Longitude 188
Pascal triangle 127 Distance along great circles 190
Problems on Binomial theorem (n is +ve) 128 Distance along small circles 192
Coefficient of term in an expansion I 129 Shortest distance between two points -
Coefficient of term in an expansion II 131 on parallel of latitude 196
Binomial theorem (n < 1) 132 Miscellaneous cases 198

CHAPTER TWELVE CHAPTER SEVENTEEN


Permutation and Combination 133 Circle geometry 201
Factorial 133 Circle theorems 201
Permutation 134 Problems on circle geometry 202
Cyclic permutation problems 134
Permutation of digits 135 CHAPTER EIGHTEEN
Permutation of letters 137 Statistics II (grouped data)
Row arrangement permutation 138 Definition of terms 210
Combination 140 Measure of location & spread 210
Problems on mathematical aspects of factorial, -
permutation and combination 144 CHAPTER NINETEEN
Statistical charts 218
CHAPTER THIRTEEN Bar chart 218
Statistics I (ungrouped data) 147 Pie chart 218
Definition of term 147 Estimation of mode & media from Histogram 219
Measure of central tendency 147 Frequency polygon 224
Basic properties of arithmetic mean 149 O-give 225
Mean of frequency distribution 151
Median 152 CHAPTER TWENTY
Mode 154 Plane geometry 228
Measure of spread 154 Rectangle 229
Range 154 Circle 231
Mean deviation 155 Parallelogram 232
Variance and standard deviation 155 Trapezium 233
Some basic arithmetic properties of variance and - Rhombus 234
standard deviation 156 Length of an arc 235
Problem on variance and standard deviation 158 Perimeter of a Sector 237
Miscellaneous cases involving measure of spread - Area of a Sector 238
and location 159 Length of a chord 239
Perimeter & area of segment 241
CHAPTER FOURTEEN Irregular plane shapes 242

Angles of elevation and depression 161


Single triangle cases in elevation 161 CHAPTER TWENTY ONE
Single triangle cases in depression 163 Solid measuration 245
Double triangle cases 165 Cylinder 245
Cone 248
CHAPTER FIFTEEN Sphere 251
Bearing and element of plane geometry 173 Right Triangle Prism 252
Element of plane geometry 173 Pyramid on a square base 253
Problems on angles between parallel lines 174 Cube 254
Bearing 178 Cuboid 255
Conditions for applying sine rule 179 Frustum 256
Conditions for applying cosine rule 180 Irregular Solids 257
Compass bearing 180
Three digits bearing 181 CHAPTER TWENTY TWO
Relationship between three digits and compass bearing 181 Construction 259
Reversed bearing 182 Foundation facts of construction 259
Problems on bearing 183 Elementary construction of triangle 260
Special angle and locus 262
Parallel lines 264
Line ratio construction 264

VI
Construction of triangle I 265 Substitution Method II 339
Construction of triangles II 267 Trignometric Substitution 341
Construction of triangles with inscribed circle 269 Trignometric integrals 343
Scaled, copied diagram construction 272 Integration by parts 344

CHAPTER TWENTY THREE CHAPTER TWENTY EIGHT


Regression and correlation 273 Partial fractions and integration 345
Scattered diagram 273 Partial fractions 345
Correlation 273 Integration by partial fraction 351
Correlation graph 274
Regression problems 278 CHAPTER TWENTY NINE
Area under curve, volume & trapezium rule 352
CHAPTER TWENTY FOUR Area under a curve 352
Normal distribution 282 Area bounded between two curves 356
Basic facts on normal distribution 282 Volumes 361
Problems on normal distribution 283 Trapezium rule 362

CHAPTER TWENTY FIVE CHAPTER THIRTY


Matrix 288 Composition and resolution of force 363
Types of matrix 288 Composition of force 363
Addition and Subtraction of matrix 288 Problems on resultant of two forces 363
Scalar multiplication of matrix 289 Problems on angles between two forces 365
Multiplication of matrix 291 Resolution of forces 366
Cases of equations on multiplication, addition - Resultant of several concurrent forces 367
and subtraction 292
Joint Cases of addition, subtraction and multiplication 294 CHAPTER THIRTY ONE
Transpose 296 Equilibrium of forces 372
2 x 2 Matrix determinant 297 Problems on three forces in equilibrium 372
3 x 3 Matrix determinant 300 Triangle of forces 376
2 x 2 Inverse of matrix 302 Moments about a point 378
Solving simultaneous Linear equation with Basic terms in moments 374
two unknown using 2 x 2 matrix 303 Problems on moments 379
3 x 3 inverse of matrix 304
Difference between cofactors & determinant 307
Crammer’s Rule 307 CHAPTER THIRTY TWO
Forces acting at inclined or horizontal planes 382
CHAPTER TWENTY SIX Definition of terms 382
Differentiation by first principle 309 Problems on force along rough horizontal -
Basic rules in differentiation 311 plane 382
Problems on chain rule 313 Problems on force along rough inclined -
Problems on product rule 317 plane 383
Problems on product and chain rule 318 Problems on smooth planes 385
Quotient rule 319
Quotient and chain rule problems 319 CHAPTER THIRTY THREE
Gradient at a given point 320 Motion 387
Turning Points 322 Definition of terms 387
Value of x for max or min point 322 Problems on velocity time graph 388
Value of y for max or min point 324 Straight line and ground surface motions 390
Value of x, y for max or min point 326 Problems on upward (vertical) motion 392
Implicit functions 327 Problems on downward (vertical) motion 395
Implicit functions and gradient at a point 328 Application of differention to motion 396
Comparing rates of change 329 Application of integration to motion 398
Curve sketching 330
CHAPTER THIRTY FOUR
CHAPTER TWENTY SEVEN Interaction between forces and motion,
Integration 332 collision and projectiles 399
Indefinite integrals I 332 Interaction between forces and motion 399
Indefinite integrals II 333 Collision 402
Definite integrals I 335 Projectiles 404
Definite integrals II 336

VII
CHAPTER THIRTY - FIVE
Vectors 407 Coordinate geometry II(circle) 475
Scalar, vector product 407 Problems on equation of a circle 475
Addition of vectors 408 Equation of a circle through 3 points 477
Other basic terms of vectors 408 Problems on circle equation’s condition 477
Sum, subtraction and scalar multiplication in vectors 408 Equation of circle with 2 opposite points -
Problems on parallel vectors 409 of it’s diameter and others 478
Problems on projection, equal & angle between - Equation of a tangent to a circle 479
vectors 410 Length of a tangent to a circle 481
Problems on magnitude, dot product of vectors 411 Conic and it’s terminologies 482
Problems on perpendicular condition of vectors 412 Parabola 482
Problems on unit vectors 413 Hyperbola 484
Problems on joint cases of vectors properties 414 Ellipse 485
Problems on position vectors 417
Problems on vectors and plane shapes 420
Problems on vectors and coordinate geometry 422
Problems on vectors and motion 423 TABLES 490
Problems on vectors and forces 424 ANSWERS 493

CHAPTER THIRTY SIX


Probability I and logic 426
Probability I 426
Definition of basic term 426
General formula problems 426
Theorical Probability 428
Empirical (Experimental) Probability 429
Addition rule 430
Multiplication rule 431
Problems on Probability 432
Logic 439
Construction of truth table 440

CHAPTER THIRTY SEVEN


Plane geometry 442
Triangles 442
Quadrilaterals 447

CHAPTER THIRTY EIGHT


Probability II 450
Binomial poisson and approximation of binomial -
to normal distributions 450
Binomial Probability distribution 453
Approximation of binomial to normal distribution 454

CHAPTER THIRTY NINE


Coordinate geometry and conic 456
Distance between two points 456
Mid-points 457
Collinear points 459
Equations of straight lines 461
Gradients 464
Angle of slope 465
Gradient and parallel condition 466
Gradient and perpendicular condition 467
Perpendicular form of linear equation 469
Tangent and normal 469
Problems on tangents 470
Problems on normal 473

VIII
Chapter one
Indices and logarithm = (64r –6)1/2 1/3

Indices = (64r – 6)1/6


Laws of indices The expression shows that 64 can be expressed in powers of
1. xa × xb = xa + b but xa × yb  xa + b 6 as well. Since r is already in power of 6.
2. xa  xb = xa – b = (26 r – 6 )1/6
OR xa = xa – b = 26 × 1/6 r – 6 × 1/6
= 26/6 r – 6/6
xb
0 = 21 r – 1
3. x = 1
= 2 (D)
4. x – a = 1 and r
xa 1990/8 UME
1 a Simplify 160r2 + 71r4 + 100r8
 y x 1 1
1
   Also   x most common x 
a

x y  x x A.9r2 B.12 3r C.13r D. 13r


a b
5. ( x ) = x ab Solution
Do not get frightened by the mere sighting of a question.
xa a x
1
6. Just stick to the principles and you will see the question

7. xa 
b
 x
a
b crumbling like a piece of cake.

These are the basic laws of indices; they will be applied 160r2 + 71r4 + 100r8
in the problems that follow. 100r8 = (100r8)1/2
Starting from the inner - most term. = 102/2 r 8/2
= 10r4
Ordinary indices problem
Here, we are only required to apply the indices laws to
number problems = 160r2 + 71r4 + 10r4
We add the values under one square root,
1988/8 UME
Evaluate 81/3  52/3 This operation is different from 3 + 2 in which
102/3 case, we don’t add
2 5
A. /3 B. /3 C. 3 5 D. 3 2
= 160r2 + 81r4
Solution
Out of the numbers given, 8 can break further, 10 will
= 160r2 + 9r2
do the same,
81/3 × 52/3 = (23)1/3 × 52/3 = 169r2
102/3 (2×5)2/3 = 13r (C)
= 23/3 × 52/3
22/3 × 52/3 1991/8 UME
What is the product of 27 , (3) – 3 and 1 –1
?
When denominators move to numerator they gain
5 5
negative power. A.5 B.3 C.1 D 1
= 21 × 2– 2/3 × 52/3 × 5– 2/3 25
Recall addition of powers when they are in same base Solution
= 21– 2/3 × 5 2/3 – 2/3 i.e 27 × (3) – 3 × ( 1/5 ) – 1 = 27 × 1 × 5
3 2
= 0 5 5 33
2 3  5 = 27 × 1 × 5
= 21/3 × 1 5 27
= 21/3 All terms will cancel out
= 3 2 (D) = 1 (C)
1989/5 UME
Simplify 3 (64r – 6 )1/2 1995/6 UME
Evaluate (81)3/4 – (27)1/3
A. r B.2r C. 1 D. 2
3 × 23
2 2r r
A.27 B.1 C.1 D.1
Solution 3 8
Continuous practice gets us use to the laws application
Solution
Recall that 81 = 9 × 9 and 27 = 3 × 9
3 (64r – 6 )1/2 = 32 × 32 i.e 34 = 31 × 32 i.e 33
1
1994/8 PCE
Thus (81)3/4 – (27) 1/3
Simplify (0.2)3 × 8
3 × 23
= ( 34 )3/4 – ( 33 )1/3 (0.4)2
3 × 23 A.40 B.0.4 C.0.2 D.0.04
4 3 31 Solution
= 3 4 3 3 (0.2)3 × 8 = ( 2/10 )3 × 8
3  23 (0.4)2 ( 4/10 )2
= 33 – 31 = 8 × 8  16
3 ×23 1000 100
= 27 –3 = 24 i.e 1 (B) = 8 × 8 × 100
3×8 24 1000 16
= 4 i.e 0.4 (B)
2002/2 UME 10
Without using tables, evaluate 1998/10 PCE
(343)1/3 × (0.14) – 1 × (25) – 1/2
A.12 B.10 C.8 D.7 ( 3 3 )2/3 + (272/3 × 9–1/2)
Solution A.6 B.3 C.1 D.1
When faced with a big number like 343 to be expressed 3 6
in power of 3 to enable us clear the cube root, we try the Solution
elementary method. (3 3 )2/3 + (272/3 × 9–1/2)
Its prime factors starting from 2,3,5,7… seven can go = 32/3 × (31/2)2/3 + ( 33(2/3) × 32 (–1/2) )
7 343 = 32/3 × 32/6 + 36/3 ×3–2/2
7 49 = 32/3 × 31/3 + 32 × 3–1
7 7 = 32/3 + 1/3 + 32–1
1 = 33/3 + 31
Also (0.14) – 1 = 14 – 1 = 100 = 31 + 31
100 14 = 6 (A)
Thus, (343)1/3 × (0.14) – 1 × (25) – ½
2001/6 PCE Exercise 1.1
= (73)1/3 × 100 × 1 If 182/3 × 121/3 = 2m × 3n
14 (52 ) ½ Find the value of m + n
= 7 × 100 × 1 A.1 B.2 C.3 D.4
14 5 = 10 (B)
Exercise 1.2
1989/14 UME Simplify the expression
Find the value of the expression 21/2 × 31/2 × 61/2 × 81/3 × 271/3
32x3 – 64x2 – 81x – 3 When x = – 3/4 A.36 B.27 C.24 D.6
81 27 16
A.101/2 B.101/6 C.33/8 D. – 131/2 2001/2 UME Exercise 1.3
Solution
Wherever we see x; we shall put – 3/4
Simplify  64a 
3 3
1

32 x3 – 64 x2 – 81 x – 3 A.4a B.1 C.8a D. 1


81 27 16 8a 4a
= 32 –3 3 – 64 –3 2 – 81 –3 – 3
81 4 27 4 16 4 2005/25 PCE Exercise 1.4
Odd powers minus sign remain the same. Simplify 27 – 2/3 × 251/2 × 50
Even powers minus sign will change to plus. A.9 B.5 C. 9/5 D. 5/9

= 32   27   64  9   81   64  2006/44 UME Exercise 1.5


81  64  27  16  16  27 
Simplify 25  273  121 2  625
 12 1
1  14
Reducing the terms
= 1 –1 – 4 1 – 3 – 4
A.14 B. 9 C. 3 D.34
3 2 3 1 1 1
5 11 275 55
= – 1 – 4 + 12
6 3
2006/21 PCE Exercise 1.6
= –1 – 8 + 72 i.e 72 – 9
Simplify (0.8)3 × (0.05)2
6 6
= 63 i.e 103/6 0.32
6 = 101/2 (A) A. 0.008 B. 0.042 C. 0.003 D. 0.004
2
Indices in algebraic terms and 1994/7 PCE
algebraic powers Simplify the expression
Here our knowledge of applying indices laws come to 1 × q–k/3
–4k/3
play. Of course our ability to solve fractions is not left q
out. A. q5k/3 B. qK C. q–k D. q–5k/3
Solution
2003/18 Neco Applying laws of indices
Simplify 4(2n + 1) – 2 n + 2 = q4k/3 × q–k/3
2n + 1 – 2n 4k
 k 4k  k
A. 2 n+2
B. 2n + 1 C. 2n D. 2 E. 4 = q 3 3
q 3

Solution 3k

First let us disarm the indices as: = q 3


i.e qk (B)
4(2n + 1) – 2n+ 2 = 4(2n × 21) – 2n × 22
2 n+1
– 2 n
2n × 21 – 2n 1999/8 PCE
= 4(2n  2) – 4(2n)
2n (2 – 1) If x4
3
 y y
x
5
8
= xm yn, find nm
C. 2¾
n
We can factor 4 × 2 at the numerator A. 32 – 1 B. 3 – 2 D. 3
= 4  2n(2 – 1) ¾ ¾
24/3 2 3 2
2n
= 4 (E) Solution
¾
= x y½ y5/8
2005/7 (b) Neco

Simplify 3(5n + 1) + 5n
5n – 2 – 5n – 1 = x¾ (y½ × x –½ )y5/8
Solution
= x¾ × x –½ × y½× y5/8
Factorizing out 5n from numerator and denominator
3(5n + 1) + 5n = 5n [(3×51 + 1)] ¾ – ½ ½ + 5/8
= x y
5n – 2 – 5n – 1 5n (5-2 – 5–1) 32 45
n
5 will cancel out = x 4
y 8

= 15 + 1 ¼
= x y 9/8
1
/25 – 1/5
Compare what we have to xm yn
=  16 m = ¼ and n = 9/8
4 m
Next, we find n
25
Changing division to multiplication nm = ( 9/8)¼
 25  = ( 32 )¼
=  16  
 4 ¼
( 23 )
= – 100
= 32/4 i.e 3½
1990/11 UME
Simplify 1  1 –1 2¾ 2¾ (D)
x –1 y –1
1984/14 UME Exercise 1.7
A. x B. xy C. y D. (xy) –1 Simplify 3n – 3n – 1
y x 33 ×3n – 27 ×3n – 1
Solution A. 1 B. 0 C. 1/27 D. 3n – 3n – 1 E. 2/27
Starting from the bracket,
1 1
 1 1  1 1 1979/19 UME Exercise 1.8
 1  1   1 1 Simplify 5x × 25x– 1
x y   
x y 125x+ 1
2x – 1
= ( x  y) –1 A. 5 B. 5 x + 2 C. 5 – 5 D. 5 x + 1 E. 53

= x –1 1995/5 PCE Exercise 1.9


y Simplify ( 2 2x )3 ( 3 3x )2
= y 66x
x (C) A. 2 B. 1 C. 2/3 D.3x/ 2x
3
Indices involving equations Equating powers
x=2
There are cases where the resulting indices of a given
2 3 x=4
problem become equation. These equations are
3 (B)
sometimes:
- simple linear or simultaneous linear
1995/1 (Nov)
- quadratic in nature
If 81 – 2x = 1 , find x
Whatever the equation may be, we are required to solve
4x
accordingly.
A ¾ B /3 2
C 3/ 8 D – 3/ 4 E – 3/2
Simple linear cases Solution
1997/2 Representing the indices properly
81– 2x = 1 , becomes
If
3
42 y = 8, find y. 4x
A. ¼ B. ½ C. ¾ D. 9/4 E. 9/2 8 1 – 2x
=4 –x

Solution Expressing the indices in base 2


Presenting the indices properly 23(1– 2x) = 22(–x)
Since they are in same base, we equate powers
3
42 y = 8 becomes 3(1– 2x) = 2(–x)
4  2y
1
3
= 8 3 – 6x = –2x
2y 3 = –2x + 6x
i.e 4 3 = 8 3 = 4x
Expressing both terms indices in base 2 ¾ = x (A)

2
  2
2y
3 2003/8 ( i ) Neco
= 23
Since both sides are in same base, we can equate powers
Solve for x in the equation given as
10x  5(2x – 2)  4(x – 1) = 1
 2y 
2  = 3 i.e 4 y = 3 Solution
 3  3 Of course 10 is same as 2 × 5 and 4 is same 22.
Cross multiply to get So let us change the bases to 2 and 5
4y = 9 (2 × 5)x × 52x – 2 × 22(x – 1) = 1
y = 9 /4 (D) 2x × 5x × 52x – 2 × 22x – 2 = 1
Collect like bases together
2x × 22x – 2 × 5x × 52x – 2 = 1
2004/46 UME
22x – 2 + x × 52x – 2 + x = 1
Given that 4  16 , find the value of x
3 2x
23x – 2 × 5 3x –2 = 1
A.4 B.6 C.3 D.2
Since we can not take cube root of 42x, take cube of i.e 23x × 53x = 1
both sides to remove the cube root. 22 52
42x = (16)3 Cross-multiply to clear fractions
Express both sides in the lowest base power 23x × 53x = 22 × 52
42x = 46 Regrouping
Equating powers (2 × 5)3x = (2 × 5)2
2x = 6 We can equate powers as:
x = 6 i.e 3 ( C ) 3x = 2
2 x = 2/ 3

1979/49 UME 1992/1 (Nov)


If 52x – 4 – 1 = 0, solve for x
If 3  x 3
9 then the value of x is A 4.5 B 2.5 C 2.1 D 2.0 E 1.0
A.3 B.4 C.1 D.2 E. 1 Solution
4 3 3 3 2 Any number raise to power zero is 1, thus
Solution 52x – 4 – 1 = 0 becomes
3  52x – 4 – 50 = 0
1 1
x 2
 93
52x – 4 = 50
To enable us equate powers, we express both sides in
Since they are in same base, we equate powers
same base
2x – 4 = 0
 
x 1

3  32
2 3
2x = 4
x 2 x = 4/2 ie 2 (D)
3  33
2

4
1996/5 42x = 2  ( 22 )2x = 2
x–1
Solve for x : 272x = 1 (3 )
43x ( 22 )3x
9
A – 3 /5 B – 1/7 C 3/7 D 2/3 E 7/4 24x = 2
Solution 26x
Representing the indices properly 24x × 2– 6x = 21 yes, 2 is same as 21
272x = 1 (3x – 1) becomes 24x – 6x = 21
9 2–2x = 21
272x = 3– 2(3x – 1) Since they are in same base, we equate powers
Expressing all the indices in base 3 – 2x = 1
33(2x) = 3x–1– 2 x = – 1/2 (B)
Since they are in same base, we equate powers
3(2x) = x –1 – 2
6x = x – 3 2000/7 PCE
6x – x = – 3 If 52x+1 + 52x = 150, Find the value of x
5x = – 3 A.4 B.3 C.2 D.1
x = – 3/5 (A) Solution
52x+1 + 52x = 150
1993/13 Nov Factor out common term on the LHS
If 33x – 8 – 1 = 0 find x 52x (51 + 1) = 150
x–2
3 52x (6) = 150
A –3 B 5/ 2 C 3 D 5 52x = 150 i.e 25
Solution 6
Presenting the indices properly, we have 52x = 52
33x – 8 – 3– (x – 2) = 0 Equating powers
Re – arranging 2x = 2
33x – 8 = 3– (x – 2) i.e x = 1 (D)
Since both sides are in same base, we equate power
3x – 8 = – (x – 2) 1993/4
3x – 8 = – x + 2 Solve for y in the equation
Collect like terms together 10y × 5( 2y – 2 ) × 4 ( y – 1) = 1
3x + x = 2 + 8 A. 3 B. 2 C.1 D. 5
4x = 10 4 3 4
x = 10/4 Solution
= 5/2 (B) 10y × 5(2y – 2) × 4 (y– 1) = 1
(2 ×5)y × 5(2y – 2) × 2 2 (y – 1) = 1
2004/36 2y × 5y × 5(2y – 2) × 22y– 2 = 1
Solve the equation Collect like base together
82x – 1 = 1 2y × 22y – 2 × 5y × 52y – 2 = 1
512 2y +2y – 2 × 5 y +2y – 2 = 1
A. 4 B. 1 C. –1 D. –2 23y – 2 × 53y – 2 = 1
Solution 23y × 53y = 1
Express both indices in same base
82x – 1 = 8– 3
22 52
Equating powers 2 × 53y = 1
3y

2x – 1 = –3 4 25
2x = –3 + 1 23y × 53y = 1
2x = –2 100
x = – 2/2 i.e – 1 (C) 103y = 1
100
103y = 100
1992/4 UME 103y = 102
What is the value of x satisfying the equation Equating powers
42x
= 2? 3y = 2
43x y = 2 (B)
A. – 2 B. – 1/2 C.1/2 D. 2 3
Solution
Expressing all terms in powers of 2

5
Simultaneous linear equations in Nature Quadratic in nature
2000/3 (Nov) 2000/1(c) Neco
Solve for x and y in the equation Solve the equation
2x + 3 y = 9 42x– 3 – 3(4x– 2) – 1 = 0
2x + 2 – 3y + 1 = 8 Solution
Leave your answer to 3 decimal places 42x–3 – 3(4x– 2) –1 = 0
Solution First, we seek to clear the minus powers as follow:
Presenting the indices properly to reflect 2x and 3y 42x  4x 
2x + 3y = 9  3 2  –1 = 0
43 4 
4(2 ) – 3(3y) = 8
x

Next let 2x = p and 3y = q To clear fraction we multiply by the highest power i.e 4 3
p + q = 9 -------- (1) 42x – 3 × 41(4x) – 43 = 0
4p – 3q = 8 ------- (2) Next, we re – arrange as
Multiply (1) by 3 and add (4x)2 – 12(4x) – 64 = 0
3p + 3q = 27 Let P = 4x
4p – 3q = 8 P2 – 12P – 64 = 0
7p = 35 Factorizing
P = 35/7 ie 5 P2 – 16P + 4P – 64 = 0
Substituting P = 5 into (1) P(P – 16) + 4(P – 16) = 0
5 + q = 9 becomes (P – 16)(P + 4) = 0
q= 9–5 P – 16 = 0 or P + 4 = 0
q=4 P = 16 or – 4
Changing back to 2x and 3y Substituting for P value
2x = p and 3y = q becomes P = 4x becomes
2x = 5 and 3y = 4 16 = 4x
Changing from indices to log Changing to same base
x = log25 and y = log 3 4 4 2 = 4x
Next we change base as: Thus 2 = x
As for P = – 4, it does not apply here.
x  log25 = log105 and y  log 3 4 = log10 4
log10 2 log10 3
x = 0.6990 and y = 0.6021 2005/7 (a) Neco
0.3010 0.4771 Solve for x if 8ex + 15e– x = 26
x = 2.322 and y = 1.262 to 3 decimal places Solution
We cannot work directly with negative power here i.e e–x
1986/25 UME Thus, 8ex + 15e– x = 26 becomes
If 5(x+2y) = 5 and 4(x+3y) = 16, find 3(x+y) 8ex + 15 = 26
ex
A.0 B.1 C.3 D.27
Clearing fraction at LHS
Solution 8e2x + 15 = 26
This problem is simultaneous equations in nature.
ex
5(x+2y) = 51 Cross-multiply to clear fraction
Equating powers 8e2x + 15 = 26ex
x + 2y = 1 ……..(1) Re – arranging
Also 4(x+3y) = 16 8(ex)2 – 26ex + 15 = 0
4(x+3y) = 42 Let P = ex
Equating powers 8P2 – 26P + 15 = 0
x +3y = 2 ………….(2) Factorizing
Solving (1) and (2), 8P2 – 20P – 6P + 15 = 0
x + 3y = 2 4P(2P – 5) – 3(2P – 5) = 0
– ( x + 2y = 1 ) (2P – 5) – (4P – 3) = 0
y=1 2P – 5 = 0 or 4P – 3 = 0
Substitute y = 1 into (1) 2P = 5 or 4P = 3
x +2y = 1 will become P = 5/2 or ¾
x + 2(1) = 1 Thus, ex = 5/2 or ¾
x=1–2 Changing from indices to logarithm ie ln
x =–1
x = ln 5/2 or ln 3/4
Next we find 3(x+y)
x = ln 5 – ln 2 or ln3 – ln4
3(x + y) = 3(– 1 + 1 ) 0.916 or -0.288
= 30 i.e 1 (B)
6
2007/10 1996/2 (Nov)
Solve 9e2x – 30ex + 25 = 0 Solve for x : 22x + 3 – 129(2x) + 16 = 0
A. eln5 – ln3 B. e ln5/3 C. ln3/5 D. ln5 – ln3 E. eln5 + ln3 Solution
Solution Nothing is common, so we express in 2x and (2)2
Showing the quadratic equation properly to show the quadratic expression properly
9e2x – 30ex + 25 = 0 becomes 8(2x)2 – 129(2x) + 16 = 0
9(ex)2 – 30ex + 25 = 0 Let P = 2x
Let P = ex 8P2 – 129P + 16 = 0
9P2 – 30P + 25 = 0 Factorizing Factorizing
9P – 15P – 15P + 25 = 0
2
8P2 – 128P – P + 16 = 0
3P(3P – 5) – 5(3P – 5) = 0 8P (P – 16) – 1 (P – 16) = 0
(3P – 5) (3P – 5) = 0 (P – 16)(8P – 1) = 0
3P – 5 = 0 twice P – 16 = 0 or 8P – 1 = 0
3P = 5 twice P = 16 or 8P = 1
P = 5/3 twice P = 16 or 1/8
x
Changing back to e Substituting for P value
e x = 5/ 3 2x = 16 or 2x = 1/8
From indices to ln Expressing both sides to base 2
x = ln5/3 2x = 24 or 2x = 2– 3
= ln5 – ln3 (D) Since they are in same base, we can equate powers
2007/35 Neco x = 4 or – 3
Solve for such that 2 ( 2 x
2
 15 x  7 )
=1 1997/9 (a)
Solve for x : 32x + 1 – 4(3x) + 1 = 0
A. (1, 2) B. (–1, –2) C. (–7, – 1/2)
D. (–2, 7) E. (–7, 1/2) Solution
Noting is common so we express the terms in 3x and (3x)2 to
Solution
show quadratic properly
From indices rule
2 3(3x)2 – 4(3x) + 1 = 0
2 ( 2 x 15x  7 ) = 1 is same as Let 3x = P
3P2 – 4P + 1 = 0
2
2 ( 2 x 15x  7 ) = 20
Equating powers, we have Factorizing
2x2 + 15x + 7 = 0 3P2 – 3P – P + 1 = 0
Factorizing the resultant quadratic equation 3P(P – 1) – 1(P – 1) = 0
2x2 + 14x + x + 7 = 0 (P – 1)(3P – 1) = 0
2x(x + 7) + 1(x + 7) = 0 P – 1 = 0 or 3P– 1 = 0
(x + 7)(2x + 1) = 0 P = 1 or 1/3
x + 7 = 0 or 2x + 1 = 0 Substituting for P value
x = –7 or 2x = –1 3x = 1 or 3x = 1/3
x = –7 or – 1/2 (C) i.e 3x = 30 or 3x = 3–1
1992/9a (Nov) x = 0 or –1
Solve the equation 3(22x + 3) – 5(2x + 2) – 156 = 0 2000/36 (Nov)
Leaving your answer in logarithm form. Solve the equation
2
Solution 2( x 24) = 4x
Applying the law of indices
A 6 or 4 B – 6 or – 4 C 6 or – 4 D – 6 or 4
3 × 22x ×23 – 5×2x ×22 – 156 = 0
Solution
Divide through by 4 ie 22 is common
Expressing both sides indices in base 2
3×22x × 21 – 5×2x – 39 = 0 2
Arranging them properly 2( x 24) = 4x
6(2x)2 – 5(2x) – 39 = 0 Expressing both sides indices in base 2
x
Let 2 = P 2

6P2 – 5P – 39 = 0 2( x 24) = (22)x same as 22x


Factorizing Since both sides are in same base, we equate powers:
6P2 – 18P + 13P – 39 = 0 x2 – 24 = 2x
6P(P – 3) + 13(P – 3) = 0 Re – arranging
(P – 3)(6P + 13) = 0 x2 – 2x – 24 = 0
P – 3 = 0 or 6P + 13 = 0 Factorizing
P = 3 or – 13/6 x2 – 6x + 4x – 24 = 0
Substituting for P = 2x x(x – 6) + 4(x – 6) = 0
2x = 3 or 2x = – 13/6 (x – 6)(x + 4) = 0
We can not raise the RHS to base 2; so changing indices to log x – 6 = 0 or x + 4 = 0
x = log23 x = 6 or – 4 ( C )
7
1999/9a (Nov) Exercise 1.10 2003/7 Exercise 1.21
Solve the equation: 1
32x + 2 – 10(3x + 1) + 9 = 0 Given that 23y
= 2 y + 2, find y
8
1 7 1
2002/38 (Nov) Exercise 1.11 A B C 1 D 1
Solve the equation 5 8 5
9(1 – x) = 1 x+1
27 2003/20 Exercise 1.22
A. – 5 B. –1 C. 1 D. ½ Simplify 8n × 22 n  43 n
A 2 –n B 21 – n C 2n D 2 n+1
2005/10b Exercise 1.12
Solve the equation: 32x – (10)3x + 9 = 0 1996/5 PCE Exercise 1.23
If 5 = 125 and 34y – x = 243, Find the value of 2x – 2y
x– y
2009/2 Neco Exercise 1.13 A.225/3 B.21/3 C.2 –1/3 D.2 –25/3
Solve 4(3x+1) – 32x = 27
A –1 or 2 B 1 or 2 C –1 or –2 1991/10 PCE Exercise 1.24
D 1 or –2 E 1 or 3 Find the value of x in
8–1 x 2(2x+1) = 64
2001/18 Neco Exercise 1.14 A.1 B. 2 C. 3 D. 4
1

 625  4
  × 3
125 p3q 3r  6 simplified is 2002/4 PCE Exercise 1.25
 1296  If 1 of 64x = 16x– 1 find the value of x.
A qr2 B 5p C 25p D 5p E 6p 2
2 2
6p 6qr 6qr qr qr2 A. –2 B. – 3 C. –1 D. – 2
2 3
2009/4(Nov) Exercise 1.15
Solve the equation 22y + 1 – 15(2y) – 8 = 0. 2003/36 PCE Exercise 1.26
A–2 B2 C3 D4 If 5 × 25x+1 = 125x, find the value of x.
A.4 B.1 C.2 D.3
2009/15(Nov) Exercise 1.16
1 1985/18 Exercise 1.27
Solve  x2  = 81(1 – x) If 32y – 6(3y) = 27, find y
27
A.3 B. –1 C.2 D. –3 E.1
1 2
A–1 B – C – D 10 1990/5 PCE Exercise 1.28
2 7 What is the value of y if
2009/17 Exercise 1.17 32y – 10 × 3y = – 9
A.1 or 9 B.3 or –3 C.0 or 2 D. 1or 3
Simplify 216 3 × 0.16 2
2 3

A 125 B 2 C 4 D 2 1997/6 PCE Exercise 1.29


288 125 225 225 Solve for x in the equation 32x – 12 (3x) + 27 = 0
A.3, 9 B.2, 1 C. –3, 9 D. –1, 2
2010/4 Exercise 1.18
1 2007/6 PCE Exercise 1.30
If 16 3x = ( 32x – 1 ), find the value of x. Solve the equation 125(x + 3) = 5
4
A. – 8/3 B. – 4/3 C. 8/3 D. 3
1 3 5
A–1 B– C– D–
3 7 19

2003/1 Exercise 1.19


x 3 3
2 8 4
Solve the equation 8 = (2 ) (4 )
3 3 4 5
A B C D
8 4 5 4
2003/3 Exercise 1.20
2
 1  3
Simplify  
 64 
1 1
A. – 4 B. – C. D. 4
4 8
8
Theory of logarithm (3) logbNk = k logbN
This aspect of logarithm deals with applying: Examples on Simple applications
(a) the relationship between indices & logarithm and (a) log10100 = log10102
(b) the laws of logarithm to solve problems. = 2log1010
Here we do not use logarithm tables. (b) log7343 = log773
= 3log77
Relationship between indices and logarithm.
If bx = N Then, (c) ½ log525 = log525 ½
x = logb N Special cases under law (3)
Note: Candidates should observe that both indices and
logarithm make use of the same base when we are (i) Logbb = Logbb1
converting from one form to another. = 1Logbb
Examples on simple Applications =1
i.e logbb = 1
(1) Given that 102 = 100, Stated as: Any logarithm to its base is one.
rewrite the expression in logarithm form. Examples
Solution (a) log1010 = 1
If 102 = 100
Then 2 = log10 100 (b) log22 = 1
(c) log66 = 1
(2) If 26 = 64,
rewrite the expression in logarithm form ( ii ) logb1 = logbb0 (Since b0 = 1)
Solution = 0 logbb
26 = 64 =0
Then 6 = log2 64 i.e. logb1 = 0
Stated as: logarithm of one to any base is zero.
(3) Express 4x = 256 in logarithm form Note:
Solution The examples given above were not fully solved,
If 4x = 256 It was deliberately done to allow for step by step
Then x = log4256 comprehension of the stated concepts (principles).
For better understanding, logarithm problems will be treated
2005/29 PCE Exercise 1.31A under subheadings:
Given that log81 S = 1 , What is S ?
2 Simplification in logarithm
A. 41.0 B. 27.0 C. 9.0 D. 3.0
1996/10 (Nov)
Evaluate log 2  94 
LAWS OF LOGARITHM 3

A. 2 B. 2/3 C. 1 D. 0 E. –2
(1) logb (mn) = logbm + logbN
Solution
Examples on simple applications Pretend as if you are not seeing a fraction,
(a) log630 = log6(6×5) rather terms such as log b b
= log66 + log65 Then log 2  94  = log 2  23 2
3 3

(b) log272 = log2 (8×9) = 2Log 2 2


3
3
= log28 + log29 = 2 (A)
(c) log39 + log327 = l0g3(9×27) 2003/3 (Nov)
= log3243 Evaluate log 2 9
4
3
and not log39 + log327 = log3 (9 +27) A. 2 B. 2/3 C. – 2/3 D. – 2
Solution
(2) logb M = logbM – logbN Pretend not to see the fraction but treat as a whole in terms of log2
N 4– 1 .
log 2  23 
2
Examples on simple applications Thus, log 2 9
4
=
(a) log10 100 = log10100 – log103 3 3

3 =  2 log 2 2
3
3
(b) log6 36 = log636 – log617 = – 2 ( D)
17 1994/2
(c) log381 – log39 = log3 81 Evaluate, without using tables
9 Log101.44 – log10 90 + log10 0.0625
A –3 B –2 C –1 D 0.0001 E 0.001
9
Solution Solution
Applying logarithm rules Applying the laws of log at top and under differently, since
Log10 1.44 – log10 90 + log10 0.0625 becomes they are in same base,
= Log10 1.44 × 0.0625 = log10 ( 8/4 )
90 log10 ( 4/2 )
= log10 0.001
= log1010–3 = log102
= –3 Log1010 log102
= – 3 (A) = 1 (E)

1994/6 (Nov) counter example 1994/11 PCE


Simplify log 49 ÷ log 343 Without using tables, evaluate
A.1/7 B. 2/3 C. 7 D. log 1/7 E. –1 Log24 + log42 – log255
Solution A. 2 B. 1/5 C. ½ D. 0
Log 49 ÷ log 343 = Log 49 Solution
Log 343 A critical observation shows
= Log 72 Log24 + log42 – log255 = log222 + log44 ½ – log2525 ½
Log 73
= 2 log22 + ½ log44 – ½ log2525
= 2 Log 7 = 2/3 (B)
Recall that logarithm of any number to its base is one
3 Log 7
=2×1+½×1–½ ×1
2004/15 (Dec) Neco =2+½–½
Simplify log2 
 27  – log  9  + log  1  = 2 (A)
 2  2  1991/9 UME
 4  8  24 
A. –5 B. – 4 C. –3 D. –2 E. – 1 Simplify 2log 2 – log 72 + log9
Solution 5 125
Applying logarithm laws, the expression becomes A. 1– 4log3 B. –1 + 2log3 C. –1 + 5log2
 27 9 1  D. 1– 2log2
= log 2     Solution
 4 8 24 
Applying the laws of logarithm,
 27 8 1  = log (2/5)2 – log 72 + log9
= log 2    
 4 9 24  125
Any logarithm problem without base is automatically a base 10 logarithm.
 1
= log 2  
 4 = log10 4 ÷ 72 × 9
= log2 2 –2 25 125
= –2 log 2 2
= – 2 (D) = log 10 4 × 125 × 9
25 72
Canceling out terms, we have
1979/42 UME = log10 5
What is log7(49a) – log10 (0.01) ? 2
A. 49a B. a/2 + 2 C.72a + 2 = log105 – log102
D.2a + 2 E.2a/2 = log10 10 – log102
Solution 2
We observe that the two items are not of same base; = log1010 – log102 – log102
= log7 [ ( 72 )a ] – log10 (1/100) = 1 – 2 log 2 (D)
–2 Whether we show the base 10 or not, it is still base 10
= log77 – log1010
2a

= 2a log77 – (–2)log1010 1993/7 UME


= 2a + 2 (D) Solve without using tables
Log5 (62.5) – log5 ( 1/2 )
A.3 B.4 C.5 D.8
1982/16 UME Solution
Simplify log108 – log104 Simplifying and applying the log laws,
= log5 625 – log5 ( 1/2)
log104 – log102
10
A.log102 B.log108 C.0 D.log104 E.1 Since they are in same base
10
= log5 625 ÷ 1 2003/36 (Nov)
10 2 If log10 x2 ≤ 2 + log10 y, express y in terms of x.
= log5 625 × 2 A y  100 B y≤
x2 C y
x2 D y≤
10
10 1 x2 10 100 x2
Cancel out terms, Solution
= log5 125 Applying laws of logarithm
= log553 log10 x2 ≤ 2 + log10 y becomes
= 3log55 log10 x2 ≤ log10102 + log10y
= 3 (A) log10 x2 ≤ log10 102 × y
1994/3 UME Equating terms
x2 ≤ 100y
Evaluate log5 (0.04)
Making y the subject formula
log318 – log32 x 2 (C)
i.e y 
A. 1 B. –1 C. 2/3 D – 2/3 100
Simplify by applying log laws, 1996/9
= log5 ( 4/100) If log5 (y2) = log5 (3x) + 2, express y in terms of x
A. y2 = 5x B. y = 75x C. y2 = 75x
log3 18 – log32 2
D.y = 6x E. y = 5x
log 5 251 Solution
=
log 3 182 Applying laws of logarithm
Log5(y2) = log5(3x) + 2 becomes
= log55 – 2 Log5(y2) = log5(3x) + log552
log39 Log5 (y2) = log5 (3x) + log525
= –2 log55 Log5 (y2) = log5 (3x × 25)
log332 Equating terms
= –2 y2 = 3x × 25
2log33 y2 = 75x (C)
= –2 i.e –1 ( B )
2 1992/13 (Nov)
1995/7 UME If 2Logx + log(y – 2) = 0, find y in terms of x
Find the value of (16)3/2 + log10 0.0001 + log232
A. 2 + 12 B. 2 – 12 C. 2 + 2x
A.0.065 B.0.650 C.6.500 D.65.000 x x
Solution D. 1 + 12 E. 2 + 22
This problem is interesting because of its uniqueness x x

 
3

= 42 2
 log 10 10 1000  log 2 32 Solution
Applying the various laws of logarithm
= 43 + log1010 –4 + log225 2Logx + log(y – 2) = 0 becomes
= 64 + – 4 log1010 + 5 log22 Logx2 + log(y – 2) = log1
= 64 – 4 + 5 Log[x2 ×(y – 2)] = log1
= 65 (D) Equating terms
x2(y – 2) = 1
1997/5 UME Making y the subject of the formula
Simplify log296 – 2log26
y – 2 = 12
A. 2– log23 B. 3 – log23 C.log23 – 3 D.log23–2 x
Solution
y = 2 + 12 (A)
Applying log laws, x
= log296 – log26 2
2003/49 Neco
= log296 – log236 Express 3Logx – ½ Logy + 1 as a single logarithm.
Since they are in same base 10 x 3
C log x 1
3
 12
A log B log 5x 2 y
= log2 96 y y
36
3 3
Canceling out terms D log x E log
x
= log2 8 y 1 y
3 Solution
= log28 – log23 Applying the logarithm rules and bearing in mind that any log not
= log223 – log23 given is assumed to be log to base 10
= 3 log22 – log23 3Logx – ½ Logy + 1
= 3 – log23 (B) 1
= logx3 – log y 2 + log1010

11
= log x3 × 10 Substitution in logarithm
1
y 2 Some logarithm problems involves substituting a given
logarithm value into it. By this we are constrained or limited
10 x 3
= log (A) to solving the problem by simplifying to get either exactly
y the given log value or its multiples.

1983/33 UME Exercise 1.31 2003/9b (i) Neco (Dec)


Given that x = log2 and y = log3,
Without using tables find the numerical value of
log 7 49  log 7  17  express log 3 72 in terms of x and y
Solution
A.1 B.2 C.3 D.7 E.0 1
log 3 72 = log 72 3

1
1983/22 UME Exercise 1.32 = /3 Log 72
1 = 1/3 Log 8 × 9
Simplify log 10 a  14 log 10 a 
3 1
12 log 10 a 7 = 1/3 Log 8 + 1/3 Log 9
A.1 B. 7/6 log10a C.0 D.10 E. a = 1/3 Log23 + 1/3 Log32
= 3/3Log2 + 2/3 Log3
1987/14 UME Exercise 1.33 Substituting for x and y
= x + 2/3y
Simplify without using tables
log26 – log23
1988/9 UME
log28 – 2 log2 ½ If log102 = 0.3010 and log103 = 0.4771, evaluate without
using logarithm tables, log104.5
A. 1/5 B. ½ C. – ½
D. log23 A.0.3010 B.0.44771 C.0.6532 D.0.9542
log27 Solution
2002/7 PCE Exercise 1.34 We observe that log104.5 = log10 45
Evaluate log7 – log3 10
log3 – log7 = log10 9
A. –2 B. –1 C. 2 D. 3 2
= log10 9 – log10 2
2003/42 PCE Exercise 1.35 = log10 32 – log10 2
Simplify log8 + log4 = 2 log10 3 – log10 2
Log8 – log4 We can now substitute for the given values
A.3 B.4 C.5 D.2 = 2( 0.4771 ) – ( 0.3010)
= 0.9542 – 0.3010
1995/8 PCE Exercise 1.36 = 0.6532 ( C )
Evaluate log354 + lo36 – log34, without using tables.
A.3 B.4 C.9 D.81 1997/4 UME
Given that loga2 = 0.693 and loga3 = 1.097, find loga13.5
2006/42 UME Exercise 1.37 A.1.404 B.1.790 C.2.598 D.2.790
Calculate the logarithm to base 9 of,
3– 4 × 92 × (81) –1
Solution
A.0 B.2 C. – 4 D. –2 loga13.5 is loga 135 = loga 27
10 2
2000/11 UME Exercise 1.38 = loga27 – loga2
–3log 2 = loga33 – loga2
Evaluate 5 5 × 22log23
A.8 B.1 1/8 C. 2/5 D. 1/8 = 3 loga3 – loga2
Substituting for the given values,
2006/16 PCE Exercise 1.39 = 3(1.097) – 0.693
Simplify 2 – log255 = 3.291 – 0.693
A. 1 1/2 B.2 1/2 C. 5 1/2 D. 1/2 = 2.598 (C)

1978/39 UME
2007/7 PCE Exercise 1.40 Assuming loge4.4 = 1.4816 and loge 7.7 = 2. 0142
Then the value of loge 7 is
log 5
Evaluate 4
log 5 A.0.5326 B.3.4958 C.0.4816 D.0.0142 E.1.3594
1
A. 2 B.1 C. /2 D. – 1/2

12
Solution Solution
The logarithm values given are that of the multiples of Take note that log28x = log28 + log2x
the problem. But good a thing, we are multiplying both log24y = log24 + log2y
numerator and denominator by the same multiple i.e. Thus, log28x + log24y = log28 + log2x + log24 + log2y
7  1.1 = 7.7 = log223 + log222 + log2x + log2y
4  1.1 4.4 = 3log22 + 2 log22 + [ log2x + log2y]
So 7 is same as 7.7 Thus, = 3 + 2 + (8)
4 4.4 This is so, because log of any number to its base is one, and
Loge 7 = loge 7.7 the bracket term is 8 as given earlier.
4 4.4 = 13 (C)
= loge7.7 – loge4.4 2 .0142
2005/28 UME Exercise 1.41
= 2.0142 – 1.4816 – 1 .4816
If log102 = 0.3010 and log103 = 0.4771
= 0.5326 (A) 0 .5326
Evaluate log104.5
1991/11 PCE A.0.4771 B.0.3010 C.0.6532 D.0.9542
Given that log107 = x and log102 = y,
evaluate without using tables, log1035 2007/33 UME Exercise 1.42
A.x + y B. x – y + 1 C. x +1 D. x – y If log102 = x , express log10 12.5 in terms of x
y A. 2(1 + x) B. 2 + 3x C. 2(1 – x ) D. 2 – 3x
Solution
We will try as much as possible to express 35 as a
product of 2 & 7 and its multiples.
log1035 = log10( 7  5 )
= log107 + log105 But 5 = 10
2
= log107 + log1010/2
= log107 + log1010 – log102
Of course log1010 is one
= log107 – log102 + 1
= x – y + 1 (B)

1998/6 PCE
Given that log10y = 2, log102x = 1,
Evaluate log10 y
3x
A. 1 B. 2 2 C. 10 D.3 2
15 3 3 Change of base
Solution (4) If any given base is not conducive or good enough to
We have a case of indirect substitution in our hands. work with , we can change base
If log10y = 2, logbN = logcN
Then log 10 y Which is log y ½ becomes
10 logcb
i.e. ½ log10y = ½ of 2 Here the new base is not common to the numbers given but
= 1 it suit our work.
And log102x = 1; changes to indices as There are cases where the number whose logarithm, we are
2x = 101 looking for will be used :
i.e. x = 10 i.e 5 logbc = logcC = 1
2 logcb logcb
Then 3x = 35.
= 15 2002/2 (Nov)
Find the value of log 0.25 32
Thus, log10 y = 1 (A)
A 2.5 B 0.4 C – 0.4 D – 2.5
3x 15 Solution
1999/7 PCE log 0.25 32 is same as log  1  32
4
Given that log2x + log2y = 8, Applying change of base
Find log28x + log24y
A.4 B.12 C.13 D.20
13
log 2 32 1993/14 (Nov)
log 41 32 = 1
log 2 4 1 Given that logx 256 = 2, find log 8  
 x
log 2 25
= A. – /3
4
B. –1 C. – /12
1
D. 5/4 E. 3/2
log 2 2 2
Solution
= 5 log22 Changing from logarithm to indices
– 2 log22 Logx 256 = 2 becomes
= – 5/2 i.e – 2.5 (D) x2 = 256
2002/23 (Nov)
Raising both sides to power two
Evaluate log4 0.3 – log40.48 + log40.05 x2 = 162
A. – 8 B. – 5/2 C. – 2/5 D. – 1/8 x = 16
Solution 1 1
Next, we find log 8   = log 8  
Applying logarithm laws  x  16 
log4 0.3 – log40.48 + log40.05 becomes
1
Solving log 8   by Change of base
= log4 0.3 × 0.05
0.48  16 
= log4 3×5 log 2 161
480 =
log 2 8
= log 4 32
1

Changing base log 2 2  4  4 Log 2 2


= =
log 2 321 log 2 2 3
3Log 2 2
log 4 1
32
=
log 2 4 = – 4/3 (A)
5
= log 2 2
log 2 2 2 1995/14 (Nov)
If Log 2 = x and Log 3 = y, find in terms of x and y,
=–5 log22
2 log22 the value of Log27 721 
= – 5/2 (B) A –2x – 2y B – 3x + 2y C 2x – 2y
2004/23 y 6y y
Evaluate log0.20 125 D –2x + 2y E – 3x – 2y
A. 1/5 B. – 1/3 C. – 3/5 D. –3 y 6y
Solution Solution
Log0.20 125 is same as log 1 125 It is obvious, we are to work in base 10, changing base

 = log 10  72 
5 1
1

2
Changing base Log27 1
72
log 10 27
log 1 125 = log 5 125
log 5 log 10  819 
5 1 1
5
2
= log5 53 =
log 10 33
log5 5– 1
= 3log5 5 =–3(D) = ½ [log(2–3 + 3–2)]
– 1 log5 5 3Log3
= ½ [–3Log2 – 2Log3]
2005/23 3Log3
Solve the equation: log 2
16 = x Substituting for x and y
A 8 B 4 C2 D 2 = ½ (–3x – 2y)
Solution 3y
Changing base at the LHS = –3x – 2y
log 2 16 = x becomes 6y (E)
log 2 16 = x
log 2 2 1997/13
log 2 2 = x 4 Find without using tables, the value of
1 log 2  94  × log 0.1 10
log 2 2 2 3

4 log2 = x 2 A –2 B – 0.2 C 0.2 D 0.5 E 2


1
/2 log22 Solution
Thus, x = 8 (A) We proceed as follows:
log 2  94  × log 0.1 10 = log 2  23 2 × log 101 10
3 3

Next we try change of base for the last term


14
= 2Log 2 2
×
log 10 10 Substituting
3
3
log 10 10 1 S– r = 4
= 2×–1 log2(2N) – 2 log8N = 4
= – 2 (A) We apply change of base to log8N
2003/1 UME log2 [2N] – 2 log2N = 4
Evaluate log 2 4  log 1 16  log 4 32
2 log28
A. –5.5 B. – 2.5 C.2.5 D.5.5
Solution log2 [2N] – 2 log2N = 4
A more look at this question will disorganize us. But if
log223
we change their entire base to 2 (two) and not 4 since
four can not bail us out. We are home.
log2 [2N] – 2 log2N = 4
log 2 4 log 2 16 log 2 32
=   3log22
log 2 2 log 2 12 log 2 4
log2 [2N] – 2 log2N = 4
= log222 + log224 – log225
½
3
log22 log22–1 log222 log2(2N) – 2 log2N = 4
= 2log22 + 4log22 – 5log22 3
2

½ log22 –1Log22 2log22 log 2 2 N  log 2 N 3  log 2 2 4


= 2 + 4 – 5  2N 
½ –1 2 log 2  2   log 2 2 4
= 4– 4 – 5 N3 
2 Equating terms
1  23
= –5 i.e –2.5 (B) 2N  24
2 1

1985/8 UME
2N 3  24
Without using tables, evaluate Multiply both sides by 2-1
log24 + log42 – log255
1
N 3  23
A. ½ B.1/5 C.0 D.5 E.2 To remove cube root from N we take cube of both sides
Solution N = ( 23 )3 i.e 29
By observation, we can see that only change of base = 512 (A)
will save us. We must always stick to the rule of change
of base as this question is meant to try us along that line 1984/10 Exercise 1.43
= log24 + log22 – log55 Find, without using logarithm tables, the value of
log22 log24 log525 log327 – log ¼ 64
The three items are different, it’s just by coincidence that log3 1
items one and two are using the same number 2 for changing 81
base. Item three must not use two (2) for its change of base, It
is five that suits it . A.7/4 B. – 7/4 C. – 3/2 D. 7/3 E. – ¼
= log22 2
+ log22 – log55
2
log22 log22 log552
= 2 log22 + log22 – log55
log22 2 log22 2 log55
= 2 + ½ – ½
= 2 (E)

1993/8 PCE
If log8N = r , log2(2N) = S and S – r = 4, find N
2
A. 512 B. 128 C. 64 D. 32
Solution
Our main working tool is S– r = 4
Observe that if log8N = r then 2 log8N = r
2
15
Logarithmic equations Equating terms
At times, questions in logarithm involve equations, (5x – 6)(2x + 3) = 10x2 – 3x –5
10x + 15x – 12x –18 = 10x2 – 3x –5
2
which may be simple linear, simultaneous linear
3x + 3x = – 5 +18
equations or quadratic equation that is factorizable in 6x = 13
nature. x = 13/6
Which ever pattern the questions follows; we solve
using the knowledge of the method of solving such 2004/1
equation. Solve the simultaneous equations:
8y = 4(2x + 5)
2004/30 Neco log3y = log3 x + 2
Solve for x in the equation`n Log2x4 + Log2x = 5 Solution
A. 5 B. 4 C. 3 D. 2 E. 2/3 From log3y = log3 x + 2, we have
Solution log3y = log3x + log332
Applying logarithm laws log3y = log3 (x × 9)
log2 x4 + log2 x = 5 becomes Equating terms
log2 x4+ log2 x = log225 y = 9x
log2 ( x4 × x) = log225
Equating terms Also from 8y = 4(2x + 5)
x4 × x = 25 Raising both sides to same base
x5 = 25 23y = 22(2x +5)
x = 2 (D) Equating powers
3y = 2(2x + 5)
2000/17 3y = 4x + 10
y Substituting for y = 9x
Given that 8 = log2 16, Find the value of y. 3(9x) = 4x + 10
4

A. 4/3 B. 8/3 C. 4 D. 6 27x = 4x + 10


Solution 27x – 4x = 10
Applying logarithm law to RHS 23x = 10
y x = 10/23
8 4
= log 2 2
4 Thus y = 9(10/23 ) i.e 90/23
2006/2
= 4Log22
y
If 22x – 3y = 32 and log y x = 2,
find the values of x and y
8
= 4
4
Solution
Changing both sides to same base Converting logarithm to indices
y
3(4) Log yx = 2 becomes
2 = 22 y2 = x ----------------------(1)
Equating powers Note that 22x – 3y = 32 is same as
3(y/4) = 2 22x – 3y = 25
y
/4 = 2/3
Equating powers
y = 4 × 2/3
2x – 3y = 5 ----------------(2)
y = 8/3 (B)
Substituting for x = y2
2y2 – 3y = 5
1999/3 (Nov) Rearranging
Given that log 2x + log 3 81 = 1, find x 2y2 – 3y – 5 = 0
A. 8 B. 1/6 C. 1/8 D. 1/32 Factorizing
Solution 2y2 + 2y – 5y – 5 = 0
Applying the laws of logarithm 2y(y + 1) –5(y + 1) = 0
(y + 1)(2y – 5) = 0
log 2x + log 3 81 = 1 becomes
y + 1 = 0 or 2y – 5 = 0
log 2x + log 3 34 = 1 y = –1 or 2y = 5
log 2x + 4 = 1 y = –1 or 5/2
log 2x = 1– 4 when y = –1; x = y2 becomes x =1
log 2x = – 3 when y = 5/2; x = y2 becomes x = 25/4
Changing from Logarithm to indices
x = 2– 3 i.e 1/8 ( C) 1997/9 (b)
Solve the simultaneous equation:
2005/1c Neco 3x + 2y = 20
Solve the equation 2Log10x – log10y = 1
Loga(5x – 6) + Loga(2x + 3) = Loga(10x2 – 3x –5)
Solution
Applying logarithm laws ; the problem becomes

Loga (5x – 6)(2x + 3) = Loga (10x2 – 3x –5)


16
Solution Solution
3x + 2y = 20 ………. (1) Applying logarithm laws
2log10x – log10y = 1………… (2) Log2 (x – 1) + log2 (x + 2) = 2 becomes
Log2 (x – 1)(x + 2) = log 2 2 2
From (2) log10x2 – log10y = 1
Applying the law of indices Equating terms
x 2 = log1010 (x – 1) ( x + 2) = 22
log 10
y
x + 2x – x – 2 = 4
2

2
x2 + x – 6 = 0
Equating terms x = 10 Factorizing
y x2 + 3x – 2x – 6 = 0
x2 = 10y ------** x(x + 3) – 2(x + 3) = 0
From (1) y = 20 – 3x (x + 3)(x – 2) = 0
2 x + 3 = 0 or x – 2 = 0
Put y value into ** x = – 3 or 2 (C)
x2 = 10 20 – 3x 2003/1 (a) (Nov)
2 Solve the equation
x2 = 5(20 – 3x) Log10 (4P2 + 1) – 2 Log10 P – Log10 2 = 1
x2 = 100 – 15x Solution
Re – arranging Applying logarithm laws
x2 + 15x – 100 = 0 Log10 (4P2 + 1) – 2 Log10 P – Log10 2 = 1 becomes
Factorizing
Log10 (4P2 + 1) – Log10 P2 – Log102 = Log1010
x + 20x – 5x –100 = 0
2

x (x + 20) – 5(x + 20) = 0 Log10 4P2 + 1 = log1010


(x + 20)(x – 5) = 0 P2 × 2
x + 20 = 0 or x – 5 = 0 Equating terms
x = – 20 or 5 4P2 + 1 = 10
when x = 5 from ** 2P2
ie x2 = 10y becomes Cross multiply
25 = 10y 4P2 + 1 = 20P2
25
/10 = y ie y = 5/2 Collect like terms together
1 = 20P2 – 4P2
when x = –20 from **
1 = 16P2
x2 = 10y 1
/16 = P2
(–20)2 = 10y
400 = 10y P =  ¼
40 = y 2000/37 Neco
Solve the equation: Log10 (x – 8)2 = 2 – Log10 (x + 1)2
2000/9a (ii) A. x = –2 or –7 B. x = –2 or –9 C. x = –2 or 9
Find the values of x for which Log2 2x + Log2 (x + 4) = 6
Solution
D. x = 1 or –18 E. x = 2 or 9
Applying logarithm law Solution
Log2 2x + Log2 (x + 4) = 6 becomes Applying logarithm laws
6 Log10 (x – 8)2 = 2 – Log10 (x + 1)2 becomes
Log2 2x(x + 4) = log 2 2
Log10 (x – 8)2 = Log10102 – log10 (x + 1)2
Equating terms Log10 (x – 8)2 = Log10 102
2x(x + 4) = 26
(x + 1)2
2x2 + 8x = 64
Rearranging
Equating terms
2x2 + 8x – 64 = 0 (x – 8)2 = 102
x2 + 4x – 32 = 0 (x + 1)2
Factorizing Since both sides are in power two; It follows that
x2 + 8x – 4x – 32 = 0 x – 8 = 10
x(x + 8) – 4(x + 8) = 0 x+1
(x + 8)(x – 4) = 0 Cross multiply
x + 8 = 0 or x – 4 = 0 (x – 8)(x + 1) = 10
x = – 8 or 4 x2 + x – 8x – 8 = 10
x2 – 7x – 18 = 0
2002/11 Factorizing
Solve the equation Log2 (x – 1) + Log2 (x + 2) = 2
x2 – 9x + 2x – 18 = 0
A. x = 0 or 1 B. x = 1 or – 2 x(x – 9) + 2(x – 9) = 0
C. x = 2 or – 3 D. x = 1/2 or 0
(x – 9) (x + 2) = 0
x – 9 = 0 or x + 2 = 0
x = 9 or –2 (C)
17
1989/11 UME Solution
Find p in terms of q , if Applying the laws of logarithm
log3 p + 3 log3q = 3 logx26 – logx33 = log5 (0.2)3
3 3 q 1/3 q 3 3 1/3
A. ( /q ) B. ( /3) C.( /3 ) D. ( /q )
logx 26 = log5 2 3
Solution
Applying the laws of logarithm. 33 10
log3 p + log3q3 = log333 But we must express 2 in terms of 5 i.e Reducing to 1
10 5
Since they are same base,
Log3 (p × q3) = log333
logx 26 = log5 1 3

Equating terms 33 5
p × q3 = 33 = log5 (5 –1)3
pq3 = 33 = log5 5 –3
p = 33 = – 3log55
q3 logx 26 = –3
It follows that 33
p = ( 3/q )3 A. Changing from log to indices
1993/6 UME x –3 = 26 i.e 22 3
If 2log3 y + log3 x2 = 4, then y is 33 3
A. 4 – log3x2 B. 4 C. 2 D.  9 To enable us get x, we express the RHS in
2 log3x2 x x power of minus three i.e by inverting it.
Solution
Applying the laws of logarithm, x –3 = 3 –3
2
log3y2 + log3 x2 = 4 2
Since they are in the same base Hence x = 3
log3 ( y2 × x2 ) = 4 22
Converting to indices = ¾ (C)
34 = y2 x2 1992/3 UME
1
4
3 = y 2
Evaluate log b a n If b  a n
x2 A.n2 B. n C. 1/n D.1/n2
32 2
= y2 Solution
x Let log b a n  x
y =  9 2 Changing from log to indices
x an = bx
1
=  9 (D) But, we were given that b  a n

 
x 1 x
1999/3 UME an  a n
If log810 = x, evaluate log85 in terms of x. x

A. ½ x B. x – ¼ C. x – 1/3 D. x – ½ an  a n
Solution Equating powers
log85 = log8 /2 10 n = x
Since we have log810 n
= log810 – log82 x = n2 (A)
= x – log82
but changing base to enable us know the numerical 1981/24 UME
value of log82, Given that 10x = 0.2 and log102 = 0.3010, find x.
= x – log22 A. –1.3010 B. – 0.6990 C.0.6990 D.1.3010 E.0.02
log28 Solution
=x– 1 10x = 0.2
log223 Changing indices to logarithm
=x– 1 x = log100.2
3 log22 = log10 2
= x – 1 (C) 10
3 = log102 – log1010
2004/43 UME
If 6 logx2 – 3logx3 = 3log50.2, find x = 0.3010 – 1
A. 8 B. 4 C. 3 D. 3 = – 0.6990 (B)
3 3 4 8
18
1996/6 PCE 2000/9 PCE
Solve for m in the equation If logx27 + logy4 = 5 and logx27 – logy4 = 1
 
5m  1
1 2m  3 find x and y respectively.
log 2 16  log 2 8 3
A.3, 3 B.3, 2 C.2, 3 D.2, 2
A. 2 B. 1 C. –1 D. – 2 Solution
Solution We can not apply log laws of multiplication (addition) or
division (minus) since they are the same terms except for
log 2 161 
5m  1
2m  3
 log 2 8 3
plus and minus.
3 ( 5 m  1) We let:
log 2 2  4 ( 2 m  3)  log 2 2 3
Logx27 = p and logy4 = q
Equating powers Then,
– 4 (2m +3) = 3(5m+1) p + q = 5 …(1) and
3 Adding (p – q = 1 ) – ( 2)
– 4 (2m + 3) = 5m + 1 2p = 6
– 8m – 12 = 5m + 1 p = 6/2 i.e. 3
– 12 – 1 = 5m + 8m Substitute p value into (1).
– 13 = 13m p + q = 5 will become
–1= m (C) 3+q=5
q = 5 – 3 i.e. 2
Applying indices & log relation.
1998/4 UME logx27 = p and logy4 = q
Given that log4(y – 1) + log4 ( 1/2 x) = 1 and logx27 = 3 and logy4 = 2
log2(y +1) + log2x = 2 , x3 = 27 and y2 = 4
solve for x and y respectively x3 = 33 and y2 = 22
A.2, 3 B.3, 2 C. –2, –3 D. –3, –2 Thus, x = 3 and y = 2 (B)
Solution
Applying the laws of logarithm
log4(y – 1) + log4 ( 1/2 x) = log4 41
log4 (y – 1) ( 1/2 x) = log4 41
We can now equate terms
(y – 1) ( 1/2 x) = 41 1986/14 Exercise 1.45
Find n If log24 + log27 – log2n = 1
4 = (y–1)x
A.10 B.14 C.27 D.28
2
8 = yx – x ………(1)
Also log2(y +1) + log2x = 2 1981 Exercise 1.46
Applying the laws of logarithm, If log2y = 3 – log2 x ( 3/2 ), find y when x = 4
Log2 [(y +1)x] = log2 22 A.8 B.0 C.2 D.3 E.1
We can now equate terms
22 = (y + 1)x 1982/9 Exercise 1.47
4 = yx + x …….(2) Write the equation
Solving (1) and (2) Simultaneously, we eliminate yx. 2 log2x – x log2 (1 + y) = 3 in a form not involving logarithms
8 = yx – x A.2x – x (1 + y) = 3 B.2x – x (1 + y) = 8
– (4 = yx + x)
C.x2 = 8 (1+ y)x D.x2 – x (1+ y) = 8
4 = – 2x
E.x – (1+y) = 8x
2 2
x = 4 i.e – 2
–2
put x = – 2 into (2) 1985/10 Exercise 1.48
4 = yx + x will become Find x if log9 x = 1.5
4 = y(–2) + (–2) A.72.0 B.27.0 C.36.0 D.3.5 E.24.5
4 = –2y – 2
4 + 2 = –2y
6 = –2y
6 = y
–2
y = –3
x, y means –2, –3 (C)

19
Common Logarithm E.g (3) With the aid of tables evaluate (42.95)3
Logarithm to base 10 is known as the common logarithm.
There are other bases of logarithm e.g base e (natural, No Log
Naperian base). (42.95)3 1.6330 x 3
The logarithm of a number consists of two parts called the
Characteristics and the mantissa.
79250.0 4.8990
Characteristics is a whole number while Mantissa is a
decimal. To get the characteristics of any given number, Note; Since antilog xtics is 4, movement 5 steps from the
express the number in standard form, the index is the imaginary decimal point before the first digit i.e 7
characteristics. Hence 7925 = 79250.0
E.g, Study carefully the table below:
Numbers Standard Logarithms Answers E.g (4) Evaluate the 3 66.32 using tables
form Xtics mantissa Solution
(a) 5 5.0 x 100 0 .6990 Log of 5 No Log
= 0.6990 (66.32) 1/3
1.8216 ÷ 3
(b) 60 6.0 x 101 1 .7782 Log of 60
= 1.7782 4.048 0.6072
(c) 8156 8.156 x 103 3 .9115 Logof 8156 Note
=3.9115
Log problems involving roots, we divide the log value by
(d) 8.2 x 10 –3 .9138 Log of 0.0082
the equivalent root e.g
0.0082 -3 or 3 = 3.9138
(e) 0.2 2.0 x 10 - 1 .3010 Log of 0.2 , divide by 2
- 1 or 1 = 1.3010
3 , divide by 3
You will agree with the author that your ability to find
the characteristics of any given problem depend on
basic knowledge of standard form. 4 , divide by 4 etc

Logarithm of numbers greater than one Logarithm of numbers less than one
Multiplication (Addition of log values). E.g 1 Evaluate 0.5624 x 0.0378
E.g. (1) Using log table, evaluate 69.24 x 8.31
No Log
No Log 0. 5624 1.7500 Addition –1 + 1 – 2 = - 2
69.24 1.8403
8.31 0.9196 0.0378 2.5775
575.3 2.7599 0.02125 2.3275

Analysis  0.5624 x 0.0378 = 0.02125


69.24 = 6.92 x 101
8.31 = 8.31 x 100 E.g 2 Evaluate 0.003512  0.6207
Check 69 under 2 difference of 4 No Log
i.e 8401 + difference (2) = 8403
Check 83 under 1 0.003512 3.5455 Subtraction
add up log values 0.6207 1.7929
Check the Antilog of 75 under 9, diff. of 9
0.005657 3.7526
5741 + 12 = 5753
Antilog xtics 2  0.003512  0.6207 = 0.005657
Movement 3 steps from the imaginary decimal before E.g. (3) Evaluate (0.07392)4
the first digit i.e 5
No log
Eg (2) (0.07392)4 2.8628 x 4
Using logarithm table, evaluate 7031  4.911.
Solution 0.00002826 5.4512
No Log
7031 3.8471  (0.07392)4 = 0.00002826
4.911 0.6912 Multiply the xtics and the mantissa separately and add the
1432 3.1559 result i.e 0.8628 x 4 = 3.4512
and 2 x 4 = 8
Division in log, we subtract the log values and find anti
log. Thus 8 + 3.4512 = 5.4512
20
E.g (4) Evaluate 4 0.00692 1991/2a (Ordinary maths)
Using logarithm table, evaluate
No Log
(0.00692) 1/4 3.8401  4 is expressed as: 3 1.376
( 4 +1.8401)  4
4 0.007 Correct to 3 s.f.
0.2884 1 . 4600
Solution
 (90.00692)1/4 = 0.2884 No Log
Think of a number to be added to 4 to give 3 on 1/3
the xtic side only i.e 4 + 1 = 3 (1.376) 0.1386  3
Then divide 4 by 4 and 1.8401 by 4. 0.0462 0.0462

(0.007)1/4 3.8451  4 is expressed as:


Problems on logarithm
1988/27 (Ordinary maths) 4 +1.8451  4
If log10 q = 2.7078, what is q? 1.4613 1.4613
A.5102 B.849.9 C. 510.2 D. 84.99 E .51.02
Analysis and Solution 3.85 0.5849
To get q value, we check the reverse of the log value of
q to base 10 i.e. antilog of 2.7078 implies 70 under 7
difference 8. Then we move three step forward. Although the anti log of 0.5849 is 3837 + 8 (difference )
Antilog of 2.7078 = 510.2 (C) = 3.845  3.85 to 3 s.f otherwise you lose the mark for
correct answer.
1989/4 (Ordinary maths)
Evaluate, using log table 5.34 x 67.4 1993/6b (Ordinary maths)
2.7 Use mathematical table to evaluate
A.1.332 B.13.32 C.133.2 D.1332 E. 3320
No Log 2.067 =A½
5.34 0.7275 Addition (multiplication) 0.0348 x 0.538
67.4 1.8287
2.5562 Solution
No Log
2.7 0.4314 Subtraction (Division) 2.06 0.3154 0.3154
0.038 2.5416
133.2 2.1248 0.538 1.7308
2.2724 2.2724
1990/6 (Ordinary maths) ½
Evaluate, using logarithm table, log (0.65)2 A 2.0430  2
A. 1.6258 B. 0.6272 C.0.6258 D. 1.6258 E 1.6272 10.51 1.0215
No Log

(0. 65)2 1 .8129 x 2 1994/6b (Ordinary maths)


Use logarithm table to evaluate
1. 6258 (A) 0.897 x 3.536 Correct to 3 s.f
0.00249
Note: The question did not ask us to evaluate (0.65)2, Solution
Hence no need to check anti log. We have 1 because 2 x No Log
(-1) plus the positive 1 carried across will give 2+1 0.897 1.9528
=1 3.536 0.5485
0.5013
1991/6 (Ordinary maths) 0.00249 3.3962
Find the number whose logarithm to base 10 is 2.6025 1274.0 3.1051
A 400.4 B 0.4004 C 0.04004 D 0.004004 E 0.000
4004
As usual we reverse the log (i.e anti log the given value) 1995/36 (Ordinary maths)
Thus, antilog of 2.6025 = 400.4 If log10 x = 2.3675 and log10 y = 2.9738, what is the value of
Actually we have 4004 - the xtics is 2 which means x + y, correct to 3 s.f ?
3 steps to decimal point. A. 0.117 B.0.118 C. 0.903 D. 0.944 E.0.946
21
Solution VTR – 11/1a NTI TCII (Ordinary maths)
To get x and y, we reverse the log values i.e anti log Evaluate using logarithm tables
operation.
Antilog of 2.3675. (3.65 )2 x 145.6
Check 36 under 7 diff 5 = 2331 anti log xtics is 2. 0.4631
Hence 1 step backward. (Negative movement is one step
less; while positive movement is one step ahead). Solution
No log
 anti log of 2.3675 = 0.023 i.e x
(3. 65)2 0. 5623 x 2 1.1246
Similarly anti log of 2.9738 = 0.0094 i.e y
145.6 2. 1632 2.1632
Thus x + y = 0.117
Numerator 3. 2878 3. 2878
1995/6c (Ordinary maths)
Use logarithm table to evaluate (0.4631) 1/2 1.6657 ÷ 2
(3.68)2 x 6.705 Denominator 2 + 1.6657 ÷ 2 1. 8329
2850.0 3. 4549
0.3581
No Log
VTR – 13/1b NTI TCII (Ordinary maths)
(3.86)2 0.5658 x 2 1.1316
Evaluate using four figure tables.
6.705 0.8264 0.8264
1.9580 Solution
(0.3581) 1/2
1.5540  2 No log
7.62 0.8820
2 + 1.5540 2 1.7770 96.45 1.9843
151.7 2.1810
(7.62 x 96.45 )1/4 2.86 63 ÷ 4 0.7166
1997/2 (Ordinary maths) (1.36)2 0.1335 x 2 0.2670
What is log10 (6.95 x 10-3 )? 2.816 0. 4496
A. 0.8420 B. 3.8420 C. 3.8420 D. 3.9777
2004/6a NABTEB Exercise 1.50 (Ordinary maths)
E 3.9777
Solution Evaluate 82.41  76.52 using logarithm tables.
Here we are only required to find the log value. The 7 .839
given number has been expressed in standard form
hence 2004/4a (Nov) Exercise 1.51 (Ordinary maths)
Log10 (6.95 x 10-3) = 3 . 8420 (C) Use logarithm tables to evaluate
Candidates should not confuse the above problem with 0 . 00341
log (6.95 x 10)-3 in which case we multiply the log
value by –3 2004/16 (Nov) Exercise 1.52 (Ordinary maths)
If log10 x = 2. 7087, find x
1997/6a (Ordinary maths) A. 0.01866 B. 0. 02913 C. 0.05113 D. 0.08504
Use logarithm table to evaluate
1998/19 (Nov) Exercise 1.53 (Ordinary maths)
15.05 x 0.00695 correct to 3 sf The logarithm to base ten of a number is 2.415. find the square of
6.95 x 102 the number.
Solution A. 2.600 x10- 2 B. 2.600 x 10- 4 C. 6.761 x 10 – 4
No Log D. 6.761 x 104 E. 67.6 x 104
15.05 1.1775 1.1775
2000/1a Neco Exercise 1.54 (Ordinary maths)
(0.00695)1/2 3.8420  2 Use logarithm table to evaluate,
4 + 1.8420 2 2. 9210 12.06 x 0.541/3 correct to 3 d.p
0. 0985 54.8 x101
6.95 x102 2.8420 2. 8420
0.001805 3. 2565 2001/6 Neco Exercise 1.55 (Ordinary maths)
 0.00181 to 3sf. Use logarithm table to evaluate
48. 53  (0.5324)2
VTR – 10/28 NTI TCII (Ordinary maths) ½
Evaluate 10 2.6457 ( 3.58)
A 26.457 B 102.6457 C 442.3 D 482.6 E 4423.0
1998/5 Exercise 1.56 (Ordinary maths)
Solution
Find the number whose logarithm to base 10 is 2.6025.
We simply take the antilog of . 64 under 5 difference 7:
A. 0.0004 B. 4.004 C. 40.04 D. 400.4 E. 4004
and characteristics of 2
10 2.6457 = 442.3 (C)
22
Chapter two Solution
Simultaneous linear, quadratic and First, We rearrange and label.
Cubic equations and roots symmetric functions 3x – 5y = 3 …………..(1)
Simultaneous linear equations 2y – 6x = – 5 ……….. (2)
Simultaneous linear equations as the name imply is a pair To make the coefficient of x uniform i.e. 6 in both equation
of linear equations (highest degree of the unknown is (1) and (2) ; We multiply (1) by 2
one) occurring together. Though in this case there are 6x – 10y = 6
two different unknowns unlike in the simple equation, Add (– 6x + 2y = – 5 )
which has only one unknown. Unless otherwise stated, – 8y = 1
we can apply either elimination or Substitution method – 8y = 1 i.e y = 1
in solving any given simultaneous linear equations. –8 –8 –8
Though a third method of solving exist; which is Substitute y value into (1)
graphical solution it is given little or no attention, except 3x – 5y = 3 will become
for deductive use. 3x – 5 –1 = 3
1990/23 PCE 8
The respective values of x and y in the equations 3x + 5 = 3
4x – y = 7 and x + 4y = 23 are 8
A.5 and 3 B.2 and 1 C.3 and 5 D. – 3 and –5 3x = 3 – 5
Solution 8
4x – y = 7 ………..(1) 3x = 24 – 5
x + 4y = 23 …………(2) 8
To make the coefficient of y uniform in both equations 3x = 19
(1) and (2) ; we multiply (1) by 4 8
16x – 4y = 28 3x = 19  3
Add ( x + 4y = 23 ) 3 8
x = 19 × 1 i.e 19 (x, y) ; 19 , – 1 (D)
17x = 51 8 3 24 24 8
17x = 51
17 17 1983/4 UME
x = 3 Solve the following equation
Substitute x value into (1) 4x – 3 = 3x + y and 3x + y = 2y + 5x – 12
4x – y = 7 will become A. x = 5 , y = 2 B. x = 2, y = 5
4(3) – y = 7 C. x = –2, y = –5 D. x = 5, y = –2 E. x = –5, y = –2
12 – y = 7 Solution
12 – 7 = y First, we formalize the equation by collecting like terms
5=y together
x is 3 and y is 5 (C) 4x – 3 = 3x + y
4x – 3x – y = 3
1991/22 PCE x – y = 3 ……..(1) and,
If 3x – y = 23 and 2x + 5y = 4, Find the value of y. 3x + y = 2y + 5x – 12
A. –20 B. – 2 C.2 D.7 12 = 2y – y + 5x – 3x
Solution 12 = y + 2x i.e.
We may not need to solve fully since the question is 2x + y = 12 --------(2)
interested in y value only. So we eliminate x from the Add (1) to (2)
onset. x–y=3
3x – y = 23 ………..(1) + ( 2x + y = 12 )
2x + 5y = 4 ……….(2) 3x = 15
To make x uniform for elimination; multiply (1) by 2 3x = 15
and (2) by 3 then subtract 3 3
6x – 2y = 46 x = 5
– (6x + 15y = 12) Substitute x value into (1)
–17y = 34 x – y = 3 will become
–17y = 34 5– y = 3
5–3 = y
–17 –17 y = – 2 (B) 2 = y Solution: x = 5, y = 2 (A)
1989/21 UME 1993/5 PCE
Given that 3x – 5y – 3 = 0 and 2y – 6x + 5 = 0 Given that 2x – y = 6
the value of (x, y) is 2y – x 1
A. 1, 19 B. 8, 24 C. – 8, 24 D. 19, – 1 Find the ratio x : y
8 24 19 19 24 8 A. 11: 7 B. 8 : 11 C.6 : 8 D.13 : 8
23
Solution Simultaneous linear equation with fractions
Here it is easier and faster to cross multiply and collect like Two types of problems exist under this subheading:
terms than equating and Solving in the simultaneous a . ( i ) Fraction with numbers as denominators – we
equations way. But require carefulness at the end Thus, solve by multiplying through by the LCM of the
2x – y = 6 denominator
2y – x 1
( ii ) Fractions with letters as denominators – Here
(2y – x )6 = (2x – y)1
we change variable which is reconverted after solving.
12y – 6x = 2x – y
This aspect shall be of more concern to us as we go
12y + y = 2x + 6x
along
13y = 8x
Recall that ratio x : y will be Example on a . ( i )
Divide both sides by y 20002/2 Neco (Nov) Ordinary Maths
13 = 8x Solve for x and y if
y x + y = 1 and x – y = 1
13 = x i.e. 13 : 8 in x : y (D)
2 4 2 3 5 9
8 y
Solution
x + y = 1 ----- (1)
1982/5 UME
2 4 2
Seven years ago, the age of a father was three times that
x – y = 1-------(2)
of his son, but in six years time the age of the son will
3 5 9
be half that of the father. Representing the present ages
To eliminate fractions, we multiply (1) by 4 and (2) by 45
of the father and son by x and y respectively, the two
which is their LCM respectively.
equations relating x and y are
A. 3y – x = 0, 2y – x = 0 B. 3y – x = 14, x – 2y = 6 2x + y = 2 ------ (a)
C. 3y – x = 7, x – 2y = 6 D. 3y – x = 14, y – 2x = 6 15x – 9y = 5 ----(b)
E. x + 3y = 7, x + 2y = 12 Multiply (a) by 9 and add to eliminate y
18x + 9y = 18
Solution + ( 15x – 9y = 5 )
Let the son’s age be y 33x = 23
Let the father’s age be x x = 23
‘Seven years ago, the age of a father was three times that of
his son’
33
 Father Son
Substitute x value into (a)
( x – 7 ) = 3( y – 7 ) …….1 2x + y = 2 will become
‘but in six years time the age of the son will be half that of 2 23 + y = 2
the father’ 33
 Son father 46 + y = 2
( y + 6 ) = 1 ( x + 6 ) …….2 33
2 y = 2 – 46
When we open up bracket : 33
Equation 1 will become 3y – x = 14
y = 20
Equation 2 will become x – 2y = 6 ( B )
33
2002/1 UME Exercise 2.0 Example on a . ( ii )
Given that x + y = 3 1989/23 UME
x + 3y 5 Solve the pair of equation for x and y respectively
What is the ratio of x to y? 2x –1 – 3y –1 = 4
A.1 : 2 B. 3 : 5 C. 5 : 9 D. 2 : 1 4x –1 + y –1 = 1
A. –1, 2 B.1, 2 C.2, 1 D.2, –1
1982/38 UME Exercise 2.1
The Solution to the simultaneous equations
Solution
The question is simply put as 2 – 3 = 4 and
3x + 5y = 4 and 4x + 3y = 5 is
x y
A. (– 13/11, 1/11) B.( 13/11,1/11)
C.( 13/11, – 1/11) D.( 11/13, 1/11) E.(13,11) 4 + 1 = 1
x y
We change variable to that which will be reconverted after solving.
Let p = 1 and q = 1
x y
Then our new equation is
2p – 3q = 4 …………(1)
4p + q = 1…………(2)
Multiply (2) by 3 and add.
24
2p – 3q =4 Factorization method
+ ( 12p + 3q = 3 ) Our discussion shall be based on the general form of a
quadratic equation i.e. ax2 + bx + c = 0. Thus when the
14p =7 author mentions “a” he is referring to the coefficient of
p = 7 i.e 1 x2 or any other variable used in power of two. Similarly
14 2 b implies coefficient of x and c refers to the constant
Substitute P value into (2)
4p + q = 1 will become When a is unity ( + 1)
4( ½ ) + q = 1 1979/12 UME
2 + q=1 The solution of the equation
q = 1– 2 x2 – 2x = 8 is
q= – 1 A. x = 0 or 12 B. x = – 2 or 4 C. x = 2
Reconverting our variables D. x = – 4 E. x = 2 or 4
p = 1 and p = 1 Solution
x 2 x2 – 2x = 8 will be rearranged as
Then x2 – 2x – 8 = 0
1= 1 We find the factors of c that when added will give b
2 x c = – 8 and b = – 2
Cross multiply Factors of – 8 Added values
x=2 (i) 8 and – 1 +7
Also q = 1 and q = – 1 (ii) – 8 and 1 –7
y (iii) – 2 and 4 +2
Then (iv) – 4 and 2 –2
–1 = 1
y The last factor fit in. Thus
–y=1 x2 – 2x – 8 = 0 will become
y = –1 i.e. (D) x – 4x + 2x – 8 = 0
2

x (x – 4) + 2(x – 4) = 0
1997/12 UME Exercise 2.3 (x – 4)(x + 2) = 0
Solve the simultaneous equation x – 4 = 0 or x + 2 = 0
2 – 3 = 2, 4 + 3 = 10 x = 4 or – 2 (B)
x y x y 1991/23 UME
A. x = 3/2 , y = 1/2 B. x = 1/2 , y = 3/2 Solve the equation (x – 2)(x – 3) = 12
1 3
C. x = – /2 , y = – /2 D. x = 1/2 , y = – 3/2 A.2, 3 B.3, 6 C. – 1, 6 D.1, – 6
Solution
First, we expand the LHS
(x – 2)(x – 3) = 12 will become
Quadratic equation x(x – 3) – 2(x – 3) = 12
A quadratic equation (function) is noted by the highest x2 – 3x – 2x + 6 = 12
power of two among others. Its general form is x2 – 5x – 6 = 0
ax2 + bx + c = 0 where a  0 Next, we find the factors of C i.e. – 6 that when added
If we allow a case where a = 0, then ax2 = 0; thus gives us b i.e. – 5. They are – 6 and + 1
reducing the equation to a linear one since the next x2 – 5x – 6 = 0 will become
power of x is one i.e bx. Other such as b and c can x – 6x + x – 6 = 0
2

take the value of zero. x(x – 6) + 1(x – 6) = 0


Examples (x – 6)(x + 1) = 0
( i ) x2 + 3x + 10 = 0 (a = 1, b = 3 c = 10) x – 6 = 0 or x + 1 = 0
( ii ) 7x2 – x – 13 = 0 (a = 7, b = – 1, c = – 13) x = 6 or – 1 (C)
( iii ) 19x2 + 3 = 0 (a = 19, b = 0, c = 3) The order of the solution is from small then bigger i.e –1, 6
( iv ) 14x – x2 = 0 (a = – 1, b = 14 c = 0) 1995/14 UME
( v ) x2 = 0 (a = 1, b = 0, c = 0)
The graph of f (x) = x2 – 5x + 6 crosses the x-axis at the
points
Methods of solving quadratic equations A.( – 6, 0), (–1, 0) B.( –3, 0), (– 2, 0)
Quadratic equations can be solved (finding their roots) C.( – 6, 0), (1, 0) D. (2, 0),(3, 0).
by the following methods; Solution
(i) Factorization The question is asking for the roots of the equation in
(ii) Completing the square coordinate forms
(iii) Graphical x2 – 5x + 6 = 0
Factors of C i.e + 6 that when added will give b i.e. – 5
are – 2 and – 3. Thus,
25
x2 – 5x + 6 = 0 will become When b = 0
x – 2x – 3x + 6 = 0
2
Examples: Solve the following quadratic equation
x(x – 2) – 3( x – 2) = 0 (i) x2 – 4 = 0
(x – 2)(x – 3) = 0 (ii) c2 = 9
x – 2 = 0 or x – 3 = 0 (iii) 4q2 = 36
x = 2 or 3 (iv) 16k2 = 49
At this points where the curve cuts the x – axis, y (v) 2b2 = 2
values are always zero. So our answer in Quadratic problems like the ones listed are solved by either
coordinate forms is; applying our knowledge of “difference of two squares” i.e.
(x, y) and (x, y) i.e. x2 – y2 factorises into (x – y) (x + y)
(2, 0) and (3, 0) (D) or making the unknown the subject formula.
( i ) If x2 – 4 = 0 solve for x
When a is not unity (– 1 inclusive) Solution
x2 – 4 = 0 then
1982/36 UME x 2 – 22 = 0
The solution to the quadratic equation 5 + 3x – 2x2 = 0 is (x – 2)(x + 2) = 0
A.( 5/2, 1) B.(5, 3) C.(- 5/2, 1) D.(- 5/2, -1) x – 2 = 0 or x + 2 = 0
E.(5/2, -1) x = + 2 or – 2
Solution
Alternatively
Here we find the factors of ac i.e. – 2 × 5 = – 10
x2 – 4 = 0
that when added will give us b i.e. + 3
x2 = 4
They are – 2 and + 5 coincidentally what we had
x= 4
before. But this is not always the case
x= 2
5 + 3x – 2x2 = 0 will become
x = + 2 or – 2
5 + 5x – 2x – 2x2 = 0
5(1 + x) – 2x(1 + x) = 0
(1 + x )(5 – 2x) = 0 ( ii ) Solve the equation C2 = 9
1 + x = 0 or 5 – 2x = 0 Solution
x = – 1 or 5 = 2x C2 = 9
C= 9
x = – 1 or 5/2 (E)
=3
1993/15 UME = + 3 or – 3
Solve the following equation
(3x – 2)(5x – 4) = (3x –2)2 (iii) Solve the equation 4q2 = 36
A. – 2/3, 1 B.1 C.2/3, 1 D.2/3, 4/5 Solution
Solution 4q2 = 36
First, we expand both sides of the equation q2 = 36
(3x – 2)(5x – 4) = (3x – 2)2 will become 4
3x(5x – 4) – 2(5x – 4) = (3x – 2) (3x – 2) q2 = 9
15x2 – 12x – 10x + 8 = 9x2 – 6x – 6x + 4 q = 9
15x2 – 22x + 8 = 9x2 – 12x + 4 =  3 i.e. + 3 or – 3
Collect like terms together
15x2 – 9x2 – 22x + 12x + 8 – 4 = 0 (iv) Solve the equation 16k2 = 49
6x2 – 10x + 4 = 0 Solution
Factors of ac i.e. 6 × 4 = 24 that when added will 16k2 = 49
k2 = 49
give b i.e. – 10. They are – 6 and – 4. This is also 16
coincidental – I mean the factors with respect to our ac
6x2 – 10x + 4 = 0 will become k = 49
6x – 6x – 4x + 4 = 0
2
16
6x(x – 1) – 4(x – 1) = 0
(x – 1) (6x – 4) = 0 =  49
x – 1 = 0 or 6x – 4 = 0 16
x = 1 or 6x = 4
x = 1 or 2/3 (C) =  7 i.e. + 7 or – 7
4 4 4
1993/3 (Nov ) Exercise 2.4 k = + 1 /4 or – 1 /4
3 3

Solve the equation 8x2 – 2x – 3 = 0


A. ½ or ¾ B. – 3/4 or –1/2 C. – 3/4 or ½
D. – /2 or ¾
1
E. ½ or /34

26
(v) Solve the equation 2b2 = 2 Completing the squares & general formula.
Solution From the general form of a quadratic equation ax2 +bx +c = 0, we
2b2 = 2 can derive the general formula by completing the squares method.
b2 = 2 The general formula is used to solve all quadratic problems that are
2 not factorisable..
b = 12 so b =  1 i.e. + 1 or – 1
2
General form of a quadratic equation
ax2 + bx + c = 0
applying completing the squares method;
When C = 0 Divide through by a to make the coefficient of x2 unity
Examples: Solve the following quadratic equation x2 + b/ax + c/a = 0
(i) x2 – 2x = 0 x2 + b/ax = – c/a
(ii) 2f2 + 3f = 0
Add the square of half the coefficient of x to both sides
(iii) 5m2 = 10m
i.e. 1 of b 2 = b 2
Cases of quadratic problems similar to the 2 a 2a
examples above are solved by factorizing out “one” x2 + b x + b 2 = – c + b 2
of the variable (unknown) and equating the whole a 2a a 2a
result to zero.
( i ) Solve x2 – 2x = 0 = – c + b2
Solution a 4a2
x2 – 2x = 0 = – 4ac + b2
Factoring out x, we have 4a2
x(x – 2) = 0 x + b 2 = b2 – 4ac
x = 0 or x – 2 = 0 2a 4a2
x = 0 or 2 Take the square root of both sides
x+ b =  b2 – 4ac
(ii) Solve the equation 2f2 + 3f = 0
2a 4a2
Solution
2f2 + 3f = 0 = b2 – 4ac
Factoring out f, 2a
f(2f + 3) = 0
f = 0 or 2f + 3 = 0 x=– b  b2 – 4ac
f = 0 or 2f = –3 2a 2a
f = 0 or – 3/2
x = – b  b2 – 4ac
2
(iii) Solve the equation 5m = 10m 2a
Solution Important property of the general formula
5m2 = 10m x = – b  D where D = b2 – 4ac
Rearranging 2a
5m2 – 10m = 0 D is called the discriminant.
5m(m – 2) = 0
The illustrative picture of the various roots are shown
5m = 0 or m – 2 = 0
m = 0 or m = 2
below
5
Since 0/5 is zero
m = 0 or 2

1991/22 UME D<0 D 0 D=0


Find the positive number n, such that thrice its square is Complex roots or two distinct two equal roots
equal to twelve times the number. Imaginary roots roots
A.1 B.2 C.3 D.4 in the case of D = 0 i.e equal roots implies
Solution b2 – 4ac = 0
3n2 = 12n b2 = 4ac take note
Rearranging,
1990/25 PCE
3n2 – 12n = 0
Solve the quadratic equation
Factoring out 3n,
cx2 – ax + b = 0
3n(n – 4) = 0
3n = 0 or n – 4 = 0 A. x = – b  b2 – 4ac B. x = b  b2 + 4ac
3n = 0 or n = 4 2a 2a
0 can not help us here C. x = – a  a2 + 4bc D. x = a  a2 – 4bc
So n = 4 (D) 2c 2c
27
Solution 1988/22 UME Exercise 2.6
Firstly, we compare our given equation with the The solution of the quadratic equation px2 + qx + b = 0 is
general form
A. – b  b2 – 4ac B. – b  p2 – 4pb
ax2 + bx + c , Before substituting into the General 2a 2a
formula
ax2 + bx + c = 0 and cx2 – ax + b = 0 C. – q  q2 – 4bp D. – q  p2 + 4bp
Here a = c, b = – a, c = b 2p 2p
Substituting into
x=–b b2 – 4ac
2a
Making a given incomplete quadratic equation a
= – (– a)  (– a)2 – 4 × c × b perfect square.
2×c The general formula for quadratic equation i.e.
= a a2 – 4bc (D) x=– b D
2c 2a
Where D is called the discriminant i.e. D = b2 – 4ac
1986/21 UME The useful aspect of the discriminant D to us is when D = 0
Solve the equation 3x2 + 6x – 2 = 0 This is a case where the equation (quadratic) has equal roots.
A. x = –1  3 B. x = – 1  15 The quadratic equation then is a perfect square
3 3 If D = 0 then
b2 – 4ac = 0 we have that
C. x = – 2  2 3 D. x = – 2  2 15
b2 = 4ac
3 3
We can conclude that if a quadratic equation is a perfect
Solution
square (complete square) then b2 = 4ac
It is obvious that the equation is not factorisable
Hence applying the general formula – we first of all
1989/24 UME
compare the given equation 3x2 + 6x – 2 = 0
What value of Q will make the expression
with the general form ax2 + bc + c = 0
4x2 + 5x + Q a complete square ?
a = 3, b = 6 and c = – 2
Substituting into A.25/16 B.25/64 C.5/8 D.5/4
Solution
x=–b b2 – 4ac To make 4x2 + 5x + Q a complete square then
2a b2 = 4ac
Comparing the given equation 4x2 + 5x + Q with
= –6 62 – 4 × 3 × (–2) ax2 + bx + c
2×3 a = 4, b = 5, c = Q
=–6 36 + 24 Substituting into b2 = 4ac
6 52 = 4 × 4 × Q
25 = 16Q
=–6 60 Q = 25/16 (A)
6 1990/24 PCE
Find the value of k which will make the quadratic
=– 6  4 × 15 expression 9x2 – 12x + k is a perfect square.
6 6 A.4/9 B. 4 C. 2/3 D. 16
= – 1  2 15 Solution
6 9x2 – 12x + k a perfect square
= – 1  15 if b2 = 4ac
3 (B) Comparing the given equation 9x2 – 12x + k with
ax2 + bx + c
1982/7 UME Exercise 2.5 Here b = – 12, a = 9, c = k
The solution of the quadratic equation bx2 +cx +a = 0 is Substituting into b2 = 4ac
given by (–12)2 = 4 × 9 × k
A. x = – c  c2 – 4ab B. x = – b  b2 – 4ac –12 × –12 = k
2b 2a 4×9
4 = k (B)
C. x = c  b2 – 4ac D. x = b  b2 + 4ac 1994/16
2b 2a Find the value of the constant k for which the equation
2x2 + (k + 3)x + 2k = 0, has equal roots
E. x = – c  c2 – 4ab A. 3 or 7 B. –1 or 0 C. –9 or –1
2b D. 9 or1 E. 1 or – 9
28
Solution 1989/31 UME Exercise 2.7
Condition for equal roots is: What will be the value of k so that the quadratic
b2 = 4ac equation
i.e (k + 3)2 = 4 × 2 × 2k kx2 – 4x + 1 = 0 has two equal roots ?
k2 + 6k + 9 = 16k A. 2 B. 3 C. 4 D. 8 E. ¼
k – 10k + 9 = 0
2

Factorizing
k2 – 9k – k + 9 = 0
k(k –9) – 1(k – 9) = 0
(k – 9)(k – 1) = 0
k – 9 = 0 or k – 1 = 0 Radical equations (Simple form)
k = 9 or 1 (D)
2003/1 (b) Nov 1992/12 UME
Find the possible values of m with which Solve the equation y – 11 y + 24 = 0
x2 + (m – 2)x + m + 1 = 0 has two equal roots . A. 8, 3 B. 64, 9 C. 6, 4 D. 9, -8
Solution Solution
Condition for two equal roots is; We solve by substitution
b2 = 4ac
i.e (m – 2)2 = 4 × 1 × (m + 1) Let t = y then t2 = y
m2 – 4m + 4 = 4m + 4 t2 – 11t + 24 = 0 becomes our new equation
m2 – 8m = 0 Factorizing
m(m – 8) = 0 t2 – 8t – 3t + 24 = 0
m = 0 or m – 8 = 0 t(t – 8) – 3(t – 8) = 0
m = 0 or 8 (t – 8)(t – 3) = 0
2002/3a Neco t – 8 = 0 or t – 3 = 0
For what value(s) of k does the equation t = 8 or 3
3y2 – 6y + k = 0 have two equal roots ? Substituting for t values into t2 = y
When t = 8, y = 64
Solution When t = 3, y=9 (B)
Condition for two equal roots is:
b2 = 4ac 1990/21 UME
(– 6)2 = 4 × 3 × k Find the solution of the equation x – 8 x + 15 = 0
36 = 12k A. 3, 5 B. -3, -5 C. 9, 25 D. -9, 25
i.e k = 3
Solution
1987/21 UME
Find the values of m which make the following Let t = x , then t2 = x
quadratic function a perfect square. t2 – 8t + 15 = 0 is our new equation
x2 + 2(m + 1)x + m + 3 Factorizing
A. –1, 1 B. –1, 2 C. 1, –2 D. 2, –2 t2 – 5t – 3t + 15 = 0
Solution t(t – 5) – 3(t – 5) = 0
The expression (t – 5)(t – 3) = 0
x2 + 2( m + 1) x + m + 3 is a perfect square t – 5 = 0 or t – 3 = 0
if b2 = 4ac t = 5 or 3
Comparing the given equation with ax2 + bx + c Substituting for t values
Here b = 2 (m + 1), c = m + 3 and a = 1 When t = 5, x = 25
Substituting into b2 = 4ac When t = 3 x = 9 (C)
[2(m + 1)]2 = 4 × 1 × (m + 3)
(2m + 2)2 = 4(m + 3) 2007/32 (Neco) Exercise 2.8
2
4m + 8m + 4 = 4m + 12 Find the solution of the equation
4m2 + 8m – 4m + 4 – 12 = 0 x+2 x – 8 = 0
4m2 + 4m – 8 = 0 A. –16, 4 B. –4, –16 C. 4 twice D. 16, 4 E. 2, –4
Divide through by 4
m2 + m – 2 = 0 REQ 1 Exercise 2.9
Factorising Find the solution of the equation
m2 + 2m – m – 2 = 0
m(m + 2) – 1(m + 2) = 0 x–5 x + 6 = 0
(m + 2) (m – 1) = 0 REQ 2 Exercise 2.10
m + 2 = 0 or m – 1 = 0 Solve the equation
m = – 2 or + 1 (C)
d + 10 d – 24 = 0

29
Radical equation II 2003/27 Neco Exercise 2.11
Here the key word is isolation of the radical, if it is not Solve the equation
already isolated, next we square both sides. The preceding
examples will show the ways to follow: a + 12 = 2 + a
2004/4 Neco A. 2 B. – 2 C. 4 D. 12 E. 16
Solve the equation
Simultaneous Linear and Quadratic equations
x+5 = 5 – x 2000/8 Neco
Solution Solve the equations:
Isolating the radicals x + 2y = 8
x+5 + x = 5 x – xy + 2y2 = 16
2

Squaring both sides Solution


( x+5 + x )2 = 52 x + 2y = 8 -----------(1)
x – xy + 2y2 = 16 --------- (2)
2

x + 5 + 2 (x + 5)x + x = 25 From (1) x = 8 – 2y


Next, we substitute into (2) and not x = 4 – y it is premature reduction.
2x + 5 + 2 (x + 5)x = 25 (8 – 2y) 2 – y (8 – 2y) + 2y2 = 16
Isolating the radical again 64 – 32y + 4y2 – 8y + 2y2 + 2y2 = 16
Re – arranging
2 (x + 5)x = 20 – 2x 8y2 – 40y + 48 = 0
Reducing further, we divide both sides by 2 We can now reduce terms, if it can
(x + 5)x = 10 – x y2 – 5y + 6 = 0
Squaring both sides Factorizing
(x + 5)x = (10 – x)2 y2 – 3y – 2y + 6 = 0
x2 + 5x = 100 – 20x + x2 y (y – 3) – 2(y – 3) = 0
Some terms will cancel out (y – 3)(y – 2) = 0
25x = 100 y = 3 or 2
x=4 Substituting for y values into x = 8 – 2y
Checking our result just to be sure When y = 3, x = 8 – 2(3) i.e 2
When y = 2, x = 8 – 2(2) i.e 4
x+5 = 5 – x will become
9 = 5 – 4 which is true 2002/16 Neco
What are the values of x and y in the simultaneous equation
2003/9a Neco (Dec ) below?
Solve the equation and check your answer; 2x + y = 4
2x + 9 = 1 + x+4 (x – y)2 = 25
Solution A. x = -3 or 1/3, y = 8 or 42/3
Isolating the radicals B. x = 3 or 1/3, y = -8 or 42/3
C. x = 3 or - 1/3, y = 8 or 42/3
2x + 9 – x+4 = 1 D. x = 8 or 4/3, y = 3 or 1/3
Squaring both sides E. x = -8 or 2
/3, y = - 3 or -1/3
2x + 9 – 2 ( 2x + 9)(x + 4) + x+4 = 1 Solution
Isolating the radical 2x + y = 4 --------------- (1)
3x + 13 – 1 = 2 (2x + 9)(x + 4) (x – y)2 = 25--------------- (2)
From (1) y = 4 – 2x
3x + 12 = 2 (2x + 9)(x + 4) Expanding LHS of (2)
Squaring both sides (x –y)2 = 25 will become
(3x + 12)2 = 4(2x + 9)(x + 4) (x – y)(x – y) = 25
9x2 + 72x + 144 = 4(2x2 + 17x + 36) x2 – 2xy + y2 = 25
9x2 + 72x + 144 = 8x2 + 68x + 144 Substituting for y = 4 – 2x
x2 + 4x = 0 x2 – 2x(4 – 2x) + (4 – 2x)2 = 25
x(x + 4) = 0 x – 8x + 4x2 + 16 – 16x + 4x2 = 25
2

x = 0 or – 4 9x2 – 24x – 9 = 0
Checking the result as demanded by the question Reducing further
When x = 0 3x2 – 8x – 3 = 0
Factorising
9 = 1 + 4 true 3x2 – 9x + x – 3 = 0
When x = – 4 3x(x – 3) + 1(x – 3) = 0
(x – 3)(3x + 1) = 0
1 = 1 + 0 true x – 3 = 0 or 3x + 1 = 0
30
x = 3 or – 1/3 2002/13
Substituting for x values into y = 4 – 2x Solve the equation;
When x = 3; y = 4 – 2(3) i.e – 2 x2 – 9y2 = 24
When x = –1/3; y = 4 – 2(– 1/3) i.e 42/3 x – 3y = 8
A. x = –5/6, y = –11/ 2 B. x = – 5/6, y = 11/2
C. x = /2, y = – /6
11 5
D. x = 2/11, y = 6/5
2000/9 (Nov) Solution
Solve the equations x2 – 9y2 = 24 ------------- (1)
x+y=1 x – 3y = 8 -------------- (2)
x2 +y2 = 13 From (2) x = 8 + 3y
A. x = –2, y = –3 or x = –1, y=2 Substituting for x into (2)
B. x = –2, y = 3 or x = 3, y = –2 (8 + 3y)2 – 9y2 = 24
C. x = 4, y = –3 or x = –1, y=2 64 + 48y + 9y2 – 9y2 = 24
D. x = 3, y = 3 or x = –2, y = –2 48y = 24 – 64
Solution 48y = – 40
x + y = 1-------------- (1) y = –40/48
x2 + y2 = 13------------ (2) y = –5/6
From (1), y = 1 – x and substitute into (2) Substituting for y into x = 8 + 3y
x2 + (1 – x)2 = 13 x = 8 + 3(–5/6)
x + 1 – 2x + x2 = 13
2 x = 8 – 5/2
2x2 – 2x – 12 = 0 x = 11/2 (B)
Reducing 1986/11 UME
x2 – x – 6 = 0 The ages of Tosan and Isa differ by 6 and the product of their ages
Factorizing is 187. Write their ages in the form (x, y) when x  y
x2 – 3x + 2x – 6 = 0 A.(12, 6) B.(23, 17) C.(17, 11) D.(18, 12)
x(x – 3) + 2(x – 3) = 0 Solution
(x – 3)(x + 2) = 0 The equation are;
x – 3 = 0 or x + 2 = 0 x – y = 6 and xy = 187
x = 3 or x = –2 From x – y = 6
Substituting for x value into y = 1 – x x =6+y
When x = 3; y=1–3 i.e –2 Substitute x = 6 + y value into xy = 187
When x = –2; y = 1 – (–2) i.e 3 (B) (6+ y)y = 187
6y + y2 = 187
Rearranging
2000/11 y2 + 6y – 187 = 0
Solve the equations Factorizing
x + y = 3, y2 + 17y – 11y – 187 = 0
x2 + y2 – 2y = 1 y (y + 17) – 11 (y + 17) = 0
A. x = 1/2, y = 5/2 B. x = 1, y = 2 (y + 17) (y – 11) = 0
C. x = 2, y = 1 D. x = 1/2 , y = 1 y + 17 = 0 or y – 11 = 0
Solution y = – 17 or 11
x + y = 3, --------- (1) But age can not be negative so y = 11
x2 + y2 – 2y = 1---------- (2) Substitute y = 11 value into x – y = 6
From (1) y = 3 – x x – 11 = 6
Substituting for y into (2) x = 6 + 11
x2 + (3 – x)2 – 2(3 – x) = 1 x = 17
x + 9 – 6x + x2 – 6 + 2x = 1
2 Ages (x, y) = (17, 11) ( C)
2x2 – 4x + 3 = 1
2x2 – 4x + 2 = 0 1996/2 Exercise 2.12
Reducing further Solve the simultaneous equation
x2 – 2x + 1 = 0 x–y=3;
Factorizing x2 – y2 = 9
x2 – x – x + 1 = 0 A. x = 0, y = 3 B. x = 6, y = 0 C. x = 0, y = 6
x(x – 1) –1(x – 1) = 0 D. x = 3, y = 0 E. x = 3, y = –3
(x – 1)(x – 1) = 0
x – 1 = 0 twice 2003/20 Exercise 2.13
x = 1 twice Which of the following is a solution to the equation
Substituting for x = 1 into y = 3 – x x + y = 10
When x = 1, y = 3 – 1 i.e 2 (B) x2 – 2xy + y2 = 36 ?
A. x = -2, y = 12 B. x = -2, y = 8
C. x = -8, y = 2 D. x = 8, y = 2
31
1991/20 UME Exercise 2.14 x2 + x – 6
Find the two values of y which satisfy the simultaneous
x +1 x3 + 2x2 – 5x – 6
equation 3x + y = 8 , x2 + xy = 6
– ( x 3 + x 2)
A. – 1 and 5 B. – 5 and 1 C. 1 and 5 D. 1 and 1
x2 – 5x
– ( x2 + x )
2001/10 UME Exercise 2.15
– 6x – 6
Solve the equations; m2 + n2 = 29, m + n = 7
– ( – 6x – 6)
A.(5, 2) and (5, 3) B.(5, 3) and (3, 5)
0 0
C. (2, 3) and (3, 5) D. (2, 5) and (5, 2)
The resultant quadratic x2 + x – 6 is factorized as
Cubic equation x + 3x – 2x – 6 = 0
2

2002/28 x(x + 3) – 2 (x + 3 ) = 0
Solve for x in the equation (x – 2)(x + 3 ) = 0
x3 – 5x2 – x + 5 = 0 x – 2 = 0 or x + 3 = 0
A. – 1, 1 or – 5 B.1, –1 or 5 C.1,1 or 5 D.1,1 0r –5 x = 2 or –3 (B) answer x = -3 , -1 , 2
Solution
First we get one zero of the function by trial and error 1998/17 PCE
usually 1 or –1 y

Try x = 1
(1)3 –5(1)2 – 1 + 5 = 1 – 5 + 4 -2 0 1
x
= 5 – 5 i.e. 0
Thus, x = 1 is a zero of the function then x – 1 is a
factor and not x + 1. Dividing we have Which of the following functions is represented by the curve
x2 – 4x – 5 above?
A. y = x3 – 2x2 – x B. y = x3 + x2 – 2x
x –1 x3 –5x2 – x + 5 C. y = x3 – x2 – 2x D. y = x3 – x2 + 2x
– ( x3 – x2 )
Solution
– 4x2 – x
The zeros of the function are: – 2, 0 and 1 then
– ( – 4x2 + 4x) x = – 2 or 0 or 1
– 5x + 5 x + 2 = 0 or x = 0 or x – 1 = 0 implies
– ( – 5x + 5) x ( x + 2 )( x – 1 ) = 0 We simply expand
0 0 x [x2 – x + 2x – 2 ] = 0
x [x2 + x – 2 ] = 0
The resultant quadratic x2 – 4x – 5 is factorized as
x3 + x2 – 2x = 0 ( B )
(x – 5)(x + 1 ) = 0
x – 5 = 0 or x + 1 = 0 2005/49
x = 5 or –1 (B) A polynomial in x whose zeros are – 2, –1 and 3 is
A.x3 + 7x + 6 B.x3 – 7x + 6 C.x3 – 7x – 6 D.x3 + 7x – 6
Solution
If – 2, –1 and 3 are zeros of the polynomial then
x = – 2 or –1 or 3
x + 2 = 0 or x + 1 = 0 or x – 3 = 0
( x + 2 )( x + 1 )( x – 3 ) = 0 We simply expand
2004/31 [x2 + x + 2x + 2 ]( x – 3 ) = 0
Find the values of x where the curve [x2 + 3x + 2 ]( x – 3 ) = 0
x3 + 3x2 + 2x – 3x2 – 9x – 6 = 0
y = x3 + 2x2 – 5x – 6 crosses the x – axis.
x3 – 7x – 6 = 0 ( C )
A. – 2, 1 and –3 B.2, – 1 and – 3
C.2, 1 and 3 D. – 2, –1 and 3 1992/17 PCE Exercise 2.16
Factorize x3 + 4x2 + x – 6
Solution
A. (x + 1) (x+2) (x –3) B.(x –1) (x + 2) (x +3)
First we get one zero of the function by trial and error
C.(x +1) (x–2) (x +3) D. (x –1) (x-2) (x – 3)
usually 1 or –1
2006/32 UME Exercise 2.17
Try x = 1 Find the roots of x3 – 2x2 – 5x + 6 = 0
(1)3 +2(1)2 – 5(1) – 6 = 1 + 2 – 5 – 6 A. 1, 2 , –3 B. 1, – 2, 3 C. –1, 2, –3 D. –1, – 2, 3
= – 8 not zero
Try x = – 1 2005/41 PCE Exercise 2.18
(– 1)3 +2(– 1)2 – 5(– 1) – 6 = –1 + 2 + 5 – 6 Factorize x3 – 6x2 + 11x – 6 completely, given that x – 3 is a factor
= 0 o.k A. (x –3)(x – 2) (x – 1) B. (x –3)(x – 2) (x + 1)
Thus, x = –1 is a zero of the function then x + 1 is a C. (x –3)(x +2) (x + 1) D. (x –3)(x + 2) (x – 1)
factor and not x – 1. Dividing we have
32
Deriving of quadratic equation with Solution
known roots. The variable here is m not x. so the formula will be adjusted
Given any two roots of a quadratic equation, we can accordingly as;
derive the required equation. m2 – (sum of roots) m + product of roots = 0
Examples: m2 – –3+ 4 m + –3 × 4 = 0
If 2 and 3 are the roots of a quadratic equation, then the 4 3 4 3
equation is.
Solution m2 – –9 +16 m + ( – 1 ) = 0
Let x be our variable (unknown) 12
Then x = 2 or 3 m2 – 7 m – 1 = 0
x = 2 or x = 3 12
x – 2 = 0 or x – 3 = 0 Multiply through by 12 to clear fraction
(x – 2)(x – 3) = 0 12m2 – 7m – 12 = 0 (C)
x(x – 3) – 2(x – 3) = 0 1992/3 Nov
x2 – 3x – 2x + 6 = 0 If the root of x2 – 7x + k = 0 are m and m – 1, find the value
x2 – 5x + 6 = 0 of the constant k.
The above method can be generalized as: A. – 4 B. 3 C. 4 D. 12 E. 16
Let m and n be the roots of any quadratic equation
Then Solution
x = m or n First, we find m from sum of root
x = m or x = n m + (m – 1) = 7 i.e  +  = – b/a
x – m = 0 or x – n = 0 2m = 8
(x – m)(x – n) = 0 m = 4
x(x – m) – n(x – m) = 0 The roots of the equation are m and m – 1 implies
x2 – mx – nx + mn = 0 The roots of the equation are 4 and 3
x2 – (m + n)x + mn = 0 ----*** i.e x = 4 or 3
m + n implies sum of roots x – 4 = 0 or x – 3 = 0
mn implies product of roots (x – 4)(x – 3) = 0
We can restate *** as Expanding
x2 – (Sum of roots )x + product of roots x2 – 3x – 4x + 12 = 0
We may use any of the two formulae above depending x2 – 7x + 12 = 0
on ability to recall. Comparing the result with the initial equation
x2 – 7x + 12 = 0 and x2 – 7x + k = 0
1984/23 UME It follows that k = 12(D)
The quadratic equation whose roots are
Symmetric functions of roots
1– 13 and 1 + 13 is Given that  and  are the roots of a quadratic equation then;
x =  or 
A.x2 + (1 – 13 )x + 1 13 = 0
x –  = 0 or x –  = 0
B. x2 + (1 + 13 x + 1 – 13 = 0 C. x2 + 2x + 12 = 0 (x – ) (x – ) = 0
D.x2 – 2x + 12 = 0 E. x2 – 2x – 12 = 0 x(x – ) – (x – ) = 0
Solution x2 – x – x +  = 0
The formula is x2 – ( + )x +  = 0
x2 – (sum of root)x + product of roots = 0 i.e. x2 – ( + )x +  = 0 --------*
comparing * with the general form of quadratic equation
Sum of roots = 1 – 13 + 1 + 13 = 2
ax2 + bc + c = 0.
Product of roots = (1 – 13 ) ( 1 + 13 ) (Making the coefficient of x2 unity )
Applying surd principles x2 + b/a x + c/a = 0 ------**
= 12 – 13 i.e 1 – 13 we have that from * and **
= – 12 – ( + ) = b/a
Substituting into the formula;  +  = – b/a and  = c/a
x2 – (2)x + (–12) = 0 These result for  +  and  can enable us solve quadratic
x2 – 2x – 12 = 0 (E) problem without actually finding the roots of the equation
Some important derivations are;
1994/16 UME 2 + 2 = ( + )2 – 2
Find the quadratic equation in m whose roots are
– 3/4 and 4/3 ( + )2 = ( – )2 + 4 or ( – )2 = ( + )2 – 4
A. 2m2 + 7m – 12 = 0 B.12m2 – 9m – 12 = 0 3  3 = (  ) (2   + 2)
C.12m – 7m – 12 = 0
2
D.12m2 + 9m – 12 = 0 3 + 3 = ( + ) (2 + 2 – )
3 + 3 = ( + ) [( + )2 – 3]
33
2002/36 Neco From the given equation: x2 + px + q = 0,
If the roots of 2x2 + 5x + 3 = 0 are  and ,  +  is – p and  is q
what is the value of 2 + 2 ? Substituting
A. 91/4 B. 31/4 C. – 31/4 D. –51/2 E. –91/4 3 + 3 = (– p) [(– p)2 – 3q]
Solution = – p(p2 – 3q)
Recall that 2 + 2 = ( + )2 – 2  = – p3 + 3pq
From 2x2 + 5x + 3 = 0 = 3pq – p3 QED
 +  = – 5/2 and  = 3/2
Substituting 1992/10 (Nov)
2 + 2 = (– 5/2)2 –2(3/2) If p and q are the roots of the equation
= 25/4 – 6/2 3x2 – 16x + 2 = 0, find the value of p + q(1 + p)
= 25 – 12 A. – 14 B. – 6 C. – 42/3 D. 6 E. 18
4 Solution
= 13/4 ie 31/4 (B) Note here and always that ,  are dummy variables
2003/25 Neco hence their replacement with p and q.
If  and  are the roots of the equation 2x2 – 6x + 5 = 0, p + q(1 + p) = p + q + pq
Evaluate  +  i.e  +  + 
  From the given equation: 3x2 – 16x + 2 = 0
A. 24 8
B. /5 C. 5/8 D. 1/24 E – 8/5  +  = 16/3 ;  = 2/3
16 2
Solution p + q + pq = /3 + /3
we simplify  +  = 2 + 2 i.e 2 + 2 = 18/3 i.e 6 (D)
    1993/11 (Nov)
If  and  are the roots of the equation 4 + 3x – 2x2 = 0 find
= (  + )2 – 2 the value of 1 + 1
  2
2
From 2x2 – 6x + 5 = 0 A. 9/4 B. 25/16 C. 9/16 D. – 7/4 E. – 7/16
 +  = 6/2 and  = 5/2 Solution
1 + 1 = 2 + 2
= 32 – 2( 5/2 )
2 2 2 2
5
/2
= 2 + 2
= ( 9 – 5)  5/2
( )2
= 4 × 2/ 5 = ( + )2 – 2 
= 8/5 ( B )
2003/10(i) Neco ( )2
If the sum of the squares of the roots of the equation From the given equation: 4 + 3x – 2x2 = 0
ax2 + bx +c = 0 is 1, show that b2 = 2ac + a2  +  = –3/–2 i.e 3/2 and  = 4/–2 i.e – 2
Solution
We are informed that: 2 + 2 = 1 = (3/2)2 – 2(–2)
But 2 + 2 = ( + )2 – 2 (–2)2
From the given equation : ax2 + bx +c = 0 9
= /4 + 4
 +  = – b/a and  = c/a
4
Substituting
= 25/16 (B)
2 + 2 = ( + )2 – 2
1 = (– b/a)2 – 2 a/c
Joint problems
1 = b2 – 2c 1997/10
a2 a If  and  are the roots of the equation 3x2 – 7x + = 0, Find
1 = b – 2ac
2 the value of
a2 1
( i ) 2 + 2 ( ii ) + 1
a = b – 2ac
2 2
 
Thus, b2 = 2ac + a2 QED (iii) Hence form the equation whose roots are
1
2005/8 (i) Neco 2 + 1 and 2 +
If  +  are the root of x2 + px + q = 0,  
Prove that 3 + 3 = 3pq – p3 Solution
Solution  +  and   of the given equation 3x2 – 7x + 5 = 0
 + 3 = ( + ) (2 –  + 2 )
3
 +  = 7/3 and  = 5/3
= ( + ) [ ( + )2 – 3]
34
( i ) 2 + 2 = ( + )2 – 2 2003/32 Neco Exercise 2.22
= ( 7/3 )2 – 2(5/3) Which of the following identities correctly
represents 3 + 3 ?
= 49/9 – 10/3 ie 19/9
A.(  - ) [( - )3 + 2] B. ( + ) [ ( + )2 - 2]
(ii) 1
/ + 1/ =  +  ie  + C.(  - ) [( - )2 + 3] D. ( - ) [( - )2 - 3]
  E.(  + ) [( + )2 - 3]
= /3 ÷ 5/3
7
Equations formed from defined roots
= 7/3 × 3/5 1996/4 (Nov)
7 2
= /5 i.e 1 /5 The quadratic equation whose roots are 3 - 3 and 3 + 3 is
A. x2 – 6x + 6 = 0 B. x2 + 6x + 6 = 0
(iii) Sum of roots of new equation 2
C. x + 6x + 9 = 0 D. 6x2 – x + 6 = 0
(2 + 1/) + (2 + 1/ ) = 2 + 2 + 1/ + 1/ E. 6x2 + 6x + 1 = 0
19
From ( i ) and ( ii ) results = / 9 + 7/ 5 Solution
The general quadratic forms of sum and product of roots is
= 95 + 63 = 158/45
45 x2 – ( + )x +  = 0
Product of roots of new equation From the given roots
(2 + 1/)(2 + 1/ ) = 2(2 + 1/ ) + 1/ (2 + 1/ )  +=3– 3 +3+ 3
= 6
= 22 +  +  + 1
2 2

   Also  = (3 – 3 ) (3 + 3 )
1 = 32 – 3 i.e 6
= () +  +  +
2
Substituting x2 – 6x + 6 = 0 (A)

= ( 5/3 )2 + 7/3 + 3/5 2005/30 Neco
25 7 3 Find the equation whose roots are the squares of the roots of
= /9 + /3 + /5
2a2 + 3a + 7 = 0
= 125 + 105 + 27 = 257 A. 4a2 – 18a + 49 = 0 B. 4a2 + 19a + 49 = 0
45 `45 C. 9a – 4a + 9 = 0
2
D. 9a2 – 4a + 49 = 0
E. 9a – 18a + 9 = 0
2

New equation x2 – (sum of root)x + product of root = 0 Solution


x2 – 158/45 x + 257/45 = 0 The new equation is to have the form
45x2 – 158x + 257 = 0 x2 – (2 + 2)x + 22
From our given equation: 2a2 + 3a + 7 = 0
2002/8 (Nov)  +  = – 3/2 and  = 7/2
If  and  are the of the equation 2x2 – 3x – 7 = 0,
But 2 + 2 = ( + )2 – 2  
find the value of ( + 1)( + 1).
A. – 4 B. – 3 C. – 3/2 D. – 1 = (– 3/2)2 – 2(7/2)
Solution = 9/4 – 7 i.e – 19/4
( + 1)( + 1) =  +  +  + 1 Also 2 2  ( )2 = ( 7/2)2 i.e. 49/4
From 2x2 – 3x – 7 = 0;  +  is 3/2 while  is – 7/2 Therefore the new equation is
( + 1)( + 1) = – 7/2 + 3/2 +1 a2 + 19/4 a + 49/4 = 0
–7 + 3 + 2 Clearing fraction
2 4x2 + 19x + 49 = 0 (B)
= – 2/2 ie –1 (D) 2005/8 (ii) Neco
If  and  are the roots of x2 + px + q = 0,
2000/9c Exercise 2.19
prove that: the equation whose roots are  – –1
If  and  are the roots of the equation 3x2 – 5x + 4 = 0,
and  – –1 is qx2 + p(q – 1)x + (q – 1)2 = 0
find the value of / + /
Solution
From the given equation: x2 + px + q = 0,
2000/1 (Nov) Exercise 2.20
 +  is – p and  is q
If  and  are the roots of 2x2 – 3x + 6 = 0,
We are to form the equation whose roots are  – 1/
evaluate: ( a ) 1/ + 1/ ( b ) / + /
and  – 1/
The sum of roots: ( – 1/) + ( – 1/ )
2002/6 Exercise 2.21
If  and  are the root of the equation 2x2 – 4x + 7 = 0, =  +  – 1/ – 1/
find 2 + 2. =  +  – [1/ +1/]
A. -3 B. -2 C. 4 D. 11
= + –  +

35
Substituting = –p – –p Its product of roots is :
q –3 –3 = 9
  
= – p + p/q
= 9 ÷ (– ¾ )
= – pq + p
q = – 9 × 4/3
Product of roots: ( – 1/) ( – 1/ ) = – 12
=   – / – / + 1/  The new equation
x2 – (– 5)x + (–12) = 0
=   + 1/   – 2 x2 + 5x – 12 = 0 (C)
Substituting = q + 1/q – 2
2003/23 (Nov)
= q2 + 1 – 2q ie q2 – 2q + 1
q q If  and  are the roots of the equation 2x2 – 3x – 1 = 0, find
Factorizing the numerator the equation whose roots are 1/ and 1/
= ( q – 1 )2 A. x2 – 3x – 2 = 0 B. x2 + 3x – 2 = 0
q C. x2 – 3x + 2 = 0 D. x2 + 3x + 2 = 0
The new equation is Solution
x2 – – pq + p x + ( q – 1)2 = 0 x2 – ( + )x +  = 0 is the general form.
q q From our given equation: 2x2 – 3x – 1 = 0
Multiply through by q to clear fraction  +  = 3/2 and   = – ½
qx2 – (– pq + p)x + (q – 1)2 = 0
Factor out – p from (– pq + p) The new equation sum of roots is:
1
qx + p(q – 1)x + (q – 1)2 = 0 QED
2 / + 1/ =  + 

1994/1 = /2 ÷ (– 1/2 )
3

If  and  are the roots of the equation x2 – 2x – 5 = 0, = – 3/2 × (– 2/1 ) i.e –3


find the equation whose roots are 2  and  2 Its product of roots is:
Solution 1 1 = 1
From the given equation: x2 – 2x – 5 = 0,   
 +  = 2 and  = – 5 = 1 ÷ (– 1/2 )
The new equation sum of roots = 1 × (– 2/1 ) i.e – 2
2  +  2 =  ( + ) The new equation is:
= – 5(2) i.e –10 x2 – (–3)x + (–2) = 0
Its product: (2 ) ( 2) = 3 3 x2 + 3x – 2 = 0 (B)
= ( )3
= (– 5)3 i.e –125 2001/8b Neco Exercise 2.23
Substituting into the general equation format If  and  are the roots of the equation 3x2 – 2x – 1 = 0,
x2 – (–10)x + (– 125) = 0 Find the equation whose roots are  and 
x2 + 10x – 125 = 0  
2001/4 Neco Exercise 2.24
2000/15 If  and  are the roots of 1 x2 + 3x – 2 = 0,
If  and  are the roots of 4x2 + 5x – 3 = 0, 4
find the equation whose roots are – 3/ and – 3/ 2 2
the value of + is
A. x2 – 5x + 12 = 0 B. x2 + 5x – 3 = 0  
C. x2 + 5x – 12 = 0 D. x2 + 12x – 5 = 0 A.-20 B. -3 C. 3 D. 8 E. 12
Solution 2001/6 Neco Exercise 2.25
General form of the required equation is The quadratic equation whose roots are log2 4 and log2 1 is
x2 – ( + )x +  = 0 2
From our given equation : 4x2 + 5x – 3 = 0, A.x2 – x – 2 = 0. B. x2 – 3x – 4 = 0.
 +  = – 5/4 and  = – ¾ C.x2 – 3x + 4 = 0. D. x2 – 3x + 2 = 0. E. x2 + x – 2 = 0.
The new equation sum of root 2008/10 a Exercise 2.26
– 3/ + (– 3/) = –3 – 3 If  and  are the roots of the equation 2x2 + 5x – 6 = 0,
 find the equation whose roots are ( – 2) and ( – 2)
= –3( + )
2009/9 Exercise 2.27
 The roots of the quadratic equation 2x2 – 5x + m = 0 are
Substituting = –3 (– 5/4) ÷ ( – 3/4 )  and  , where m is a constant. Find (2 + 2 ) in terms of m
= /4 × ( – 4/3 )
15
A
25
m B 25  2m C 25  m D 25  2m
= –5 4 4 4 4
36
2000/22 UME Exercise 2.28 2x2 – 9x – 4 + 3 = 3
If  are  are the roots of the equation 3x2 + 5x – 2 = 0. 2x2 – 9x – 1 = 3
Find the value of 1/ + 1/  On the graph, draw a line at point y = 3 to cut the curve.
A. – 5/2 B. – 2/3 C. 1/2 D. 5/2 These (two) points traced to x – axis gives the solution:
-0.4 & 4.9
2000/12 PCE Exercise 2.29 (c) (ii) At point x = 3 traced to the curve, we draw an
equidistant line as shown, then take the slope.
If the roots of ( 1/2x – 1 ) ( x + 1 ) = 0 are  and ,
Slope = Change in y
Find the value of  +  Change in x
A.1/2 B. 1 C. 2 D. 3 = -5 – (-15) = 2.78
4.8 – 1.2
1996/10 PCE Exercise 2.30
If  + are the roots of the equation 3x2 – 5x + 6 = 0. 1988/8 (ordinary maths )
Find the value of / + / The table below is for the relation y = 2 + x – x2
A. 11/9 B.11/18 C. – 11/18 D. – 11/9 x -2 -1.5 -1 -0.5 0 0.5 1 1.5 2 2.5 3
y -4 -1.75 0 1.25 2 2.25 2 1.25 0 -1.75 -4

Graphical method of solving quadratic equations


(a) Using a scale of 2cm to 1 unit on each axis, draw
1994/10 (ordinary maths )
(a)Copy and complete the following table of values the graph of the relation in the interval – 2  x  3
for y = 2x2 – 9x – 1 (b) From your graph, find the greatest value of y and the
value of x for which this occurs.
x -1 0 1 2 3 4 5 6 (c) Using the same scale and axis, draw the graph of
y -1 -8 -11 17 y = 1 – x.
(b)Using a scale of 2cm to represent 1 unit on the (d) Use your graphs to solve the equation 1 + 2x – x2 = 0
x - axis and 2 cm to represent 5 units on the y - axis, Analysis and Solution
draw the graph of y = 2x2 – 9x – 1 for -1  x  6 From the table , we observe that:
(c) Use your graph to find the: x – axis
(i) roots of the equation 2x2 – 9x = 4 correct to Has bigger (+ ve side) 3
1 decimal place Has smaller (-ve side ) – 2
(ii) gradient of the curve y = 2x2 – 9x – 1 at x = 3 y - axis
Analysis and Solution Has smaller ( + ve sides) 2.25
(a) Table for y = 2x2 – 9x - 1 Has bigger (-ve side) – 4
Graph format:
x -1 0 1 2 3 4 5 6
2x2 2 0 2 8 18 32 50 72
-9x 9 0 -9 -18 -27 -36 -45 -54
-1 -1 -1 -1 -1 -1 -1 -1 -1

y 10 -1 -8 -11 -10 -5 4 17
( a ) Graph is as shown on page 38
(b) From the table we observe that:-
x – axis (b) y = 2.25 and x = 0.5
Has very small – ve (-1)
Has very big + ve (6) ( c )Using three non-too close points from the given range,
y – axis we have the table as;
Big negative side (-11) compared to Table for y = 1 – x
bigger + ve side (17) x -2 -1 1
Thus the graph will be ruled in the format below: 1 1 1 1
-x 2 1 -1
y 3 2 0

(d) The equation 1 + 2x – x2 = 0


is a combination of the equations. Let us confirm it;
y = 2 + x – x2 and y = 1 – x then, it follows that
2 + x – x2 = 1 – x
Graph is as shown on page 38 2 – 1 + x + x – x2 = 0
(c) (i) Rearranging the equation, 1 + 2x – x2 = 0
2x2 – 9x = 4 Thus the solution to the resultant equation is the x
2x2 – 9x – 4 = 0 values of the points of intersection of the two graphs
Add 3 to both sides to get our original equation, x = - 0.4 and 2. 4
37
20
y 1994/10 Graph

y = 2x2 – 9x – 1
15

10

x
0
-2 -1 0 1 2 3 4 5 6 7

-5

-10

-15

3.5 1988/8 Graph


y
3

y=1–x 2.5

1.5

0.5 x
0
-2.5 -2 -1.5 -1 -0.5 0 0.5 1 1.5 2 2.5 3 3.5
-0.5

-1

-1.5

-2

-2.5

-3

-3.5

-4
y = 2 + x – x2
-4.5

38
Chapter three ( iv ) Solve the inequality x +3  4x
Inequalities Solution
Equality sign (=) does not always meet our need in x + 3  4x
mathematics. Cases such as Bread is more or less Collect like terms together
carbohydrate, Nigeria’s naira is less valued compared to x – 4x  – 3
the American dollar ($) will not fit equality sign. Thus, the use – 3x  – 3
of inequality signs: Divide both sides by the coefficient of x i.e –3
< “is less than” – 3x  – 3
> “ is greater than” – 3 –3
 “is greater than or equal to” x  1
 “is less than or equal to” Checking
Strict inequalities Let’s try the values of x
Less than < and greater than > are called strict When x = 1
inequalities. x + 3  4x will be
Examples. 1 + 3  4(1) true
(1) x > 3 means x could be any of 4,5,6… But x can not be 3. When x = 2
(2) y < 5means y can take values 4,3,2,1,0,-1,-,2… But not 5 x + 3  4x will be
Weak inequalities 2 + 3  4 (2) true
Greater than or equal to  and Less than or equal to  i .e 5 < 8
are called weak inequalities. The phrases “at most” and When x = 3
“at least” are used for weak inequalities too. Here at x + 3  4x will be
most means  while at least is  3 + 3  4(3) true
Examples i.e. 6 < 12
(1) W  12 means W can take values 12,13,14,…
Let’s now try the non-reversed inequality i.e.
(2) x  12 means x can take values 12 ,11,10,… – 3x  – 3
Linear inequalities in one variable Divide both sides by –3 but do not reverse the sign as some
Basic operational principles with linear equations apply people will argue - that both sides are negative so no need for
to linear inequalities except when we multiply or divide sign change when dividing by a negative number
both sides of the equation by a negative number – then – 3x  – 3
we reverse the inequality symbol concerned. –3 –3
Examples x 1
( i ) Solve 3x – 4 > 8 Lets substitute for x values
Solution When x = 1
3x > 8 + 4 x + 3  4x
3x > 12 1 + 3  4 (1) true
Divide both sides by the coefficient of x i.e 3 When x = 0
x > 12 x + 3  4x will be
3
0 + 3  4(0)
x>4
i.e. 3  0 is it true?
( ii ) For what values of t is ½ (x + 3) < 3 When x = – 1
Solution x + 3  4x will be
½ ( x + 3) < 3 – 1 + 3  4(– 1)
To clear fraction, we multiply both sides by 2 i.e. 2  – 4 is it true?
x+3<6 Please adhere strictly to sign changes when dividing or multiplying
x < 6 –3 both sides of inequality equations by negative numbers
x<3 1990/27 PCE
( iii ) Solve the inequality 6x – 14  13x Solve the inequality y + 9 > 5y – 7
Solution A y > 4 B. y > – 4 C. y < 4 D. y < – 4
6x – 14  13x Solution
Collect like terms together y + 9 > 5y – 7
6x – 13x  14 Collect like terms together
y –5y > – 7 – 9
–7x  14
– 4y > – 16
Divide both sides by the coefficient of x i.e –7
Divide both sides by the coefficient of x i.e – 4
–7x  14
and reverse sign
–7 – 7 – 4 < – 16
Notice the inequality symbol changes from  to  – 4 – 4
x  –2 y < 4 (C)
39
1987/26 UME Linear inequalities in one variable involving fractions
Solve the inequality x – 1 > 4 (x + 2) The use of LCM of the denominator to multiply through the
A. x > – 3 B. x < – 3 C. 2 < x < 3 D. – 3 whole fractional inequality is very essential here.
<x<–2
Solution 1981/5 UME
x –1 > 4(x + 2) Find the value of x satisfying.
Open up the bracket in the RHS x – 1 < 2x + 1
x –1 > 4x + 8 2 3 5 6
Collect like terms together A. x < 5 B. x > 7 ½ Cx<7½ D. x < – 5 E. x  5
x – 4x > 8 + 1 Solution
– 3x > 9 x – 1 < 2x + 1
Divide both sides by the coefficient of x i.e – 3 & 2 3 5 6
reverse sign Collect like terms together
– 3x < 9 x – 2x < 1 + 1
– 3 –3 2 5 6 3
x < – 3 (B) Solving both sides of the inequality separately by using
LCM.
1992/19 UME LHS, LCM of 2 and 5 is 10 while RHS, LCM of 6 and 3 is 6.
Solve the inequality 2(x – 3) < 3( x – 1) 5x – 4x < 1 + 2
A. x > – 3 B. x < – 3 C. x > 3 D x < 3 10 6
Solution x < 3
2(x – 3 ) < 3(x – 1) 10 6
Open up the brackets
2x – 6 < 3x – 3 x < 1
Collect like terms together 10 2
2x – 3x < – 3 + 6 Multiply both sides by 10
– x<3 x < 10
Divide both sides by –1 & reverse sign 2
x > – 3 (A) x < 5 (A)
Alternatively
1999/15 PCE Apply the general method of multiplying through by LCM of
The solution set of the inequality x (x – 1) < (x + 1)2 the denominators. We have that LCM of 2,3, 5 and 6 is 30
is given by i.e 30 × x – 30 × 1 < 30 × 2x + 1 × 30
A.{x : x < –3} B.{x : x > –3} 2 3 5 6
C.{x : x > – 1/3} D.{x : x < –1/3} 15x – 10 < 12x + 5
Solution Collect like terms together
x (x – 1) < (x + 1)2 15x – 12x < 5 + 10
Open both brackets 3x < 15
x2 – x < x2 + 2x + 1 x < 5 (A)
Collect like terms together
x2 – x – x2 –2x < 1 1986/19 UME
– 3x < 1 Find all real numbers x which satisfy the inequality
Divide both sides by the coefficient of x i.e – 3 & 1 (x + 1) – 1 > 1 (x + 4)
reverse sign 3 5
– 3x > 1 A. x < 11 B. x < – 1 C. x > 6 D. x > 11
– 3 – 3. Solution
x > – 1/3 ( C ) This kind of fraction with whole numbers is trouble-some to
clear , lets multiply through by the LCM of the denominators
1995/20 PCE Exercise 3.0 3, 1 and 5 i.e. 15. Thus,
For what range of values of x is 2x –3 > 3x + 2? 15 × 1 (x + 1) – 1× 15 > 15 × 1 (x + 4)
A. x > 5 B. x < 5 C. x < – 5 D. x > – 5 3 5
5(x + 1) – 15 > 3(x + 4)
2005/46 UME Exercise 3.1 5x +5 – 15 > 3x +12
Find the range of values of x for which 7x–3 >25 +3x. 5x – 10 > 3x + 12
A. x < – 7 B. x > – 7 C. x < 7 D. x > 7 Collect like terms together
5x – 3x > 12 + 10
2007/21 UME Exercise 3.2 2x > 22
Solve the inequality –3( x – 2 ) < –2( x + 3 ) Divide both sides by the coefficient of x i.e 2
A. x >12 B. x < 12 C. x > –12 D. x < –12 x > 11 (D)

40
1993/22 PCE 1994/17 UME
What is the range of value for t if Find the range of values of x for which 1 > 2 is true.
3t – t + 3 > t ? x
4 5 2 A. x < 1 B. x < 0 or x > 1 C. 0 < x > 1 D. 1 < x < 2
A. t < – 60 B. t < 60 C. t > – 60 D t > 60 2 2 2
Solution Solution
We multiply through by the LCM of the denominators 4, 5 We are simply asked to solve. Thus,
and 2 i.e20 1 > 2
20 × 3t – t × 20 + 3 × 20 > t × 20 x
4 5 2 1 > 2x
15t – 4t + 60 > 10t Divide both sides by 2
11t + 60 > 10t 1 > x
Collect like terms together 2
11t – 10t > – 60 i.e. 1/2 is greater than x which means x is less than 1/2
t > – 60 (C) x < 1 (A)
2003/25 PCE 2
Solve the inequality 1997/17 UME
1x+1< 1x–1 Let f(x) = 2x + 4 and g(x) = 6x + 7 where g(x) > 0.
3 2 2 Solve the inequality f(x) < 1
A. x > 3 B. x > 9 C. x < – 3 D. x < – 9 g(x)
Solution A. x < – 3 B. x > – 4 C. x > –3 D. x > – 12
We multiply through by the LCM of the 4 3 4
denominators 3, 2 and 2 i.e 6. Solution
6×1x + 1×6 < 6×1x – 1×6 We are simply asked to solve
3 2 2 2x + 4 < 1
2x + 3 < 3x – 6 6x + 7
Collect like terms together The additional information of g(x) > 0 is in line with the rule
2x – 3x < – 6 – 3 of fraction that does not allow for zero in the denominator.
–x < –9 Thus we cross multiply
Multiply through by –1 & reverse sign 2x + 4 < (6x + 7) ×1
x > 9 (B) Open bracket and collect like terms together
1989/25 UME 2x – 6x < 7 – 4
Find the range of values of r which satisfies the – 4x < 3
following inequality, where a , b and c are positive. Divide both sides by the coefficient of x i.e – 4& reverse sign
r + r + r > 1 – 4x > 3
a b c –4 –4
A. r > a bc B. r > a b c C r > 1 + 1 + 1 x > –¾ (C)
bc + ac + ab a b c 1998/17 UME
If x is a positive real number, find the range of values for which
D. r > 1 1 + 1 > 1
abc 3x 2 4x
Solution A. x > – 1 B. x > 0 C. 0 < x < 4 D. 0 < x < 1
The general format of using LCM of the denominators 6 6
to multiply through is applicable here. Solution
r + r + r >1 First we find the LCM of the denominators 3x, 2 and 4x i.e.
a b c 2 3x 2 4x
We multiply through by the LCM of a, b and c i.e a b c x 3x 1 2x
a a b c 2 3 1 2
b 1 b c 3 3 1 1
c 1 1 c 1 1 1
1 1 1 LCM = 2 × 2 × 3 × x i.e 12x
Thus, we multiply through by the LCM
LCM = a × b × c i.e a b c
12x × 1 + 12x × 1 > 1 × 12x
abc × r + r × abc + r × abc > 1 × abc
3x 2 4x
a b c
Recall that 12x is same as 4 Since the x will cancel out
bcr + acr + abr > abc
3x
Factor out r on the LHS
4 + 6x > 3
r (bc + ac + ab) > abc
6x > 3 – 4
Divide both sides by the coefficient of r
6x > – 1
r > abc
bc + ac + ab (A) x > – 1/6 (A)
41
1978/23 UME (y – 6) (y + 3) > 0
–½ By quadratic principle
If 3x – ( ¼ ) > ¼ – x,
y – 6 > 0 or y + 3 > 0
Then the interval of values of x is
y > 6 or y > – 3 (C)
A. x > 1 B. x < 1 C. x < 1 D. x < 9 E. x > 9
3 3 4 16 16
1997/18 UME
Solution Find the range of values of x which satisfies the inequality
3x – ( ¼ ) – ½ > 1– x 12x2 < x + 1
4 A–1<x< 1 B. 1 < x < 1
½ 4 3 4 3
3x – ( 4/1 ) > 1– x
4 C. 1<x<1 D.–1 < x < – 1
3x – 2 > 1 – x 4 3 4 3
4 Solution
Multiply through by 4 to clear fraction 12x2 < x + 1
12x – 8 > 1 – 4x Rearranging
Collect like terms together 12x2 – x – 1 < 0
12x + 4x > 1 + 8 Factorizing the LHS
16x > 9 12x2 – 4x + 3x –1 < 0
x > 9 4x (3x –1) + 1 (3x – 1) < 0
16 (E) ( 3x – 1)(4x + 1) < 0
By quadratic format
1991/21 UME Exercise 3.3 3x – 1 < 0 or 4x + 1 < 0
Find the range of values of x which satisfy the 3x < 1 0r 4x < – 1
inequality x < 1 or x < – 1
x + x + x < 1 3 4
2 3 4 The values shows that x is between – 1/4 and 1/3 . Thus
A. x < 12 B. x < 13 C. x < 9 D. x < 13 – 1/4 < x < 1
/3 A.
13 12
2000/21 UME
2005/39 PCE Exercise 3.4 Solve the inequality 2 – x > x2
For what range of values of m is m – 1 + m + 3  m – 2? A. x < – 2 or x > 1 B. x > 2 or x < – 1
3 7 C. –1 < x < 2 D. – 2 < x < 1
A. m  1 B. m  – 1 C. m  4 D. m  – 4 Solution
4 2 – x > x2
2006/30 UME Exercise 3.5 Rearranging
Solve the inequality for which 2 – x – x2 > 0
x + 4 – ( x –3 ) < 4 Factorizing the LHS
3 2 2 – 2x + x – x2 > 0
A. x > 7 B. x < 7 C. x < – 7 D. x > –7 2 (1– x) + x(1– x) > 0
(1– x)(2 + x) > 0
2006/43 PCE Exercise 3.6 By quadratic principle
Solve the inequality 1/2 ( 4 – 3x )  3x – 2 1– x > 0 or 2 + x > 0
A. x  8/9 B. x  9/8 C. x  – 9/8 D. x  – 8/9 – x > –1 or x > – 2
x < 1 or x > – 2
Inequalities involving quadratic expression combining the two equality from the smallest – 2
Here the basic operational principles of solving –2<x<1 (D)
quadratic equations by method of factorization is mostly Note that
applied. x > – 2 means – 2 < x to enable us combine the two
inequalities
1993/20 UME
Solve the inequality y2 – 3y > 18 2006/33 UME Exercise 3.7
A. – 3 < y < 6 B. y < –3 or y > 6 If y = x2-x-12, find the range of values of x for which y  0.
C. y > – 3 or y > 6 D. y < –3 or y < 6 A. x  – 3 or x  4 B. x < – 3 or x > 4.
Solution C. – 3  x  4 D. – 3 < x  4
y2 – 3y > 18
Rearranging 2007/19 UME Exercise 3.8
y2 – 3y – 18 > 0 The solution of the quadratic inequality ( x2 + x – 12 )  0 is
Factorising the resulting quadratic expression on the LHS,
A. x  – 3 or x  4 B. x  3 or x  – 4
y2 – 6y + 3y – 18 > 0
y (y – 6) + 3 ( y – 6) > 0 C. x  3 or x  – 4 D. x  3 or x  – 4
42
Inequality combination, splitting and 1985/4 UME
range List all integers satisfying the inequality
Two inequalities about a particular variable can be – 2  2x – 6 < 4
combined as follows: A. 2,3,4,5 B.2,3,4 C.2,5 D3,45 E.4,5
Solution
Examples
We split the given inequality and solve the halves
1) If – 4 < x and x < 2 : combine the two inequalities
separately
and give the range of x that are whole numbers.
– 2  2x – 6 < 4 will be
Solution – 2  2x – 6 and 2x – 6 < 4
We have that x values is between – 4 and + 2. – 2 + 6  2x 2x < 4 + 6
Thus, combining, 4  2x 2x < 10
–4<x<2 4  2x 2x < 10
Range of x = – 3, – 2, –1, 0, 1. 2 2 2 2
The two limits – 4 and 2 are not included because of the strict
inequalities 2  x x < 5
Thus, combining the result we have,
2) Given that t  3 and t > 0 , then combine the two 2x<5
inequalities and give the range of t that are whole x values are 2,3,4 (B)
numbers. 5 is not included because of the strict inequality there
Solution
1988/20 UME
Combine the inequalities 0 < t  3 and
List the integral values of x which satisfy the inequality
Range of t = 1,2,3
– 1 < 5 – 2x  7
0 is not included as a result of the strict inequality there.
A. –1,0,1,2 B.0,1,2,3 C. –1,0,1,2,3 D. –1,0,2,3
Splitting of inequalities Solution
The splitting of inequality has to do with double As usual, we split the given inequality and solve the halves
inequalities. For instance mathematically, separately.
2  x  10 –1 < 5 – 2x  7 will be
Implies or mean –1 < 5 – 2x and 5 – 2x  7
2  x and x  10 –1 – 5 < – 2x –2x  7 – 5
So also when we write –3 < 7 + 3x < 12 – 6 < – 2x –2x  2
Implies or mean – 6 > – 2x –2x  2
– 3 < 7 + 3x and 7 + 3x < 12 –2 –2 –2 –2
The middle term in any double inequalities affects both 3 > x x  –1
halves of the inequality. To enable us solve problems in Thus, combining the result
double inequalities with ease, it is advisable to split the 3>x–1
double inequalities to halves and solve. After which we x values are 2,1,0, –1 (A)
combine the result as shown below 3 is not included because of the strict inequality there

1980/7 UME 1992/16 UME


List all integers’ values of x satisfying the inequality Find all values of x satisfying the inequality
– 1 < 2x – 5  5 – 11  4 – 3x  28
A. 2,3,4,5 B. 2,5 C. 3,4,5 D. 2,3,4 E. 3,1 A. – 5  x  8 B. 5  x  8
Solution C. – 8  x  5 D. – 5 < x  8
We apply the split method to solve. Solution
– 1 < 2x –5  5 will mean We split the given inequality and solve the halves separately.
– 1 < 2x – 5 and 2x – 5  5 – 11  4 – 3x  28 will be
Solving the two halves, –11  4 –3x and 4 – 3x  28
–1 + 5 < 2x and 2x  5 + 5 –11 – 4  – 3x and –3x  28 – 4
4 < 2x and 2x  10 – 15  – 3x and –3x  24
4 < 2x and 2x  10 –15  –3x and –3x  24
2 2 2 2 –3 –3 –3 –3
2 < x and x  5 5  x and x  –8
Thus, combining the results we have To combine the two resultant inequalities
2<x 5 5  x means x  5 and x  – 8 means – 8  x,
x values are 3,4 and 5. (C) Starting from the smallest – 8,
2 is not included because of the strict inequality there. – 8  x  5 (C)

43
2004/29 UME Exercise 3.9 Solution
What are the integral values of x which satisfy the Solving the inequality first,
inequality – 1 < 3 –2x  5 ? x + 4  4x – 2
A. –1,0,1,2 B. – 2,1,0, – 1 C. 0,1,2 D. –1,0,1 Collect like terms together
x – 4x  – 2 – 4
2003/13 UME Exercise 3.10 – 3x  – 6
Find the range of values of x satisfying the Divide both sides by – 3 and reverse the inequality sign
inequality 5 + x  8 and 13 + x  7 –3x  – 6
A. –3  x  3 B.3  x  6 C. – 6  x  3 D. – 6  x  –3
–3 –3
x  2
Representing the solution in a number line;
Representation of inequalities in number line.
x
Ο Open dot is used to represent strict inequalities of 0 1 2 3 4 5
less than < and greater than >
Double (combined inequalities) number line
Thick dot represents weak inequalities of less than
1992/3 ( ORDINARY MATHS WAEC )
or equal to  and greater than or equal to 

There is a confusing fact about Negative numbers that


–4 –3 –2 –1 0 1 2 3 4 5
affect students, result in number line unlike positive
If x varies over the set of real numbers, which of the
numbers, the more you go on the negative axis, the
following is illustrated in the diagram above?
smaller the number. – 6 is less than –1
A. –3 < x  2 B. –3  x < 2 C. –3 < x < 2
D. –3  x  2 E. x  2
Single inequality number line Solution
A splitting of the inequality shows that x is between –3 and 2.
Eg. 1 What is the range of values of x for which 1/2 (1– x) < 3 ? But at – 3, shows that x  – 3 which means –3  x and
( ORDINARY MATHS NECO 2002 )
at 2, shows x < 2
A. B. Thus, combining both
–5 0 5 –5 0 5 –3  x < 2 (B)

1994/22 PCE
C. D.
–5 0 5 –5 0 5
–3 0 2
Solution
The inequality which represents any point x on the number
First we solve the inequality for the x range before
line shown above is
showing it on the number line
1 (1– x) < 3 A. –3  x < 2 B. –3 < x < 2 C. –3  x  2 D. –3 < x  2
2 Solution
Multiply through by 2 Though the examiner gave a poor diagram here, going by the
1–x < 6 available option , the range is between –3 and 2 as earlier
–x < 6 – 1 treated in example 1. But in this case open dot is at – 3
–x < 5 i.e. –3 < x ,while thick dot is at 2 i.e. x  2
Multiply through by –1 and change inequality sign Thus, combining – 3 < x  2 (D)
x >–5
Thus, the number line is 2001/16 UME

(A)
-5 -4 -3 -2 -1 0 1 2 3 4 5
–5 0 5
The graph above represents the inequality
2000/19 PCE A.-2  x  4 B.-2< x  4 C.-2 > x > - 4 D.-2 > x  -4
Which of the following is the correct representation of Solution
the solution of the inequality x + 4  4x – 2? The range of x is between – 2 and 4.
At –2 it is open dot and it is the greater than part that is
A x B. x implied here i.e. x > – 2. At 4, it is the less than or equal to
0 1 2 3 4 5 0 1 2 3 4 5 part that is implied i.e. x  4.
Combine the inequalities x > –2 and x  4;
C x D. x
Note that x > –2 means –2< x .Thus,
0 1 2 3 4 5 0 1 2 3 4 5
– 2 < x  4 (B)
44
2003/27 PCE – 3x  6
If x – 3  4 and x + 1 < 5, which of these diagrams –3 –3
represents the solution set? x  –2
Representing the solution on a number line, we must recall
that “the bigger the number the smaller its value on the
A. B. negative axis”.
3 4 7 8 3 4 7 8 Thus x  – 2 is x
(B)
-4 -3 -2
C. D. 2005/50 PCE Exercise 3.12
3 4 7 8 3 4 7 8
x
Solution -7 -6 -5 -4 -3 -2 -1 0 1 2 3 4 5 6 7
Solving for x in both inequalities, we have
x–34 and x + 1 < 5 The above graph represents the inequality
A. –3 < x  6 B.–3  x  6 C. –3 < x < 6 D. –3  x < 6
x4+3 x<5–1
x7 x<4 2007/16 PCE Exercise 3.13
Showing both result on inequality line will give a “ run The correct representation of the solution of set of
away” type of number line starting from the smallest { t : t > – 5 } U { t : t  – 2 } on a number line is
number x < 4 with open dot
A B.
-5 -2 -5 -2
(D)
3 4 7 8 C D.
-5 -2 -5 -2

2002 /17 PCE Exercise 3.11 Graphical representation of linear inequalities


Which of the following represents the range –5 < x  5 in two variables
on a number line?
This is another phase of inequality that is related to linear
equation in two variables. We shall restrict ourselves here to
graphical solution of inequality in two variables.
Here broken lines represent strict
A. B.
inequalities (< or >). While thick line
-5 0 5 -5 0 5
shows weak inequalities ( or ).
We always use the linear equation equivalent of an inequality
to get the two points necessary to plot the straight line.
C. D. Examples of inequality graph are shown below:
-5 0 5 -5 0 5
( 1 ) Show the graphical region represented by the inequality
y < 4x + 2
Solution
1996/17 UME
First we consider the line equality by replacing the inequality
Which of the following number lines is a solution of the
with equals to sign
equation 2(x –3)  5x ? y < 4x + 2 will become
x x y = 4x + 2
A. B. When x = 0, y = 4( 0 ) + 2 i.e. 2 ( 0, 2 )
-2 -2 when y = 0 , 0 = 4x + 2
–2 = x i.e. – 0.5 ( – 0.5, 0 )
x x 4
C. D. We use these points to draw as shown below:
2 2 y

Solution 2
Firstly, we solve the inequality
2(x –3)  5x
2x – 6  5x -0.5
x
0
Collect like terms together
2x – 5x  6
– 3x  6 The shaded region satisfies the inequality y < 4x + 2 ; since
Divide both sides by –3 and change inequality sign less than implies the lower region.
45
( 2 ) Show graphically the region represented by x + y > 5 1994/18 UME
y
Solution

0
2=
4
Firstly, we consider the line equation

y-
-
x + y > 5 will be

2x
x+y=5 2
When x = 0, y = 5 ( 0, 5 )
When y = 0, x = 5 ( 5, 0 ) x
0 1 2 3
Using the above points to plot as shown below:
y

-2

Find the inequality which represents the shaded portion in the


0 5
x
diagram above
A. 2x – y –2  0 B. 2x – y –2  0
C. 2x – y –2 < 0 D. 2x – y – 2 > 0
The Shaded region is x + y > 5; since greater than Solution
implies the upper region. The linear equation equivalent is already given. The next is
the appropriate inequality sign.
( 3 ) Show graphically the region represented by 2x + y  8 The line is thick i.e. weak inequality. And the lower part is
Solution shaded i.e. less than or equal to in this case
Considering the linear equivalent, Thus changing the equality sign,
2x + y  8 will be 2x – y –2 = 0 will be come
2x + y = 8 2x – y –2  0 ( A )
When x = 0, y = 8. ( 0, 8 )
When y = 0, x = 4 ( 4, 0 ) 2004/33 UME
Using the above points to plot as shown below: y
y

(0, 6)
8

0 x
4
x
(0, 0) (2, 0)

Some inequality regions are shown below (shaded).


y y
The shaded area in the diagram above is represented by
A.{(x, y) : y –3x < – 6} B. {(x, y) : y +3x < 6}
C.{(x, y) : y + 3x < – 6} D. {(x, y) : y – 3x < 6}
Solution
x equation of the line is not given. That will be our first
The
0 x 0
issue to resolve. Applying the concept of gradients of a line
and line equation
y = mx + c
y y Here c = 6 intercept at the y – axis.
m (gradient) = change in y
change in x
x 0
= 0 – 6 and Not 6 – 0
x 2– 0 0–2
0 x 0 Since it is a negative gradient
x 0
= – 6 i.e – 3
2
Substituting m = –3 and c = 6 into y = mx + c
y = –3x + 6
Rearranging,
y + 3x = 6
Since it is the lower portion that is shaded and a broken line
is involved, i.e. less than is this case. Thus,
y + 3x = 6 will become
y + 3x < 6 (B)
46
2002/34 UME Solution
y
The two major lines have their lower portions involved.
Since there are no broken lines here,  will hold for the two
major lines
(0, 2) i.e. 2y – x –2 = 0 will be
2y – x –2  0
and y +2x + 2 = 0 will be
(-4, 0) y + 2x + 2  0
x Triangle SPT is located at positive side of y– axis i.e y  0
and the negative side of x – axis i.e x  0.
Thus the set of the inequalities are:
2y – x –2  0, y + 2x + 2  0, y  0 and x  0 (A)

2003/19 UME
Using the graph above to find the values of p and q if y
px + qy  4
A. p = 2, q = – 1 B. p = – 1 q = 2 P

0
x=
y-
C. p = 2, q = 1 D. p = 1, q = 2
Solution
Here the inequality is given; we then convert to its
x
linear equivalent i.e. O

x+1=0
px + qy  4 will be

y
+
px + qy = 4

x=
Substituting ; At point (– 4, 0) i.e. x = – 4 and y = 0

0
Q
p(– 4) + (0) q = 4
– 4P = 4 Triangle OPQ above is the solution of the inequalities
p= 4 A. x + 1  0, y + x  0, y – x  0
–4 B. y + x  0, y – x  0, x – 1  0
p = –1 C. x – 1  0, y – x  0, y + x  0
Substituting ; At point (0, 2) i.e. x = 0 and y = 2 D. x – 1  0, y + x  0, y – x  0
p(0) + q(2) = 4 Solution
2q = 4 No broken line(s) in the graph
q= 4 Thus, the use of either  or  to replace the equality signs.
2 Analyzing the lines,
q = 2 Thus p = – 1 , q = 2 (B) y – x = 0 has its upper portion involved i.e. 
y–x0
System of inequalities (more than one y + x = 0 has its lower potion involved i.e. 
equation) y+x0
The solution of a system of linear inequalities in two x + 1 = 0 has its front involved, i.e.  0
variables is the region of intersection of all the
So, x + 1  0
equations involved. The set of the inequalities:
2001/13 UME
x + 1  0, y + x  0, y – x  0 (A)
=0
2 1999/16 PCE
x-

.
-
2y

.
y
y
y
+

P
2x

1
+
2
=

.
0

1
+

P 1 x-1
x

y= 2
=

. . .
y

-2 T
S 0 x
S -1 x

.
.
-2
R
Q

Triangle SPT is the solution of the linear inequalities Which of the points P,Q,R,S in the diagram above satisfies
A. 2y – x – 2  0, y + 2x + 2  0, y  0, x  0 the simultaneous inequalities y  x + 1, y < 1/2x – 1 ?
B. 2y – x – 2  0, y + 2x + 2  0, x  0
C. 2y – x – 2  0, y + 2x + 2  0, – 2  x  –1
D.–2  x  2, y  0, y + 2x +2  0 , x  0 A. P B. Q C. R D. S
47
Solution 2006/28 UME Exercise 3.14
The line y = x + 1 is y  x + 1 which is the lower
4 y=x
portion. This condition eliminates R and S.
As for line y = 1 x –1, it is y < 1x – 1 which is the lower
3
portion. 2 2
This condition eliminates P leaving us with Q which 2
satisfy both inequalities of option (B)
1
1998/18 UME
y
0 0 1 2 3
y+x=4
(0, 3) The solution set of the shaded area above is
A. y + x  4, y  x B. y  x, y + x  4 and y 0
C. y  0, y  x and y + x  4 D. y  x , y + x  4

2006/50 PCE Exercise 3.15

x (0, 4)
(2, 0)
The shaded area above represents
A. x  0, 3y + 2x  6 B. x  0, y  3, 3x + 2y  6
C. x  2, y  0, 3x + 2y  6 D. x  0, y  0, 3x + 2y  6
Solution
Though this problem seems to be a single linear
inequality, it is not. A reference to the beginning part of
(0, 0)
this subtopic page will convince us.
y  0 is shaded, (6, 0)
x  0 is shaded The inequality represented by the graph above is
Lastly, the line with points (0,3) and (2,0) A. 2x + 3y > 12 B. 2x – 3y  12
Applying y = mx + c C. 2x – 3y > 12 D. 2x + 3y  12
Here c = 3, intercept with the y – axis
m(gradient) = change in y
change in x
= 0–3
2–0
Since it is negative gradient
m=–3
2
Substitute m = – 3 and c = 3 into y = mx + c,
2
y=–3x+3
2
Rearranging
y+3 x=3
2
Clear fraction by multiplying through by 2
2y + 3x = 6
Changing the equality sign, we have that, the shaded
region is the upper portion and a thick line is involved
i.e.  in this case
3x + 2y  6 (D)
i.e. x  0, y  0, 3x + 2y  6

48
Chapter four Solution
Undefined expressions, With just 1 at the numerator let us concern our self with
denominator only.
expression equals zero and limits
For the expression to be undefined, we equate the
Undefined expressions denominator to zero.
For any expression of the form a (x) 2x2 – 13x + 15 = 0
b(x) Here a  1, ac = 2 x15 i.e 30
to be undefined then b(x) = 0 Factor of 30 that when added gives -13 are – 3 and –10
Factorising
1991/4 (ordinary maths) 2x2 –3x – 10x + 15 = 0
For what value of y is the expression x( 2x – 3) –5(2x – 3) = 0
(y + 2) undefined (2x – 3 ) (x –5) = 0
y2 – 3y – 10 2x –3 = 0 or x –5 = 0
A. y = 0 B. y = 2 C. y = 3 D. y = 5 E. y = 10 2x = 3 or x = 5
Solution x = 3/2 or 5 (A)
Factorize the denominator y2 – 3y –10
= y2 + 2y – 5y –10 2002/21 (ordinary maths)
= y (y +2) –5 (y + 2) For what values of x is the expression
= (y + 2) (y – 5) 3x – 2 undefined.
Thus y+ 2 = 1 4x2 + 9x – 9
(y +2) (y – 5) y–5 A -3 or 3 B. – 2 or –3 C. 2 or 3 D 3 or –3
For the reduced expression to be undefined 4 3 3 4
y–5=0 Analysis and solution
y = 5 (D) We factorize the denominator
1994/16 (ordinary maths) Note that a  1 and ac = - 36, b = + 9;
For what value of x is the expression Factors of – 36 that will give + 9 are – 3 and 12
x2 + 15x + 50 not defined 4x2 + 9x – 9 = 4x2 – 3x + 12x – 9
x–5 = x (4x – 3) + 3 (4x – 3)
A. – 10 B. – 5 C. 0 D. 5 E 10 = (4x – 3 ) (x+3)
Solution Thus, 3x – 2 = 3x – 2
The denominator is in its lowest term, 4x2 + 9x – 9 (4x – 3) (x + 3)
Thus, for the expression to be undefined Since non of the terms cancelled out, we equate the
x–5=0 denominator to zero.
x = 5 (D) (4x – 3) (x + 3) = 0
4x –3 = 0 or x +3 = 0
1995/11 (ordinary maths) x = 3/4 or x = -3 (D)
Find the values of x for which the expression is
undefined.
6x – 1 1998/10 (ordinary maths) Exercise 4.0
x2 + 4x – 5 For what values of x is the expression
A. + 4 or +1 B. – 5 or + 1 C. –5 or -1 x–5 not defined
D. +5 or –1 E. + 4 or –1 x2 – 2x – 3
Solution A 3, 1 B –1, -3 C –1, 3 D 3, -2 E 1, -3
We factorize the denominator
x2 + 4x – 5 = x2 + 5x – x – 5 2000/6 b(i) Neco (ordinary maths) Exercise 4.1
= (x + 5) (x – 1) For what value (s) of x is the expression
= (x + 5) (x – 1) x2 - 4x – 21 not defined ?
Thus, 6x –1 = (6x –1) x +11
x2 + 4x – 5 (x + 5) (x – 1)
Since non of the terms cancelled out we equate the 2002/26 Neco (ordinary maths) Exercise 4.2
denominator to zero to satisfy our condition. Given that x = 2a – 16 find the value (s)
2
(x + 5) (x - 1) = 0 a - 11a +24
x + 5 = 0 or x – 1 = 0 of a for which x is not defined.
x = – 5 or 1 (B) A. 8 B. 4 or 6 C. 3 or 8 D. – 3 or – 8 E. – 4

2003/25 (ordinary maths) 2003/49 Neco (ordinary maths) Exercise 4.3


Find the values of x for which 1 What values of x make the equation. 2x + 1 undefined?
2x2 - 13x + 15 2
x + 6x +5
is not defined. A x = +5, x =1 B x = 3, x = 2 C x = -3, x = -2
A 5 or 3 B 1 or 15 C 2 or 15 D 13 or 15 D x = -5, x = 1 E x = -5, x = -1
2 13
49
(ii) Expression equals zero Since no term cancelled out
For any algebraic expression a (x) = 0 Then a(x) = 0 We equate the numerator to zero
b (x) (y – 2 )(y – 1 ) = 0
Examples y – 2 = 0 or y – 1 = 0
(1) For what value (s) of x is the expression equal zero y = 2 or 1 (B)
x+3
x2 + 10x – 25 Exercise 4.4
Solution For what values of y is the expression y + 1 equal zero
Since the numerator cannot factor further, we impose y2 – 3y – 10
the condition
x+3 =0
Exercise 4.5
x + 10x – 25
2
For what values of x is the expression 9x2 – 1 equal zero
if x + 3 = 0 x2 + 2x – 8
x = -3
(2) For what value (s) of x is the expression
x2 - 2x – 15 equals zero Limit of a function
x2 – 25
Firstly, we factorize the numerator x2 – 2x – 15 The limiting value or limit of f(x) = 3 – x as x approaches
= x2 + 3x – 5x – 15 2 is equal to 1/4 2+x
= x(x +3) – 5(x + 3)
= (x +3 ) (x –5 ) This is written more briefly as Lim 3–x = 1
Thus x 2 2+x 4
x2 – 2x – 15 = (x +3) (x – 5)
x2 – 25 (x –5) (x +5) In general, when dealing with algebraic functions and when a
= x+3 function approaches a limit value; to find its value we first
x+5 reduce the function to its lowest terms. We then insert in the
Imposing the given condition on the reduced form result the value that x approaches. As long as the
x+3=0 indeterminate quality
x = -3 0 or  is not obtained.
0 
2000 / 6b ii Neco (ordinary maths)
For what value (s) of x is the expression It follows that the result is the required value.
x 2 – 4x – 21 equals zero If 0 or  is obtained, it means that the function has
2x + 11 0  not been reduced sufficiently.
Solution
Firstly we factorize the numerator 1997/40 PCE
x2 - 4x – 21 = x2 – 7x + 3x – 21 What is the value of y as x approaches 5 in the equation.
= x(x –7) + 3 (x –7) y = x2 + 5x – 50
= (x – 7) (x + 3) x–5
Thus x2 - 4x – 21 = (x – 7) (x + 3) A.0 B. 3 C. 5 D. 15
2x + 11 2x + 11 Solution
Since no term cancelled out, we impose the condition, First, we factorize ( simplify) ‘the non-simplified aspect of
expression equal zero i.e the question to see if any term(s) will cancel out.
(x –7) (x + 3) = 0 y = x2 + 5x – 50
x – 7 = 0 or x + 3 = 0 x–5
x = 7 0r – 3 = x2 + 10x – 5x – 50
2002/ 27 Neco (Nov) (ordinary maths) x–5
For what values of y is the fraction = x ( x + 10) – 5( x + 10 )
y2 – 3y + 2 equal to zero x–5
3y – 12
A. 0 or 4 B. 1 or 2 C. – 1 or - 2 = ( x + 10 ) ( x – 5 )
D. – 1 or – 3 E. – 2 or – 4. x–5
Solution x – 5 will cancel out
Firstly, we factorize the numerator y = x + 10
y2 - 3y + 2 = y2 – 2y - y + 2 Value of y as x approaches 5
= y (y – 2) – 1 (y – 2) y = 5 + 10
= (y – 2) (y – 1) y = 15 (D)
y - 3y + 2 = (y – 2) (y – 1)
2

3y –12 3 (y – 4)
50
1992/36 1995/40 PCE
Evaluate lim ( x – 2)( x2 + 3x – 2) Calculate lim of cos x
x2 x2 – 4 x0 x–2
A. 0 B. 2 C. 3 D. 4 A. – ½ B. 0 C. ½ D. 1
Solution Solution
First, we simplify ( factorize) the unsimplified term(s) The terms cannot reduce further
to see whether any will cancel out Lim cosx = 1 ( A ) since cos 0 = - 1/2
( x – 2)(x2 + 3x – 2) = ( x – 2) ( x2 + 3x – 2) x0 x–2 -2 0–2
x2 – 4 x2 – 22
= (x – 2)(x2 + 3x – 2 ) 1996/36 PCE
(x – 2)(x + 2) If f(x) = 3x3 – 2x and g(x) = 1/x
what is the limit of h(x) = f(x) g(x) as x  2 ?
= x2 + 3x – 2
x+2 A. 20½ B. 20 C. 10 D. 3
Thus, Lim (x – 2) ( x2 + 3x – 2) = lim x2 + 3x – 2 Solution
x–2 x2 – 4 x–2 x+2 h(x) = f(x) g(x)
= 8 i.e. 2 (B) = (3x3 – 2x)( 1/x)
4 = x(3x2 – 2 )( 1/x)
2002/37 PCE h(x) = 3x2 – 2
Find the L t x2 – 4
lim h(x) = lim 3x2 – 2
x2 x–2
x2 x2
A. – 4 B. 0 C. 4 D. 8 = 3(2)2 – 2
Solution = 3(4) – 2
First, we reduce the expression to its lowest terms. = 10 ( C )
x2 – 4 = x-2 – 22 2005/5 PCE
x–2 x–2 Evaluate lim x2 – 4x + 3
=(x–2)(x+2)
x3 x2 – 9
x–2 1 1
A. 3 /3 B. 2 /3 C. 0 D. 1/3
=x+2
Solution
Thus,
First we factorize the numerator & denominator to see
L t x2 – 4 = L t x+2
whether some terms will cancel out.
x2 x – 2 x2 x2 – 4x + 3 = x2 – 3x – x + 3
= 4 (C ) x2 – 9 x 2 – 32
1998/39 PCE `= x(x – 3) – 1 (x – 3 )
The limit of x2 – a2 as x tends to a is (x – 3 )(x + 3)
x–a = (x – 3 )( x – 1 )
A. 2a B. a C. 0 D. – 2a ( x – 3 ) ( x + 3)
Solution x – 3 will cancel out
First, we reduce to its lowest term = x–1
x2 – a2 = (x – a) ( x + a ) x+3
x–a x–a Thus, lim x2 – 4x + 3 = lim x–1
=x+a x3 x –9
2
x3 x+3
Thus, lim x2 – a2 = lim x + a = 3 – 1 = 2/6 i.e 1/3 (D)
xa x–a xa 3+3
= 2a (A) 2000/36 PCE Exercise 4.6
Evaluate lim x2 – 16
2004/36 PCE x–4 x+4
Find the limit of x2 – x as x  0. A. – 8 B. – 2 C. 0 D. 4
x
A. – 1 B. 0 C. 1 D. 2 2003/47 PCE Exercise 4.7
Solution Evaluate lim x2 – x – 2
Reducing the expression to its lowest term, we have x2 x2 – 4
x2 – x = x ( x – 1) A. /2 B. ¾
5 1
C. /3 D. ½
x x
= x–1 2007/39 PCE Exercise 4.8
Find the limit of the function x2 – 9x + 20 as x tends
Lim x2 – x = lim x – 1 x2 – 3x – 4 to 4
x0 x x0 1 1
= – 1 (A) A. –5 B. – /5 C. /5 D. 5
51
Chapter five 1995/12 UME
Remainder and factor theorems and surd Find the positive value of p if the expression 2x2 – px + p
Remainder and factor theorems leaves a remainder 6 when divided by x – p
These theorems take their roots from long division with A.1 B.2 C.3 D.4
numbers. Solution
Now, let us study some long division cases Applying remainder theorem
Evaluate ( i ) 13  2 ( ii ) 12  2 i.e. f ( p ) = 6
Solution We now substitute p for x in the given equation
(i) 6 2( p )2 – p(p) + p = 6
2 13 2p2 – p2 + p = 6
12 p2 + p = 6
1 p2 + p – 6 = 0
When 2 divide 13, we get 6 remainder 1 factorizing
13 = 2 × 6 + 1 p2 + 3p – 2p – 6 = 0
divided = divisor × quotient + remainder p( p + 3 ) –2( p + 3 ) = 0
(p–2)(p+3)=0
( ii ) 6 p – 2 = 0 or p + 3 = 0
2 12 p = 2 or – 3; but the positive value is 2 ( B )
12
00 1995/11 UME
When 2 divides 12, we get 6 there is no remainder If x – 1 and x + 1 are both factors of the equation
Thus 2 is factor of 12. x3 + px2 + qx + 6 = 0 evaluate p and q.
In both cases, 2 is their divisor and 6 is their quotient. A. – 6, – 1 B. 6, 1 C. 1, –1 D.6, – 6
while 13 and 12 are divided for i and ii respectively. Solution
Remainder theorem becomes factor theorem when the If x – 1 is a factor; then
divisor divides the divided without a remainder. x–1=0
and x = 1,
Remainder thereom states that when we divide a put x = 1 into the given equation
polynomial f(x) by another polynomials ax – b and (1)3 + p(1)2 + q(1) + 6 = 0
there is a remainder r, then the value of the polynomial 1+p+q+6= 0
f(x) for the value x, p + q = –7 ………………… ( 1 )
Where ax – b = 0
Also if x + 1 is a factor, then
ax = b
x+1=0
x = b
and x = – 1,
a
put x = – 1 into the given equation
i.e. f  a  = r,
b
(– 1 )3 + p (– 1 )2 + q (– 1 ) + 6 = 0
Also if f( c ) = r –1+p–q+6=0
Then r is the remainder when f(x) is divided by x – c p – q = – 5 ………… (2)
Meanwhile Solving the simultaneous equations
if f  ba  = 0 p + q = –7
adding p – q = – 5
Then ax – b is a factor of the polynomial f( x ) 2p = – 12
Also if f(c ) = 0 Thus p = – 6
Then x– c is a factor of f( x ) this is the factor theorem. Substitute p value into (1)
p + q = – 7 will become
1990/18 UME –6+q= –7
Divide x3 – 2x2 – 5x + 6 by ( x – 1) q=–7+6
A.x2 – x – 6 B.x2 – 5x + 9 C.x2 – 7x + 6 D.x2 – 5x – 6 =–1
Solution p, q will be – 6, – 1 ( A )
Performing the long division
x2 – x – 6 1994/12 UME
x –1 x3 – 2x2 – 5x + 6 Find values of p and q such that ( x – 1 ) and ( x – 3 ) are
– ( x3 – x2) factors of px3 + qx2 + 11x – 6
– x2 – 5x A. – 1, – 6 B. 1, – 6 C.1, 6 D.6, –1
– (– x2 + x ) Solution
– 6x + 6 If ( x – 1 ) is a factor then
(– 6x + 6 ) x–1=0
0 0 and x = 1,
Thus, x – 2x – 5x + 6  ( x – 1 ) = x2 – x – 6 ( A )
3 2 put x = 1 into the given equation
p(1)3 + q (1)2 + 11 (1) – 6 = 0
52
p + q + 11 – 6 = 0 Solving (1) and (2) simultaneously
p+q+5 = 0 p–q=3
p + q = – 5 …………( 1) –(p+q=1)
– 2q = 2
If x – 3 is a factor then q=–1
x–3=0 Substitute q value into (1)
and x = 3, p + q = 1 will become
put x = 3 into the given equation p–1=1
p( 3)3 + q( 3 )2 + 11(3 ) – 6 = 0 p=1+1
27p + 9q + 33 – 6 = 0 p=2
27p + 9q + 27 = 0 p, q is 2, – 1 ( A )
27p + 9q = – 27
Divide through by 9 1998/15 UME
3p + q = – 3 …………( 2 ) Divide 2x3 + 11x2 + 17x + 6 by 2x + 1
Solving the simultaneous equations A.x2 + 5x + 6 B.2x2 + 5x + 6 C.2x2 – 5x + 6 D.x2 – 5x
3p + q = –3 +6
–(p+q=–5) Solution
2p =2 x2 + 5x + 6
p =1 2x + 1 2x + 11x2 + 17x + 6
3
Substitute p value into (1)
– ( 2x3 + x2 )
p + q = – 5 will become
10x2 + 17x
1+q=–5
– ( 10x2 + 5x )
q=–5–1
12x + 6
q=–6
12x + 6
p , q is 1, – 6 ( B)
0 0
The quotient is x2 + 5x + 6 (A)
1997/11 UME
If the function f ( x ) = x3 + 2x2 + qx – 6 is divisible by
x + 1, find q 1999/21 Special case
A –5 B. – 2 C. 2 D. 5 Divide 4x3 – 3x + 1 by 2x – 1
Solution A.2x2 – x + 1 B.2x2 – x – 1 C.2x2 + x + 1 D.2x2 + x
Since x + 1 divides the function, –1
x + 1 is a factor. Thus Solution
x+1=0 2x2 + x – 1
and x = – 1 2x – 1 4x3 – 3x + 1
put x = –1 into the given equation – (4x3 – 2x2 )
(–1)3 + 2(–1)2 + q(–1) – 6 = 0 2x2 – 3x
–1 + 2 – q – 6 = 0 – (2x2 – x )
–1 – 6 + 2 = q – 2x + 1
–5 = q ( A ) (– 2x + 1 )
0 0
1998/10 UME The quotient is 2x2 + x – 1 (D)
When the expression pm2 + qm + 1 is divided by
( m – 1 ), it has a remainder 2 and when divided by 2002/15 PCE Special case
( m + 1 ), the remainder is 4. find p and q respectively Divide 1 – k3 by k – 1
A.2, – 1 B. –1, 2 C.3, –2 D. – 2, 3 A.k2 – k + 1 B. – k2 – k + 1 C. – k2 – k – 1 D.k2 + k – 1
Solution Solution
When the expression is divide by ( m – 1), it has Note that 1 – k3 is same as – k3 + 1
remainder 2 – k2 – k – 1
implies f( 1 ) = 2 k – 1 – k3 + 1
i.e. p(1 )2 + q(1 ) + 1 = 2 – (– k3 + k2 )
p+q+1=2 – k2 + 1
p+q=2–1 – (– k2 + k )
p + q = 1 ………………… ( 1 ) – k+1
Also division with ( m + 1 ), remainder 4 –(–k+1)
Implies f(–1 ) = 4 0 0
i.e p(– 1 )2 + q(– 1 ) + 1 = 4 The quotient is – k2 – k – 1 ( C )
p–q+1= 4
p–q=4–1 2001/11 UME
p – q = 3 ………….. ( 2 ) Divide a3x – 26a2x + 156ax – 216 by a2x – 24ax + 108
A.ax – 18 B.ax – 6 C.ax – 2 D.qx + 2
53
Solution 1992/15 PCE Exercise 5.9
ax – 2 If x – 2 is a factor of 2x2 – x + k then k is
a – 24a + 108 a – 26a2x +156ax – 216
2x x 3x
A.6 B.2 C. – 3 D. – 6
– ( a3x – 24a2x + 108ax )
–2a2x + 48ax – 216
– (– 2a2x + 48ax – 216 ) Remainder cases
0 0 0 2004/35
2004/12 PCE Find the remainder when 3x3+5x2–11x + 4 is divided by x +3.
Find the value of r if ( x – 5 ) is a factor of A. – 4 B.4 C.1 D. – 1
( 1 + r )x2 – 2x – 190. Solution
A- 7 B. – 5 C.5 D.7 Since x + 3 divides the expression
Solution Then f(–3 ) = remainder
If x – 5 is a factor, then Thus; 3(–3 )3 + 5 (–3 )2 – 11 (–3 ) + 4
x–5=0 = 3(–27) + 5(9) + 33 + 4
x=5 = – 81 + 45 + 33 + 4
Put x value into the given expression = – 81 + 82
( 1 + r ) (5)2 – 2 (5) – 190 =1 (C)
= (1 + r )25 – 10 – 190 2003/ 26 PCE
= 25 + 25r – 200 Find the remainder when the polynomial
200 – 25 = 25r f(x) = 3x3 + x2 – 8x + 4 is divided by x + 2
175 = 25r A.0 B. – 2 C.2 D.1
175 = 25r Solution
25 25 Since x + 2 divides the given expression
7 = r (D) Then f(–2 ) = remainder
1997/15 PCE Exercise 5.1 = 3(– 2 )3 + (– 2 )2 – 8(– 2 ) + 4
If ( x – 2 ) and ( x + 1 ) are factors of the expression = 3(– 8 ) + 4 + 16 + 4
x3 + ax2 + bx + 1, what is the sum of a and b ? = – 24 + 24
A. 3/2 B.3 C. – 3/2 D. – 3 = 0 Thus the remainder is 0 ( A )

1999/12 PCE Exercise 5.2 1995/14 PCE special case


if ( x – 2 ) is a common factor of x3 – x2 – 2mx + 3n and Find the remainder when 6x3 – 5x + 4 is divided by 2x – 1
nx3 – mx2 + x + 2, find the values of m and n A.2 ¼ B.2 ½ C.2 ¾ D.3
respectively
Solution
A.0,1 B.1,0 C.1,2 D.2,1 Since 2x – 1 divides the expression
2005/48 PCE Exercise 5.3 Then f( ½ ) = Remainder from 2x – 1 = 0
What is the remainder when 5x3–13x + 4 is divided by x + 2? 6 ( ½ )3 – 5 ( ½ ) + 4 = 6 ( 1/8 ) – 5/2 + 4 2x = 1 i.e x = ½
A. – 4 B.14 C. – 10 D.4 = ¾ – 5/2 + 4
2005/39 Exercise 5.4 = 3 – 10 + 16
Divide 6x2 – 13x + 5 by 2x – 1 4
A.5x + 3 B.5x – 3 C.3x + 5 D.3x – 5 = 19 – 10
4
2004/13 PCE Exercise 5.5 = /4 i.e. 2 ¼ ( A )
9

If ( x + 1 ) divides x3 – 4x2 + kx + 6 without a


2006/29
remainder, then the value of k is
Find the value of k if the expression kx3 + x2 – 5x – 2 leaves
A. – 2 B. – 1 C.1 D.2
a remainder 2 when it is divided by 2x + 1.
A.8 B. – 10 C.10 D. – 8
1996/9 PCE Exercise 5.6
Analysis & solution
Find the value of k for which the expression
If 2x + 1 divides the function leaving a remainder 2
x3 + kx2 – x + 2 is divisible by ( x + 2 )
then 2x = –1
A. – 2 B. – 1 C.1 D.4
x=–½
1995/19 PCE Exercise 5.7 Then f(– ½ ) = Remainder
Divide 2x3 – x2 + x – 48 by ( x – 3 ) f(– ½ ) = 2
A.2x2–3x + 4 B.2x2 + 5x +16 C.x2 + 4x +5 D.x2 + 5x + 6 i.e. k(– ½ )3 + (– ½ )2 – 5(– ½ ) – 2 = 2
–k + 1 + 5 – 2 = 2
2007/13 PCE Exercise 5.8 8 4 2
If ( x + 1 ) and ( x – 6 ) are both factors of the equation Multiply through the Lcm 8
x3 + px2 + qx + 6 , find p and q . – k + 2 + 20 – 16 = 16
A. – 6, – 6 B. 6, 1 C. –6, – 1 D. 6, – 6 – k + 22 = 32
– k = 10 i.e k = – 10 ( B )
54
1994/14 PCE
b. Simplify 80 + 20 - 45
When a polynomial is divided by c + 2, the quotient is 3c – 3
and the remainder is 5. find the polynomial. Solution
A.3c2 +3c –6 B.3c2 +3c –1 C.3c2 +3c +5 D.3c2 –3c +1 We bring out the perfect square factors in the given surd as:
Solution = 16 5 + 4 5 - 9 5
Recall that
Divided = divisor × quotient + remainder = 16  4  5 - 9 
5 + 5
Polynomial a(x) = divided by b(x) × quotient q(x) + remainder
Polynomial = ( c + 2 ) × ( 3c – 3 ) + 5 = 4 5 + 2 5 -3 5
Opening the brackets = (4 + 2 -3) 5
= 3c2 – 3c + 6c – 6 + 5
= 3c2 + 3c – 1 ( B ) =3 5
2007/14 PCE Exercise 5.10 C. Simplify 60
Find the quotient when x3 + x2 – x – 1 is divided by x – 1. Solution
A x2–2x + 1 B. x2 – 2x –1 C.x2 + 2x +1 D.x2 + 2x – 1
60 = 415
1993/18 PCE Exercise 5.11
What is the remainder when x3 + 5x2 – 3x – 6 is
= 2 15
divided by x + 1 ? D. Simplify 3  3
A.6 B.1 C. – 1 D. – 11
Solution
2005/ 48 PCE Exercise 5.12 3  3 = 3  3
What is the remainder when 5x3 – 13x + 4 is divided by x + 2 ?
A. – 4 B.14 C. – 10 D. 4 = 9
=3
SURD Generally, a  a =a
Irrational or non rational numbers are numbers whose Division in surds
value have no ending
m m
Examples are 2 , 3 , 5 e.t.c the square root of =
n n
all non perfect squares are irrational likewise their
multiples and fractions. Also  is an example of This true for all surds
irrational numbers. we refer to the square root of Examples
non-perfect square as SURD.
35
Surd has it own basic rule of operation with addition, 1. Simplify
subtraction, division and multiplication. 5
Addition of surds 35 35
Example Solution =
5 5
Simplify 2 + 3
Solution = 7
2 + 3 = 2 + 3 17
2. Simplify
But 2 + 3 ≠ 2  3 or 5 4
Solution
Generally
17 17
a + b ≠ ab =
4 4
Also a - b ≠ a b
Rationalization of surds
Multiplication of surds Examples
a  b = a b 1
1. Rationalize
This true for all surds 3
Solution
Examples
1 1 3
a. simplify 2  6 = 
Solution 3 3 3
2  6 = 2 6 1  3 3
= =
= 12 3 3 3
= 4 3 = 4  3 =2 3 Recall that a a =a
55
To rationalize, when the denominator is a single surd 2006/ 9a
we multiply the numerator by the surd in the 75  3
denominator. Simplify , leaving your answer in the form
3 1
10 2
2. Simplify a + b c , where a, b and c are rational numbers
5
Solution
Solution Rationalizing the given surd
10 2
=
10 2

5 75  3 75  3  3  1
5 5 5 3 1
=
3 1  3  1
2 5 Simplifying the numerator and denominator separately.
= 10 =2 10
5 225  3 3  75  3
=
6 3 3  12
3. Simplify 2 3 - +
3 27 15  3  3 3  5 3
Solution =
6 3 2
2 3 - + 18  8 3
3 27 =
2
6 3 3 27
=2 3 - + =9–4 3
3 3 27  27
2009/1b
6 3 3 27
=2 3 - + 5  2 10
3 27 Express in the form m 2 + n 5 ,
3 5 2
9 3 Solution
=2 3 -2 3 + Rationalizing the given surd
9
5  2 10 5  2 10 3 5 2
3 3 = 
= = 13 3 3 5 2 3 5 2 3 5 2
9 Simplifying the numerator and denominator separately
Rationalization of surd by Conjugate
Surd Conjugate their product 15 5  5 2  6 50  2 20
=
95  2
2+ 3 2- 3 22 – 3
15 5  5 2  6  5 2  2  2 5
2- 3 2+ 3 22 – 3 =
45  2
Generally
a + b a- b a2 – b
19 5  35 2
=
The last term is a general expression for Surd and their 43
conjugates. Also from the table above we have seen that  35 19
= 2 + 5 as required
the product of a surd and its conjugate is not a surd since 43 43
(a + b ) (a - b ) = a (a - b)+ b (a - b)
2000/ 8
= a2 - a b + a b - b  b 1 1 1
=a2 – b Simplify 4 - 2 + 3
Generally, in solving a given question, sometimes we
32 8 2
employ one or more of the principles treated above. 2 3 2 3 3 2 3 2
A B C D E
2005/1 3 2 2 2 2 2
Simplify (2 + 3 5 ) – (2 – 3 5 ) 2 2 Solution
Simplifying the given surd in their fractional forms
A 24 5 B 90 C0 D 4 + 18 5
Solution 1 1 1
4 -2 +3
Applying the difference of two squares rule 32 8 2
(2 + 3 5 )2 – (2 – 3 5 )2 = (2 + 3 5 + 2 – 3 5) 1 1 1
=4 + 2 +3
[2 +35 - (2 – 3 5 )] 16  2 4 2 2
= 4(2 + 3 5 - 2 + 3 5 ) 1 1 1 1 1
= 4 - 2  +3
= 4(6 5 ) = 24 5 A 4 2 2 2 2
56
1 1 1 2005 / 4a Neco
=
2
-
2
+3
2 5 2 52
Simplify +
1 52 5 2
3
=3 i.e. Solution
2 2 Applying simple fractional operation principle
3 5 2 52
Rationalizing = 2 (D) +
2 52 5 2
  5  2   5  2 
2002 / 11 Neco
7 52 52
 5  2  5  2
Evaluate =
1
5 1
50
=
5  2
5 2 5 4  52 5 2 5 4   
2 3 2 3 5  22
A3 2 B C 2 D E
3 5 7 94 5 94 5 18
= = i.e. 18
Solution
Simplifying from the core
54 1
7 7
= 1990/3 (Nov)
1 50  1
5 1 5 2 2 5
50 50 Given that: =p+q 10
7 3 22 5
7
= = Where p and q are constants, find the value of p – q
49 49 Solution
5 5
50 50 Rationalizing the given surd
7 2 2 5 2 2 5 3 2 2 5 
×  
=1 
1 2 (C)
= = = 
5 7 2 3 22 5 3 22 5 3 2 2 5 
5 2 Simplifying the numerator & denominator separately
2002/ 4a Neco
6  2  4 10  3 10  2  5
If 6 = 2.45, evaluate without using tables the value of
=
9  2  6 10  6 10  4  5
2 12  7 10  10
correct to two decimal places =
3 2 18  20
.Solution
Rationalizing the given surd 22  7 10
=
2 2 3 2 2
= 
3 2 3 2 3 2 = -11 +
7 10
6 2 2
= = 6 -2 7
3 2 Thus p – q = –11 –
2
Substituting for the given values
= 2.45 – 2 = 0.45 to 2 d.p  22  7 29
2004 / 29 Neco = =  i.e – 14 1/2
2 2
If 7 = 2.646, evaluate 1 to 3 significant figures
7 1 2003/17 Neco Exercise 5.13
A 6.607 B 6.606 C 0.6080 D 0.6077 E 0.6076 2 3 3 2
Solution If =m+n 6
Rationalizing the given surd 2 3 3 2
7 1 Find the values of m and n respectively
1 1
=  A – 5, 3 B 5, - 2 C - 5, - 2 D 2, - 5 E 3, 5
7 1 7 1 7 1
=
7 1 = 7 1 2000/4 (Nov) Exercise 5.14
7  12 6 4 3
3.646 Given that =p+q 3 , where p and q are
= 2.646  1 = = 0.60767  0.608 to 3 s.f. 4 3
6 6
constants, find the value of (p + q).
A 5/13 B 11/13 C 19/13 D 25/13

57
Chapter six Now let us debate it
Mapping Will any new baby have weight? Answer capital yes
Mapping is simply an association or a relation between Can any new baby have two or three different weights
two sets. at the same time? Answer capital NO
Let S be the set of cars { Benz 190, 505, Camry}
and T the set of car producer Thus, for a relation to be a mapping; it must be that:
{ Peugeot, Mercedes Benz, Toyota}, then there is a mapping - Every element of the domain has an image in the co domain
which associate each car with its producer and we write as - The image of every element of the domain is unique
f
f : S → T or S → T You will agree that two or three or more new babies can have
Also by mapping diagram or arrow graph, we can the same weights
show it as
S T Types of mapping
Benz190 M ercedes Injective (one-one)
505 Peugeot A mapping f : S → T is said to be injective or one to one
Cam ry Toyota when elements of S are uniquely paired to elements of T
or if no element of T corresponds to more than one element of S
S T P K
a 1
S = { Benz 190, 505, Camry} is called Domain Benz 190 Mercedes b 2
505
T ={ Peugeot, Mercedes Benz, Toyota} is called co domain Camry
Peugeot
Toyota
c 3
d 4
5
Example MP2 f : P → K as shown below
f : S → T is Injective f : P → K is Injective
P K
a 1 Surjective (onto)
b 2 A mapping f : S → T is said to be surjective or onto if every
c 3 element of T is an image
d 4
5 S T P K
a 1
Domain Co domain Benz 190 Mercedes b 2
505 Peugeot c 3
Note that not all the elements of K are paired or related to P Camry Toyota d 4
Domain of the mapping f is {a, b, c, d} 5

Co domain of the mapping f is{1, 2, 3, 4, 5}


f : S → T is surjective f : P → K is NOT surjective
Image or range of the mapping is {1, 2, 3, 4}
Bijective
Example MP2 is a mapping and is also a function:
A mapping that is both injective and surjective is bijective
reason the co domain is a set of real numbers. So you T
S
can say that:
Benz190
A function is a mapping whose co domain is numbers 505
M ercedes
Peugeot
The mapping f : H → L Cam ry Toyota
H L
a 1
b 2 f : S → T is bijective
c 3
d composition of mapping
Let f : S → T and g : T → U be any function then we can
is acceptable since every element of the domain H is compose then to get a mapping h : S → U, given by
uniquely paired to an element of the co domain L
h(x) = g( f(x) ) or gof or gf for xS
Here the co domain is same as Image or range
we can represent it as:
T
The relations represented below are not mapping
Fig I Fig II f g
L T U
a 1
V
b 2 a 1 S U
c 3 b 2 h
d 3
c 4 Identity mapping
A mapping S : T→ T, where the domain is the same as the
Reasons Reasons co domain i.e S(x) = x for all xT
All the elements in the domain one element a in the T T T R
are not paired to the co domain domain is not uniquely paired a a a 1
b b b 2
i.e paired to more than one element c c c 3
Example MP5
With each newborn baby associate its weight in grams
to the nearest gram. This is a function from the set of identity mapping not identity mapping
newborn babies to the set of natural numbers domain = co domain domain ≠ co domain

58
Constant mapping To determine position of a mapping
A mapping T : R→ S, which assigns all the elements of (Whether it is injective,surjective or both
the domain R to a single element in the co domain S and to locate its domain and co domain)
Here we work based on the given problem; if it has any
R S
x formula, its application helps to determine whether it is
y t injective, surjective or both and to locate its domain and co
z r domain depending on the question.
w
2009/26 Neco
Inverse function
Consider the mapping below
For any function f to have an inverse f- –1 it must be A g B
bijective i.e the function must be injective and p 1
surjective q
r 2
s 3
2002/12 Neco Exercise 6.0 t
Which of the following illustrates a function?
What are the elements in the co-domain of g?
a 1 A.{1, 2, 3} B.{g, 1, 2, 3} C.{p, q, g, l}
A b
c
2
3 D.{p, q, r, s, t} E.{p, q, r, s}
d 4
Solution
co-domain of g = { 1, 2, 3} i.e set B
a 2
b
C E c
d
1
4
Counter illustration
e 3
5 A g B
p 1
q
2005/28 Neco Exercise 6.1 r 2
Suppose f : A  B be defined in the diagram below. s 3
t
A B 4
1 s
Here co-domain of g = { 1, 2, 3, 4} i.e set B
2 t Here image of g = { 1, 2, 3} i.e subset of B
3 w
2003/22 Neco
Then f is said to be Find the domain of f : x  x2 + 6 ,
A1–1 B. both 1 – 1 and onto C. inverse if the range of f is {6, 7, 10, 15}
D. onto E. orthogonal A. {0, 1, 2, 3} B. {-1, 0, 1, 2} C. {5, 6, 8, 12}
D. {-1, 0, 1, 2} E. {-2, -1, 0, 1}
2010/3 Neco Exercise 6.2 Solution
A mapping f : x → y described in the diagram below Here we are to reverse the process as
refers to what type of mapping? range 6
x y
x2 + 6 = 6, find x
x2 = 0
x=0
range 7
A Constant B Equality C Identity D One-one E Onto x2 + 6 = 7, find x
x2 = 1
Ordered pair mapping x =  1 i.e –1, 1
A typical ordered pair mapping is the Euclidean plane
which is made up of (x, y); e.g any point on the graph range 10
is ordered pair: (3,7) you know that x is 3 and y is 7 x2 + 6 = 10, find x
Mathematically X ×Y = {(x,y) : xX, yY} x2 = 4
All elements of x make up the domain x =  4 i.e –2, 2
All elements of y make up the co-domain
Example range 15
Identify with reason a mapping from the pairs below: x2 + 6 = 15
i{(a,4), (b,5), (c,6), (d,7)} ii.{(a,2), (b,2), (c,1), (d 0)} x2 = 9
Solution. Judge by yourself diagrammatically it is: x =  9 i.e –3, 3
Alphabet Natural no. Alphabet Natural no.
a a 2
The domain set = -3, -2, -1, 0, 1, 2, 3
4
b 5 b Thus the domain here = {0, 1, 2, 3}(A) or {-3, -2, -1, 0}
c 6 c 1
d 7 ii d 0
i

59
1997/12 2010/1
Let the function f: w  R be defined by f : x2 – x – 2 , Find the domain of f(x) = x , where x  R,
where W = {- 1, 0, 2, 5, 11} and R the set of real 3 x
numbers. Find the range of f. the set of real numbers.
A. {- 2, 0, 18, 108} B. {- 2, 18, 108} A {x : x R, x ≠ 3} B {x : x R, x ≠ 1}
C. {-1, 0, 2, 5, 11} D. {-2, 0, 18} E. {0} C {x : x R, x ≠ 0} D {x : x R, x ≠ - 3}
Solution Solution
What we are to find here is the image of the function The domain here will be R – f(x) undefined
(though identified here as “range of f”) i.e R minus 3 – x = 0
f(x) = x2 – x – 2 i.e R minus x = 3
Taking the value domain w = {-1, 0, 2, 5, 11} one i.e R , x ≠ 3 ( A)
by one Because if x is equal to 3 then f(x) becomes undefined
f(-1) = (-1)2 – (- 1) – 2 f(5) = 52 – 5 – 2
=1+1–2 = 25 – 7 2010/16 Exercise 6.3
=0 = 18 Given that P = {x : x is a factor of 6} is the domain of
g(x) = x2 + 3x - 5, find the range of g(x).
f(0) = 02 – 0 – 2 f (11) = 112 – 11 – 2 A { -1, 5, 13 } B {5, 13, 49 } C {1, 2, 3, 6 }
=–2 = 121 – 13 D { -1, 5, 13, 49 }
= 108
f(2) = 22 – 2 – 2 2002/2 (Nov) Exercise 6.4
=4–4 Given that the domain of f (x) = x2 + 1 is
=0 {-2, -1, 0, 1, 2}, find the range of f..
Thus, the required set = {-2, 0, 18, 108} (A) A.{3, 0, -1} B.{5, 2, 1} C.{3, 1, 5} D.{2, 0, -1}
2004/32 2009/11 Neco Exercise 6.5
If the domain of f : x  x2 – 2 is {-2, -1, 0, 1}, Find the domain of f: x→ x2 +1 if the range is {1, 2, 5, 10}
find its range. A. {0, 1, 2, 3} B. {-3, -2, -1, 0} C. {-2, -1, 0, 1, 2}
A{2, 1, 0, -1} B{2, -1, -2} C{0, -1, -2} D{2,1, -2, -1} D. {-3, -2, -1, 0, 1, 2, 3} E. {0, 5, 26, 101}
Solution
Its “range” here is the result of the values gotten by To determine injectivity and surjectivity
substituting {-2, -1, 0, 1} into f (x) = x2 – 2 Examples
f (- 2) = (-2)2 – 2 f (0 ) = 02 – 2 Which of the following function f defined below is injective,
= 4 – 2 i.e 2 = 0 – 2 i.e –2 surjective and bijective
(1) f : R  R, f(x) = 2x+1
f (- 1) = (-1)2 – 2 f (1) = 12 – 2 (2) f : R  R, f(x) = ex
= 1 – 2 i.e – 1 = 1 – 2 i.e –1
(3) f : R  +R, f(x) = ex
The set is = {- 2, - 1, 2} (B) (4) f : R  R, f(x) = x2 + 2
2007/17 Neco x2 + 1
If f : x  sec x , find the range ; if the domain is Solution
the set {x : 00  x  600} (1) Let a : bR and suppose that
1 f(a) = f(b)
A. {y : 1  y  2} B. {y :  y  1}
2
Then 2a + 1 = 2b + 1
1 1
C. {y : 0  y  1} D. {y : – 2  y  2 } So 2a = 2b and hence a = b
E. {y : –1  y  1} Thus, f is injective
Solution
Here we substitute x value between 00 and 600 To show surjectivity
When x = 00 Let yR be such that
Sec x = 1 f(x) = y
Cos x so 2x + 1 = y
= 1 = 1 i.e 1 x = 1 (y – 1)R
2
cos 00 1
Even if we take y = 1 then x = 0R
When x = 600 Accordingly f is surjective and hence bijective
Sec x = 1
cos 60 (2) Let a, bR such that
= 1 i.e 2 f(a) = f(b)
½
Then, ea = eb
Introducing y to represent the range and minding our
inequalities. Loge e a = Log e e b a Log e e = b Log e e
Domain {x : 00  x  600} becomes range  a = b Hence f is injective
{y : 1  y  2 } (A)
60
To show surjectivity Value of a function
Let yR be such that A function takes another value when the function is
f(x) = y redefined; it’s redefinition can be real numbers or the
Then ex = y multiple of the former variable.
Examples
Thus x = Log e y (a) If f(x) = 2x + 1, find f(3x)
Log of negative numbers do not exist (Range = R) Solution
Hence f is not surjective hence not bijective The question means substituting 3x for x
i.e. f (3x) = 2(3x) + 1
(3) However, since we restrict our range to R+ , we = 6x + 1
conclude here that f is surjective and so it is bijective
(b) If f(x) = 10x – 3, find f(2)
(4) ) Let a, bR such that Solution
f(a) = f(b) The question means, substituting 2 for x,
Then a2 + 2 = b2 + 2 f (2) = 10(2) – 3
a2 + 1 b2 + 1 = 20 – 3
You can observe that a2 = b2 or a =  b. Accordingly = 17
it is possible to have a ≠ b when f(a) = f(b).
(c) If f (2x – 3) = 4x2 + 5x + 2 find f (1)
Thus f is not injective ( and so not bijective)
Solution
First, we solve for x
To show surjectivity
2x – 3 = 1
Let yR be such that
2x = 1 + 3
f(x) = y
2 x = 4/2 i.e 2
Then , x + 2 = y
Then f(1) is obtained by substituting x = 2 in the function
x2 + 1
f(2x – 3) = 4x2 + 5x + 2
Solving x =  y  2 Thus, indirect substitution
1 y
f(1) = 4(2)2 + 5(2) + 2
y 2 = 16 + 10 + 2
Observe that is not defined for all values of = 28
1 y
y( y = 0 makes the result becomes 2 which is 1990/15 UME
If f (x – 4) = x2 + 2x + 3, find f (2)
not real.Thus, f is not surjective A.6 B.11 C.27 D.51
Solution
2003/8 Exercise 6.6 First, we solve for x
Which of the following functions is/are one – to –one? x–4 =2
f : x →x2 , g : x →2x + 1 , h : x → x x=2+4
A f only B g only C h only D f and h only =6
Then f (2) is obtained by indirectly substituting for 6
2000/10 theory Exercise 6.7 and not 2
( a ) If g : x  2x – 1 , for all x R, the set of f (2) = (6)2 + 2 (6) + 3
5x + 3 = 36 + 12 + 3
real number, find the : ( i ) domain of g; = 51 (D)
(b) Determine whether or not g in 10(a) is onto
(c) find the range of value of x for which 2x2 + x - 6< 0 1993/17 UME
If the function f is defined by
f(x + 2) = 2x2 + 7x – 5, find f(–1)
A. – 10 B. – 8 C.4 D.10
Solution
First, we solve for x
i.e. x + 2 = – 1
x = –2 –1
x=–3
Then f (–1) is obtained by indirectly substituting –3 for x
and not – 1
f (–1) = 2 (– 3)2 + 7 (– 3) – 5
= 2 (9) – 21 – 5
= 18 – 26
= – 8 (B)

61
1988/16 UME For f(4), only 3x – 2 fits into it
If g (y) = y – 3 + 11 what is g (y + 3) ? For f(–3), only 5x – 3 fits into it
11 y2 – 9 Thus, f(4) – f(–3 ) = [ 3(4) – 2 ] – [ 5(–3) – 3 ]
A. y + 11 B. y + 11 = (12 – 2 ) – (–15 – 3)
11 y (y + 6) 11 y (y + 3) = 10 + 18
= 28 ( A )
C. y + 30 + 11 D. y + 3 + 11
11 y (y +3) 11 y (y – 6)
1996/20
Solution
Find the linear function f(x) such that f /(x) = –3
The question means substituting y + 3 for y
and f (1) = 4
i.e g (y + 3) = ( y + 3) – 3 + 11
11 (y + 3)2 – 9 A. –3x + 4 B. – 3x – 4 C.4 + 3x D. – 3x – 7 E –
= y+3–3 + 11
3x + 7
11 (y + 3)2 – 32 Solution
The several form of a linear function is gotten from
To enable us solve by difference of two squares
y = mx + c i.e f (x) = mx + c
= y + 11
11 (y + 3 – 3) (y + 3 + 3) Condition 1, f /(x) = –3
= y + 11 Implies gradient m = 3
11 y (y + 6) (A)
Condition 2 f(1) = 4
1989/16 UME i.e m (1) + c = 4
If f (x) = 2x2 – 5x + 3, find f (x + 1) m+c=4
A.2x2– x B.2x2 – x + 10 C.4x2 + 3x + 2 D.4x2 + 3x + 12 Substituting for m value
Solution –3+c=4
The question says substitute x + 1 for x
c=7
i.e. f (x +1) = 2 (x +1)2 – 5 (x + 1) + 3
Substituting for m and c, we have that the linear function f
= 2(x2 + 2x + 1) – 5x – 5 + 3
(x) = –3x + 7 (E)
= 2x2 + 4x + 2 – 5x – 5 + 3
Collect like terms
= 2x2 + 4x – 5x – 5 + 3 + 2 2003/4 (Nov)
The function f and g are defined on the set R of real numbers
= 2x2 – x (A)
by: f : x  x2 – 2x – 1, g : x  x – 1
1996/18 Find the value of x for which f(x) = g(x) – 2
A. 0 or 3 B. 0 or 2 C. –1 or 1 D. 1 or 2
Let the function f : R  R be defined by
Solution
 4 x  7 if x  3  f(x) = g(x) – 2
  x2 – 2x – 1 = x – 1 – 2
f (x) =  x 2  3 if  2  x  3
 2 x  3 if x  2  x2 – 2x – x – 1 + 3 = 0
  x2 – 3x + 2 = 0
find f (–1) Factorizing
A. – 11 B. – 4 C. – 2 D. – 1 E. 5 x2 – 2x – x + 2 = 0
Solution x (x – 2) – 1(x – 2) = 0
f(x) is defined for 3 three conditions. For the present (x – 2)(x – 1) = 0
problem of f (–1) , it fits into –2 ≤ x ≤ 3 x – 2 = 0 or x – 1 = 0
i.e. f (x) = x2 – 3 x = 2 or 1 (D)
Thus f (–1) = (–1)2 – 3
= –2 (C) 1991/16 UME
Note: In problem like this, care must be taken in If g (x) = x2 + 3x + 4, find g(x + 1) – g (x)
identifying the case that applies. A.(x + 2 ) B.2(x + 2) C.(2x + 1) D.(x2 + 4)
Solution
2013/20 g(x) is already given,
The function f : R → R is defined by Next we find g (x + 1)
3x + 2 : x > 4 By substituting x +1 for x
f( x ) = 3x – 2 : x = 4 g(x + 1) = (x + 1)2 + 3(x + 1) + 4
5x – 3 : x < 4 = x2 + 2x + 1 + 3x + 3 + 4
Find f(4) – f(–3 ) = x2 + 5x + 8
A 28 B –26 C –26 D –28 Hence g (x + 1) – g (x)
Solution = x2 + 5x + 8 – (x2 + 3x + 4)
f(x) is defined for three conditions here = x2 + 5x + 8 – x2 – 3x – 4
x > 4 i.e 5, 6, 7… +∞ = x2 – x2 + 5x – 3x + 8 – 4
x=4 = 2x + 4
x < 4 i.e 3, 2, 1… – ∞ = 2(x + 2) (B)
62
2004/29 Neco (Dec)
A.1 + 1 B. x + 1 C. – 1 – 1
If g (x) = x2 + 2x + 1.
x x–2 2x – 1 x x–2
Find the expression for g(x + 2) – g(x + 1)
A. 2x2 + 10x + 13 B. x2 + 6x + 9
D. –1 – 1 E. 1 + 1
2
C. x + 4x + 4 D.2x + 5 E. – 2x – 3
x 2x – 1 x x –2
2
Solution
g (x + 2) – g(x + 1) Composition of function
= (x + 2)2 + 2(x + 2) + 1 – [ (x + 1)2 + 2(x + 1) + 1] If f(x) = x2 and g(x) = 2x – 1, find ( i ) gf(x) ( ii ) fg(x)
= x2 + 4x + 4 + 2x + 4 + 1 – (x2 + 2x + 1 + 2x + 2 +1) Solution
= x2 + 6x + 9 – (x2 + 4x + 4) gf(x) = g( f(x) )
= x2 + 6x + 9 – x2 – 4x – 4 = g(x2 )
= 2x + 5 (D) = 2x2 – 1

2004/34 Neco But fg(x ) = f( g(x) )


Given that g : x  ax2 + bx is a mapping on the set of = f ( 2x – 1 )
real number. Given that g(2) = 6 and g(–1) = –6, = ( 2x – 1 )2
find g(3) + g(4) = 4x2 – 4x + 1
A. – 2 B. 2 C. 6 D. 4 E. 10 2000/20 Neco
Solution If f(x) = 2x + 1 and g(x) = x2 – 2, find the expression
g(x) = ax2 + bx which defined the product g▪f
g(2)  a(2)2 + b(2) = 6 A. 4x2 + 4x + 1 B. 4x2 + 4x – 1 C. 4x2– 4x + 1
i.e 4a + 2b = 6 ----------(1) D. 2x3 + x2 – 4x – 2 E. 2x3 – x2 + 4x –2
Solution
Also g(–1)  a(–1)2 + b (–1) = – 6 g▪f = g(2x + 1)
a – b = – 6 --------- (2) = (2x + 1)2 – 2
Next, we solve the resulting simultaneous equations = 4x2 + 4x + 1 – 2
From (2) a = b – 6 = 4x2 + 4x – 1 (B)
Substitute a in to (1)
4(b – 6) + 2b = 6 2000/36 Neco
4b – 24 + 2b = 6 If f : x  2x + 1 and g : x  2(x – 2),
6b = 6 + 24 1
6b = 30 what is the value of f▪g ( 2 ) ?
6 6 A. 8.0 B. 1.0 C. 0.5 D. 0.0 E. –5.0
b = 5 Solution
Thus, a = 5 – 6 First, we solve f▪g
a = –1 f▪g = f [ 2(x – 2) ]
 g(x) = 5x – x2 = f (2x – 4)
g(3) + g(4) = 5(3) – 32 + 5(4) – 42 = 2(2x – 4 ) + 1
= 15 – 9 + 20 – 16 = 4x – 8 + 1
= 10 (E) = 4x – 7
1 1
Thus f▪g( 2 ) = 4( 2 ) – 7
2008/1 Exercise 6.8
A function f is defined on the set R, of real numbers by =2–7
=–5 (E)
f: x → px2 + qx + 2, where p and q are constants.
2004/43 Neco (Dec)
If f(-2) = 0 and f(1) = 3, find f(- 4).
If f : x  x – 1 and g : x  x – 3 , find the
1980/43 UME Exercise 6.9 x–1 2x – 1
If a function is defined by f (x + 1) = 3x2 – x + 4, find f (0) expression for gof
A.4 B.6 C.0 D.8 E.2
A. 2x + 4 B. – 2x – 4 C. 2x – 1
1982/17 UME Exercise 6.10 x–3 x–3 x–3
If f(x – 2) = 3x2 + 4x + 1. find f (1) D. 2x – 4 E. x – 4
A.8 B.40 C.7 D.32 E.21 x+3 2x – 2
Solution
1994/15 PCE Exercise 6.11 gof = g  x  1 
 x 1
If f(x – 2) = 4x2 + x + 7, find f(1)  
A. 10 B. 12 C. 26 D. 46 x 1  x 1 
= –3  2   – 1
1983 /40 UME Exercise 6.12 x 1  x  1 
If f (x) = 1 + x – 1 , find f (1 – x) = x – 1 – 3(x + 1)  2x – 2 – 1
x–1 x2 – 1 x+1 x+1
63
= x – 1 – 3x – 3  2x – 2 – 1(x + 1) = 3x ÷ 2x2 – 2x + 1
x+1 x+1 x–1 (x - 1)(x - 1)
= - 2x – 4  2x – 2 – x – 1
= 3x × (x - 1)(x - 1)
x+1 x+1
x–1 2x2 – 2x + 1
= - 2x – 4  x–3
x+1 x+1 = 3x (x - 1)
= - 2x – 4  x +1 2x2 – 2x +1
x+1 x–3
Thus gof(3) = 3×3 (3 - 1) = 18
= – 2x – 4 (B)
2×32 – 2×3 + 1 13
x–3
2007/6 Neco
Let f : x  y and g : y  z be mapping on the set of real
1996/19 (Nov)
numbers defined by f(x) = x + 1 and g(y) = (y + 1)2
The function f and g are defined over the set R of real
find gof.
numbers as follows; f : x  tan x , g : x  (1  x 2 ) . A. 2x2 – 8x + 4 B. 2x2 – 4x – 4 C. 2x2 + 4x + 4
Find gof
2
D. x + 4x + 4 E. x – 4x – 4
2

A sec2 x B. cos2 x C. sin x D sec x E. cos x Solution


Solution gof = g(x + 1)
By composition rule in functions Next, y is in the place of the independent function hence.
gof = g (tan x) = (x + 1 + 1)2
= (x + 2)2
= 1  (tan x) 2 = x2 + 4x + 4 (D)
= 1 tan 2 x 2000/14 UME
By trig identities If g(x) = 2x + 3 and f(x) = 3x2 – 2x + 4, find f [ g(–3 )]
A. 37 B. 1 C. – 3 D. – 179
= sec 2 x i.e (sec)
2
  1
2
Solution
= sec x (D) Taking the bracket i.e. g( – 3). Thus
From g(x) = 2x + 3
2005/1 g(– 3 ) = 2(–3) + 3
The functions f and g are defined on the set R of real numbers by =–6+3
f:x  x , x  1, g : x  3x xR =–3
x – 1, x2 + 1 Therefore, f [ g(– 3 ) ] means substitute – 3 into
find: (a) fog B. gof(3) f(x) = 3x2 – 2x + 4
Solution f[ g(– 3 ) ] = 3 (–3 )2 – 2 (– 3) + 4
(a) fog =  3x  = 3(9) + 6 + 4 = 37 (A)
 x 2 1  2002/39 (Nov)
 
If f(x) = x2 – 3x + 2 and g(x) = x – 1 for all x R, find the
 3x   3x 
=  2  ÷  2  1 value of x for which fg(x) = 0
 x 1  x 1  A. 1 or 2 B . 2 or 3 C. 1 or 3 D. –1 or 3
Solution
= 3x ÷ 3x – (x2 + 1) fg(x) = 0
x2 + 1 x2 + 1 f(x – 1) = 0
(x – 1)2 – 3(x – 1) + 2 = 0
= 3x ÷ 3x – x2 – 1
2 x – 2x + 1 – 3x + 3 + 2 = 0
2
x +1 x2 + 1 x2 – 5x + 6 = 0
= 3x × x2 + 1 Factorizing
2
x +1 3x – x2 – 1 x2 – 2x – 3x + 6 = 0
x(x – 2) – 3(x – 2) = 0
= 3x (x – 2)(x – 3) = 0
3x – x2 – 1 x – 2 = 0 or x – 3 = 0
x = 2 or 3 (B)
(b) First we find gof
2002/3
gof = g  x  Given that g : x  3x and g : x  3x – 2, find fog(2)
 x 1 
 
2
A. 81 B. 36 C. 27 D. 9
 x    Solution
= 3   ÷  x   1
First , we find fog
 x 1   x 1 
fog = f (3x – 2)
= 3x ÷ x2 + 1 = 33x – 2
x–1 (x - 1)(x - 1) Thus, fog(2) = 33( 2 ) – 2
= 3x ÷ x2 + (x - 1)(x - 1) = 36 – 2
x–1 (x - 1)(x - 1) = 34 i.e 81 (A)
64
2000/10 obj Solution
Given that f : x  1 – 2x , and g : x  2x + 1, xR f(x) = 1 x – 3
(set of real number), find [fog (x) – gof (x)] 2
A. – 8x B. – 4 C. 0 D. 2 Let y = 1 x – 3
Solution 2
fog(x) = f (2x + 1) gof (x) = g (1 – 2x) Next , we make x , subject of formula
= 1– 2(2x + 1) = 2(1– 2x ) + 1 y+3= 1x
= 1– 4x – 2 = 2 – 4x + 1 2
= – 4x – 1 = 3 – 4x 2(y + 3) = x
Thus, f – 1(x) = 2(x + 3) (B)
Thus, [fog (x) – gof (x)] = – 4x – 1 – (3 – 4x)
= – 4x – 1 – 3 + 4x i.e – 4 (B)
2000/37 (Nov)
1997/5 If f : x 2x – 1 , where x is a real number , find the value of
Two function f and g are defined over the set R of real x for which f(x) = f – 1 (x)
A. 1 B. 2 C. 1 D. 3
numbers as follows ; f : x  2x – 3 g : x  x2 + 5
Find f [g(x – 1)] 2 3
A 2x2 – 4x + 9 B. 2x2 + 11 C. 2x2 – 20x + 30 Solution
First , we find f – 1(x)
D.4x – 16x + 21 E. 2x2 + 5
2
Let y = 2x – 1
Solution
Find f [g(x – 1)] Next, we make x the subject formula
Starting with g(x – 1) : y + 1 = 2x
from g(x) = x2 + 5 y+1=x
2
g(x – 1) = (x – 1)2 + 5
= x2 – 2x + 1 + 5 Thus f – 1(x) = x + 1
= x2 – 2x + 6 2
Thus, f [g (x – 1)] = f (x2 – 2x + 6) Now ,we can find x value for which f(x) = f – 1 (x)
= 2(x2 – 2x + 6) – 3 i.e 2x – 1 = x + 1
2
= 2x2 – 4x + 12 – 3
= 2x2 – 4x + 9 (A) Cross multiply
2(2x – 1) = x + 1
2006/6 Exercise 6.13 4x – 2 = x + 1
Two functions f and g are defined by f : x→3x – 1 and 4x – x = 1+ 2
3x = 3
g : x →2x3, evaluate fg (-2)
x=1 (C)
A - 49 B - 47 C -10 D -9

2009/29 (Nov) Exercise 6.14 2004/6


The functions f and g are defined on the set R, real Given the function f (x) = 5x – 2 , find f – 1 (–1)
numbers by f : x →2x2 – 3 and g : x → 4 – x. A. – 3 B. 1 C. 1 D. 3
Find fog. 5 5
A x2 – 16x + 29 B x2 + 16x – 29 Solution
C 2x – 16x – 29
2
D 2x2 + 16x – 29 First we find f – 1(x) from f(x)
f(x) = 5x – 2
2004/35 Neco Exercise 6.15
Let y = 5x – 2
Given f(x) = x2 – 1 and g(x) = 2x + 3,determine the
Next , we make x subject formula
formula for gf(x)
y + 2 = 5x
A. 2x2 + 4x + 1 B. 2x2 + 1 C. 2x2 – 2
2 1 (y + 2) = x
D. x + 4x + 5 D. x + 1
5
Thus, f – 1 (x) = 1 (x + 2)
Inverse of a function 5
If y = f (x) is a function, we obtain the inverse function f –1 Hence, f – 1(–1) = 1 (–1 + 2) = 1 (B)
by solving the equation y = f (x) for x in terms of y then 5 5
later change y to x 1995/4 (Nov)
If g : x → 2x – 3, find g – 1 (– 3)
2007/ 47 Neco A. – 9 B. – 9 C. – 4 D. 3 E. 9
The function f over the set of real numbers is defined by 2 4 5 4 4
f(x) = 1x – 3, find f –1(x) Solution
2 First, we find g – 1 (x)
A. – 2(x – 3) B. 2(x + 3) C. 3(x – 3) Let y = 2x – 3
D. 5(x + 3) E. 10(x – 3) 4
Next, we make x subject formula
65
4y = 2x – 3 y(3x + 2) = 2x – 1
4y + 3 = 2x 3xy + 2y = 2x – 1
4y + 3 = x Collect terms in x together
2 3xy – 2x = – 2y – 1
Changing y to x for proper identification x(3y – 2) = – 2 y – 1
Thus g – 1 (x) = 4x + 3 x = – (2y + 1)
2 3y – 2
and g – 1 (– 3) = 4(–3) + 3 Thus, f –1(x) = – (2x + 1)
2 3x – 2
=–9
The largest domain of f –1(x) is 3x – 2 ≠ 0
2 (A)
1992/7 (Nov) i.e x ≠ 2/3 (B)
If f: x → x2 – 4, where x is a real number, find f – 1(5) 1994/12
If g: x → x + 3 , x  2 , find the value of x for which
A. 1 B. 3 C.  3 D. 21 E. 21 3x – 2 3
Solution g – 1 (x) is not defined.
First, we find f – 1 i.e inverse of f A. – 2 B. – 1 C. 1 D2 E. 3
Let y = x2 – 4 3 3 3 3
y + 4 = x2 Solution
x= y4 First , we find g– 1 (x)
Let y = x + 3
Changing y to x for proper identification
3x – 2
f – 1 (x) = x4 Next, we make x , subject formula
y(3x – 2) = x + 3
Thus f – 1 (5) = 5 4 3xy – 2y = x + 3
Collect terms in x together
= 9 =  3 (C) 3xy – x = 2y + 3
2003/24 Neco x(3y – 1) = 2y + 3
If f : x  3x + 4 ; x  1, find f – 1(2) x = 2y +3 Thus g– 1 (x) = 2x + 3
x–1 3y – 1 3x – 1
For the function not to be defined
A. – 6 B. – 3 C. –1/6 D. 3 E. 6 The denominator 3x – 1 = 0
Solution i.e. 3x = 1
First, we find f –1 (x) from x=1
f (x) = 3x + 4 3 (C)
x–1 1999/ 15 (Nov)
let y = 3x + 4 If g : x  3 – x xR, the set of real number ,
x–1 3+x
Next, we make x subject formula find the value of x for which g – 1(x) does not exist
y(x – 1) = 3x + 4 A. –3 B. –1 C. 4/3 D. 4
xy – y = 3x + 4
Solution
Collect terms in x together
First , we find g – 1(x )
xy – 3x = y + 4
x(y – 3) = y + 4 Let y = 3 – x
x=y+4 3+x
y–3 Next , we make x the subject formula
f –1(x) = x + 4 y(3 + x) = 3 – x
x–3 3y + xy = 3 – x
Thus, f –1(2) = 2 + 4 Collect terms in x together
2–3 xy + x = 3 – 3y
= 6 i.e – 6 (A) x(y + 1) = 3 – 3y
–1 x = 3 – 3y
2004/33 Neco y+1
If f (x) = 2x – 1 , the largest domain of f – 1 (x) is when Thus, g – 1(x) = 3 – 3x
3x + 2 x+1
A. x = 2 B. x ≠ 2 C. x  - 3 E. x = 3 If g – 1 (x) does not exist then the denominator
3 3 2 x+1=0
Solution x = – 1 (B)
First, the find f – 1 (x) 2006/9 Exercise 6.16
f (x) = 2x – 1
3x + 2
1 1
If f (x) = , x ≠ 2, find f – 1 (- ).
Let y = 2x – 1 2 x 2
3x + 2 A 4 B 0 C -2 D -4
66
2009/8 (Nov) Exercise 6.17 Joint cases
A function f is defined by f: x →3x - 2 ,
5 1992/10 (Nov)
find the inverse of f. Two functions f(x) and g(x) are defined on the set of real
1 1 numbers by f(x) = 3x2 – 2 and g(x) = x + 3.
A (3x + 2) B (3x + 6)
15 15 Find: (a) f(–2) ; (b) g – 1   3 
1 1  4
C (5x + 2) D (5x + 6) (c) the value of x for which f (g (x)) = g (f(x))
15 15
(d) f – 1 (g (4))
2003/35 Exercise 6.18 Solution
4 x 1 (a) To find f(–2)
If f : x → , where x R, find f – 1 (- 2).
3 f(x) = 3x2 – 2 becomes
A–3 B 5 C 4 D 1 f(–2) = 3 (–2)2 – 2
4 5 3 = 12 – 2 i.e. 10

2010/11b Neco Exercise 6.19 (b) g – 1   3  First, we find g – 1


Find the inverse of the function f(x) = 5x + 11  4
2x – 4 Let y = x + 3
Next, we make x subject of formula
2000/9 Exercise 6.20 y–3=x
Given that f : x  7 – x , xR (the set of numbers), Changing y to x for proper identification
find f – 1  g – 1 (x) = x – 3
A. x + 7 B. x C.–7 D.7 – x Thus g – 1   3  =  – 3
3
7 x  4 4
15
2002/25 Exercise 6.21 = 
4
Given that f; x  1 , x  – 1 , find f – 1(x)
(c) f (g (x)) = g (f (x))
2x + 1 2
f (x + 3) = g (3x2 – 2)
A. 2x + 1 B. 1 – x , x  0
3(x + 3)2 – 2 = 3x2 – 2 + 3
2x
3(x + 6x + 9) – 2 = 3x2 + 1
2
C. 2x , x  1 D. 2x – 1
3x2 + 18x + 27 – 2 = 3x2 + 1
1–x
24 = –18x
24 4
2002/20 Exercise 6.22 x=  = 
3 18 3
If f (x) = x – 4 , (x  4,  2 ), for all (d) f – 1 (g (4))
2x – 3 xR, find the value of x for which First, we find g(4) from g(x) = x + 3.
–1
f does not exist. g (4) = 4 + 3 i.e 7
A. 1 B. 3 C. 2 D. 4 Next, f (g (4)) = f (7) , But we find f – 1 of f(x)
–1 –1

2 2 Let y = 3x2 – 2
2004/16 Neco (Dec) Exercise 6.23 Next, we make x the subject of formula
If f : x  3 – 2x, find f – 1 y + 2 = 3x2
A. x – 3 B. x – 1 C. 3 – 3x y + 2 = x2
2 3
E. 3x – 3 D. 2x – 3 y2 =x
3
2000/10 theory Exercise 6.24 Changing y to x for proper identification
( a ) If g : x  2x – 1 , for all x R, the set of x2
5x + 3  f – 1 (x) =
real number, find the : 3
( ii ) g – 1 (iii) value of x for which g – 1 does not exist 72
Thus ,f – 1 (7) = =  3
3
2005/33 Neco Exercise 6.25
Let g : x  x3 + 5 be a function on the set of real 1998/4 (Nov)
number onto the set of real number, find the (a) If f(x) = x – 1 , x  1 and x 2, find f – 1 (x)
inverse of f(x) x– 2
Solution
A. x 5 B. 3 x 5 C. 4
x2  5 Let y = x – 1
D. E. x2  5 x– 2
3 x 5
67
Next, we make x the subject formula x
Thus, gf – 1 = g ( 3 )
y(x – 2) = x – 1
xy – 2y = x – 1 = 2 sin x (B)
Collect terms in x together 3
xy – x = 2y – 1 2007/ 26 Neco
x(y– 1) = 2y – 1 If : x  2 , g : x  x + 1 and h : x  x ,
x = 2y – 1 x 2
–1 –1
y–1 find f ogoh (1)
Thus f – 1 (x) = 2x – 1 A. 1 B. 2 C. 4 D. 3 E. 2
x–1 3 3 3 2
( b) If f : x  2x – 5 and g : x  sin x, Solution
where x is a real number , find : First , we find f –1 and h –1
f(x) = 2 h(x) = x
x 2
( i ) (fog)(x) (ii) (gof)    let y = 2 let y = x
2 x 2
Solution x= 2 x = 2y
( i ) (fog) = f(sin x) y h –1(x) = 2x
= 2(sin x) – 5 –1
f (x) = 2
= 2 sin x – 5 x
(ii) (gof)    First, we find f –1ogoh–1(x) = f –1og(2x)
 
2
= f –1(2x + 1)
First (gof) (x) = g(2x – 5) = 2
= sin(2x – 5) 2x + 1
Thus, (gof)  = sin 2 ×  – 5 Thus, f –1 ogoh–1 (1) = 2 = 2 (B)
2 2 2 (1) + 1 3
= sin 175
= 0.08716 2004/9 Neco Exercise 6.26
Given f(x) = 1 , and g(x) = x where xR,
2002/14 Neco x2 – 1 x+2
If f : x  x2 + 2 and g : x  2x + 2, find f – 1g(2) find the : (i) inverse of g(x) ;
A. 1/2 B. 2 C. 4 D. 36 E. 38 (ii) expression g–1 f(x)
Solution (iii) largest domain of f(x) and g(x) respectively
To find f – 1 g(2) first, we resolve f – 1
f(x) = x2 + 2 2001/1 Neco Exercise 6.27
Let y = x2 + 2 (a) State the condition necessary for a function to have
Next, we make x subject formula
inverse.
y – 2 = x2
4x  3 4
x= y2 (b) If f : x → , and g : x → 1+ ,
2x 1 2x 1
f – 1 (x) = x  2 find
(i) f –1 (x)
Next, we find f – 1 g(x) (ii) g –1 (x)
= f – 1 (2x + 2) (iii) f –1 o g –1 (0)
= 2x  2  2 (iv) g –1 of –1 (0)
= 2x
Thus f – 1 g (2) = 2 2 = 4 i.e 2 (B)

2002/15 Neco
If f:x – 3x, and g:x  2sin x, the formula for gf – 1 is
A. 2 sin x B. 2 sin x C.2sin 3x D. 3 sin 2 x
3 3 3
Solution
To find gf – 1 , first, we find f – 1
f (x) = 3x
Let y = 3x
y =x
3
f –1(x) = x
3
68
Chapter seven Non tabular binary operation for more than
Binary operation one property
Binary operation is simply the rule of combining any
1994/10
two elements of a given non-empty set. It is denoted by
A binary operation Ө is defined on the set R of real numbers
mathematical symbols ∗, Δ, ʘ, , , Ө etc. by x Ө y = x + y – 2xy, where x, y belong to R
Defined operation (a) Determine whether or not the operation Ө is commutative
This is the basic rule to be followed in given problem (b) Find the Identity element of R under Ө
Eg a  b = a × b (c) Find the inverse of a general element P, where P belongs to R
a  b = 2a + 3ab (d) Find the element of R under the operation Ө that has no inverse
a Δ b = 7 – a / b etc (e) Calculate (7 – 5 2 ) Ө (10 + 7 2 ) giving your answer
There are some basic properties of binary operation namely: in the form a + b 2 , where a, b, are rational numbers.
closure, commutative, associative, identity inverse and Solution
distributive. We shall treat these properties with live (a) The operation Ө on a, bR is commutative
examples to enable us grab the required concept if a Ө b = b Ө a
a Ө b = a + b – 2ab
Closure Property and b Ө a = b + a – 2ba
A set D is said to be closed under the binary operation  Which same as a + b – 2ab Thus the operation Ө is
if for any a  b = c a, b, c D commutative
on the other hand if a, bD and c D then  is not closed
(b) Properties of identity shows that:
Either a Ө e = a or e Ө a = a
eg a  b = a × b and a × b = c where a, b, c 
Now using a Ө e = a ____________ (1)
2  3 = 2×3 i.e 6 Here 2, 3, 6   hence  is closed Next we apply the defined operation to (1)
a Ө e = a + e – 2ae ______________ (2)
But 2  6 = 2× 6 For an identity element, (2) = (1)
i.e a + e – 2ae = a
=2  6 complex number hence  is not closed e – 2ae = 0
e(1 – 2a) = 0
Commutative property
e = 0 or 1 – 2a = 0
A binary operation Δ is said to be commutative if
Identity element e = 0
xΔy =yΔx (c) The inverse of P is P–1
eg 5 × 7 = 7 × 5 thus × is commutative under  Condition of inverse is that P Ө P –1 = e
But 5 – 7 ≠ 7 – 5 thus – is not commutative under  From (b) identity element is zero
Thus P Ө P–1 = 0 ________ (1)
Associative property Applying the defined operation
P Ө P–1 = P + P–1 – 2PP–1 ______ (2)
A binary operation Ө is said to be associative if for
Equating (2) to (1)
any x,y,z element P + P–1 – 2PP–1 = 0
(x Ө y) Ө z = x Ө (y Ө z) Next, we make P–1 the subject formula
eg (2 × 3 )× 4 = 2 × (3 × 4 ) thus × is associative under  P–1 – 2PP–1 = – P
But ( 7 × 5 ) – 2 ≠ 7 × (5 – 2) thus – and × are not associative P–1(1 – 2P) = – P
P
P–1 =
Identity property 1  2P
An element e of the binary operation  is said to be the (d) Operation has no inverse implies the value for which the
identity if for any b b∗e=b P
inverse from (C) is undefined
1  2P
Inverse property
An element a has an inverse a–1 under the binary i .e 1 – 2p = 0 (denominator equal zero)
operation  if 1 = 2P
a  a–1 = e where e is the identity element 1
= P
2
Distributive property (d) Applying the defined operation x Ө y = x + y – 2xy
Then  is Left distributive over ʘ (7 – 5 2 ) Ө (10 + 7 2)
a  (b ʘ c) = (a  b)ʘ(a  c)
= (7 – 5 2 ) + (10 + 7 2 ) – 2[ (7 – 5 2 ) (10 + 7 2 ) ]
(a  b)ʘ c = (a ʘ c)  (b ʘ c) =7–5 2 + 10 + 7 2 – 2[70 + 49 2 – 50 2 – 35(2)]
 Right distribution over ʘ = 17 + 2 2 –2[– 2 ]
For better understanding we shall treat this topic under = 17 + 4 2
some smaller headings
69
1994/9 (Nov) Solution
A binary operation  is defined on the set R of real (a) ( i ) operation ∗ is commutative on x, y  R if
x∗y=y∗x
numbers by a  b = */ a2 + b2 – ab for all a, b  R
x ∗ y = x + y + xy
(a) show that  is commutative and y ∗ x = y + x + yx
(b) Calculate (i) 2 ∗ (– 4) They are same, thus, commutativity holds
(ii) 2 ∗ [ (– 4)  1 ] (a) ( ii ) Operation ∗ is associative on x, y and z R if
(iii) [ 2 ∗ [ (– 4) ]  1 (x + y )  z = x + ( y  z )
(c) Show that  is not associative First apply the defined operation to the brackets
(d) Solve the equation 4  x = 28 LHS (x + y + xy) ∗ z = x + y + xy + z +(x + y +xy)(z)
Solution = x + y + xy + z + xz + yz + xyz
(a) The operation  is commutative on x, yR if RHS x ∗ (y ∗ z) = x ∗ (y + z + yz)
x∗y=y∗x = x + y + z + yz + x (y + z + yz)
x ∗ y = x2 + y2 – xy = x + y + z + yz + xy + xz + xyz
and y ∗ x = y2 + x2 – yx both results are same LHS = RHS thus, associativity holds
Thus operation ∗ is commutative
(b) Identity element property shows that
(b) ( i ) 2 ∗ (– 4) = 2 + (– 4) – 2×(– 4)
2 2 Either x ∗ e = x or e ∗ x = x
= 4 + 16 + 8 Using x ∗ e = x _________ (1)
= 28 Applying the defined operation to (1)
( ii ) 2 ∗ [ (– 4)  1 ] x ∗ e = x + e + xe ___________ (2)
First, we solve bracket item For an identity element (2) = (1)
(– 4) ∗ 1 = (– 4)2 + 12 – (– 4) × 1 x + e + xe = x
= 16 + 1 + 4 Next, we make e subject formula
= 21 e + xe = x – x
Thus 2 ∗ [ (– 4) ∗ 1 ] = 2 ∗ 21 e(1 + x) = 0
= 22 + 212 – 2×21 e = 0 or 1 + x = 0
= 4 + 441– 42 Thus identity element e = 0
= 403
1997/3
(iii) [ 2  (– 4) ] ∗ 1
First we solve the bracket item A binary operation ∗ is defined on the set R of real
2 ∗ (– 4) = 22 + (– 4)2 – 2 × (– 4) numbers by a ∗ b = a + b – 2ab, where a, bR
= 4 + 16 + 8 Determine whether or not ∗ is
= 28 (i) Closed
Thus [ 2∗ (– 4) ] 1 = 28 1 (ii) Commutative
= 282 + 12 – 28 × 1 (iii) Associative on the set R.
= 784 + 1 – 28 Solution
= 757 ( i ) Let a, b  R be any two numbers, then
(C) From (b) (iii) and (ii) we have see that the a ∗ b = a + b – 2ab also belongs to R, so closure holds
operation  is NOT associative as: For e.g 2 ∗ 3 = 2 + 3 – 2 × 2 × 3 = 5 – 12 ie - 7R
2 ∗ [ (- 4) ∗1 ] ≠ [ 2 ∗ (- 4)] 1 No matter the values you use, we will never arrive et
complex numbers say  2 in this operation which is outside
(d) 4  x = 28
Applying the operation definition the range of R
42 + x2 – (4)x = 28 ( ii ) Operation ∗ is commutative on a, bR if
16 + x2 – 4x = 28
a∗b=b∗a
Rearranging: x2 – 4x – 12 = 0
Factorizing x2 – 6x + 2x – 12 = 0 a ∗ b = a + b – 2ab
x( x – 6) + 2(x – 6) = 0 and b ∗ a = b + a – 2ab
( x – 6) (x + 2) = 0 We observe that they are same hence commutativity holds.
x – 6 = 0 or x + 2 = 0
x = 6 or – 2 (iii) Associativity will hold if for any a, b & c R
(a ∗ b) ∗ c = a ∗ ( b ∗ c)
1996/ 4 (Nov) First we apply the defined operation to the brackets
Let R denote the set of real numbers. A binary operation LHS (a + b – 2ab) ∗ c = a + b – 2ab + c – 2(a + b – 2ab)c
∗ is defined on the set R -  -1 by x  y = x + y + xy = a + b + c – 2ab – 2ac – 2bc + 4abc
for all x, y  R We can’t simplify further
(a) Show whether or not  is RHS a ∗ (b + c – 2bc) = a + b + c – 2bc – 2a(b + c – 2bc)
(i) Commutative (ii) Associative = a + b + c – 2bc – 2ab – 2ac + 4abc
(b) Determine its identity element They are same, hence associativity holds.
70
2003/4 (Nov) Solution
A binary operation is defined on R, the set of real The table will be of the form
numbers, by x ∗ y = xy + x + y for all x, yR  -2 -1 0 1 2
4 -2
(a) Find (i) the identity element eR -1
(ii) the inverse of an element rR 0
(b) Solve the equation x ∗ 6 = 36 1
Solution 2
a ( i ) By identity element property Applying the defined operation a  b = a + ab
x∗e = x – 2 Row
Applying the defined operation to LHS __
2 __
2 = __2 + (__ 2)(__ 2) i.e 2
xe + x + e = x __
2  __ 1 = __ 2 + (__ 2)(__ 1) i.e 0
4 __
2  0 = __ 2 + (__ 2)(0) i.e __ 2
Make e the subject formula __
2  1 = __ 2 + (__ 2)(1) i.e __ 4
xe + e = x – x
4
__
2  2 = __ 2 + (__ 2)(2) i.e __ 6
__
Simplified the LHS 1Row
__
1  2 = __ 1 + (__1)(__ 2) i.e 1
__
e  x  4  = 0
 4 
__
1  __ 1 = __ 1 + (__1)(__ 1) i.e 0
e=0
__
1  0 = __ 1 + (__1)(0) i.e __1
a ( ii ) The inverse of r is r–1
__
1  1 = __ 1 + (__1)(1) i.e __2
By condition of inverse __
1  2 = __ 1 + (__1)(2) i.e __3
r ∗ r –1 = e 0 Row
here r ∗ r–1 = 0 0  __ 2 = 0 + (0)(__ 2) i.e 0
Applying the defined operation to the LHS 0  __ 1 = 0 + (0)(__1) i.e 0
r r–1 + r + r–1 = 0 0  0 = 0 + (0)(0) i.e 0
4 0  1 = 0 + (0)(1) i.e 0
We are to make r –1 subject formula 0  2 = 0 + (0)(2) i.e 0
r r–1 + r –1 = – r 1 Row
4 1  __ 2 = 1 + 1(__ 2) i.e __ 1
r r–1 + 4r–1 = – r
4
1  __ 1 = 1 + 1(__ 1) i.e 0
r–1( r + 4) = – r 1  0 = 1 + 1(0) i.e 1
4 1  1 = 1 + 1(1) i.e 2
r–1 = – 4r 1  2 = 1 + 1(2) i.e 3
r + 4 2 Row
2  __ 2 = 2 + 2(__2) i.e __ 2
(b) x ∗ 6 = 36 applying the defined operation 2  __ 1 = 2 + 2(__1) i.e 0
x × 6 + x + 6 = 36 2  0 = 2 + 2(0) i.e 2
4 2  1 = 2 + 2(1) i.e 4
6x + x = 36 – 6
2  2 = 2 + 2(2) i.e 6
4
6x + 4x = 30 Thus our resulting table is:
4  -2 -1 0 1 2
10x = 30 × 4 -2 2 0 -2 -4 -6
x = 12 1 0 -1 -2 -3
-1
0 0 0 0 0 0
1 -1 0 1 2 3
Tabular binary operation for more than
2 -2 0 2 4 6
one property
(b) (i) Closure
2005/11 Neco (Dec)
Resultant elements of – 4, – 6, 4 and 6 are outside the set A.
Let  be a binary operation over a set A such that
Thus operation ∗ is not closed
A = -2, - 1, 0, 1, 2 and is defined by a ∗ b = a + ab
(b) (ii) Commutativity will hold if for any a, b A
(a) Design a table for this operation
(b) From the table determine whether the operation  a∗b=b∗a
over the set A is Let us try the negative and positive elements
(i) Closed (ii) commutative and (iii) associative To show whether __ 2 ∗ 1 = 1 ∗ __ 2
(C) From the table, determine the identity element Result from the table -4 ≠ -1
Only one proof is enough commutativity does not hold
71
(b) (iii) Associatively will hold if for any a, b cA To confirm :
a * (b * c) = (a * b) * c To show whether p * q = q * p
Let us try the negative and positive elements From the table s = s Thus * is commutative
To show whether __2 * (__1 * 2 ) = (__2 * __1) * 2
( a ) iii The identity element
Starting with the bracket and taking the table values
__ By identity element property
To show whether 2 * __3 = 0 * 2
__ __ p*e = p
2 * 3 is outside the table
From the table at p row p * r = p
Applying the defined operation a * b = a + ab To confirm :
__
2 * __3 = __2 + __2 (__3 ) From the table q * r = q
=4
s * r = s Thus r is identity element
Thus, __2 * __3 = 0 * 2 becomes
4 ≠ 0 ( a ) iv The inverse of s
Hence associativity does not hold The inverse of s is s–1
By condition of inverse
(c) Let the identity element be e
s * s–1 = e
By identity property
i.e s * s–1 = r since r is the identity here
a * e = a for any aA
From the table s * p = r thus p is the inverse of s
1st Row
__
2 * 0 = __2 (b) Given (n * p) * (r * s) = p, find n.
2nd Row From the table:
__
1 * 0 = __1 Starting with (r * s) = s
Thus (n * p) * (r * s) = p becomes
3rd Row is trivial
(n * p) * s = p
4th Row From the table : q * s = p
1 * 0 = 1 Hence identity element is 0
Thus (n * p) = q
2003/2 From the table : p * p = q thus n = p
A binary operation * is defined on the set
T = {p, q, r, s} and shown in the table.
* p q r s Tabular binary operation for identity property
P q s p r 1993/1 (Nov)
q s r q p A binary operation ∗ on a set S =  a, b, c, d, e  is defined
r p q r s by the entries in the table below;
S r p s q * a b c d e
(a) Determine with reasons: a e c a b d
b d e b a c
(i) Whether T is closed with respect to 
c a b c d e
(ii) Whether * is commutative
d c a d e b
(iii) The identity element
e b d e c a
(iv) The inverse of s.
(b) Given (n * p)  (r * s) = p, find n. What is the identity element of the set under the operation ?
Solution A. a B. b C. c D. d E. e
( a ) i To show that T is closed with respect to  Solution
is to observe the table result to see whether there is any Taking the first element, i.e a, we have that for identity to
element there outside {p, q, r, s} as highlighted below: hold
* p q r s a  e = a or e  a = a
P q s p r Starting from the first column
a  a = e which is not a
q s r q p
Next, second column
r p q r s
b  a = d, thus b is not identity
s r p s q
c  a = a thus C is identity (C)
Since all the resultant elements are members of T Further confirmation
thus, T is closed with respect to * First row: a  c = a
( a ) ii To show whether  is commutative is to pick Second row: b  c = b
any two elements of T and proceed as follows: Fourth row: d  c = d
To show whether r * s = s * r
From the table s = s

72
1998/21 UME 2000/22 PCE
 p q r r  0 1 2 3 4 5
p r p r p 0 0 1 2 3 4 5
q p q r s 1 1 2 3 4 5 0
r r r r r 2 2 3 4 5 0 1
s q s r q 3 3 4 5 0 1 2
The identity element with respect to the multiplication 4 4 5 0 1 2 3
shown in the table above is 5 5 0 1 2 3 4
A. P B. q C. r D. s In the table above, P  K is defined as what remains of the sum of
Solution P and K when divided by 6. What is the identity element?
Taking the first element i.e. p we have that, A. 0 B. 1 C. 4 D.5
for identity to hold Solution
p  e = p or e  p = p Taking the first element i.e. 0 we have that,
Starting with the first row for identity to hold.
p  p = r which is not p 0  e = 0 and e  0 = 0
Hence p is not the identity i.e. e ≠ p Starting with the first row
Taking the next element to multiply p. 00=0
pq=p and 0  1 = 1
Also from the last row, we confirm Also 1  0 = 1 from the 2nd row
sq=s Confirmation test
Thus, q = e identity element (B) 2 0 =2
3  0 = 3 and so on
Thus, 0 is the identity element (A)
. Tabular binary operation for inverse property
2004/19 PCE 2003/17 (Nov)
A binary operation * is defined as shown in the
 1 2 3
table. Find, the inverse of c , under the operation 
1 2 3 1
2 3 1 2 * a b c d
3 1 2 3 a d c b a
b c a d b
The table above defines a binary operation on {1, 2, 3}. c b d a c
Find its identity element. d a b c d
A. 0 B. 1 C. 2 D. 3
Solution A. a B. b C. c D. d
Taking the first element i.e. 1, Solution
we have that for identity to hold The inverse of c is c–1
1  e = 1 and e  1 = 1 And c ∗ c–1 = e identity
Starting with first row Thus we first of all find identity e for  on the c row
1  1 = 2, thus, 1 is not identity for identity c ∗ e = c From the table we notice that
Next element 2 c ∗d =c
1  2 = 3, thus, 2 is not identity To confirm whether d is truly the identity on a row
Next element 3 a∗d=a
1  3 = 1, thus, 3 is identity on the b row b  d = b confirmed.
also, 3  1 = 1 confirms 3 as identity Next, on C row where can we find
Thus, 3 is the identity under this table of binary c ∗ c–1 = d (identity)
operation (D) c ∗ b = d hence b is c–1 (B)
further investigation confirm it
1994/23 UME
23=2
33=3 Mod 10 2 4 6 8
2 4 8 2 6
4 8 6 4 2
6 2 4 6 8
8 6 2 8 4
The multiplication table above has modulo 10 on the set
S = { 2, 4, 6, 8}. Find the inverse of 2.
A. 2 B. 4 C. 6 D. 8
73
Solution Applying the defined operation to the LHS
2 × inverse of 2 = e ( identity) 2a + 3a–1 – 5 = 5 – a
First, we have to find the identity element on the Row of 2. 3
But 2 × e( identity) = 2 We make a–1 subject formula
2 × 6 = 2 ( Row of 2 in the table) 3a–1 = 5 – a – 2a + 5
hence 6 is the identity 3
Next, we are to find the inverse element on the Row of 2. Applying basic fraction operation to the RHS
2 × inverse of 2 = e (identity) 3a–1 = 5 – a – 6a + 15
2 × inverse of 2 = 6 3
From the row of 2. 3a–1 = 20 – 7a
2×8=6 3
Hence, the inverse of 2 = 8 (D) To get a–1 multiply both sides by 1/3
a–1 = 20 – 7a (B)
2002/22 PCE 9
* 1 2 3 4 2010/2 Neco
1 1 2 3 4 The operation  is defined over the set R, of real numbers by
2 2 4 1 3 x  y = 2xy + 3x – 6y . Find :
3 3 1 4 2 ( a ) the inverse of the operation
4 4 3 2 1 ( b ) 9  11
In the table above, find the inverse of 4 Solution
A. 1 B. 2 C. 3 D.4 We are to find inverse with the aid of identity element
Solution Let e be the identity element
4 × inverse of 4 = e (identity) Then by identity property
First we have to find the identity element on the Row of 4. x  e = x ------ (1)
But 4 × e(identity) = 4 Applying the defined operation to (1)
4×1 = 4
Hence the identity is 1
x  e = 2xe + 3x – 6e --------(2)
Next, we are to find the inverse element on the Row of 4. For an identity element (2) = (1)
4 × inverse of 4 = e i.e 2xe + 3x – 6e = x
4 × inverse of 4 = 1 We make e subject formula
From the row of 4 2xe – 6e = x – 3x
4×4=1 e(2x – 6) = –2x
Hence the inverse of 4 = 4 (D) e = –2x or –x
2x – 6 x–3
Non-tabular binary operation for Next is the inverse , the inverse of x is x– 1
inverse property By condition of inverse
2010/8 Neco x  x–1 = e
The binary operation ∗ on the set of R of all real i.e x  x–1 = –x
numbers is defined as a ∗ b = 2a + 3b – 5. Find the x–3
inverse element in ∗ Applying the defined operation to the LHS
A 20 – 7a B 20 – 7a C – 20 + 7a 2xx–1 + 3x – 6x–1 = –x
9 9 x–3
D –a+5 E – 3a + 10 We make x–1 subject formula
3 9 2xx–1 – 6x–1 = – x – 3x
Solution x–3
We are to find inverse with the aid of identity element Factor out x –1 at the LHS and simplify RHS
Let e be the identity element x–1(2x – 6) = – x – 3x(x – 3 )
Then by identity property x–3
a ∗ e = a -------- (1) x–1(2x – 6) = –x –3x2 + 9x
Applying the defined operation to (1) x–3
a ∗ e = 2a + 3e – 5 ------ (2) x–1(2x – 6) = 8x –3x2
For an identity element (2) = (1) x–3
i.e 2a + 3e – 5 = a To get x-1 multiply through by 1
We make e subject formula 2x – 6
3e = a – 2a + 5 x–1 = 8x –3x2
e= 5–a (x – 3) (2x – 6)
3 (b) 9  11 = 2 × 9 × 11 + 3 × 9 – 6 × 11
The inverse of a is a– 1 and by inverse condition = 198 + 27 – 66
a ∗ a–1 = e = 159
i.e a ∗ a–1 = 5 – a
3
74
1996/11 (Nov) Counter example Solution
A binary operation ∗ with identity element zero is Let e be the identity element
defined on the set R of real numbers by Then by identity element property
m ∗ n = m + n + 2mn, m, n R . x ∗ e = x _________ (1)
For what value of m does the operation have no inverse? Applying the defined operation to (1)
A – 1/ 2 B 1/2 C 0 D 1 E 2 x ∗ e = 5xe _________ (2)
Solution For an identity element (2) = (1)
First we find the inverse of the operation  with the aid of 5xe = x
identity element: x 1
Let e be the identity element: Then by identity property Next, we make e subject formula e = i.e (C)
5x 5
m∗e=m ___________
(1)
2000/8 (Nov)
Applying the defined operation to (1)
A binary operation  defined on R, the set of real numbers is given
m ∗ e = m + e + 2me ___________ (2)
by x ∗ y = x + y – 2 for all x, yR. Find its identity element.
For an identity element (2) = (1)
1 1
m + e + 2me = m A–2 B  C D2
We make e subject formula 2 2
e + 2me = m – m Solution
e (1 + 2m) = 0 Let the identity element be e.
e = 0 or 1 + 2m = 0 Then by identity element property
Thus identity element e = 0 x ∗ e = x _________ (1)
The inverse of m is m–1 Applying the defined operation to (1)
By condition of inverse m  m–1 = e x + e – 2 = e __________
(2)
i.e m  m–1 = 0 __________
(1) For any identity element (2) = (1)
Applying the defined operation to (1) x + e – 2 = x
m + m–1 + 2mm–1 = 0 Next, we make e subject formula
–1
Make m subject formula e = x – x + 2
m–1 + 2mm–1 = __ m e = 2 (D)
m–1(1 + 2m) = __ m
m–1 = __ m Non-tabular binary operation for
(1 + 2m) commutative property
For there to be no inverse RHS must be undefined 2000/3
i.e 1 + 2m = 0 (denominator equals zero) A binary operation ∗ is defined over R (the set real numbers)
2m = __1 Thus, m = __ 1/2 (A) by x ∗ y = xy + x2 + y2 for all x, yR .
(a) Determine whether or not ∗ is commutative
Non-tabular binary operation for (b) If x ∗ (x + 2) = 49, find x.
identity property Solution
1995/17 (Nov) (a) Commutativity holds if
A binary operation ∗ is defined on the set R of real numbers x ∗ y = y ∗x
by x  y = x + y – xy , where x, y belong to R xy + x2 + y2 = yx + y2 + x2
Find, under the operation , the identity element The results are same, hence commutativity holds
A1 B0 C 1y D 1x E2 (Pick any real number, say 3 and 4 )
Solution 3 × 4 + 3 2 + 4 2 = 4 × 3 + 4 2 + 3 2)
Properties of identity show that (b) x ∗ (x + 2) = 49
Either x ∗ e = x or e ∗ x = x for any xR Applying the defined operation
Using x ∗ e = x __________ (1) x(x + 2) + x2 + (x + 2)2 = 49
Next we apply the defined operation to (1) x + 2x + x2 + x2 + 4x + 4 = 49
2
x ∗ e = x + e – xe _________ (2) 3x2 + 6x – 45 = 0
For an identity element (2) = (1) Factorizing
x + e – xe = x 3x2 + 15x – 9x – 45 = 0
Next we make e the subject formula 3x(x + 5) – 9(x + 5) = 0
e – xe = x – x (x + 5)(3x – 9) = 0
e(1– x) = 0 x + 5 = 0 or 3x – 9 = 0
e = 0 or 1 – x = 0 x = – 5 or 3
Thus identity element e = 0 (B)
1997/14 2000/14
If a binary operation ∗ is defined on the set Q of rational Which of the following binary operations is not
numbers by x ∗ y = 5xy, For all x, y  Q, find its commutative?
identity element. A a ∗ b = a + b – 2ab B a ∗ b = 3a + 3b – ab
A_5 B _ 1 C 1 D 5 E 1 C a∗b= 2  2 D a ∗ b = a – b + 2ab
5 5 5x a b

75
Solution 2001/33 Neco
Option A A binary operation ∗ is defined on the set of real numbers
Commutative if a ∗ b = b ∗ a ( R ) by
a + b – 2ab = b + a – 2ba same x∗y= x+y – x–y
Option B where x, y R, the value of 6  2 is
Commutative if a ∗ b = b ∗ a A 8 8 B 10 C 8 – 2 D 6 E 2
3a + 3b – ab = 3b + 3a – ba same
Solution
Option C Applying the defined operation
Commutative if a ∗ b = b ∗ a
6 2 = 6 +2 – 6–2
2 2 2 2
+ = + Same
a b b a = 8 – 4
Option D
Commutative if a ∗ b = b ∗ a = 8–2
a – b + 2ab ≠ b – a + 2ba
Reason : a – b ≠ b – a for any a,b  R = 6 (D)
e.g 2 – 3 ≠ 3 – 2 option D not commutative
1994/9
A binary operation ∗ on the set of rational number is defined
by
a ∗ b = b  a , find – 3 ∗ 2
2 2
2005/1 (Nov)
Two binary operations Δ and  are defined on the set R 2ab
real numbers by; 13 5 7 5  13
A B C D E
x Δ y = 2(x + y) + 3xy 12 12 12 12 12
x ∗ y = 2x + y Solution
Applying the defined operation
Find: (a) x Δ (y  z) (b) (x Δ y)(x Δ z)
–3 ∗ 2 = 22 – (–3)2
Solution
2 (–3) × 2
(a) Applying the defined operation ∗ to the bracket
= 4–9
x Δ (y ∗ z) = x Δ (2y + z)
– 12
Next, we apply the defined operation Δ
= 5 (B)
x Δ (2y + z) = 2(x + 2y + z) + 3x(2y + z)
12
= 2x + 4y + 2z + 6xy + 3xz
(b) Applying the defined operation Δ to the brackets.
1996/8
(x Δ y) ∗ (x Δ z) = [2(x + y) + 3xy] ∗ [2(x + z) + 3xz]
A binary operation ∗ defined on the set R of real numbers by
= (2x + 2y + 3xy) ∗ (2x + 2z + 3xz)
a ∗ b = a  b , where a and b belong to R
Next, we apply the defined operation  b a
= 2(2x + 2y + 3xy) + (2x + 2z + 3xz) Simplify (x + 1)  2 = 3
= 4x + 4y + 6xy + 2x + 2z + 3xz A x2 + 2 x + 2 = 0 B x2 – 4x + 1 = 0 C x2 - 4x -1 = 0
= 6x + 4y + 2z + 6xy + 3xz D x2 + 2x + 5 = 0 E x2 – 2x – 5 = 0
Solution
Defined binary operation problems Applying the defined operation
2006 / 4 (x + 1) ∗ 2 = 3 implies
A binary operation ∗ is defined on the set R, of real x+1 + 2 = 3
ab 2 x+1
numbers by a ∗ b = . Find the value of 2 ∗ 6
4 Applying basic LCM operation to the LHS
(x + 1)(x + 1) + 2 × 2 = 3
A 3 B 3 2 C 3 D 2 2 (x + 1)
4 2 2 x2 + 2x + 1 + 4 = 3 [2(x +1)]
Solution x2 + 2x + 5 = 6x + 6
Applying the defined operation x + 2x – 6x + 5 – 6 = 0
2

x2 – 4x – 1 = 0 (C)
2 ∗ 6 = 2 × 6
4 1999/5 (Nov)
A binary operation ∗ is defined by x ∗ y = x2 – y2 – y
= 12
4 for all x, y R, the set of real numbers. Find the possible
values of m for which 3 ∗ m = 7
= 2 3 = 3 (C) A 2 or – 1 B 1 or – 2 C 1 or 2 D –1 or –2
4 2
76
Solution collect like terms together
3∗m=7 x2 – x + y – y = 0
Applying the defined operation to the LHS x2 – x = 0
32 – m2 – m = 7 x(x – 1) = 0
9 – m2 – m = 7 x = 0 or 1 (A)
 m2 + m – 9 + 7 = 0
m2 + m – 2 = 0 1999/20 PCE Exercise 7.0
Factorizing  1 2 3 4
m2 + 2m – m –2 = 0 1 1 2 3 4
m(m + 2) – 1(m + 2) = 0
2 2 4 1 3
(m + 2)(m – 1) = 0
3 3 1 4 2
m + 2 = 0 or m – 1 = 0
4 4 3 2 1
m = – 2 or 1 (B)
In the table above, a * b = ab mod 5 that is, a * b is defined
2002/4
as what remains of the product of a and b when divided by 5.
A binary operation  defined on R, the set of real find the identity element.
numbers A. 4 B. 3 C. 2 D. 1
by x ∗ y = xy for all x, yR. If x ∗ (2 ∗ 8) = 6, find
x 2005/40 PCE Exercise 7.1
A 2 B 4 C 9 D 10  k l m n
Solution k l n k m
x ∗ (2 ∗ 8) = 6 l m n l k
Applying the defined operation to the bracket m k l m n
n l k n m
2∗8 = 2×8
In the above table,  is the operation defined on
= 16 i.e 4 S = { k, l, m, n }. Find the identity element of the set with
Next, x ∗ (2 ∗ 8) = 6 becomes respect to 
x ∗ 4 = 6 A. l B. k C. m D. n
Applying the defined operation
2003/33 PCE Exercise 7.2
x ×4 = 6
 p q r S
To remove square root; take square of both sides
P q s p q
4x = 36
q p r q S
x = 9 (C)
r p q r s
2002/18 (Nov) s q p s r
A binary operation ∗ defined on the set R of real If  is the operation defined on the set M = {p, q, r, s}
numbers is given by m  n = m + n2, m, nR . as shown on the table above, find the identity element of the
If K ∗ 3 = 7 ∗ 4, KR, Find the value of K. set with respect to 
28 A. S B. r C. q D. P
A8 B C 14 D 20
3
Solution 2001/17 UME Exercise 7.3
K 3 =7 ∗ 4  K L M
Applying the defined operation to both sides K L M K
K + 3 2 = 7 + 42 L M K L
K + 9 = 7 + 16 M K L M
K = 23 – 9
= 14 (C) The identity element with respect to the multiplication shown
in the table above is
2009/3 (Nov) A. K B. L C. M D. O
Two binary operation  and ∗ are defined on the set of
real numbers R, such that
x  y = x2 + y and x ∗ y = x + y, x , yR 2002/1 Neco Exercise 7.4
If x  y = x  y, what are the possible values of x ? Find the identity element (e) in the binary operation  on the
A 0,1 B 0, 2 C 1, 1 D 1, 2 set of real numbers defined by
Solution x ∗ y = 3x + 3y – 1
Applying their different definitions 3
x  y = x ∗ y becomes A1 B 3/ 4 C 2/ 3 D 1/2 E 1/ 3
x2 + y = x + y
77
2002/2 Neco Exercise 7.5 2005 / 1 Neco (Dec) Exercise 7.13
Calculate the inverse function a1 in the binary operation The operation ⨀ is defined over the set of real numbers by
ab x  y = x + y + x2y.
∇such that for all a, b R a ∇ b = ( i ) is  commutative ?
5
25  25 a a a ( ii ) Find the identity element e under ⨀
A B C D E (iii) Given a real numbers a, find a* such that a ⨀ a = e
a a 25 5 5
2004/31 Exercise 7.14
2002/3b Neco Exercise 7.6 Which of the following binary operations is not
The operation ⨀ is defined on the set of real numbers commutative?
1 A a ∗ b = a + b – ab B a ∗ b = 3a + 3b – ab
R by a ⨀ b = a + b __ ab for all a, bR. Find the
2 1 1
( i ) Identity element Ca∗b= + D a ∗ b = b – a + ba
a b
( ii ) Inverse element.
2004/33 Exercise 7.15
2003/40 Neco Exercise 7.7 A binary operation is defined by a ∗ b = a2 – b2 + ab, where a
A binary operation ∆ is defined by x ∆ y = xy .
If x ∆ 2 = 2 – x, find the possible values of x. and b are real numbers. Evaluate 3 ∗ 2
A 2, 2 B 1, 2 C 2, _ 2 D 1, __1 E 1, __2
A 6 __ 1 B 1+ 6 C 1 __ 6 D 6
2004/36 Neco (Dec) Exercise 7.8
Find the identity element under the binary operation  2005/4 Exercise 7.16
defined by c  d = c + d + 2cd, for all c, dR The binary operation ∗ is defined on the set (R) of real
A 1 + 2c B 1 + 2d C c+d D 0 E 1 numbers by a ∗ b = a + b – 2, a, b R.
Find the identity element under ∗
2004/41 Neco (Dec) Exercise 7.9 A4–a B 0 C 2 Da–2
a c a  c
In (Q+, ), ∗ is defined as ∗ = , 2005/10a Exercise 7.17
b d bd
A binary operation ∗ is defined on the set (R) of real numbers
1
3 1
15 by x ∗ y = + x + y, where x, y R
Calculate 2 ∗ 2
5 16 (i) Find the identity element of the operation
A 13 B 37 C 37 D 27 E 13 (ii) Determine the inverse of 2 under ∗
160 80 160 42 37

2004/14 Neco Exercise 7.10 2007/ 27 Neco Exercise 7.18


Find the identity element e under this operation, if the A binary operation ∗ is defined on the set of
binary operation  is defined by a 2  b2
integers (z) by a ∗ b = , for all a, bZ.
c ∗ d = 2cd + 4c + 3d for any real number c and d ab
A  3c B – 3c C – 2c + 3 D 3c E 2c + 3 Find the value of x if 3 ∗ x = 23
2c  3 2c  3
A 26 B 23 C 20 D 17 E 3

2005 / 3 Neco Exercise 7.11 1997/20 Exercise 7.19


The operation  defined for any two real numbers Which of the following subsets of N, the set of natural
1 numbers, is/are closed under the operation of multiplication?
a and b by a  b = a + b + ab is
2 I P =  x : x is prime
A Associative B Both associative and commutative II Q =  x : x = 2n 
C Commutative D Distributive III M=  x : x is even
E Neither associative nor commutative. A I only B 1 and III only C II and III only D III only E II only.

1999/4 (Nov) Exercise 7.20


2005 / 9 Neco Exercise 7.12
A binary operation ∗ is defined on R, the set of real numbers
The operation ⨀ on the set R of real is defined by:
4x  2 y  2 by a ∗ b = a + b – ab, where a bR
x⨀y= for all x, yR. (a) Determine whether or not ∗ is
4 (i) Closed;
The identity element in R under the operation ⨀ is (ii) Commutative
A1 B 1/2 C0 D–1 E–2 (iii) Associative; on the set R
(b) Find the identity element

78
Chapter eight Then Tn = 1 + ( n – 1 ) 1
AP, GP & Sequence =1+n–1
=n
Sequence(AP & GP ) and series. As for ( 2 )
When numbers appear or are presented one after the T1 = 12
other in an orderly manner, we then refer to the set of T2 = 19 = 12 + ( 2 – 1 ) 7
numbers as sequence. = 12 + 7
Examples; T3 = 26 = 12 + ( 3 – 1 ) 7
1, 2, 3, 4,… = 12 + 14
5, 10, 20, 40,…
-3, -1, 1, 3,… T4 = 33 = 12 + ( 4 – 1 ) 7
= 12 + 21
Series is the summation (addition) of the terms of
a sequence. For example, the series equivalent of Then Tn = 12 + ( n – 1 ) 7
the above sequence are; = 12 + 7n – 7
1 + 2 + 3 + 4 +… = 5 + 7n
5 + 10 + 20 + 40 +… Generally if we are given any A.P: a, a + d, a + 2d,… Then
( - 3 ) + ( - 1 ) + 1 + 3 +…
Tn = a + ( n – 1 )d
Arithmetic Progression ( AP ) This is the formula for finding the nth term of an AP
Arithmetic progression is a form of sequence in Examples:
which each term is gotten by the addition of a 1. Find the 4th term of an AP whose first term is 2 and the
common difference to the preceding one. Alternatively, common difference is 0.5 ( WAEC )
we can say that APs or linear sequence are sequence A. 0.5 B. 2.5 C. 3.5 D. 4
that follows simple addition rule. Solution
Examples; let us examine the pattern of the APs below; Tn = a + ( n – 1 )d
( 1) 1, 2, 3, 4, 5,… Here n = 4, a = 2, d = 0.5
We observe that; Substituting, we have
Items pattern T4 = 2 + ( 4 – 1 ) 0.5
1 = 2 + ( 3 ) 0.5
2 =1+1 = 2 + 1.5
3 =2+1 = 3.5 ( C )
4 =3+1
5 =4+1 ( 2 ) Find the 9th term of the Arithmetic progression,
Conclusion; 1 is the common difference and first term is 1. 18, 12, 6, 0, – 6... ( WAEC ORDINARY MATHS )
A – 54 B – 30 C 30 D 42
( 2 ) 12, 19, 26, 33,… Solution
We observe that Students need to be careful when dealing with numbers such as the
Terms pattern ones above. Since the common difference is negative. It is at times
12 like this, that you show your ability to stick to the rule in AP
19 = 12 + 7 For any AP, a, a + d, a + 2d
26 = 19 + 7 Common difference d = ( a + d ) – a or
33 = 26 + 7 = ( a + 2d ) – ( a + d )
Conclusion; 7 is the common difference and first term is 12 In words: it must be 2nd term minus 1st term or
3rd term minus 2nd term, no matter the numbers given.
nth term of an AP Tn = a + ( n – 1 )d
Conventionally, we denote the first term by ‘a’ and the Here a = 18, d = 12 – 18 = – 6 or d = 6 – 12 i.e – 6
common difference by ‘d’. Let n represent the nth term, Substituting, we have
we observe that the examples given above follow a T9 = 18 + ( 9 – 1 ) ( – 6 )
simple pattern: = 18 + 8 ( – 6 )
a, a + d, a + 2d, a + 3d = 18 – 48
1st 2nd 3rd 4th = – 30 ( B )
For ( 1 ) 2006/1
T1 = 1 The sum of the 2nd and 5 terms of an arithmetic progression
T2 = 2 = 1 + ( 2 – 1 ) 1 (A.P) is 42; if the difference between the 6th and 3rd terms is
=1+1 12, find the;
(a) Common difference; (b) First term; ( c ) 20th term
T3 = 3 = 1 + ( 3 – 1 ) 1 Solution
=1+2 First statement: T2 + T5 = 42
T4 = 4 = 1 + ( 4 – 1 ) 1 i.e (a + d) + (a + 4d) = 42
=1+3 2a + 5d = 42……..(1)

79
Second statement: T6 – T3 = 12 Here n = 6, a = 1, T6 = 11, d = ?
(a + 5d) – (a + 2d) = 12 Substituting
a + 5d – a –2d = 12 T6  11 = 1 + ( 6 – 1 )d
3d = 12 11 = 1 + 5d.
d = 12/3 i.e 4 11 – 1 = 5d
(b) Substitute d = 4 into (1) 10 = 5d
2a + 5(4) = 42 d = 10 i.e 2 ( D )
2a + 20 = 42 5
2a = 42 – 20 2001/19 UME
2a = 22 The sixth term of an arithmetic progression is half of its
a = 22/2 i.e 11 twelfth term. The first term is equal to
(c) Tn = a + (n – 1)d A. zero B .half of the common difference
T20 = 11 + (20 – 1)4 C. double the common difference D. the common difference
= 11 + 19(4) Solution
= 11 + 76 i.e 87 The question is simply
T6 = ½ T12 ; find a
1999/18 (Nov)
The coefficient of the fifth, sixth and seventh terms in Apply the formula Tn = a + ( n – 1 )d
the expansion of (1 + x)7, in ascending powers of x, T6 = a + ( 6 – 1 )d
form a linear sequence (A.P). Find the common difference . = a + 5d
A. –14 B. 3 C. 6 D. 14 Similarly,
Solution T12 = a + 11d
The general term in the expansion of (a + x )n But T6 = ½ T12
is Tr +1 = nC r a n – r x r i.e a + 5d = ½ ( a + 11d )
Here Tr + 1 = 7C r 17 – r x r 2 ( a + 5d ) = a + 11d
T 5 by Tr + 1 is r = 4 i.e. the coefficient of x4 2a + 10d = a + 11d
Collect like terms together
T 6 by Tr + 1 is r = 5 i.e. the coefficient of x5 2a – a = 11d – 10d
a=d
T 7 by Tr + 1 is r = 6 i.e. the coefficient of x6 i.e first term equal to the common difference ( D ).
Thus, T = 7C4 1 7 – 4 i.e 35 2002/29 UME
T = 7C5 17 – 5 i.e 21 If the 9th term of an A.P is five times the 5th term,
T = 7C6 1 7 – 6 i.e 7 Find the relationship between a and d.
In ascending powers of x, the A.P is 35, 21, 7, … A.3a + 5d = 0 B. a + 2d = 0 C. 2a + d = 0 D. a + 3d = 0
By A.P formula for common difference d Solution
d = 21 – 35 or 7 – 21 The question is simply
= –14 T9 = 5 × T5, find the relationship b/w a & d
Applying the formula Tn = a + ( n – 1 )d
1995/21 PCE T9 = a + ( 9 – 1 )d
Given that 443 is the nth term of the arithmetic = a + 8d
progression 5 + 8 + 11 +…, find n Similarly,
A. 89 B. 144 C. 148 D. 441 T5 = a + 4d
Solution But T9 = 5 × T5
Tn = a + ( n – 1 )d  Thus a + 8d = 5 (a + 4d )
Here a is 5; d is 3, Tn = 443 a + 8d = 5a + 20d
443 = 5 + ( n – 1 )3 Collect like terms together
443 = 5 + 3n – 3 a – 5a = 20d – 8d
443 = 2 + 3n –4a = 12d
443 – 2 = 3n – a = 3d
441 = 3n i.e. 3d + a = 0 ( D )
441 = 3n 147 = n ( B ) 2007/20 PCE Exercise 8.1
3 3 The 4th term of an A.P is 8 while the common difference
1994/19 UME is – 2 . Find the 15th term of the progression.
If the 6th term of an arithmetic progression is 11 and the A. 32 B. 28 C. –14 D. –32
first term is 1, find the common difference 2009/22 Neco Exercise 8.2
A. 12 B. 5 C. – 2 D. 2 The 12th term of an AP is four times 4th term. What is the
5 3 relationship between the first term (a) and the common
Solution difference (d)?
Apply the formula A. 5a – 23d = 0 B. d – 3a = 0 C. d + 3a = 0
Tn = a + ( n – 1 )d D. 4d – 3a = 0 E. 23d + 5a = 0
80
Unknown a and d resulting to Solution
simultaneous linear equations The mere fact that sum is mentioned here does not mean
application of the summation formula. So always watch out
There are cases in AP where two terms of a progression are for examiner’s tricks
given or sometimes a third term having a relationship with T8 + T9 = 72 and T4 = – 6, find d
any of the two given or any other given situation that may Applying the formula Tn = a + ( n – 1 )d
have same form. Then we are asked to find the first term ‘a’ T8 = a + ( 8 – 1 )d
or common difference ‘d’ or both or use the result. The
resulting problem leads us to simultaneous linear equations;
= a + 7d
which can be solved either by: Similarly,
Elimination method or T9 = a + 8d
Substitution method But T8 + T9 = 72
of solving our usual simultaneous linear equations i.e. a + 7d + a + 8d = 72
2a + 15d = 72 ……… ( 1 )
1993/4 (Nov) Also T4 = – 6
The third term of a linear sequence (A.P) is 16 and its i.e. a + 3d = – 6 ………………… ( 2 )
6th term is 34. Find the second term. Now, since we are asked to find d then, we eliminate a. Thus
A. 12 B. 10 C. 8 D. 6 E. 5 ( 2 ) × 2 and subtract
Solution 2a + 15d = 72
The 2nd term can be gotten, if we know a and d of – ( 2a + 6d = – 12 )
the term A.P.
In A.P, Tn = a + (n – 1)d 9d = 84
T3  16 = a + 2d…………….(1) d = 84
9
T6  34 = a + 5d …………… (2)
= 9 3/9 i.e 9 1/3 ( D )
Solving both equations simultaneously
34 = a + 5d 1995/17 UME
– (16 = a + 2d ) The 4th term of an AP is 13 while the 10th term is 31.
18 = 3d Find the 21st term
18
/3 = d Thus d = 6 A. 175 B. 85 C. 64 D. 45
Substitute d value into (1) Solution
16 = a + 2(6) T4 = 13, T10 = 31, find T21
16 = a + 12 Applying the formula
16 – 12 = a ie a = 4 T4  13 = a + ( 4 – 1 )d
Thus T2 = a + d 13 = a + 3d …………( 1 )
= 4 + 6 i.e 10 (B) Also for T10, 31 = a + 9d …………( 2 )
2000/12 Neco Here, we will find a and d to enable us find T21
The 4th and 13th terms of an A.P are 3 and 39 Rearranging and subtracting
respectively, find the nth term of the progression. a + 9d = 31
A. 2n – 4 B. 2n – 1 C. 4n – 13 – ( a + 3d = 13 )
D. 4n + 13 E. 4n + 17
Solution 6d = 18
For any A.P Tn = a + (n – 1)d d = 18 i.e 3
T4  3 = a + 3d 6
T13 39 = a + 12d Substituting d value into ( 1 )
Solving the two equations simultaneously 13 = a + 3d will become
39 = a + 12d 13 = a + 3( 3 )
– ( 3 = a + 3d ) 13 = a + 9
a = 13 – 9
36 = 9d a= 4
Thus d = 36/9 i.e d = 4 Now, T21 = 4 + ( 21 – 1 )3
Substituting d value into 3 = a + 3d = 4 + ( 20 )3
3 = a + 3(4) = 4 + 60
3 = a + 12 = 64 ( C )
3 – 12 = a ie a = – 9
Tn = – 9 + (n – 1) 4 2000/13 UME
= – 9 + 4n – 4 The 3rd term of an AP is 4x – 2y and the 9th term is 10x – 8y.
= 4n – 13 (C) Find the common difference.
1988/28 UME A.19x – 17y B. 8x – 4y C. x – y D. 2x
If the sum of 8th and 9th terms of an arithmetic Solution
progression is 72 and the 4th term is – 6, find the It is said ‘forget about what is written on vehicle’s body and
common difference. enter inside’. It is true here. Always stick to the principle of
A.4 B.8 C.6 2/3 D. 9 1/3 solving any problem. Do not mind examiner’s tricks.
81
2p – 10 – ( p + 1 ) = 1 – 4p2 – ( 2p – 10 )
T3  a + ( 3 – 1 )d = 4x – 2y 2p – 10 – p – 1 = 1 – 4p2 – 2p + 10.
i.e a + 2d = 4x – 2y ……….( 1 ) Arranging the right side to fit quadratic form
Also for T9; a + 8d = 10x – 8y …………( 2 ) 2p – p – 11 = 11 – 2p – 4p2
We will seek to eliminate a, since d is our target, p – 11 = 11 – 2p – 4p2
Rearranging and subtracting, we have Rearranging the whole terms to fit quadratic form,
a + 8d = 10x – 8y p – 11 – 11 + 2p + 4p2 = 0
– ( a + 2d = 4x – 2y ) 4p2 + 3p – 22 = 0
6d = 6x – 6y Factorizing, we have
d = 6x – 6y 4p2 – 8p + 11p – 22 = 0
6 4p ( p – 2 ) + 11 ( p – 2 ) = 0
d = 6x – 6y ( p – 2 ) ( 4p + 11 ) = 0
6 6 p – 2 = 0 or 4p + 11 = 0
p = 2 or – 11 (C)
d=x–y (C) 4
2002/18 PCE
1997/21 PCE Exercise 8.3 If 3, x, y, 24 are in A.P., find x + y.
The 6th term of an AP is 20 and the 11th term is – 5. A.3 B.10 C.17 D.27
find the sum of the first three terms. Solution
A.45 B.85 C.120 D.150 The idea of common difference ‘d’ comes into play here.
x – 3 = 24 – y
1992/20 PCE Exercise 8.4 x + y = 24 + 3
The third term of an arithmetical progression is – 5. x + y = 27 D.
If the fifth term is 4.5, find the eleventh term.
A.19 B.31 C.77 D.33 1994/24 PCE Exercise 8.5
In a family, the ages of children arranged from the youngest
to the oldest are 2, 6, y, 14 and z. If the sequence of the ages
Missing terms is in an arithmetic progression what is the ratio z : y ?
1989/30 UME A. 9 : 4 B. 4 : 9 C. 9 : 5 D. 5 : 9
If – 8, m, n, 19 are in arithmetic progression, find ( m, n ).
A. 1, 10 B. 2, 10 C. 3, 13 D. 4, 16 1993/23 PCE Exercise 8.6
Solution Find the first term of the sequence x + 1, 2x – 18, 2x – 1,
The idea of common difference ‘d’ will come into play here. if this sequence is an arithmetic progression.
m – ( – 8 ) = 19 – n A. 37 B.12 C.10 D.5
m + 8 = 19 – n
m + n = 19 – 8
m + n = 11 ……………( 1 ) Term’s formula for a particular AP
Also, m – ( – 8 ) = n – m
m+8=n–m It has been noted earlier that:
m+m–n= –8 Tn = a + ( n – 1 )d
2m – n = – 8 ………….( 2 ) This is the general formula for finding the nth term of any
Adding ( 1 ) and ( 2 ) A.P. But, every AP has it’s own particular term’s formula
m + n = 11 which is a short cut to deriving other members of the
+ ( 2m – n = – 8 ) progression
3m = 3 Examples:
m = 3/3 i.e 1 ( 1 ) Find the nth term Un of the A.P. 11, 4, –3,...
Substituting m value into ( 1 ) A.Un = 19 + 7n B.Un = 19 – 7n C.Un = 18 – 7n
m + n = 11 will become D. Un = 18 + 7n ( WAEC ORDINARY MATHS )
1 + n = 11 Solution
n = 11 – 1 Applying the general formula
n = 10 Thus ( m, n ) = 1, 10 (A) Tn = a + ( n – 1 )d
Here Tn = Un, a = 11, d = 4 – 11 i.e –7
1998/19 UME Substituting,
If P + 1, 2P – 10, 1 – 4P2 are three consecutive terms of Un = 11 + ( n – 1 ) ( –7 )
an arithmetic progression, find the possible values of P. Ửn = 11 – 7n + 7
A.- 4, 2 B.- 3, 4 C. – 11 , 2 D. 5, -3 = 18 – 7n ( C )
11 4
Solution ( 2 ) Find the nth term of the sequence 4, 10, 16,...
Here, our firm knowledge of common difference is A. 2( 3n –1 ) B. 2( 2 + 3 n – 1 ) C.2n + 2
required D. 2( 3n + 2 ) ( WAEC ORDINARY MATHS)

82
Solution Solution
It is obvious, that the sequence is AP (a) A.P first and last term sum formula is
Applying the formula ( General ) Sn = n (a + L)
Tn = a + ( n – 1 )d 2
a = 4, d = 10 – 4 i.e 6 196 = n (– 12 + 40)
Tn = 4 + ( n – 1)6 2
= 4 + 6n – 6 196 × 2 = n(28)
T n = 6n – 2 196 × 2 = n
= 2( 3n – 1 ) ( A ) 28
14 = n
2004/34 UME (b) To find d, we apply the general sum formula for AP
The numbers – 1/2, 11/2 , 31/2, 51/2, … form a linear Sn = n [ 2a + (n – 1)d ]
sequence (A.P). Find its nth term. 2
A. 1/2(4n – 3) B. 1/2(3 – 4n) C. 1/2(4n – 5) S14  196 = 14 [ 2(–12) + (14 – 1)d ]
D. /2(5 – 4n)
1
2
Solution 196 = 7 (– 24 + 13d)
For any A.P, Tn = a + (n – 1)d 196
A is – 1/2 and d is 11/2 – (– 1/2) i.e 2 /7 = – 24 + 13d
28 + 24 = 13d
Thus; Tn = – 1/2 + (n – 1)2 52
/13 = d
= 2n – 2 – 1/2 4=d
= 2n – 5/2 (c) Tn = a + (n – 1)d
= 1/2(4n – 5) (C) T12 = –12 + (12 – 1) 4
= –12 + (11)4
1993/25 PCE Exercise 8.7 = –12 + 44
Given the progression 5,8,11,14…. = 32
Find the expression for the nth term of the progression
2004/35 (Dec ) Neco
A. n( n + 1 ) B. 5 + 3n C. 3n + 2 D. 5 + 2n
Find the sum of the first thirty odd integers
A. 400 B. 469 C. 480 D. 900
Sum of an A.P Solution
The sum of terms of an AP refers to the addition of all Odd numbers are numbers that two cannot divide i.e opposite
the terms mentioned or required in the given AP. It is of even numbers: they follow a simple A.P
usually denoted by Sn where n is the number of terms 1, 3, 5, 7, 9,…
in the progression. For another day, their Tn = 1 + (n – 1)2
Given any AP, where Tn = 2n – 1
T1 = a first term n
Sn = [2a + (n – 1)d]
T2 = a + d ( 2nd term ) 2
T3 = a + 2d ( 3rd term ) 30
Tn = a + ( n – 1 )d (nth term ) S30 = /2 [2(1) + (30 – 1) (2)]
Then the sum Sn from the 1st term to the nth term = 15 (2 + 58)
Sn = a + (a + d) + (a + 2d) +... + [ a + ( n – 1)d] …….1 = 15 (60)
Also the sum Sn from the nth term to the first term = 900 (D)
Sn = [a +(n –1)d ] + [ a + ( n –2)d ] +…+ (a + d) + a …..2 1992/20 UME
Adding 1 and 2 termly as they occur For an arithmetical sequence, first term is 2 and the common
2Sn = [2a + (n – 1)d ] + [2a + ( n – 1)d ] +…+ [ 2a + ( n – 1 )d] difference is 3. Find the sum of the first 11 term.
we observe that [ 2a + ( n – 1 )d ] is reoccurring n times A.157 B.187 C.197 D.200
in the RHS of the equation. Thus, we can say Solution
2Sn = n ×  2a + ( n – 1 )d S11 = ? but Sn = n  2a + ( n – 1 )d 
Sn = n  2a + ( n – 1 )d 2
2 Here a = 2, d = 3, and n =11 Thus,
A case where the first term ( a ) and last term ( L ) only are given. S11 = 11  2 × 2 + ( 11 – 1 )3
Sn = n ( a + L ) 2
2 = 11  4 + ( 10 )3 
2
2004/4 = 11  4 + 30
The first and last term of a linear sequence (A.P) 2
are – 12 and 40 respectively. If the sum of the sequence = 11 × 34
is 196, find: 2
(a) the number of terms; = 11 × 17
(b) the common difference; (c) the 12th term. = 187 ( B )
83
1990/27 UME Solution
Find the sum of the first twenty terms of the arithmetic Sn = 252, a = –16 , L = 72, find n
progression loga, loga2, loga3,…. Applying the formula for 1st and last term
A.loga20 B.loga21 C.loga200 D.loga210 Sn = n ( a + L )
Solution 2
S20 = ? but Sn = n  2a + ( n – 1 )d  252 = n ( –16 + 72 )
2 2
Here n = 20, a = loga, d = loga2 – loga = n ( 56 )
= 2loga – loga i.e loga 2
Thus, S20 = 20  2 × loga + (20 –1) loga 252 = n × 28
2 252 = n
= 10 2loga + 19 loga 28
= 10 ( 21 loga ) Reducing gradually,
= 210 loga n = 63 i.e 9 ( D )
7
= loga210 (D)
2002/19 PCE
2001/19 PCE How many terms are there in the series 5 + 10 + 15 +...+ 750?
How many terms of the sequence 10, 12, 14,… A.50 B.100 C.150 D.200
will add up to 190? Since series is the sum equivalent of any sequence. The
A. 10 B. 14 C. 15 D. 19 above series is an AP. Once we know the position of 750, we
Solution have arrived.
Sn = 190 Tn = 750
But Sn = n  2a + ( n – 1 )d But a = 5, d = 15 – 10 or 10 – 5 i.e 5
2 Thus, from Tn = a + ( n – 1 )d
Here a is 10, d is 2, n is not given. Tn = 750
n  2 × 10 + ( n – 1 )2 = 190 5 + ( n – 1 ) 5 = 750
2 5 + 5n – 5 = 750
n ( 20 + 2n – 2 ) = 190 5n = 750
2 n = 750 i.e 150 ( C )
n ( 18 + 2n ) = 190 5
2 1996/18 PCE
18n + 2n2 = 190 The sum of all the series 1½ + 1¼ + 1 +… to the 12th term is
2 2 A. 4 ½ B. 3 ½ C.2 ½ D.1 ½
9n + n2 = 190 Solution
i.e n2 + 9n – 190 = 0
Sn = n  2a + ( n – 1 )d
Factorizing
2
( n – 10 ) ( n + 19 ) = 0
n = 10 or – 19 a = 1 ½ , d = 1 ¼ – 1 ½ i.e 5/4 – 3/2 = – 1/4
Sum of positive of terms cannot be negative S12 = 12  2 × 3/2 + ( 12 – 1 )( – 1/4 ) 
Thus; n = 10 ( A ) 2
1991/29 UME = 6  3 + ( 11 )(– 1/4 ) }
Find the sum of the 20 terms in an arithmetic = 6  3 – 11/4 }
progression whose first term is 7 and last term is 117 = 6  1/ 4 }
A.2480 B.1240 C.620 D.124 = 3/2 i.e 1 ½ D.
Solution
S20 = ? But only; First term a = 7 and Last term L = 117 2005/42 UME Exercise 8.8
were given Find the sum of first 20 terms of the series 8, 12, 16,… 96.
Applying the formula for the first & last term A.1040 B. 960 C. 920 D. 1400
Sn = n ( a + L)
2 2005/42 PCE Exercise 8.9
S20 = 20 ( 7 + 117 ) Find the sum of the first 12 terms of the series 5 + 9 + 13 +…
2 A. 612 B. 702 C. 119 D. 324
= 10 ( 124 )
= 1240 (B)

2003/22 UME
The sum of the first n terms of an arithmetic
progression is 252. If the first term is – 16 and the last
term is 72, find the number of terms in the series.
A.6 B.7 C.8 D.9
84
Tn & Sn cases Tn = 1/3 + (n – 1) 13/3
2000/16 = 1/3 + 13n/3 – 13/3
If the sum of the first n terms of a linear sequence (A.P) = 1/3 – 13/3 + 13n/3
is 1 – 9n2, find the third term. = 13n/3 – 12/3
A. –117 B. –53 C. – 45 D. – 8 2005/44 UME
Solution If the 7th term of an AP is twice the third term and the sum of
T3 = S3 – S2 the first four terms is 42, find the common difference.
S2 = 1 – 9(2)2 A. 1 B. 2 C. 3 D. 6
= 1 – 9(4) Solution
= 1 – 36 i.e –35 T7 = 2( T3 )
and S3 = 1 – 9(3)2 Applying AP terms formula Tn = a + ( n – 1)1
= 1 – 9(9) T7  a + ( 7 – 1 )d = 2 [ a + ( 3 – 1 ) d]
= 1 – 81 i.e –80 a + 6d = 2(a + 2d)
Therefore T3 = S3 – S2 a + 6d = 2a + 4d
= –80–(–35) 0 = 2a – a + 4d– 6d
= –80 +35 i.e a – 2d = 0 ………………(1)
= –45 ( C ) Also we are informed that S4 = 42
2005/3 Applying the sum formula for AP
The sum of the first 2 terms of an A.P is 24. The sum of the Sn = n  2a + ( n- 1 ) d
4th and 5th terms is 36. Find its common difference. 2
A. 6 B. 4 C. 2 D. – 2 S4  4  2a + ( 4 – 1 )d  = 42
Solution 2
n 2( 2a + 3d ) = 42
Sn = [2a + (n – 1)d]
2 Divide both sides by 2
First statement: S2  24 = 2/2 [2a + d] 2a + 3d = 21………… (2)
24 = 2a + d ……… (1) Solving the two simultaneous equations 2a + 3d = 21
Second statement: T4 + T5 = 36 ( a – 2d = 0 )× 2 and subtract to eliminate a
i.e (a + 3d ) + (a + 4d) = 36 2a + 3d = 21
2a +7d = 36….. (2) – ( 2a – 4d = 0 )
Solving (1) and (2) simultaneously
2a + 7d = 36 7d = 21
– ( 2a + d = 24 ) d= 3 (C)
6d = 12 2003/35 PCE
d = 12/6 i.e 2 (C) The sum of the first three terms of an AP is – 18. The third
2002/9 term is half the seventh term. Find the sum of the first four
The sum of the first n terms of a linear sequence (A.P) terms of the progression.
is 2n2 – n – 1. Find its nth term. A.30 B.24 C. – 28 D. – 30
A. 4 (n – 1) B. 2n – 3 C. 2 (n – 1) D. 4n – 3 Solution
Solution S3 = – 18
For any A.P, Tn = a + (n – 1)d, i.e 3
/2  2a + ( 3 – 1 )d  = – 18
We need to know a and d from 2n2 – n –1
S2 = 2(22) – 2 – 1 also S3 = 2(32) – 3 – 1 /22a + 2d = – 18
3

= 2 (4) – 3 =2× 9–4 Multiply both sides by 2/3


=5 = 14 2a + 2d = – 18 × 2/3
n 2( a + d ) = – 6 × 2
From the general formula Sn = [2a + (n – 1)d] Divide both sides by 2
2
a+d=– 6×2
S2  5 = 2/2 [2a + d ] 2
5 = 2a + d ………….(1) a + d = – 6 ………. (1)
Also S3  14 = 3/2 [2a + 2d]
Next T3 = ½ T7
28 = 3(2a + 2d)
a + (3 – 1)d = ½ ( a + ( 7 – 1 ) d )
28 = 6a + 6d ……(2)
Solving (1) × 3 and (2) a + 2d = ½( a + 6d)
28 = 6a + 6d Multiply through by 2
– ( 15 = 6a + 3d) 2a + 4d = a + 6d ………(2)
13 = 3d From (1) a = – d – 6
d = 13/3 and 2a = 5 – d Substitute a value into (2)
2a = 5 – 13/3 2(– d – 6 ) + 4d = – d – 6 + 6d
2a = 2/3 – 2d – 12 + 4d = 5d – 6
a = 1/ 3 2d – 12 = 5d – 6

85
– 12 + 6 = 5d – 2d Geometric progression ( GP )
– 6 = 3d A geometric progression or exponential sequence is a form
Thus, d = – 2 of sequence that has a common ratio between any of the term
Also from (1) a + d = 6 and its preceding one.
a–2=–6 Examples
a=–6+2 ( 1 ) 2, 4, 8,...
a=– 4 We observe that common ratio i.e 4 = 8
Therefore, S4 = 4/2 {2 (– 4) + ( 4 – 1 ) (– 2) } 2 4
= 2– 8 + ( 3) (– 2 ) Common ratio ( r ) = 2
= 2 – 8 – 6  ( 2 ) 14, 98, 686,…
= 2 × (– 14 ) Common ratio ( r ) = 98 = 686 i.e 7
= – 28 ( C ) 14 98

2004/17 PCE Exercise 8.10 nth term of a GP


The 5th term of an arithmetic progression is 41 and the Conventionally, we denote the first term by ‘a’ and the
sum of the first four terms is 94. find the common common ratio by ‘r’
difference. Analysing the given examples, we have
A.13 B.11 C.7 D.5 No T1 T2 T3 Tn
Eg1 2 2×21 = 4 2 ×22 = 8 2 × 2n –1 same as
1990/29 PCE Exercise 8.11 = arn – 1
If the sum of the 2nd and 7th terms of an arithmetic Eg2 14 14 × 71 = 98 14 × 72 14 × 7n – 1 same as
progression is 25 and the fifth term is 15, find the = 686 = arn – 1
common difference.
A. 5 B. 10 C. 3 D. 2 In general for any given GP a, ar, ar 2, …arn
3
2005/44 UME adjusted Exercise 8.12 Tn = ar n – 1
If the 8th term of an AP is thrice the 3rd term and the Where a is first term, common ratio r = ar or ar2
sum of the first four terms is 32, find the common a ar
difference
A. 1 B.2 C.3 D.4 2003/19 Neco
Determine the third term of a geometric progression whose
2002/34 (Nov) Exercise 8.13 first and fourth term are 4 and 108 respectively.
The 3rd and 6th terms of a linear sequence are 16 and A. 112 B. 108 C. 36 D. 27 E. 3
34 respectively. Find the sum of the first six terms. Solution
A. 38 B. 50 C. 57 D. 114 For G.P: Tn = arn – 1
To determine the 3rd term, we need to know r apart from the
2004/4 Exercise 8.14 given ‘a’ first term.
The first and the last term of a linear sequence (A.P) T4  108 = 4r4 – 1
are – 12 and 40 respectively. If the sum of the sequence 108 = 4r3
is 196, find: 108 = r3
(a) the number of terms; (b) the common difference; 4
(c) the 12th term. 27 = r3
Raise both sides to same power
33 = r 3
3 = r
Thus, T3 = 4(3) 3 – 1
= 36 (C)

2002/10 ( Nov)
The common ratio of the exponential sequence (G.P)
log x, log x2, log x4, … is
A. x B. 2 C. log x D. log 2
Solution
For this G.P common ratio “r” is
= Log x2
Log x
= 2 log x
log x
=2 (B)

86
2005/36 Neco
Find P if P – 3, 3P + 5 and 18P – 5 are three = 3×2 3 i.e 3
consecutive terms of a geometric progression. 6
A. – 10 or – 1/9 B. – 8 or – 1/9 C. – 9 or 6 T8 = 6 ( 3 )7
1 2
D. 10 or – /9 E. 89 or –10  12 
Solution = 6 ( 3 )6 ( 3 ) recall that  a   a
 
For any G.P, the common ratio says  
3P + 5 = 18P – 5
P–3
 6
3P + 5 = 6  3 2  3
Cross multiply  
(3P + 5)2 = (P – 3)(18P – 5)
9p + 30P + 25 = 18P2 – 5p – 54P + 15
2
= 33 ( 6 )( 3 )
9P2 + 30P + 25 = 18P2 – 59P + 15
Re – arranging as = 27 ( 18 )
18P2 – 9P2 – 59P – 30P + 15 – 25 = 0
9P2 – 89P – 10 = 0 = 27 × 3 2
Factorising 9P – 90P + P – 10 = 0
2
= 81 2 (C)
9P(P – 10) + 1(P – 10) = 0
(9P + 1)(P – 10) = 0
9P + 1 = 0 or P – 10 = 0
Unknown a and r leading to equations
9P = –1 or 10
There are cases where two different terms of a GP which are
P = –1/9 or 10 (D)
not close will be given and we are asked to either find the
2003/20 (Nov)
The 1st and 5th terms of an exponential sequence (G.P) are
first term a and common ratio r or another term in the
16 and 9 respectively. Find the 7 th term of the sequence sequence.
A. 9/16 B. 3/2 C. 27/4 D.1024/27 Examples:
Solution The third term of a geometric progression( GP ) is 360 and
T7 = 16(r)6 the sixth term is 1215. Find the first term ( WAEC )
A.3/2 B.360 C.40 D.160
We find r from T5  9 = 16(r)4
Solution
9
/16 = r4 Apply the general formula
9
1
4 Tn = ar n – 1
  = r T3 = 360 = ar 3 – 1
 16  360 = ar2 ………….( 1 )
6
 9 4
1 6
Also
 9 4
T7 = 16   = 16   T6  1215 = ar 6 – 1
 16    16  1215 = ar5 ………….( 2 )
3 3 Divide ( 2 ) by ( 1 )
 9 2 3
= 16   = 16   = 27 (C)
 16  4 4 ar5 = 1215
ar2 360
2005/2 by gradual reduction,
Given that 6 , 3 2 , 3 6 , 9 2 ,… are r3 = 27
the first four terms of an exponential sequence (G.P) 8
find, in its simplest form, the 8th term r3 = 33
A. 27 2 B. 27 6 C. 81 2 D. 81 6 23
To remove the cube, we take cube root
Solution ( r3 )1/3 = [ ( 3/2 ) 3 ]1/3
For any G.P: Tn = arn – 1
r = 3/ 2
Here a is 6 and r is 3 2 or 3 6 Substitute r value into ( 1 )
360 = ar2 will become
6 3 2 360 = a ( 3/2 )2
Applying surd rules on the first item 360 = 9 a
4
= 3 2 × 6
Cross multiply.
360 × 4 = 9a
6 × 6 360 × 4 = a
9
= 3 12 40 × 4 = a
6 a = 160 ( D )

87
2003/34 PCE a = 12 × 25
The 2nd term of a G.P is 9 and the 7th term is 1 . Find the 25 4
common ratio. 27 a=3 Thus, a, r is 3, 2/5 ( B )
A. 1/3 B. – 1/9 C. 1/9 D. – 1/3
Solution
Applying the formula: Tn = arn – 1 2010/17 Exercise 8.15
Second term T2  ar2 – 1 = 9 The third of a geometric progression (G.P) is 10 and
i.e ar = 9 ……. ( 1 ) the sixth term is 80. Find the common ratio
T7 implies ar7 – 1 = 1 A2 B3 C4 D8
27
i.e ar6 = 1 …………( 2 ) 2001/19 Neco Exercise 8.16
27 If the second term of a geometric progression is 4 and the
Divide ( 2 ) by ( 1 ) 1
fifth term is , the seventh term is
ar6 = 1  9 16
ar 27 1 1
A. B. C. 8 D.256 E. 4096
r5 = 1 × 1 4096 256
27 9
2003/34 Exercise 8.17
r5 = 1
An exponential sequence (G,P) is given by
33 × 32
1 1 1 1
5
r = 1 , , , … What is the common ratio?
35 27 18 12 8
since 15 is same as 1 2 2 3 3
r5 = ( 1/3 )5 A B C D
3 3 2 2
1
Thus; r= /3 ( A )
2000/20 PCE 2004/32 UME Exercise 8.18
The common ratio of a GP is negative. The fifth term is Given that the first and fourth terms of a GP are 6 and 162
respectively. Find the sum of the first three terms of the
48
/625 and the third term is 12/25. Calculate the first term
progression.
and common ratio respectively.
A. 27 B. 8 C. 78 D. 48
A.3, - 2/5 B. 3, 2/5 C.- 2/5, 3 D. -3, - 2/5
Solution
Applying Tn = arn – 1 Sum of n terms of a G.P
48 The sum Sn of n terms of a geometric progression GP is.
T5 = /625 Sn = a + ar + ar2 + ... + ar n – 2 + ar n – 1 ……….( 1 )
48
i.e ar5 – 1 = /625 Then,
48
ar4 = /625 ………..( 1 ) rSn = ar + ar2 + ar3 +…+ arn – 1 + arn …………( 2 )
also, T3 = 12/25 Subtracting ( 2 ) from ( 1 )
On the RHS, there will be elimination of all terms except a &
ar3 –1 = 12/25 arn
ar2 = 12/25 ……………(2) Sn – rSn = a – arn
Divide (1) by (2) Sn( 1 – r ) = a ( 1 – rn )
ar4 = 48  12 Sn = a ( 1 – rn )
ar2 625 25
1– r ( formula one )
We change the RHS from division to multiplication
This formula holds only when r < 1
r2 = 48 × 25
625 12 Also, subtracting ( 1 ) from ( 2 )
2
r = 4 rSn – Sn = arn – a
25
r2 = 2 2 i.e r = 2 Sn ( r – 1 ) = a( rn – 1 )
5 5 Sn = a ( rn – 1 )
Substitute r value into ( 2 )
r–1 ( formula two )
ar2 = 12 will become
25 This formula is used when r > 1
a 2 2 = 12
5 25 Candidates should note that the usage of either formula one
a × 4 = 12 or formula two depends only on the common ratio ( r ) of the
25 25 given geometric progression ( GP )

88
Examples S4 = 3[ 1 – (–2 )4 ]
( 1 ) If the second and fourth terms of a GP are 8 and 32 1 – (– 2 )
respectively. What is the sum of the first four terms ? Since the power of – 2 is even then the result is positive.
( WAEC ORDINARY MATHS) S4 = 3( 1 – 16 )
A.28 B.40 C.48 D.60 1+2
Solution = 3(– 15 )
We need to know a and r before finding sum. Thus 3
Tn = arn – 1 = – 15 ( B )
T2  8 = ar2 – 1
8 = ar ………( 1 ) 1989/29
Also The sum of first two terms of a geometric progression is x
T4  32 = ar4 – 1 and the sum of the last two terms is y. If there are n terms in
32 = ar3 ……..( 2 ) all, then the common ratio is
Divide ( 2 ) by ( 1 ) 1

 x n2
1
ar3 = 32  y  n 1
C.  
y
A. x B. D.  
ar 8
r2 = 4
y x  y x
r2 = 22
Solution
r=2
a + ar = x
Put r = 2 into ( 1 )
a( 1 + r ) = x ………I
a( 2 ) = 8
and
a = 8 i.e a = 4
arn – 1 + arn = y
2
arn – 1 ( 1 + r ) = y ……..II
But common ratio r = 2 > 1
Divide eqn II by I
Thus, we apply the formula for r > 1
arn – 1 ( 1 + r ) = y
Sn = a ( rn – 1 )
a( 1 + r ) x
r–1
rn – 1 = y
S4 = 4 ( 24 – 1 ) x
2–1 To remove power, we take root, of that power. This case the
= 4( 16 – 1 ) power is n – 1, then root of it is 1
= 4 × 15 n–1
= 60 ( D ) 1

 y  n 1
r =  D.
( 2 ) If the 2nd and 5th terms of a GP are – 6 and 48 x
respectively, find the sum of the first four terms
( WAEC ) 1999/19 PCE
A. – 45 B. –15 C.15 D.33 Find the sum of the first five terms of a geometric
Solution progression, whose first term is 3 and its common ratio is ¼
Firstly, we find a and r to four significant figures.
T2  – 6 = ar2 – 1 A.4.000 B.3.996 C.3.699 D.3.561
– 6 = ar …….. ( 1 ) Solution
T5  48 = ar5 – 1 Since common ratio ¼ < 1, we use the formula
48 = ar4 ………. ( 2 ) Sn = a( 1 – rn )
1–r
Divide ( 2 ) by ( 1 )
S5 = 3( 1 – ( ¼ )5 )
ar4 = 48
1–¼
ar –6
r3 = – 8 = 3 ( 1 – 1/1024 )
r3 = (– 2 )3 ¾
r = –2 = 3( 1024 – 1 )  3
Put r = – 2 into ( 1 ) 1024 4
= 3( 1023 ) × 4
a(– 2 ) = – 6
1024 3
a = – 6 Thus, a = 3
= ( 1023) × 4
–2
1024
Since common ratio r = –2 < 1, we apply the formula
= 1023
for r < 1
256
Sn = a( 1 – rn )  3.996 B
1–r

89
2006/27 UME Thus, S∞ = 8
How many terms of the series 1 – 1/2
3, – 6, + 12, –24, +... are needed to make a total of 1– 2 ?
8

A.9 B.12 C.8 D.10 = 8 ÷ 1/2


Solution = 8 × 2 i. e 16 (D)
The series is a GP with r = – 2 1992/9b (Nov)
Thus, formula for r < 1 in sum of GP The sum to infirmity of an exponential sequence (G.P) with a
Sn = a(1 – rr) positive common ratio is 9 and the sum of the first two terms
1–r is 5, find the:
1 – 2 = 3[ 1 – ( -2 )n]
8
(i) Fifth term:
1–(-2) (ii) Sum of the first six terms
Solution
1-28 = 3( 1- ( -2 )n
S∞ = a
3
1–r
1- 28 = 1 – ( -2 )n 9 = a Thus 9(1 – r) = a
since n is 8 which is even – 2 = 2 Thus 1–r
n = 8 C.
we are told that r is positive and since S∞ is mentioned  r < 1.
Thus S2  5 = a (1 – r2)
1–r
5(1 – r) = a (1 – r2)
5 – 5r = a – ar2
1995/22 PCE Exercise 8.20 Substituting for a = 9(1 – r)
Find the sum of the first 8 terms of the geometrical 5 – 5r = 9 – 9r – (9 – 9r)r2
progression 1 + 2 + 4 + 8 +… 5 – 5r = 9 – 9r – 9r2 + 9r3
A.64 B.128 C.255 D.256 Re – arranging
9r3 – 9r2 – 9r + 5r + 9 – 5 = 0
1994/23 PCE Exercise 8.21 9r3 – 9r2 – 4r + 4 = 0
The sum of the first 7 terms of the series Applying factorizing of polynomial formats
1
/7 + 2/7 + 4/7 +… is Try r = 1, it is a zero of the function
A.128 B.127 C. 128 D.127 Thus 9r2 – 4
7 7 14 21 r – 1 9r – 9r2 – 4r + 4
3

– (9r3 – 9r2)
1988/29 UME Exercise 8.22 – 4r + 4
If 7 and 189 are the first and fourth terms of a G.P – (– 4r + 4)
respectively, find the sum of the first three terms of the
0 0
progression.
(r – 1) (9r2 – 4) = 0
A.182 B.91 C.63 D.28
r – 1 = 0 or 9r2 – 4 = 0 ie a difference of two squares
r = 1 or (3r – 2)(3r + 2) = 0
r = 1 or 2/3 or –2/3
Sum to infinity ( S )
r cannot be 1 it will make the formula undefined. r positive.
A G.P whose common ratio is between – 1 and + 1 say
Thus r = 2/3
- ½ , ½ , - ¼ , ¼ etc has a sum which approaches a Substituting for r value into
finite value as n approaches infinity. It is given as; 9(1 – r) = a
S = a 9(1 – 2/3) = a
9(1/3) = a
1–r
Thus, a = 3
2000/3 (Nov)
The 1st and 6th terms of a geometric progression (G.P)
are 8 and 1/4 respectively. What is its sum to infinity? T5 = 3 ( 2/3)4
A. 1/2 B. 4 C. 8 D. 16 = 3( 16/81) i.e 16
/27
Solution
S∞ = a (ii) S6 = 3[1 – ( 2/3)6]
1–r 1 – 2 /3
r can be gotten from: T6  1/4 = 8 (r)5
= 3[1– ( 2/3)6 ] ÷ 1/3
Multiply through by 1/8
1
/32 = r5 Changing division to multiplication
Raising both sides to same powers = 9(1 – 64/729)
(1/2)5 = r5 = 9(665/729) = 665/81 i.e 817/81
Equating terms
1
/2 = r
90
1992/18 UME 2001/22 Neco Exercise 8.23
Find the sum to infinity of the following series The sum to infinity of the sequence;
3 + 2 + 4 + 8 + 16 +… 4 4
3 9 27 4, 4 , , , … is
A.1270 B. 190 C.18 D.9 5 5 5 5
Solution 5
‘sum of infinity’ is mentioned here, thus A. – 4 B C. 5 – 5 D. 5 E. 5 + 5
S = a 5 4
1–r 1992/21 PCE Exercise 8.24
a = 3, r = 2. Find the sum to infinity, of the geometrical progression
3 1 + 1/3 + 1/9 + 1/27 + …
S = 3 A.1, B.11/3 C.1½ D. 113/27
1 – 2/3
= 3 same as 3  1/3 change to multiplies Miscellaneous cases
1
[
/3 2003/10a
=3×3 The sum of the first n terms of a geometric progression (G.P)
= 9 (D) is given by Sn = 6 – 3
2n–1
1995/23 UME Calculate, the common ratio
Find the sum to infinity of the following sequence Solution
1, 9 , 9 2 , 9 3 ,… To find r, we must know at least three members of the G.P
10 10 10 Applying the given Sn = 6 – 3
2n–1
A. 1 B. 9 C. 10 D.10 T1  S1 = 6 – 3
10 10 9 21 – 1
Solution = 6 – 3 i.e 3
‘ Sum to infinity’ is mentioned here, thus T2  S2 – S1
S = a But S2 = 6 – 3
1–r 22 – 1
a = 1, r = 9  1 i.e. r = 9 = 6 – /2 = 12 – 3 i.e 9/2
3

10 10 2
S = 1 Thus, T2 = S2 – S1 = /2 – 3 i.e /2
9 3

1 – 9/10 T3  S3 – S2
But S3 = 6 – 3 = 6 – 3/4 i.e 21/4
= 1 same as 1  1 2 3–1
1
/10 10 Thus, T3 = S3 – S2 = 21/4 – 9/2 = 21 – 18 i.e 3/4
= 1 × 10 4
3
= 10 ( D ) Therefore, the G.P: 3, /2, ¾
With a common ratio:
3
2002/20 PCE /2 ÷ 3 or 3/4 ÷ 3/2
3
Find the sum of the following series to infinity /2 × 1/3 or 3/4 × 2/3 = 1/2
/3 + ( 1/3 )2 + ( 1/3 )3 +….
1

A ½ B. 2/3 C. ¾ D.1 1998/20 UME


Solution The sum of the first three terms of a geometric progression is
S = a half its sum to infinity. Find the positive common ratio of the
1–r progression.
Here a = 1/3, r = [ 1/3 ]2  1/3 A. ¼ B. ½ C. 1 D. 1
= 1 × 3 i.e. 1 3 3 3 2
9 1 3 Solution
S = 1  1 – 1 S3 = ½ S
3 3 Since sum to infinity is mentioned, then, r < 1 thus
=1  2 Thus sum formula for S3 is that of r < 1. i.e.
3 3 a(1–r3) = ½ × a
= 1 × 3 = ½ (A) 1– r 1– r
3 2 Some terms common to both sides will cancel one
2006/13 Exercise 8.23A 1 – r3 = ½
Find the sum of the exponential series 96 + 24 + 6 +… 1 – ½ = r3
A 144 B 128 C 72 D 64 r3 = ½
91
To remove cube, take cube root of both sides Hence S = ½
1–(-½)
r = 3 ½ = ½
cube root of 1 is one 1+½
r = 1 = ½  3/2
3 2 (D) = ½ × 2/3 i.e 1/3
Therefore S – 1 = 1/3 – 1
1999/20 = – 2/ 3 ( C )
The first term of a geometrical progression is twice its 2009/13 Exercise 8.25
common ratio. Find the sum of the first two terms of the
The first term of a geometric series is 350. If the sum to
progression. If its sum to infinity is 8.
A .8 B. 8 C. 72 D. 46 infinity is 250, find the common ratio
5 3 25 9 A –5/7 B –2/5 C 2/5 D 5/ 7
Solution
a = 2r find S2 if S = 8
We cannot find S2 without knowing a and r. Problems on Sequence
Thus from S = 8 The Author deliberately left problems on sequence for the later
i.e. 8 = a but a = 2r part of this chapter since the term ‘sequence’ could mean AP,
1–r GP or any other particular pattern of numbers.
Once a sequence or a progression is mentioned without
8 = 2r specifying whether it is an AP or a GP, or any other patterned
1–r numbers it is the candidates responsibility to identify the type
Cross multiply, of sequence ( progression ) and deal with it accordingly.
8( 1 – r ) = 2r 2005/10a Neco
8 – 8r = 2r A sequence is given by the formula Ur+1 = 3Ur + 2
8 = 10r If the third term is twice the second term, find the first
r= 8 = 4 and second terms.
10 5 Solution
But a = 2r From the given condition
a= 2 × 4 U3 = 2U2
5 Applying the given formula : Ur+1 = 3Ur + 2
= 8 3U2 + 2 = 2(3U1 + 2)
5 3U2 + 2 = 6U1 + 4
We now have a and r, also since S is mentioned our Next, we substitute for U2 = 3U1 + 2
formula for S2 will be that of r < 1 3(3U1 + 2 ) + 2 = 6U1 + 4
Sn = a( 1 – rn ) 9U1 + 6 + 2 = 6U1 + 4
1–r 9U1 – 6U1 = 4 – 8
S2 = 8 ( 1 – ( 4/5 )2 ) 3U1 = – 4
5 1 – 4/5 U1 = – 4/3
= 8 ( 1 – 16/25 ) Second term: U2 = 3U1 + 2
1 = 3(– 4/3) + 2
5 /5
= – 4 + 2 i.e – 2
9
= 5 × 8 ( /25 )
5 2003/10b
= 8 × 9/25 = 72/25 ( C ) A sequence of numbers U1, U2, U3, …Un, satisfies the relation
Un + 1 + n2 = nUn + 2 for all integers n  1.
2000/16 If U1 = 2, find the:
Evaluate ( ½ - ¼ + 1/8 - 1/16 +... ) – 1 ( i ) Values of U2, U3 and U4
( ii ) Sum of the first n terms of the sequence.
A. 2/3 B.0 C.- 2/3 D. - 1 ( iii ) Value of Un + Un + 1 + Un + 2, when n = 20
Solution Solution
The examiner was very technical here. The series here ( i ) U2  U1 + 1 + 12 = 1U1 + 2
is sum to infinity because of +… Thus Substituting for the given U1 value
S = a to find S – 1 U2 + 1 = 1(2) + 2
1–r U2 = 4 – 1
Here a = ½ , r=-¼ ½ U2 = 3
= - ¼ × 2/1 i.e - ½
92
U3  U2 + 1 + 22 = 2U2 + 2 Thus U3 = 1 4 + 1
U3 + 4 = 2U2 + 2 3 9
Substituting for U2 value = 1 13
U3 + 4 = 2(3) + 2 3 9
U3 = 8 – 4 i.e 4 = 13
27 (B)
Similarly U4  U3 + 1 + 32 = 3U3 + 2 1995/5 (Nov)
Substituting for U2 value U4 + 9 = 3(4) + 2 Find the nth term of the sequence 2, 12, 36, 80, …
U4 = 14 – 9 i.e 5 A. n3 – n B. n3 + 2n C. n3 + n D. n3 + n2 E. n3 – 2n
Solution
(ii) To find the sum, first, we know the number pattern AP test
U1 , U2 , U3 , U4 For any AP a, a + d, a + 2d ,…
2 3 4 5 Common difference d = a + d – a or a + 2d – ( a + d )
Simple A.P. with d = 1, a = 2 12 – 2 ≠ 36 – 12
Sn = n {2 × 2 + (n – 1) 1} It is not an AP
2 GP test
= n { 4 + n – 1} i.e n ( 3 + n) For any GP a, ar, ar2,….
2 2 Common ratio r = ar or ar2
a ar
( iii ) When n = 20 12 ≠ 36
= U20 + U21 + U22 2 12
Applying the number pattern deducted at (ii) The A.P and G.P test fails when tried here.
= 21 + 22 + 23 Thus we list the terms and see their pattern.
= 66 T1 2
2003/34 (Nov)
Find the nth term of the sequence 1 , 1 , 1 , … T2 12 : i.e 2 × 6, 4 × 3
3 18 108 T3 36: 6 × 6, 9 × 4
A. 2(1– n) 3(n – 1) B. 3–n 2–n A pattern is seen
C. 2(n – 1) 3–1 D. 2(1 – n) 3–n T1 : 2
Solution T2 : 12 i.e 22 × 3
Taking the denominators alone T3 : 36 i.e 32 × 4
T1 3 T4 : 80 i.e 42 × 5
T2 18 2 × 32 Thus Sn : n2 × (n + 1)
T3 108 But 108 = 2 × 54 It implies Sn = n2 (n + 1)
Also 108 = 4 × 27 = n3 + n2 (D)
Pattern: 22 × 33 1994/7
T3 108 : 22 × 33 How many terms are there in the series
3 + 6 +12 + …+ 384?
Tn : 2n– 1 × 3n i.e 2(n – 1) 3n A. 5 B. 6 C. 7 D. 8 E. 128
Next, we represent them in denominators as they appear AP test
Tn = 1 For any AP a, a + d, a + 2d ,…
2(n – 1) 3n Common difference d = a + d – a or a + 2d – ( a + d )
= 2–1 (n–1) 3– n 6 – 3 ≠ 12 – 6 It is not an AP
= 2(1 – n) 3– n (D) GP test
For any GP a, ar, ar2,….
1997/18 Common ratio r = ar or ar2
If for all integers n, a sequence is given as a ar
Un + 1 = 1 (Un + 1) and U1 = 1 , find U3 12 = 6 Thus, it is a G P
3 3 6 3
A. 13 B. 13 C. 4 D. 5 E. 4 Next, we are to find nth position of the last term 384.
9 27 9 27 27 Tn = arn–1
Solution 384 = 3(2)n–1
From the given formula U3 = 1 (U2 + 1) 384 = 2n– 1
3 3
But U2 = 1 (U1 + 1) 128 = 2n
3 2
= 1 1 +1 128 × 2 = 2n
3 3 Raise both sides to same base
= 4 28 = 2n
9 Equating powers 8 = n (D)
93
2005/10b Neco = r3 – 2r – [( r3 – r2 + r – 1 – 2r + 2]
The first four terms of a sequence are 2, 4/3, 8/9 ,16/27 = r3 – 2r – r3 + r2 + r – 1
find the: Ur = r2 – r – 1
( i ) Sum of the first n terms in the simplest form, Next we find the first four terms
( ii )Least value of n for which the sum to infinity U1 = – 1
exceeds the sum of n terms by less than 1 U2 = 22 – 2 – 1 i.e 1
Solution
( i ) By G.P test: common ratio 4/3 ÷ 2 and 8/9 ÷ 4/3 U3 = 32 – 3 – 1 i.e 5
i.e 4/3 × 1/2 = 8/9 × 3/4 U4 = 4 – 4 – 1
2
i.e 11
common ratio is 2/3 2010/26 Neco
Since r < 1, We apply Sn = a (1 – rn) What is the sum of all the numbers between 5 and 130 which
1–r are divisible by 4?
= 2[1 – (2/3)n ] A 2108 B 3102 C 4216 D 4116 E 5826
Solution
1 – 2/ 3 We start by generating the numbers and get out the pattern
= 2[1 – ( 2/3 )n ] ÷ 1/3 with three members of the sequence
= 6[1 – ( 2/3 )n ] 5, 6, 7, 8 , 9, 10, 11, 12 , 13, 14, 15, 16,…
( ii ) S∞ = a The sequence 8, 12, 16,… a simple AP
1– r It is not advisable to list all the members of the sequence
Here, S∞ = 2 i.e 6 before we find the sum
1
/3 In this case, we apply:
we are to find n, when A.P first and last term sum formula
S∞ – 1 = 6 [1 – (2/3)n] Sn = n (a + L)
5 = 6 [1 – (2/3)n ] 2
Last term L can be gotten by simple test of division by 4
5
/6 = 1 – (2/3)n starting from 130 backward 130/4 no, 129/4 no, 128/4 yes
2 n
( /3 ) = 1 – 5/6 Thus, L = 128
(2/3)n = 1/6 Next we find n from L = 128
Tn = a + ( n – 1 )d
128 = 8 + ( n – 1 )4
128 = 8 + 4n – 4
2000/11a Neco
n = 31
If U1 = 2, U2 = 6 and Ur = a + br,
31
(a) Find a, b and U5 Therefore Sn  S31 = ( 8 + 128)
Solution 2
Ur = a + br, 31
= ( 136 ) = 2108
r is 1 hence U1  2 = a + b -------(1) 2
r is 2 hence U2  6 = a + 2b -------(2)
Solving (1) and (2) simultaneously 2010/1b Neco Exercise 8.27
6 = a + 2b Determine the general term of the sequence –256, –64, –16, …
–(2 = a + b ) Hence find the 7th term of the sequence
4 = b
Substitute b value into (1) 2008/9a Exercise 8.28
2=a+4 The sum of the first n terms of a sequence is given by S n =
2 – 4 = a Thus a = –2
5n 2 5n . Write down the first four terms of the sequence

U5 = –2 + 4(5) 2 2
= –2 + 20 and find an expression for the nth term.
= 18
2000/11b Neco 2009/18 ( Nov) Exercise 8.29
The sum of the first n terms of a series is given If Sn = 2n (n + 3) is the sum of the first n terms of a
by n = n3 – 2n. Find Ur and the first four terms series, find the 4th term of the series.
Solution A 14 B 20 C 36 D 56
For any term Ur to be gotten from the sum at a 2001/21 Neco Exercise 8.30
point The formula for the sum of n terms of a sequence is given by
Say S4 = S4 – S3 the resulting S4 becomes U4 4n 2  2n
Thus Ur = r – r– 1 Sn = . The third term of the sequence is
n
= r3 – 2r – [(r – 1)3 – 2(r – 1)] A. 4 B. 6 C. 10 D. 18 E. 22
= r3 – 2r – [( r3 – r2 + r – 1) – 2r + 2]

94
Chapter Nine Subsets
This is the child or children of a given universal set
SETS E.g. 1  = {a, b, c, d, e}
Set is the collection of objects. (Objects means numbers, X = {a, b, c}
things, places etc). Generally speaking the above Y = {a, k, c, d}
definition is acceptable but for precision. X is a subset of  denoted by  or  i.e. x  
* Set is the collection of well-defined objects. Also, we say that the universal set  contains x i.e.
Eg Supposing a new teacher is posted to your school to   x. While y   and   y means y is not a subset of 
teach Further mathematics in SSII; he goes to the library and  does not contain y respectively.
to get some textbooks. Below is the Librarian’s response
to his question. Further Mathematics teacher: Sir, where Proper and improper subsets
can I get SSII math’s texts? Any universal set is an improper subset of itself.
Librarian’s response; (I) You will get some in the Also the empty set is an improper subset of all set but every
Further mathematics shelves other subset originating from the universal set is a proper
(II) You will get some in shelf 2 subset.
of the further mathematics shelves.
Of course, both responses are correct but statement II is Power sets
precise. We usually denote sets by Capital letters while
their members called ELEMENTS by lower case letters. This is the collection of all the subsets of a given set
Elements (members) of any set are usually enclosed in a say S, denoted by 2s
curly bracket and frequently we may write for short. Example PS1
{x : x satisfies a certain condition } or List all the subsets of
{ x/x satisfies a certain condition} A = { 1, 2, 3, }
The power sets = 2A
Where the signs / or : means “such that” = 23
= 8 (eight subsets)
Universal sets
This is the mother set from which other subsets emerge They are , {1, 2, 3}, {1}, {2}, {3}, {1, 2},
{1, 3}, {2, 3}
or relate and it is denoted by  or U
E.g. I  = {All SSIII students of Esi College Warri} Listing elements of power set
These students include: To list the group of the various subsets, say one element
A = All science students subsets, two elements subsets, three elements subsets etc. we
B = All social science students use the combination formula since all the elements in a given
C = All arts students sets are the same irrespective of the order of appearance ie {a,
E.g. 2  = {x: x is the set of all workers living in Abuja} b} is same as { b, a }
Then the subsets will include: From our given example PS1 :
S = {All civil servants} Subsets of one (1) elements:
3
T = {All taxi drivers} C1 = 3!
V = {All private sector workers} (3 –1 )! 1!
= 3!
Singleton Sets 2! 1!
These are single element (member) sets = 3 × 2! = 3
E.g. 1. A = {x : x is the Principal of Ibru college 2!
Agbarha-Otor} ie {1}, {2},{3} Note that  is a special subset of its own and
not a single element subset.
E.g. 2 B = {x: x is the President of Nigeria}
Subsets of two (2) elements:
You will agree with the assertion that there is only 3
C2 = 3!
one Principal in a college and one President in a (3 – 2)! 2!
country at a given time. = 3!
Null or empty set 1! 2!
This type of set contains no element. It is the subset =3
of every set and is denoted by  or { } They are { 1, 2 }, { 1, 3 }, { 2, 3 }

E.g. W = {y : y is the set of a JSS one student who Subsets of three (3) elements
3
is a principal} C3 = 3!
= (3 – 3)! 3!
Note = 3!
The set J = {0} is not an empty set because zero is an 0! 3!
element. = 1 i.e. {1, 2, 3}

95
2002/17 Exercise 9.0 Solution
How many subsets of 3 elements are obtainable from The universal set elements will be from 2 to 14. Note that 1
the set P = {x, y, z, m, n}? and 15 are not included because of the strict inequality at
A. 10 B. 32 C. 6 D.120 both ends
U = {2, 3, 4, 5, 6, 7, 8, 9, 10, 11, 12, 13, 14}
2004/32 Neco Exercise 9.1
Given P = { 1, a, b, 3, 5}. P = {3, 5, 7, 9, 11, 13} and P1 = {2, 4, 6, 8, 10, 12, 14}
What is the power of the set P? Q = {2, 3, 5, 7, 11, 13 } and Q1 = {4, 6, 8, 9, 10, 12, 14}
A. 5 B. 10 C. 16 D. 32 E. 81 P1Q1 = {4, 6, 8, 10, 12, 14}
Thus, n(P1Q1 ) = 6 (D)
2005/4 Neco Exercise 9.2
Find the number of subsets in the set {a, b, c, d }
Union
A. 4 B. 8 C. 16 D. 61 E. 80 The union of two or more sets is the combination of elements
of the sets without repetition. It is denoted by  and not u.
Equal sets Mathematically if A and B are two sets then,
Two or more sets are equal if they contain the same
AB = {x : x  A or x B}
elements, order of arrangement immaterial
E.g If X = {a , b, c, d} E.g 1 If A = {1, 2, 3, 4, 5, 6}
Y = {b, d, a, c, d} B = { 5, 2, 2, 7,1, 3, 2}
Z = {first four letters of the alphabet} then find A  B.
K = {a, b, c, d, e, f } Solution
We say that X = Y i.e. They contain the same elements AB = {1, 2, 3, 4, 5, 6, 7}
when counted once;letter d repetition in Y is immaterial
.Also X = Z but X ≠ K, since k contains e and f which E.g. 2 List the elements of X Y
are not elements of X If X = {a, b, c, d} and Y = {a, d, e, a, f }
Solution
Equivalent sets X Y = {a, b, c, d , e, f }
Two or more sets are equivalent when they contain
equal elements denoted by ≡. Intersection
E.g Which of the sets listed below are equivalent. The intersection of two or more sets has a result of a set that
A = {1, 2, 3, 4} contains only the common elements in them. It is denoted by
B = {a, c, d, e}  and not n.
C = {a, b, b, e, c, d} If A and B are two subsets, mathematically
A ≡ B since one element of A can pair with only A  B = {x: x A and xB}
one element of B without remainder. But A and B
E,g 1. The sets of two groups of students who took part in the
are not equal. Neither A nor B is equivalent to C.
finals of shell and chevron Scholarship test are given as.
This is so because C has a sixth element which can
S = {Ada, Olu, Akpos}
not be paired; though B is equal to C when the elements
of C are counted once. C = {Ejiro, Akpos, Iluwa} Find S  C
Solution
Other set Notations S C = {Akpos}
“Element of’’ is denoted by 
E.g 1. Y ={x : x is between 10 and 15} Complement of set
Then 11Y, 12 Y and 13 Y but any number outside The complement of any set say k is the set that has every
element in the universal set except those of K.
the range is not an element of y. Denoted by . For
it is denoted by K1 or Kc
instance 30  y means that 30 is not an element of y.
i.e. U – K = Kc
E.g If U = {10, 20, 30, 40, 50} and K = {20, 50}. Find Kc
Cardinality of set: refers to the number of elements in a
Solution
given set - Say X and it is denoted by n ( X ).
Kc = {10,20, 30, 40, 50} – {20, 50}
E.g 1 If X = {i, j, k, l,}
KC = {10, 30, 40}
Then the Cardinality of X is n (X ) = 4
E.g 2 If B = {a, b, c, d, e, f,} then find n (B) Disjoint set
Solution Two sets A and B are said to be disjoint when they have no
n (B) = 6 element in common i.e. AB = 

1994/19 (Nov) Non disjoint set


P and Q are subsets of the Universal set U defined as Two sets C and D are non-disjoint when they have at least
U = { x : x is an integer and 1< x < 15} one element in common
P = { x : x is odd} and Q = { x : x is prime} i.e. CD ≠ 
Find n(P1 ∩ Q1)
A. 3 B. 4 C. 5 D. 6 E. 9
96
Examples Finite set
Given that X = {a, b, c, d}, Y = {a, b} and Z = {c, d} These are countable sets.
Then find the pair of sets that are: E.g. 1 T = {All the State Governors in Nigeria}
(a) Non – disjoint ( b) disjoint. E.g. 2 X = {All the students who registered for
Solution WAEC exams in 2010}
(a) To establish non-disjoint. We find intersection. Even a primary school pupil can easily count all the
X  Y = {a, b} governors in the 36 state of Nigeria. The example 2 is more
Thus X and Y are non-disjoint detailed because WAEC headquarter has the total figure of
(b) Y  Z =  thus, students who sat for the exam in the year 2010.
Y and Z are disjoint. Infinite set
These are uncountable sets
2005/5a Neco Exercise 9.3
What do you understand by E.g.1 D = {x : x  ℕ}
( i ) a set ={All natural or counting number}
( ii ) the “union” of two sets. E.g 2. F = {x: x  ℤ}
( iii ) The “intersection” of two sets. = {Set of all integers}
= {…, - 2, - 1, 0, 1, 2, 3, …}
Range Sets
Here elements of inequality are employed to define the given
sets. They are strict inequalities < (less than)
and > (greater than) & the weak inequalities < ( less than or
LAWS OF SET equal to ) and > (greater than or equal to).
1. IDEMPOTENT LAW Any set defined by strict inequalities does not accept numbers
A A = A; A A = A at the boundaries (limits). Similarly, any set defined by weak
inequality accept numbers at the boundaries (limit).
2. ASSOCIATIVE LAWS
Examples.
A (BC) = (AB) C
List the element of the sets below:
A (BC) = (AB) C
3. COMMUTATIVE LAW (i) G = {x : 1< x < 5}
AB = BA; AB = BA (ii) L = {y : 16 < y  19}
4. DISTRIBUTIVE LAW (iii) M = {a : 25  a < 30}
A  (BC) = (AB)  (AC) (iv) N = {b : 3  b  6}
A  (BC) = ( A B)  (AC) Solution
5. DE-MORGAN’S LAW (i) G = {2, 3, 4}
(AB)1 = A1B1 Reasons: Both boundaries are strict inequalities hence 1 and
(AB)1 = A1B1 5 are not included in the set G.
6. COMPLIMENT LAWS (ii) L = {17, 18, 19}
(A1)1 = A ; U1 =  Reason : 16 is not included because the left half inequality is
AA1 =  ; AA1 =  ; 1 =  strict, while 19 is included because the right half inequality is
7. IDENTITY LAWS weak.
A = A ; A  = A (iii) M ={25, 26, 27, 28, 29}
A =  ; A =  Reason: 25 is included because the left half inequality is weak
while 30 is not because the right half inequality is strict.
2003/5 (iv) N = {3, 4, 5, 6}
If P and Q are subsets of U, the universal set, which of Reason: 3 and 6 are included because of the weak
the following is equivalent to P[QQ1]? inequalities at both ends.
AP B P1 CQ D Q1
Solution Number sets
[QQ1] = U ( Compliment law) This type of set deals with properties of the numbers such as
Next P[QQ1] = PU = P ( Identity law) prime, odd, even, multiples, odd- prime and divisibility. For
the purpose of emphasis the author will briefly define and
1992/4 (Nov) Exercise 9.4 give examples of the above named properties.
P is a nonempty set and Ø is an empty set. Which of the Prime number
following is not true? A number is said to be prime if it has only two factors; one
A P∪Ø=P B P∩Ø=Ø C P∪P=Ø D P∩P=P E (P1)1 =P and itself.
Example:
List the elements of the set
A = {z : z is a prime number from 1 to 10}
A = {2, 3, 5, 7}
Note1 is not a prime number because it has only one factor.

97
Even numbers ( b ) B = {x/x 2 – x + 2 = 0}
These are numbers divisible by 2 without remainder. Solving the quadratic equation x2 – x + 2 = 0
Example: no factors of + 2 that when added will give – 1. Thus, it
Given that W = {x: x is even number from 5 to 15}: cannot be factorized. Applying the general formula
list the elements of W
Solution x=–b± b2 – 4ac
W = {6, 8, 10 , 12, 14} 2a
Odd numbers
Odd numbers are numbers not divisible by 2. = – (–1) ± (–1)2 – 4 × 1 × 2
Example: 2×1
List the element of the set =1± 1–8
R = {k : k is an odd number between 2 and 10} 2
Solution
The word “between’’ here means 2 and 10 are not =1 ± –7
inclusive. 2
R = {3, 5, 7, 9} Note: 1 is an odd number Without going further – 7 is a complex number.
Hence B is a complex number and U is real number.
Multiples Therefore BU
The Multiple of any number is the set which contains ( c ) C = {x / x = 2 or x = 3 or x = 20}
the product of the given number and 1, 2, 3, … except C  U Since 20 or any other positive real number is part
when limits are given. Alternatively the multiples of of the universal set.
any given number, say 4 are those numbers that can be
divided by 4 without remainder. 1999/2 theory (Nov)
Example: List the elements of the set The universal set U and its subsets A and B are described as:
J = {t: 1 < t < 10 and t is the multiple of 3} U = {x : 20 < x < 35};
Solution A = {even numbers} B = {prime numbers}
The range {1< t < 10} = {1, 2, 3, 4, 5, 6, 7, 8, 9, 10} (a) Describe the set C = {21, 25, 27, 33} in terms of A and B.
The multiples of 3 here are {3, 6, 9,}
(b) Find (A∪B)1
Therefore J = {3, 6, 9}
Solution
Odd-prime Numbers Listing the elements A and B within the given limits of the Universal set U
These are numbers that have the odd numbers property U = {21, 22, 23, 24, 25, …34}
20 and 35 is not included because of the strict inequality sign
and are further subjected to the prime number property.
Example: A = {22, 24, 26, 28, 30, 32, 34}
If U = {x: 1 < x < 10} and 20 is not included because of the strict inequality sign there.
Y = {x: x is odd – prime} B = {23, 31}
Then list the element of Y (a) Elements of C are present in the Universal set but
Y odd = {1,3, 5, 7, 9 } not in A or B
Y odd – prime = {3, 5, 7} (b) (A∪B) = {22, 23, 24, 26, 28, 30, 31, 32, 34}
DIVISIBILITY Thus, (A∪B)1= {21, 25, 27, 29, 33, 34}
This is the other term for multiple. Also not divisible
and not a multiple are the same. 2002/4
If A = { (x, y) : y2 = 4x } and B = { (x, y) : yx = 16}, find
2005/2 Neco (Dec) the elements of A  B1.
If the set U = {1, 2, 3, 4, ...}, describe the following sets Solution
by listing method and state whether they are subsets of U. Common reasoning in listing the elements shows that
A = {x/x2 – 3x + 2 = 0} If y2 = 4x
B = {x/x2 – x + 2 = 0} Then 42 = 42
C = {x/x =2 or x = 3 or x = 20} i.e y is 4 and x is 2 in (x, y) format (2, 4)
Solution Thus A = { 2, 4 }
( a ) A = {x/x2 – 3x + 2 = 0} Next if yx = 16
Solving the quadratic equation Then 42 = 16
x2 – 3x + 2 = 0 i.e y is 4 and x is 2 in (x, y) format (2, 4)
x – 2x – x + 2 = 0
2
Thus B = {2, 4}
x(x – 2) – 1(x – 2) = 0 Therefore A B = {2, 4}
(x – 2)(x – 1) = 0
x – 2 = 0 or x – 1 = 0
x = 2 or 1
and 1 and 2 are in U, thus A is a subset of U written as AU or
AU
98
2002/33 (Nov) 21. Find B1 ∪ C
Given that U = {– 10 ≤ x ≤ 10}, P = {x : – 10 < x < 10} A{1,2,3,4,5,9} B{2,4,5,7,8,10} C{1,2,4,5,7,8,10}
and Q = { x: – 5 < x ≤ 3}, where x  I, which of the D{2, 4, 5, 6, 7, 8, 10} E {2, 3, 4, 5, 6, 7, 8, 9}
following are correct?
I. P1 ∩ Q = Ø II. P ∪ Q = U III. P1  Q1 1999/2 obj (Nov) Exercise 9.6
A I and II only B I and III only Given that: P = {1,3,5,7,9,11}; Q = {2, 4, 6, 8,10,12}; R =
C II and III only D I, II and III {2, 3, 5, 7, 11}; are non – empty subsets of the universal set 
Solution = {1, 2, 3, ..12}, which of the following statements are true?
First, we list the various elements I. Q ∩ R = Ø II. R  P III. (R∩P)  (R∩E)
U = {-10, -9, -8, …,7, 8, 9,10} -10 and 10 are included A I, II and III B II and III C I and II D I and III
because of the weak inequality signs present there.
P = {-9, -8, -9 …,7, 8, 9} -10 and 10 are not included 1993/9 (Nov) Exercise 9.7
because of the strict inequality signs present there If the universal set U = {x : x is a natural number and 1≤ x ≤ 9},

Q = {- 4, -3, -2, -1, 0, 1, 2, 3} -5 was not included P = {x : 1 ≤ x ≤ 4} and Q = {2, 4, 6, 8} Find (P∪Q)1.
because of the strict inequality sign there where as 3 was A {1, 2, 3, 4, 5, 6, 7, 8, 9} B {1, 2, 3, 4, 6, 8}
included because of the weak inequality sign there C {4, 5, 6, 7} D {5, 7, 9} E {2, 4}
The other items needed to answer this question are P1 and Q1
P1 = {-10, 10} and 2000/3 Exercise 9.8
Q1 = {-10, -9, -8, -7, -6, -5, 4, 5, 6, 7, 8, 9, 10} If P={x : 2≤x<16, x is prime} and Q={x : 2≤x≤13, x is odd},
Now, going by the listed values evaluate n(P∪Q) + n(P∩Q).
A.12 B. 13 C. 26 D. 35
I. P1 ∩ Q = Ø correct
II. P  Q = P and not U as stated (since P is bigger than Q) 2000/40 (Nov) Exercise 9.9
III. P1  Q1 is correct Thus, Option C holds. P and Q are non – empty subsets of a universal set U
where U = {integers}, P = {x : 2x + 6 > 0} and
2004/9C Q = {x : (x + 5)(x – 2) < 0}. Find PQ
Given the sets P and Q defined by the ordered pairs: A {x : - 2 ≤ x ≤ 1} B {x: - 5≤ x ≤ -3 }
P = { (x, y) : 0< x < 4, 0< y < 4 }, C {x : - 3 < x < 2} D {x : - 5 < x < 2}
Q = { (x, y) : 3 ≤ x ≤ 4, 3 ≤ y ≤ 4 },
Where x and y are integers, Find:
2003/37 Exercise 9.10
( i ) P, ( ii ) Q, ( iii ) PQ Given that P and Q are sets of real numbers, such that
Solution
P = {x: 0 ≤ x ≤ 3} and Q = {x: - 3 < x < 2}, find P  Q.
( i ) P = { x – components y – components }
A -3< x < 2 B 0≤ x<3 C -3< x ≤ 3 D 0 ≤ x < 2
1, 2, 3 1, 2, 3

P = {1, 2 , 3} 2006/1 Exercise 9.11


If P = {x : 1≤ x ≤ 6} and Q = {x : 2 < x < 9},
(ii) Q = {x – components y – components} where x  R, find P  Q.
3, 4 3, 4 A. {x : 2 ≤ x ≤ 6} B. {x: 2≤ x < 6}
C. {x : 2 < x < 6} D. {x : 2 < x ≤ 6}
Q = {3, 4}
(iii) PQ = {3}

2000/19 – 21 Neco Exercise 9.5


Use the information below to answer questions 19 to 21.
 = {1,2,3,…,10}, A ={2,4,6,8,10 }, B = {1,3,9}
and C = {2,5,7}

19. Set A, B and C are defined as sets of_______,


______ and _______ numbers from 1 to 10.
A. even, prime and odd B. even, odd and prime
C. rational, non – rational and fractional.
D. Surdical, fractional and integers
E. positive, negative and neutral

20. Find A1 ∩ C1
A. {3, 9} B. {1, 3, 4} C. {1, 3, 9}
D. {6, 7, 8, 9} E. {4, 5, 6, 7, 8}
99
Venn diagram Analysis and Solution
This is a representation of set in shapes whereby subsets (a) To be able to draw the Venn diagram effectively, we
are represented by circles while rectangle enclosing the must list elements of the various intersections.
circle(s) represents the universal set. ABC = {3}
ABC1 = {5, 7, 11}
Number Venn diagram AB1C = {9}
This is a representation of a given set of numbers or
A1BC = {15}
set’s solution in Venn diagram.

Example NV1 U
A 5,7,
2 B
Find PMG1 in the diagram below 1 11
9 3 15
P 6
7 M 6,12 C
11
3
2
4 4, 8, 10, 13, 14
10 G
b( i ) CA1 = {6, 12, 15 }
A {7} B {6,3} C.{6, 11} D.{6} E.{3} (ii) A1 (BC) = {6, 12, 15, 2}
Analysis and Solution Note: After you have drawn your Venn diagram, always
If we cover G imaginarily you will discover that though check whether the elements in each set correspond with the
3 is part of P and M, but 3 belongs to P, M and G. one given i.e.
Hence PMG1 = {6} and not {6, 3) A 1 5,7
11
Example NV2 9 3
If U = {x : 0 < x < 4} A = {1, 3, 5, 7, 9, 11} as given
P = {y : 1< y < 4}, Q = { k : 1< k < 4} Also confirm whether the members of the universal sets are equal
and R = {r : 0 < r < 2} to the one given i.e A BC + (A BC)1 = 
a. Illustrate the above information in a Venn diagram
b. Find P1QR1 2002/7 (Nov) WAEC ordinary maths
Analysis and solution Given that X, Y and Z are subsets of the universal set
We would have to list the elements of P,Q and R as well  = {1, 2, 3, 4,…,10} such that
as their intersection. X = {P : 2 < P < 9; P is a prime number}
U = {0, 1, 2, 3, 4, }, P = {1, 2, 3}, Q = {2, 3, 4}and Y = {n : 1 < n < 7, n is an integer}
R = {0, 1, 2} Z = {m : 2 < m < 10, m is a multiple of 2}
PRQ = {2} List the elements of
PRQ1 = {1, } (i) X; (ii) Y; (iii) Z;
(C) (i) Draw a Venn diagram to illustrate the information in
PR1Q {3}
(b) (ii) find X1Z1Y.
P1RQ = { }
Analysis and Solution
U i. X @ = {2, 3, 4, 5, 6, 7,8}
P 3 4 X = {2, 3,5, 7,}
Q ii. Y = {1, 2, 3, 4, 5, 6, 7}
2
1 iii. Z@ ={2, 3, 4, 5, 6, 7, 8, 9,10}
0 Z = {2,4, 6, 8, 10}
R @
is the first condition

(B) P1QR1 = {4} i.e. Q members only C (i) To draw the Venn diagram effectively, we must list the
Note: the total number of elements in diagram above is elements of the various intersections i.e
equal to the universal set i.e. n (U) = 5. XYZ = {2}
XYZ1 ={3, 5,7}
XY1Z = { }
X1YZ {4, 6}
2002/2 WAEC ordinary maths
The set A = {1, 3, 5, 7, 9, 11} B = {2, 3, 5, 7, 11, 15} and U
X Y
C = {3, 6, 9, 12, 15}are subsets of  ={1, 2, 3, …, 15 } 3,5,
7 1
(a) Draw a Venn diagram to illustrate the given 2
information. 4,6
(b) Use your diagram to find: 8,10 Z
(i) CA1 (ii.) A1 (BC).
(b) (ii) X1Z1Y = {1} i.e. Y only

100
2000/2 Neco ordinary maths
A U
A,B and C are subsets of the Universal set U such that: B
U = {1, 2, 3, 4, 5 …20} 4,6 2
A = {x : x is even }, B = {y : y is odd}and 3,5
C = {z : z is prime}
(a) Draw a Venn diagram to illustrate the
1,7
information given above. C
(b) Find (i)AB (ii) A B1C (iii) BC
Analysis and Solution 2002/9 Neco ordinary maths Exercise 9. 12
To draw the Venn diagram, we must list the subsets and In the diagram below, what is XY1Z1?
their intersections.
U
U = {1, 2, 3, 4, 5, … 20} X Y
3,4
A = {2, 4, 6, 8, 10, …,18, 20} 1,6 7
B = {1, 3, 5, 7, 9, 11, 13, 15, 17, 19} 2
5
C = {2, 3, 5, 7, 11, 13, 17, 19} 8
ABC =  , ABC1 = , AB1C ={2} 9 Z
A1BC = {3,5, 7, 11, 13, 17, 19}
(a) A {1, 6} B {3, 4} C {3, 4, 5 } D {3, 4, 7} E {2, 5, 8,9}
B C A U
2004/1 (Nov) WAEC ordinary maths Exercise 9.13
3, 5 4,6,8 The sets A, B and C are subsets of the universal set
1,9 7,11 2 10,12 U = {1, 2, 3, … 14}
15 13,17 14,16
10 18,20 A = {Even numbers}, B = { Factors of 24} and
C = {3, 6, 9, 12}.
(a) list the elements of A and B
(b) ( i ). AB =  ( ii ). AB1C ={2} (b) Illustrate the information on a Venn diagram.
(iii). BC {1, 2,3, 5, 7, 9,11,13, 15, 17, 19} (c) Using your Venn diagram, determine A1∩ C1
2003/36 – 37 WAEC ordinary maths
Use the venn diagram to answer Questions 36 and 37
P
Q
R
d
f e h

c g
i
a b
36. How many elements are in PQ ?
A. 1 B. 2 C. 4 D. 6
Solution
We will pretend not to see R as part of the set
PQ = Q elements ; {f , e } 2 of them ( B )
37 . Find Q1 R
A {e} B{ c, h } C.{c, g, h} D.{ c, e, g, h}
Solution
We will pretend not to see P as part of the set
Q1 R = R only ; {c, g, h} ( C )

2004/2b Neco (Dec )


If µ = {I ≤ x ≤ 7 }, A = { 3, 4, 5, 6}
B = {2, 3, 5} and C = {1, 3, 5, 7}
Draw a Venn diagram to represent the sets and hence,
find A1B1C1
Solution
µ = {1, 2, 3, 4, 5, 6, 7,}
A = {3, 4, 5, 6,} B = {2, 3, 5} C = {1, 3, 5, 7 }
A B C = {3, 5,}
ABC1 = { }, AB1C = { }, A1BC = { }
A1 B1C1 = { }
101
Venn diagram shading A
This is the representation of set relationships such as A B B
intersections, union and complements by appropriate
shading in Venn diagrams

One subset C C

U U AB1C shaded ABC1 shaded


A
A
A B A B
A1 shaded A
shaded
C C
Two subsets
(i) When A  B (A is a subset of B) A BC shaded
1
AB C1 shaded
1

B B
A
A A B A B
1
A Shaded AB shaded

C C
AB
A1BC1 shaded A1B1C shaded
AB shaded A B
A
B
(ii) Independent subsets

A BUA B C C
(A BC) shaded
1
(BC) A C shaded
1

2004/17 Neco ( Dec)

A B
U
AB shaded AB shaded

A B A B
C

The Venn diagram above represents which of


the following sets:
A ABC1 B A1B1C
(AB)1 shaded (AB)1 shaded C A B C
1 1
D AB1C1
Solution
A B A B Shaded portion A1 B C 1

1995/10 WEAC ordinary maths


In the diagram below, the shaded portion is
A.P1R B.QR C.P1QR D.(PQ)1R
A1B shaded AB1 shaded E.(RQ)  (P R)
1

P Q
U
(iii) Three subsets
A B A B
R
Shaded portion (RQ)  (P1R) E.

2003/5 Exercise 9.14


C C If P and Q are subsets of U, the universal set, which of the
following is equivalent to P  [Q  Q1]?
ABC shaded A BC shaded AP B P1 CQ D Q1
102
2005/10 Neco Exercise 9.15 n(PM) = n(U) = 42 the statement “each student offers at
The shaded portion in the figure below represents least one of mathematics and physics” implies all the
P students participated.
n(P) = 22, n(M) = 28, n(PM) = ?
Q Substituting
42 = 22 + 28 – x
42 = 50 – x
x = 50 – 42
= 8 students
R
1
Alternatively
A. QRP B.QRP C.R1RP1
P22 M28 U42
D. Q R P1 E.RQ  P1
1992/4 (Nov) Exercise 9.16
P is a non – empty set and Ø is an empty set. Which of 22-X X 28-X
the following is not true?
A. P∪Ø = P B. P∩Ø = Ø C. P∪P = Ø
From the diagram above
D. P∩P = P E. (P1)1 = P 42 = 22 – x + x + 28 – x
42 = 50 – x
TWO SUBSETS’ PROBLEMS x = 50 – 42 = 8 students
Example TS 3.
For any two subsets A and B not disjoint i.e AB ≠ 
A survey of 100 students in an institution shows that 80
n(AB) = n(A) + n(B) – n(AB) students speak Hausa and 20 students speak Igbo. While only
Example TS1 9 students speak both languages. How many students speak
In an evening lecture organized for the teaching of neither Hausa nor Igbo?
SSCE based Mathematics and English in one of the Solution
centres in Awirhe. 25 offers mathematics and 32 offer H – Hausa
English while 13 offers both courses. How many I - Igbo
candidates enrolled for the lecture?. Each student offer x – those who speak neither Igbo nor Hausa
at least one of the two subjects. n(HI ) = n(H) + n(I) – n(HI)
Solution Here n(U) = n(HI) + x or n(U) – x = n(HI )
Let M – Maths
E – Eng. n(HI) = 100 – x, n( H) = 80, n(I) = 20, n(HI) = 9
n(ME) = n(M) + n(E) – n(ME) Substituting
n(ME) = n(U) = ? Since all the students participated. 100 – x = 80 + 20 – 9
100 – x = 91
n(M) = 25, n(E) = 32, n(ME) = 13
100 – 91 = x
Substituting
9=x
n(ME) = 25 + 32 – 13
Alternatively (Applying Venn diagram)
= 57 – 13 U100 U 100
= 44 candidates. H 80 I 20 H I
Alternatively (Applying Venn diagram) 80-9 9 20-9 9 11
= 71
M 25 E32 U U
M E
= X X
25-13 13 32-13 12 13 19
71 + 9 +11 + x = 100
91 + x = 100
n(ME) = n(U) = 12 +13 + 19 x = 100 – 91 = 9
= 44 candidates Example TS 4
In a youth Club with 94 members, 60 like modern music and
Example TS 2 50 like traditional music. The number of the members who
In a science class of 42 students, each student offers at like both traditional and modern music is three times those
least one of mathematics and physics. If 22 students who do not like any type of music. How many members like
offer physics and 28 students offer mathematics, find only one type of music?
how many student offer both subjects. Solution
Solution M – Modern Music ,
P – physics T – traditional,
M – mathematics, then x – Those who do not like any of the two music
n(PM) = n(P) + n(M) – n(PM) 3x – those who like both musics

103
M60 T50 U94 M T U94 In fig II: If that situation occurs we will have a big possible
value of n(PQ); where as in fig I, if the situation occurs, we
will have the smallest possible value of n(PQ) by principle
60-3x 3x 50-3x 36 24 26 of union of two subsets, the bigger one will dominate the
=
smaller
X 8 n(P) = 10 will be what matters
60 – 3x + 3x + 50 – 3x + x = 94 2001/1b Neco ordinary maths
110 – 2x = 94 5
110 – 94 = 2x
/6 of a class of 60 students study mathematics; 3/5 study
16 = 2x economics. Every student studies at least one of these
subjects. How many students study both subjects?
x = 16/2 i.e x = 8
Analysis and Solution
Those who like one type of music only M – mathematics
= 36 + 26 i.e 62 E – Economics
2003/3 Neco
n (M) = 5/6 of 60 = 50 students
Let A and B be sets of real numbers.
n (E) = 3/5 of 60 = 36 students
( a ) If n(A) = 23, n(B) = 12, n(AB) = 35
Here n (U) = n (ME) Reason: the statement
( i ) Find n(AB)
“every Student…”
( ii ) What can you say about sets A and B?
(ME) = n(M) + n(E) – n(ME)
( b ) If n(A)=16, n(AB)=7 and n(AB)=16, find n(B)
60 = 50 + 36 – x
Solution
60 = 86 – x
a ( i ) Let n(AB) be x
x = 86 – 60
Applying the formula for two subsets
= 26 students.
n(AB) = n(A) + n (B) – n(AB)
35 = 23 + 12 – x
2008/5 Exercise 9.17a
35 = 35 – x
In a Hotel the breakfast menu is a choice between
x = 35 – 35
yam (Y) or plantain (P) or both. The Venn diagram shows
= 0 this implies empty set
the choices made by 25 guests of the Hotel.
( ii ) sets A and B are disjoint P
Y

( b ) Let n(B) be x, 2
(2x+1) x (x-2)
Then applying the formula for two subsets
n(AB) = n(A) + n(B) – n(AB)
16 = 16 + x – 7
16 – 16 + 7 = x (a) Find the value of x?
7=x (b) What is the probability that a guest chosen at random
choose only one of the two?
1994/18
P and Q are 2 sets such that 1997/4 WAEC ordinary maths Exercise 9. 17b
n(P) = 17, n(Q) = 14 and n(PQ) = 5 A and B are two sets. The number of elements in
where n(x) denotes the number of elements in set x. AB is 49, the number in A is 22 and the number
find n(PQ) in B is 34. How many elements are in AB ?
A. 36 B. 31 C. 28 D. 26 E.19 A. 105 B. 27 C. 15 D. 12 E. 7
Solution
Applying the formula for two students 2001/4 WEAC ordinary maths Exercise 9.18
n(PQ) = n(P) + n(Q) – n(PQ) In a class of 80 students, every student studies Economics or
= 17 + 14 – 5 Geography or both. If 65 students study Economics and 50
= 26 (D) study geography, how many study both subjects?
A.15 B.30 C.35 D. 45
2002/16
P and Q are non - empty sets such that 2002/17 (Nov) WAEC ordinary maths Exercise 9.19
n(P) = 10 and n(Q) = 6. Find the smallest possible value of n(PQ) If n (P) = 21, n (R) = 33 and n (PR) = 46, where n (P) is the
A. 6 B. 4 C. 10 D. 16 number of elements in the set P, find n(PR).
Solution A.8 B.34 C.58 D.100
The two possible values are represented below:
P10 P10 2001/3a NABTEB ordinary maths Exercise 9.20
Q6
In a class of 20 students. 12 play table – tennis, 13 play
Q6 cricket ball and 3 are NOT allowed to play either of the two
games. Find how many students play.
(i) both table – tennis and cricket balls,
(ii) cricket ball only; and
Fig I Fig II (iii) table – tennis only
104
Three subsets (a )
For any three subsets A, B and C not disjoint i.e B88 U160
ABC   C70
25-x
n(ABC) = n(A) + n(B) + n(C)
x
– n(AB) – n(AC) – n(BC) + n(ABC) 42-x 20-x
If the given three-subset problem is disjoint apply only
the Venn diagram approach. Otherwise both methods M82
can be used.
25 – x is what we are to solve but we can get x by the
Example TSB 1 formula
A group of market women sell at least one of yam, n(CBM)
plantain and maize. 12 of them sell maize, 10 sell yam = n(C)+ n(B) + n(m) - n(CB) - n(CM) - n(BM)+ n(CBM)
and 14 sell plantain. 5 sell plantain and maize, 4 sell Here n(µ ) = n(CBM) since it was stated that each person
yam and maize, 2 sell yam and plantain only while 3 rides on at least one of the vehicles.
sell all the three items. How many women are in the Substituting
group? 160 = 70 + 88 + 82 – 25 – 20 – 42 + x
Analysis and Solution 160 = 240 – 87 + x
Applying Venn diagram approach 160 = 153 + x
n(m) = 12, n(Y) = 10, n(P) = 14 ; n(PM) = 5; 160 – 153 = x
n(YM) = 4, n(YP) only = 2 i.e n(YPM1) 7=x
n(MYP) = 3 ( b )Thus, 25 – x will become 25 – 7 = 18
( c ) Either one of the three vehicles” implies x i.e 7
M Y
U M
Y 2003/5 Neco (Dec)
4-3 21-1-3-2 1 In a science class with 80 students, 40 students passed
10-3-2-1
5- 3 3 2 = 2
3 2 mathematics, 45 passed chemistry and 30 passed biology
20 passed both mathematics and chemistry, 12 passed
P 14-2-3-2 mathematics and biology. If 3 students did not pass any of
P
the subjects, how many students passed all the subjects?
Solution
Let the first letter of each subject represent it.
M Y U n(MCB)
15 1 4 = n(m)+ n(C) + n(B) - n((MC)- n(mB) - n(CB)+ n(MCB)
3 Here n(U) ≠ n(MCB) since three students did not pass any subject
2 2
Rather n(MCB) = n(U) – 3
7 P Substituting
80 – 3 = 40 + 45 + 30 – 20 – 12 – 8 + x
77 = 115 – 40 + x
n (MYP) = 15 + 1 + 3 + 2 + 7 + 2 + 4 77 = 75 + x
= 35 77 – 75 = x
2=x
Two students passed all the subjects
2000/10 Neco
In a community of 160 people, 70 have cars, 82 have
2003/10b (Nov)
motorcycles and 88 have bicycles, 20 have both cars
In a survey, out of 100 out – patients who reported at a clinic
and motorcycles, 25 have both cars and bicycles, while
in a week it was found out that 70 complained of fever, 50
42 have both motorcycle and bicycles. Each person
had stomach ache and 30 had injuries. All the 100 patients
rides on at least any one of the vehicles.
had at least one of the complaints and 44 had exactly two of
( a )Draw a Venn diagram to illustrate this information
the complaints. How many patients had all the three
( b ) Find the number of people that have both cars and
complaints?
bicycle but not motor cycles.
Solution
( c )How many people have either one of the three
This questions calls for application of our knowledge of
vehicles?
solving three subsets formula / Venn diagram problems
Solution
let the first letters represents each complaint
Let the first letters of the vehicles represent them.
n(FS) = x n(FI) = y n(SI) = z
n(C) = 70 n(M) = 82, n(B) = 88
and n(FSI) = p
n(CM)=20, n(CB)=25, n(MB) = 42,
n(CBM) = x

105
F70 U100
S50 15 25
Physics 45 Chemistry
x-p

P 75
y-p z-p 30 10
I30 20
Maths
But x – p + y – p + z – p = x + y + z – 3p
and x + y + z is 44 Analysis and Solution
Thus x – p + y – p + z – p = 44 – 3p At least two of the three subjects implies two or more
From the formula method subjects. In this case, two subjects + three subjects i.e.
n(FSI) 45 + 30 + 10 + 75 = 160 (B)
= n(F) + n(S) + n(I) – n(FI) -n(FS) - n(SI) + n(FSI)
1990/11 WAEC ordinary maths
= n(F) + n(F) + n(S) + n(I) In a class of 40 students, 25 speak Hausa, 16 speak Igbo,
– [n(FI) + n(FS) + n(SI)] + n(FSI) 21 speak Yoruba and each of the students speaks at least
100 = 70 + 50 + 30 – [ 44 + 3P] + P one of these three languages. If 8 speaks Hausa and Igbo,
100 = 150 – 44 – 3P + P 11 speak Hausa and Yoruba and 6 speak Igbo and Yoruba.
100 = 106 – 2P (a.) Draw a Venn diagram to illustrate this information,
2P = 106 – 100 using x to represent the number of student that
2P = 6 speaks all three.
P = 6/2 i.e 3 (b.) Calculate the value of x.
Analysis and Solution
Example TSB 2 H - Hausa
A survey of a group of 20 patients who took one or I - Igbo
more of panadol, cofta and vix for the treatment of Y - Yoruba
catarrh and cough shows that:16 used panadol (P) Here n(HIY) = n (U) since all the students speaks
9 used cofta (C) and 8 used vix (V). No patient took vix at least one of the three languages
and cofta. 7 patients took panadol and vix. n(H) = 25, n (I) =16, n(Y) = 21
a.) Write down the last two statements in set n(HIY) = 40, n(HTY) = x
language, using the letters indicated above. n( HI) = 8, n(HY) = 11, n(IY) = 6
b.) Illustrate the given information in a Venn
diagram
c.) How many patients took panadol and cofta H25 U40
I 16
Analysis and Solution 8-x
a. VC =  and n(PV) = 7 X
b. 11-x 6-x
V8 P16 C9 U20 Y21
(a)
8-7 7 16-7-x x 9-x (b) n(HIY) = n(H) + n(I) + n(Y) – n(HI) –
n(HY) – n(IY) + n(HIY)
Substituting
40 = 25 + 16 + 21 – 8 –11 – 6 + x
40 = 62 - 25 + x
c. 40 = 37 + x
8 – 7 + 7 + 16 – 7 – x + x + 9 – x = 20 40 – 37 = x
26 – x = 20 x=3
26 – 20 = x 1991/3 WAEC ordinary maths
x=6 In a certain class, 22 pupils take one or more of chemistry,
But n(PC) = x which is 6 Economics and Government. 12 take Economics (E), 8 take
Government (G) and 7 take Chemistry (C) nobody takes
Economics and Chemistry and 4 pupils take Economics and
1989/17 WAEC ordinary maths Government.
The Venn diagram below shows the number of students (a), (i) Using set notation and the letters indicated above,
who studied physics, chemistry, and mathematics in a write down two statements in the last sentence
certain school. How many students took at least two of (ii) Draw a Venn diagram to illustrate the information
the three subjects? (b). How many pupils take
A.165 B. 160 C. 155 D. 135 E. 85. i.) Both Chemistry and Government
ii.) Government only?
106
Analysis and Solution (i) 15 + 3 + 4 + 6 + x + 3 + x = 51
A (i.) EC =  or n(EC) = 0 2x + 31 = 51
n(EG) = 4 2x = 51 – 31
(ii) 2x = 20
U22 x = 10 students
E 12 G8
( ii ) only one subject = (m + p + c) only
= 15 + x + x
12 - 4 4 X 7-x = 15 + 10 + 10
8-4 - x = 35 students
C7
The acceptable diagram is as shown above and NOT 2004/53-55 Neco (Nov) ordinary maths
The Venn diagram below shows the number of students
G E C G E C
E G offering Mathematics (M) , Biology (B) and Agriculture
science (A) in a final class of Ayegbe Comprehensive High
C School . The total number of students is 85
Use the information given in the Venn diagram to answer
Because econs and chem. are disjoint questions53 – 55
b(i.) 12 – 4 + 4 + 8 – 4 - x + x + 7 – x = 22 M B U85
23 – x = 22 7 + x 5x 21 - x
23 – 22 = x 3x
11x 9x
x=1
30 - x
n(CG) = 1
(ii) n(G) only = 8 - 4 – x
A
=4–1
=3 Exercise 9. 21
2003/10 NABTEB ordinary maths 53. How many students offer both mathematics (M) and
A recent survey of 51 students revealed that the number Biology (B) ?
studying one or more of the THREE subjects: A. 57 B.28 C. 19 D. 8 E. 5
Mathematics (M), physics (P) and chemistry (C) is as
follows: Exercise 9.22
Subject No of students 54 How many students offer all the three subjects ?
A. 85 B. 11 C. 9 D.5 E. 3
M 28
M and P 7 Exercise 9.23
M and C 10 55. How many students offer at least two of the subjects ?
P and C 7 A. 3 B. 28 C. 31 D.34 E. 57
M, P and C 4
Miscellaneous (WASC ordinary maths) Exercise 9.24
If the number of students who study Physics as their
In a class of 50 students, 25 offer mathematics, 22 offer
only subject is the same as that who study chemistry as
physics. 30 offer chemistry and all the students take at least
their only subject, Find the number of students who
one of these subjects. 10 offers physics and mathematics, 8
study
offer chemistry and mathematics, 16 offer physics and
(i) only Physics (ii) only one subject
chemistry.
Solution
(a) Draw a Venn – Euler diagram to illustrate the
If we are to apply formula method for 3 subsets
information.
problem , all the needed items are not complete;
(b) Find the number of students who offer all three
no n (P) and no n(C)
subjects.
Let “If the number of students who study physics as
their only subject is same as that who study chemistry
2002/10 Neco ordinary maths Exercise 9.25
as their only subject” be x
In a certain examination, 52 candidates offered Biology,
Applying Venn diagram
60 History, 96 Mathematics, 21 offered Biology and History,
M P U=51 22 mathematics and biology, 16 Mathematics and History. If
7-4 X 7 candidates offered all the three subjects:
28-3-4-6 (a) How many candidates were there for the examination?
i.e15 4 (b) How many candidates offered ONE subject only ?
10-4 7-4
(c) How many candidates offered TWO subjects only?
X (d) How many candidates offered at least TWO subjects?
C

107
Chapter Ten Special angles trigonometric ratios
Trigonometry Angle of 450
Trigonometry is simply the study of triangles in relation to
their sides and angles.
To achieve the desired goal here; candidates must allow for
For any right angled triangle of the form below. the mathematical assumption that follows. A right – angled
B triangle whose  is 450 with opposite and adjacent sides of
unit 1. Then by Pythagoras theorem the hypotenuse is 2
a c
2
O 1
C A
Fig I b 450
1
Where  is facing is opposite (BC) Applying SOH CAH TOA
Where right angle is facing, is the Hypotenuse (AB)
While the remaining side is the Adjacent (AC) Sin450 = Opp = 1
Also, where the capital letter A is facing is the small letter a; Hyp 2 rationalizing, we have = 2
same for B and C 2
Diagrammatically, we have Cos 450 = Adj = 1
Hyp 2 rationalizing, we have = 2
Hy

2
Opposite

p ot

Tan 450 = Opp = 1 = 1


he
nu

Adj 1
se

O
Adjacent Fig II Angles of 300 and 600
Here the mathematical assumption is that of an equilateral
The three basic trig. ratios are Sine, Cosine and Tangent triangle of sides 2 unit. Then on bisecting the triangle from
written in short form Sin, Cos and Tan respectively. the top angle to the base. We have 300 at top and 1 unit at the
We will apply SOH CAH TOA in Fig I above to base of the resulting two right – angled triangles.
define them as : Then by Pythagoras theorem the bisector (height) is √3
Applying SOH in defining Sin  represented below by Fig. 1 and Fig. II
Here S – sine, O - opposite, H – hypotenuse
Sin  = Opposite = a
0
60 0
300 30
2 2 2 2
Hypotenuse c
600 0

Applying CAH in defining Cos  60


0
600 60

Here C - cosine, A - adjacent, H – hypotenuse Fig I 2 1 1 FigII


In practice, we adopt one part of Fig II i.e.
Cos  = Adjacent = b
Hypotenuse c
300
Similarly we apply TOA to define Tan  2 3
Here T -tangent , O – opposite, A – adjacent,
600
Tan  = Opposite = a
1 Fig III
Adjacent b
The three basic trig ratios have reciprocals namely Applying SOH CAH TOA to fig III
Cosec (cosecant) = 1 Sin 300 = 1/2
Sine
Sec (secant) = 1 Cos 300 = 3
Cosine 2
Cot (cotangent) = 1 Tan 300 = 1 = 3
Tangent
3 3 by rationalization.
Deductively, we have that
Similarly,
Cosec  = 1 = Hypo = c
Sine  Opp a Sin 600 = 3 Cos 600 = 1

Sec  = 1 = Hypo = c 2 2
Cos Adj b Tan 600 = 3 = 3
and Cot  = 1 = Adj = b 1
Tan Opp a
108
Eg 1 Find the value of the unknown sides in the E.g 2. Without using tables evaluate
diagrams below: 1 + tan 600 tan 300
i ii tan 600 + tan 300
4 Solution
y
x
Substituting for the various special trig. ratio
45
0 0 = 1 + √3 (1/√3) = 2
45
8 √3 + 1/√3 √3 + 1/√3
iv We simplify as in ordinary fractions:
iii z 9 = 2 = 2 = 2 = 2√3 = √3
3 √3x√3 + 1 3+1 4 4 2
600 30 0
√3 √3 √3
k
Solution Derived trig. ratio problems
(i) x – Opp; 4 – Hypo (OH  SOH) 2000/23 Neco
Sin 450 = Opp = x If sin  = m2 – n2 find cot  if m  n and  is acute.
Hypo 4 m2 + n2
2 2
Substituting for Sin 450 from special trig. ratio A m +n B 2mn C 2mn
1 =x m –n
2 2 2
m +n 2
m2 – n2
2 4 D m2 – n2 E 2 m2n2
2mn m2 – n2
4 = x Rationalizing, 4 x √2 = x Solution
2 √2 √2 Cot  = 1 = Adj
4√2 = x i.e x = 2√2 tan Opp
2 But Sin  = m2 – n2 i.e Opp , Adj not given
(ii) 8 – Adj; y – Hypo (AH  CAH) m2 + n2 Hyp
D
ia
gra
m a
tic
ally
Cos 450 = Adj = 8 2
m 2
+n
2 2
Hypo y m-n
Substituting for Cos450 from special trig. ratio
1 = 8 A
dj
√2 y By Pythagoras theorem:
(m2 + n2 )2 = Adj2 + ( m2 – n2 )2
y = 8√2
( m + n ) – (m2 – n2 )2 = Adj2
2 2 2
(iii) 3 – opp; z – Hypo (OH  SOH) Applying difference of two squares to LHS
Sin 600 = opp = 3 (m2 + n2 + m2 – n2 ) [ m2 + n2 – (m2 – n2 ) ] = Adj2
Hypo z 2m2 (m2 + n2 – m2 + n2) = Adj2
Substituting for Sin 600 from special trig. ratio 2m2 (2n2) = Adj2
4m2n2 = Adj2
√3 = 3
2 z  Adj = 4m2n2 = 2mn
z √3 = 6 Thus, Cot  = 1 = Adj
z=6 rationalising , we have = 6 x √3 tan Opp
= 2mn (C)
√3 √3 x √3
m2 – n2
= 6√3 = 2√3
3 2000/33 Neco
(iv) 9 – Adj; k – Hypo (AH  CAH) If  is acute and cos = 1/2 , find the value of cot2
Cos 300 = Ad j = 9 A 1 B 1 /3 C 2 D 1/2 E 3
Hypo k 3 3
Substituting for Cos300 from special trig. ratio Solution
Cot2  = 1 2 i.e Adj 2
√3 = 9
tan Opp
2 k
1
k√3 = 18 But cos  = i.e Adj , Opp not given
2 Hyp
k = 18 Rationalising, k = 18√3 = 6√3
√3 3

109
D
ia
gra
m a
tic
ally 2000/1
Given that tan Φ = – 3
, 00 Φ1800, find cosΦ + sinΦ
O
pp 2 4
A – 3 /5 B – 1/5 C 1/5 D 4/5
1 Solution
By Pythagoras rule : For us to find cosΦ i.e CAH and sinΦ i.e SOH, we need to
22 = 12 + opp2 know Hypotenuse from tan Φ = – 3 ,
2 – 12 = opp2
2 4
(2–1) (2 + 1) = opp 2 D
ia
gra
m a
tic
ally
(1) (3) = opp2 H
yp
-3
Thus, opp = 3

Hence, cot2  = Adj 2


= 1 2
= 1 (B) 4
Opp 3 3 By Pythagoras rule :
Hyp2 = (–3) + 42
= 9 + 16
Hyp2 = 25
2003/21 Neco
Express in surd form by rationalizing; 1 + tan 600 Hyp = 25 i.e 5
1 – tan 600 Thus, cosΦ + sinΦ = 4 – 3
5 5
A 3 + 2 B 3– 2 C 1–2 3 = 1 (C)
D–1 + 2 E –2 – 3 5
Solution 2000/4
By special trig ratios; tan 600 = 3 Given that Sin = 3 Cos α = 3 ,Where  and α are acute,
5 5 find sin( + α)
Thus, 1 + tan 600 = 1 + 3 A1 B 24
/25 C 1/2 D – 7/25
1 – tan 600 1 – 3 Solution
Rationalizing Sin ( + α ) = sin cosα + cos sinα
We are given sin , but cos is not given
= 1+ 3 × (1+ 3 )
We are given Cosα , but Sinα is not given
(1– 3 )× (1+ 3 ) 3
Sin = 3 i.e SOH and cos α = i.e CAH
5 5
= 1 + 2 3 + 3 Diagramatically
1 – 3
= 4 + 2 3 5
5
–2 3 Opp
= –2 – 2 (E)
3
Adj
1996/4 1 By Pythagoras rule By Pythagoras rule
Express 2 – tan600 in surd from 52 = 32 + Adj2 52 = 32 + Opp2
A –2– 3 B 2– 3 C –2 + 3 5 – 3 = Adj
2 2 2
5 – 32 = Opp2
2
2
16 = Adj 16 = Opp2
D 2+ 3 E 2+ 3 Adj = 4 Opp = 4
7 4 4
Cos  = Sin α =
Solution 5 5
1 = 1 Sin ( + α ) = sin cosα + cos sinα
2 – tan600 2 – 3 = 3 ×3 + 4 × 4
Rationalizing = 2 + 3 5 5 5 5
= 9 + 16 = 1 (A)
(2 – 3 )( 2 + 3 ) 25 25
= 2 + 3
4 – 3 1991/41 UME
If cos x = a , find cosec x
= 2 + 3 (E) b

A. b B. b C. b D. b – a
b–a a b–a b

110
Solution = 4 2 + 42
cos x = a = 16 + 16
b = a RZ2 = 32
b RZ = 32 i.e 4 2
Applying CAH diagrammatically, it means
In PRZ, the relevant sides are;
PR opposite and RZ hypotenuse
b OH  SOH
Thus Sin 300 = opp i.e PR
hyp RZ
x 1 = PR
a 2 4 2
We need to know all the sides of  before we can compute
cosec x .By Pythagoras theorem, the opposite can be solved 4 2 = PR
for as 2
PR = 2 2 cm ( B)
Opposite squared = Hypotenuse squared – adjacent
squared 1989/42 UME
4 sin2 x – 3 = 0 , find x, if 0  x  900
= ( b )2 – ( a )2
Opposite squared = b – a A. 300 B. 450 C. 600 D. 900
Solution
Hence, opposite = b – a Thus,
4 sin2 x – 3 = 0
Cosec x = 1 = Hyp 4 sin2 x = 3
Sin x Opp sin2 x = 3
4 To clear square, take square root
= b sin x = 3
b-a 4
sin x =3
= b
2
b–a (C)
Since we have 3 and 2 implies trig ratios of 600 and 300
1995/27 PCE
R
30
2
3

P
Q 60
1
30
sin x = 3 when x = 600 ( C )
60 2
Y Z
i.e. sin 600 = 3 (C)
In the diagram above, calculate PR if YZ = 2cm.
2
A 2cm B2 2 cm C. 2 3 cm D. 2.5cm 1996/33 PCE Exercise 10.0
Solution If tan  = 5 where  is an acute angle, evaluate cos 
We can only find PR, If we know RZ with the aid of 12 Sin  + cos 
QZ in QYZ A. 17 B. 13 C. 12 D. 11
In QYZ, the relevant sides are; 13 17 17 17
YZ = 2cm Adjacent, and QZ = ? Hypotenuse
AH  CAH
Thus Cos 600 = Adj i.e 2
Hyp QZ
1 = 2
2 QZ 2009/3 Neco Exercise 10.2
Cross multiply, 2 tan 
QZ = 2 × 2 Evaluate given that  = 300
1  tan 2 
QZ = 4cm
But QZ = QR in QRZ from the diagram 3
A. B. 3 C. 2 D. 2 E.3
By Pythagoras theorem, 3
RZ2 = QR2 + QZ2

111
2000/34 PCE Exercise 10.3 Fig. I (1st quadrant between 00 and 900)
J N The resulting right – angled triangle has both x and y
positive, the hypotenuse that is 1 unit plays little or no role.
Here Sin , Cos  and Tan  are all positive.
m 60 0 M
Fig. II (2nd Quadrant between 900 and 1800).
10

The right – angled triangle here has negative x, which is the


30 0 adjacent side and a positive y - which is the opposite side.
K
L Hence, only
In the figure above, JKL = 300 , LMN = 600, Sin  = Sin (1800 - )
= +y = + y is positive
JLK = LJN = LMN = 900 and /JL/ = /LN/. If /JK/.
1
= 10m, find /MN/.
A.`10 3 m B. 10 6 m C. 5 6 m D. 5 3 m Others are negative in 2nd Quadrant i.e Cos  and Tan 
3 3 3 3
Fig III (3rd Quadrant between 1800 and 2700).
The right - angled triangle formed here has both x and y
negative.
Trigonometric ratios of angles Only Tan  = Tan ( - 1800)
between 00 and 3600 =-y
These type of trig ratios does not carry the condition -x
“acute angle” or 00    900. Apart from our = y is positive
knowledge of special angles of 450, 300 and 600 ; we are x
also required to know the required quadrant we are Others are negative 3rd quadrant i.e Cos  and Sin 
dealing with as discussed below:
Diagram of trigonometry ratio of angles Fig. IV (4th Quadrant between 2700 & 3600)
between 00 and 3600. Our right – angled triangle here has negative y which is the
opposite side but a positive x which is the adjacent side.
Y Hence only
Cos  = Cos (360 - )
1u

= +x
nit

+y
nit
1u

+y 1
= x is positive
180- Others are negative in 4th quadrant i.e Tan  and Sin 
x x
+x -x
1 quadrant
st
2 quadrant
nd Summarily
We use “Acts” to recall the mentioned properties of trig.
Y ratio between  0 and 3600 where
A is for all positive
-x x C – Cosine only positive
T – Tangent only positive
- 180 S – Sine only positive
-y Fig III
Written
nit S A
1u
nd st
3rd quadrant 2 Quad 1 Quad

T C
Y 3rd Quad 4th Quad

+x 1993/39 UME
360 -0 x If sin = cos , find between 00 and 3600
Fig iv A. 450, 2250 B. 1250 , 3150
0 0
-y C. 45 , 225 D. 1350, 2250
1u

Solution
ni t

Sin  = cos 
Then, divide both sides by cos 
The x and y axes divide the plane into four quadrants Sin  = cos 
as shown in Fig. I to Fig.IV. Any angle measured Cos  cos 
positively in an anticlockwise direction will be located
in any of these quadrants. Tan  = 1 = 1 and tan  is positive
1
112
Since we have 1 , then it must be 450 special trig. ratios Thus,  – 1800 = 300
1  = 30 + 180
If tan  = 1/1 then  = 450 Thus 450 in the first = 2100 (E)
quadrant 1995/7 (Nov)
Also tan is positive in the 3rd now with formula  – 1800, Find the values of x, for which 2cos x + 1 = 0, 0  x  3600
then A 00, 1800 B 900, 2700 C – 600 , 600
 – 180 = 45
0
D 600, 3000 E 1200, 2400
 = 45 + 1800 Solution
= 2250 2cos x + 1 =0
 = 450, 2250 ( A ) 2cos x =–1
1
Cos x = –
2002/34 PCE 2
The tangent of 1350 has the value x = cos–1(– 0.5)
A. – 1 B. – 1 C. 1 D. 1 But cos–1(0.5) is 600 and that cosine is negative
2 2 in the 2nd and 3rd quadrants with formula
Solution 180 –  and  – 180
Tan 1350; tan is negative in the second quadrant where Thus, x = 180 – 60 or  – 180 = 60
135 falls into. The formula here is 180 –  = 1200 or 2400 (E)
Thus, Tan 1350 = tan (180 – 1350)
= – tan 450 2
1996/1 (Nov)
From special angles 450 1 Find the values of x and y between 00 and 1800 which
tan 450 = 1 i.e 1 satisfy the simultaneous equations
45
1 sin( x + y ) = 2
Hence tan 1350 = – tan 450 1
2
= –1 ( A ) cos (2x) = –1
2
1990/37 PCE Solution
Evaluate cos 600 + sin 300
sin( x + y ) = 2
Sin 1500
2
A. 2 B. 1 C. – 2 D. – 1
x + y = Sin–1 2
2 2
2
Solution
Ordinarily, on seeing cos 600 and sin 300 the author By special trig ratio triangle 2 appears in 450
would have treated this question under acute angles trig ratios 2
but for the sin 1500 which is in a quadrant beyond 900. Thus, x + y = 450
S A Sin 1500 is the 2nd quadrant where sine is positive Also Cos(2x) = – 1
T C with the general formula 180 –  . Thus
0 2
Sin 1500 = + sin ( 1800 – 1500 ) 1
2x = cos–1 (– ) between 00 and 1800
= sin 300 2
We can now conveniently say; By special trig ratios triangles appears 1/2 in 600 Δ But for the minus
cos 600 + sin 300 = cos 600 + sin 300 2x = 180 – 600 (cosine is –ve in 2nd quadrant only here)
Sin 150 sin 300 2x = 120
=1 + 1 x = 600
2 2 From x + y = 45
1 y = 45 – 60
2 y = –15
=1  1 =1×2
2 1 2005/18
=2(A) If sinӨ = 1 , 00 Ө  4500 , find the possible values of Ө
2
1994/3 (Nov)
A 300 , 2100, 3900 B 600 , 3300, 3900
1
If sin = – and 0    2700 find  C 600, 2100, 4500 D 300, 1500, 3900
2 Solution
A 300 B 600 C 1200 D 1500 E 2100 Sin Ө = 1/2
Solution Ө = sin–1( 0.5)
Sin = – 0 .5 = 300 or ( 180 – 300 ) Sine is +ve in 2nd quad.
 = sin –1(–0.5) = 300 or 1500
But sin–1 0.5 is 300 and sin is negative in = 300 or 1500 or 30 + 3600 i.e Plus complete cycle
3rd and 4th quadrants but our restriction here is 3rd = 300 or 1500 or 3900 (D)
with formula  – 1800

113
1993/43 PCE Exercise 10.4 1978/44 Exercise 10.5
Evaluate tan 450 + cos 600 Evaluate without using tables sin (– 12900 )
Sin 1500 A. – 3 B. 3 C. 2 D. 1 E. – ½
A. – 3 B. 3 C. – ( 2 + 1 ) D 2 + 1 2 2 2

Trigonometry of Negative angles ( i.e –  ) 1987/38 Exercise 10.6


They are treated by adding 3600 to it or its multiples as the sine, cosine and tangent of 2100 are respectively
the case demands. We use the resulting angle to identify A. – 1, - 3, 3 B. - 1, - 3, 3
the quadrant it belongs. 2 2 3 2 2 3
C. 3, 3, 1 D. 3, 1, 1
1994/38 UME
2 3 2 2
What is the value of sin (– 6900 )
A. 3 B. – 3 C. – 1 D. 1
2 2 2 2
Solution Identities
– 6900 + 3600 = –3300 Complementary angles
So sin (–6900 ) = sin (– 3300 ) If  is an acute angle, then the angles  and 900 –  are
and 3300 falls into the 4th quadrant where sin is negative complementary which appear together in the right angled
with the general formula 3600 –  . Thus triangle below;
sin (– 6900 ) = sin (– 3300 ) B
= – sin ( 3600 – 3300 )
= – sin 300 From special trig ratios
=–1 (C) 90-
c
2 a

2001/35 PCE
What is the value of cos (– 8400 ) C A
b
A. – 3 B. – 1 C. 1 D. 3 1 sin  = a/c = cos (90 - )
2 2 2 2 cos  = b/c = sin (90 - )
Solution
– 8400 + 7200 = – 1200 ( 720 is 2 × 3600)
tan  = a/b = cot (90 - )
0 0
So cos (– 840 ) = cos (– 120 ) 2. Tan  = a/b = sin 
and 120 falls into 2nd quadrant where cosine is negative
with quadrant formula180 – . thus
cos 
Cos (– 8400 ) = cos (– 1200 ) Reason sin  =a b
= – cos ( 1800 – 1200 ) Cos  c c
= – cos 600 = a xc = a
=–1 (B) c b b
2
3. sin2 + cos2 = 1
2003/17 PCE Proof
Given that sin  = a – 1 and cos  = 1 – a , find tan  By Pythagoras rule
x x a 2 + b2 = c 2
A. 2450 B. 450 C. 3250 D. 1350 Divide through by c2
Solution a 2 + b2 = c 2
tan  = sin  c2 c2 c2
Cos 
=a–1 1–a But a/b = sin  hence a2/ c2 = sin2 
x x Similarly b2/c2 = cos2  but c2/c2 = 1
=a–1 × x Then substituting
x 1– a sin2  + cos2  = 1 or
=a–1 cos2  + sin2  = 1
1–a Since addition is commutative i.e. 2 + 3 = 3 + 2
We have to factored out – 1 in the denominator
= a–1 From identity 3 we can derive others
– 1( a – 1) i.e. cos2  + sin2  = 1
tan  = – 1 Divide through by cos2 
but tan  = 1 means  = 450 from trig ratios of
special angles Cos2  + sin2  = 1
Tan is only negative in the 2nd and 4th quadrants Cos2  Cos2  Cos2 
2nd quadrants 1800 – 450 = 1350 ( D ) 4 1 + tan2 = Sec2 

114
Similarly dividing identity 3 by sin2  Problems on identities I
Cos2  + Sin2  = 1
Sin2  Sin2  Sin2  1999/1b (Nov)
Simplify 1 – Cos2
5. cot2  + 1 = cosec2 
Sec 2 – 1
Solution
By trig identities
Complementary angles
1 – Cos2 = Sin2
2002/19 (Nov)
Given that sin(900 – 5) = cos (1800 –  ) find the value of .
Sec 2 – 1 tan2
A 150 B 22.50 C 300 D 450 = Sin
Solution tan
Sin (90–5) = cos (180 – ) = Sin ÷ Sin
Under complementary angle Cos
Sin(90 – 5 ) = Sin[ 90 – ( 180 –  ) = Sin × Cos
 90 –5 = 90 – 180 +  Sin
90 – 5 =  – 90 = Cos 
90 + 90 =  + 5 2003/20
180 = 6 Thus,  = 300 (C) Given that sin  = a – b what is 1 – cos2
a+b
2005/3a Neco
If sin(x – ) = cos(x + ), prove that tan x = 1 A a–b B a–b C a+b D a–b 2

Solution a+b a+b a–b a+b


Under complementary angle
sin(x – ) = cos(x + ), implies E a+b 2
sin (x – ) = sin [ 90 – ( x + ) ] a–b
Thus x –  = 90 – x –  Solution
By trig identities Sin2  + cos2  = 1
x + x –  +  = 90
2x = 90 Sin2  = 1 – cos2
x = 450 Taking the square root of both sides
Hence tan x  tan 450 = 1/1 = 1 QED Sin  = 1 – cos2
Thus , a–b = 1 – cos2 (A)
1981/6 UME a+b
If sin x0 = a. , what is sin ( 90 – x )0 ?
2000/7b Neco
b
If a cos2 + bsin2  = c show that tan2  = c – a
A. a B. 1 – a C. b2 – a2
b–c
b b b
Solution
tan2  = sin2
D. a 2 – b2 E. b2 – a2
b cos2
But in terms of a, b and c as given in the problem,
Solution
we apply trig identities
Recall that, under complementary angles
Sin2 + cos2 = 1
Cos  = sin ( 90 –  ). So that question is indirectly
asking us to find cos x. Cos2 = 1 – sin2
First, let us complete the trig ratio triangle. Thus, a(1 – sin2 ) + b sin2 = c
Applying SOH
b a – a sin2 + b sin2 = c
a
Sin x0 = a implies b sin2 – a sin2 = c – a
x sin2 (b – a) = c – a
0
B
sin2 = c – a
By Pythagoras rule the adjacent is equal to b2 – a2 b–a
Hence sin ( 90 – x )0  cos x0 = b2 – a2 ( C ) Also sin2 = 1 – cos2
b Thus, a cos2 + b(1 – cos2 ) = c
2010/9 Exercise 10.7 a cos2  + b – bcos2  = c
If sin x = – sin 700 , 00< x < 3600, determine the two
a cos2  – bcos2 = c – b
possible values of x
A 1100, 2500 B 1100, 2900 C 2000, 2500 D 2500, 2900 a cos2 (a – b ) = c – b
cos2 = c – b
2000/7a Exercise 10.8 a–b
Solve the equation Cos7 = Sin 5 Where 0    900 Hence tan  = c – a ÷ c – b
2

b–a a–b
115
= c–a × a–b sec2 300 = 1 = hyp 2
b–a c–b
= c–a × a–b cos2 300 adj
0
– (a – b ) c–b From special angles 30 trig ratio
30
= c–a 2
3
b–c QED cos 300 = 3
2 60
From above deduction 1
1982/20 UME
If sin  = m – n , find the value of 1 + tan2 . 2
sec2 300 = 2
m+n
A. 2 ( m2 + n2 ) B. 2 (m2 + n2) 3
m+n m+n
=4
C. m2 + n2 + 2mn D. m 2 + n2 + 2mn E. ( m + n )2 3
2mn 4mn 4mn Hence 1 + sec2 300 = 1 + 4 = 7 i.e 2 1/3 ( A )
3 3
Solution
Recall that 1 + tan2  = sec2  1985/38 UME
and sec  = 1 If cos  = 3 and  is less than 900, calculate
Cos  2
Then we are to find sec2  but first, lets complete our cot ( 90 –  )
trig ratio triangle.
sin2 
Sin  = m – n implies m+ n
m+n m-n
A. 4 3 B. 4 3 C. 3 D. 1 E. 2
Applying SOH 3 2 3 3
By Pythagoras rule, the adjacent side Solution
= (m + n ) – (m – n)
2 2 Recall that cot ( 90 –  ) = tan  and sin2 = 1 – cos2
Such terms are not in our options so we simplify further.
But the latter is longer to solve for
Hence cot ( 90 –  ) = tan 
= [m + n – ( m – n ) ] [ m + n + ( m – n)] Sin2  sin2
Let now complete the trig ratio triangle 2
= [m + n – m + n ] [ m + n + m – n] Cos  = 3  applying CAH
2
= ( 2n ) ( 2 m ) By Pythagoras rule, the 3

= 4mn opposite = 22 – ( 3 ) 2
i.e Adj = 2 mn
= 4-3
Hence sec2 = ( sec )2
2 2 = 1 i.e 1
= Hypo = m+n
Adj 2 mn So, tan = 1 and sin  = 1

= ( m + n )2 3 2
Hence tan  = 1  1 2
(2 mn )2
sin2 3 2
=(m+n)(m+n)
4 mn = 1  4
= m + n2 + 2mn (D )
2
3 1
4mn
= 4 rationalizing 4 3 ( A )
3 3
1985/34 UME
Without using tables, calculate the value of 1 + sec2 300
2008/2
A. 2 1/3 B. 2 C. 1 1/3 D. ¾ E. 3/2
3 15
Solution If sin A = and cos B = , where A is obtuse and
5 17
Recall that 1 + tan2  = sec2 
B is acute, find the value of cos(A + B)
tan2 = sec2 – 1 Solution
– tan  = 1 – sec2
2
cos(A + B) = cosA CosB – sinA sinB
The only way out here is direct substitution To proceed we first find cosA and sinB
116
By trig identities To recall the aforementioned formula easily, note the following
3 characteristics; the signs in the two sides of each of the sine formula
If sin A = it follows that Cos A = 1 sin 2 A are the same but that of the cosine is reverse.
5
The signs for the tangent formula are the same on both sides except
9 4 4 for the denominator that have the reverse signs.
Cos A = 1 =  i. e  ( since A is
25 5 5
Obtuse)
MULTIPLE ANGLES FORMULAE
15
If cos B = it follows that Sin B = 1 cos 2 B These angles are derived from compound angles where
17 A = B . Hence we have A + A instead of A + B
225 Sine formula
1 8 8 Sin 2A i.e sin ( A + A ) = sin A cos A + cos A sin A
Sin B = =  i. e  ( since B is Acute )
289 17 17 = 2 sin A cos A
1. sin 2A = 2 sin A cos A
cos(A + B) = cosA CosB – sinA sinB
Cosine formula
=–4 × 15 – 3 × 8 cos 2A i.e cos ( A + A ) = cos A cos A – sin A sin A
5 17 5 17 = cos2 A – sin2 A * *
= – 12 – 24
Substituting 1 – cos = sin2 A
2
17 85
= – 84 = cos2 A – ( 1 – cos2 A )
85 = cos2 A – 1 + cos2 A
= 2 cos2 A – 1
1990/39 UME Exercise 10.9 2 (a ) cos 2A = 2 cos2 A – 1 or
If cos2  = 12 , find 1 + cot2  Substituting 1 – sin2 A = cos2 A into **
13 cos 2A = ( 1 – sin2 A ) – sin2 A
A. 169 B. 25 C 169 D 144 = 1 – 2 sin 2 A
25 169 144 169
2002/31 UME Exercise 10.10 2 (b ) cos 2A = 1– 2 sin2 A
If tan  = 4 , calculate sin2  – cos2 
3 3. tan 2A i.e tan ( A + A ) = tan A + tan A
A. 16 B 24 C. 7 D. 9 1 – tan A tan A
25 25 25 25 = 2 tan A
1 – tan2 A
2006/9 UME Exercise 10.11
If tan  = 5 , find sin2  – cos2  MULTIPLE ANGLES FORMULAE II
4 Sin 3A = sin(A + 2A)
A. 41 B. 9 C. 1 D. 5 = sinA cos2A + cosAsin2A
9 41 4 = sinA(1 – 2 sin2A ) + cosA(2sinA cosA )
= sinA – 2 sin3A + 2sinA(cos2A )
1986/32 UME Exercise 10.12
= sinA – 2sin3A + 2sinA(1– sin2A)
If cos  = a, find 1 + tan2 
= sinA – 2sin3A + 2sinA– 2sin3A)
b
= 3sinA – 4sin3A
A. b2 B. a2 C. a2 + b2 D. 2a2 + b2
a2 b2 b2 – a 2 a 2 + b2 cos3A = cos(A + 2A)
= cos A cos2A – sinA sin2A
COMPOUND ANGLE FORMULAE = cosA(2cos2A – 1 ) – sinA (2SinACosA)
( 1) Sin( A  B ) = sin A cos B  cos A sin B = cosA(2cos2A – 1 ) –2CosA (Sin2A)
Broken down as = 2cos3A – cosA – 2cosA(1 – cos2A )
Sin( A + B ) = sin A cos B + cos A sin B and = 4cos3A – 3cosA
Sin( A – B ) = sin A cos B – cos A sin B
tan 3A = tan ( A + 2A)
( 2 ) Cos ( A  B ) = cos A cos B  sin A sin B tan A  tan 2 A
=
Broken down as 1  tan A tan 2 A
Cos ( A + B ) = cos A cos B – sin A sin B and 2 tan A
Cos ( A – B ) = cos A cos B + sin A sin B. tan A 
= 1  tan 2A
( 3) tan ( A  B ) = tan A  tan B 2 tan A
1  tan A .
1  tan A tan B 1  tan 2A
Broken down as tan A(1  tan 2A)  2 tan A
tan ( A + B ) = tan A + tan B =
1 – tan A tan B and (1  tan 2A)  2 tan 2A
tan ( A – B ) = tan A – tan B
= 3 tan A  tan
3
A
1 + tan A tan B 1  3 tan 2A

117
Problems on identities II
= 2 1 × 3 ÷ (– 2 ) 1 × 1
2005/9i Neco 2 2 2 2
Express Cos2A and SinAsin2A in terms of cosA
= 3 ÷ –1
Solution
By trig identities 2 2
Cos2A = cosA cosA – SinAsinA
= 3 × – 2
= cos2A – Sin2A
= cos2A – (1 – cos2A ) 2 1
= cos2A – 1 + cos2A
= – 3 ( A)
= 2cos2A –1
Sin A sin 2A = sinA(sinAcosA + cosA sin A )
= sinA(2sinAcosA) 1994/11
= 2sin2A cosA Given that sin(A + B) = sinA cosB + cosA sinB
= 2(1– cos2A) cosA (a) Show that: ( i ) sin 3x = 3sin x – 4sin3 x
( ii ) Where x = 180 , sin3x = cos2x
2007/45 Neco Solution
Given that sin(A– B) = sinA cosB – cosA sinB. (a) ( i ) Sin3x = (x + 2x)
Find the value of sin 150 = sinx cos2x + cosx sin2x
A 6 – 2 2 B 2 – 6 C 2 6 – 2 Simplifying further for cos2x and sin2x
4 8 4 = sinx(1–2sin2x) + cosx(2sin x cos x )
= sinx – 2sin3x + 2sinx(cos2 x )
D 6 – 2 E 6 – 2 = sinx – 2sin3x + 2sinx (1–sin2x)
8 4 = sinx –2sin3x + 2sinx – 2sin3x
Solution = 3sinx – 4sin3x QED
Sin150  Sin (450–300 ) = sin45 cos30 – cos45 sin30
(a) (ii) When x = 18 , show that sin 3x = cos 2x
= 1 3 1 1
Under complementry angle
2 2 2 2 sin3x = sin (90 – 2x)
 3x = 90 – 2x
= 3 1 3× 18 = 90 – (2 × 18) QED
2 2 2 2 1995/13 (Nov)
Find the value of Sin 750 – Sin 150
= 3 – 1 Sin 750 + Sin 150
2 2 A 3 B 1 C 32 D  3 E 1 3

Rationalizing = ( 3 – 1 ) ( 2 2 ) Solution
4× 2 Sin 750 – Sin 150 = Sin(45 + 30) – Sin(45 – 30)
Sin 750 + Sin 150 Sin(45 + 30) + Sin(45 – 30)
= 2 6 – 2 2 = Sin45Cos30 + Cos45Sin30 – (Sin45Cos30 – Cos45Sin30)
8 Sin45Cos30 + Cos45Sin30 + Sin45Cos30 – Cos45Sin30
2 will cancel out
= 2 Cos 45 Sin 30
= 6 – 2 (E) 2 Sin 45 Cos 30
4 = 1 × 1 ÷ 1 × 3
2 2 2 2
1992 /18 (Nov)
Find, without using mathematical table, the value of = 1 × 2 2
Cos 750 + Cos 150
2 2 3
Cos 750 – Cos 150
= 1 (E) rationalizing 3
A– 3 B– 1 C 1 D 1 E 3
3 3
3 3
Solution 2000/1a
Cos 750 + Cos 150 = Cos(45 + 30) + Cos(45 – 30) Find in surd form, the value of cos150
Cos 750 – Cos 150 Cos(45 + 30) – Cos(45 – 30) Solution
= Cos45Cos30 – Sin 45Sin30 + Cos45Cos30 + Sin45Sin30 Cos150  cos (45 – 30) = cos45 cos30 + sin45 sin30
Cos45Cos30 – Sin 45Sin30 – ( Cos45Cos30 + Sin45Sin30)
= 1 × 3 + 1 × 1
= 2Cos45 cos30
– Sin45 Sin30 2 2 2 2
118
Sin450 cos150 – cos450 sin150
= 3 + 1 = sin45 cos(45–30) – cos45 sin(45–30)
2 2 2 2 = sin45[Cos45Cos30 + Sin45Sin30]– cos45[Sin45Cos30 + Cos45Sin30]
= sin45cos45cos30 + (sin45)2sin30 – sin45cos45cos30 + (cos45)2sin30
= 3 + 1 = (sin45)2 sin30 + (cos45)2 sin30
2 2 = 1 2
× 1 + 1 2
× 1
Rationalizing = ( 3 + 1) 2 2 2 2 2 2
( 2 2 ) (2 2 )
= 1 + 1 i.e 1 (B)
4 4 2
= 2( 6 + 2 ) = 1 ( 6 + 2 )
4 × 2 4 2002/24
Express 2cos (60 +)0 in terms of cos and sin
2000/15
Simplify 1 – tan2x A cos + 3 sin B 3 cos  – sin 
1 + tan2x C cos – 3 sin D 3 cos + sin
A sin 2x B 0 C cos2x D –1 Solution
Solution 2Cos(60 + ) = 2(cos 60 cos – sin 60 sin)
1 – tan2x = 1 – (sec2x –1) = 2 1 cos  – 3 sin
1 + tan2x 1 + (sec2x –1) 2 2
= 2 – sec2x
= cos – 3 sin (C)
sec2x
= 2 – 1
sec2x 2003/7 (Nov)
Recall that sec x is 1 Thus 1 = cos x Simplify Cos2 – 1
Cos x sec x Sin 2
= 2 cos2x – 1 A – tan B – cot C tan D cot 
By trig identities Solution
= cos2x (C) Cos2 – 1 = 2 Cos2 – 1 – 1
Sin 2 2 sin cos
2000/9a (Nov) = 2 Cos2 – 2
Given that tan = 4 , 00    3600, 2 sin cos
3
= Cos2 – 1
find the value of 3tan + tan2 sin cos
Solution By identities sin  + cos2 = 1  cos2 –1 = – sin2
2
3tan + tan2 = 3tan + tan + tan
= – sin2
1 – tan tan
sin cos
= 3tan + 2tan
= – sin
1 – tan2
cos
4  4  = – tan (A)
=3× +2 × 1   2 
4
3
3  3 
2005/2a
8  16 
= 4 +  1   Express cos(2 )  1 in terms of tanӨ
3  9 cos(2 )  1

= 20 ÷ – 7 Solution
3 9 sin 
We know that tan  = we work with this in mind,
= 20 × – 9 cos 
3 7 Cos2Ө = 1 – 2sin2Ө
= – 60 i.e – 8 4/7 Cos2Ө – 1 = 1– 2sin2Ө – 1
7 = – 2sin 2Ө
1996/2 (Nov) Similarly, Cos(2Ө) = 2 cos2Ө – 1
Evaluate, without using tables, sin450 cos150 – cos45 sin150 Cos (2Ө) + 1 = 2cos2Ө – 1 + 1
= 2cos2Ө
A 3 B 1 C 3 D 1 E 3 +1
Thus cos(2 )  1 =  2 sin2 
2
4 2 2
Solution cos(2 )  1 2 cos 
= – tan2 Ө
119
2006/10 b 2010/2 Exercise 10.14
  Find the value of cos (600 + 450) leaving your
Given that A = 450 and B = 300 answer in surd form.
Sin (A + B) = sin A cos B + sin B cosA 6 2 3 6 2 6
and Cos (A+B ) = cosA cosB – sin A sin B A B C D 3 6
4 4 4 4
( i ) Show that :
6 2 6 2 2003/18 Exercise 10.15
Sin 150 = and Cos 150 = Evaluate sin150.
4 4
(ii) Hence find tan 150 A
6 2
B
3 1
C
 3 1  D
6 2
Solution 4 2 2 2 2
Sin150  sin (45–30 ) = sin45 cos 30 – cos45 sin30
2006/33 Exercise 10.16
1 × 3 1 1 Find, in surd, the value of Cos 1650
 ×
   

2 2 2 1 1
2 A 6 2 B 6 2
3 1 4 4
 
2 2 2 2 C 
1
4
 6 2  D
1
4

6 2 
3 1
 2009/17 ( Nov) Exercise 10.17
2 2 Given that tan p = x, where 00 < p < 900, find the
Rationalizing value of sin 2p.
 3  1 2 2 
2 6  2 = 6 2 A
x 1  x2
B 2 C
x
D
2x
=
2 2 2 2  =
4 2 4
QED
x 2 1 x 1 x 2 1 x 2 1
Cos 15  Cos (45 – 30 ) = Cos 45cos30 + sin45 sin30
2009/5 ( Nov) Exercise 10.18
3 1 Evaluate sin 150 cos 1350, leaving your answer
= 1 × + 1 × in surd form.
2 2 2 2
1 1
A- (1 + 3 ) B (1 - 3 )
3 1 4 4
= 
2 2 2 2 1 1
C ( 3 - 1) D (1 + 3 )
3 1 4 4
=
2 2
Rationalizing Identities and equations
 3  1 2 2 2  6  2  2003/2 Neco (Dec)
Solve the following equation for values of 
=
2 2  2 2  =
42 between 00 and 3600 : 2tan2 – 7sec + 8 = 0
Solution
6  2 2tan2 – 7sec + 8 = 0
=
4 But by trig identities 1 + tan2 = sec2
sin 15 tan2 = sec2 – 1
(ii) tan 15 =
cos 15 Substituting
2tan2 – 7sec + 8 = 0 becomes
6  2 6 2
=  2(sec2 – 1) – 7sec + 8 = 0
4 4 2sec2  – 2 – 7sec + 8 = 0
6 2 4 2sec2 – 7sec + 6 = 0
=  Put sec = p
4 6 2 2p2 – 7p + 6 = 0
6 2 2p2 – 4p – 3p + 6 = 0
= 2p(p–2) –3(p–2 ) = 0
6 2
(p – 2 )(2p – 3 ) = 0
Rationalizing
p = 2 or 3/2
 6 2 6 2  = 8  2 12 = 8  4 3 Thus sec = 2 or 3/2
=
 6 2 6 2  4 4 But sec  = 1
cos
= 2 3  Cos  = 1/2 or 2/3
 = cos 0.5 or cos–1 0.6667
–1

= 600 or 480
120
1978/32 UME Sine is negative in 3rd and 4th quad
If sec2  + tan2 = 3, then the angle  is equal to x = 900 or (300 + 1800 or 360 – 30 )
A. 300 B. 450 C. 600 D. 900 E. 1050 = 900 or 2100 or 3300
Analysis and solution
1
Recall that 1 + tan2  = sec2  Example IE 4 Solve tan x + = –2
Sec2  + tan2  = 3 becomes tan x
1 + tan2  + tan2  = 3 Solution
1 + 2 tan2  = 3 Applying simple fraction principle
2 tan2 = 3 – 1 1
tan x + = – 2 becomes
2 tan2  = 2 tan x
tan2  = 2 tan 2x  1
 2
2 tan x
tan2  = 1 tan2 x + 1 = – 2tan x
( tan  )2 = 1 2
tan x + 2tan x + 1 = 0
To clear square, we take the square root of both sides Let tan x = P
tan  = 1 P2 + 2P + 1 = 0
tan  = 1 Factorising
This is only possible in the 450 triangle for special trig ratio. P2 + P + P + 1 = 0
i.e.  = 450 ( B ) P(P + 1 ) + 1(P + 1 ) = 0
(P + 1 )(P + 1 ) = 0
P = –1 twice
Example IE 1 Solve sin2x + 2cosx = 0 tan x = –1
Solution x = tan–1x (–1)
Sin2x + 2cosx = 0  2sin x cos x +2cosx = 0 Tangent is negative in 2nd and 4th quad
2cosx (sinx + 1 ) = 0 = 180 – 45 or 360 – 45 tangent
2cosx = 0 or sinx + 1 = 0 = 1350 or 3150
Since 0 2 is zero
Cosx = 0 or sinx = – 1 2002/8
x = cos–1 0 or sin–1(–1) 3
Given that tan(x – y) = and tan x = 2, find the value of tan y
Sine is negative in 3rd and 4th quad 4
x = 900 or (900 + 1800 or 360 – 90 ) A 1 /2 B 2 C 5 /2 D 11
/2
= 900 or 2700 Solution
Tan ( x – y) = tan x – tan y
Example IE 2 Solve cos2 x – 1 = 0 1 + tan x tan y
Solution Substituting for the given value
Let Cos x = p then 3 = 2 – tan y
Cos2x –1 = 0  p2 – 1 = 0 4 1 + 2tan y
(p + 1) (p –1) = 0 3(1 + 2tan y) = 4(2 – tan y)
p= ±1 3 + 6tan y = 8 – 4tan y
Thus, cosx = 1 or cosx = – 1 6tan y + 4tan y = 8 – 3
x = cos –11 or cos–1 (–1) 10tan y = 5
Cosine is negative in 2nd and 3rd quad tan y = 5/10 i.e 1/2 (A)
x = 00 or (1800 – 00 or 0 + 180 )
= 00 or 1800
2003/2 (Nov)
Example IE 3 Solve cos2x + sin x = 0 1
Given that tan ( x – y) = 
2 and tan x = 
Solution 3 2
Applying trig identities Calculate the value of : (a) tan y ;
cos2x + sinx = 0 becomes (b) ( x + y ) for 00 x + y < 3600 correct to the nearest degrees
(1– 2sin2x ) + sin x = 0 Solution
Rearranging by multiplying through by – 1 (a) tan ( x – y ) = tan x – tan y
2sin2x – sinx – 1 = 0 1 + tan x tan y
Let Sinx = p Substituting for the given values
2P2 – P – 1 = 0  12  tan y
2P – 2P + P – 1 = 0
2

2 =
2P(P–1) + 1(P–1) = 0 3 1  12 tan y
(P–1) (2P + 1 ) = 0 Cross multiply
P = 1 or – 1 2 –2( 1 – ½ tan y) = 3(– ½ –tan y)
Thus, sin x = 1 or – 0.5 –2 + tan y = – 3/2 – 3tan y
x = sin–11 or sin–1 (– 0.5) tan y + 3tan y = – 3/2 + 2
121
1 IE 5 Exercise 10.19
4 tan y =
2 Solve the equation : 3sinx – 4sin3x = 1 – 2sin2x
Multiply through by 1/4
2003/33 Exercise 10.20
tan y = 1/8
Solve the equation 1– cos2x = 0, for 00≤ x ≤ 2700.
(b) If tan y = 1/8 and tan x = – 1/2 A x = 00 or 2700 B x = 00 or 1800
–1
y = tan (0.125) x = tan– 1 (- 0.5) C x =180 0
D x = 2700
0
y = 7.125
But tan– 1 (0.5) is 270 to the nearest degree
tan is negative in 2nd and 4th quadrants with formula Half angles
180 -  and 360 -  We put A = B = 1
2 Ө in our former expressions
Thus x = 180 –270 or 360 – 27
= 1530 or 3330 Sin 2Ө = 2 SinӨ Cos Ө becomes
sinӨ = 2sin 1 2 Ө cos 1 2 Ө
Thus (x + y ) = 153 + 7 or 333 + 7
= 1600 or 3400
Cos 2Ө = 1 – 2 sin2 Ө becomes
2000/9b (Nov)
Cos Ө = 1 – 2 sin2 1 2 Ө or = 2cos2 1 Ө–1
If cos(2k + Ө) + cos(2k– Ө) = – 3 CosӨ, find the 2

possible values of the constant k (Note: cos2A= 2cos2A– 1)


tan 2Ө becomes
Solution
tan Ө = 2 tan 2 2 
1
LHS Cos(2k + Ө) + Cos(2k– Ө)
= Cos2kCosӨ - Sin2kSinӨ + Cos2kCosӨ + Sin2kSinӨ 1  tan 12 
= 2Cos2kCosӨ 2t
tanӨ = note this expression
Substitute for cos2k = 2cos2k– 1 1 t 2
= 2(2cos2 k – 1 ) CosӨ Where t = tan 1 2 Ө
Thus, Cos(2k + Ө) + Cos(2k – Ө) becomes Other expressions are
2(2cos2 k – 1 ) CosӨ = – 3 cosӨ 1  t2
2t
CosӨ will cancel out sinӨ = and cos Ө =
2(2cos2 k – 1) = – 3 1 t 2 1  t2

 3 Example HA 1 Solve 4 Cosx – 3Sinx = 2


2cos2 k – 1 =
2 for values of x between 00 and 7200 correct to 0.10
Solution
3 2 3
2cos2k = 1  i.e Let t = tan 1 2 x
2 2 2t
Then cos x = 1 t 2 and sin x =
2

Multiply through by 1
2 1 t 1 t 2
2 3 4 Cosx – 3Sinx = 2 becomes
cos2k =  2t 
4 
 1 t 2
4   – 3  2 
2
1
 1 t
2
 1 t 
Cos k =  2  3 
2

 4  To clear fractions multiply through by LCM (1 + t2 )


 
4(1 – t2 ) – 3(2t) = 2 (1 + t2 )
cos 1 ( 0.38268) 2 i.e 680
1
k= 4 – 4t2 –6t = 2 + 2t2
2005/ 9iii Neco 0 = –2 + 6t + 6t2
Solve the equation i.e 3t2 + 3t – 1= 0
3cos3x0 – 4cos2x0 cosx0 + 3cosx0 = 0,  3  9  12  3  21
giving the values of x in the range 00  x  3600 t= = = -1.2633 or 0.2633
Solution 6 6
Since our given problem is in cos x at its lowest We were given that 00  x  720
value, we will try to simplify to it or its powers using Hence 00  1 2 x  3600
trig identities Thus, tan 1 2 x = – 1. 2633 or tan 1 2 x = 0 .2633
3cos3x0 – 4cos2x0 cosx0 + 3cosx0 = 0 Either 1 2 x0 = 180 – 51.640 or 360 – 51.640
3(4cos3x0 –3cox0) – 4(2cos2x0 – 1)( cosx0 ) + 3cosx0 = 0
12cos3x0 – 9cosx0 – 8cos3 x0 + 4cosx0 + 3 cosx0 = 0 128.360 or 308. 360
4cos3x0 – 2cosx0 = 0 OR tan 1 2 x = 14.75 or 180 + 14.75
2cosx0(cos2x0 – 1) = 0 = 14.75 or 194.75
2cosx0 = 0 or cos2 x0 – 1 = 0 Starting from the smallest, and multiplying through by 2
1
cosx0 = 0 ( 0/2 is zero) or cosx0 = ±1 ( 1 2 is 1) x = 29.50, 256.70, 389.50, 616.70 to 0.10
x0 = cos–1 0 or cos–1 1 or cos–1 – 1
= 900 or 00 or 1800
122
2005/35 Neco Characteristics of a Sine curve
Express tan 22.50 in surd form When plotting or sketching a sine curve on the positive x – axis
only (fig 1), it starts from the origin (0 0, 0) and reach its peak of +1
A  2 2 B 1– 2 C  2 1 at 900 (/ 2) then falls to 0 at x = 1800 ( ) from this point, it
descends downward to a lowest point of – 1 at
D  2 1 E  2 2 x = 2700 ( 3/2). It then attains 0 at x = 3600. The cycle continues
Solution in that order; other forms of sine curves are multiples of this basic
one.
2t
tan Ө = where t = tan 1 2 Ө
1 t 2 Sine graph for AMPLITUDE multiples
i.e t = tan 22.5 thus tan Ө = 450 When y = sin 
2t Actually the amplitude here is the coefficient of sin, which is 1.
tan450 = recall that tan 450 = 1 Thus, the highest and lowest points of the graph
1 t 2 are + 1 and –1 respectively
2t
1 = When y = 2 sin .
1 t 2 The amplitude (coefficient of sin ) is 2
1– t2 = 2t Thus, the highest and lowest points are +2 and – 2 respectively.
1– t2 – 2t = 0 Similarly, when y = 3 sin .
Multiply through by – 1 The amplitude (coefficient of sin ) is 3
t2 + 2t – 1 = 0 Thus, the highest and lowest points are +3 and – 3 respectively.
This quadratic equation is not factorizable so, we apply the general formula y
+1 y = sin
2 2 2  4 1  (1)
t =
2 1 
00 1800 3600
2 44 -1
=
2 y
2 8 
2 2 2 +2 y = 2sin
= = =  1  2 (C)
2 2 
00 1800 3600
Trigonometric graph
-2
Trigonometric functions like others have their graphical
representation, which is of great importance to scientists.
Thus, it is one of the basic knowledge required in y
mathematics. The sine, cosine and tangent can be represented
+3 y = 3sin
graphically in either degrees or radians as units of
measurements. Though degree is often used.

Sine Graph 00 1800 3600
y -3
+1 Fig I

Sine graph for PERIODICITY Multiples


x
The periodicity of trigonometry graph is a term for the
00 900 1800 2700 3600 repetition of the graph pattern at intervals; or completing of
the graph cycle. Usually within 3600
-1 When y = sin 
Its periodicity is 3600 . This is the basic sine graph shown
y Fig II earlier.
+1 When y = sin2
Its periodicity is 1800 – since two multiplies 1800 to give
x 3600. The graph pattern is shown below:
.
-1800 -900 00 900 1800 2700 3600 y
–1 +1 y = sin2
-
y 
+1 Fig III 00 900 1800
-1

x
0 /2  3/2 2
–1

123
When y = sin 3  y = -cos
The periodicity is 1200 – since three multiplies 1200 to give +1
3600.. The graph pattern is shown below.
.
y
+1 y = sin 3  900 2700 

 -1
00 600 1200
-1 Cosine Graph for PERIODICITY Multiples
The same principle applies as in the case of sine function.
Cosine Graph
+1 y = cos2  +1 y = cos3 
y
+1
 
Fig I 450 1350 300 900
-1 -1
x
00 900 1800 2700
Tangent graph
y
-1
y
+1
00 1800 3600
Fig II

x
0 /2  3/2
The curve of tangent function is not a continuous one like that of
-1 sine or cosine. It appears in three distinct curves as shown above
y
1996/6
+1 Fig III
The sketch represents the function
y
3
2
1
x
x 0
-1 180 360 540 72
0
90
0 0
108 126
0 0
144 1620 180
0

-270 -90 90 270 -2


-3
-1 A y = 3cos 5x B y = sin 5x C y = 3sin 5x D y = 3cos x E y = 3sin x
Solution
Characteristic of a cosine curve First, we look at the starting point of the graph. It started from
Sketching cosine curve on the positive x – axis as shown in fig,
origin i.e 0 is for Sine graph.
I and II. It starts from its peak of +1 (00,1) and then falls to zero
Next, we look at the amplitude (y-axis), it’s highest point is + 3
at x = 900 (/2). From this point it continues to descend till it and – 3 (i.e amplitude of 3 )
get to its lowest point of –1 at x = 1800 () . It then starts Next, we look at the periodicity (x- axis) i.e 360 an compared to
climbing to zero at 1800 which is the half cycle of sine or 720 as compared to 3600
x = 2700 (3/2). which is the first full cycle of sine graph. In either case we get 5 as
The cycle continues in this pattern. the period i.e 180 36 or 360 72
Other forms of cosine graph are either multiples of its
amplitude or periodicity. Thus, the function is y = (Amplitude) sine ( Period )x
y = 3sin5x (C)
Cosine graph for AMPLITUDE multiples 2004/26 Exercise 10.21
The principles applied in sine graph multiples are the same Y
2
here. Thus, we have: y
+2 y = 2cos 
1

X
0 0
00 900 2700  300 600 900 120 0 150 0 180
-2
-1
y
+3 y = 3cos
-2
The sketch above represents the graph of
00 900 2700  A y = 2sin x B y = 2cos x C y = cos 2x D y = sin 2 x
-3
124
GRAPH PLOTTING IN TRIGONOMETRY  00 300 600 900 1200 1500 1800
The methods applied in table of trigonometry curves are y -1. 0 0 1. 0
similar to that quadratic of graph.
E.g 1 (a) Copy and complete the table below
Analysis and solution
for the relation y = 2 sin3x – 1
10 (a) Table for y = 3 sin2 - cos
X 00 300 600 900 1200 1500 1800  00 300 600 900 1200 1500 1800
y =2 sin3x 3sin2 0 2.6 2.6 0 -2.6 -2.6 0
-1 -1 1 - cos -1.0 - -0.5 0 0.5 0.9 1.0
0.9
(b).Using a scale of 2cm to 300 on the x - axis and y -1.0 1.7 2.1 0 -2.1 -1.7 1.0
1 cm to 1 unit on the y - axis, draw the graph of
y = 2 sin 3x –1 for 00  x  1800 10 (b) Graph is shown on page 126
(c)On the same axes draw the graph of y = 1 (x –360) 10 (c) (i) These are the points where the curve cut
180
(d). Use your graph to find :
the  – axis 9 0, 900 and 1700
i. values of x for which 2 sin3x – 1 = 0 (ii) Highest point on the curve is 2. 3 as
ii. Roots of the equation 2 sin 3x – x + 1 = 0 shown by dotted line on the graph
180
Analysis and Solution
2 (a) Applying tabular format as in quadratic , we have

Table for y = 2 sin 3x – 1


x 00 300 600 900 1200 1500 1800
2 sin 3x 0 2 0 -2 0 2 0
-1 -1 -1 -1 -1 -1 -1 -1
y -1 1 -1 -3 -1 1 -1 2010/10 Exercise 10.22
(a) copy and complete the table for the relation
2 (b) Graph is shown on page 126 y = 2cosx + 3sinx for 00  x  3600
2 (c) The relation y = 1 ( x – 360 )
180 x 00 300 600 900 1200 1500
is a linear function since the highest degree of x is one. y 2.00 3.23 1.60
Thus preparing a three points table for the function; we
have
1800 2100 2400 2700 3000 3300 3600
y = 1 (x –360) i.e y = x – 2
180 180 -3.23
Table for y = x /180 – 2
(b) Using a scale of 2cm to 300 on the x-axis and 2cm to one
x 0 90 180
unit on the y-axis, draw the graph of y = 2cosx + 3sinx
x/180 0 0. 5 1
-2 -2 -2. 0 - 2. 0
for 00  x  3600
y -2. 0 -1. 5 -1 (c) From your graph , find the
( i ) maximum value of y, correct to two decimal place
2 (d) i. These are the points where the curve cut the ( ii ) solution of the equation 23 cosx + sinx = 56
x – axis 90, 500, 1290 and 1700
ii.These are the points of intersection between the linear
line and the curve; traced to the x – axis 660 , 1170
and 1800 Since the given equation is a combination of
the two functions
1993/10 Ordinary maths
Copy and complete the following table of values
for y = 3sin 2 – cos
(b).Using a scale of 2cm to 300 on the  - axis and
2cm to 1 unit on the y – axis, draw the graph of
y = 3 sin 2 – cos  for 00  0 1800
(c).Use your graph to find the:
(i) Solution of the equation 3sin 2 - cos  = 0
correct to the nearest degree;
(ii) Maximum values of y, correct to one decimal
place
125
2 y

y=2sin3x - 1
1

x
0
0 30 60 90 120 150 180 210

-1

-2
x
y  2
180

-3

-4

3
y
y = 3sin 2 – cos
2


0
0 30 60 90 120 150 180 210

-1

-2

-3

126
Chapter Eleven (a + x)5 = 1a5x0 + 5a4x1 + 10a3x2 + 10a2x3 + 5a1x4 + 1a0x5
Pascal triangle and binomial theorem Here (1.002)5 is same as (1 + 0.002)5
By ordinary multiplication of algebraic expansions we have that By the expansion (a + x)5 where a is 1 and x is 0.002
(a + x)0 = 1 = 1(1)5(0.002)0 + 5(1)4(0.002)1 + 10(1)3(0.002)2
(a + x)2 = a2 + 2ax + x2
+ 10(1)2(0.002)3 + 5(1)1(0.002)4 + 1(1)0(0.002)5
(a + x)3 = a3 + 3a2x + 3ax2 + x3
The following features are possessed by the expressions above =1+5(0.002) +10(0.002)2 +10(0.002)3 +5(0.002)4+(0.002)5
n
(i) The no. of terms in the expansion of (a + x) is n = 1 + 0.01 + 0.00004 + 0.000,000,08 + 0.000,000,000,08
(ii) The highest degree of each term in the expansion
+ 0.000,000,000,000,032
of (a + x)n is n
(iii) The coefficient of the term in the expansion of = The last two terms will not matter to 6 d.p.
(a + x)n are given by the nth line of Pascal’s triangle = 1 + 0.01 + 0.00004 + 0.000,000, 08
= 1.010040 (C)
PASCAL’S TRIANGLE
1
Example PT2
1 1
1 2 1 Without using tables, evaluate 2 5   – 2 5 
3 3

1 3 3 1 Solution
1 4 6 4 1 First step, we expand each term with the help of Pascal’s triangle
1 5 10 10 5 1 1
1 6 15 20 15 6 1 1 1
1 7 21 35 35 21 7 1 1 2 1
1 3 3 1
Each line of Pascal’ Triangle starts and ends with 1,
terms in between are obtained by adding together the 2 5  3
= 1× 23  5 0
+ 3×22  5 1
+ 3×21  5 2

terms on the either side of it in the row above.  5 + 1×20


3

= 8 + 12 5 + 6  5  +  5 
2 3
Thus applying row 4 to expansion of (a + x)4
Expansion a4 x0 + a3 x1 + a2 x2 + a1 x3 + a0x4
Coefficient 1 4 6 4
= a4 + 4a3x + 6a2x2 + 4ax3 + x4
1
2 5  = 1×2  5  + 3×2  5  +
3
3
0
2
1

There are two ideas to be deducted from the above i.e. 3×2  5  + 1×2  5 
2 3
1 0
in any binomial expansion while the power of the first
Take note of powers of  5 ; odd gives minus, even gives plus
term of “a” starts from the given power and then
subsequently decreasing, the other term x starts from
the power zero increasing upwards till it attains the
= 8 – 12 5 + 6  5 –  5
2 3

given power. Of course the coefficients of each term Thus, 2 5  – 2 5  = 24
3 3
 5
5 +2
3

5 + 2  5  5
combined are supplied by Pascal triangle.
2
Thus applying row 5 to expansion of (a + x)5 = 24
Expansion a5x0 + a4x1 + a3x2 + a2x3 + a1x4 + a0x5 = 24 5 + 10 5
Coefficient 1 5 10 10 5 1
= a5 + 5a4x + 10a3x2 + 10a2x3 + 5ax4 + x5 = 34 5
Coefficient 5C0 5C1 5
C2 5
C3 5
C4 5C5 Example PT3
Generally, it can be deducted that
(a + x)n = an + nC1an– 1x + nC2an– 2x2 + … + nCr an– r xr
Without using tables, evaluate 1 3 – 1 3   4

4

Solution
+ … + nCn x n First step: we expand each term with the help of Pascal’s triangle
This is binomial theorem for all values of a and x, 1
provided n is a positive integer. 1 1
1 2 1
Problems on Pascal’s triangle 1 3 3 1
2003 /11 Neco 1 4 6 4 1
Use Pascal’s triangle to evaluate (1.002)5
correct to six places of decimal. 1 3  4
= 1×14  3 0
 3
+ 4×13  3 1
+ 6×12
2

+ 4 ×1  3  + 1× 1  3 
3 4
A 1.000100 B 1.001004 C 1.010040 1 0
D 1.100040 E 2.010440
= 1 + 4 3 + 6  3 + 4  3 +  3
2 3 4
Solution
Use of Pascal’s triangle here is to help in placing of coefficients

1
1
1
1 3  = 1×1  3  + 4×1  3  + 6×1  3 
4
4
0
3
1
2
2

+ 4×1  3  + 1×1  3 
3 4
1 2 1 1 0
1 3 3 1
1 4 6 4 1 Take note of powers of  3 ; odd gives minus, even gives plus
1 5 10 10 5 1
127
= 1– 4 3 + 6  3 2
– 4  3 +  3
3 4 Powers higher than three will be discarded
= 1 +20x – 10x2 + 180x2 – 180x3 + 960x3
= 1 +20x + 170x2 + 780x3
  
Thus, 1 3 – 1 3
4
 4

3 + 8  3
=8
3 Example BE
Find the first sixth term of the expansion of (1 – 2x)8 in
3 + 8  3
2
=8 3 ascending powers of x
= 8 3 + 24 3 Solution
The first six terms of the expansion of (1 – 2x)8
= 32 3 = 18 + 8C1(1)7 (-2x)1 + 8C2(1)6 (-2x)2 + 8C3 (1)5 (-2x)3
+ 8C4(1)4 (-2x)4 + 8C5(1)3 (-2x)5
2002/40 (Nov) Exercise 11.1 Note: Odd powers of – 2x gives negative values, while
Simplify  3 1 3 even powers gives positive values
= 1+ 8(-2x) + 28(4x2) + 56(-8x3) +70(16x4) +56(-32x5)+…
A6 3 -9 B 2 3 -10 C 2 3 - 9 D 6 3 - 10 = 1 – 16x + 112x2 – 448x3 + 1120 x4 – 1792 x5 +...

2002/10
(a)Write down the binomial expansion of (1 + y)8,
simplifying all the terms.
(b)Using the substitution y = x – x2 in (a), deduce the
expansion of (1 + x – x2)8 in ascending powers of x,
as far as the term in x4
(c) Find, by inspection, a value of x such that 1 + x – x2 = 1.09
Problems on binomial theorem Hence, evaluate (1.09)8 correct to three decimal places.
(where n is positive integer) Solution
(a) ( 1 + y )8 = 8C018 y0 + 8C117 y1 + 8C216 y2 + 8C315 y3
Recall that the general binomial theorem for + 8C414 y4 + 8C513 y5 + 8C612 y6 + 8C711 y7 + 8C810 y8
n that are positive integer is Since 1 is positive, we can do without showing it, after the first
(a + x )n = an + nC1 xn – l a1 + nC2 xn – 2 a2 + term
n
C3 xn – 3 a3 + … + nCr xn – r ar +…+ nCn xn = 1 + 8y + 28y2 + 56y3 + 70y4 + 56y5 + 28y6 + 8y7 + y8
Where nCr xn – r ar is the general term
(b) where y = x – x2
(1 + x – x2 )8 = 1 + 8(x – x2) + 28(x – x2)2 +
2004 / 9a
56(x – x2)3 + 70(x – x2)4 + …
Write down the binomial expansion of ( 1 + x )4
Solution
The bracket terms expanded individually gives
(1+x)4
4 4 0 4 3 1 4 2
= C0(1) (x) + C1(1) (x) + C2(1) (x) + C3(1) (x) + 2 4 1 3 (x – x2)2
4
C4(1)0 (x)4 = C0(x) (-x2)0 + 2C1(x)1 (-x2)1 + 2C2 (x)0 (-x2)2
2 2

Since 1 is positive here we can do without it. i.e. = x2 – 2x3 + x4


= 1 + 4x + 6x2 + 4x3 + x4 (x – x2)3
= C0(x) (-x ) + 3C1(x)2(-x2)1 + 3C2(x)1(-x2)2 + 3C3(x)0(-x2)3
3 3 2 0

Example BE = x3 – 3x4 + … powers higher than 4 is discarded


Expand (1 + 2x – x2)10 in ascending powers of x, as far (x – x2)4
as the term in x3 = C0(x) (-x2)0 + 4C1(x)3(-x2)1 + 4C2(x)2(-x2)2 +
4 4
4
Solution C3(x)1(-x2)3 + 4C0(x)0 (-x2)4
(1 + 2x – x2)10 is taken as [ 1 + ( 2x – x2 ) ]10 = x + … powers higher than 4 is discarded here
4

Expanding, we have
= 1 + 10C1(2x - x2) + 10C2(2x - x2)2 + 10C3(2x - x2)3 + . . . Substituting, we have
= 1+8(x – x2)+28(x2–2x3+x4)+56(x3 – 3x4+…)+70(x4 +…)
Expanding each of the bracket terms
= 1+ 8x–8x2 +28x2–56x3 + 28x4 + 56x3 – 168x4 +70x4 +…
(2x – x2)2
= 1 + 8x + 20x2 – 70x4
= C0(2x) (–x ) + 2C1 ( 2x)1(– x2)1 + 2C2(2x)0 (-x2)2
2 2 2 0

= 4x2 – 4x3 + x4 (c) 1 + x – x2 = 1.09 implies


( 2x – x2)3 1 + x – x2 = 1 + 0.09
= C0(2x) (-x ) + 3C1(2x)2 (-x2)1 + 3C2( 2x)1 (-x2 )2
3 3 2 0
Thus, x – x2 = 0.09
+ 3C3(2x)0(-x2 )3
x(1 – x) = 0.09
= 8x – 12x + 6x – x
3 4 5 6
x = 0.09 or 1 – x = 0.09
(1+2x – x2)10 = 1 + 10(2x - x2) + 45(4x2 - 4x3 + x4) + x = 0.09 or 1 – 0.09
120(8x3– 12x4 + 6x5– x6) x = 0.09 or 0.91
Smallest value is better to work with by inspection with a calculator

128
Substituting into 1 + 8x + 20x2 – 70x4 Coefficient of term in an expansion I
Thus, (1.09)8 = 1 + 8(0.09) + 20(0.09)2 – 70(0.09)4 Under this subtopic the general term nCr xn – r ar of any
= 1 + 0.72 + 0.162 – 0.00 4593 binomial expansion will be used extensively. It will be
= 1.882 – 0.004593 referred to as (r + 1)th term
= 1.877407
 1.877 to 3 d.p 2003/50 Neco
Confirmation by calculator (1.09)8 = 1.993 Find the coefficient of x5 in the expansion of (2x – 1)8
A 1792 B 896 C 448 D – 896 D – 1792
Example BE Solution
Expand (x + 2y)5 and hence evaluate to the nearest The general term in the expansion of (a + x )n
naira the Compound interest on N 2,000,000.00 is ( r + 1 )th term = nC r a n – r x r
(2 million) in 5 years at 20% per annum. In our present case n is 8, a is 2x and x is - 1
Solution ( r + 1 )th term = 8Cr (2x)8 – r (-1)r
Here we treat 2y as single term Separating terms in x and making it last
(x + 2y)5 = 5C0 x5(2y)0 + 5C1 x4(2y)1 + 5C2 x3(2y)2 + = (-1)r 8Cr 28 – r x8 – r
5
C3 x2(2y)3 + 5C4 x1(2y)4 + 5C5 x0(2y)5 Equating powers of x to the given x5
= x + 5x .2y + 10x3.4y2 + 10x2.8y3 + 5x.16y4 + 32y5
5 4
8–r=5
= x5 + 10x4y + 40x3y2 + 80x2y3 + 80xy4 + 32y5 r=3
Amount in compound interest is given as: Thus, the required coefficient = (-1)3 8C3 2 8 – 3
= - 156  32
n
 r 
A  P 1   = - 1792 (E)
 100 
5
Example BE
 20  Find the Coefficient of x20 in the expansion ( 2x – x2)12
Thus, A = 1m 1 
 100  Solution
The ( r + 1 )th term = 12Cr (2x)12 – r (-x2)r
= 1m [1 + 0.2]5
Here x = 1 , 2y = 0.2 = 12Cr 2 12 – r x12 – r (-1)r x2 r
0 .2 applying indices to the powers of x ( 12 – r + 2r )
y= i.e y = 0.1 = (-1)r 12Cr 2 12 – r x12 + r
2 Equating the powers of x to that of the given x20
Substituting into x5 + 10x4y + 40x3y2 + 80x2y3 + 80xy4 + 32y5
1m[1+0.2]5= 1m[1+10(0.1)+40(0.1)2 +80(0.1)3 +80(0.1)4 +32(0.1)5] 12 + r = 20
= 1m [1 + 10(0.1) + 40(0.01) + 80(0.001) + r=8
80(0.0001) + 32(0.00001)] Required coefficient = (-1)8 12C8 2 12 – 8
= 1m [ 2 + 0.4 + 0.08 + 0.008 + 0.00032 ] = 495  24
= 1m [ 2.48832 ] = 7920
= N 2,488,320.00 2004/13 Neco
What is the coefficient of x3 in the expansion of ( 2 + x)6 ?
A 12 B 15 C 20 D 60 E 160
Solution
The general term in the expansion of (a + x )n
is ( r + 1 )th term = nC r a n – r x r
In our present case n is 6, a is 2 and x is x
( r + 1 )th term of the expansion
= 6Cr ( 2 ) 6 – r xr
Since x is in its simplified form, we equate power to the given x3
r=3
Thus the required coefficient is
= 6C3 2 6 – 3
= 20  8
= 160 (E)
1998/ 2a (Nov)
If Kx4 is a term in the binomial expansion of ( 1 + 2x)5,
find the value of K
Solution
The question wants us to find the coefficient of x4 in the
expansion of ( 1 + 2x)5
( r + 1 )th term = 5Cr ( 1 ) 5 – r ( 2x) r
Separating the term in x
= 5Cr 15 – r 2 r xr

129
Equating powers of x to x4 Similar terms will cancel out
r=4 2 3
The required coefficient K is =
= 5C4 1 5 – 4 2 4 4 n  3
= 51 16 = 80 6=n–3
n=9
2003 35 (Nov)
Find the coefficient of x8 in the binomial expansion
of (x + 3)12 Example BE
A 495 B 1336 C 40095 D 64152 Find the term independent of x in the expansion of
12
Solution  3 1 
( r + 1 ) th term = 12Cr x12 – r 3 r x  
 2x 
i.e. = 12Cr 3 r x12 – r Solution
The power of x is in its simplified form, we can equate to x8
The general term here is
12 – r = 8
 
r
r=4 3 12  r  1 
Tr + 1 = 12Cr x  
Thus, the required coefficient is  2x 
= 12C4 3 4 r
= 495  81 = 12Cr x 36 - 3r   1  x r
= 40,095 (C)  2
2005 / 24 r
Determine the coefficient of x6 in the binomial  1  36 3r  r
= 12Cr    x
expansion of (x + y)8  2
A 8y2 B 28y2 C 56y2 D 70y2 r
 1
Solution = Cr    x 36 4 r
12

The ( r + 1 ) th term = 8Cr x8 – r y r  2


The x power is in its simplified form, we equate it to x6 The term independent of x, implies “Power of x equals zero”
8–r = 6 i.e. 36 – 4r = 0
r = 2 r=9
Thus, required coefficient is 9
= 8C2 y 2  1
The required term = 12C 9  
= 28y2 (B)  2
2009 / 9b Since the power of  1 is odd, the result will carry minus sign
The ratio of the coefficient of x4 to that of x3 in the 2

binomial expansion of (1 + 2x )n is 3 : 1. 220


=–
Find the value of n. 512
( r + 1 ) th term = nCr 1 n – r ( 2x ) r 55
=–
Since 1 is positive here its effects is not relevant 128
= n Cr 2 r xr and the term is T9 +1 i.e. T10
Coefficient of x4  r is 4
Thus Coefficient of x4 = 24 nC4 2002/37 (Nov)
6
Similarly coefficient of x3 = 23 nC 3  1
The constant term in the expansion of  x   is
Their ratio is 3 : 1  x
24 n
C4 : 2 3 n C3 A -20 B 15 C 60 D 120
2 4 n C4 Solution
3
Implies 3 n
= The general term here is
2 C3 1
 1 
r
Simplify and cross multiply
2 n
C4 = 3 n
C3
Tr+1 = Cr
6
x  6 r
 
 x
n! n! 6
= Cr x6 – r (–1)r x – r
2 = 3
n  4!4! n  3! 3! Resolving indices in x
= 6Cr (–1)r x6 – r – r
2 3
= = 6Cr (–1)r x6 – 2 r
n  4!4! n  3!3! The constant term of x; implies “power of x equated to zero”
Simplifying the factorial to get similar terms 6 – 2r = 0
2 3 r=3
= The required term = C 3 (–1)3
6

n  4! 4  3! n  3n  4!3! = – 20 (A)

130
2006/10 Exercise 11.2 Solution
Find the coefficient of x4 in the binomial expansion of (1-2x)6 Tr+1  T 3 i.e. r = 2
A 320 B 240 C – 240 D – 320
The general term in the expansion of (a + x )n
is Tr +1 = nC r a n – r x r
2009/6 Exercise 11.3 2
Find the coefficient of x4 in the binomial expansion of (2+x) 6
A 120 B 80 C 60 D 15 Here T3 = 3C2 2x3  2  3 y 
 4 
2
2009/10 (Nov) Exercise 11.4
Find the value of the coefficient of x2y3 in the binomial
3
= C2 2x  1  3y 
 
 4 
expansion of (2x + 3y)5
9 2
A 243 B 720 C 810 D 1080 =32x  y
16
27 2
= xy
8
27
Thus the coefficient of the third term is (B)
8
Coefficient of term in an expansion II
2000 /45 Neco Counter example
Under this subtopic the general term nCr xn – r ar of
Given that P1 = x3 - x2 + x +2 and P2= 2x3 - x2 – 2.
any binomial expansion will be used extensively. It will What is the coefficient of x5, if P1is multiplied by P2 ?
be referred to as T(r + 1) term
A3 B1 C–1 D–2 E–3
2004/ 9 Neco Solution
What is the coefficient of x in the fifth term of the
This is a case of ordinary expansion whereby
expansion of (1 + x )6 ? P1 P2 = (x3 – x2 + x + 2 )( 2x3 – x2 – 2 )
A1 B5 C6 D 15 E 20 = 2x6 – 2x5 + 2x4 + 4x3 1st bracket by 2x3
Solution – x5 + x4 – x3 – 2x2 1st bracket by – x2
The general term in the expansion of (a + x )n – 2x3 + 2x2 – 2x– 4 1st bracket by –2
is Tr +1 = nC r a n – r x r
Here we want to find T5 i.e r = 4 in (1 + x )6 P1 P2 = 2x6 – 3x5 + 3x4 + x3 + 0x2 – 2x – 4
[The coefficient of x4] Thus, Coefficient of x5 is – 3 (E)
Thus T 5 = 6C 4 16 – 4
= 15 (D) 2004/24 Neco Counter example
2000 / 9c (Nov) What is the coefficient of x3 in the expansion of
The coefficients of the 5th, 6th and 7th terms in the (4x2 – 3x + 1)(x2 – 4x + 1)?
expansion of (1 + x )7 , in ascending powers of x are in A7 B4 C1 D – 16 E – 19
linear sequence (A . P) ,find their common difference Solution
Solution (4x2 – 3x + 1) (x2 – 4x+ 1)
The general term in the expansion of (a + x )n = 4x4 – 3x3 + x2 1st bracket by x2
is Tr +1 = nC r a n – r x r –16x3 + 12x2 – 4x 1st bracket by – 4x
Here Tr + 1 = 7C r 17 – r x r + 4x2 – 3x + 1 1st bracket by + 1
T 5 by Tr + 1 is r = 4 i.e. the coefficient of x4 = 4x4 – 19x3 + Readers to complete it.
T 6 by Tr + 1 is r = 5 i.e. the coefficient of x5 Thus, coefficient of x3 is –19 (E)

T 7 by Tr + 1 is r = 6 i.e. the coefficient of x6


2010/6 Exercise 11.5
Thus, T = 7C4 1 7 – 4 i.e 35 The coefficient of the 7th term in the binomial expansion
T = 7C5 17 – 5 i.e 21 of
T = 7C6 1 7 – 6 i.e 7
In ascending powers of x, the A.P is 35, 21, 7, … 2 – x 10
By A.P formula for common difference d 3
d = 21 – 35 or 7 – 21 in ascending powers of x is
= –14 A 560 B 841 C 1120 D 4481
243 243 243 243
2002 / 31 Neco
What is the coefficient of the 3rd term in the expansion
3
 3y 
of  2 x   ?
 4 
27 27
A B C8 D9 E 12
64 8
131
Binomial theorem n  1 1 2 1  2  5 
It has been proven that for all values of n, integral or       
3 3 3  3  3 
fractional, positive or negative, if n is numerically less =1 –
2 x +
2
4x +  8 x 3 +…
than unity then (1 + x )n is the sum of the infinite series 3 2 6
This series is known as binomial series: 2 8 x2 1 1
(1 + x )n = 1 + nC1 x + nC2 x2 + nC3 x3 +… =1 – x –   – 80 x3   + …
in its simplified form is
3 9 2 27  6 
2 4 40
= 1 + nx + nn  1 x 2 + nn  1n  2 x 3 + … =1– x – x2 – x3 + …
2! 3! 3 9 81
1 1
This simplified pattern is easy to master and is But (1.05) 3 = (1 + 0.05) 3
useful for when n  1 We equate the two given items as
1 + 0.05 = 1 – 2 x
When n = – 1 it follow that (1+ x) – 1 Thus 0.05 = – 2 x
= 1 + (-1)x + (-1)(-1-1)x2 + (-1)(-1-1)(-1-2)x3 +… 0.05
= x i.e. – 0.025
2! 3! 2
= 1 – x + x 2 – x3 + … Substituting
2 4 40
When n = – 2 it follow that (1 + x ) – 2 =1– (- 0.025) – ( - 0.025)2 – (- 0.025)3 + ….
3 9 81
= 1 + (-2)x + (-2)(-2-1)x2 + (-2)(-2-1)(-2-2)x3 +… Bearing in mind, the 4 s.f condition of our answer, we write
2! 3! down our values a little above it
= 1 – 2x + 3x – 4x + …
2 3 = 1 + 0.01667 – 0.000278 + 0.000,0078 + …
= 1.0163998
When n = 1/2  1.016 to 4 s.f
11  confirmation using calculator 3 1.05 = 1.016 to 4 s.f
1   1
1 22  2
(1 + x ) = 1 +
2
x + x +
2 2!
1  1  1  1 
1  1  1 
  1  2    1  2   3 
2  2  2  x3 + 22  2  2  x 4 +…
3! 4!
1 1 1  1  3 
      
1 2 2 2 2  2  2  3
=1+ x + x + x +
2 2! 3!
1  1  3  5 
      
2  2  2  2  4
x + ...
4!
1 1 2 1 3 5 4
=1 + x – x + x – x +…
2 8 16 128

2004 / 9b
1
Using the linear expansion of (1 – 2x) 3 , find, correct
1
to 4 significant figures, the value of (1.05) 3

Solution
In a typical problem like this, the first four terms are
OK except otherwise stated.
11 
  1
  2 x  +
1 2
1 33
(1 – 2x) 3 = 1 + (–2x) +
3 2!
1  1  1 
  1  2 
3  3  3  (2 x) 3 + …
3!

132
Chapter Twelve
Permutation and combination
Factorial EgPC4 Consider the alphabets a, b, c from which two letters
Factorial is the representation of numbers denoted by ! at a time are to be chosen. It’s
Eg.PC1 0! =1
1! =1 Permutation Combination
2! =2×1 Applying the formula Applying the formula
3 3
3! =3×2×1 P2 = 3! C2 = 3!
4! =4×3×2×1 ( 3 – 2 )! ( 3 – 2 )! 2!
5! = 5 × 4 × 3 × 2 × 1 etc = 3! i.e 6 = 3! i.e 3
1!2!
Also we can manipulate factorials to suit our workings ( order recognized ) ( order not recognized )
Eg.PC2 5! = 5(5 – 1 )!  ab ba etc  ab = ba etc
= 5 × 4! Agree ab, ba, ac, ca, bc, cb ab, ac, bc
Also 5! = 5( 5 – 1 )( 5 – 2 )! EgPC5 Four candidates P, Q, R and S applied for scholarship
=5×4×3! in Chevron and Shell, each of the scholarship must go to a
different candidate. List the possible ways it
Similarly n! = n( n – 1 )! can be awarded.
Or = n( n – 1 )( n – 2 )! Solution
Or = n( n – 1 )( n – 2 )( n – 3 )! We write PQ as ‘P gets chevron award and Q gets shell award’
Depending on the desired result. Question 1 : will PQ be the same as QP ?
Answer : NO
Examples PC3 Question 2.Is this a permutation or a combination case?
Simplify the following
Answer : permutation since PQ QP
( i ) 6! ( ii ) 7! ( iii ) n!
4!2! 6! (n – 3)!
Question 3
Which formula is to be applied ?
Solution
( i ) If you check the denominator you notice there is 4! Answer: npr = n!
( n – r )!
; our aim will be to eliminate it
6! = 6 × 5 × 4! 4
P2 = 4! = 4!
4!2! 4! 2! ( 4 – 2 )! 2!
= 6 × 5 i.e 15
2
= 4 × 3 × 2! = 12
( ii ) 7! = 7 × 6! i.e 7 2!
6! 6! The list then is PQ, QP, QR, RQ, RS, SR, QS etc
Conclusion – The scholarship can be awarded in 12 different ways.
( iii ) n! = n( n – 1) ( n – 2 ) ( n – 3 )!
(n – 3)! (n – 3)! Eg PC6. Out of the five Science Club members of a school,
= n( n – 1 )( n – 2 )
A, B, C, D and E; just three are to be chosen to
represent the school in an exhibition. In how many ways
Permutation & Combination can this be done?
The terminology permutation & combination are a bit
confusing to students like identical twins; so before we go
Solution
into the topic proper, we shall define both terms, given their Question 1. If we write ABC BCA etc as three members;
various formula and take simple examples involving both will ABC be the same as CAB, BAC and ACB
concept. Answer: YES; reason Audu, Bello, Cain is the same students
Permutation Combination no matter how you arrange them or their names
A permutation of n A combination of n different Question 2
distinct objects taken r objects taken r at a time is a Is this a permutation or combination problem?
at a time is an selection of r out of n object Answer: combination – since ABC = CAB = BAC = ACB
arrangement of r out the with no attention given to the Question3
n objects with attention order of arrangement Which formula will apply?
given to order of arrangement
Answer: nCr = n!
n n
pr = n! Cr = n! ( n – r)!r!
( n – r )! ( n – r)! r! 5C3 = 5! = 5! = 5 × 4 × 3 !
( 5 – 3 )!3! 2! 3! 2! × 3 !
Also both formulae are related as : =5×2
n np = 10ways
n
Cr is    r
r r! The list then is ABC, BCD, CDE, ADE , ABD, BCE, ABE,
ACE, BDE, ACD
133
Permutation Permutation of distinct n objects of which n1 are of one
Already, we have defined permutation and given its kind, n2 of 2nd kind,… kk of kth kind is given as :
formula; in this kind of arrangement order is given n! where n = n + n2 + … + nk
priority. n1! n2! ... nk!
Eg PC7. Evaluate 8P2 = 8!
( 8 – 2 )! Eg PC12. In how many ways can 3 yellow, 4 red and 5 blue
= 8! balls be arranged in a string of xmas tree light with 12 sockets
6! Solution
= 8 × 7 × 6! = 56 Total numbers of distinct arrangement
6! = 12!
3!4!5!
Eg PC8 . Evaluate 7P3 = 7!
( 7 – 3 )! Eg PC13. How many ways can the letter of the word
= 7! = 210 MATHEMATICIAN be arranged
4! Solution
Eg PC9. Total letters = 13
How many ways are there of arranging 3 different jobs Ms = 2, As = 3 , Ts = 2, H = 1, E = 1, Is = 2 ,
between 5 men where any man can do only one job C = 1, N = 1
Solution Arrangement = 13!
Putting ABC as man A does job1, man B does job 2, 2! 2! 2! 3! 1! 1! 1! 1!
and man C does job 3, etc. It is clear that different But in most cases 1! is not reflected since 1! = 1
arrangements of the letters mean different men do = 13!
different jobs. Hence this is a permutation problem. 2! 2! 2! 3!
Possible arrangement is 5P3 = 5!
( 5 – 3 )! Cyclic permutation
= 5! (a) n different objects can be arranged in a circle - round
2! table in ( n – 1 )! different ways.
= 5 × 4 × 3 × 2!
(b) If the objects are fixed in a ring which can be turned
2!
over, the number of arrangement is.
= 5 × 4 × 3 i.e 60
( n – 1 )!
Eg PC10. In how many ways can 8 people be seated in 2
a bench if only 3 seats are available? as an arrangement with the ring facing one way will be one
This problem is similar to EgPC9 . Hence of the arrangements with the ring turned over.
8
P3 = 8!
(8–3)! Cyclic permutation problems
= 8! = 8 × 7 × 6 × 5!
2000/28 (Nov)
5! 5!
In how many ways can 8 men be seated at a round table,
=8×7×6
if 2 particular men refuse to sit together?
= 336 ways
A.1440 B. 2000 C. 3600 D. 4320
Permutation of n objects taken n at a time Solution
i.e nPn = n! Normally, 8 men seating in a round table
( n – n )! Arrangement = (8 – 1)!
= 7!
= n! Normally
0! The two men seat together in two ways and
= n! Next the remaining arrangement is in 6!
nP = n!
n Arrangement = 2 × 6!

Eg PC11. If the mathematics department of a college Thus, two men refuse to sit together
has 5 members of staff and they are to pose for a = 7! – 2 × 6!
photograph by standing in a row. How many different = 6! (7 – 2 )
arrangements are possible?. = 6! × 5 = 3600
Solution 1999/14a (Nov)
5
P5 = 5! = 5! A family of 7 is to be seated round a table. In how many
( 5 – 5 )! 0! ways can this be done, if the father and the mother are to sit
i.e 5! = 5 × 4 × 3 × 2 × 1 together?
= 120 ways Solution
Father and the mother sitting together is to be
arranged in 2 ways.
And their position is to be taken as 1
134
The remaining 5 people plus this one making six can be Thus, the last digit can be one of these four.
arranged Once the last digit has been filled, we have eight digits
= 2 × (6 – 1)! remaining
= 2 × 5! First digit can be any of these eight digits
= 240 2nd digit can be any of the remaining seven digits
3rd digit can be any of the remaining six digits
1992/12 (b) (Nov) Arrangements = 4 × 8 × 7 × 6
In how may ways can eight boys be arranged around a = 1344
circular table? i.e 4 × 8P3
Solution Example PD2
Arrangement in a circular table How many numbers greater than 4000 can be formed using
= (n – 1)! some or all the digits, 6, 5, 4, 3 and 2 without repetition?
= (8 – 1)! Solution
= 7! i.e 5040 The numbers formed using
2003/35 Neco Exercise 12.0a All 5 digits will be greater than 4000.
The number of ways of arranging ‘n’ objects in a i.e 5! Or 5P5 ways
circular ring is
A. n! B. n! C. (n – 1)! If we form a four-digit number > 4000;
(n – r)! r!(n – r)! First digits must be either of 6, 5, or 4.
D. (n – 1)! E. 2 It is done in 3 ways.
2 (n – 1)! 2nd digit can be chosen from the left over 4 digits after the
1992/50 PCE Exercise 12.0b picking of the first one since repetition is not allowed.
Five persons are to have dinner jointly. In how many It is done in 4 ways.
ways can they sit at around table? 3rd digit can be chosen in 3 ways.
A.120 B.40 C.24 D.10 4th digit can be chosen in 2 ways
Arrangement = 3 × 4 × 3 × 2
2002/47 PCE Exercise 12.0c Total Arrangement = 5! + 3 × 4 × 3 × 2
In how many ways can 6 persons sit in a ring ? = 192 ways.
A. 60 B. 100 C. 120 D. 240
Example PD3
How many numbers greater than 2000 can be formed using
Permutation of digits the digits 1, 3, 4, 5 and 6? How many of these will be even ?
(No repetitions allowed)
Example PD1
How many 4 digits numbers can be formed with the Solution
digits 1, 2, …,9 if: If all five digits are used, every number will be > 2000.
(i) Repetitions are allowed it is in 5!
(ii) Repetitions are not allowed If only four digits are used, the first (left – hand) digit
(iii) The last digit must be even and repetitions cannot be 1.
are not allowed. It has to be any of (3, 4, 5 or 6) done in 4 ways.
Solution 2nd digit will be chosen out of the remaining 4 digits.
3rd digit will be chosen out of the remaining 3 digits.
Repetition Allowed
4th digit will be chosen out of the remaining 2 digit.
( i )The first digit can be any one of the 9 digits.
Arrangement = 4 × 4 × 3 × 2
2nd digit is also 9 choices since repetition is = 96
allowed. Total Arrangement = 5! + 96
3rd digit is 9 choices also = 120 + 96 = 216
4th digit is 9 choices similarly. If all five digits are used and the number to be even,
Arrangement = 9 × 9 × 9 × 9 The last (right hand) digit must be 4 or 6
= 6561 which gives 2 choices.
Repetition not allowed 1st digit has four choices.
( ii ) The first digit can be any one of the 9 digits. 2nd digit has three choices.
2nd digit can be any of the remaining 8 digits. 3rd digit has two choices.
3rd digit can be any of the remaining 7 digits. 4th digit has one choice.
4th digit can be any of the remaining 6 digits. Arrangement = 2 × 4 × 3 × 2 × 1
Arrangements = 9 × 8 × 7 × 6 = 48
9
= 3024 i.e P4
If only four – digits are used,
( iii ) The last digit must be even and repetitions are not
allowed. the last (right hand) digit must also be 4 or 6
Here the last digit must be even and we have four even which gives 2 choices
digits: 2, 4, 6 and 8. 1st digit cannot be 1, only be any of 3 or 5 or
135
any of 4 or 6 digit one depending on the one taken Arrangement = 1 × 4 × 3
among them to fill the right hand. = 12 ways
It is done in 3 ways. b( i ) Odd number (is negation of even number)
2nd digit can be done in 3 ways again since it can take 1 as a last digit will be any of 1 or 5 in 2 ways
digit. 1st digit will be chosen from the remaining 4 digits
3rd digit can be done in 2 ways 2nd digit will be chosen from the remaining 3 digits
Arrangement = 2 × 3 × 3 × 2 = 36 Arrangement = 2 × 4 × 3
Total Arrangement = 48 + 36 = 24 ways
= 84 Thus Pr(Multiple of 5 or odd number) = 12 + 24
60 60
2002/29 Neco
= 36
How many four – digit numbers greater than 5000 can
be formed using the digits 1, 2, 5, 6, and 7 if no digit 60 i.e 3/5
can be repeated? 2002/2a
The digits 1, 2, 3, 4, 5, 6 are written on six identical discs. Three
A.420 B. 210 C.72 D. 24 E.12
discs are drawn at random and placed in order of drawing. What is
Solution the probability that the number formed is less than or equal to 500?
1st digit will be any of 5, 6, 7, in 3 ways. Solution
2nd digit will be chosen from the remaining 4 digits. We are to find the probability of the number formed to be
3rd digit will be chosen from the remaining 3 digits. less than or equal
4th digit will be chosen from the remaining 2 digits. Pr ( ≤ 500) = 1 – Pr (> 500)
Arrangements = 3 × 4 × 3 × 2 Three digits greater than 500
= 72 (C) 1st digit must be any of 5 or 6 in 2 ways.
2003/5b (Nov) 2nd digit will be chosen from the remaining 5 digits.
How many numbers greater than 300 can be formed 3rd digit will be chosen from the remaining 4 digits.
from the digits 0, 2, 3 and 4, if repetition of digits is not Arrangement = 2 × 5 × 4
allowed? = 40
Solution Pr (> 500) = 40
6
If all four digits are to be used; P3
1st digit will be any of 2 or 3 or 4 in 3 ways. = 40 i.e 1
2nd digit will be chosen from the remaining 3 digits. 120 3
3rd digit will be chosen from the remaining 2 digits. Therefore Pr (≤ 500) = 1 – Pr (> 500)
4th digit will be chosen from the remaining 1 digit. =1–1
Arrangement = 3 × 3 × 2 × 1 3
= 18 = 2
If three digits are to be used; 3
1st digit will be any of 3 or 4 in 2 ways. 1999/14(b)
2nd digit will be chosen from the remaining 3 digits. How many 4 – digit odd numbers can be formed with the
3rd digit will be chosen from the remaining 2 digits digits 1, 2, 3, 4, if repetition is allowed?
Arrangement = 2 × 3 × 2 Solution
= 12 Since odd is the opposite of even and repetition is allowed.
Total Arrangement = 18 + 12 Last digit must be any of 1 or 3 in 2 ways.
= 30 ways 1st digit will be chosen from 4 digits
2nd digit will be chosen from 4 digits.
2003/5 3rd digit will be chosen from 4 digits.
Three-digit numbers are formed from the digits 1, 2, 4, Arrangement = 2 × 4 × 4 × 4
5, 6. If repetition of digit is not allowed and a number is = 128 ways
picked at random, find the probability that it is a
Assuming repetition is not allowed.
multiple of 5 or an odd number. Since odd is the opposite of even and repetition is not allow
Solution Last digit must be any of 1 or 3 in 2 ways.
Since probability is involved; 1st digit will be chosen from the remaining 3 digits
we find the sample space for 3 digits formed 2nd digit will be chosen from the remaining 2 digits.
from 5 digits i.e 5P3 3rd digit will be chosen from the remaining 1 digit.
(a) Probability (multiple of 5) = multiple of 5 ways Arrangement = 2 × 3 × 2 × 1
5
P3 = 12 ways
(b) Probability (odd number) = odd number ways 1998/6 (Nov)
5
P3 3 – digit numbers are to be formed from 1, 2, 3, ...9.
If repetition of digits is not allowed,
a( i )Multiple of 5 (Recall that: condition for multiple (a) How many numbers can be formed?
of 5 is that the last digit should be 5) i.e 1 way. (b) What is the probability of forming a number
1st digit will be chosen from the remaining 4 digits. divisible by 5?
2nd digit will be chosen from the remaining 3 digits
136
Solution Permutation of letters
3– digit numbers and repetition not allowed Example PL1
(a) First digit can formed in 9 ways. Out of the letters A, B, C, p, q, r how many arrangements
2nd digit can formed in 8 ways. can be made.
3rd digit can formed in 7 ways. (i) beginning with a capital.
Arrangement = 9 × 8 × 7 (ii) beginning and ending with a capital.
= 504
Solution
We can solve directly in this case as: 9P3
( i ) A capital letter out of the given letters can be
(b) Pro (number divisible by 5) = number ÷ 5 way chosen in 3 ways.
9
P3 Remaining five letters can be arranged among
3– digit number divisible by 5 themselves in 5! ways.
Last digit must be 5 in 1 way. Therefore total number of arrangements beginning
1st digit will be chosen from the remaining 8 digits. with a capital letter = 5! × 3
2nd digit will be chosen from the remaining 7 digits. ( ii ) Two capitals out of the given letters can be
Arrangement = 1 × 8 × 7 i.e 56 chosen in 3P2 = 6 ways.
Thus, Pr (number divisible by 5) = 56 i.e 1 For each choice of these two letters,
504 9 remaining four can be arranged in 4! ways
Hence the required number of arrangements
2003/13 Exercise 12.0d = 6 × 4!
How many numbers less than 3000, can be formed from 2000/22
the digits 1,2,3,8 and 9, if no digits is used more than How many ways can the letters of the word CHEMISTRY be
once? arranged, if the vowels must always be together?
A. 12 B. 36 C. 48 D. 72 A.725760 B.362880 C.181440 D.80640
Solution
Vowels are: a, e, i, o and u
The ones present here are E, I
The two letters can be arranged in 2 ways
If they are to be together; they will be treated as one
The 9 letters word minus the two vowels to be seven and
since the two vowels will be treated as one
Making them to be 8 letters.
Thus arrangement = 2 × 8!
= 80640(D)
Example PL2
How many words can be formed from the letters of
VALEDICTORY provided the letters A, E, I, O, Y are not to
be separated at all?
Solution
Put together A, E, I, O, Y and count them as one letter. Thus
we have V, L, D, C, T, R, (AEIOY).
These seven letters can be arranged in 7! ways.
But the group (AEIOY) consisting of five letters can itself be
arranged in 5! ways.
Hence the required number = 5! × 7!
Example PL3
How many different words can be formed with the letters of
‘HARYANA’?
How many of these words have H and N together?
How many of these words begin with H and end with N?
Solution
Total number of letters in ‘HARYANA’ is 7 with 3A’s.
So total number of different words
= 7! = 840
3!
Counting HN as one letter, we are left with 6 letters having
3A’s.
H and N can be put together in two different manners namely
HN and NH.

137
Required no of words in which H and N are together 2005/12
= 2 × 6! = 240 The registration number of every car in a state consists of 2
3! letters from X, Y, P, R and 3 digits from 2, 4, 6, 8. How
Finally the number of words beginning with H and many cars can be registered under this system?
ending with N is same as the number of arrangements A. 80 B. 228 C. 512 D. 1024
of ARYAA, which is equal to Solution
5! = 20 same as [1 × 1 × 5! ] Here you will agree that XY ≠ YX and 246 ≠ 426 in car
3! 3! number registration; hence a permutation problem.
Letter digits
Example PL4
In how many ways can the letters of the word Selection = 4P2  4
P3
STRANGE be arranged so that the vowels occupy only 4!  4!
the odd places. (4 – 2) (4 – 3)
= 4!  4!
Solution
2! 1!
Number of letters in STRANGE is 7 so there are 4 odd
places namely 1st, 3rd, 5th, 7th. = 24  24
Vowels are only 2. 2 = 288
The number of ways in which 2 vowels can occupy
4 places is 4P2 2004/5 (a) Exercise 12.1
For each of such way, the remaining 5 places are to be How many seven – letter words starting with the letter T can
occupied by five remaining consonants. be formed from the word FURTHER?
This can be achieved in 5! different ways.
These vowels can be arranged within themselves in two 2005/11 Exercise 12.2
ways. How many distinct arrangements can be made from the word
Hence the required number = 4P2 × 5! × 2 STATISTICS?
= 2880 A. 5400 B. 6400 C. 50400 D. 60400
2004/45 (Dec) Neco
2002/48 PCE Exercise 12.3
Find the number of arrangements in the letter of the Taking two letters at a time obtain the number of permutation of
word CONGRATULATIONS if the letter ‘A’ must be the alphabetic vowels.
placed next to each other. (Leave your answer in terms A. 10 B. 20 C. 25 D. 40
of factorial)
A.16! B.15! C.14! D.13! E.12! 2001/39 Exercise 12.4
Solution In how many ways can the letters in the word ‘IRRITATE’
Normally; CONGRATULATIONS is 15 letter word has be arranged?
2(O), 2(N), 2(A), 2(T), 1(S), 1(R) 1(G) A 40 320 B 5 040 C 2 520 D 960 E 120
Congratulations arrangement = 15!
2! 2! 2! 2! 1! 1! 1! Row - arrangement permutation
If ‘A’ must be place next to each other i.e going together
Two As arranged in two ways and when taken as one 2000/12b
letter, along with the other becomes 14 and Three boys and four girls are to be seated in a row. How
we have 2(O), 2(N), 2(T) many ways can this be done if the boys are to sit in the
(AA) CONGRTULTIONS = 2 × 14! middle?
2! 2! 2! Solution
The 3 boys sitting in the middle together as one and the 3
2002/12b
boys can be arranged in 3!
How many words, taking 6 letters at a time from
The remaining 4 girls can inter change their positions in pairs
COUNTRYSIDE, will contain N, T and C?
as 4P2 = 12 ways
Solution
Total Arrangement = 3! × 4P2
COUNTRYSIDE without N, T and C becomes
= 6 × 12
OURYSIDE i.e 8 of which we are to choose 3 from and
= 72 ways
the arrangement = 8P3
2000/5b
= 336
Ten different books are to be arranged in a row on a
2004/10
bookshelf, which can take only seven books. If two of the
How many arrangements are there in the word
books are to occupy the ends, how many ways can the books
EVALUATE?
be arranged?
A.10080 B.20160 C.30240 D.40320 E.50400
Solution
Solution The two books that are to occupy the ends can be done in two ways.
We are left with five books positions out of the left over 8 books.
Arrangts = 8! (Total no of letters)! And since this is row arrangement AB ≠ BA i.e permutation
2!2! (Similar E)! (Similar A)! Thus, arrangement = 2 × 8P5
= 10080 (A) = 13440
138
Example RA3 (ii ) Total number of arrangement is 6! = 720
In how many ways can 3 boys and 5 girls be arranged in Hence the number of arrangements in which English papers
a row so that all the 3boys are together? are not consecutive is
Solution = 720 – 240
We count the 3 boys as one boy so that the number of = 480
persons involved is now 6.
They can be arranged in 6! ways 2002/32 (Nov)
But these 3 boys themselves can be arranged in 3! ways Six men and 2 women are to be seated on reserved seats at a
Hence the required numbers of arrangement is 6! × 3! conference. If the seats of the 2 women are labeled with their
names, in how many ways can the group be seated?
2003/5a A. 360 B. 720 C. 1080 D. 40320
In how many ways can 2 boys and 5 girls be seated in a Solution
row, such that the boys do not sit together? Since the 2 women are labeled, they are left or arranged as 1.
Solution Remain six men will seat in 6! ie 6! = 720.
Without any condition arrangement = 7! Total number of ways = 1  720
Two boys sit together can be done in 2 ways = 720 B
And the two is to be taken as 1 adds to the 5 girls to 1994/23
make 6, which can be arranged 6! Five students are lined up in row. How many arrangements
Thus, arrangement = 2 × 6! could be made if the position of the last boy remains
Arrangements such that the boys do not sit together unchanged?
= 7! – 2 × 6! A. 120 B. 60 C. 25 D. 24 E. 23
= 6! (7 – 2) Solution
= 6! × 5 Arrangement = 1  4!
= 3600 = 24 (D)

1993/22 (Nov) 2002/27 (Nov) Exercise 12.5


In how many ways can three people be seated on eight The number of ways of arranging 9 men and 8 women in a
seats in a row? row, when the women occupy the even places, is
A. 6720 B.336 C. 240 D.128 E.56 A. 17! B. 17!16! C. 9!8! D. 9!8! E. 72
Solution 9!8! 2! 2
It is a permutation problem because in rows position
matters a lot i.e AB ≠ BA 1994/22 (Nov) Exercise 12.6
Arrangements = 8P3 It is required to seat 5 men and 4 women in a row so that the
= 336 (B) women occupy the even numbered seats. How many of such
arrangements are possible?
2004/11 Neco A.362880 B.2880 C.120 D.24 E.5
In how many ways can 2 boys and 3 girls be arranged to
sit in a row if the 2 boys must sit together? 1999/34 (Nov) Exercise 12.7
A. 6 B. 4 C. 3 D. 2 E. 1 Seven men and eight women are to be seated in a row. If the
Solution women are to occupy the odd places, in how many ways can
2 boys sitting together is to be taken as one and 3 girls this be done?
to make 4 which can be arranged in 4! A. 15! B 15! C. 7!8! D. 7!8!
And the two boys can be interchanged in 2 ways. 7!8! 15!
Thus sitting arrangement = 2 × 4!

Example RA7
Six papers are set in terminal exams of which two are
English. In how many different orders can the papers be
arranged so that: 1992/49 UME Exercise 12.9
i. The two English papers are together. Five people are to be arranged in a row for a group
ii. The two English papers are not consecutive. photograph. How many arrangements are there if a married
Solution couple in the group insisted on sitting next to each other ?
(i )Counting the two English papers as one, A 48 B 24 C 20 D 10
total number is now 5 and can be arranged in 5! ways
The two English papers can be arranged within
themselves in 2 ways.
The arrangements where the English papers are
together.
= 2 × 5!
= 2 × 120 i.e 240
139
Combination (ii) There is no restriction on teachers – hence its selection is
In combination orders or repetitions are not allowed still 9C4 as before. Since one particular principal must be
i.e ab = ba included, we are left to choose 2 additional principals from
Also recall that combination is denoted by the remaining 4 ;which can be done in 4C2 ways.
n
Cr or ( nr ) = n! Selection = 9C4 × 4C2
(n – r )! r! = 9! × 4!
The relationship between permutation and combination ( 9 – 4 )! 4! (4 – 2 )! 2!
is n
Cr = nPr = 9! × 4!
r! 5!4! 2!2!
Example PC 24 = 9×8×7×6×5! × 4×3×2!
In how many ways can a disciplinary committee of 3 be 4×3×2×5! 2×2!
formed from 10 members of staff of a college? =9×2×7 × 2×3
Solution = 756
The members of committee are always the same since ( iii ) Since 2 particular teachers can not be on the committee
there is no condition placed on the membership. we are left with 7 teachers to choose 4 from and this can be
Combination problem done in 7C4 ways. There are 5C3 ways to select the principals
10
C3 = 10! – since no condition is attached.
( 10 – 3 )! 3! Selection = 7C4 × 5C3
= 10! = 7! × 5!
7! 3! (7 – 4)!4! ( 5 – 3 )!3!
= 10×9×8×7! = 7! × 5!
3×2×7! 3!4! 2!3!
= 10×3×4 = 120 = 7×6×5×4 × 5×4×3
Example PC 25 3×2×4! 2×3!
In how many ways can a committee of 3 maths teachers = 7×5 × 5×2
and 5 chemistry teachers be formed from 6 maths = 350
teachers and 10 chemistry teachers 1992/50 UME
Solution A student has 5 courses to take from mathematics and
The maths teachers can be selected in 6C3 ways and the physics. There are 4 courses in mathematics and 3 in physics
chemistry teachers can be selected in 10C5 ways. which he can choose from at will. In how many ways can he
Number of ways = 6C3 × 10C5 choose his courses so that he takes exactly two courses in
= 6! × 10! physics ?
( 6 – 3 )! 3! (10 – 5 )! 5! A.18 B.12 C.10 D.7
= 6! × 10! Analysis and Solution
3!3! 5! 5! The student must, take exactly 2 courses in physics out of 3,
= 6×5×4×3! × 10×9×8×7×6×5! in 3C2 ways and will be left with 3 courses to take in maths
3×2×3! 5×4×3×2×5! out of 4 in 4C3 ways to complete his 5 compulsory courses
= 5× 4 × 2×9 ×2× 7 The three courses in maths and 2 in physics are the same; no
= 5040 matter the arrangement
Hence – a combination problem.
Example PC 26 Number of ways = 4C3 × 3C2
Out of 9 teachers and 5 principals, a committee = 4! × 3!
consisting of 4 teachers and 3 principals is to be formed. ( 4 – 3 )!3! (3 – 2 )!2!
In how many ways can this be done if = 4! × 3!
( i ) any teacher and principal can be included. 1!3! 1! 2!
( ii ) one particular principal must be on the committee = 4×3! × 3×2!
( iii ) two particular teachers can not be on the 3! 2!
committee ? = 4 × 3
Analysis and Solution 9C4 × 5C3 = 12 ways ( B )
( i ): 4 teachers out of 9 can be selected in 9C4 ways
and 3 principal out of 5 can be selected in 5C3 ways. 2006/13b
Hence the required number of ways is A man travels from Nigeria to Ghana by air and from Ghana
= 9! × 5! to Liberia by ship. He returns by the same means. He has 6
( 9 – 4 )! 4! (5 – 3 )! 3! airlines and 4 shipping lines to choose from. In how many
= 9! × 5! ways can he make his journey without using the same airline
5! 4! 2! 3! or shipping line twice?
= 9×8×7×6×5! × 5×4×3! Solution
4×3×2×5! 2×3! Without using the same airline or shipping line twice simply
= 9×2×7 × 5×2 mean combination
= 1260 Note: boxes are optional steps
140
‘Travels and returns by same means’ implies 2 Solution
Nigeria to Ghana : Ghana to Liberia Since there is no restriction on girls, it is 8C3.
6 airlines 4 shipping lines As for the boys, one is already there, we are to choose 3 from
6 4
C2 C2 the leftover 9 in 9C3 ways
No of ways = C2  C2
6 4
Selection = 8C3  9C3
= 6!  4! = 8!  9!
4!2! 2!2! (8 – 3)3! (9 – 3)! 3!
= 6 5  4 3 = 8!  9!
2 2 5! 3! 6! 3!
= 90 ways = 8765!  9876!
2005/13b 325! 326!
A committee of 3 men and 4 women is be formed from = 8  7  3  4 7
5 men and 7 women. How many different committees = 4704
can be formed if : 2004/20
(i) a particular man must be on the committee A football team of 11 players is to be selected from 15
(ii) Two particular women must not be on the committee players. In how many ways can this be done, if a particular
(iii) There are no restrictions? player must be included?
Solution A. 364 B. 1001 C. 1365 D. 3003
First let us get the required selections without Solution
restrictions If Elisha, Glory, Chukwu, Isa, Akpos are players no matter
Men women how you select them, when you pick these four players, they
5 7
C3 C4 remain the same people ie AB = BA hence a combination
( i ) There is no restriction on women, hence its problem. Since a particular player is to be included, we are
selection is 7C4 . Since a particular man must be on the left to choose 10 players from 14
committee, we are left to choose 2 additional men from Selection = 14C10
the remaining 4, which can be done in 4C2 ways = 14!
Selection = 7C4  4C2 (14 – 10)!10!
= 14!
= 7!  4!
4! 10!
(7 – 4)! 4! (4 – 2)! 2!
= 14  13  12  11  10!
4  3  2  10!
= 7!  4! = 7  6  5  4!  4  3  2!
= 1001 (B)
3! 4! 2! 2! 3  2  4! 2!  2
=765  43 2003/13b (Nov)
32 2 A committee of 6 is to be formed from 9 women and 3 men.
= 210 In how many ways can the members be selected so as to
(ii) Since two particular women cannot be on the include:
committee, we are left with 5 women to choose 4 from ( i ) At least one man ( ii ) No man
and this can be done in 5C3 ways. No restriction on the Solution
men, so it is 5C3 ( i ) At least one man selection will include
Selection = 5C4  5C3 one man + two men + three men
= 5!  5! = 5  4!  5  4  3! One man included in the selection and we take 5 women to
1! 4! 2! 3! 1!  4! 2! 3! complete 6 members
= 5  52 3
C1  9C5
= 50 Two men included in the section and we take 4 women to
(iii) There is no restriction complete 6 members
Selection = 5C3  7C4 3
C2  9C4
= 5!  7! All the three men included in the section and we take 3
(5 – 3)!3! (7 – 4)!4! women to complete 6 members
= 5!  7! 3
C3  9C3
2! 3! 3! 4! Thus: at least one man selection
= 543!  7654! = 3C1  9C5 + 3C2  9C4 + 3C3  9C3
23! 324! = 3  126 + 3  126 + 1  84
= 10  7  5 = 378 + 378 + 84
= 350 = 840
2005/6 (a) Nov ( ii ) Selection with no man = 9C6
A committee of 4 boys and 3 girls is to be formed from = 9!
a club of 10 boys and 8 girls. In how many ways can 3! 6!
this be done, if a particular boy must be on it? = 84

141
Example PC 27 ( iii ) it is the total sample space = 11C4
A committee consisting of 5 members is to be formed = 330
out of 6 men and 4 women. How many committees can
(b) Pr = number of required outcome
be formed so that at least one woman is always there in
Number of possible outcome
the committee?
= C3  6C1
5
Solution 11
C4
Total number of committees = 10C5 i.e 252 = 10  6 = 2/11
Number of committees in which no woman is included = 6C5 i.e 6
330
Hence the number of committees in which at least one
woman is included = 252 – 6
1994/5
= 246 A committee of 3 members is to be formed from 4 men and 5
2002/10 (Nov) women. How many committees of 3 members each, are
A committee of 5 health officers is to be formed from a possible if there will be :
group of 6 doctors and 4 nurses. Find the probability (a) no restrictions
that it consist of 3 doctors (b) 1 man and 2 woman
A. 1/21 B. 13/126 C. 15/64 D. 10/21 (c) 2 men and 1 women such that a certain man must be on
Solution each of the committees?
Since probability is involved; Solution
Total sample space will be required which is 10C5 (a) No restrictions here means total sample space
Selection consisting of 3 doctors: 6C3  4C2 = 9C3 i.e 84 committees
This is so because the committees’ members are 5 and (b) 1 man and 2 women
since 3 members must come from doctors, the = 4C1 × 5C2
remaining 2 will come from nurses. = 4 × 10 i.e 40
Thus, Pr(3 doctors) = 6C3  4C2 (c) a certain man that must be on each of the committees
10
C5 leaves us with one man and one woman
10
= /21 = 3C1 × 5C1
= 3 × 5 i.e 15
1997/22
2000/5 (Nov)
What is the probability of selecting a committee of 3
An examination paper has three sections, namely A, B and C.
men and 2 women from a group of 10 men and 6 women.
There are 5 questions in section A, 4 questions in B and
A. 10C3  6C2 B. 10C3  6C2 C. 10C3 + 6C2 4 questions in C. A candidate is required to answer all the
10 10 10
C5 C3 C5 questions in A, 3 from B and 2 from C.
D C3  C2
10 6
E. C3  C2
10 6
(a) How many ways can he make his choice?
15 16
C6 C5 (b) How many selections are possible if :
Solution (i) a question in B is made compulsory?
Pr(x) = number required outcome (ii) one particular question in C cannot be chosen
Number of all possible outcome because the question is out of the syllabus?
= 10
C3  6C2 Solution
16
C5 (E) Here answering question 8 and 9 is same as answering
question 9 and 8; hence a combination problem.
1996/12 (Nov) (a) Selection = all 3 from B 2from C
An excursion group of 4 is to be drawn from among 5
C5 × 4
C3 × 4
C2
5 boys and 6 girls =1× 4×6
(a) Find the number of ways of choosing the excursion = 24
group if the group (b) i Since a question in B is made compulsory the
( i ) is to be made up of an equal number of boys and girls candidate is left with three questions in B to choose
( ii ) is to be either all boys or all girls 2 from
( iii ) has no restrictions on its composition all 2 from 3 in B 2from C
(b) What is the probability that a random choice of members Selection = 5C5 × 3
C2 × 4C2
for the group will result in 3 boys and 1 girl? =1× 3×6
Solution = 18
(a) ( i ) Required group number : 4 (b) ii Since one particular question in C cannot be chosen
Equal number of girls and boys i.e 2 boys: 2 girl the candidate is left with 3 questions instead of 4 in C
Selection = 5C2  6C2 to choose 2 from
= 150 all 3 from B 2from 4 in C
( ii ) Required group number : 4 Selection = 5C5 × 4
C3 × 3
C2
Either all boys or all girls =1× 4×3
Selection = 5C4 + 6C4 = 12
= 5 + 15
= 20
142
2002/1b
In how many ways can a delegation of at least 1994/12 (Nov) Exercise 12.15
7 members be selected from 10 officials? A panel of 3 women and 4 men is to be set up from 8 women
Solution and 7 men. Find the number of ways in which the panel can be
Here it is a delegation of people hence AB = BA, combination (a) Formed
At least 7 members = 10C7 + 10C8 + 10C9 + 10C10 (b) Formed if it must have at least 2 women
= 120 + 45 + 10 +1 (c) Formed if Miss A refuses to serve on the panel if
= 176 ways Mr.B is a member.
1992/23 (Nov)
Two books are to be chosen from three paperbacks and 2000/4 Neco Exercise 12.16
three hardbacks. Find the number of ways in which this In how many ways can 2 men and 3 women be chosen from
can be done, if at least one must be paper back. 5 men and 6 women?
A. 24 B. 15 C. 12 D. 9 E. 3 A. 6 B. 100 C. 200 D. 240 E. 300
Solution
Since the number of books to be chosen is 2 1994/24 (Nov) Exercise 12.17
At least one paper back = How many committees of size 5 constituting of 3men and 2
one paper backone hard back+2 paper back  no hard back women can be selected from 8 men and 6 women if a certain
man must not be on the committee?
Paper hard Paper hard A. 315 B. 525 C. 840 D. 1287 E. 2002
3
C1 × 3C1 + 3C2 × 3C0
= 3×3 + 3 1
= 12 (C)

1993/30 (Nov)
In how many ways can 9 bulbs be selected from 4 red,
5 green and 6 yellow bulbs if 3 of each colour are to be 2005/28 Exercise 12.19
included? A committee of 3 men and 4 women is to be formed from 5
A. 800 B. 120 C. 40 D. 27 E. 15 men and 6 women. If a particular woman must be included,
Solution in how many ways can this be done?
Arrangements = 4C3 × 5C3 × 6C3 A. 50 B. 100 C. 150 D. 200
= 4 × 10 × 20
= 800 (A) 2003/16 (Nov) Exercise 12.20
A committee of 5 members is to be formed from a group of 8
1999/14 (Nov) Exercise 12.11 men and 6 women. How many different committees are
Eight football teams are to play one another in pairs on possible if at least a woman must be included?
a home – and – away basis. How many matches will be A. 120 B. 420 C. 1946 D. 2058
played in all?
A. 16 B. 28 C. 56 D. 112 2003/15a Neco Exercise 12.21
In how many ways can a committee of 3 students be formed
1997/12a Exercise 12.12 from a class of 15 students?
A panel of 5 jurists is to be chosen from a group of 6 men and
6 women. Find the number of different panels that could be 2003/33 Neco Exercise 12.22
formed if A population consists of the five members 2, 3, 6, 8, and 11.
(i) a particular man must serve on the panel How many different ways are there of selecting a sample of
(ii) There is no restriction two members when the sampling is with replacement?
A 5 ways B15 ways C 25 ways D 51 ways E 52 ways
1996/28 (Nov) Exercise 12.13
In how many ways can a committee of 5 men and 3 2003/8a Dec Neco Exercise 12.23
women be chosen from 8 men and 5 women, if a A bag contains five identical balls of which there is one red,
particular man must be on the committee? one blue and the rest are white. In how many ways can three
A. 35 B. 45 C. 350 D. 700 E. 792 balls be chosen if at least one must be white.

1996/27 Exercise 12.14 2003/13d Exercise 12.24


In how many ways can a committee of 3 men and 4 A carton contains 12 eggs, 1 of which is bad. Find the
women be selected from a group of 7 men and 9 women number of ways of selecting three good eggs from the carton
A. 126 B. 161 C. 230 D. 440 E. 11440

143
Problems on Mathematical aspect of We were told at the early part of this topic that
n
factorials, permutation and combination Cr = nPr
r!
1994/17 PCE i.e. = 1 nPr (B)
Factorise 14! – 10( 13! ) r!
A.14 B.2(13!) C.3(13!) D.4 ( 13! )
2005/13a
Solution
n
13! is the lowest factorial among the two , thus, we Show that n C r  n 1C n  r 
express 14! in terms of 13! factor r
14! – 10( 13! ) = 14 × 13! – 10( 13! ) Solution
Factor out 13! We are to show that LHS = RHS
= 13! ( 14 – 10 ) LHS: nCr = n!
= 13! ( 4 ) D. (n – r)! r!
(n  1)! n
2002/22 Neco n 1
C nr 
n
= 
(n  1)  (n  r )! (n  r )! r
RHS:
What is the value of (6! – 4!)2 ? r
A. 24 B. 48 C. 696 D. 720 E. 1392
Opening up the denominator bracket item n – 1 – n + r becomes r – 1
Solution
We bring out the lowest factor (n  1)! n
= 
(6! – 4!)2 = [4!(65) – 1)]2 (r  1) ! (n  r )! r
= [4! (30 – 1)]2
= 24 (29)2 = n(n – 1)!
= 1392 (E) r(r – 1)! (n – r)!
2002/32 Neco Can you see any factorial logic here
What is the factorial expression of a (a –1)(a –2)(a –3)? = n!
A. a! B. a! C. a! D. a! E. a! r! (n – r)! QED
(a – 2)! a–3 a–4 (a – 3)! (a – 4)! 2005/6 (b) Nov.
Solution Given that 10Cr + 2 = 10Cr , find the value of r,
We should recall that a! can be expressed as: Solution
a! = a (a – 1)! 10
Cr + 2 10
Cr
a! = a (a – 1) (a – 2)! 10! = 10!
a! = a (a – 1) (a – 2) (a – 3)! [10 – (r + 2)]! (r + 2)! (10 – r)! r!
a! = a (a – 1) (a – 2) (a – 3)(a – 4)! and so on
10! = 10!
a(a – 1)(a -2)(a – 3) = a(a – 1)(a – 1)(a – 3)(a – 4)! (8 – r)! (r + 2)! (10 – r)! r!
(a – 4)! Cross multiply
= a! 10!(10 – r)! r! = 10! (8 – r)! (r + 2)!
(a – 4) ! 10! can cancel out
2000/24 (10 – r)! r! = (8 – r)! (r + 2)!
Simplify 1 – 1 + 1 Applying factorial notations to make both sides similar
(n + 1)! n! (n – 1)! (10 – r) (9 – r) (8 – r)! r ! = (8 – r)! (r + 2) (r + 1) r!
A. n B. n2 + 2 C. n2 – n +2 D. n2 Similar terms will cancel out
(n + 1)! (n + 1)! (n + 1)! (n + 1)! (10 – r)(9 – r) = (r + 2)(r + 1)
Solution Expanding both sides
By factorial notations LCM of (n + 1)! , n! and (n – 1)! is (n + 1)!. 90 – 10r – 9r + r2 = r2 + 3r + 2
Reason: (n + 1)! can be expressed as: r2 will cancel out
(n + 1)! = (n + 1) n! i.e (n + 1)[(n+1)– 1]! 90 – 2 = 3r + 19r
(n + 1)! = (n + 1)n(n – 1)! and so on 88 = 22r Thus, r = 4
2000/5a
1 – 1 + 1 = 1 – (n +1) + (n +1)n
Given that nC6 = n + 1C5 , find the value of n
(n + 1)! n! (n – 1)! (n + 1)!
Solution
= 1 – n – 1 + n2 + n n n+1
C6 C5
(n + 1)!
= n2 n! = (n + 1)!
(n + 1)! (n – 6)! 6! (n + 1 – 5)! 5!
2001/47 PCE n! = (n + 1)!
The value of nCr is the same as the value of (n – 6)! 6! (n – 4)! 5!
A.r! ( nPr ) B. 1 !( nPr ) C.(n – r)! ( nPr ) D. 1 !( nPr ) Simplifying the factorials to get similar terms on both sides
r n–r n! = (n + 1) n!
Solution (n – 6)! 6(5!) (n – 4)(n – 5)(n – 6)! 5!
The examiner wants to know whether you can recall or Similar terms will cancel out
recognize the relationship between combination and 1 = n+1
permutation. 6 (n – 4) (n – 5)
144
Cross multiply Solution
(n – 4)(n – 5) = 6(n + 1) n
P2 ÷ nC3 = 1/4
n2 – 9n + 20 = 6n + 6 n! ÷ n! = 1
n – 15n + 14 = 0
2
(n – 2)! (n – 3)! 3! 4
Factorizing
n!  (n – 3)! 3! = 1
(n – 1) (n –14) = 0
(n – 2)! n! 4
n = 1 or 14
We accept n = 14 as 1C6 and 2C5 can’t help us here. 1  (n – 3)! 3! = 1
(n – 2)! 1 4
Simplifying the factorials to get similar terms (n – 2)! = (n – 2)(n – 3)!
2004/6
Solve the equation n
P5 = 12 1  (n – 3)! 3! = 1
n
C3 (n – 2)(n – 3)! 1 4
Solution 1  3! = 1
n
P5 ÷ nC3 = 12 (n – 2) 1 4
Cross multiply
n! ÷ n! = 12 3!  4 = n – 2
(n – 5)! (n – 3)! 3!
24 = n – 2 Thus, n = 26
n!  (n – 3)!3! = 12 2003/13 (c)
(n – 5)! n! Show that 6C2 + 6C3 = 7C4
3! (n – 3)! = 12 Solution
(n – 5)! We are to show that LHS = RHS
Applying factorial notations (reducing it by one at ago) LHS 6C2 + 6C3
6 (n – 3) (n – 4) (n – 5)! = 12 = 6! + 6!
(n – 5)! 4! 2! 3! 3!
6 (n – 3) (n – 4) = 12 = 6  5  4! + 6  5  4  3!
(n – 3) (n – 4) = 2 4! 2! 3! 3!
n2 – 7n + 12 = 2 = 65 + 654
n2 – 7n + 10 = 0 2! 3!
n – 2n – 5n + 10 = 0
2
= 15 + 20 = 35
n (n – 2n) 5 (n – 2) = 0 RHS 7C4 = 7!
(n – 2) (n – 5) = 0 3! 4!
n – 2 = 0 or n – 5 = 0 = 7  6  5 4!
n = 2 or 5 3! 4!
We accept n = 5 since 2P5 and 2C3 can’t help us here. = 7  6  5 = 35
3!
2003/29 (Nov) Therefore LHS = RHS QED
n
Simplify P3 + nC0
n
C2 2002/1 (a)
A. n – 2 B. n – 3 C. 2n – 5
D. 2n – 3 If 8Cn : 6Cn – 1 = 56 : 15, find n
Solution Solution
n
P3 ÷ nC2 + nCo Recall that ratio is division i.e
8
Cn = 56
= n! ÷ n! + n! 6
Cn – 1 15
(n –3)! (n – 2)! 2! (n – 0)! 0!
= n!  (n – 2)! 2! + n! 8! ÷ 6! = 56
(n – 3)! n! n! (8 – n)! n! [6 – (n – 1) ]! (n – 1)! 15
Some terms will cancel out 8!  (7 – n)! (n – 1)! = 56
= 1  (n – 2)! 2! + 1 (8 – n)! n! 6! 15
(n – 3)! 1 Simplifying the factorials to get similar terms
Simplifying the factorials to get similar terms (n – 2)! = (n – 2)(n – 3)! 8  7  6!  (7 – n)! (n – 1)! = 56
= 1  (n – 2)(n – 3)! 2! + 1 (8 – n)(7 – n)! n(n – 1)! 6! 15
(n – 3)! 1 Some terms will cancel out
Some terms will cancel out 8  7  1 = 56
= 1  (n – 2)  2! + 1 (8 – n)n 1 15
= 2n – 4 + 1 Cross multiply
= 2n – 3 (D) 56  15 = 56(8 – n)n
15 = (8 – n)n
2003/13 (b) 15 = 8n – n2
Solve the equation n
P2 = 1 n – 8n + 15 = 0
2
n
C3 4 Factorizing
(n – 5)(n – 3) = 0 Thus n = 3 or 5
145
2002/39 Solution
If 6Pr = 6Cr , find the possible values of r 6
P3 – 6C3 = 6! – 6!
A.0, 1 B. 0, 5 C. 1, 5 D. 0, 1, 5 (6 – 3)! (6 – 3)! 3!
Solution = 6! – 6!
6
Pr = 6Cr 3! 3!3!
6! = 6! Factor out the lowest term
(6 – r)! (6 – r)! r ! 6!  1 
6!  1
Similar term on both sides will cancel out = 1   i.e 1  
1 = 1 3!  3!  3!  6 
1 r! 5
Cross multiply = 654
r! = 1 6
Implies r = 0 or 1 (A) Since 0! = 1 and 1! = 1 = 100 (E)

2000/12b (Nov) Exercise 12.25


Evaluate 10P5  8C3 If nP5 ÷ nC4 = 144, find n
Solution
10
P5  8C3 Exercise 12.26
= 10!  8! With the usual notation, if nP3 ÷ nC2 = 120, find n
(10 – 5)! (8 – 3)! 3!
2006/18 Exercise 12.27
= 10!  8! n
P4
5! 5! 3! Simplify n
To cancel out some terms, we apply factorial
C4
= 10  9  8  7  6  5!  8  7  6  5! A 24 B 18 C 12 D6
5! 5! 3!
= 10  9  8  7  6  8  7  6 2003/9 Exercise 12.28
3 2 Given that nP r = 90 and nC r = 15, find the value of r
= 1693440 A5 B4 C3 D2

Example PC 29 2009/19 Exercise 12.29


8
If n! /4! ÷ nC3 = 30, find n Px
Solution If 8
= 6, find the value of x
Cx
Applying the various formulae:
A1 B2 C3 D6
n! /4! ÷ nC3 = 30 becomes
n! ÷ n! = 30
2010/26 Exercise 12.30
4! (n – 3)! 3!
Find the least value of n for which 3 n C 2 > 0, n R.
Changing division to multiplication
n!  (n – 3)! 3! = 30 1 1 2
A B C D1
4! n! 6 3 3
n! will cancel out
1  (n – 3)! 3! = 30 2009/40 Neco Exercise 12.31
4! 1 If 2n-3C4 = 2n-3C7, find the value of n.
But recall that 4! is same as 4  3! A. 7 B. 8 C. 9 D. 10 E. 11
1  (n – 3)! 3! = 30
4(3!) 1 2009/43 Neco Exercise 12.32
1
/4 (n – 3)! = 30 Given that nC6 = n+1C5, find the value of n.
Cross multiply A. -0.5 or – 1.4 B. -1 or – 14 C. 1 or – 14
(n – 3)! = 30  4 D. 1 or 14 E. 2 or 16
(n – 3)! = 120
Express both sides to factorials by changing 120 2002/30 (Nov) Exercise 12.33
(n – 3)! = 5! For what value of n is n + 1C3 = 7 ( nC2)?
Equating terms A. 16 B. 20 C. 21 D. 22
n–3=5
n = 5 + 3 i.e 8 2003/28 Neco Exercise 12.34
Evaluate 6C2 + 6C3 + 6C4 + 6C5
2002/33 Neco A. 6C6 B. 6C5 C. 7C5 D. 8C6 E. 8C5
Evaluate the expression
6
P3 – 6C3
A. 20 B. 40 C. 60 D. 80 E. 100
146
Chapter Thirteen E.g 3 The ages of 25 students in JSS I of Ibru college
Statistics I (ungrouped data) Agbarha – Otor is given as follows;
Statistics is the science of collecting, organizing and 7 8 10 15 8 9 7
analyzing data for any given purpose. It helps us to 8 8 9 10 12 7 13
reduce large and scattered data to an understandable 14 8 8 9 10 15 8
level, thereby enabling us to make decisions in the face 7 10 11 11
of uncertainty. At one time or the other in our day to arrange the above data in a frequency table.
day activities, we must have engaged statistical values Solution
in taking decision. Arranging the above data in a frequency table.
For instance, Musa’s parent wants to send Musa to a Ages Tally No of student ( freq.)
college in Garki – Abuja. You will agree with them that 7 IIII 4
there are so many schools to choose from. Through 8 IIII II 7
enquiry about various schools qualities such as : 9 III 3
(i) School performance in JSCE & SSCE 10 IIII 4
(ii) Qualification of teaching staff 11 II 2
(iii) School location 12 I 1
(iv) Fees charged, 13 I 1
They will make a good decision on the best school that 14 I 1
suit them. By the above act, they have engaged in the 15 II 2
statistical activities of collecting, organizing and analysis Total = 25
of data for the purpose of sending their child to school.
The centrality and spread of any given data are of great
Population importance to statisticians. These two mentioned concepts
This is the totality of observation with which we are enable us to make appropriate decision.
concerned. The whole Secondary Schools in Garki-
Abuja forms the population of the above example. Measure of central tendency
(measure of location)
Sample Measure of central tendency can be thought of as a measure
It is basically the subset of population. You will agree which given the location of the “center” of the data. The most
with the author that it will not be Naira (Penny) wise for commonly used ones are the mean, median and mode.
Musa’s parents to check on all the schools in Garki –
Abuja. Rather they will check on a few, say about 3 to 8
schools (sample) among the over 50 secondary schools
MEAN
The mean (arithmetic mean) denoted by x ( x bar) of a set n
Raw data
numbers x1, x2, … xn is
They are collected data which have not been organized
x = x1 + x2 + … + xn
numerically, i.e they have not been arranged either in
n
ascending or descending order.
= x (where  is summation)
E.g The scores of 10 students who sat for a
n
mathematics test are: 3, 5, 10, 0, 1,8, 10, 4, 2, 6.
E.g 4 Find the mean of the following set of numbers 4,7,3,6 and 0
ARRAY Solution
This is the arrangement of raw numerical data in Mean ( x ) =  x
ascending or descending order of magnitude. From the n
data supplied above: =4+7+3+6+0
Ascending order: 0, 1, 2, 3, 4, 5, 6, 8, 10, 10 5
Descending order: 10, 10, 8, 6, 5, 4, 3, 2, 1,0. = 20 = 4.
5
Frequency & Tally
Frequency: this is the number of occurrence of any value E.g 5. The following are the measurements of the diameter of
in a given data while tally is the stroke representation of a set of cylinders. Find the mean diameter of the cylinders 2.2,
frequency where : 3.1, 1.3, 1.0, 2.4, and 3.0 (in mm)
I represents 1 Solution
II represents 2 Mean =  x
III represents 3 n
IIII represents 4 = 2.2 + 3.1 + 1.3 +1.0 + 2.4 + 3.0
IIII represents 5 6
IIII I represents 6 = 13.0  2.17 mm
IIII II represents 7 and so on. 6

147
E.g 6 If the mean of 4, 6, 9, y, 16 and 19 is 13, what is E.g 7. If the mean of 30 observations in an experiment is 5.
the value of y? And one of the observed values is 34, find the mean of the
Solution remaining observations.
Mean = x Solution
n Let k be the sum of the observations. Then
13 = 4 + 6 + 9 + y + 16 +19 Mean =  x
6 n
13 = 54 + y 5= k
6 30
78 = 54 + y k = 150
78 – 54 = y Thus y = 24 The observed value 34 is to be subtracted from 150 values
and 1 observation from the 30 observations.
2002/17 Neco (Nov) Ordinary Maths
The average of x, 3, (x –1), 5 and (2x – 2) is 5, find the Thus, the remaining observation mean = 150 – 34
value of x. 30 – 1
A.6 B.5 C.4 D.3.5 E. 0 = 116 = 4
Analysis and Solution 29
Average = x + 3 + (x – 1) + 5 + (2x - 2) = 5 1995/47 Ordinary Maths
5 The mean of 20 observations in an experiment is 4. If the
observed largest value is 23, find the mean of the remaining
x + 3 + x – 1 + 5 + 2x – 2 = 5 observations.
5 A. 4 B. 3 C. 2.85 D. 2.60 E. 2.56
4x + 5 = 5 x 5 i.e 25 Analysis and Solution
4x = 25 – 5 i.e 20 Let k be the sum of the observation i.e  x. Then,
x = 20 / 4 = 5 (B) From mean =  x
2004/2a (Nov) Ordinary Maths n
The marks obtained by a student in an examination are 4= k
27, 32, 33, 45, 59, 63, 75 and 90. Calculate the mean 20
Solution k = 80
Mean x = x The largest value 23 is to be subtracted from 80 and one
n observation from the 20 observations. Thus,
= 27 + 32 + 33 + 45 + 59 + 63 + 75 + 90 Remaining observation mean = 80 – 23
8 20 – 1
= 424 = 53 = 57 = 3 (B)
8 19
1994/47 Ordinary Maths
2004/21 Ordinary Maths The mean heights of the three groups of students consisting
Find, correct to two decimal places, the mean of of 20, 16 and 14 students are 1.67 m, 1.50m and 1.40m
9, 13, 16, 17, 19, 23, 24. respectively. Find the mean height of all
A. 23.00 B. 17. 29 C. 16. 50 D. 16. 33 the students.
Solution A. 1.52m B. 1.53m C. 1.54m D. 1.55m E 1.56m
Mean ( x) =  x Analysis and Solution
n Let x, y and z be the total heights for each of the three
= 9 + 13 + 16 + 17 + 19 + 23 + 24 groups of students. Then
7 From mean =  x
= 121 n
7 X Y Z
= 17 . 286  17. 29 to 2 d . p (B) x =1.67 y = 1.50 z =1.40
2004/31 Ordinary Maths 20 16 14
Find the average of the first four prime numbers greater x = 33.4 y = 24 z = 19.6
than 10 Thus, the mean height of all the students.
A. 20 B. 19 C. 17 D. 15 Mean = x + y + z
Solution 20 + 16 + 14
Recall that: prime numbers has two factors only, one = 33.4 + 24 + 19.6
and itself. Thus, the first four prime numbers greater 50
than 10 are: 11, 13, 17, and 19. = 77 = 1.54m (C)
Mean (average) = 11+ 13 + 17 + 19 50
4 Note:
= 60 = 15 (D) The Solution is not mean = 1.67 + 1.50 + 1.40 = 1.52m (A) wrong
3
4
148
1991/34 Ordinary Maths 2002/20 Neco (Nov) Exercise 13.4 Ordinary Maths
The mean of 30 observations recorded in an experiment The average age of 12 boys and 18 girls in a class are 16
is 5. If the observation largest value of 34 is deleted, years and 15 years respectively.
find the mean of the remaining observations Calculate the average age of the student in the class.
A. 5 B. 4 C. 3.9 D. 3.4 E. 3.0 A. 15.4 years B. 15.5 years C. 15.6 years
Analysis and Solution D. 16 years E. 18 years.
Let the sum of the observations be y; then, 2007/43 Neco Exercise 13.5
Mean =  x An elitist club has 11 members. Their mean age was 50years.
n A member died and the mean age of the remaining members
5 = y was 48 years. If the late member was twice the age of his son
30 who replaced him in the club now, calculate the mean age of
5 x 30 = y the club now to the nearest year.
y = 150 A 48 years B 47 years C 46 years D 45 years E 44 years
But if 34 is deleted, then the value 34 is subtracted from 2010/21 Exercise 13.6
150 and one observation is taken away from the 30 The mean age of 15 pupils in a class is 14.2 years. One new
observations. Thus pupil joined the class and mean changed to 14.1 years.
Mean = 150 – 34 Calculate the age of the new pupil.
30 – 1 A 12.4 years B 12.6 years C 13.2 years D 14.1 years
= 116 = 4 (B)
29
2006/16 Exercise 13.7
The mean age of n men in a club is 50 years. Two men, aged
2004/58 Neco Ordinary Maths 55 and 63, left the club and the mean age reduced by 1 year.
Four pupils have an average age of 12 years and two Find the value of n.
other pupils of average age of 10. 5 years are added. A 30 B 20 C 18 D 14
What is the average age of the six pupils
A. 10.5 years B. 11 years C. 11.5 years Basic properties of Arithmetic mean
D. 12 years E 12 .5 years 1. Weighted arithmetic mean.
Solution This is a case where certain degree of importance is attach to
Let R be the sum total ages of the four students and a subject. For instance university courses has credit load
Let S be the sum total ages of the other two students attached to them say 5 credit, 3 credit and so on
Mean = x Case I
n E.g A student in one hundred level math dept of unijos has
12 = R ; R = 4 x 12 Similarly, 10. 5 = S ; S = 2 x10.5 the following scores; MTH 101 70%,Use of English 40%,
n 2 STA 101 65% , PHY 101 60% , GNS 102 30%. If the
Average of six pupils credit loads of the courses are 5, 3, 4, 2 and 1 respectively.
= 4 x 12 + 10. 5 x 2 Find the average grade.
6 Solution
= 48 + 21 = 69 70  5  40  3  65  4  60  2  30  1
Average grade =
6 6 5  3  4  2 1
= 11. 5 yrs (C) 350  120  260  120  30
=
2001/55 Neco Exercise 13 .1 Ordinary Maths 15
If the mean of 3, 5, 8, k, 14 and 17 is 11, what is the 880
=  58.67%
value of k? 15
A. 58 B. 38 C. 19 D. 11 E. 9.67 Case II
A student in hundred level math Dept. in UniLag has the
2002/55 Neco Exercise 13. 2 Ordinary Maths following scores; MTH 101 A, use of English E, STA 101B,
The mean heights of 3 groups of students consisting of PHY 101 B, GNS 102 F. If the credit load of the courses are
20, 16 and 14 students are 1.67m ,1.50m and 1.40m 5, 3, 4, 2and 1 respectively. Find the average grade.
respectively What is the mean height of all the students? Note: A = 5, B = 4, C = 3, D 2, E = 1, F = 0
A. 1.52m B. 1.53m C. 1. 54m D. 1.55m E. 1.56m Solution
5  5  3  1  4  4  2  4  1 0
Average grade =
2001/56Neco Exercise 13 .3 Ordinary Maths 5  3  4  2 1
The mean age of 12 boys involved in a survey is 19 25  3  16  8  0
=
years, 3 months. If the age of one of the boys is 22 year, 15
what is the mean age of the other boys? 52
A.10.5 years B. 19.0 years C. 35.4 years = i.e. 3.47
15
D. 264.0 years E. 423.5 years

149
Generally, if x1, x2, x3, … xn occurs with weights w1,
55 = 150  60  100  g
w2, w3, … wn respectively, then the weighted arithmetic 250
mean
w1 x1  w2 x2  w3 x3  ...  wn xn 55 × 250 = 9000 + 100 g
x =
w1  w2  w3  ...  wn 13750 – 9000 = 100 g
 wx 4750 = 100 g
x =
w
(2) The algebraic sum of the deviations of a set number g = 47.50
from their arithmetic mean is zero i.e.
2005 / 27
 d = 0 where d = x – x If the mean of the numbers x1, x2, x3, … xn is k,

 x 
n
Find the mean of the numbers (3x1 + 2),(3x2 + 2),(3x3 + 2),
better still i x =0 … (3xn + 2)
i 1
A 9k + 6 B 3 (k + 2) C 3k + 2 D 9k + 2
(3) If x1, x2, x3, … xn has mean x , then any multiple Solution
of set of x i say Applying the basic properties of Arithmetic mean
Kx1, Kx2, Kx3, … Kxn is K x Mean = 3k + 2 (C)

(4) If x1, x2, x3, … xn has mean x , then the mean of 2004 / 37
The mean of 12 numbers is 18. If each of the number is
the numbers (3x1 - 1), (3x2- 1), (3x3 - 1), … (3xn- 1)
increased by 5, what is the new mean?
is (3 x -1) A 13 B 17 C 18 D 23
Solution
CASE III Applying the basic properties of Arithmetic mean
If a1, numbers have mean d1, a2 numbers have mean Mean = 18 + 5
d2,… = 23 (D)
an numbers have mean dn. Then the mean of all the set
of numbers is 1994 / 30
a1d1  a2 d 2  ...  an d n A set of two numbers P = {x1, x2} has mean 0.5 while
a = another set of three numbers Q = {x3, x4, x5} has mean
a1  a2  ... an
3.0. Find the mean of the set R = { x1, x2 , 2x3, 2x4, 2x5}
Eg. A class of 20 girls took a test in English; 5 of them A 6.5 B 6.0 C 4.0 D 3.8 E 2.375
scored 58 marks each, 6 of them scored 47 marks each Solution
and 9 girls scored 37 marks each. Calculate the mean Here we apply basic properties of arithmetic mean (3) and case III of (1)
mark. P ={ x1, x2 }, P = 0.5
Solution
58  5  47  6  37  9 905 Q = { x3, x4, x5 }, Q = 3.0,
Mean = = = 45.25 Thus part of R where Q1 ={2x3, 2x4, 2x5}
569 20
Q 1 = 2 ( Q ) i.e. 2 (3.0)
2002 / 14a (i) Neco Applying case III
The mean of a1 numbers is d1and that of a2 number is d2. 2  0.5  3  2  3
R =
What will be the mean of the set derived from a 5
combination of the two sets of numbers? 19
Solution = i.e. 3.8 (D)
5
By basic properties of arithmetic mean
a1d1  a 2 d 2 1994 / 29 (Nov)
Mean of the set of numbers =
a1 a 2 If x denotes the sample mean of the random sample
n
2002 / 14a (ii) Neco
In an examination, the mean score of 250 students x1, x2, x3, … xn , the value of i 1
(x i - x ) is
tested was 55. Of the 250 students, 150 were boys. If
the mean score of the boys was 60, what was the mean A -x B0 Cx Dn x Exi
score of the girls? Solution
Solution By the basic properties of arithmetic mean
Applying Arithmetic mean properties to the two set of n
mean of boys (150) and girls (100) Case III  (x i - x ) = 0 i.e.  d = 0 (B)
a1d1  a2 d 2  ...  an d n i 1
a =
a1  a2  ... an

150
1996/22 Exercise 13.8 Solution
If two samples consisting of 7 and 3 observations Mean =  fx
fx
have means x and y respectively, then the mean of the
x f fx
combined sample is 12 5 60
     
3 y7 x x y 7 x 3 y 13 12 156
A B C 14 10 140
10 10 4 15 3 45
 
16 3 48
7 y 3x
D3 y +7x E 17 5 85
10 18 2 36
2003/15 (Nov) Exercise 13.9 f = 40 fx = 570
The mean of the set { x1, x2, x3 } is 4 and the mean of Mean = 570 = 14.25 years
the set { x4, x5} is 1. Calculate the mean of the 40
set {2x1, 2x2, 2x3, x4, x5}. But 0. 25 years = 0. 25 x 12 months = 3 months
A 6.5 B 5.2 C 4.0 D 2.8 Thus, mean = 14yrs . 3 months (B)
2007/41 Neco Exercise 13.10
If the mean of K1, K2, K3, … Kn is 8, then the 2004/5a(i) Ordinary Maths
1 1 1 1 The table shows the marks scored by a group of students in
mean of K 1 – , K2 – , K3 – , … Kn – a class test.
4 4 4 4
is Marks 0 1 2 3 4 5
33 31 1 Frequency 1 4 9 8 5 3
A B C4 D2 E
4 4 32 Calculate the mean marks
Solution
Mean of frequency distribution Mean (x) = fx
For any given data say x1, x2, …, xk that occur f1, f2, f
…,fk times respectively, then the mean marks (x) f fx
0 1 0
x = f1x1 + f2x2 + … + fk xk
1 4 4
f1 + f2 + …+ fk
2 9 18
= fx
3 8 24
f 4 5 20
E.g The following are the sizes of shoes worn by a 5 3 15
group of students: 9,7,6,10,11,6,8 ,8 ,9 ,11 ,7 ,8 9, 8, 11, f = fx = 81 Mean = 81 = 2.7
8, 6, 9, 8, 6 . Find the mean size of shoe 30 30
Solution 2004/12c NABTEB (Dec) Ordinary Maths
First, we arrange the given data in a tabular format. Find the mean of the distribution of the test scores below.
Shoe size (x) Tally No. of stud fx Score 30 35 40 45 50
6 IIII 4 24 No of student 8 27 41 15 9
7 II 2 14 Mean = fx
8 IIII I 6 48 f
9 IIII 4 36
10 I 1 10 x f fx
11 III 3 33 30 8 240
f = 20 fx =165 35 27 945
Mean = fx 40 41 1640
f 45 15 675
= 165 = 8.25 50 9 450
20 f =100 fx = 3950 Mean = 3950 = 39.50
100
2001/54 Neco Ordinary Maths Instead of using table, it is also allowed to go this way
Below are the ages of 40 students in form V1. Alternatively
Age 12 13 14 15 16 17 18 = 30 x 8 + 35 x 27 + 40 x 41 + 45 x 15 + 50 x 9
No of 5 12 10 3 3 5 2 8 + 27 + 41 + 15 + 9
students = 240 + 945 + 1640 + 675 + 450
100
Using the above table, what is the mean age of the
= 3950
students?
100 = 39. 50
A 12yrs B 14yrs. 3 months C 14yrs. 6months
D 15yrs E 15yrs . 3month.
151
1992/10b Exercise 13.11 Ordinary Maths 1996/22 (Nov) Exercise 13.13
The table below shows the frequency distribution of The numbers of words per line on a page of a book is
marks scored by 30 candidates in an aptitude test. recorded as follows:
Marks 4 5 6 7 8 9 No. of words per line 6 7 8 9 10
Number of candidates 5 8 5 6 9 2 No. of lines 4 10 13 x 5
Find the mean score to the nearest whole number
If the mean is 8.0 words per line, calculate the value of x.
A6 B7 C8 D9 E 10
2000/11 Neco Exercise 13.12
Find the mean of the distribution 2002/12 (Nov) Exercise 13.14
Class Mark 61 64 67 70 73 The table below shows the distribution of the ages of pupils
Frequency 5 18 42 27 8 in a School.
Age (in years) 10 11 13 14 18
A 61.5 B 64.45 C 67.00 D 67.45 D 70.00
Frequency 4 x 4 3 1
Miscellaneous problems on mean If the mean age of the pupils is 12 years, find x.
E.g If the mean of the frequency distribution A3 B4 C5 D8
below is 5.2, find y
Median
Class mark 2 4 6 8
The median of a set of number arranged in an array is the
Frequency 4 y 6 5
middle number (if their total number is odd) or the arithmetic
Solution
mean of the two middle number (if the number of the given
Mean = fx values is even). Median divides the data into two equal parts.
f
E.g 1 Find the median of the following
5.2 = 4 x 2 + y x 4 + 6 x 6 + 5 x 8
(a) 6, 1, 0, 3, 10, 2, 5
4+y+6+5
(b) 5, 7, 11, 9, 15, 12, 18, 6
5.2 = 8 + 4y + 36 + 40
Solution
y + 15
Arranging the value in array, we have:
Cross multiplying , we have
(a) 0, 1, 2, 3, 5, 6, 10
5.2 (y + 15) = 84 + 4y
The given values is odd, thus median = 3 (middle)
5.2y + 78 = 84 + 4y
(b) 5, 6, 7, 9, 11, 12, 15, 18
5.2y - 4y = 84 – 78
The given values is even, thus
1.2y = 6
Median = 9 + 11 Arithmetic mean of the two
y=6
2 middle values
1.2
= 10
=5
1992/38 Ordinary Maths
1997/47 Ordinary Maths Find the median of the following set of scores :
The table shows the scores of a group of students in the 65, 72, 55, 48, 78.
test. If the average score is 3.5, find the value of x A. 55 B. 50 C. 60 D. 65 E. 72
Solution
Scores 1 2 3 4 5 6
The given data is raw i.e not arranged. Thus arranging in
No of students 1 4 5 6 x 2 ascending order 48, 55, 65, 72, 78
A. 1 B. 2 C. 3 D. 4 E.5 Since there are 5 (odd) scores
Analysis and Solution Median = middle term 65 (D)
Average score = mean
2005/10
= fx
The mean of the numbers 4, 6, 4, 7, (x + 1), 8, and 2 is 5.
f Find the median of the numbers.
3.5 = 1 × 1 + 4 × 2 + 5 × 3 + 6 × 4 + x × 5 + 2 x 6 A4 B 4.5 C5 D 5.5
1+4+5+6+x+2 Solution
3.5 = 1 + 8 + 15 + 24 + 5x + 12 To find median, we first find the value of x +1 from
18 + x x
3.5 = 60 + 5x Mean =
1 18 + x n
60 + 5x = 3.5 (18 + x) 4  6  4  7   x  1  8  2
5=
60 + 5x = 63 + 3.5x 7
5x – 3.5x = 63 – 60 Cross multiply
1.5x = 3 5  7 = 32 + x
x= 3 35 – 32 = x
1.5 Thus x + 1 = 4
= 2 (B)
152
To find median, we arrange the given numbers in Solution
ascending or descending order f = 7 + 21 +12 + 10 + 8 + 2 i.e 60 is even
8, 7, 6, 4, 4, 4, 2 Median class = f
The numbers are odd i.e.7, the middle number here is 4(A) 2
2009/33 Exercise 13.15 = 60 i.e 30th class
If the mean of –1, 0, 9, 3, k, 5 is 2, where k is a 2
constant, find the median of the set of numbers. Adding the frequency from the upper side till we get
A0 B 3/ 2 C 7/2 D6 to 30th class 7 + 21 + 12 i.e 3rd column, where
we have 30th class.Thus, median = 7 Not given
2002/31(Nov) Exercise 13.16 Ordinary Maths
The numbers –15, -11, -5, m, 3 and 8 are arranged in Example 3
ascending order of magnitude. If the median is – 3, The distribution of marks in a mathematics test is shown
find the value of constant m. below:
A. – 1 B. 0, C. 1 D. 2 Marks 0 1 2 3 4 5 6 7 8 9 10
Frequency 2 0 0 2 3 6 3 2 2 1 0
1995/48 Exercise 13.17 Ordinary Maths
Find the median of the following numbers: Use the data above to answer questions 7 & 9
2.64, 2.50, 2.72, 2.91 and 2.35 7. How many candidates took the test?
A. 2. 91 B. 2.72 C. 2.64 D. 2.50 E. 2.35 A. 20 B. 21 C. 22 D. 23 E. 24
Solution
Median (frequency distribution) How many candidates took the test simply means addition of
all the frequency i.e
To get median for frequency data, we apply the idea of f = 2 + 0 + 0 + 2 + 3 + 6 + 3 + 2 + 2 + 1 + 0
odd and even numbers to get the median class or = 21 (B)
column among other columns or classes. f is another
9. What is the median score ?
important term for us to know, which is “ Summation
A. 3 B. 4 C. 5 D. 6 E. 7
of frequency”. The two ways of getting our median
class or column are: Solution
f is 21 from question 7. Since it is odd
(a) Median class (column) = f if f is even Median class = f + 1
2 2
(b) Median class (column) = f + 1 if f is odd = 21 + 1
2 2
After we have gotten our median class, then keep adding the = 22 = 11th class
frequency of the given data till we get to the class or column 2
where our number falls in – that position is our median. Adding the frequency; 2 + 0 + 0 + 2 + 3 + 6,
Examples on this will help us better 11th class is in the 6th column with mark 5
Example 1 Thus, median = 5 (C)
The table below shows the number of days of
absenteeism during a term for a class of 30 students. VTR – 13/49 NTI TCII Ordinary Maths
Use the table to answer question 33
Score 0 1 2 3 4 5 6 7
Days Absent 0 1 2 3 5 35 Frequency 3 2 3 4 5 7 2 1
Frequency 14 5 4 3 3 1
The above table shows the scores obtained by a group of
33 From the table, what is the median ?
pupils in mathematics test.Find the median scores
A. 35 B. 5 C. 3 D. 1 E. 0
A. 3 B. 4 C. 5 D. 6 E. 7
Solution
Solution
f = 30 since it is even
 f = 3 + 2 + 3 + 4 + 5 + 7 + 2 +1
Median class = f = 27 odd
2
Thus, median class =  f + 1
Median class = 30
2
2 = 15th class
= 27 + 1 = 28
Tracing 15 along the frequency 14 + 5, 15th class is in
th
2 2
the 2nd column. Day 1
Adding the frequencies till we get to the 14th class:
Thus, median = 1 (D)
3+2=5
Example 2 5+3=8
Use the table to answer question 57. 8 + 4 = 12
12 + 5 = 17
Number 5 6 7 8 9 10
14 falls into
Frequency 7 21 12 10 8 2
That is the 5th class starting from the frequency side. The 5th
57. Calculate the median class score is 4
A. 10 B. 11 C. 12 D. 13 E. 14 Thus, Median = 4 (B)
153
1998/24(Nov) Exercise 13.18 Ordinary Maths Mode of frequency distribution
Age (in years) 4 5 6 7 8 9 10 E.g. The mass of 100 students is as shown below:
Frequency 2 1 2 4 3 6 2 Mass (kg) No of student
The table shows the age of distribution of a group of 52 5
children. Find the median age. 55 18
A. 9 years B. 8 years C. 7 years 58 42
D. 4 years E. 3 years 61 27
64 8
2002/ 57 Neco Exercise 13.19 Ordinary Maths
Below is a table showing the age of 40 SS3 Find the modal mass.
students in Ayegbe Comprehensive High School Solution
Age 12 13 14 15 16 17 18 Mode = 58 kg (it has the highest frequency)
No of students 5 12 10 3 3 5 2
2004/55 Neco Exercise 13.23 Ordinary Maths
Using the above table, calculate the median age of the The table below shows the number of goals scored by 32
students. football teams in a world cup tournament.
A. 13 years 6 months B 14 years 3 months No of goals 1 2 3 4 5 6 7
C. 14years D. 15years E. 15 years 3 months No of teams 4 1 8 7 5 4 3
Find the modal goal
2004/34 Exercise 13.20 Ordinary Maths A. 3 B. 4 C. 5 D. 7 E.8
The table below gives the marks scored by a group of
students in a test Measures of Spread (variation/dispersion)
Mark 0 1 2 3 4 5 This is the degree to which data are scattered (dispersed) about
Frequency 1 2 7 5 4 3 its average values. The several measures of spread are the
range, mean deviation, inter quartile range, variance and
What is the median mark ? standard deviation
A. 1 B. 2 C. 3 D.4
1991/33 Ordinary Maths
Mode Which of the following is not a measure of dispersion?
The mode of a set of numbers is that value which occurs A. Mode B. Mean deviation C. Range
with greatest frequency. i.e. it is the most common value. D. Interquartile range E. Standard deviation
The mode may not exist or it may not be unique (when Solution
we have more than one mode) Mode is not a measure of dispersion (A)

E.g. 1. Find the mode of the following set of numbers: 2002/26 ( Nov) Exercise 13.24 Ordinary Maths
(a) 2, 5, 7, 10, 8, 12, 8, 9 Which of the following is / are not the measures (s) of
(b) 2, 3, 4, 7, 9, 8 dispersion?
(c) 1, 2, 4, 4, 3, 3, 4, 7, 9, 7, 7 I Range
Solution II Variance
(a) Mode = 8 III mode
(b) The mode does not exist A 1 only B. II only C. III only D. 1 and II only.
(c) Mode = 4 and 7 (Bimodal)
Range
1995/46 Exercise 13.21 Ordinary Maths This is the simplest measure of dispersion and is defined as the
What is the mode of the numbers difference between the largest and smallest number in the set.
8, 10, 9, 9, 10, 8, 11, 8, 10, 9, 8, and 14? E.g1 Find the range of the following set of numbers,
A. 8 B. 9 C. 10 D. 11 E. 14 (i) 2, 4, 5, 7, 9, 11, 13, 17, 15
(ii) 7, 7, 2, 6, 6, 17, 7, 6, 6
1992/42 Exercise 13.22 Ordinary Maths Solution
A group of students measured a certain angle (to the (i) range = 17- 2 = 15
nearest degree) and obtained the following results:
750, 760, 720, 730, 740, 790, 720, (ii) range = 17 – 2 = 15
0 0 0 0 0
72 , 77 , 72 , 71 , 70 , 780, 730. Note
42. Find the mode Range is a poor measure of spread as it uses only two
A.790 B. 780 C. 740 D. 730 E. 720 extreme values.

154
2002/19/Neco (Nov.) Ordinary Maths Solution
What is the range of these numbers: ½, 9/4 3
/ 4 , 1/ 4 , 1 ? M.D =   x – x 
2 -1 n

1 3 3
A. 2 /4 B. 2 C. 1 /4 D. /4 E. ½
Analysis and Solution mean x = 0 + (-1) + (-3) + 4 + 5 + 1
The numbers are ½ , 3/2, 3/4, 1/4 and 2 ; to know the 6
biggest number multiply through by their Lcm i.e 4 = -1–3+4+5+1
4 × ½, 4 × 3/2, 4 × 3/4, 4 × 1/ 4, 4 × 2 6
6
2 6 3 1 8 = /6
Range is between 8 and 1 i.e = 2 – 1/4 =1 Thus,
= 7/4 i.e 13/4 (C) x x -x /x - x /
2002/59 Neco Exercise 13.25 Ordinary Maths 0 -1 1
What is the range of the following scores; -1 -2 2
17.1, 50.3, 6.27, 33.2, 15.9, 66.05 and 24? -3 -4 4
A 59.78 B 44.03 C 29.89 D 24.0 E 15.75 4 3 3
5 4 4
2002/12a(i) NABTEB Exercise 13.26 Adapted 1 0 0
The profit made by a hawker in 5 consecutive days of
 /x – x / =14
selling are: ₦8, ₦15, ₦20. ₦12, ₦15 .What is the
range of the profits ?
Mean deviation = 14 = 7/3 (C)
6
Mean deviation (M.D)
The mean deviation of a set of n numbers x1, x2,… xn is 2005/9 Exercise 13.27
defined as: Find the mean deviation of the numbers 2, 5, 3, 6, 8, 6,
M.D = x – x A 1.30 B 1.40 C 1.67 D 1.75
n 2004/5 Neco Exercise 13.28
Where x is the mean of the numbers and  x - x is the Calculate the mean deviation of the following data:
absolute value of the deviation of x from the mean ( x ) 3, 5, 8, 11, 12, and 21
and n is the number of digits given A 28.0 B 18.3 C 10.0 D 8.5 E 4.7

E.g1.Find the mean deviation of numbers: 4, 6, 5, 8, 12 2004/28 Exercise 13.29 Ordinary Maths
Find the mean deviation of 6, 7, 8, 9, 10.
Solution A. 1.2 B. 1. 5 C.2 D.8
M.D =   x – x 
n
Mean Deviation (frequency Distribution).
But x = x = 35 = 7
If x1, x2 ,… xk occur with frequency f1, f2 ,… fk
n 5
then the mean deviation is given by
M.D = /4 – 7/ + /6 – 7/ +/5 – 7/ + /8 – 7/ + /12 – 7/
5
M.D =  f /x – x/
= / -3 / + / -1/ + / -2/ + /1/ +5/
5 f
=3+1+2+1+5 Variance & Standard deviation.
5 The variance of a set of numbers x1, x2 … xk is given as
= 12 = 2.4 V =  ( x – x )2
5 n
Alternatively Where V is the variance and (x – x )2 is the square of the
x x–x /x – x/ derivation from the mean while standard deviation is the
4 -3 3 square root of variance.
6 -1 1
5 -2 2 S.D =  (x – x )2
8 1 1 n
12 5 5 If the numbers x1, x2 ….xk occur with frequency
 /x – x/ f1, f2 …, fk respectively, then,
= 12
M.D = 12 = 2.4 Variance V =  f (x – x )2
5 f
2002/38 (Nov) Ordinary Maths
Calculate the mean deviation of the numbers S.D = f (x – x )2
0, -1, -3, 4, 5, 1 f
A. 0 B. 2 C.7/3 D. 24/5
155
Some basic arithmetic properties of variance EgV1. Find the variance of the numbers, 2, 5, 6, 3 and 4
and standard deviation Solution
(i) If a set of numbers x1, x2, x3,…xn has a mean x and Variance =  (x – x )2
variance 2 then a new set of numbers x1 + 3, x2 + 3, n
… x n+3 has a mean x + 3 but variance remain But mean = 20 =4
unchanged, the same applies to Standard deviation. 5
(ii) If a set of numbers x1, x2, x3,…xn has a mean x x x-x (x – x )2
2 -2 4
and variance 2 then a new set of numbers 5x1, 5x2,
5 1 1
5x3,…5xn has mean 5 x and variance 52 2 and its 6 2 4
standard deviation can be deduced from it 3 -1 1
1996 / 26 (Nov) 4 0 0
The variance of the Sample 7, 5, 9, 8 and 6 is 5/3  (x – x )2
Find the variance of the sample 10, 8, 12, 10, 11 and 9. = 10
A 5/ 9 B 5/3 C125/27 D14/3 E 5 Var = 10 = 2
Solution 5
Observe that 7, 5, 9, 7, 8, 6 is basically +3 to the data
1996 / 29
10, 8, 12, 10, 11, 9
The table gives the distribution of marks obtained by a group
and by basic arithmetic property of variance the
of 10 students in a test. Calculate the variance of the
variance for the two set of numbers is the same i.e 5/3 (B)
distribution.
1993 / 26 (Nov) Marks 1 3 5 7 9
Calculate the variance of the numbers 19, 21, 27 and 37 No. of Students 4 2 2 1 1
A0 B7 C 14 D26 E 49
Solution A 33.16 B 20.20 C 12.96 D 7.24 E 2.69
Variance =  ( x – x )2 Solution
n For frequency data
But mean = x = 19 + 21 + 27 + 37 = 104 i.e. 26 Variance = f (x – x )2
n 4 4 f
But Mean = 14+32+52+71+91 = 36 i.e 3.6
Var = (19-26)2 + (21-26)2 + (27-26)2 + (37-26)2 4+2+2+1+1 10
4 We prepare a table to reflect the formula items for
= (-7)2 + (-5)2 + (1)2 + (11)2
variance x, x- x and (x - x )2, f, f(x - x )2
4
= 196 i.e. 49 (E) x x -x (x - x )2 f f (x - x )2
4 1 – 2.6 6.76 4 27.04
2000 / 29 3 – 0.6 0.36 2 0.72
Calculate the variance of the following quantities: 5 1.4 1.96 2 3.92
(x – 2), (x + 1), (x + 2) and (x + 3) 7 3.4 11.56 1 11.56
1 1 1 1 9 5.4 29.16 1 29.16
A4 B3 C 3 D1
2 2 3 2 f f (x - x )2
Solution =10 = 72.4
For simple data like this ones
72 .4
Variance =  ( x – x )2 Variance = = 7.24 (D)
n 10
But mean = x = (x-2)+(x+1)+(x+2)+(x+3)
2004/47 Neco Exercise 13.30
n 4 The variance of the numbers 2, 3, 5, 9, 11 is
= 4x + 4 = x + 1 A 12.00 B 6.00 C 3.46 D 3.00 E 0
4
x x– x (x – x )2 Exercise 13.31
x–2 x –2 – (x +1) = –3 9 Calculate the variance of the frequency distribution below.
x+1 x + 1– (x +1) = 0 0 Mark(x) 1 2 3 4 5
x+2 x +2– (x +1) = 1 1 frequency 2 1 2 1 2
x+3 x +3 –(x +1) = 2 4
(x - x ) = 14
2
2003/10 Exercise 13.32
1
Thus, Variance = 14 = 3 /2 (B) Calculate the variance of the numbers 2 , (1 2 ), (2  2 ).
4
A0 B 2/3 C 2 D2
3

156
2002/29 Solution
Calculate the standard deviation of the numbers
–6, –2, 5, 7, 11
S.D =
 
 xx
2

A 2.76 B 3.00 C 5.80 D 6.16 n


Solution  X 1  y  1  2 y  1 3y  3
  2 But x = = = =y+1
 xx n 3 3
S.D = Next, we prepare a table to reflect the formula items i.e.
n
X  6   2  5  7  11 x, x - x , (x – x )2
But x = = x
n 5 x– x (x - x )2
15 1 1 – (y + 1) = – y y2
= i.e. 3 y+1 y + 1 – (y + 1) = 0 0
5
Next, we prepare a table to reflect the formula items, 2y + 1 2y +1 – (y + 1) = y y2
x, x - x and (x - x )2 (x – x )2 = 2y2
x
 
x -x (x - x )2 2
–6 –9 81  xx
–2 –5 25 S.D =
n
5 2 4
7 4 16 2y2
11 8 64 6 =
3
 (x - x )2 = 190 Remove square root sign from both sides
2
190 6 = 2y
Thus S.D = = 38  6.16 (D) 3
5 63
= y2
1997 / 24 2
A group of eight students obtained the following scores 9 = y2 Thus, y =  9 positive value is + 3 (C)
in a test: 66, 36, 72, 46, 87, 56, 79, and 94. Calculate the
standard deviation of the scores. 1999 / 9 (Nov)
A 352.69 B 67.00 C 58.00 D 18.78 E 17.88 The standard deviation of the numbers 1, 2, 3, 4, 5, 6, 7, 8, 9
Solution is 2.58. What is the standard deviation of the numbers 101,
102, 103, 104, 105, 106, 107, 108, 109?

S.D =
 
 xx
2

But x =
X =
536
= 67
A 0.258 B 2.580 C 25.80 D 102.58
By basic properties of variance and standard deviation stated
n n 8 earlier; the standard deviation of the second set is same as
 
Next we prepare a table to reflect, x, x- x and (x - x )2 2.58 (B)
1, 2, 3, 4, 5, and so on
x  
1+100, 2 +100, 3+100, 4+100, 5+100,
x- x (x - x )2
Readers to verify by calculating variance and standard
66 -1 1
deviation for the two set of data
36 - 31 961
72 5 25 2000 / 34
46 - 21 441
Marks 3 4 5 6 7 8 9
87 20 400
No. of Students 2 3 4 3 1 1 1
56 - 11 121
79 12 144
94 27 729 Using the above table, find the standard deviation
 A 1.66 B 5.33 C 6.23 D 8.42 E 80.00
 (x - x )2 = 2822 Solution
For frequency data
Thus, S.D =
2822
8
= 352 .75  18.78 (D)
S.D =

 f xx 2

But x =  fx
f f
3  2  4  3  5  4  6  3  7 1  8 1  9 1
2003 / 5 Neco x =
Find the positive value of y if the standard deviation of 2  3  4  3 111
1, y + 1 and 2y + 1 is 6 6  12  20  18  7  8  9 80
= = i.e. 5.3 =  5
A1 B2 C3 D4 E5 15 15

157
Next, we prepare a table to reflect, the formula items Solution
i.e. x, x - x , (x - x )2, f and f(x - x )2 Variance = f (x – x )2
f
x x- x (x - x )2 f f(x - x )2 But mean = 2 x 1 + 1 x 2 + 2 x 3 + 1 x 4 + 2 x 5
3 -2 4 2 8 2+1+2+1+2
4 -1 1 3 3 = 24 = 3
5 0 0 4 0 8
6 1 1 3 3 x x– x (x – x )2 f f (x – x )2
7 2 4 1 4 1 -2 4 2 8
8 3 9 1 9 2 -1 1 1 1
9 4 16 1 16 3 0 0 2 0
4 1 1 1 1
f  f (x - x )2 5 2 4 2 8
=15 = 43 8 18
V = f (x – x )2
43 f
Thus, S.D =  1.693 (A)
15 = 18
8
Variance and standard deviation
= 2.25
2002/3 ordinary maths
and S.D = square root of variance i.e 2.25 = 1.5
The variance of given distribution is 25. What is the
standard deviation?
A. 125 B. 75 C. 25 D. 5 2002 / 31 (Nov)
Solution 2
The variance of the numbers, 9, 7, 8, 5, 7, 6 is 1 . Find the
Standard deviation is the square root of variance. 3
Thus,SD = var iance standard deviation of 12, 10, 11, 8, 10, 9.
SD = 25 = 5 (D) 14 5 5 125
2000/20 ordinary maths A B C D
3 3 9 27
Calculate the variance and standard deviation of the
Solution
following marks: 2, 3, 6, 2, 5, 0, 4, 2.
If we observe the numbers carefully, you will see that
Solution
A = {9, 7, 8, 5, 7, 6}
Variance =  ( x – x )2 A i +3 = {12, 10, 11, 8, 10, 9}
n
Standard deviation is the square root of variance. By simple arithmetic properties of variance and standard

But here mean x = 3
deviation, their Std is same
Thus Std = var iance
Next, we prepare a table to reflect, the formula items
2 5
i.e. x, x - x and (x - x )2 = 1 = (B)
x
3 3
x-x (x - x )2
2 -1 1
3 0 0
6 3 9 2001/34 Neco Exercise 13.33
2 -1 1 What is the standard deviation of the following set of scores
5 2 4 to one decimal place: 2.1, 2.6, 3.1, 3.6, and 4.1?
0 -3 9 A 2.6 B 2.5 C 0.7 D 0.5 E 0.3
4 1 1
2 -1 1
2005/1 Exercise 13.34
(x – x )2 = 26 Calculate the standard deviation of 3, 4, 5, 6, 7, 8, 9
26 A 2.0 B 2.4 C 3.6 D 4.0 E 4.2
Variance = = 3.25
8
Thus, SD = var = 3.25 i.e 1.8 (C) 2009/11 (Nov) Exercise 13.35
Calculate, correct to one decimal place, the standard
Note:Candidate should note that 0- 3 = -3 and not 0
deviation of 7, 10, 6, 8 and 14.
Eg 1. Calculate the variance and standard deviation of A 1.3 B 2.8 C 2.9 D 3.3
the frequency distribution below.
x 1 2 3 4 5 2009/39 Exercise 13.36
The ages, in years, of 5 boys are 5, 6, 6, 8 and 10. Calculate,
f 2 1 2 1 2 correct to one decimal place, the standard deviation of their ages.
A 3.2 years B 2.6 years C 1.9years D 1.8 years

158
Miscellaneous cases involving measures of 1
Q1 = of Cum frequency (Total frequency)
spread & location 4
1994 / 13 3 1
Inter quartile range = (20) – (20)
The times, in hours, taken by 20 athletes in a cross – 4 4
country race is given as: = 15 – 5
3.2, 2.8, 2.5, 3.7, 2.3, 1.9, 2.9, 3.3, 2.8, 2.1, = 10
2.7, 3.8, 1.8, 2.0, 2.5, 2.4, 3.0, 2.5, 3.2, 1.9 2003/13a (Nov)
Calculate, correct to one decimal place, the The deviation of a set of numbers from an assumed mean 7
(a) mean are: –4, –2, –1, 0, 1, 3, 4, 5.
(b) standard deviation of the data Calculate, correct to two decimal places, the
Solution (i) mean;
2 (ii) standard deviation, of the numbers.
 fx   fx 
2
Mean =  fx and S.D =   Solution
f  f   f  By assumed mean formula
Next, we prepare a table to reflect the formula items x, (i) Mean = A +  f d
tally, f, fx, x2, fx2 f
x Tally f fx x2 f x2 d
Since no f, it reduces A + here  d = 6
1.8 I 1 1.8 3.24 3.24 n
1.9 II 2 3.8 3.61 7.22 6
2.0 I 1 2.0 4.00 4.00 =7+
2.1 I 1 2.1 4.41 4.41 8
2.3 I 1 2.3 5.29 5.29 = 7 + 0.75 i.e 7.75
2.4 I 1 2.4 5.76 5.76 2
2.5 III 3 7.5 6.25 18.75  f d2  f d 
2.7 I 1 2.7 7.29 7.29 (ii) Standard deviation =   
2.8 II 2 5.6 7.84 15.68 f  f 
2.9 I 1 2.9 8.41 8.41
3.0 I 1 3.0 9.00 9.00
Since we have no f it reduces to
2
3.2 II 2 6.4 10.24 20.48
d 2  d 
3.3 I 1 3.3 10.89 10.89
=  
3.7
3.8
I
I
1
1
3.7
3.8
13.69
14.44
13.69
14.44
n  n 
f=20 fx= fx2=  d2 = 16 + 4 +1 + 0 +1 + 9 +16 + 25 i.e. 72
53.3 148.55
72 6
53 .3 = 
Thus mean = = 2.665 = 2.7 to 1dp 8 8
20
2 = 9 0.75
148.55  53.3 
S.D =   = 8.25
20  20 
= 2.8723  2.87 to 2dp.
= 7.4275  7.1022
= 0.3253 2003/6
= 0.5704  0.6 to1dp The mean of the numbers 1, 4, 11, x, (x + 4) is x + 1.
2002/14 b Calculate:
Given are the Mathematics scores of 20 students. (a) the value of x,
42 64 76 30 58 46 57 67 48 66 (b) the standard deviation of the distribution.
62 45 50 50 48 52 60 53 70 58 Solution
Find the median and inter quartile range of the scores. x
(a) Mean =
Solution n
With a few data like this one we work out our median 1  4  11  x   x  4 
by arranging in descending order. x+1=
5
76, 70, 67, 66, 64, 62, 60, 58, 58, 57, 53, 52,
50, 50, 48, 48, 46, 45, 42, 30 Cross multiplying, we have
Since 20 is even, 5 ( x + 1 ) = 20 + 2 x
5 x + 5 = 20 + 2 x
57  53
The median = = 55 5 x – 2 x = 20 - 5
2 3 x = 15
Inter quartile range = Q 3 – Q 1 and 15
x= i.e. 5
3 3
Q3 = of cum frequency (Total frequency)
4 Thus the numbers are 1, 4, 11, 5, 9, with mean ; x + 1= 6

159
 xx Solution
(b) (i) mean deviation = The given data are raw thus; we must arrange them to enable
n us work effectively.
1 6  4  6  11 6  5  6  9  6 Scores: 2 3 4 5 6 7 8 9
= Frequency: 4 4 3 6 2 7 2 2
5
Thus mode = score 7(E) with highest freq. of 7
 5   2  5  1  3
= 29 . What is the median score?
5 A. 3 B. C. 5 D. 6 E. 7
5  2  5 1 3 16 Solution
= = i.e. 3.2
5 5 The sum of the frequency is 30 and an even number,
Median class = f

(b) (ii) S.D =



 xx 
2
2
= 30 i.e 15th class
n 2
52  2 2  52 12  32 Adding the frequency from the upper side till we get
= to 15th class 4 + 4 + 3 + 6 i.e 4th column, where
5 we have 15th class.
64 Thus, median = 5 (C)
= = 12.8 i.e. 3.58
5 30. What is the range of the distribution?
2000/12a A. 2 B. 7 C. 8 D. 9 E. 10
If the mean and the variance of the numbers 1, 4, x, y, Range = biggest number – smallest number
10 is 6 and 10 respectively, find the values of x and y =9–2 =7 (B)
Solution 2003/9 Neco Exercise 13.37
x Which type of distribution are the mean and median always equal
Mean = A Symmetrical B Bi – modal C y – shaped
n
1 4  x  y 10 D Um – modal E Skew
6 =
5 1999/11 (Nov) Exercise 13.38
6 × 5 = 15 + x + y Which of the following is a measure of dispersion?
30 – 15 = x + y A mean B median C mode D variance
Thus x + y = 15 _____(1) 2004/38-39 Neco Exercise 13.39

Secondly variance =
 xx   2
The heights of 30 students is shown in the following table
n Height 160 161 162 163 164 165
No. of 4 6 3 7 8 2
1 6  4  6  x  62   y  62  10  62
2 2
Students
10=
5 38 The mean of the distribution is
10  5 = (-5)2 + (-2)2 + (x2 –12x + 36) + (y2 –12y + 36) + (4)2
A 4875.1 cm B 4001.2 cm C 3571.0 cm
50 = 25 + 4 + x – 12x + 36 + y – 12y + 36 +16
2 2
D 162.5 cm E 129.2 cm
50 = 117 + x2 – 12x + y2 – 12y_____(2)
From (1) x = 15 – y Substituting into (2) 39 The median of the distribution is
A 160 B 162 C 163 D 164 E 165
50 = 117 + (15–y)2 – 12(15–y) + y2 – 12y
50 = 117 + 225 – 30y + y2 – 180 + 12y + y2 – 12y 2006/34 Exercise 13.40
50 = 162 – 30y + 2y2 The mean and median of integers x, y, z and t are 5
y2 – 15y + 56 = 0 Factorizing and z respectively. If x < y < z < t and y = 4, find (x + t).
(y– 7)(y– 8) = 0 A 12 B 11 C 10 D8
y = 7 or 8 2009/33 Exercise 13.41
If y = 7 Then x = 15– 7 i.e 8 If the mean of –1, 0, 9, 3, k, 5 is 2, where k is a constant, find
If y = 8 Then x = 15– 8 i.e 7 the median of the set of numbers.
1989/28 – 30 ordinary maths A 10 B 3/ 2 C 7/ 2 D6
Use the following for questions 28 – 30 1994/5 Exercise 13.42 ordinary maths
For a class of 30 students, the scores in a mathematics The table below gives the frequency distributions of marks
test out of 10 marks were as follows: obtained by some students in a scholarship examination.
4 5 7 2 3 6 5 5 8 9 Scores (x) 15 25 35 45 55 65 75
5 4 2 3 7 9 8 7 7 7 Frequency (x) 1 4 12 24 18 8 3
3 4 5 5 2 3 6 7 7 2 (a) Calculate, correct to 3 sig. figures, the mean mark
28 .What is the mode of the scores? (b) Find the:
A. 3 B. C. 5 D. 6 E. 7 (i) mode (ii) range of the distribution
160
1/29 CHAPTER FOURTEEN.
Express 250 451 in decimal (Hint: 10 = 601)
A. 25.750 B. 25. 550 C. 25. 450 D. 25. 150 Angles Of Elevation & Depression
Solution
Using the hint provided we convert the “second” part of
the question i.e. 451 to degree
451 = 45/60 = 0. 750 φ
Hence, 250 451 = 25. 750 (A) φ
2002 / 41 Neco
If cos x = -5/13 where 1800  x  270. What is the value
of tan x – sin x
 
A. 11 1/13 B. 3 21/65 C. 2 14/65
D. 1 11/13 E. 1 13/65 Fig.I Fig.II
Analysis and Solution
The range specification is strictly the 3rd quadrant.
Hence the minus sign goes along with the cosine value.
Thus all our working must be restricted to the 3rd Fig.I shows a boy standing on the ground viewing a plane with
quadrant. The resulting trig ratio an angle of elevation  while the pilot views the boy at an
angle of depression .
K =12 by
S A Fig II shows a man viewing his daughter in her office at an
13 Pythagoras rule
K angle of elevation  and the daughter views her father at an
T C angle of depression .
X Deductively in both fig I and fig II
 =  (Alternate angles).
5
The angles of elevation and depression can be used in
calculating among others
Thus tan x – sin x = 12/5 - ( - 12/13)
(a) the height of an object above the ground
Recall sine is negative in 3rd quad.
(b) the distance of an object from a given point of
= 12/5 + 12/13 observation.
= 156 + 60 = 216 = 3 21/65 (B)
65 65 Single-triangle cases involving angle of elevation
E.g. 1 A mountain climber walks 500m up a hill, which
2004/5b Neco (Nov) slopes at an angle of 300. Calculate the vertical height
Evaluate cos2450 +tan2 600 through which he rises.
Analysis and Solution Analysis and Solution
cos2450 + tan2 600 = (cos450 ) 2 + (tan 600) 2 Common sense in interpreting data is important.
Substituting for the special trig ratios Here ; Opp , Hypo are relevant i.e SOH.
= 1 2
+ ( √3 ) 2
√2 Sin 300 = x
500m 500
X
= 1 + 3 1 = x
2 300 2 500
= 7
/2 = 31/2 x = 500
2
2002/10 Exercise 13. 17 x = 250m
Find in surd form, the value of
sin 450 cos 300 + cos 450 sin 300 E.g. 5 A ladder 5m long rest against a vertical wall so that
the distance between the foot of the ladder and the wall
A. √2 B. √3 C. √2 D. √ 6 + √2 is 2. 5m. Find the angle the ladder makes with the wall.
2 2 4 Analysis and Solution

2002 /23 Neco Exercise 13. 18 5m


If sin  = √3 and 00  1800, What is cos  ?
2
A. 1/√3 B. 2/√3 C. 1/2 D. √ 3/2 E. √3
2.5m

161
 is the angle the ladder makes with the wall. Here opp X
and hypo are relevant.
Hence sin  = 2.5
5 40m K
1
sin  = /2 i.e 0.500
 = sin - 1 0. 500
0
30
= 300 y

1988/10 OM Here, opp and hypo are relevant, hence


A ladder 9m long lean against a vertical wall, making sin 300. = k
an angle of 640 with the horizontal ground. Calculate, to 40
one decimal place, how far the foot of the ladder is from 1 = k
the wall. 2 40
A. 4. 0m B. 5. 8m C. 7. 1m D. 8. 1m E. 18. 5m k = 40
Analysis and Solution 2
k = 20m (B)

9m 1990/41 OM
If the shadow of a pole 7m high is = 1/2 its length, what is the
angle of elevation of the sun; correct to the nearest degree.
640 A. 900 B. 630 C. 600 D. 260 E. 00
y Analysis and Solution
In sketching the required diagram common sense must come
In the resulting triangle Adj and Hypo are relevant.Thus into play. The shadow will be the horizontal distance on the
Cos 640 = y ground.
9
y = 9 cos 640
= 9 x 0. 4384
= 3. 9453 7m
 4. 0 m to nearest metre
1988/35 OM
When an aeroplane is 800m above the ground, it’s angle
of elevation from a point P on the ground is 300. How 3.5m
far is the plane from P by the line of sight ? Here opp and adj are relevant. Hence ,
A. 400m B. 800m C. 150m
D. 1600m E. 1700m tan  = 7
3. 5
Analysis and Solution tan  = 2.000
 = tan – 1 2 . 000
= 63.430
K
 630 to the nearest degree B
800m
0 1991 / 28 Exercise 14 . 1 OM
30 The angle of elevation of the top of a tree 39m away from a
point on the ground is 300. Find the height of the tree
Here Opp, Hypo are relevant. Hence,
A. 39√3m B. 13√3m C. 13/√3 m
Sin 300 = 800
K D. 13/3√3 m E. √ 3/13 m
1 = 800
1991 /28 Exercise 14.2
2 K
The angle of elevation of the top of a tree 39m away from a
K = 1600m (D)
point on the ground is 300. Find the height of the tree
1989 /12 OM A. 39 √m B. 13√3m C. 13/√3m
The angle of elevation of x from y is 300. If xy = 40m, D. 13/3√3m E. √3/13m
how high is x above the level of y ?
A 10m B. 20m C. 20 √3m D. 40m E. 50m

162
1992/35 Exercise 14.2 OM 1999/31 Exercise 14.10 OM
A ladder leans against the wall at an angle 600 to the A pole of length L leans against a vertical wall so that it
wall. If the foot of the ladder is 5 metres away from the makes an angle of 600 with the horizontal ground. If the top
wall, calculate the length of the ladder. of the pole is 8m above the ground, calculate L.
A. 5√3m B. 5m C. 10√3m D. 5√3m E. 10m A. 16√3 m B. 4√3 m C.√3 m D. 16√3 m
3 3 16 3

1993/34 Exercise 14. 3 OM 2001 / 40 Neco Exercise 14 . 11 OM


The angle of elevation of the top of a tower from a point The angle of elevation of the top T of a pole from a point X
on the horizontal ground, 40m away from the foot of the on the horizontal ground is 320. If X is 68m away from the
tower is 300. Find the height of the tower. foot F of the pole, calculate the height FT of the pole, correct
A. 20m B. 40 √3m C. 20√3m D. 40√3m E.60m to 1 d. p
3 A.128.3m B.108.8m C. 80.2m D. 57.7m E.42.5m

1993/35 Exercise 14. 4 OM 2002 / 28 Neco Exercise 14. 12 OM


At a point 500m from the base of a water tank the angle The angle of elevation of the top (T) of a palm tree from a
of elevation of the top of the tank is 450. Find the height point R on the ground is 300. The distance from R to the
of the tank. base B of the palm tree is 120m. What is the distance
A. 250m B. 353m C. 354m D. 433m E. 500m between T and R ?
A. 60m B. 60√3 m C. 80√3 m D. 120√3 m E. 240 m
1994/ 42 Exercise 14.5 OM
A ladder 6m long leans against a vertical wall so that it 2000/10 (a) Exercise 14. 13 OM
makes an angle of 600 with the wall. Calculate the A surveyor walks 100m up a hill, which slopes at an angle of
distance of the foot of the ladder from the wall. 240 to the horizontal. Calculate, correct to the nearest metre,
A. 3m B. 6m C. 2 √3m D. 3√3m E. 6√3m the height through which he rises.

1996/43 Exercise 14. 6 OM


The angle of elevation of the top X of a vertical pole Single-triangle cases involving angle of depression
from a point P on a level ground is 600. The distance E.g. 2. The angle of depression of a boat from the top of a
from P to the foot of the pole is 55m. Without using mountain 10m high is 300. How far is the boat from the foot
tables, find the height of the pole. of the mountain?
Analysis and Solution
A. 50/3m B. 50m C. 55√3m D. 60m E. 65m Let y be the distance between the boat and the foot of the
mountain. In the resulting ∆ only opp and Adj are relevant.
1997/39 Exercise 14. 7 OM
The angle of elevation of a point T on a tower from a 300 300
point U on the horizontal ground is 300.
If TU = 54m, how high is T above the horizontal 10m 10m
ground ?
A. 108m B. 72m C. 46. 3m D. 31. 2m E. 27m 300
1998/26 Exercise 14. 8 OM y
In ΔPQR,  PQR is a right angle, QR = 2cm and Hence, tan 300 = 10
 PRQ = 600. Find PR y
y tan 300 = 10
A. 4√3 cm B. 4 cm C. 2√3 cm D. 4/3 cm E. 1 cm y = 10
P
tan 300
but tan 300 = 1/√3
=  cot 300 = √3/1
y = 10 cot 300
0
60
Q R = 10 √3m
2cm
1989/ 44 OM
1998/30 Exercise 14. 9 OM The angle of depression of a point on the ground from the top
A ladder 5m long rest against a wall such that its foot of a building is 20.30 . If the distance from the point to the
makes an angle of 300 with the horizontal. How far is foot of the building is 40m. Calculate the height of the
the foot of the ladder from the wall ? building, correct to one decimal place
A5√ 3 m B 2 1/2 m C 5√ 3 m D 10√3m E10√3m A. 37.5m B. 28. 1m C. 27. 8m
3 2 2 D. 14. 8m E. 13. 9m
163
1992/3b Exercise 14. 17 OM
20.30 A boat is on the same horizontal level as the foot of a cliff,
and the angle of depression of the boat from the top of the
Building
height (h) cliff is 300. If the boat is 120m away from the foot of the
cliff, find the height of the cliff correct to three significant
0
20.3 figures.
Point Building
40 foot 1996/4b Exercise 14. 18 OM
Here opp and adj are relevant . Hence., A hawk on top of a tree, 20 metres high, views a chick on
Tan 20. 30 = H the ground at an angle of depression of 390. Find correct to
40 2s.f the distance of the chick from the bottom of the tree.
=  H = 40 tan 20. 30
= 40 x 0. 3699 2002 / 33 Exercise 14. 19
= 14.7964 The angle of depression of a point Q from a vertical tower
 14.8 to 1 d. p (D) PR, 30m high, is 400. If the foot P of the tower is on the same
horizontal level as Q, find, correct to 2 decimal places, PQ
1999/28 OM A. 35.75m B. 25.00m C. 22.98m D. 19.28m
From the top of a cliff 20m high, a boat can be sighted
at sea 75m from the foot of the cliff. Calculate the angle 2002/ 24 Neco Exercise 14. 20 OM
of depression of the boat from the top of the cliff. A girl on top of a storey building 36m high dropped a N10.
A. 14. 90 B. 15 . 50 C. 74. 50 D. 75. 10 00 note on the ground below. If the angle of depression of the
Analysis and Solution N10. 00 note is 350, how far is the money from the girl ?
T(top) A 20. 64m B 29. 48m C 44. 49m D 49. 55m E 62. 76m

2000/17 Exercise 14 . 21 OM
A
20m
10m

75m
C
Tan  = 20 = 0. 2667 B 5m
75 In the diagram, AB is as vertical pole and BC is horizontal. If
 = tan – 1 0. 2667 AC = 10m and BC =5m. Calculate the angle of
 = 14 . 90 (A) depression of C from A.
A. 630 B. 600 C. 450 D. 270
1990/42 Exercise 14. 14 OM
From the top of a building 10m high ; the angle of
depression of a stone lying on the ground is 690.
Calculate, correct to 1 decimal place, the distance of the Further cases on single triangle
stone from the foot of the building. 1990/5 OM
A. 3. 6m B. 3. 8m C .6. 0m D. 9. 3m E. 26. 1m The feet of two vertical poles of height 3m and 7m are in line
with a point P on the ground, the smaller pole being between
1991/27 Exercise 14. 15 OM the taller pole and P and at a distance of 20m from P. The
From the top of a cliff, the angle of depression of a boat angle of elevation of top (T) of the taller pole from the top
on the sea is 600; if the top of the cliff is 25m above the (R) of the smaller pole is 300.
sea level, calculate the horizontal distance from the Calculate the :
bottom of the cliff to the boat. (i) distance RT (ii) distance of the foot of the taller pole
A 50√3m B 25√3m C 25√3 m D 25 m E√3 m from P. correct to 3. s. f
3 3 25 (iii) angle of elevation of T from P, correct to 1 d. p
Analysis and Solution
1992/36 Exercise 14.16 OM A second look at the question will provide us with the data
A cliff on the bank of a river is 300 metres high. If the necessary for sketching the required diagram.
angle of depression of a point of the opposite side of the
river is 600. Find the width of the river.
A. 100m B. 75 √3m C. 100 √3m
D. 200√3m E. 300m
164
T 1 = y
1 10. 4
y = 10. 4m
Tree height = 10 . 4 + 1. 6 = 12 . 0 m
30 0
R S
7m 2004/9b NABTEB OM
3m A student 1.5m tall, when standing up, observe that the
angle of elevation of the top of a tree 20m away is 250. Find
O the height of the tree.
P Q K Solution
I 20m I
(i) Distance RT
RQ = SK = 3m y
TS = TK – SK
= 7m – 3m
0
25
= 4m 20m
1.5m 1.5m
Hence in triangle RST
opp and hypo are relevant 20m
sin 300 = TS
RT Tree height = y + 1. 5
1 = 4 In the top ∆
2 RT Tan 250 = y
RT = 8m 20
20 tan 25 = y
(ii) Required distance is PK y = 20 x 0.4663
PK = PQ + QK ; PQ = 20m = 9.3m
But RS = QK and RS = 82 – 42 Tree height = 9.3 + 1.5 = 10.8m
RS = 64 – 16
2001 / 42 Neco Exercise 14. 22 OM
RS = √48 From the top of a storey–building, the angle of depression of
RS = 6.928 = QK a mango on a tree 20m away from the base of the
 PK = 20 + 6.928 storey – building is 680, If the mango is 4 .5m above the
= 26.928 ground, what is the height of the storey – building to the
 26.9 to 3 s. f nearest metre ?
A. 55m B. 54m C. 50m D. 49m E. 45m
(iii) To find  in Δ PKT,
Opposite and adjacent are relevant. Hence 1987/10b GCE Exercise 14.23 OM
Tan  = TK = 7 The angle of elevation of the top of a vertical pole from a
PK 26.9 height 1.54m above a horizontal ground is 400. The foot of
Tan  = 0. 2602 the pole is on the same horizontal ground and the point of
 = tan – 1 0. 2602 observation is 20m from the pole.
= 14. 580  14. 60 to 1 d. p Calculate correct to three significant figures:
(i) The height of the pole,
E.g. 4 A boy 1. 6m tall, views a bird on top of a tree
at an angle of 450. If the distance between the boy and
the tree is 10. 4m, find the height of the tree; Double triangle cases
Analysis and Solution Example
Take into consideration the boy’s height in sketching From the top of a vertical mast 150 m high, two huts on the
the diagram. same ground level are observed. One due west and the other
due east of the mast. Their angles of depression are 600 and
450 respectively. Find the distance between the huts.
y
Solution
600 450
450
1.6m 10.4m 1.6m 150m
0
Considering the 45 Δ only Opp and Adj are relevant
Tan 450 = y 600 450
10. 4 Hut II Hut I
x I y
165
In the sub – triangle of 600; opp and adj are relevant.
Hence tan 600 = 150 QT = 35
x Sin 250
x tan 600 = 150 = 82. 82m
x = 150  83m to whole metres.
tan 600
x = 150 cot 600 1994/12b OM
Similarly Two observers P and Q , 15 metres apart observes a kite (k)
y = 150 cot 450 in the same vertical plane and from the same side of the
Required distance = x + y kite.The angles of elevation of the kite from P and Q are 350
= 150 cot 600 +150 cot 450 and 450 respectively. Find the height of the kite to the nearest
= 150 (cot 600 + cot 450) metre.
Analysis and Solution
= 150 ( 1/ √ 3 + K
 √3/3 +1)
= 150 (√3 + 3) = 50 (3 + √3 ) m
3 H

2000 / 9 Neco OM
Two ships on the sea are in line with an observer 35
0
450
standing on top of a cliff. Their angles of depression are P R
15 Q x - 15
250 and 650. If height of the observer is 35m above sea X
level; In ΔRPK
(i) Find the distance between the two ships; correct to tan 350 = H
1 d. p X
(ii) Calculate the shortest distance between the observer H = x tan 350 --------- (1)
and the ship that makes an angle 250 with the horizontal Also in ΔRQK
ground, correct to the nearest whole metres. tan 450 = H
Analysis and Solution x – 15
The phrase “in line with the observer” shows that both H = tan 450 (x - 15) -------- (2)
ships and the observer are in the same side and not Equating (1) to (2)
opposite sides H implies x tan 350 = tan 450 (x – 15)
Q
0 250 0. 7002 x = x – 15 since tan 450 = 1
65 15 = 1x – 0. 7002x
15 = 0. 3x
35 x = 50m
Substituting x value into (i)
H = 50 tan 350
250 650 = 35m
T P
S 1999/3 OM
(i) The distance between the two ships is TS ABC is a triangle, right angle at C. P is the mid-point of
But TS = TP – SP, we now find TP and SP AC,  PBC = 370 and BC = 5 cm
Thus in ΔQTP, Calculate : (a) AC correct to 3 sf (b)  PBA
tan 250 = 35 B
TP
TP = 35 = 75. 06m
tan 25 370
Also in ΔQSP 5cm
tan 650 = 35
SP
SP = 35 = 16. 32m
A P C
tan 650
TS = 75. 06 – 16. 32 a) In ΔPBC
= 58. 74m tan 370 = PC
5
 58. 7m to 1d. p
(ii) Shortest distance here = QT  PC = 5 tan 370
Thus in ΔQTP = 3. 768
Sin 250 = 35  3. 77 cm to 3 sf
QT Thus AC = 2 x 3. 77 (P is mid- point of AC)
= 7. 54 cm
166
b) In ΔABC From Fig. 2
tan B = AC = 7. 54 = 1.508 T1F12 = QF12 + QT12 by Pythagoras rule
BC 5 QT12 = 102 – 4.42
B = tan – 1 1. 508 = 100 – 19. 36
= 56. 50 = 80.64
Thus  PBA = 56. 50 – 370 = 19.50 Thus QT1 = 80. 64
= 8. 98
1999/9 OM But TT1 = TQ – QT1
T
= 9. 397 – 8. 98
= 0. 417
= 0. 42m to 2 sf
(We used non – approximate value to allow for non – trivial
T
1 10m answer)

(ii) From Fig.2, find  = QF1T1


Cos  = 4. 4
0
70
Q F F1 10
 = Cos – 1 0. 44
In the diagram, a ladder TF, 10m long is placed against
= 63. 90
a wall at an angle of 700 to the horizontal.
 640 to the nearest degree
(a) How high up the wall, correct to the nearest metre,
does the ladder reach ?
2000/10 OM
(b) If the foot ( F ) of the ladder is pulled from the wall
Two boats, 70 metre apart and on opposite sides of a
to F1 by 1m
lighthouse are in a straight line with the lighthouse. The
(i) how far, correct to 2sf, does the top T slide down the
angles of elevation of the top of the lighthouse from the two
wall to T1
boats are 71.60 and 450. Find the height of the lighthouse.
(ii)calculate, correct to the nearest degree,  [ Take tan 71.60 = 3]
QF1T1. A
Analysis and Solution
The (a) part of the question is in the usual pattern but to
solve (b) part correctly candidates need to visualize the
practical aspect of the question – this will help to avoid
unnecessary mistakes. You know that the length of the H
ladder remains unchanged and that the new length on 0
71.6 45
0
the ground will be the former plus 1 metre.
In ΔQFT: B Y D 70 - y C
Sin 700 = TQ = TQ 70m
TF 10
TQ = 10 sin 700 To find H, we should know either BD or CD
= 9 . 397 In ΔABD tan 71. 6 = H
= 9m correct to the nearest metre y
H = y tan 71.6 ---------- (1)
(b) (i) T T1 is the required distance Also in ΔACD, tan 45 = h
TF1 = TF = 10m 70 – y
QF1 can be known if we know QF H = tan 45 (70 – y) ------ (2)
In ΔQFT, Equating (1) to (2)
Cos 70 = QF = QF H  y tan 71. 6 = tan 45 (70 – y)
TF 10 3y = 70 – y
Thus QF = 10 cos 70 4y = 70
= 3. 42m y = 70 / 4 = 17. 5m
 QF1 = 3. 42 + 1 = 4 . 42 Substituting y value into (1)
= 4. 4 to 2 s f H = 17.5 tan 71. 6
Resulting diagram (Fig. 2) = 17. 5 x 3
T = 52. 50m

T
1 2002/38 OM
10m In the diagram, QRT is a straight line. If angle PTR = 900,
angle PRT = 600, angle PQR = 300 and PQ = 6√3 cm,
calculate RT
Q 4.4m F1
167
√3 Therefore in ΔPQR
A. 0.3 cm B. /2 C. 3 cm D. 3√3 cm
tan  = 8 = 4.950
P 1. 616
 = tan – 1 4. 950
= 78. 60
6 3cm  790 to the nearest degree
(ii) To find RPS which is 
but  =  - 
300 60
0

In ΔQPS
Q R T  = 180 – 90 – 68 (sum of angles in a triangle ).
= 220
Analysis and Solution Also in ΔPQR,
To solve for RT, we should know PT  = 180 – 90 – 79 (sum of angles in a triangle)
Hence in ΔPQT = 110
Sin 30 = PT
Hence  =  - 
6√3 = 22 – 11
PT = 6√3 Sin 30 = 110
= 3 √3 cm 1996/2b OM
Thus in ΔPRT In the diagram below ADC is a straight line.
Tan 600 = PT
CD = 48cm BD = 36cm and  AD = y cm.
RT
Find the value of y.
RT = 3√3 = 3√3 = 3cm B
tan 600 √3

2002 / 3 ( Nov ) OM 36cm


P

8m
A Ycm D 48cm C

Analysis and Solution


68 0 To evaluate y, we need to know one of the angles in
Q
R S ΔABD. This can be done by evaluating  CBD in
ΔCBD and subtracting it from ABC = 900
In the diagram, PQR = 900, PQ = 8m, PSQ = 680 B
and QR = RS. Calculate, correct to the nearest
degree;
(i) PRQ ; (ii) RPS
Analysis and Solution 36cm
P

o w
A Ycm D 48cm C
8m

In ΔBCD
tan  = 48 = 1. 333
36
680
Q S  = tan – 1 (1. 333)
R
= 53. 130
(i) To find , QR must be known in ΔPRQ but QR can
be found from ΔQPS Thus  = 900 -  ( +  = 900 shown)
Thus, in ΔQPS  = 900 – 53 . 130
Tan 680 = PQ = 8 = 36. 87
QS QS In ΔABD
QS = 8 = 3.232 m tan  = y
Tan 680 36
But QS = QR + RS and QR = RS tan 36. 87 = y
Thus, QS = 2QR 36
Hence QR = 3.232 = 1. 616  y = 36 tan 36. 870
2 = 36 x 0. 7500
= 27. 00cm
168
1991 / 12 OM Analysis and Solution
From a horizontal distance of 8.5 km, a pilot observes Critical analysis of the above question shows elements of
that the angle of depression of the top and base of vagueness or lack of precision on the part of the examiner.
control towers are 300 and 330 respectively. Calculate, Hence the resulting diagram can be expressed in two ways ;
correct to 3 s . f : giving different answers. The author does not advise students
(i) The shortest distance between the pilot and the base to challenge examiners but for safety they should avoid any
of the control tower; question that is not clear to them during exams.
(ii) the height of control tower.
Analysis and Solution P
Pilot Fig. I
P 0
30 0
33
H
0
15 0
35
30
0 X Z
4000 Y b

5,200m
Q S
Tower 4000 + b
0
33
R T In Δ PXZ, tan 150 = H
4000 + b
(i) In Δ PTR, the shortest distance is / PT/ H = tan 15 (4000 + b) ----- (1)
Here Adj and Hypo are relevant Also in Δ PYZ tan 350 = H
Hence, cos 330 = RT b
PT =  H = b tan 350 ------ (2)
0
Cos 33 = 8. 5 Equating (i) to (ii)
PT H =  tan 150 (4000 + b) = b tan 350
PT cos 330 = 8.5 0.3 (4000 + b) = 0. 7b
PT = 8. 5 1200 + 0. 3b = 0. 7b
Cos 330 1200 = 0. 7 b – 0. 3b
= 10.14 km  10. 1 km to 3 s.f 1200 = 0. 4b
(ii) Control tower height / ST / = / QR / b = 3000m
/ QR/ = / PR / - / PQ / Substituting b value into (2)
/PR/ is in ΔPRT while /PQ/ is in ΔPQS. H = 3000 tan 35
In ΔPRT, opposite and adjacent are relevant = 2100m
Hence tan 330 = PR  Actual height = 2100 + 5200
RT = 7300m
PR = RT tan 330 i.e 8. 5 tan 330 Second diagram
= 5. 520km P
In Δ PQS, opposite and adjacent are relevant
Hence tan 300 = PQ But /QS/ = /RT/
QS
PQ = 8.5 tan 300 H
= 4. 908 KM
Substituting 150 350
/ST/ = /QR/ = 5. 520 – 4 . 908 X Y
4000 - b Z b
= 0. 612 KM
= 612m
Since height is measured in metres; not km 4000m Fig II

1993/3 OM In Δ PXZ, tan 150 H=


A simple measuring device is used at point x and y on 4000 – b
the same horizontal level to measure the angle of H = tan 15 (4000 – b) ----- (1)
elevation of the peak P of a certain mountain. If X is Also in Δ PYZ, tan 35 = H
known to be 5,200m above sea level /XY / = 4,000m b
and the measurements of the angle of elevation of P at H = b tan 350 ------ (2)
X and Y are 150 and 350 respectively, find the height of Equating (1) to (2)
the mountain H =  tan 15 (4000 – b) = b tan 35
(Take tan 150 = 0. 3 and tan 350 = 0. 7) 0. 3 (4000 – b ) = 0. 7b
1200 – 0.3b = 0. 7b
169
1200 = 0. 7b + 0. 3b Sin 650 = TS
b = 1200m PT
Substituting b value into (2) TS = PT Sin 650 = 28.2 sin 650
H = 1200 tan 35 = 25. 558m  25. 6m to 3 s.f
H = 840m
Actual height = 5200 + 840 Authors suggestion (practice only)
= 6040 m Assuming question (b) was asked before question (a) i.e.
discussing strictly right – angles triangle trigonometry, the
1994/3a OM solution, applying the same diagram is thus:
A tower and a building stand on the same horizontal T
level. From the points at the bottom of the building, the
angle of elevation of the top T, of the tower is 650.
From the top Q of the building, the angle of elevation of H - 20m
the point T is 250. If the building is 20m high, calculate
the distance PT 250
(b) Hence or otherwise, calculate the height of the Q R H
tower.
(Give your answers correct to 3 S.fs) 20m 20m
Analysis and Solution 65
0

T S
P

In ΔTQR
tan 250 = TR = H - 20
250 QR QR
Q R =  QR = cot 250 (H – 20) ---------- (1)
In ΔTPS
20m tan 650 = TS but PS = QR
650 PS
P S Hence tan 650 H
QR
By the examiner’s requirement, sine rule will be applied QR = H cot 650 ------- (2)
to solve for PT in Δ PTQ.
To apply sine rule to the diagram below, we need to Equating (1) to (2)
know angles  PQT,  PTQ QR =  cot 250 (H – 20) = H cot 650
 PQT =  RQT +  RQP 2. 145 (H – 20 ) = 0. 4663 H
= 250 + 900 2. 145 H – 42.90 = 0.4663 H
= 1150 2. 145 H – 0. 4663 H = 42. 90
 PTQ = 180 -  PQT - QPT 1. 6787 H = 42. 90
H = 25. 56
But QPT = 90 – 65 = 250
 25. 6m to 3 sf.
  PTQ = 180 – 115 – 25 = 400
To find PT
T Sin 650 = TS
PT
400 PT = TS cosec 650 but TS = H
Q = 25. 6 cosec 650
1150 = 28. 246m
?  28. 2m to 3 sf
20m

Warning
Please note that the author do not advise candidate to
P change question format before answering. The above was
done for the purpose of practice.
20 = PT
sin 400 sin 1150

PT = 20 sin 1150 = 28. 1993


Sin 400
 28 . 2m to 3 s.f

(b) Height TS in Δ PTS


170
VTR – 12/2a NTI TCII OM In ∆ ACD
The angle of elevation of the top of a flag pole from a Tan 400 = H
point on the level ground is 300. From another point on y
the ground, 20m nearer the flag pole, the angle of y tan 400 = H ----- (1)
elevation is 600.. Calculate the height of the flag pole
Also in ∆ BCD
Solution
A Tan 700 = H
2200 – y
(2200 – y) tan 700 = H ---- (2)
Equating (1) to (2) since they are all equals to H
H
(2200 – y) tan 700 = y tan 400
(2200 – y) 2.7475 = y x 0.8391
6044.5 – 2.7475y = 0.8391 y
300 600 6044.5 = 0.8391y + 2.7475y
B C 6044.5 = 3.5866y
x D 20m
x + 20 6044.5 = y = 1685.3
3.5866
Since we are looking for H, it can be gotten from Substitute y value into (1)
∆ACD H = y tan 40 will become
tan 600 = H H = 1685 . 3 x 0.8391
20 = 1414.1m
H = 20 tan 600
VTR – 12/24 NTI TCII OM
= 20 3 m Calculate /BC/ in the figure below
A

10cm

300 450
B C D

A.10 ( 3 – 1)cm B.10 (2 3 - 1) cm C.20cm


D.10cm E10( 3 - 2) cm
Solution
To find /BC/ ; we first find BD and CD
Then BC = BD - CD
In ∆ABD
tan 300 = 10
BD
VTR – 14/2B NTI TCII OM
BD tan 300 = 10
By drawing (using a suitable scale) or other wise, find
BD = 10 i.e. BD = 10 cot 300 -----(1)
the height at which an aircraft is flying if the angle of 0
tan 30
elevation when observed from points A and B are 400
and 700 respectively. A and B are 2200 metres apart and Also , in ∆ACD
the aircraft is vertically above a point on the line AB tan 450 = 10
Solution CD
C Similarly, CD = 10 cot 450 ----- (2)
Thus BC = BD – CD
= 10 cot 300 – 10 cot 450
= 10 (cot 300 – cot 450)
= 10( 3 - 1)cm (A)
H

0
1986/19 GCE OM
40 700 In the diagram below, P and Q are two places on the same
A y D 2200 - y B horizontal level. T is an aircraft which is due west of P and
2200m Q. The angle of elevation of the aircraft from P and Q are 400
and 600 respectively
171
T 2004/4 NECO
B

A
0
60 40 0

Q 13m
N 1200km P 5m
If /PQ/ = 1200km and PQN is a straight line, Calculate,
correct to the nearest 10m:
(a) The height TN of the aircraft above the horizontal, T 20m S
(b) The distance of the aircraft from P.
Solution In the above diagram, AT represents the height of a building
T -
5m high, which stands 20m away from the foot of a pole BS
13m high on the same horizontal ground. Calculate the :
(i) length AB (to 3 sig. figures):
(ii) angle of elevation of B from A
(iii) angle of depression of base T from B.

600 Solution
400
N x - 1200 Q 1200km P B
x
In ∆NPT
tan 400 = TN 13 - 5 (8m)
x
x tan 400 = TN -----(1)
A C 13m
Also, in ∆NQT
tan 600 = TN 5m
x – 1200 5m
(x – 1200) tan 60 = TN -----(2)
equating (1) to (2) since they are equals to TN T S
x tan 400 = (x – 1200) tan 600 20m
x x 0.8391 = (x – 1200) 3
0.8391x = 1.7321x – 2078. 5
In ∆ ABC
2078.5 = 1.7321x – 0.8391x
(i) length AB2 = AC2 + BC2
2078.5 = 0.893x
But by deduction TS is the same as AC
2075.5 = x i.e x = 2327.5km
= 202 + 82
0.893
= 400 + 64
Substituting x value into (1)
AB2 = 464
x tan 40 = TN will become
AB = 464
2327. 5 x 0.8391 = TN
= 21. 54  21.5 m to 3 sf
1953.00 km = TN
(ii) θ = ?.
(a) TN  2000 km
In ∆ABC tan θ = 8 i.e 0.4
( b ) TP ? 20
θ = tan – 1 0.4
In ∆NPT
= 21.800
Sin 400 = TN
TP
TP Sin 400 = TN ( iii ) Φat B = Φ at T (Alternate angles)
TP = TN Thus, in ∆ BST
Sin 400 Tan Φ = 13 i.e. 0. 65
TP = 1953. 00 km 20
0.6428 Φ = tan- 1 0. 65
= 3038.27 = 33. 020
 3000km

172
1997/3b Exercise 14 .24 OM Others
Q Right angle (i. e. 900)

300 700
P N Straight line angles is 1800
R
The diagram above represents the vertical cross- section
of a mountain with height NQ standing on a horizontal
ground PRN. If the angles of elevation of the top Q of
the mountain from P and R are 300 and 700 respectively
and PR = 500m.
Calculate correct to 3 significant figures: SOME BASIC FACTS ON THE FORMATION OF
(i) QP (ii) the height of the mountain ANGLES BETWEEN LINES

2004/7a Exercise 14.25 OM STRAIGHT LINES


The angle of depression of the top and bottom of a (i) Sum of angles on a straight line is 1800
building are 510 and 620 respectively from the top of a
tower 72cm high. The base of the building is on the Fig. I
same horizontal level as the foot of the tower. E C
Calculate the height of the building correct to two
significant figures.
b
a c
CHAPTER FIFTEEN A B
BEARING & Element Of Plane Geometry f d
e
Element of plane geometry
Angles - The units of measuring angles are degrees and F
minutes.
D
From Fig. I, the following holds :
1 degree = 60 minutes a + b + c = 1800 on line AB
i.e. 10 = 601 b + c + d = 1800 on line EF
Sometimes, smaller units like seconds are used; f + e + d = 1800 on line AB
1 minute = 60 seconds a + f + e = 1800 on line EF
i.e. 11 = 6011
Acute angle (i.e.  between 00 and 900) (ii) Sum of angles at a point is 3600
From Fig. I
a + b + c + d + e + f = 3600

(iii) Vertically opposite angles are equal

m
Obtuse angle (i.e.  between 900 and 1800)
o.
a
n
n
Reflex angle (i.e.  between 1800 and 3600)

m = n (vertically opposite angles)


o + n = a (vertically opposite angles)

173
PARALLEL LINES Examples
Two lines are parallel; if they do not and can never (i) Find the value of  in the diagram
meet (intersect) when their length is increased. Any line
cutting across a pair of parallel lines is called P C
0
transversal. 50

T
r
ans
ver
sal P
ar
al
lel 20
0

R D
Solution
(We give reasons for claims made)
Draw a line KT parallel to PC and RD
P C
0
50
a
K T
b
0
( i )Corresponding angles are equal 20
R D
x
a = 500 (Alternate angles)
a Also b = 200 (Alternate angles)
But  = a + b
y = 200 + 500 = 700
b (ii) Find the unknown angles below

x = y (corresponding angles) b
c
a = b (corresponding angles)
In corresponding angles, one angle is inside while the
other angle is outside and both angles are on the same
side. a

( ii ) Alternate angles are equal 3120


Solution
a = 360 – 312 (angles at a point)
k a = 480
m b = a i.e 480 (corresponding angles)
n b + c = 1800 (sum of angles on a straight line)
I 48 + c = 180
c = 180 – 48 = 1320
( iii ) Find the unknown angles below
k = I (alternate angles)
m = n (alternate angles) 0
70
0

30
Under alternate angles both angles are within the
parallel lines but on different sides. t

( iii ) Sum of interior opposite angles is 1800


u s

a w
c
b Solution
d t = 300 + 700 (vertically opposite angles)
t = 1000
t + s = 1800 (sum of interior opposite angles)
s = 180 – 100
a + b = 1800 (sum of interior opposite angles) = 800
c + d = 1800 (sum of interior opposite angles) w = s = 800 (vertically opposite angles)
Also, u = 30 + 70 (corresponding angles)
= 100

174
1988/ 36 OM Analysis and Solution
In the diagram below LK//PQ, reflex angle KLM = 2410 Produce RQ and PN as shown below
and QPM = 890. What is the value of LMP ? R
Q K
d Q
P
C
M
L N
0
241
550
a
890 400 b L
M
P
a + 55 = 1800 (sum of angles on a straight line)
0 0 0 0 0
A. 30 B. 61 C. 89 D. 119 E. 150 a = 180 – 55
Analysis and Solution = 1250
Produce line XY//KL and QP
Also a + b + 40 = 1800 (sum of angles in ∆)
Fig V
b = 180 – 40 – 125
X b = 150
K 0
c = b = 15 (alternate angles)
Q
d = c [ RQP] = 150 (alternate angles) A.

b a
d M
1993 / 7a OM
L 241 0
In the diagram below,BA is parallel to DE. Find the value of X
A E
0
89
Y
P
From Fig. V X0
d + 241 = 360 (sum of angles at a point) 520
d = 360 - 241 C
d = 1190 B
d + a = 1800 (sum of interior opposite angles)
a = 180 – 119 D
= 610
3120
Also 890 + b = 1800 ( sum of interior opposite angles)
b = 180 – 89 Solution
= 910 Produce another parallel line as shown below :
But  LMP = b – a A E
= 91 – 61= 300
1992 / 24 OM
In the diagram below ML // PQ and NP// QR.
If  LMN = 400 and  MNP = 550.
X
Find  RQP 0
A. 150 B. 250 C. 350 D. 400 E. 550 52
a
b
R
e

Q 3120
P
e = 360 – 312 (sum of angles at a point)
= 480
550 N b = e = 480 (alternate angles)
a = 520 (alternate angles)
400 x + (a + b) = 360 (sum of angles at a point)
M L X= 360 – (52 + 48)
= 360 – 100 = 2600

175
1994 / 34 OM Analysis and Solution
In the diagram, PQ // RS and the angles are shown. C
A
Find X 0
A. 1450 B. 1500 C. 1550 D. 1600 E. 1650 52 a
Q b
P 0 x 35
0

140

55
0
T
D
X B
R
S a = 520 (corresponding angles)
Analysis and Solution b = a = 520 (vertically opposite angles)
Joining RP to get a pentagon, we have b + x + 35 = 1800 (sum of angles in a triangle)
Q 52 + x + 35 = 180
P x = 180 – 87
1400
= 930 (B)
55
0
T 1996 / 29 OM
X In the diagram below, TRQ is a straight line. Find P, if
R P = 1/3 (a + b + c)
S A. 450 B. 60 C. 900 D. 1200 E. 1500
To get the sum of angles in a pentagon n = 5 Analysis and Solution
2 (n – 2) x 90
 2 (5 – 2) x 90 = 540
 90 + 90 + X + 55 + 140 = 540 (sum of pentagons )
X = 540 – 375 = 165 (E)

1995 / 34 OM b c
a P
In the diagram below AO is perpendicular to OB. T Q
Find x R
A. 7. 50 B. 150 C. 22. 50 D. 300 E. 38. 60
a + b + c + P = 1800(sum of s on a straight line)
i.e a + b + c + 1/3 (a + b + c) = 1800
To clear the fraction, multiply both sides by 3
3a + 3b +3c + a + b + c = 540 Alternatively
3x
2x 4a + 4b + 4c = 540 P = 1/3 (a + b + c)
4x B 4 (a + b + c) =540 Multiply through by 3
O
a + b + c = 540 = 135 3P = a + b + c
4 a + b + c + P = 180 0
If a + b + c = 135 (sum of s on a straight line)
Then, 1/3(a + b + c) = 135/3 3P + P = 1800
A 1
/3(a + b + c ) = 45 P = 180/4
4x + 3x + 2x + 90 = 360 But 1/3(a + b + c) = P = 450(A) = 45 0(A)
9x = 360 – 90
9x = 270 1996/39 Exercise 15. 1 OM
x = 300 (D) In the diagram, AB, CD and XY are straight lines
intersecting at W. Find  CWX.
1995 / 38 OM A. 200 B. 800 C. 1000 D. 1200 E.1400
C C
A X
0
52

0
x 35
A B
600 W 400
D
B

In the diagram above, AB // CD. What is the size of the Y D


angle marked x ?
A. 1030 B. 930 C. 770 D. 620 E.520
176
2001/5a OM 2002/29 (Nov) Exercise 15. 4 OM
P P
T
x
M
N 3x
x
560
Q S
y

S T
700
R
In the diagram above, PR and QS are straight lines.
Q Find the value of angle y
In the diagram, MN//ST, NP//QT and  STQ = 700 A. 300 B. 560 C. 870 D. 930
Find x.
Analysis and Solution 2002/33 (Nov) Exercise 15. 5 OM
Produce PN to cut ST at O P
P

x
M 3r m
b
N X Y
r O
640

a T
S T
O 700 Q
In the diagram, XY and PQ intersect at O.
Angle TOQ = 640, calculate m.
Q A. 930 B. 960 C. 1160 D. 1510
a = 700 (alternate angles)
b = a =700 (alternate angles) 2002/45 OM
b + x = 1800 (sum of angles on a straight line) P S
x = 180 – 70 = 1100
540
2002/1 Exercise 15. 2 OM
P 0
Q
36
110
0 x

U Q R T

In the diagram above, PQ//ST. Find the value of x.


T A. 820 B. 1080 C. 1240 D. 1640
0 Analysis and Solution
29
Produce QR to meet ST
R S P S
In the diagram  PQU = 360,  QRT = 290, PQ//RS
and UQ//RT. Find  PQR. 540
A. 940 B. 650 C. 610 D. 540

2002/3 Exercise 15. 3 OM


In the diagram, calculate the value of x 110
0 x
b a
350 Q R T
0
110 + a = 180 (interior opposite angles)
x a = 180 - 110 = 700
Also b + a + 54 = 1800 (sum of angles in a triangle)
b = 180 - 54 – 70
1150
b = 560
But b + x = 1800 (angles on straight line)
A. 350 B. 800 C. 1000 D. 1150 x = 180 – 56
= 1240 (C)
177
2001/34 Neco OM BEARING
This topic deals with measurement of points on the earth
surface based on some principles of which part has been
discussed. The resulting shapes are mostly triangles whose
lengths and angles are used to solve for the unknown ones.
This task (i.e. of solving for unknown angles and lengths)
need some prerequisite knowledge. Among these are;
- Element of plane geometry as treated in the preceding
unit.
- Types of triangles and ways of finding their unknown
angles and lengths.
Which of the statements is true of u, v, and w ?
A. u = w + v B. w – v = u C. u + w = v TYPES OF TRIANGLES (REVIEW)
D. u + v + w = 0 E. u + v + w = 3u
Analysis and Solution Equilateral triangle
u = v (alternate angles) A
v = w (alternate angles)
Now if u = v and v = w, then u = w i.e. all variables c b
are equal hence represented by one of them Thus
u + v + w = 3u (E)
or = 3w
or = 3v B a C
Properties
2002 /46 Neco OM All sides are equal a = b = c
If YZ = XZ, what is the value of the angle All angles are equal A = B = C
marked x0 ?
Y Isosceles Triangle
0 0 A
x 79
c
b

B C
a
Properties
X Two sides are equal b = c
Z Base angles are equal B = C
A. 390 301 B. 500 301 C. 790 D. 1010 E. 1100
Scalene Triangle
A
Analysis and Solution

Y c b
x0 790
c
C
a
B
Properties
b a
No sides are equal a  b  c
X
Z No angles are equal A  B  C

a = 790 (alternate angles) Right- angled Triangle


b = c (Base of isosceles triangle sinceYZ = XZ) P
But a + b + c = 1800 (sum of angles in a triangle)
a + b + b = 180 (b = c)
79 + 2b = 180 r q
2b = 180 – 79 = 101
b = 50. 5
But x0 = b (alternate angles) Q p R
x0 = b = 50 . 50
Properties
x0 = 500301 (10 = 601) (B)
One of the angle is 900 i.e. Q
178
Acute – angled Triangle Two angles and one side opposite one of the given angles
condition fulfilled hence sine rule will apply
A RS = 12
Sin 30 Sin 45

c b RS = 12 Sin 30
Sin 45
C
a = 12 x 1/2 =6÷ 1 =6x 2 =6 2
B
1/ 2 2 1
Properties
Three angles are less than 900 (b) Given that the two angles of a triangle are 300 each and
the longest side is 10 cm. Calculate the length of each of the
i. e. A < 900, B < 900, C < 900 other sides.
Analysis and Solution
Obtuse – angled Triangle The 3rd angle should be1200 facing the longest side. Since
A 30 + 30 +  = 180 (sum of  s in Δ)
 = 1200
A
c c
b b
0
3
0 0
3
0
B C
B C 1
0cm
a
All angles and one side hence sine rule applies
Properties
One of the angles ,  is greater than 900 10 = b
but less than 1800 sin 120 sin 30
The finding of the unknown angles or lengths of the last
b = 10 Sin 30 Since sine is positive in the 2nd quad.
two triangles (acute and obtuse-angled triangles) leads
Sin 60
to sine and cosine rule.
= 10 x 1/2 = 5 x 2 = 10 3 cm
The sine rule states that :
_a__ = b__ = __c__ 3 /2 3 3
Sin A Sin B Sin C
The notations a, b & c and A, B & C are as applied to
acute and obtuse angled triangles above. (c) In an acute-angled triangle PQR with PQ = 10m,
PR = 15m ,  PRQ = 400. Evaluate  PQR
Conditions for applying sine rule Analysis and Solution
P
(i) One length and all angles, or
(ii) Two side and an angle opposite one of the
given sides. 10m 15m
(iii) Two angles and one side opposite one of the
given angles
0
Examples 40
Q R
( a ) QRS is a triangle with QS = 12m,
 RQS = 300 and QRS = 450, calculate the length RS Two sides and one opposite angle, thus we can apply sine
Analysis and Solution rule.
S
10 = 15
sin 40 sin Q
12m
sin Q = 15 sin 40
300 450 10
Q R = 1.5 ( 0. 6428)
30 + 45 + S = 1800 (Sum of  s in a ∆) Sin Q = 0. 9642
S = 180 – 75 Q = sin – 1 0. 9642
= 1050 = 74. 620
179
Cosine Rule
Applying the usual notations in acute and obtuse angled = 64 + 36 – 25
triangles on 159. 96
Cosine rule states that: cos A = 0. 7813
a2 = b2 + c2 – 2bc cos A A = cos - 1 0. 7813
b2 = a2 + c2 – 2ac cos B = 38. 620  38 .60
2001 / 26 Neco Exercise 15. 6 OM
c2 = a2 + b2 – 2ab cos C
In an acute – angled ΔXYZ, XZ = 9 cm, YZ = 11 cm and
Conditions for applying cosine Rule  XZY = 350 ; calculate  YXZ, correct to the nearest
(i) Given two side and the included angle. degree A. 570 B. 450 C. 330 D. 160 E. 30
(ii) Given all the three side of the triangle.
2002/52 (Nov) Exercise 15. 7 OM
Here the above stated rule will have to be
In triangle ABC, if A = 350, B = 240, b = 16m find a,
adjusted to make cos A or cos B or cos C the
A. 1. 41m B. 11. 35m C. 11. 53m
subject of the formula as case may be i.e.
D. 22. 56m E. 22. 62m
cos A = b2 + c2 – a2
2bc TYPES OF BEARING
Any given bearing angle is represented in two ways, namely
cos B = a2 + c2 – b2 - Compass bearing (cardinal points)
2ac - Three-digit bearing.
the others follow suit.
Compass bearing (cardinal points)
Examples This concept is based on cardinal points as can be obtained in
( i ) Find /XZ/ in the triangle below any standard compass such as shown below.
N (North)
X

2
m 0 W (west) E (East)
1
20
Z
Y 1
m
Analysis and Solution S (South)
Two sides and the included angle; thus cosine rule
applies here,
/XZ/2 = 22 + 12 – 2 x 2 x 1 x cos 120 N
= 5 + 4 Cos 600 since Cosine is – ve in 2nd quad NW (North West ) NE ( North East)
= 5 + 4 x 1/2
/XZ/2 = 7 W E
/XZ/ = 7
SW( South West) SE (South East)
(ii) In ∆ ABC below, find  BAC
A S
The starting point are based on North or South which are
measured to either East or West. You will agree with the
8
author that from North to East or North to west, we can only
6 cover between 00 and 900. The same applies to South to East
or South to West. Below are examples of compass bearing
B C
5 (i) N 300 E (ii) N 150 W (iii) S 500E (iv) S 550 W
Analysis and Solution
Three sides of ∆ then ,the adjusted cosine rule will N ( ii ) N
apply. (i) 0
cos A = c2 + b2 - a2 30
150
2xcx b
E W
cos A = 82 + 62 - 52
2x 8x 6

180
N
( iii ) ( iv ) ( ii ) N

W 1300
E E 1300 =
50 EE
0
E = S S50
0
50 55 0

500
S S
S
(iii)
Three digit bearing N N
This type of bearing bases its starting point from the 0
North only; then moves in a clockwise direction. The 60
movement is between 00 and 3600. =
Below are examples of 3 - digit bearing W = N 600 W
(i) 0100 (ii) 0800 (iii) 1500 (iv) 2500 (v) 3200 3000 0
300
The concept of quadrant is important here though to be
numbered in clockwise direction instead of the usual
anti – clockwise numbering thus we have the format as: (b) Covert the following compass bearing to a 3 digit
bearing
4th Quadrant 1st quadrant (i) N 650 E (ii) S 400 W
2700 and 3600 00 and 900 Solution
3rd quadrant 2nd quadrant N
N
1800 and 2700 900 and 1800
(i) 650
Solution diagrams =
N E 650
N N
( iii )
(i) ( ii ) = 0650
1500
0 0
010 080
N N
(ii)
N
N
(v)
( iv ) W E = 2200
i.e 40 + 180
2500 400
2200
3200
S

Relationship between three digits and 1997/40 OM


compass bearing Express the true bearing of 2500 as a compass bearing.
Any three – digit bearing can be converted to a compass A. N 200 E B. S 200 E C. N 200 W
bearing and vice versa. D. S700 W E. S 700 E
Examples Solution
( a ) Covert the following 3 digit bearing to compass
bearing.
(i) 0300 (ii) 1300 (iii) 3000
Solution
N
N
0
(i) 30
=
The required angle is 
300 E = N300 E But  = 250 – 180
= 700
Also  is between South and West
Hence  = S 70 W (D)

181
2000/16 OM Bearing of Y from X= 180 - a ( s on a straight line)
Which of the following bearing is equivalent to S 500 W = 180 - 400
A. 0400 B.1300 C. 2200 D. 2300 = 1400
Solution
1996/ 42 OM
A town P is 150km from a town Q in the direction 0500 what
is the bearing of Q from P ?
A. 0500 B. 1500 C. 2300 D. 2700 E. 3100
0
5 0

N p

The compass bearing is km 500


500 150
= 50 + 180
S
= 2300 three digit bearing (D)
Q
2001/26 Exercise15.8 OM
The bearing S 400 E is the same as Solution
A. 0400 B. 0500 C. 1300 D. 1400
QPS = 50 (Alt.  s)
But the bearing of Q from P is
2002/32 (Nov) Exercise 15.9 OM
180 + 50 = 2300 ( C )
The bearing S 500 W is the same as
A. 0500 B. 1300 C. 1400 D. 2300 2003/49 OM
The bearing of P from Q is x , where 2700 < x < 3600 . Find
2004/48 Neco Exercise 15.10 OM the bearing of Q from P.
The bearing S 400 W is the same as A ( x- 90) 0 B (x- 270 ) 0 C ( x - 135 ) 0 D (x -180 ) 0
A 0400 B 0500 C 1400 D 2200 E 2300 Solution
The range of the angle x is given as between 2700 and 3600
so we sketch as
REVERSED BEARING N
If the bearing of a point A is described from another
point B. Then, the bearing of B can be described from A
based on the first description.
Examples z
1. The bearing of p from q is 0600, what is the bearing P
of q from p ?
Solution
N
x
y Q
0
p
60
0
60
The bearing of Q from P = Z
z = y ( Alternate angles)
q
But y = x – 1800
The required bearing of q from p is 180 + 600 = 2400 Thus z = x – 1800 ( D )

2. The bearing of X from Y is 3200. What is the bearing 2001/37 Neco Exercise 15. 11 OM
of Y from X The bearing of a point Y from point X is 1500
Solution What is the bearing of X from Y?
N A 3300 B. 2240 C. 1500 D 1200 E 300

N 2002/ 26 Neco Exercise 15. 12 OM


x
a If the bearing of Lagos from Abuja is 199.90
What is the bearing of Abuja from Lagos?
b Y A. 340.10 B. 250.10 C. 109.90 D. 70.10 E. 19.90

320 2004/31 Neco Exercise 15.13 OM


a = b (alt.  s) The bearing of Y from X is 0850. What is the bearing of X
but b + 320 = 3600 ( s at a point) from Y ?
b = 360 – 320 = 400 A 0050 B 0950 C 1850 D 2650 E 2750
182
Bearing Problems tan  = 3 = 1.732

E.g. I Usman starts a 3 km walk from X and on a  = tan – 1 (1.7321)


bearing 0230. He then walks 4 km on a bearing 1130 to = 600
Y. What is the bearing of Y from X The required bearing in this case is 
= 0600 three digit bearing
N
E.g. 3 A ship sails 110 km from a port P on a bearing of
z 1130 0350 and then 116 km on a bearing of 1050 to
another point Q.
r p
Calculate: ( a ) The distance PQ ( b ) the bearing of Q from P
3km 900 4km Solution
N
0
23 1050

X Y 116

m
km

k
110
r = 230 (Alt  s) N N
0
p + 113 = 1800 ( s on straight line)
0 35
p = 180 - 1130
= 670 P Q
r+p = 23 + 67
= 900
To get Y bearing from X, we should know .
Here XY = Hypo, YZ = Opposite of  and XZ =
adjacent 1050
Thus a b
km
Tan  = 4/3 110

116k
N 1100 N
Tan  = 1. 333

m
0
 = tan – 1 (1. 333) 35
= 53. 10 P Q
But the required bearing
a = 35 (alt.  s)
0
= 230 +  b + 1050 = 1800 (Sum of  s on straight line)
= 230 + 53. 10 b = 180 – 105
= 076 .10 = 750
a + b = 350 + 750
E.g. 2 = 1100
From a point Z, 90m North of X, a boy walks 90 3 m (a)Distance PQ can be solved for by cosine rule since
eastwards to another point Y. Find the bearing of Y we have two sides and an included angle. Thus
from X.
/PQ/2 = 1102 + 1162 – 2 x 110 x 116 x Cos 1100
= 12100 + 13456 – 25520 ( - Cos 70) Since cosine is
Solution
negative in the 2nd quadrant where Cos 1100 is found. The
formula there is 180 - 
N
90 3 m /PQ/ 2 = 25556. 0 + 25520 (6. 3420)
Z Y = 25556 .0 + 8727. 84
/PQ/ 2 = 34283. 84
/PQ/ = 34283. 84
90m /PQ/ = 185. 1km

(b) The bearing of Q from P can be gotten with the aid of


. Thus to solve for  , we apply sine rule for ease.
PQ = 116 But PQ = 185. 1
X Sin 110 Sin 
Here opposite and adjacent are relevant
Sin  = 116 Sin 110
Thus
185. 1
tan  = 90 3
90
183
= 116 Sin 70 Since Sine is + ve in the 2nd quad. No Log
185.1 with formula 180 -  15 1. 1761
= 116 (0. 9397) Sin 35 1. 7586
185. 1 0 . 9347
Sin  = 0. 5889 33. 41 1. 5238
 = Sin – 1 (0. 5889) 0. 2575 1. 4109
= 360 Sin  = 0. 2575
Therefore; bearing of Q from P = 350 +   = Sin – 1 0. 2575
= 350 + 360 = 0710 = 14. 920
But the required bearing is 025 + 14.92
1996/12 OM = 039. 92
Two men p and Q set off from a base camp R = 0400 to whole
prospecting for oil. P moves 20km on a bearing of 2050
and Q moves 15km on a bearing 0600. Calculate the
(a) Distance of Q from P 1999/30 OM
(b) Bearing of Q from P The diagram shows the position of three ships A, B and C at
(Give your answer in each case correct to the nearest whole number) sea. B is due north of C such that /AB/ = /BC/ and the
bearing of B from A = 0400. What is the bearing of A from
Q C?
0 15km A. 0400 B. 0700 C. 1100 D. 2900
060
R 0 B
2050 040
A
m
20k

P
Solution
Resulting diagram  ABC = 400 (Alt  s)
 BCA = 180 - 40 (Base  s of isosceles triangle).
15km Q
60
0 2
R Thus,  BCA = 70
1450 But the required Bearing =360 –  BCA
= 360 – 70
m

250
= 2900 (D)
20k

0
25
2002/ 11 OM
P Q
1300
8m
0
( a ) Distance PQ can be solved from 2nd diagram by 50
Cosine rule since we have two sides and an included 50
0 13m
angle.
/PQ/ 2 = 202 + 152 - 2 x 20 x 15 cos 145 P
= 400 + 225 – 600 ( - cos 35)
= 625 + 491. 49
/PQ/ 2 = 1116. 49 R
In the diagram, /PQ/ = 8m, /QR/ = 13m, the bearing of Q
PQ = 1116. 49 = 33. 41  33km to whole from P is 0500and the bearing of R from Q is 1300.
(a) Calculate correct to 3 significant figures,
( b ) The bearing of Q from P can not be resolved (i.) \PR\ (ii) the bearing of R from P.
without knowing . It can be solved for by sine rule. (b) Calculate the shortest distance between point Q
15 = 33. 41 and PR. Hence the area of triangle PQR
Sin  Sin 145
Sin  = 15 Sin 145
33. 41
= 15 Sin 35
33. 41

184
Analysis and Solution (ii.) To calculate the area of ∆PQR, we apply any of the
Q suitable formula for finding area of. ∆
1300
Thus Area of ∆PQR = ½ p r sin Q
8m
0
50 a = ½ x 13 x 8 x sin 100
13m = 4 x 13 x sin 80
0 =52 x 0.9848
50 = 51. 21m2
P ≅ 51.2m2

R 2001/11 Neco OM
a = 180 – 130 = 50 (sum of  s on straight line)
0 0 There are three cities A, B and C within a state. If city B is
 PQR = 500 + a = 1000 24m on a bearing of 1500 from A and city C is 32m from
(i) To find PR, we apply cosine rule; since we have two city A and city A is on bearing of 0150 from city C.
sides and an included angle. Calculate the:
Thus /PR/ 2 = 82 + 13 2 – 2 x 8 x 13 cos 100 (i) Distance between city B and city C, to two decimal places;
= 64 + 169 – 208 (- cos 800) (ii) Bearing of city C from city B.
= 233 + 36.12 Analysis and Solution
N
/PR/ 2 = 269.12
/PR/ = 269.12 A 1500
/PR/ = 16.40
≅ 16.4m to 3 s.f

24
32
(ii.) To find the required bearing we need to calculate

m
N N
for  and this can be done easily by sine rule. Thus 0
15
13 = 16. 4
Sin  Sin 100 C B
Sin  = 13 sin 80 Resulting Diagram
16.4 N
No Log
13 1.1139 A 1500
sin 80 1.9934 ab
1.1073
450
24

16. 4 1. 2148
m

N
32

N
0.7807 1. 8925 15
0

c
Sin  = 0.7807 C B
 = Sin – 1 0.7807 a = 150 (Alt  s)
= 51. 30 b + 1500 = 1800 (sum of  s on a straight line)
Hence, the required bearing is 50 + 51.30 = 101.30 b = 300
Also C = b = 300 (Alt s)
(i.) To find distance BC, we apply cosine rule, since we have
(b) (i.) The shortest distance between Q and PR will be two sides and an included angle.
the ∆PQR altitude (or the perpendicular height)
Thus /BC/ 2 = 322 + 242 – 2 x 32 x 24 x cos 45
Q = 1024 + 576 – 1086.12
8m 1000
/BC/ 2 = 513. 88
/BC/ = 513. 88
13
P 51.3
0
m /BC/ = 22. 668
≅ 22. 67 to 2 d.p
M (ii.) The bearing of C from B can not be solved easily
without finding the value of  by sine rule .Thus
R
Thus, QM = PQ sin 51. 3 32 = 22.67
= 8 sin 51. 3 Sin  Sin 45
= 8 x 0.7807 Sin  = 32 sin 45
= 6.246m 22.67
≅ 6. 25m

185
No Log (ii) What is its bearing from it i.e .  Which can be solved for
32 1.5051 easily by sine rule . Thus
Sin 45 1. 8495 450 = 1320.65
1.3546 Sin  Sin 127
22.67 1.3555 Sin = 450sin 127 = 0.2721
0.9979 1. 9991 1320.65
 = sin –1 0.27 21 = 015.790
Sin  = 0. 9979
 = sin-1 0.9979 2004/10b Neco OM
= 86.30 A courier bus travels from head office in Lagos to a town B
The required bearing is 210km away on bearing of 0550 to deliver a mail . It then
= 360 – (c + ) = 360 – (30 + 86.30) changes course and moves to its branch office also in Lagos
= 360 – 116. 3 on a bearing of 2200 . If the branch office is directly east of
= 243.70 the head office , calculate, correct to 3 significant figures ;
( i ) the distance between the head office and the branch
2002/10a NABTEB OM office
An Aeroplane leaves an airport, flies due North for ( ii ) how far is town B from the branch office.
2 hours at 500km/hr. It then flies 450km on a bearing Problem diagram
0530. How far is the plane from the airport and what is N
its bearing from it?

C B
m
450k
530 2200
B
N
km
210
N S
0
055

H
b

Resulting diagram
Resulting Diagram N
C
m
450k
530 B
B 127 0
2200
N
km 150 0
210 40
1000km
N
S
0 400
055
350 900
H
b
A

Also  ABC = 180 – 530 (sum of s on straight line)  NBb = 2200 but NBS = 1800 thus SBb = 2200- 1800 i.e 400
= 1270  BHb = 900 – 550 = 350
(i) How far is the plane from the airport i.e /AC/  NHB = HBS Alternate angles i.e 550
We apply cosine Rule since we have two sides and an But HBb = 55 – 40 i.e 150
included angle. ( i ) Also applying sine rule to find Hb
Thus /AC/2 = 450 + 10002 – 2 x 450 x 1000 x cos 1270 Hb = 210
= 202500 + 1000000 – 900000 (-cos. 53) Sin 150 Sin 1300
=1202500 + 900000 (0.6018) Hb = 210 Sin 150
/AC/2 = 1744120km Sin 1300
/AC/ = 1744120 km = 70. 95km
/AC/ = 1320. 65km = 71. 0km to 3 sf

186
( i i ) Applying Sine rule X
210 = Bb
m
Sin 1300 Sin 350 8c
210 Sin 350 = Bb
Sin 1300 45
0

157.24 = Bb Z 9 00
Bb  157km to 3 sf
2004/30 Neco OM 6c
m
A man walks 15km due North from street A to another
street B and then to a bus stop S which is 7.5 km due Y
east of B . Find the bearing of bus Stop S from street A
A 1350 B 063.40 C 0450 D 026.60 E 022.50 Since  Z is 900 , Cosine rule will reduce to Pythagoras rule
Solution because Cos 900is zero . Thus ,
N / XY/2 = 82+ 62
East 7.5km
B S = 64 + 36
0 / XY/2 = 100
90
/XY/ = 100 = 10cm ( A )
10km

15km 2003/30 Neco OM


Victoria moves from a point A on a bearing of 0350 to point
B, a distance of 9m . She then moves to point C a distance of
A 12m on a bearing of 1250. How far is she from her starting
point ?
The required bearing is θ ; but we need side AS to A 7. 9m B 15. 0m C 37. 8m D 38.2m E 54. 0m
achieve it
By Pythagoras rule B 1250
/AS/2 = ( 7.5 )2 + ( 15 )2
= 56.25 + 225

12
m
/AS/2 = 281.25 N
9m

/AS/ = 281. 25
= 16.77km C
Applying Sine rule 35 0

7.5 = 16.77 A
Sin θ Sin90
Resulting diagram
7. 5 Sin90 = 16.77 Sin θ
7. 5 Sin90 = Sin θ 1250
B
16.77
Sin θ = 0 .4472 35 0 55
0
12
m

θ = Sin - 1 0.4472 N 90 0
9m

= 26. 560 C
In three digits bearing 026. 60 ( D ) 35 0

A
2003/15 Neco OM
The bearing of two points X and Y from Z are 450 and
1350 respectively. If /ZX/ = 8cm and /ZY/ = 6cm, find The required distance is AC
/XY/ Which can be gotten by Pythagoras rule since B= 900 .
A 10cm B 10 2 cm C 14cm D 28cm E 100cm /AC/2 = 9 2 + 122
X = 81 + 144
/AC/2 = 225
m /AC/ = 225 = 15m
8c

45
0 2002/ 25 Neco Exercise 15. 14 OM
1 35
0 A boy leaves station X on a bearing of 0350 to station Y,
Z which is 21km away. He then travels to another station Z on
a bearing of 1250. If Z is directly East of X, what is the
6c
m distance from X to his present position?
A. 3.0km B. 12.04km C17.20km
Y D 34.18km E. 36.61km

187
2000/14 – 15 Exercise 15.15 & 15.16 OM 2004/6b Exercise 15.21 OM
A tree is 8km due south of a building. Kofi is standing A point P is 40km from Q on a bearing 0610. Calculate
8km west of the tree. correct to one decimal place, the distance of P to
Use this information to answer question 14 and 15 (i ) north of Q ; (ii ) east of Q.
Exercise 15.15 OM
14 How far is Kofi from the building 2004/7 ( Nov ) Exercise 15.22 OM
A. 4 2 km B. 8km C 8 2 km D. 16km The diagram shows the positions of two ships G and K
Author’s Hint: 128 = 8 2 relative to harbour H

Exercise 15.16 OM
15 Find the bearing of Kofi from the building ? K G
A. 3150 B. 2700 C. 2250 D. 1350

2001/28 Exercise 15.17 OM


Three observation posts P, Q and R are such that Q is
due east of P and R is due north of Q , if \PQ\ = 5km

km
8k
m

10
and \PR\ = 10km, find \QR\
A 5. 0km B 9. 5km C 7. 6km D 8. 7km

2001/12 Exercise 15.18 OM


H
T
K is 8km on a bearing of2900 from H and G is 10km on a
N bearing of 0500 from H .
580
0 ( a ) Determine  KHG
B 161
( b ) Calculate , correct to one decimal place:
( i ) the distance KG ;
18

( ii ) the bearing of G from K.


m

cm
15

053
0 N
N
2004/32 ( Nov ) Exercise 15.23 OM
A The bearing of a town Q from town P is 2150 . P is 80km
C north of R while R is due east of Q . Find the distance
In the diagram, three points A, B and C are on the same between Q and R correct to the nearest km .
horizontal ground. B is 15m from A, on a bearing of A 46 km B 56km C 98km D 140km
0530 . C is 18cm from B on a bearing of 1610. A
vertical pole with top T is erected at B such that angle 2004/26 Neco Exercise 15.24 OM
ATB = 580, Calculate, correct to 3 sig. Figures: A man travels 24km due North and then 7km due West.
(a) The length AC (b) The bearing of C from A; What is his bearing from the starting point correct to the
(c) The height of the pole BT. nearest degree ?
Author’s hint : convert 18 cm to metres A 160 B 740 C 1060 D 1640 E 3440
2002/16 Exercise 15.19 OM
N
CHAPTER SIXTEEN
P Latitude & Longitude
34 0
This is one of the theory based areas to be tested in certificate
exams. The formula of the length of an arc of a circle plays a
crucial role here.

Q
O
From the diagram, find the bearing of Q from P r r
A. 2360 B. 2140 C. 1460 D 1240
A B
2002/3 Neco (Nov) Exercise 15.20 OM
Two villages T and S are 12km and 8km away from the
same base R and on the bearing of 0200 and 1600 from i.e circle with centre O
R respectively. Calculate the distance of T from S to the Length of arc AB = θ x 2 r
nearest km 360

188
The Earth is spherical in shape. Any cut or slice through
it horizontally (Latitude) or vertically (Longitude) is Solution
circular in shape. To find the distance between any two N
points on such slice we have to use the above- 650
0
mentioned formula A 40 N

N
Greenwich 00
Meridian
D
Equator 350S
B
W E
S

Angular distance = 400 + 350


= 750
Note that we add when the points are in different
S hemispheres i.e South and North.
The measurement of any point on the latitudes either to
the east or west must start from the Greenwich Example 2
meridian; while that of longitude either to south or north Find the angular distance between X (600N, 200W )
must start from the equator - please note the difference. and Y ( 400 N, 200W)
Any point on the earth surface is the intersection of Solution
(latitude, longitude) like in co-ordinates (x,y) N
0 X
Small circles X 60 N
Great circles
Y 400N Y
R
Z 0
0
600 0
40
Z R

200W
S

Angular distance = 600 – 400 = 200

1989/9 OM
Units of Measurement P and Q are two places on the same circle of latitude 790S. P
Degrees and seconds are the units of measurement. is on longitude 680E, while Q is on longitude 220W. The
Latitude is from 00 to 900 south or North ; while angular distance between P and Q is
Longitude is from 00 to 1800 East or West
A. 120 B. 450 C. 480 D. 900 E. 920
Analysis and solution
0 0
30 E N
300W 220W
0
N
0
680E
60 W 600E 80 N0
60 N
300N r
00 220 68
0 r
W E 0
300S W 0E
Q
600S P
0 Q P 0
80 S 79 S
Longitudes S Latitudes
S

Angular distance = 220 + 680


= 900 (D)
Example 1 and not 68 – 22 since they are west & east and not West,
Find the angular distance between West or East, East.
A (400 N, 650E) and B (350 S, 650E)

189
1995/30 OM Note: To check the antilog value of 4.0023 candidates should
The positions of two countries P and Q are (150N, 120E) check the first line i.e 00 under 2 difference 3 then apply the
and (650N, 120E) respectively. What is the difference in antilog characteristics of 4
latitude?
A 1040 B 1000 C 800 D 500 E 240 Example 2
Analysis Solution Find the distance between G (300N, 500W)
N 12 0 E and H(300S, 500W)
Analysis and solution
65 0N N
Q G
15 0N G 300N R
P
300
W E
W E 300

R
H 300S H

S S
Difference in latitude 650 – 150 = 500N (D)
We subtracted because they are in the same hemisphere Here θ = 300 + 300 = 600
North, North Distance GH = θ x 2R
360
Thus distance GH = 60 x 2 x 6400
Distance along great circles 360
Distance along any of the longitudes and distance along = 6400
the equator only among the latitudes are called distance 3
along great circles. The formula used is No Log
Lengths of arc = θ x 2R 6400 3.8062
360  (3.142 ) 0.4972
Where R is the radius of the earth. 4.3034
3 0.4771
Example 1. Find the distance between towns 6703 3.8263
A(00 S, 30W ) and town B (00 S, 600E )
Analysis and Solution 1988/44 OM
For any latitudes and longitudes problem it is necessary Cotonou and Niamey are on the same line of longitude and
that candidates should sketch by free- hand, the Niamey is 70North of Cotonou. If the radius of the earth is
required diagram. Do not use compass since the earth 6400km, how far is Niamey north of Cotonou along the line
shape is not a circle but a sphere of longitude correct to the nearest kilometer?
N (Take = 22/7)
A. 391km B.503km C.782km
D.1006km E. 2012km.
Analysis and solution
R R The formula for distance = θ x 2R since we are talking
W 00 30
0 0
60
A B
360 about longitude.
The question specified that Niamey is 70 North of Contonou
A B
and not Niamey is 70 North  θ = 70
S We sketch as: since the questions did not specify East or
West.
Here θ = 300 + 600 = 900 N N
Thus distance AB = 90 x 2R
360 N N
Always use the quoted  & R values
R = 6400km C OR C
AB = 90 x 2 x  x 6400 = 3200 W E W E
360
No Log
3200 3.5051
 (3.142) 0.4972
10051km 4.0023 S S
190
The resultant sector is: Recall that equator is a Great circle.
N Thus Distance XY =  x 2 R
R 360
Where  = 123 – 670 = 560 (East, East )
0
70
Distance XY = 56 x 2 x 22 x 6400
R 360 7
C = 88 x 640
9
Distance CN = 7 x 2 x 22 x 6400 No Log
360 7 88 1.9445
= 11 x 640 = 782km (C) 640 2. 8062
9 4. 7507
9 0. 9542
1994/28 OM special case 6259. 0 3.7965
Two ships on the equator are on longitudes 450W and
45 0E respectively. How far are they apart along the Distance XY = 6259 km.
equator, correct to 2 significant figures
(Take the radius of the earth = 6400km and  = 22/7 ) (ii) Distance X to North pole is also a distance along
A 1500km B 10 000km C 6400km D 5000km great circle (i.e longitude 670 E)
E 3200km. Distance X to North =  x 2 R
N 360
45 0W 45 0E Here  = 90 – 0 (North is 900, Equator is 00 )
0 0

Distance X to North = 90 x 2 x 22 x 6400


360 7
= 11 x 6400
W E 7
A B = 70400
7
= 10057km.

S 1995/10b OM
Recall that only the equator among the latitudes uses the An aircraft flies due South from an airfields on latitude 360N,
formula for Great circles. longitude 1380E to another airfields on latitude 360S longitude
Distance AB = θ x 2R 1380E.
360 (i) Calculate the distance travelled, correct to 3 S.f
Here θ = 450 + 450 = 900 (ii) If the speed of the aircraft, is 800km per hour, calculate the
Distance AB = 90 x 2 x 22 x 6400 time taken, correct to the nearest hour
360 7 (Take  = 22/7 , R = 6400km).
= 11 x 6400 Analysis and solution
7 N
= 10057.14 1380E
 10,000km to 2 sf. (B) 360N 0
36 N
N
2000/13a OM Special case R
Two places X and Y on the equator are on longitudes 670 36
W E
E and 1230 E respectively. 36
(i) What is the distance between them along the equator? R
0
(ii) How far is X from the North pole? S 0
36 S 36 S
(Take = 22/7 and radius of the earth = 6400km)
Analysis and solution S
N 0
67 E
(i) Distance NS =  x 2R (Distance along great circle)
1230E 360
Angular distance = 360 + 360 = 720 (Different hemisphere)
 Distance NS = 72 x 2 x 22 x 6400
W E 360 x 7
X Y
= 88 x 640
7
= 8045.7  8050 km to 3sf.

191
(ii) Speed = Distance Distance along small circles
Time (Distance along parallel of latitudes)
800 = 8050 This is any distance along the latitudes except the equator.
Time
Time = 8050 = 10. 06  10 Hrs N
800 A
1997/15 OM B
Two points P and Q are on longitude 670W. Their C
latitudes differ by 900. Calculate their distance apart in
terms of  ( Take radius of the earth = 6400km ) D
A. 6400km B. 6400/ km C. 3200km E
D. 3200km E.3200/ km. Here distance between any
Analysis and solution S
two points on latitudes along A, B,
Read the question once more, you will see that the C, D, E is derived as shown next
question talks about distance along the longitude which
implies great circle.
Though their point of location either in North or South
was not specified but their angular distance is given.
N
Hence, two diagrams represent the problem; either of
P r X
them is correct Y
N N
670W 670W R
P Z
Q OR O

W E W E
P
S
Assuming X to be a point on the earth surface with latitudes Ф
Q
(phi), of course XY is a small circle with radius r. The resulting
S S ∆ OPX has
Resulting sector: Cos Ф = r
P R
R Where R is the radius of the earth
90
0 r – small r
R – Big R
R Thus to calculate the distance of any two points on the
Q parallel of latitudes, we apply the formula
Distance = θ x 2 r but r = R cos Ф
Distance PQ =  x 2 R 360
360
= θ x 2 R cos Ф
= 90 x 2 x 6400 x  360
360 = 3200 km (C) Where θ is the angular distance of the points on the
2004/36 OM longitudes to East or West.
Two places are 2816km apart on the same line of Ф is the angle between the given parallel of latitude and the
longitude. Calculate the angular difference between
equator i.e latitude measurements is to North or South
their latitude ( R = 6400km )
Note: θ and Ф are all dummy variables they can be changed
A 25.20 B 26.10 C 51. 30 D 63. 90 E 64.80
Solution but always remember what is attached to them.
By the phrase “same line of longitude” shows that we
are to make θ the subject formula in :
Distance along longitude = θ x 2  R
360
2816 = θ x 2 x 3.142 x 6400
360
θ = 2816 x 360
2 x 3.142 x 6400
= 25.20 ( A )
192
E.g1 Find the distance XY if X ( 300N, 500E) The question says Abidjan is 40 west of Accra not Abidjan is
and Y (300N, 100W) 40 west.
Solution θ = 40 ( angular difference )
N 50 0E r = R cos Ф is given as 6370km i.e radius of cicle of lat.
10 0 W
Abidjan and Accra.
X 30 0N N
Y
30 0 r
W E AB
AC
40 r
AB
W E
AC

S
Distance XY = θ x 2R Cos Ф
360 S
Here θ = 10 + 50 = 600 and Ф =300
Distance AB to AC = 4 x 2 x 22 x 6370
XY = 60 x 2R Cos 30 360 7
360 = 22 x 2 x 91
= R√3Km 9
6  445km (B)

E.g 2 Two towns A and B are on the same parallel of 1988/9 OM


latitude 600S. Their longitudes are 350E and 1550E Two towns K and Q are on the parallels of latitude 460N. The
respectively. Calculate their distance AB along the longitude of town K is 1300W and that of town Q is 1030 W.
parallel of latitude.(take  = 22/ 7 R = 6400) A third town P also on latitude 460 N, is on longitude 230E
Solution Calculate.
N (i) The length of the parallel of latitude 460 N, to the
nearest 100km,
350E (ii) The distance between K and Q, correct to the
1550E nearest 100km,
(iii) The distance between Q and P measured along
E the parallel of latitude, to the nearest 10km.
W
(Take  = 3.142 , R = 6400km)
Solution
B
600S The sketch is as shown below :
A

S 103 W
0
N
AB = θ x 2R Cos Ф 230E
360 1300W
0
Here θ = 1550 – 350 = 1200 and Ф = 600 46 N
R
Q
P r 0
23
0 r
27
AB = 120 x 2 x 22 x 6400 cos 60 W E
360 7 R P
1300W
= 22 x 6400 230E
3 x7 = 6705km Q
1030W
1988/37 OM
Abidjan is 40 west of Accra and on the same circle of S
latitude. If the radius of this circle of latitude is
6370km. How far is Abidjan west of Accra, correct to
the nearest kilometer? ( = 22/7 ) (i) The length of the parallel of latitude formula
A. 222 km B. 445km C. 890km = θ x 2 r, Where r = R cos Ф
D. 5005km E.10010km. 360
Analysis and solution will apply because the distance is along latitude which is not
The formula for distance = θ x 2R CosФ the equator.
360 Also θ = 3600 since it is a complete circle and Ф = 460
will apply here since we are talking about distance Alternatively, reason it as a circumference of the latitude
along latitude. = 2 r where r = R cos Ф
Which ever way can serve

193
Length of the parallel of latitude 460N Analysis and solution
= 2 x 3.142 x 6400 x cos 460
= 6.284 x 6400 x cos 460 N
400W 200E
No Log_
6.284 0.7983 40N
6400 3.8062 P Q
cos460 1. 8418 0
40 20
0

27950 4.4463 W E
 28000km P Q

(ii) The distance between K and Q.


Since it is along the latitude 460N;
Distance KQ = θ x 2R Cos Ф S
360 Here, distance PQ = θ x 2R cos Ф
But θ = angular distance between K and Q 360
= 1300 – 1030 = 270 and Ф = 460 Angular distance between P & Q is θ = 400 + 200= 600
Substituting ; and Ф = 40
Thus distance KQ Distance PQ = 60 x 2R cos40
= 27 x 2 x 3.142 x 6400 x Cos 460 360
360 = R cos40 (D)
= 3 x 3.142 x 6400 x Cos 460 3
20 1994/12 a OM
= 9.426 x 6400 x Cos 460 Two points X (320 N , 470W) and Y (320 N , 250E) are on
20 the earth’s surface. If it takes an aeroplane 11 hours to fly
= 9.426 x 320 x Cos 460 from X to Y along the parallel of latitude. Calculate its speed,
No Log correct to the nearest kilometer per hour.
9.426 0. 9744 (Take the radius of the earth = 6400 km and  =22/7 )
320 2. 5051 Analysis and Solution
cos 460 1. 8418 N
2095 3. 3213 47 0W 25 E
0

 2100km X
Y 32 0N
(iii) Distance between Q and P
= θ x 2R Cos Ф W
32 0
E
360
Reason – distance along parallel of latitude other than
the equator.
Here θ = angular difference= 1030 + 230 = 1260and Ф =
460 S
Substituting Speed = Distance from X to Y
Distance QP = 126 x 2 x3.142 x 6400 x cos 460 Time taken
360 We are asked to find speed; time is given but distance from
= 7 x 3.142 x 640 x cos 460 X to Y is not. But it can be solved for from the given data.
Distance X to Y along parallel of latitude
No Log = θ x 2R cos Ф
7 0.8451 360
3.142 0.4972 θ = 47 + 25 0 = 720 and Ф = 320
0
640 2.8062 Substituting
cos46 1.8418 Distance XY = 72 x 2 x 22 x 6400 x Cos 320
9779 3.9903 360 7
 9780km = 88 x 640 x Cos 320
7
1991/24 OM No Log
Two towns, P and Q are on (40N, 400W) and (40N, 88 1.9445
200E) respectively. What is the distance between them 640 2.8062
along their line of latitude? ( Give your answer in terms Cos 320 1. 9284
of the  and R , radius of the earth ) 4.6791
A. 2R cos40 B. Rsin40 C. R/2 cos 40 7 0.8451
D. R/3 cos4 0
E. R/6 cos4 0
6823km 3. 8340 Distance XY = 6823 km
194
Therefore Speed = 6823 1996/11a OM
11 P and Q are points on the parallel of latitudes 68.70S, their
longitude being 1240W and 560E respectively. What is their
No Log distance apart measured along the parallel of latitude?
6823 3.8340 [ Take R = 6400km,  = 3.142 ]
11 1.0414 (Given your answers to 3 s.f. )
620.3 2.7926 Analysis and solution
N
Speed  620km/Hr 1240W 560E

1995/3b OM
P and Q are two towns on the earth’s surface on latitude
560 N. Their longitudes are 250E and 950E respectively. W E
Find the distance PQ along their parallel of latitude,
correct to the nearest km. 68.70 S
P Q
(Take radius of the earth as 6400km and  = 22/7 )
Analysis and solution
N 95 0E S
Parallel of latitude other than the equator mentioned implies
25 0E small ‘r ’ .
56 0N Also, the mentioning of points P in the west and Q on the
P
Q east implies angular distance is addition between the two
angles.
W E
Distance PQ =  x 2 R Cos Ф
360
Where  = 1240 + 560 = 1800 and Ф = 68.70
Substituting
S Distance PQ = 180 x 2 x 3.142 x 6400 x cos 68.7
Distance along latitude =  x 2R Cos Ф 360
360 = 3.142 x 6400 x cos 68.7
 = 95 – 250 = 700 and Ф = 560
(Reason :  is in East, East ) No Log
Substituting 3.142 0.4972
6400 3.8062
Distance PQ = 70 x 2 x 22 x 6400 x cos 560
cos 68.7 1.5602
360 7
7305 3.8636
= 11 x 6400 x Cos 560
 7310km (to 3.s.f)
9
No Log
1998/17 OM
11 1.0414
The radius of a geographical globe is 60cm. Find the length
6400 3.8062
of the parallel of latitude 600N
Cos560 1. 7476
4.5952 A. 66cm B. 60 cm C. 30cm D.15cm E.6cm
9 0.9542 Analysis and solution
4375. 0 3.6410 The question refers to the total length of the parallel of latitude
and not distance between two points on it.
Distance PQ = 4375km Length of the parallel of latitude 600N
= 360 x 2 R Cos 600
1996/24 Exercise 16.1 OM 360
Two towns X and Y both on latitude 600S have longitude Here; R is 60cm and  is 3600 angles of a circle i.e
270 E and 330W respectively. Find to the nearest complete revolution
1
kilometres, the distance between X and Y measured = 2 x 60 x /2
along the parallel of latitude. = 60 cm. (B)
(Take 2R = 4 x 104km, where R is the radius of the Alternatively
earth). Perimeter of circle (circumference) 2r
A 28850km B 16667 km C 8333km Here r = R Cos Ф
D 6667 km E 3333km = 2 x  x 60 x Cos 60
= 60cm (B)

195
2003/13 Neco OM Example 1
Given that the radius of the earth is 6400km , find the P and Q are two points on latitude 550N and their longitudes
parallel of latitude whose radius is 1600km correct to are 330 W and 230 E respectively. Calculate the distance
the nearest degree between P and Q measured along:
A 140 B 450 C 760 D 900 E 1430 (a) the parallel of latitude
Solution (b) great circle
By the phrase “parallel of latitude whose radius” shows (Take  = 22/7, Radius of the earth = 6400km)
that we are to make Φ the subject formula in Solution
r = R cos Φ 33 W
0
N
1600 = 6400 cos Φ
0
23 E

1600 = cos Φ
0
55 N
P Q
6400
0.25 = cos Φ W
55
E
Thus, Cos – 1 0.25 = Φ
75.520 = Φ
Φ  760 to the nearest degree ( C )
S
2003/8 Neco Exercise 16.2 OM (a) From the resulting diagram,
A town X is on longitude 400W and latitude 500S . Distance PQ = θ x 2R Cos Ф
Another town Y is on the same latitude as town X but 360
on longitude 300E . Find the ; Here θ = 33 + 23 i.e 560 and Ф =550
( i )circumference of the circle of latitude 500S correct PQ = 56 x 2 x 22 x 6400 x cos 55
to the nearest 100km 360 x 7
( ii ) distance between X and Y measured along their = 44 x 1280 x 0. 5736
parallel of latitude to the nearest 10km . 9
(Take radius of the Earth as 6400km and  = 22/7 ) = 3589 km
(b) The distance between P and Q measured along the great
circle is also the shortest distance between P and Q.
Method I (SPLIT)
Shortest distance between any two points 0 0
33 W
N
56 230E
on the parallel of latitude i . e
X
0
(Distance along the great circle of two 55 N
r r P Q
points on the parallel of latitude). 0
28 28
0 P Q
R
Apart from the normal questions on distance along the E
great circle or along the parallel of latitudes, there are
P Q W R R
X
other questions on the shortest distance between any
two points on the parallel of latitudes or distance along Fig II
the great circle of any two points on the parallel of Fig I
latitude. Below is a sketch of how the problem looks S
like: From fig I
The length of chord PQ = 2 XQ
= 2r sin θ
2
C = 2 R Cos Ф sin θ
A B 2
= 2 x 6400 x cos 550 sin 28
D = 2 x 6400 x 0.5736 x 0.4695
= 3447 km

From fig II
This form of question are interwoven in the sense that the Arc PQ is the required distance
data given concerns mainly arc AB through D but = 2α x 2 R
candidates are required by the question to find out some 360
facts on arc AB through C. This can be done in two ways α is derived from fig. II as shown below:
namely:
Sin α = XQ
Split and direct formula methods
R

196
= 1/2 length of chord PQ (a) Distance PQ along parallel of latitude
R =  x 2 R Cos Ф
= 1/2 x 3445 360
6400
Where  = 200 +350 = 550, Ф = 310
Sin α = 0.2691 Substituting
α = sin –1 (0.2691) PQ = 55 x 2 x 3.142 x 6400 x Cos 31
= 15.60 360
Thus Arc PQ = 2 x 15.6 x 2 x 22 x 6400 = 55 x 6.284 x 6400 x cos 31
360 x 7 360
= 3486.47
 3486 km No Log
55 1.7404
Method II (Applied formula ) 6.284 0.7983
To avoid error due to students’ judgment of three- 6400 3.8062
dimensional shape in two dimensions, the above cos31 1. 9331
procedures can be summed up as follows; 6.2780
360 2.5563
Distance PQ along great circle 5269.0 3.7217
= [2 sin – 1 (cos Ф sin θ /2 ) ] x 2 R
Therefore distance PQ = 5269 km
360
Where θ = angular distance or longitude difference.
Ф = is the given latitude to North or South
Substituting (b) Distance PQ along great circle sketched diagram
Distance PQ = [ 2 sin-1 (cos 55 sin 56/2)] x 2 x 22 x
6400 P Q
360 x 7
= [ 2 sin-1 ( 0.2691)] x 281600
360 x 7
= 31.20 x 28160
36 x 7
= 3486km = 2 sin-1 (cos Фsin /2) 2R
Note 360
θ and Ф are dummy variables, they can be changed to r Where  = 550, Ф =31
and s, or a and b etc. But the values attached to them Substituting,
remain unchanged. Distance PQ along great circle
The Author applied, method I to solve the given problem = 2sin – 1 (cos 31 Sin 55/2) 2 x 3.142 x 6400
for students to appreciate and be able to recall easily 360
–1
formula II which can be used without drawing much = 2 sin (0.8572 x 0.4617) x 6.284 x 6400
diagrams. Candidate should note that the answer to the 360
great circle distance is always smaller in value than that = 2sin – 1 (0.3958) x 6. 284 x 160
of the distance along parallel of latitude. 9
= 46.63 x 6.284 x 160
2001/4 Neco OM
9
P (lat. 310N long. 200 W) and Q(lat. 310 N. long 350E)
= 5209km.
are two points on the earth’s surface. Calculate correct
Note:
to the nearest km, the distance along
Answer to (b) is smaller than (A) implying the shortest
(a) the parallel of latitude
distance.
(b) a great circle
( Take radius of the earth = 6400km and  = 3.142)
2004/10 Neco ( Nov ) OM
N A and B are points on the parallel of latitude 540N their
200W 350E longitudes being 440W and 1360E respectively . What is their
310N distance apart measured along :
P Q ( a ) the parallel of latitude
550 ( b ) a great circle (correct your answer to the nearest km ) .
W E [Take  to be 22/7 and radius of the Earth (R) = 6400km]
P Q

S
197
Solution Analysis and solution
( a ) Distance apart measured along parallel of latitude 620W N
180E O
= θ x 2  r where r = R cos Φ
360 780N
N FigI V U FigII
0
44 W r 800 r
0
136 E 0
62 18 0
W E
0
54 N
A B U V
W E
S
(i) Distance UV along parallel of latitude
=  x 2 R Cos Ф
360
S
Here θ is the angular distance of points on the latitude Where  = 62 +18 = 800 andФ =780
i.e Substituting,
440 + 1360 = 1800 Distance UV = 80 x 2 x 3.142 x 6400 x Cos78
and Φ is latitude measurement to North or South i.e 540 360
No Log
(a ) Distance along = 180 x 2 x 22 x 6400 x cos 540 160 2.2041
parallel of lat 360 7 3.142 0.4972
= 44 x 6400 x 0.5878 6400 3.8062
14 cos780 1.3179
= 11823.18km 5. 8254
 11823 km to the nearest km 360 2. 5563
1858.0 3. 2691
(b ) Distance apart measured along the great circle
= [ 2 Sin –1 ( Cos Φ Sin θ/2 ) ] x 2  R  DistanceUV = 1858 km.
360
= [ 2 Sin –1 ( Cos 54 Sin 180/2 ) ] x 2 x 22 x 6400 (ii) Length of the chord UV.
360 x 7 Recall that: to determine the length of any chord, we need to
= [ 2 Sin – 1 ( 0. 5878 x 1 ) ] x 44 x 6400 know the angles it subtends at the centre Thus,
360 x 7 Chord length UV = 2 r sin /2 = 400
= ( 2 x 36 ) x 44 x 6400 Where  = angle subtended at centre = 800 Thus, /2 = 400
360 x 7 r = R cosФ
= 8045.71 km Where Ф= 780 is the latitude angle.
 8046km Substituting,
Chord UV = 2R Cos 78 Sin 40
= 2 x 6400 Cos 78 sin 40
Miscellaneous cases = 12800 Cos 78 sin 40
No Log.
2000/5 Neco OM 12800 4. 1072
The latitudes and longitudes of two points U and V on Cos 78 1. 3179
the earth’s surface are: Sin 40 1. 8081
U (Latitude 780N; Longitude 180E ) 1711. 0 3. 2332
V (Latitude 780N; Longitude 620 W)
=1711.0 km
Calculate :
Alternatively,
(i) The distance of arc UV along the parallel of latitude. Diagrammatically, we have that
(ii) The length of the chord UV
O
(Taking R = 6400 km,  = 3.142) 800
r 0
r
40 40
0

U V
W
Fig III

UW = VW
Hence UV = 2VW
But VW = sin 400
198
r But distance PQ = 1450km (Given) and Ф = 400
VW = r sin 400 But r = R cos 780 Substituting
VW = R cos 780 sin 400 1450 =  x 2 x 3.142 x 6400 x cos 40
Thus, UV = 2 VW 360
= 2 R cos 780 sin 400 1450 =  x 2 x 3.142 x 6400 x 0.7660
= 2 x 6400 cos 780 sin 400 360
= 12800 cos 78 sin 40 1450 x 360 =
= 1711. 0 km 6.284 x 4902.4
 = 16. 940
Longitude x0 E 52 + 16. 94
Table work for 12800 cos78 sin 40 here
 Longitude of Q = 68.94  690E
No log
(c )
12800 4.1072 S
cos 78 1.3179

N
60 0
sin 400 1. 80 81
1711.0 3. 2332

2002/6Neco OM 20
0
Q
40 N
0
A moving object takes off from a town P (latitude 400N,
longitude 520E) and after flying 1450km due East, it
reaches a town Q, it then flies due North to another town The distance between Q and S along the parallel of
S on latitude 600N. longitude has  as 200
Calculate: = 20 x 2 R
(a) The radius of line of latitude through P. 360
(b) The longitude of Q correct to the nearest degree = 20 x 2 x 22 x 6400
(c) The distance between Q and S along the parallel of 360 7
longitude correct to 3 significant figures. = 2234.92 km
(Take R = 6, 400km,  = 3.142)  2230 km to 3 s.f
Analysis and solution
S 2001/13b Exercise 16.3 OM
520E A point X is on latitude 280N and longitude 1050 W.
600N Y is another point on the same latitude as X but on longitude
S
400N 350E
Q (i) Calculate, correct to 3 s.f.., the distance between X
W P
E and Y along latitude 280N
(ii) How far is X from the equator?
(Take  = 22/7 and the radius of the Earth as 6,400km).

2002/11 Neco (Nov) Exercise 16.4 OM


N Two cities X and Y are on latitude 380S and their longitudes
(a) Radius of parallel of latitude 400 N are 500 E and 1300 W respectively.
r = R cos 40 Calculate to 3 significant figures.
= 6400 x 0.7660 (a) the speed of x due to the rotation of the Earth.
= 4902. 4 km (b) the distance between X and Y along a great circle
(b) (c) the distance between X and Y along the parallel of
latitude.
[Take  = 22/7 and radius of the Earth (R) as 6400km]
Q 2003/11 OM
0
XE an aeroplane flies due west for 3 hours from P ( lat 500
N,long 600W)to a point Q at an average speed of 600kmhr-1
P The aeroplane then flies due South from Q to a point Y
520E 500km away.
Calculate , corrct to 3 significant figures,
To answer the question we should first find  ; which ( a ) the longitude of Q
can be gotten from: ( b ) the latitude of Y
Distance PQ along parallel of latitude (Take radius of the Earth to be 6400km and  = 22/7 )
=  x 2 R cos Ф
360
199
Solution Solution
6 0 0W N 5 0 0W
N
0
50 N K
0
60 N
Q L
P
N
W E W E

S
( a ) The longitude of G can be gotten from distance S
PQ along parallel of latitude
( i ) Longitude of L can be gotten from :
= θ x 2  r where r = R cos Φ
360 Distance along parallel of latitude KL = θ x 2r
Distance PQ is not given . But from : Speed = Distance 360
where r is R cos Φ
Time
i.e 3520 = θ x 2 x 22 x 6400 x cos 600
Distance PQ = speed x time
360 7
= 600 x 3
θ = 3520 x 360 x 7
= 1800km
Substituting , 2 x 22 x 6400 x 0.5
1800 = θ x 2 x 22 x 6400 x cos500 θ = 630
360 Longitude of L = 500 + θ
θ = 1800 x 360 x 7 = 500 + 630
2 x 22 x 6400 x 0.6428 Moving from 50 W to east , we can only move 500reversed
0

after which we find ourselves in the east. The remaining 130


θ = 25.060
is on the east .
Longitude of Q is 600 - θ i.e 600 - 25.060 = 34.940W
Longitude of L = 130E
 34.90W to 3 sf
( b ) The latitude of Y can be gotten from :
( ii ) The latitude of N can be gotten from :
Distance along longitude Q to Y = θ x 2  R
Distance along longitude KN = θ x 2  R
360
360
500 = θ x 2 x 22 x 6400
i.e 10951 = θ x 2 x 22 x 6400
36 0 7
360 7
θ = 500 x 360 x 7 θ = 10951 x 360 x 7
2 x 22 x 6400 2 x 22 x 6400
= 4.470 θ = 97.9990
Latitude of Y is 500 - 4.470 = 45. 530N The latitude of N = 600 + θ
 45. 5 0N to 3sf = 600 + 97.9990
We subtracted because the flight toY was due south Moving from 60 N to South we can only move 600reversed
0

from point Q after which we find ourselves in the Southern hemisphere


.The remaining 37. 9990 is in the South.
2004/10 OM i.e The latitude of N is 37. 9990S
K ( lat. 600N , long. 500W ) is a point on the earth’s
surface . L is another point due east of K and the Third 2003/8 NABTEB Exercise 16.5 OM
point N is due south of K. The distance Klis 3520km An aeroplane took 4 hours to fly from a town
and KN is 10951km. P(lat450N, long 750E ) to another town, T(lat 450,long150E) .
( a ) Calculate : The aeroplane changed course and eight hours after leaving T,
( i ) the longitude of L ( ii ) the latitude of N. arrived at a third town Q ( lat 00 , long 150E ) . If the flight
(Take  = 22/7 and radius of the Earth = 6400km ) from P to T was along the line of parallel of latitude, and that
from T to Q along the meridian , calculate , correct to 3
significant figures,
( i ) the total length of the journey
( ii ) average speed of the aircraft .
Take ( R = 6400km and  = 22/ 7 )

200
Chapter Seventeen = 2 x APB
From (3) AÔB = B ÔQ – AÔQ
Circle geometry =2y2 – 2x2
A well-grounded knowledge about the theorems
= 2 (y2 – x2)
involved in circle geometry is what forms the basis for
attempting questions under this topic. = 2 x APB
Added value to this is a good background on properties In all cases AÔB = 2 x APB
of triangle and straight lines.
(2) Theorem
Angles in the same segment of a circle are equal
P Q
CHORD
x1 x2

O
Properties of Chords
(a)The line joining the center to the mid-point of a A 2x
B
chord is perpendicular to the chord as indicated
in the diagram below
Given: Points P,Q on the circumference of a circle APQB.
To prove : APB = AQB
Construction: Join A and B to O, the center of the circle.
Proof:
AÔB = 2x1 (s at centre = 2 x at circum)
(b) Equal chords of a circle are equidistant from the = 2x2
centre and conversely  x 1 = x2
(3) Theorem
OTHER BASIC PROPERTIES OF CIRCLES The angle in semi- circle is 900
P
(1) THEOREM
The angle which an arc of a circle subtends at the centre
is twice that which it subtends at any point on the
remaining part of the circumference.
P A B
O
P
y2
x2
x2 y 2
y1
x1
y1 x1 B P = 900
O B A
O
Q (4) Theorem
A Q Angles in opposite segments are supplementary.
(2)
(1) (opposite angles of a cyclic quadrilateral are supplementary)
B
O A p
Q x2 P x
y2
x1
y1
A (3)
B y
q
C
Given: a circle APB with centre O D
To prove: AÔB = 2 x APB
Construction: Join PO and produce it to any point Q x + y = 1800
Proof: p + q = 1800
/OA/ = /OB/ (radii)
 x1 = x2 (Base s of iso. ∆ ) (5) Theorem:
The exterior angle of a cyclic quadrilateral is equal to the
 AÔQ = x1 + x2 (Ext.  of ∆ AOP)
interior - opposite angle.
AÔQ = 2x2 (x1 = x2) B
Similarly , BÔQ =2y2 A
x
(1) AÔB
(2) reflex AÔB = AÔQ + BÔQ
= 2x2 + 2y2
= 2(x2 + y2) x
C D
201
(6) Theorem ( 10)Theorem
A tangent is perpendicular to the radius at the point of Q

contact R

K
O

S
A B
C

(7)Theorem If QK is a secant and QS is a tangent to a circle, then


The angle between a tangent to a circle and a chord SQ2 = QR . QK
through the point of contact is equal to the angle in the
alternate segment. ( 11)Theorem
B B
Transverse common tangent to two circles
S

r
C a
OR K
a C W
O
r
a a
Q A T Q A T T
SK = OK = r
( 8)Theorem TK WK r
Two chords PT and RS of a circle intersect outside or
inside a circle at a point Q Problems on circle geometry
Q
2005/12 UME
T In the diagram below, O is the center of the circle, UOT =
P
R 70o and RST =100o. Calculate RUO.
P S
A. 50o B. 20o C. 80o D. 25o
R
Q S

.
o
100
S O

R 70
T

PQ ∙ QT = QR ∙ QS U T

Solution
( 9)Theorem  OTU is an isosceles one; since sides OT and OU are radii
Two tangents SR and SK to a circle with cenre O are of the circle.
equal Thus; û = T (Base angles of Isosceles )
in length Ô + Û + T = 180o (sum of s in )
R
70 + x + x = 180
70 + 2x = 180
2x = 180 – 70
2x = 110
O S 2x = 110
2 2
x = 55o
Now,  RUT + RST = 180o (opposite segment angles)
K
RUT + 100 = 180
RUT = 180 – 100
SR = SK,  RUT = 80o
OSR = OSK RUO = RUT – OUT
SOR = SOK = 80 – 55O
= 25O (D)
R = K = 90o

202
2004/9 UME 2003/27 UME
Q

.
R
o
35

O
R

20 o . O
P
40
S

S
In the diagram above, PQR is a straight line and PS is a
P
tangent to the circle QRS with /PS/ = /SR/ and SPR = 40o .
P, R and S lie on a circle of center O as shown above, Find PSQ
while Q lies outside the circle, Find PSO. A.20o B.10o C. 40o D. 30o
A. 45o B.55o C. 35o D. 40o Solution
Solution In PRS
In  PQR P = R = 40o (base angles of isosceles )
P + Q + R = 180o (sum of angles in ) PSQ = R. = 40o ( Tangent and alternate segment angles)
20 + 35 + R = 180
= 180 – 55 2004/24 PCE
J
= 125o
PRS is lying on straight line QRS K
 PRS = 180 – 125o
= 55o. 88 M
But POS = 2  PRS (circumference & center angles) O
POS = 2  55
= 110o
Note that OPS is isosceles (Both sides are radii of the circle) L
Thus PSO = SPO The figure above is a cyclic quadrilateral JKLM with
i.ePSO = 180 – 110 KOM = 88o. Find  JMK.
2 A. 22o B. 44o C.132o D. 136o
= 70 Solution
2 = 35o (C) KLM = KOM  2 (Circumference & center angle)
KLM = 88  2 i.e 44o
2003/23 UME KJM + KLM(44o) = 180o(opposite angles in cyclic quad.)
P KJM = 180 – 44 i.e 136o
KJM is isosceles. Thus JMK = 180 – 136
N

42
o
.
O
2
= 44 i.e 22o (A)
2
2003/20 PCE
Q
M

In the diagram above, O is the center of the circle, POM O

is a diameter and MNQ = 42o . Calculate QMP 156


A. 138o B. 132O C. 42o D. 48o P
Solution x
R

MNQ = MPQ = 42o (angles in the same segment)


PQM = 90o (angle in semi circle) In the diagram above, find the value of the angle x.
QMP + MPQ + PQM = 180o (s in ) A. 112o B. 102o C. 122o D. 132o
QMP + 42o + 90o = 180 Solution
QMP = 180 – 132 POR (reflex)  2 = x (Circumference & center angles)
= 48o (D) But POR is 360 – 156o = 204o (angles at a point)
POR  2 = 204  2
= 102o i.e x (B)

203
2000/24 UME 2000/25 PCE
S
g

0
97
0
63
50
P R
30

Q
In the diagram above, if  RPS = 50O, RPQ = 30o and
PQ = QR, find the value of PRS. The angle marked g in the diagram above is
A. 50o B. 60o C. 70o D. 80o A. 43o B. 37o C. 34o D. 24o
Solution Solution
PQR is isosceles. Thus Let us re-label the diagram as.
PQR = 180 – (2  30o) [Base angles of isosceles ] g
= 180 – 60
a
= 120o 0
97
 PSR +  PQR = 180o (opposite angles of cyclic quad.) 63
0
b
 PSR + 120o = 180o
PSR = 180 – 120 i.e 60o
In PSR
SPR + PRS +  PSR = 180o (sum of angles in )
50 + PRS + 60 = 180
PRS = 180 – 110 a + 97o = (angles on a straight lie)
= 70o (C) a = 180 – 97 i.e 83o)
b = 63o (interior  = Exterior opposite )
2002/26 PCE Now, a + b + g = 180o (sum of s in )
83 + 63 + g = 180o

.
g = 180 – 146 i.e 34o (C)
X Y
O 1998/26 PCE
Q

R
P

In the diagram above, O is center of the circle. Find the O


relationship between X and Y.
A. x + y = 90o B. x – y = 90o
C. x + y = 180 o
D. x – y = 180o
45 0 25 0
T S
Solution
In the figure above O is the centre of the circle PQR,
Z
 RTS = 45o, TSP = 25o, Find PQR.

X
.
O
Y
A.70o
Solution
B. 110o C. 125o

Let us re-label the diagram as:


Q
D. 315o

R
P
o
Z = 90 (angle in semi-circle) O d

Z + x + y = 180o (sum of angles in ) c a


b
90 + x + y = 180o
x + y =180 – 90o
x + y = 90o (A)
45 0 25 0
T S
(a + b +c) 2 = PQR (Centre and angle at circumference)
But in  TOS
b + 45 + 25o = 180o (sum of s in )
b = 180 – 70
b = 110
204
But at point O 1998/28 UME
b = d = 110 ( vertically opposite angles ) X

a + b + c + d = 360o (angles at a point)


(a + b + c)o = 360 – d i.e 360 – 110 32 0
(a + b + c)o = 250o
 PQR = (a + b + c)o  2
= 250  2 R
Y
= 125o (C)
1999/27 UME
40 0

T Q

P T TQ is tangent to circle XYTR, YXT = 32o, RTQ = 40O.


Q
x Find  YTR
A. 108 o B.121o C. 140 o D.148 o
Solution
RXT = 40o (Tangent and alternate segment angle)
1100 RXY = 40o + 32o i.e 72o
S
But YTR + RXY = 180 (opposite angles in cyclic quad)
R
YTR + 72o = 180
In the figure above PQRS is a circle with ST//RQ. YTR = 180 – 72
Find the value of x if PT = PS = 108o (A)
A. 70o B. 55o C. 40o D. 35o
Solution 1995/26 UME Exercise 17.1
x = T (Corresponding angles)
SPT + 110o = 180o (opposite angle of cyclic quad)
SPT = 180o – 110 i.e 70o
SPT is isosceles. Thus 810
x
T = 180 – 70
2
= 110
2 = 55o i.e x = 55o (B)
53 0

1998/25 UME

Determine the value of x in the figure above.


A. 134o B. 81o C. 53o D. 46o

1995/27 UME Exercise 17.2


Z

Y
X

In the diagram above, PR is a diameter of circle PQRS.


50 0
PST and QRT are straight lines. Find QSR.
A. 20 o B. 25o C. 30o D. 35o P T
Solution
PRQ = 30o + 35o (Exterior  = 2 interior opposite s) PT is a tangent to the circle TYZX, YT = YX and PTX =
PRQ = 65o 50o. Calculate TZY.
A. 50o B. 65o C. 85o D.130o
Also PSQ = 65o (s in same segment)
Since PR is a diameter
PSR = 90o ( in semi-circle)
Thus; QŜR = 90 – 65
= 25O (B)

205
1994/27 UME Exercise 17.3 1994/32 PCE
R

.
R
P O
38 0 O
S
48 0

S P T
The diagram above shows a circle of center O with P,R,T as
In the diagram above, O is the center of the circle. If points on the circumference and TS as a tangent, if RTS =
SOQ is a diameter and  PRS is 38o , 48o find the value of  ORT.
what is the value of PSQ? A.24o B.42o C.48o D.96o
A.148o B.104o C.80o D.52o Solution
P = 48o(Alternate segment angle and tangent)
1994/31 UME Exercise 17.4
2  P = ROT (circumference and center s )
R 2  48o = ROT i.e 96o
 ROT is isosceles. Thus
60 0 ORT = 180 – 96
S 2
= 84 i.e 42O (B)
2
50 0
Q T 1993/32 PCE Exercise 17.5
Q
T
x

.
In the diagram above, PTS is a tangent to the
P
circle TQR at T. Calculate RTS. O
A. 120o B. 70o C. 60o D. 40o 350

1994/28 PCE R

In the diagram above, O is the center of the circle.


O x0 y0
TQ is a tangent at Q and angle PRQ = 35O,
While angle QTR = x , find the value of x.
A. 55o B. 40o C. 35o D. 20o

In the diagram above, O is the centre of the circle. The 1992/25 UME Exercise 17.6
relationship between xo and yo is P
A. x + y = 180o B. ½ xo + yo = 180o
o
C. x + y = 240 D. ½ xo + yo = 240o 500
Solution
If you say x + y =180o (opposite segment angles)
You are wrong. Rather Q
(360 – x ) 2 = y (circumference and center angles) O x
180 – ½ x = y
180 = y + ½ x (B)

If point O is the center of the circle in the diagram above,


calculate the value of x.
A. 40o B. 45o C.50o D. 55o

206
1991/35 PCE Exercise 17.7
Q
2  R = XOY (circumference and center angle)
OXP = 90o (Tangent and radius angles)
OYP = 90o (Tangent and radius angles)
 + 90o + 140o + 90o = 360o (sum of angle in quad)
O  + 320o = 360 – 320
y
= 40o (D)
x

P R 1993/28 UME
P

In the diagram above, P, Q, R are three points on a


circle center O. Angle QPO is x and angle QRO is y.
Which of the following is equal to the reflex angle
T S
.
O
Q

POR?
A.180-x-y B.2x + 2y C.360-x -y D.360 -2x - 2y

R
1991/35 PCE Exercise 17.8 In the diagram above, PQRS is a circle with O as center and
P
PQ//RT. If RTS = 32o find PŜQ
A. 32o B. 45o C. 58o D. 90o
Solution
O
RTS = PQS = 32O (alternate angles)
SPQ = 90o (angle in semi – circle)
S Q PŜQ + SPQ + PQS = 180o (sum of angles in )
140 0 PŜQ + 90o + 32o = 180
R PŜQ = 180 – 122
PQRS is a cyclic quadrilateral, with O as the centre of = 580 (C)
the circle. The value of angle  is? 1990/37 UME Exercise 17.9
A. 280o B. 80o C. 70o D. 40o P

Q
1990/36 PCE
X
0 x
81 220 T
S R

R
P In the figure above, PQRS is a circle. If PQT and SRT are
straight lines, find the value of x.
A. 59o B. 77o C. 103o D. 121o
Y

In the figure above PX and PY are tangents to the circle


1989/31 UME Exercise 17.10
and XRY = 70o. Find the value of XPY.
A. 140o B. 110o C. 70o D. 40o M

Solution
Let us re-label the diagram applying the relevant
theorem N

R
0
O 40 0
R 70
P MN is a tangent to the given circle at M. MR and MQ are two
chords. If QMN is 60o and MNQ is 40o, find RMQ
A. 120o B.110o C. 60o D. 20o
Y

207
1987/31 UME Special cases
Q
1999/31 UME
x
Q

R T
30
0 9
O
P

x
P
6
In the diagram above, POQ is a diameter, O is the Z
T

center of the circle and TP is a tangent.


Find the value of x. In the figure above, TZ is tangent to the circle QPZ.
A. 30o B. 40o C. 450o D. 50o Find x if TZ = 6 units and PQ = 9units
Solution A. 3 B. 4 C. 5 D. 6
OQR is isosceles(OQ and OR are same radii ) Solution
Thus ORQ = x (Base angles of isosceles ∆ ) Applying tangent and secant theorem
In PQR ZT2 = PT ∙ QT
QRP = 90o (angle in semi- circle) ZT2 = x( PQ + x )
In ORP 62 = x( 9 + x )
ORP = 90 – x. 36 = 9x + x2
In PRT x + 9x – 36 = 0
2

 PRT = 180 – (90 – x) ( s in a straight line = 1800 ) ( x + 12)(x – 3) = 0


 PRT = 180 – 90 + x x = 3 A.
= 90 + x
Also,  TPR = x (Tangent and alternate segment s) 1993/34 UME
S
Thus 30 + 90 + x + x = 180o(sum of angles in  TPR)
120 + 2x = 180
5 cm

2x = 180 – 120
2x = 60 Q
O
x = 30o (A)

2 cm
2005 /13 PCE Exercise 17.11
K
T
80o
In the figure above, the line segment ST is a tangent to the
two circles at S and T. O and Q are the centers of the circles
P yo with OS = 5cm, QT = 2cm and OQ = 14cm. Find ST
Q
76o
A. 7 3 cm B. 12cm C. 87 cm D. 7cm
Solution
to Let the point of intersection of the two lines be Z. Then let us
re-label applying transverse common tangent theorem
M S
Given that PKQM is a quadrilateral inscribed in a
circle, find the value of (t + y)o
5cm

A. 100o B. 103 o C. 104o D. 204o Z x Q


O 14 - x
2cm

2006 / 8 UME Exercise 17.12


S

T
145 o
R
Then
P
xo
. O
Q
OZ = 5
QZ 2
14 – x = 5
x 2
2(14 – x) = 5x
In the diagram above, POQ is a diameter of the circle 28 – 2x = 5x
PQRS, If PSR = 145o, find xo 28 = 5x + 2x
A 45o B. 25o C. 55o D. 35o 28 = 7x
29
208
Thus, x = 4 Solution
We can only get ST by Pythagoras rule in each of the Applying two chords of circle intersecting outside theorem
right – angled triangles. IG.GH = EG.GF
In  OSZ 6  GH = 8  3
(14 – x)2 = 52 + SZ2 GH = 8  3
(14 – 4)2 = 52 + SZ 2 6
102 = 52 + SZ2 GH = 4cm (C)
10 – 52 = SZ2
2

(10 – 5)(10 + 5) = SZ2 1986/33 UME Exercise 17.14


5  15 = SZ2 In the diagram below, PQ and RS are chords of a circle
center O which meet at T outside the circle. If TP = 24cm,
553 = SZ
TQ = 8cm
5 3 = SZ and TS = 12cm, find TR
Also in QTZ
P
x2 = 22 + T Z2
Q
42 = 22 + TZ2
4 – 22 = TZ2
2
O T
(4 – 2)(4 + 2) = TZ2
2  6 = TZ2 R
S
223 = TZ
2 3 = TZ
A.16cm B.14cm C.12cm D.8cm
Therefore; ST = SZ + TZ
=5 3 + 2 3
= 7 3 cm (A)

1991/35 UME Exercise 17.13


T

O Q P

PMN and PQR are two secants of the circle MQTRN


and PT is a tangent.
35. If PM = 5cm, PN = 12cm and PQ = 4.8cm ,calculate
the respective lengths of PR and PT in centimeters.
A. 7.3, 5.9 B. 7.7, 12.5 C. 12.5, 7.7 D. 5.9, 7.3

1991/33 PCE
I
H

E
G
.O F

Two chords EF and IH intersect at G when produced,


If EG = 8cm, GF = 3cm, and IG = 6cm, find GH.
A. 8cm B. 6cm C. 4cm D. 2cm

209
Chapter Eighteen Class marks ( Midpoints )
This is the mid point of the class interval and is obtained by
Statistics II (Grouped data) adding the lower and upper class limits and dividing by two
Frequency Distribution (2). Thus the class mark of the interval 40 – 42 is
When summarizing large masses of raw data, it is often 40 + 42 = 41
useful to distribute the data into classes or categories and 2
to determine the number of individuals belonging to each Measure of location for grouped data
class called the class frequency. The tabular arrangement (Mean, median & mode)
of data by classes together with their corresponding class
frequency is called frequency distribution or frequency Mean
table. Mean for grouped data can be calculated in two ways:
Example (i) Mean for problems without assumed mean
Mass (kg) Number of Students x = fx
40 – 42 5 f Where x is the class mark or class midpoint
43 – 45 18 (ii) Mean for problems with assumed mean
46 – 48 42 x = A + fd
49 – 51 27 f where A = Assumed mean ;
52 – 54 8 Total = 100 d = deviation from the mean (x – A)
The table above shows a frequency distribution of The first method though easy to manipulate involves bigger
masses (recorded to the nearest Kg) of 100 students values and if the question did not restrict you to any particular
attending SSCE lesson. Any data organized and method i.e if the word “otherwise” is used in questions
summarized as in the above frequency distribution are involving assumed mean ; then Student should stick to the
often called Group data. easiest one for safety.
Class interval & Class limits 1990/7 OM
A symbol defining a class such as 40 – 42, 43 – 45 in the The weights to nearest kilogram, of a group of 50 students in
above table is called a class interval – while the end a college of technology are given below:
numbers 40 and 42, 43 and 45 are called class limits; the 65 70 60 46 51 55 59 63 68 53
smaller numbers 40, 43, ... are the lower class limits and 47 53 72 58 67 62 64 70 57 56
the bigger numbers: 42, 45; ... are the upper class limits 73 56 48 51 58 63 65 62 49 64
53 59 63 50 48 72 67 56 61 64
Class boundaries 66 52 49 62 71 58 53 69 63 59
If masses are recorded to the nearest kg, the class interval
40 – 42 theoretically includes all measurements from (a). Prepare a grouped frequency table with class
39.5 to 42.5 kgs which are called class boundaries or intervals 45 – 49, 50 – 54, 55 – 59 etc.
True class limits. The smaller number 39.5 is the lower (b). Using an assumed mean of 62 or otherwise calculate
class boundary while the bigger number 42. 5 is the upper the mean of the grouped data, correct to one decimal
class boundary. Displaying the given table to reflect class place
boundaries, we have Solution
Class interval Class boundaries Frequency 7 (a).
40 – 42 39. 5 – 42 .5 5 Class Interval Tally Frequency (f)
43 – 45 42. 5 – 45 .5 18 45 – 49 IIII I 6
46 – 48 45. 5 – 48 .5 42 50 – 54 IIII III 8
49 – 51 48.5 – 51. 5 27 55 – 59 IIII IIII I 11
52 – 54 51.5 – 54 . 5 8 60 – 64 IIII IIII II 12
65 – 69 IIII II 7
Total = 100 70 – 74 IIII I 6
In practice, the class boundaries are obtained by Applicable to all raw or scattered data problems
adding the upper limit of one class interval to the lower Note
limit of the next higher class interval and dividing by The problem of grouping data when they are scattered or raw
two can be solved, if students use four figure table plus
i.e 43 + 42, 46 + 45, 49 + 48 mathematical set or any opaque object to cover the data
2 2 2 leaving one column or row at a time. Allocate the numbers
Class Size there to their various class intervals with the aid of tally. Then
This is the difference between the lower and upper class move to the next row or column as shown below. Alternatively
boundaries or the difference between two upper limits, or write out the data as they appear in the column or row; then
two lower limits. The class size is also called the class allocate them to the various class intervals with the aid of tally.
width or class strength. In this case 42.5 – 39.5 = 3 Repeat the process for the next column or row depending on
Or 43 – 40 = 3 or 45 – 42 = 3 your chosen pattern.
210
Class interval Tally Method II (Assumed mean used)
65 45 – 49 I x = A + fd where d = x – A ( 62 )
47 50 – 54 I f
73 55 – 59 We then prepare a table to reflect x, f, d, and fd items in the
53 60 – 64 formula.
66 65 – 69 II Class interval x f d=x–A fd
70 – 74 I 45 – 49 47 6 -15 -90
Next, we attend to 2nd column 50 – 54 52 8 -10 -80
70 Class interval tally 55 – 59 57 11 -5 -55
53 45 – 49 I 60 – 64 62 12 0 0
56 50 – 54 III 65 – 69 67 7 5 35
59 55 – 59 II 70 – 74 72 6 10 60
52 60 – 64 f fd
65 – 69 II = 50 = - 130
70 – 74 II x = A + fd
f
Next, we attend to 3rd column = 62 + - 130
60 Class interval tally 50
72 45 – 49 Il l = 62 – 2.6
48 50 – 54 III = 59. 4 to 1 dp
63 55 – 59 II
49 60 – 64 ll 2004/11 a & b ( i ) Neco OM
65 – 69 II The table below gives the masses in Kg of 35 students in
70 – 74 IIl a particular school .
45 43 54 52 57 59 65
The pattern continues, each addition to the tally was
50 61 50 48 53 61 66
differentiated for emphasis.
47 52 48 40 44 60 68
The lesser the figures the better. A student who chooses
51 47 51 41 50 62 70
the row aspect of this data will likely have problems or
58 42 51 49 55 71 60
slower compared to the person picking his data from
(a ) using the above given data , construct a group
column. The pattern to be followed depends on the way
frequency table with class interval 40 – 44 , 45 – 49,
the data given are presented.
50 – 54 etc
Column
( b ) From the table , using an assumed mean of 52.0 and
65
correct to 2 places of decimal, calculate the ( i ) mean
47
73 Solution
53 Here assumed mean was specified
66 Mean = A +  f d
Row: 66, 52, 49, 62, 71, 58, 53, 69, 63, 59 f
where d = x - A (i.e 52.0 )
7(b) Candidate should note the use of the word We then prepare a table to reflect x, f, d and fd items in the
“otherwise” here. Thus, they required any of the two formula
types of solutions displayed below: Class interval x Tally f d=x -A fd
Method I (Assumed mean not used) 40 – 44 42 IIII 5 -10 - 50
45 – 49 47 IIII I 6 -5 - 30
x = fx
50 – 54 52 IIII IIII 10 0 0
f
55 – 59 57 IIII 4 5 20
We then prepare a table to reflect x, f and fx which are
60 – 64 62 IIII 5 10 50
items in the formula
65 – 69 67 III 3 15 45
Class interval Class mark (x) f fx
70 – 74 72 II 2 20 40
45 – 49 47 6 282
f fd
50 – 54 52 8 416
55 – 59 57 11 627 35 75
60 – 64 62 12 744 Mean = A +  f d
65 – 69 67 7 469 f
70 – 74 72 6 432 = 52. 0 + 75
f =50 fx 35
= 2970 = 52. 0 + 2.143
= 54.143
x = fx = 2970
 54 . 14 to 2dp
f 50
= 59.4 to 1 dp.
211
2003/14 Neco 2005/14 Neco (Dec)
The table below shows the frequency distribution of the The number of eggs collected daily by a poultry farmer for
intelligence quotients (I.Q) X of students in an 50 days are as follows:
institution. 72 45 84 69 28 49 61 42 50 76
X 82-85 86-89 90-93 94-97 98-101 102-105 106-109 110-113 69 53 75 64 63 48 60 35 25 78
F 6 19 32 49 71 92 75 56
45 68 35 66 62 54 79 43 57 80
(a) Calculate, correct to the nearest whole number and 67 72 52 78 62 56 74 57 50 65
using an assumed mean of 99.5, the; 30 50 51 40 73 63 82 58 69 66
( i ) mean (correct to 2 d.p.) (a)Using the above data, and a class interval of 20 – 29,
( ii ) standard deviation (correct to d.p.) 30 – 39, … e.t.c, Calculate the
(b) If a student is randomly selected, What is the ( i ) mean ( ii ) standard deviation
probability that his I:Q is at least 94? (correct to 2.d.p.) (b) If a day is picked at random, what is the probability
Solution that 66 eggs were collected that day?
For grouped data assumed mean formula Solution
Here we work without assumed mean; for grouped data
Mean = A +  f d and S.D = 2
 f f x f x 2

2 Mean =  fx and S.D =   


 f d2   f d  f f  f 
  
f Next, We prepare a table to reflect x, f, fx, x2 and fx2 items in
 f 
the formula.
We then prepare a table to reflect x, f, d, fd,d2 and fd2
Class x Tally f fx x2 fx2
items in the formula. where d = x – A( 99.5 ) interval
Class Class 20 - 29 24.5 II 2 49.0 600.25 1,200.50
interval mark x
f d= x-A fd d2 f d2 30 - 39 34.5 III 3 103.5 1190.25 3,570.75
82-85 83.5 6 -16 -96 256 1536 40 - 49 44.5 IIII II 7 311.5 1980.25 13,861.75
86-89 87.5 19 -12 -228 144 2736 50 - 59 54.5 IIII IIII I 11 599.5 2970.25 32,672.75
90-93 91.5 32 -8 -256 64 2048 60 - 69 64.5 15 967.5 4160.25 62,403.75
IIII IIII IIII
94-97 95.5 49 -4 -196 16 784
70 - 79 74.5 IIII IIII 9 670.5 5550.25 49,952.25
98-101 99.5 71 0 0 0 0
102-105 103.5 92 4 368 16 1472 80 - 89 84.5 III 3 253.5 7140.25 21,420.75
106-109 107.5 75 8 600 64 4800 f = 50 fx= f x2 =
110-113 111.5 56 12 672 144 8064 2,955 185,082.5
 f= fd  f d2
400 = 864 =21,440 2955
(i) Mean =
50
a (i) Mean = A +  f d = 59.1
f
864 2
= 99.5 + 185082.5  2955 
400 (ii) S.D =  
= 99.5 + 2.16 50  50 
= 101.66 correct to 2 dp.
2
= 3701.65  3492.81
 f d2   f d 
( ii ) S.D =    = 208.84
f  f 
= 14.45
2
21440  864  No of 66eggs from the raw data
=   (b) Pr of 66 eggs =
400  400  Total number of eggs
2
= 53.6  4.6656 =
50
1
= 48.9344 =
25
= 6.9953
 7.00 correct to 2 dp. 2000/15 Neco
b Prob. (I.Q) at least 94 = Summation of 94 freq upward to 113 Below is the distribution of marks of 90 candidates in a JSC
Total freq summation examination.
49  71  92  75  56 Mark 10-19 20-29 30-39 40-49 50-59 60-69 70-79 80-89
= Frequency 5 14 16 15 12 10 8 10
400
343 Using an assumed mean of 54.5, calculate the;
= ( i ) mean mark ( ii ) variance of the distribution.
400
212
Solution (d) % < 31 days or % > 45 days
For grouped data, assumed mean formula Since they are strict inequalities, they are not part of the
show, thus we take the lower part and upper part respectively
 f d and variance =  f d
2
Mean = A + – 4 62
 f f % < 31 days =  100 or % > 45 days =  100
40 40
2
fd  = 10% or 20%
 
 f  2005 / 13 (Nov)
Mass 20-29 30-39 40-49 50-59 60-69 70-79
Next, We prepare a table to reflect x, f, d, fd,d and fd 2 2
(kg)
items in the formula. where d = x – A( 54.5 ) Number
of Sheep
187 252 196 130 88 47

Class Class f d=x-A fd d2 fd2


interval mark x The table shows the distribution of the masses of sheep in a pen.
10 – 19 14.5 5 -40 -200 1600 8000 (a) Using an assumed mean of 34.5, find:
20 – 29 24.5 14 -30 -420 900 12600 (i) the mean
30 – 39 34.5 16 -20 -320 400 6400 (ii) the standard deviation, of the distribution.
40 – 49 44.5 15 -10 -150 100 1500 (b) If a sheep is taken at random from the pen, what is
50 – 59 54.5 12 0 0 0 0 the probability that it’s mass is at least 50kg?
60 – 69 64.5 10 10 100 100 1000 Solution
70 – 79 74.5 8 20 160 400 3200 For grouped data, assumed mean formula
80 – 89 84.5 10 30 300 900 9000 2
 f d2   f d 
Mean = A +  f d and S.D =
f= fd= fd2 =   
90 –530 41,700  f f  f 
  530  Next, We prepare a table to reflect x, f, d, fd,d2 and fd2 items
( i ) Mean = 54.5 +   in the formula. where d = x – A( 34.5 )
 90 
(a)
= 54.5 – 5.8889 Class Class f d=x - A fd d2 fd2
 48.61 interval mark x
2 20 - 29 24.5 187 - 10 -1870 100 18700
41700   530  30 - 39 34.5 252 0 0 0 0
( ii ) Variance =   40 - 49 44.5 196 10 1960 100 19600
90  90  50 - 59 54.5 130 20 2600 400 52000
= 463.333 – 34.679 60 - 69 64.5 88 30 2640 900 79,200
= 428.654 70 - 79 74.5 47 40 1880 1600 75200
f = fd = fd2 =
2009 / 14 c and d 900 7210 244,700
The distribution of the lives (in days) of 40 transistor
batteries is shown in the table 7210
( i ) mean = 34.5 +
Battery life
(in days)
26-30 31-35 36-40 41-45 46-50 51-55
900
Frequency 4 7 13 8 6 2 = 34.5 + 8.011
(c) Using an assumed mean of 43days, calculate the = 42.51
mean of the distribution. 2
244700  7210 
(d) What percentage number of batteries will live for ( ii ) S.D =  
less than 31days or more than 45days? 900  900 
Solution
(c) Assumed mean formula = A +  f d = 271.889  64.178 = 207.711 = 14.41
 f
Next, We prepare a table to reflect x, f, d and fd, items ( b ) Pr (at least 50 kg) = Summation of 50kg freq upward to 79
Total freq summation
in the formula. where d = x – A( 43 )
130  88  47
=
Class Class mark x f d=x-A fd 900
interval  0.29
26 – 30 28 4 - 15 - 60
31 – 35 33 7 - 10 - 70 1998/13b (Nov) Exercise 18.1
36 – 40 38 13 -5 - 65 The table shows the distribution of marks obtained by 100
41 – 45 43 8 0 0
46 – 50 48 6 5 30
pupils in a test.
51 - 55 53 2 10 20 Marks 0-9 10-19 20-29 30-39 40-49 50-59 60-69 70-79 80-89
f = 40 fd = - 145 Freq 6 14 9 16 18 21 10 4 2
  145  (b) Using the assumed mean of 44.5, calculate, correct
Mean = 43 +  
 40  to 3 significant figures, the
= 43 – 3.625 ( i ) mean; ( ii ) standard deviation of the distribution
= 39.375
213
2004/14 Neco Solution
The distribution below shows the number of eggs laid Mean =  fx , Next, We prepare a table to reflect x, f and fx
by 20 fowls on daily bases within 30 days.  f
34 35 43 45 42 45 41 43 52 42 items in the formula.
56 53 60 50 59 60 53 54 35 47
Age (years) 15-24 25-34 35- 44 45-54 55-64
57 37 62 58 50 48 47 54 48 48 Class Mark (x) 19.5 29.5 39.5 49.5 59.5
Using the above given set of data No with 10 40 35 13 2 f1 = 100
Malaria
(a) construct a grouped frequency table using an f1x 195.0 1,180.0 1382.5 643.5 119.0 f1x = 3520.0
interval of 31 – 35, 36 – 40, 41 – 45 e.t.c No free of 10 20 35 30 5 f2 = 100
(b) From the table, using an assumed mean of 50 Malaria
f2x 195.0 590.0 1382.5 1485.0 297.5 f2x = 3950.0
correct to 2 places of decimal, calculate;
( i ) the mean, and;
( ii ) standard deviation of the data.  f1 x 3520
Mean age of women with malaria = = i.e. 35.20
(c) If a day is chosen at random, what is the probability  f1 100
that 20 fowls laid 60 eggs?
Solution  f2 x 3950
Mean age of women without malaria = = i.e.39.50
For grouped data  f2 100
2
fd fd  2
Mean = A +  f d and S.D =     f x  f x
 f f   f  Mean age of all the women =
1 2
strictly by mean rule
 f  f
1 2
Next, We prepare a table to reflect x, f, d, fd,d2 and fd2
items in the formula. where d = x – A( 50 ) 3520  3950
( a ) a grouped frequency table =
100 100
Class x Tally f d = x-A f d d2 fd2
7470
interval = = 37.35
31 - 35 33 III 3 –17 –51 289 864 200
36 - 40 38 I 1 –12 –12 144 144
41 - 45 43 IIII II 7 –7 –49 49 343
46 - 50 48 IIII II 7 –2 –14 4 28 1999 / 12 (Nov)
51 - 55 53 IIII 5 3 15 9 45 The table gives the distribution of males and females in a
56 - 60 58 IIII 7 8 64 64 448 village, classified by age, in years.
II
Age (in years) 0-9 10-19 20-29 30-49 50-59 60-69 70-79 80-89
f = fd = fd2 =
30 –55 1875 Males 36 47 56 55 35 30 15 10
Females 44 53 54 65 40 25 20 5
  55  (a) Using an assumed mean of 24.5, calculate, correct to
( b ) ( i ) Mean = 50 +  
 30  one decimal place, the mean of the distribution.
(b) If one person between ages 30 and 79 is selected at
= 50 – 1.8333
random to run an errand for the village, what is the
= 48.1667  48.17 to 2dp
probability that the person is:
2
(i) at most 59 years old;
1875   55  (ii) a male
( ii ) S.D =   Solution
30  30 
For grouped data, assumed mean formula;
= 62.5  3.3611 Mean = A +  f d For easy working, we will produce two tables
 f
= 59.1389 Males table
= 7.6902  7.70 to 2dp Class Class f d = x-A fd
interval mark x
No of 60eggs daily 0-9 4.5 36 - 19.6 -705.6
(c) Pr of 60 eggs = 10 - 19 14.5 47 - 10.0 -470.0
Total number of eggs
20 - 29 24.5 56 0 0
2 1 30 - 49 39.5 55 15.0 825.0
= i.e
30 15 50 - 59 54.5 35 30.0 1050.0
2004 / 13 a 60 - 69 64.5 30 40.0 1200.0
The table shows the distribution of the ages of women 70 - 79 74.5 15 50.0 750.0
in a village who had malaria during pregnancy and 80 - 89 84.5 10 60.0 600.0
those who did not. f = f =
Age (years) 15-24 25-34 35-44 45-54 55-64 284 3,249.4
No. with malaria 10 40 35 13 2
No. free of malaria 10 20 35 30 5

(a) Calculate the mean age of all the women.


214
Females table 2005 / 14
Class Class f d = x-A fd The table shows the distribution of rainfall in some locations,
interval mark x in a year.
0-9 4.5 44 - 19.6 -862.4 No. of
days of 0-9 10-19 20-29 30-39 40-49 50-59 60-69 70-79 80-89 90-99
10 - 19 14.5 53 - 10.0 -530.0 rainfall
No. of 3 5 15 18 28 14 7 5 3 2
20 - 29 24.5 54 0 0 locations
30 - 49 39.5 65 15.0 975.0
50 - 59 54.5 40 30.0 1200.0 (a) Using an assumed mean of 44.5, calculate, correct to
60 - 69 64.5 25 40.0 1000.0 one decimal places,
70 - 79 74.5 20 50.0 1000.0 ( i ) the mean
80 - 89 84.5 20 60.0 300.0 ( ii ) the standard deviation
f = f = (b) If one of the locations is selected at random, calculate
306 3082.6 the probability that it is from the modal class.
Solution

Distribution mean = A + 
  fd male   fd female 

For grouped data, assumed mean formula;
2
 f d2   f d 

 f male   female 
f Mean = A +  f d and S.D =   
 f f  f 
3249.4  3082.6 Next, We prepare a table to reflect x, f, d, fd,d2 and fd2 items
= 24.5 +
284  306 in the formula. where d = x – A( 44.5 )
Class Class f d = x–A fd d2 fd2
6332
= 24.5 + interval mark x
590 0-9 4.5 3 - 40 -120 1600 4800
10 - 19 14.5 5 - 30 -150 900 4500
= 24.5 + 10.7322 20 - 29 24.5 15 - 20 - 300 400 6000
= 35.2322  35.2 to one dp. 30 - 39 34.5 18 - 10 - 180 100 1800
40 - 49 44.5 28 0 0 0 0
(b) Between 30 and 79 table is displayed as: 50 - 59 54.5 14 10 140 100 1400
Age 30-49 50-59 60-69 70-79 60 - 69 64.5 7 20 140 400 2800
70 - 79 74.5 5 30 150 900 4500
(in years) 80 - 89 84.5 3 40 120 1600 4800
Males 55 35 30 15 90 - 99 94.5 2 50 100 2500 5000
Females 65 40 25 20 f = f = fd2 =
100 – 100 35,600
( i ) Pr( at most 59 years ) = Either a male or female
= 55 + 35 + 65 + 40   100 
284 306 a (i) Mean = 44.5 +  
= 0.3169 + 0.3431  100 
= 0.66 = 44.5 – 1
( ii ) Pr(Male) = A male between 30 and 79 = 43.5 to one decimal place
= 55 + 35 + 30 + 15 2
284 35600   100 
(ii) S.D =  
= 135 100  100 
284
= 0.475 = 356  1
2006/6 Exercise 18.2 = 355
The table shows the distribution of the ages of a group
of people in a village = 18. 84  18.8 to 1 dp.
Ages
(in years)
15-18 19-22 23-26 27-30 31-34 35-36 ( b ) Pr (modal class) = modal class freq
Frequency 40 33 25 10 8 4 Total freq summation
28
Using an assumed mean of 24.5, calculate the mean of =
the distribution. 100
= 0.28
2003/14a Exercise 18.3 1992/24 Exercise 18.4
The table below shows the distribution of mass of Marks 0-2 3-5 6-8 9-11 12-14 15-17
women in a survey. Number 5 6 4 3 1 1
Mass(kg) 40-49 50-54 55-59 60-64 65-69 70-74 75-84 of Students

No. of 2 5 5 8 10 12 8 Find the mean mark


women 4 13 1 1 2
A. 5 B. 7 C. 8 D. 19 E. 22
(a) Using an assumed mean of 67, calculate the 5 20 2 3 3
mean mass of the women.
215
VTR – 12 / 4 a & b Exercise 18.5 OM (ii) We find the class interval where the median lies,
The following are the prices in Naira of 50 articles in a shop. with the aid of the cumulative frequency 20 lies in
21 35 52 70 55 42 9 48 57 36 the cf after 15 i.e class interval 30 – 39
46 15 35 12 29 61 48 22 43 58 L1 = 29.5, Cfb = 15, C = 10, i.e 30 minus 20 or 20 minus 10
25 42 1 60 44 38 54 47 69 30 F median = 12
47 18 35 32 21 50 11 34 41 50
53 30 54 47 34 48 60 45 16 33 Median = 29.5 + 20 - 15 x 10
12
( a ) Make a frequency distribution table using intervals = 29.5 + 5 x 10
1- 10 , 11 – 20 , 21 – 30 and so on. 12
( b ) Find the mean and standard deviation of these prices = 29.5 + 4.17 = 33.67
1997 / 30
2002/26 Exercise 18.6 Calculate the median of the distribution;
The table below shows the distribution of ages in
years of a group of 25 men Class 60-64 65-69 70-74 75-79
Frequency 4 10 12 12
Age in 19-21 22-24 25-27 28-30
years 80-84 85-89 90-94
Frequency 4 10 8 3 8 6 2
A 75.42 B 75.35 C 75.00 D 74.92 E 74.50
Calculate, correct to the nearest year, the mean age of
Solution
the men. For grouped data
A. 22 B. 23 C. 24 D. 25
n 
  cf b 
Median Median = L1 +  2  c
Median formula for grouped data is given as  fm 
n
/2 – Cfb x C  
Median = L1 +  
fm n 54
Where First, we find = = 27 median position
2 2
L1 = Lower class boundary of the median class Secondly, Adding Cumulatively, 27 position is in 75 – 79
n = Total frequency
Here, L1 = 74.5, Cfb = 26, C = 79.5 – 74.5 i.e 5, Fm = 12
Cfb = Cumulative frequency before the median class
Fm = Frequency of median class  27  26 
Thus, median = 74.5 +    5
C = Size of the median class  12 
= 74.5 + 0.4166
1996/3c OM  74.92 (D)
The table below shows the marks obtained by forty
pupils in a mathematics test 1998/13a (Nov) Exercise 18.7
The table shows the distribution of marks obtained by 100
Marks 0-9 10-19 20-29 30-39
pupils in a test.
Number of pupils 4 5 6 12
Marks 0-9 10-19 20-29 30-39 40-49
40 – 49 50 –59 Frequency 6 14 9 16 18
8 5
(C). Calculate the median of the distribution 50-59 60-69 70-79 80-89
Solution 21 10 4 2
Median table (a) Estimate the median of the distribution
Marks Class Boundaries f cf
2002/27 Exercise 18.8
0– 9 0 – 9.5 4 4
The table below shows the distribution of ages in years
10 – 19 9.5 – 19. 5 5 9
20 – 29 19.5 – 29. 5 6 15
of a group of 25 men
30 – 39 29.5 – 39. 5 12 27 Age in 19-21 22-24 25-27 28-30
40 – 49 39.5 – 49. 5 8 35 years
50 – 59 49.5 – 59. 5 5 40 Frequency 4 10 8 3
n Find, in years, the median of the distribution
Median = L1 + /2 – Cf b C
fm A. 21.50 B. 22.95 C. 22.85 D. 23.45
n
(i) First, find /2 = 40
2
= 20 median position
216
Mode  13 
Mode formula for grouped data is given as Mode = 39.5 +    10
 13 11 
Mode = L1 + △1 x C 13
= 39.5 +  10
△1 + △2 24
Where : L1 = lower class boundary of the modal class = 39.5 + 5.417 = 44.917  44.9 (B)
△1 = Difference between the modal frequency 1986/12 GCE Exercise 18.9 OM
and the frequency of the next lower class i.e class The marks scored by fifty students in an examination paper
before it . are given below :
△2 = Difference between the modal frequency and the 30 45 48 55 39 25 31 12 18 21
frequency of the next higher class i.e class after it. 54 59 51 33 43 44 10 38 19 26
41 35 37 41 46 33 51 37 58 48
C = Size of the modal class 17 19 23 26 29 38 57 36 35 44
43 27 31 43 22 31 47 34 18 15
1992/12c OM Prepare a frequency table with the class intervals
The table below shows the weekly profit in Naira from 10 – 19, 20 – 29, 30 – 39 , etc.
a mini-market. Using the mid-values, calculate the median and mode of the
Weekly 1-10 11 –20 21-30 31- 40 41-50 51-60 distributions
profit
Freq. 6 6 12 11 10 5 2000/26 – 28 Exercise 18.10
The table gives the distribution of marks obtained by 50
(c) What is the modal weekly profit candidates in a test.
Solution Marks 30-39 40-49 50-59 60-69 70-79
Modal table Frequency 1 8 15 19 7
Weekly profit Class boundaries f
1 – 10 0.5 – 10.5 6 Use the information to answer Questions 26 – 28.
11 – 20 10.5 – 20.5 6
26. What is the lower class boundary of the modal class?
21 – 30 20.5 – 30.5 12
A 49.5 B 59.5 C 60.5 D 64.5
31 – 40 30.5 – 40.5 11
41 – 40 40.5 – 50.5 10 27. If 52% of the candidates are successful, What is the
51 – 60 50.5 – 60.5 5 minimum pass mark?
A.60 B.61 C.67 D.68
(i ) Identify the modal class i.e class with highest
frequency which is 21 – 30 28. What is the class mark of the median class?
A 49.5 B 55.0 C 59.5 D 64.5
Mode = L1 + △1 xC
2004/17–18 Exercise 18.11
△1 + △2 The table below shows the distribution of marks of 20
L1 = 20.5, △1 = 12–6 = 6 ,△2 = 12–11 = 1 and C = 10 students in a test.
Mode = 20.5 + 6 x 10 Marks 0-19 20-39 40-59 60-79 80-89
6+1 Frequency 2 6 8 3 1

= 20.5 + 6 x 10 = N 29.07 Use this information to answer Questions 17 and 18


7
17. What is the upper class boundary of the median class?
2004 /44 Neco
A 39.5 B 44.5 C 59.5 D 60.5
Calculate the mode in the distribution of marks of
students shown below: 18. If a student is selected at random from the group, what
Mark % 20-29 30-39 40-49 50-59 60-69 70-79 is the probability that he obtained at least 39.5?
No. of 4 7 20 9 6 4 3 3 2 3
Student
A B C D
10 20 5 5
A 49.0 B 44.9 C 44.5 D 40.0 E 39.5
2001/37 Neco Exercise 18.12
Solution In the table below, L is the true lower class limit of the 2nd
The modal class here is 40 – 49 with frequency 20
class and P is the true upper class boundary of the 5th class.
   2  What is L – P?
Mode = L1 +  1  c
 1  Class interval 11-15 16-20 21-25 26-30 31-35 36-40
C = 49.5 – 39.5, L1 is 39.5, 1 = 20 – 7  2 =20 – 9 Freq 8 3 5 6 7 1

=13 = 11 A 49.0 B 44.9 C 44.5 D 40.0 E 39.5

217
Chapter Nineteen Solution
Graphs (Charts) Total number of periods = 36
Subject s Periods Sector (degrees)
Statistical graphs (Charts)
English 10 10 x 360 = 1000
BAR CHART 36
A bar chart is a chart in which rectangular shapes are Mathematics 7 7 x 360 = 700
used to represent frequency. The rectangles have equal 36
width and unequal length. The lengths depend on the Biology 3 3 x 360 = 300
frequency of each class. The distinct bars are separated 36
by a uniform gap all through. Students choose the scale Statistics 4 4 x 360 = 400
used but it must be reasonable. 36
Ibo 3 3 x 360 = 300
1988/1(i) OM 36
The number of items produced by a company over a Others 9 9 x 360 = 900
five years period is given below: 36
We then proceed to draw the required diagram with the aid of
Year 1978 1979 1980 1981 1982 a protractor as show below:
Number
produced 4100 2500 1500 1800 9200
(i) Plot a bar chart from this information.
Solution Others

English

10000
Ibo
9000
Mathematics
8000 Statistics

Biology

7000

6000
Note: The total angles of all sector in a given circle is 3600
5000

4000
1997/45 OM
The pie chart represents the fruits on display in grocery shop. If
3000 there are 60 oranges on display, how many apples are there?
A. 40 B.80 C. 90 D.120 E.270
2000

1000
Bannana
Orange

1978 1979 1980 1981 1982

1989/2 Exercise 19.0 OM 6


The table shows the weights, to the nearest kilogram of Pawpaw 1
twelve students in a further mathematics class
1
Apple

Weight in kg 55 57 59 61 63
Number of Students 2 1 2 4 3
Draw a bar chart to illustrate the above information. Solution
The only sector that has useful information is that of Oranges.
Numbers of orange = 60
Sector angle of orange = 360 – (60 + 100 + 120 )
PIE CHART = 360 – 280 i.e 800
A pie chart is a cyclic diagram in which each sector of But, 60 x 360 = 800
the circle represents a given frequency expressed in Total fruits
degrees. Total fruits = 60 x 360 = 270
80
1988/4 OM
Let the number of apples be x, then.
In a certain school, the lesson periods for each week are
x × 360 = 120
as itemized below: English 10, Mathematics 7, Biology
270
3, Statistics 4, Ibo 3, others 9.
Draw a pie chart to illustrate this information. x = 120 x 270 = 90 (C)
360
218
1993/42 Exercise 19.1 OM Solution
The distribution by state of 840 students in the faculty To draw histogram, we need class boundaries column
of Science of a Nigerian University in a certain session Marks Class boundaries Frequency
is as follows: 10 – 29 9.5 – 29.5 2
Bendel 45 30 – 39 29.5 – 39.5 8
Kwara 410 40 – 49 39.5 – 49.5 14
Ogun 105 50 – 59 49.5 – 59.5 14
Ondo 126 60 – 69 59.5 – 69.5 28
Oyo 154 70 – 79 69.5 – 79.5 10
In a pie chart draw to represent this distribution, the 80 – 89 79.5 – 89.5 4
angle subtended by Ondo is?
A 360 B. 420 C. 450 D. 480 E 540 Graph is shown on page 221
From the graph, modal class = 59.5 + 4
1995/23 (Nov) Exercise 19.2 = 63.5
Which one of the following is true about the 1994 / 5 (Nov) Unequal class width case
construction of a pie chart? The angle The table below gives the distribution of distances in Kn of
A in a sector is proportional to the total frequency 60 villages from a state capital;
B in a sector is proportional to the class width Distances 0-19 20-29 30-39 40-49 50-69 70-99 100-149
C in a sector varies inversely as the corresponding Number of 12 7 6 8 5 9 10
class frequency. Villages
D at the centre of the circle is proportional to the total
frequency Construct a histogram for the distribution.
E at the centre of the circle is equal to the class Solution
midpoint. To construct a histogram, we need class boundaries column
Distances Class Frequency Class size
boundaries
Histogram 0 – 19 0 – 19.5 12 10
Histogram is almost the equivalent of bar chart. It is used 20 – 29 19.5 – 29.5 7 10
for frequency distributions. Unlike the bar chart, 30 – 39 29.5 – 39.5 6 10
histogram bars have no spaces between them. We use 40 – 49 39.5 – 49.5 8 10
class boundaries to plot against the frequencies. Here 50 – 69 49.5 – 69.5 5 20
Histogram will be used to estimate mode and median 70 – 99 69.5 – 99.5 9 30
100 – 149 99.5 – 149.5 10 40
2003 / 6 (Nov)
A scientist measured the length, x mm of the tails of We observe that the three last boundaries are not equal to the
100 small replies of the same species. The table below first four boundaries as indicated by the class size (class
is the summary of the results. width) hence their rectangles width will reflect same
Graph is shown on page 221
Tail length (x mm) Frequency
75.0 < x  77.0 8
2000 / 6 a and c
76.0 < x  76.5 12 The table shows the distribution of the ages (in years)
76.5 < x  77.0 28 of a group of people.
77.0 < x  77.5 22 Ages 25-27 28-30 31-33 34-36 37-39 40-42
(in years)
77.5 < x  78.0 12 Frequency 2 9 15 19 10 5
78.0 < x  79.0 10
(a) Draw a histogram of the distribution
79.0 < x  80.0 8 (c) Estimate the median, expressing your answer correct to 1
decimal place
Graph is shown on page 221 Solution
To draw a histogram, we need to show class boundaries column
Ages (yrs) Class Frequency
1996 / 6 (Nov)
boundaries
The table gives the distribution of marks obtained in a
25 – 27 24.5 – 27.5 2
test by 80 students.
28 – 30 27.5 – 30.5 9
Mark 10-29 30-39 40-49 50-59 50-69 70-79 80-89 31 – 33 30.5 – 33.5 15
%
34 – 36 33.5 – 36.5 19
No. of 2 8 14 14 28 10 4
Students 37 – 39 36.5 – 39.5 10
40 – 42 39.5 – 42.5 5
(a) Draw the histogram of the distribution
(b) Use your histogram to estimate the modal mark. Graph is shown on page 222 Next graph analysis
219
( c ) Median from the graph Next, we are to move about 8 little boxes in the median
f = 60 i.e 30 10 .5
rectangle to get 10.5 i.e ( 8)
2 2 1 .3
The cumulative frequency shows 2+9 = 11, 11+15 = 26 Reading our value 19.5 + 8 = 27.5
and 26 +19 = 45 our 30 falls in here
Thus the median class is 34 – 36 2010/6
Cumul freq(Cfb) before the median class is 26(2+9+15) The table shows the marks obtained by a group of students in
n a class test.
Next, – Cfb = 30 – 26 i.e 4
2 Marks 40-44 45-49 50-54 55-59 60-64 65-69
No. of 4 9 18 23 10 6
Note that Cf at Median class –
f = 45 – 30 i.e 15 students
2 (a) Draw a histogram for the distribution.
Hence we share the rectangle of the median class into (b) Use your histogram to estimate the median of the distribution.
ratio 4 : 15 ( this is the median class frequency 19) Solution
To achieve this , Class. frequency i e 19 = 1.9 To draw a histogram, we need to show class boundaries column
X – axis scale 10 Marks Class boundaries Frequency
Next, we to move about 2 little boxes in the median 40 – 44 39.5 – 44.5 4
4 45 – 49 44.5 – 49.5 9
rectangle to get 4 i.e ( 2 )
1 .9 50 – 54 49.5 – 54.5 18
Reading our value 33.5 + (0.3 + 0.3)  34.1 to 1 dp 55 – 59 54.5 – 59.5 23
60 – 64 59.5 – 64.5 10
2005 / 6 65 – 69 64.5 – 69.5 6
The table shows the distribution of marks scored by Graph is shown on page 223 Next graph analysis
some candidates in an examination.
( c ) Median from the graph
Marks 0-9 10-19 20-29 30-39 40-59 f = 70 i.e 35
No. of candidates
3 7 13 8 10 2 2
The cumulative frequency shows 4+9=13 and 13+18=31 Then
(a) Draw a histogram of the distribution 31+23=54 our 35 falls in here
(b) Calculate, correct to the nearest whole number, the Thus the median class is 55 – 59
median mark. Cumul freq(Cfb) before the median class is 31(4+9+18)
Solution n
To draw a Histogram, we need class boundaries column, Next, – Cfb = 35 – 31 i.e 4
Marks Class Boundaries Freq Class size 2
0 – 9
10 – 19
0 – 9.5
9.5 – 19.5
3
7
10
Note that Cf at Median class –
f = 54 – 35 i.e 19
10
20 – 29 19.5 – 29.5 13 10 2
30 – 39 29.5 – 39.5 8 10 Hence we share the rectangle of the median class into
40 – 49 39.5 – 59.5 10 20 ratio 4 : 19 ( this is the median class frequency 23)
To achieve this , Class frequency i e 23 = 2.3
Since the last class boundary has times two of the other
X – axis scale 10
size, we reflect it in the graph.
Next, we are to move about 2 little boxes in the median
Graph is shown on page 222 Next graph analysis
rectangle to get 4.6 But we need 4 Here we apply common
( c ) Median from the graph sense to read Median = 55marks instead of 55.5
f = 41 i.e 20.5
2 2 2009/7 (Nov) Exercise 19.3 OM
The cumulative frequency shows 3+7=10 and 10+13=23 our The marks scored by 50 students in a Geography
20.5 falls in here examination are as follows:
Thus the median class is 20 – 29
Cumul freq(Cfb) before the median class is 10(3+7) 60 54 40 67 53 73 37 55 62 43
n 44 69 39 32 45 58 48 67 39 51
Next, – Cfb = 20.5 – 10 i.e 10.5 46 59 40 52 61 48 23 60 59 47
2
65 58 74 47 40 59 68 51 50 50
Note that Cf at Median class –
f = 23 – 20.5 i.e 2.5
71 51 26 36 38 70 46 40 51 42
2 (a) Using a class interval of 21 – 30, 31 – 40, …, prepare a
Hence we share the rectangle of the median class into frequency distribution table
ratio 10.5 : 2.5 ( this is the median class frequency 13) (b) Draw a histogram to represent the distribution.
To achieve this , Class frequency i e 13 = 1.3 (c) Use your histogram to estimate the modal mark
X – axis scale 10 (c) If a student is selected at random, find the probability that
he/she obtains a mark greater than 63
220
Number of villages F re qu ency

5
10
15
30

20
25

10

0
5
75.0

19.5 76.0
2003/6(Nov)

76.5
29.5
77.0
39.5
77.5

49.5 78.0

1994/5(Nov)
79.0

80.0
69.5

Length (x mm)

221
No. Of students

5
10
15
25
30

20

9 .5
99.5
2 9 .5

3 9 .5
1996/6(Nov)

4 9 .5

5 9 .5

6 9 .5

149.5 7 9 .5

8 9 .5
Marks

Distances
Frequency
No. Of Candidates

20

5
15

10

6
8

2
4
14

10
12
2000/6

2005/6
24.5
9.5

27.5
19.5

10.5
M edian 30.5

2.5
29.5

222
33.5
4

39.5 Median
15

36.5

3 9.5
59.5

Marks
Ages(yrs)

42.5
No. Of students

5
15
20
25

10
2010/6

39.5

44.5

49.5

223
54.5
4

Median
19

59.5

64.5
Marks

69.5
2008/6 (Adjusted) Exercise 19.4 Frequency polygon
The table shows the distribution of marks obtained by Frequency polygon is a graph gotten by plotting class mark
some candidates in a test. (mid point) against its frequency. After the usual plotting of
Marks 10-14 15-24 25-29 30-39 40-44 45-59 histogram, we then join the mid point of each rectangle to
No. of 14 30 22 18 12 4 each other by a straight line.
candidates
1996/3 OM
(a) Draw a histogram for the distribution. The table below shows the marks obtained by forty pupils in
(b) Use your graph to estimate correct to the nearest a mathematics test.
whole number, the median mark.
Marks 0–9 10 – 19 20 – 29
Number of pupils 4 5 6
1988/46-47
The histogram below shows the number of candidates, 30 - 39 40 - 49 50 –59
in thousands, obtaining given ranges of marks in a state 10 8 5
Examination. Use the graph to answer Question 46 and 47.
(a) Draw a histogram for the mark distribution.
100
(b) Draw the frequency polygon on the same graph
90 Solution
80 We shall solve the problem by preparing a table for the graph:
Marks Class Boundaries frequency
70
0 –9 0 – 9.5 4
60 10 – 19 9.5 – 19.5 5
20 – 29 19.5 – 29.5 6
50
30 – 39 29.5 – 39.5 10
40 40 – 49 39.5 – 49.5 8
30 50 – 59 49.5 – 59.5 5
20
( a ) The required diagram is a graph showing class
10 boundaries plotted against frequency as shown below.
0
0-10 10-20 20-30 30-40 40-50 50-60 60-70 70-80 80-90 90-100 10
No of Pupils

46.Find the total number of candidates that sat for


examination. 8
A80, 000 B120,000 C250,000 D260,000 E270,000
Solution
Total number of candidates = sum of each bar’s Length (freq) 6
Bar Position Range% Freq.(thousands)
1st Bar 0 – 10 5
2nd Bar 10 – 20 10 4
3rd Bar 20 – 30 20
4th Bar 30 – 40 60
5th Bar 40 – 50 80 2
6th Bar 50 – 60 50
7th Bar 60 – 70 30
8th Bar 70 – 80 5 0
29.5

39.5

49.5
19.5

59.5

80 – 90
9.5

9th Bar 5 Marks


10th Bar 90 – 100 5
f = 270 1995/21 (Nov) Exercise 19.5
The curve obtained by joining the mid points of the top of the
Total No of candidates in Thousands = 270,000 (E) bars of a histogram is called
A relative frequency polygon B ogive C frequency polygon
47. How many candidates scored at most 30% D relative frequency curve E normal curve
A. 15,000 B.20,000. C. 25,000
D. 35,000 E.60,000 1994/21 (Nov) Exercise 19.6
Solution A frequency polygon is a line graph of the class
At most 30% = 0 – 10 + 10 – 20 + 20 – 30 A frequency against class boundaries
= 5 + 10 + 20 B relative frequency against the class mode.
= 35 thousand (D) C midpoints against their medians.
D frequency against assumed mean.
E frequency against the class midpoint.
224
Cumulative Frequency Curve (O - give)
Cumulative frequency curve or O–give is a statistical 2006/14
graph gotten by plotting the upper class boundaries
against cumulative frequencies. It is used to determine
among others:

Cumulative frequency
80
 Median
 Percentiles (100 divisions) 70
 Deciles (10 divisions) 60
 Quartiles (4 divisions)
Median in O - give 50

The median is simply half of the cumulative frequency


40
traced to the curve then to the class boundaries to get
the required value. Also the 50th percentile is a measure 30
of median likewise the 5th Deciles (D5) and 2nd Quartile (Q2)
20
Percentile
This is the division of the cumulative frequency into 10
100 points. For instance
75% = 75 x cum. Freq 29.5 39.5 49.5 59.5 69.5 79.5
100 Time(hours)

20% = 20 x cum freq


100 (b) ( i ) median is simply half of the cumulative frequency
Then we trace the required values in the graph as discussed in median traced to the curve then to the class boundary to get
Deciles the required value
This is the division of the cum. Freq into 10 portions. 1
of 80 = 40. Reading from graph is 47.5+0.8 = 48.3
For instance, D1 = 1 x cum. Freq 2
10 40
(ii) 40th percentile =  80
D5 = 5 x cum. Freq 100
10 = 32 at cum freq traced to the curve then to the class boundaries
Quartiles Reading from graph is 45.5+0.6 = 46.1
This is the division of the cumulative frequency into 4 portions. Where:
(iii) We traced 50.5 (50 hours 30 minute) from class
(Lower quartiles ) Q1 = ¼ of cum freq,
boundary to the cumulative frequency and subtract the
(Middle quartile) Q2 = 2/4 of cum freq lower part from upper. Reading from graph
(Upper quartile) Q3 = ¾ of cum. freq. No. of Employees that spent at least 50.5hrs
Inter quartile range = Q3 – Q1 = 80 – 44 i.e 36
Semi – inter quartile range = Q3 – Q1 2003 / 14 (Nov)
2 The following table shows the distribution of nurses in some
2006 / 14 hospitals in a state.
The table below the distribution of hours spent at work No. of 0-9 10-19 20-29 30-39 40-49
by the employees of a factory in a week. Nurses
Time 20-29 30-39 40-49 50-59 60-69 70-79 No. of 2 7 9 11 13
(hours) Hospitals
No. of 8 11 23 25 8 5
persons
50-59 60-69 70-79 80-89 90-99
(a) Draw an O–give for the distribution
(b) Using your graph, estimate the 14 18 15 8 3
(i) median (ii) lower quartile (iii) 40 percentile
(iv) number of employees that spent at least 50
hours 30 minutes (a) Construct a cumulative frequency table for the
Solution distribution
O–give is the plotting of upper class boundaries against (b) Use your table to draw a cumulative frequency curve
cumulative frequencies, so we prepare a table to show it (c) From your curve, estimate the number of hospitals
having at least 55 nurses
Time (hours) Class boundaries Freq Cum. Freq
(d) If an hospital is estimate at random, what is the
20 – 29 19.5 – 29.5 8 8 probability that:
30 – 39 29.5 – 39.5 11 8 + 11 = 19 (i) it has between 40 and 49 nurses
40 – 49 39.5 – 49.5 23 19 + 23 = 42 (ii) it has less than 35 nurses?
50 – 59 49.5 – 59.5 25 42 + 25 = 67
Solution
60 – 69 59.5 – 69.5 8 67 + 8 = 75 (a) Cumulative frequency table contains class boundaries
70 – 79 69.5 – 79.5 5 75 + 5 = 80 and cumulative frequency as shown next
225
No. of Nurse Class boundaries Freq Cum. Freq Solution
0– 9 0 – 9.5 2 2 Mean =
 fx , Next, We prepare a table to reflect x, f and fx items
10 – 19 9.5 – 19.5 7 2+ 7= 9  f
20 – 29 19.5 – 29.5 9 9 + 9 = 18 Age (years) 15-24 25-34 35- 44 45-54 55-64
30 – 39 29.5 – 39.5 11 18 + 11= 29 Class Mark (x) 19.5 29.5 39.5 49.5 59.5
40 – 49 39.5 – 49.5 13 29 + 13= 42 No with
Malaria
10 40 35 13 2 f1 = 100
50 – 59 49.5 – 59.5 14 42 + 14= 56 f1x 195.0 1,180.0 1382.5 643.5 119.0 f1x = 3520.0
60 – 69 59.5 – 69.5 18 56 + 18= 74 No free of 10 20 35 30 5 f2 = 100
Malaria
70 – 79 69.5 – 79.5 15 74 + 15= 89 f2x 195.0 590.0 1382.5 1485.0 297.5 f2x = 3950.0
80 – 89 79.5 – 89.5 8 89 + 8= 97
90 – 99 89.5 – 99.5 3 97 + 3=100 Mean age of all the women =
 f x  f x
1 2
strictly by mean rule
 f  f
1 2
100 2003/14 3520  3950
=
100 100
Cumulative frequency

80 7470
= 37.35
=
200
(b) Cumulative frequency curve is the plotting of
60 cumulative frequency against upper class boundaries
55 Nurses with Malaria table
40
49 Nurses Age Class boundaries Freq Cumul. Freq.
15 – 24 14.5 – 24.5 10 10
40 Nurses
25 – 34 24.5 – 34.5 40 10 + 40 = 50
35 Nurses
35 – 44 34.5 – 44.5 35 50 + 35 = 85
20
45 – 54 44.5 – 54.5 13 85 + 13 = 98
55 – 64 54.5 – 64.5 2 98 + 2 = 100
0 Malaria free table
9.5 19.5 29.5 39.5 49.5 59.5 69.5 79.5 89.5 99.5
No. Of Nurses
Age Class boundaries Freq Cumul. Freq.
(c) From 49. 5 to 59.5 is ten marks, so we share the 10 15 – 24 14.5 – 24.5 10 10
among the little boxes. Trace 55 to the curve then 25 – 34 24.5 – 34.5 20 10 + 20 = 30
to the cumulative frequency. Graph Reading is 48. 35 – 44 34.5 – 44.5 35 30 + 35 = 65
But at least 55 nurses implies highest cumulative minus 48 45 – 54 44.5 – 54.5 30 65 + 30 = 95
= 100 – 48 55 – 64 54.5 – 64.5 5 95 + 5 = 100
= 52 Hospitals
(d) (i) Trace 40 Nurses to the curve then 100
to the cumulative frequency and do same for 49
r ia

40  49
ala

Pr (40 and 49) =


M

100
ith

ree

80
W

aF

= 0.11
i
lar

24  0
Ma

(ii) Pr (less than 35) =


100
60
= 0.24
2004/13
The table shows the distribution of the ages of women
in a village who had malaria during pregnancy and 40
those who did not.
Age (years) 15-24 25-34 35-44 45-54 55-64
No. with malaria 10 40 35 13 2
No. free of malaria 10 20 35 30 5 20
(a) Calculate the mean age of all the women.
(b) On the same graph sheet, draw separately, the
cumulative frequency curves of women:
24.5 34.5 44.5 54.5 64.5
(i) who had malaria;
(c) (i) Mean 37.35 yrs traced to the curve of women with
(ii) without malaria.
malaria then to cumulative frequency = 100 – 62.5
(c) Use your curves to determine the number of women
= 37.5 38women
above the mean age, who: (c) (i) Mean 37.35 yrs traced to the curve of women
(i) had malaria; malaria free then to cumulative frequency = 100 – 38 = 62
(ii) did not have malaria.
226
2002 / 14 No of 0-25 26-50 51-75 76-100 101-125 126-150 151-175 176-200
Below is the cumulative frequency table of the days
Cum. Freq 5 21 40 60 80 91 98 100
life–span of 100 rabbits in a controlled environment. Frequency 5 21-5 40-21 60-40 80-60 91-80 98-91 100-98
Life Span =16 =19 =20 =20 =11 =7 =2
22 50 75 100 125 150 175 200
in Days (d) Find the probability that a rabbit choosen at random in
Cumulative 5 21 40 60 80 91 98 100 the environment will live beyond 125 days.
No.of Rabbits
11  7  2
(a) Draw a cumulative frequency curve of the distribution. Pro (Beyond 125 days) =
100
(b) Use your curve to estimate:
20
(i) the curve to estimate: = = 0.2
(ii) the number of rabbits still alive after 130 days 100
Solution 2002 / 30
Here we plot the o-give directly from the given table It is desired that 60% of the candidates who took a test
since it has cumulative frequency already should pass. Which of the following percentile marks should
be made a pass mark?
100 A 20th B 30th C 40th D 60th
2002/14
Solution
Cumulative frequency

Those to pass are at the upper scores portion


80 Q3 100  60
=  100 (percentile)
100
40
=  100 = 40th percentile (C)
60 100
2000 / 25 – 27 (Nov)
The table shows the distribution of marks obtained by a
group of pupils in a test.
40
Marks 1-9 10-18 19-27 28-36 37- 45
Q1 Boys 3 10 9 5 5
Girls 1 5 10 2 0
20
Use this information to answer Questions 25 – 27
25. If a pupil is randomly selected from the class, what is the
probability that the pupil scored at least 28 marks
25 50 75 100 125 150 175 200 Days A 0.14 B 0.24 C 0.76 D 0.90
Q3  Q1 Solution
b (i) Semi – inter quartile range = The whole data is about one class. Thus,
2
3 55 2 0
But Q3 = of 100 = 75 At the graph we trace 75 Pr (at least 28 marks) =
4 50
at cumulative freq. to the curve then to x – axis (days) 12
= i.e. 0.24 ( C )
Q3 = 120 50
1 26. What is the median score for the girls?
and Q1 = of 100 = 25 At the graph we trace 25 at A 19.0 B 21.2 C 21.4 D 23.0
4
cumulative frequency to the curve then to x–axis (days) Solution
Q1 = 55 18
f for the girls is 18 i.e. even. Thus median class = = 9th class
120  55 2
Thus, semi – inter quartile range = = 32.5 days
2 Adding the frequency cumulatively; 9 occur at 19 – 27 with
b (ii) Trace 130 days to the curve, then to the 19  27
class mark as = 23 (D)
cumulative frequency i.e. 82 After here means 2
upper cumulative 100 minus 82 =18 rabbits 27. If 19 boys passed the test, what was the pass mark?
A 27.5 B 23.0 C 18.5 D 18.0
(c) Using the cumulative frequency table, copy and Solution
complete table below: Pass is at the upper level. Thus taking the frequency
No. of 0-25 26-50 51-75 76-100 101-125 126-150151-175 176-200 cumulatively from the upper side (Highest score); 19(5+5+9)
Days is gotten at the class 19 – 27 with class mark 23 (B)
No. of 5 7
Rabbits

Solution
Since cumulative frequency is simple addition from the
first frequency to the second frequency so on, we
reverse it by subtraction as
227
2004 /46 Neco (Dec) 1996/21 Exercise 19.10
What is the upper quartile of the distribution, if the A graph obtained by plotting cumulative frequencies against
semi–inter quartile range is 24 and the lower quartile is 18? their upper class boundaries is called
A 66 B 48 C 42 D 24 E6 A a Frequency polygon B a percentile curve
Solution C a histogram D o-give E a range chart
Q3  Q1 2008/14 Exercise 19.11
Semi – inter quartile range =
2 The following table shows the distribution of marks obtained
Q  18 by some students in an examination.
24 = 3
2 Marks 0-9 10-19 20-29 30-39 40-49
24 2 = Q3 – 18 No. of 50 50 40 60 100
48 = Q3 – 18 Students
Q 3 = 48 + 18
50-59 60-69 70-79 80-89 90-99
= 66 (A)
100 50 25 15 10
2000 / 49 Neco
If 24 students sit for an examination and 18 students (a) Construct a cumulative frequency table for the
are to be promoted to the next class, which of the distribution.
following should be made the least pass mark? (b) Draw an ogive for the distribution.
A lower quartile mark B median mark (c) Use your graph in (b) to determine the;
C Upper quartile mark D the 18th percentile mark (i) semi-inter quartile range;
E the 24th percentile mark (ii) number of students who failed, if the pass
Solution mark for the examination is 37;
18 students to be promoted will be the upper portion (iii) probability that a student selected at random
(highest part), and what will be left of 24 is 6 scored between 20% and 60%.
i.e. 24 – 18 = 6 2008/13 Exercise 19.12
6 1 The table gives the distribution of marks of 60 candidates in a test.
it follows that = (Lower quartile) A.
24 4 Marks 23-25 26-28 29-31 32-34 35-37 38-40
Alternatively by mere observation from 24 – 18 = 6 Freq 3 7 15 21 10 4
666 6 (a) Draw a cumulative frequency curve of the distribution.
= 24 (b) From your curve, estimate the
18 last six
(i) 80th percentile; (ii) median; (iii) semi-interquartile.
1
We can see clearly that it is the last Six, which is 2001/ 9 Neco Exercise 19.13 OM
4
i.e. lower quartile. A The following in a data represents the scores of students in a
Mathematics mock examination:
1999/12 (Nov) Exercise 19.7 90 40 60 70 46 53 40 60 91 70
If n values of a distribution are arranged in ascending 55 35 24 63 66 20 55 25 45 78
 n  1  th 66 25 38 68 78 30 38 82 63 55
order of magnitude, the   value is the 78 82 51 59 80 45 51 66 60 50
 4 
A median B mean C lower quartile D upper quartile (a) Prepare a grouped cumulative frequency table with
` class intervals 20 –29, 30 –39 40 –49 and so on.
2004/19 Exercise 19.8 (b) Draw a cumulative frequency curve for the distribution
The cumulative frequency curve can be used to estimate the, (c) Use your curve to estimate the:
A mean of a distribution i. Fail mark if 35% of the students failed the examination;
B relationship between two variables ii. Interquartile range.
C median and percentiles of a distribution
D variance of a distribution

2003 / 8 Neco Exercise 19.9 Chapter Twenty


The items of a distribution are in numerical order; the Plane mensuration
upper quartile is the value of
Area & Perimeters Of plane Shapes
 n  1  th Area and perimeter are terms used in mensuration. Students
A the middle item B  item
 2  tend to confuse the two terms thus, interchanging the formulae
to be used.
3n  1 th n  1 th To give a clear difference between them, let us consider an
C item D item
4 4 example of a football field that is rectangular in shape. The 11
 n  th players of both teams play within the pitch area while the
E  item. referee officiates within the pitch (area)
2
228
L = (2x + 5 )m and b = (x + 1) m
Players andreferee = (2 x 4 + 5)m = (4 + 1 )m
= 13 =5
Thus, Area of a rectangle = L x b
= 13 x 5
= 65m2 (B)

2004/8b NABTEB OM
As for the assistant referee (linesmen ) they work within A rectangle has its dimensions in a and b . If a pair of
the lines outsides the field (perimeter). They run round opposite sides are ( 4a )cm and ( 2a + 3b )cm and the other
the length and breadth of the field but not within the pair of the opposite sides are ( 5 + a ) cm and
field as the players ( 2a – b ) cm long , find the
( i ) values of a and b
( ii ) area of the rectangle
Solution
( i ) Recall from the properties of rectangle that : the pair of
opposite sides are equal in length. Thus ,
4a = 2a + 3b i.e 2a – 3b = 0 ------ ( 1 )
Also 5 + a = 2a – b i.e a – b = 5 ----- ( 2 )
From ( 2 ) a = 5 + b substituting into ( 1 )
Assistant referee (line’s men) 2a – 3b = 0 will become
2 ( 5 + b ) – 3b = 0
Thus, area is the content of any plane shape while the 10 + 2b – 3b = 0
perimeter is the distance round the shape. 10 – b = 0
10 = b
Rectangle Substituting b value into ( 2 )
a – b = 5 will become
a – 10 = 5
a = 5 + 10 i.e a = 15
b
( ii ) Area of rectangle = L x b
= 4a x ( 5 + a )
= 60 x 20
L = 1200cm2
1998/4a OM
Area = L x b The area of a rectangular floor is 13.5m2. One side is 1.5m
Perimeter = 2L + 2b longer than the other.
= 2 ( L + b) (a) Calculate the dimension of the floor.
Analysis and Solution
1998/42 OM In a rectangle, the length is always longer than the breadth
The diagram below is a rectangle. If the perimeter is Thus,
36m, find the area of the rectangle Breadth = x
A 64m2 B. 65m2 C. 84m2 D124m2 E. 275m2 Length = Breadth + 1.5m
= x + 1.5m
Area = L x b
(x+1)m 13.5 = ( x + 1.5) (x )
13.5 = x2 + 1.5x
Rearranging,
(2x+5)m x2 + 1.5x – 13. 5 = 0
Solution x2 + 3/2x – 27/2 = 0
Perimeter of a rectangle = 2 (L + b) 2x2 +3x – 27 = 0
36 = 2 [(2x + 5 ) + (x + 1)] Factorising,
= 2 ( 2x + 5 + x + 1) 2x2 + 9x – 6x – 27 = 0
= 2 (3x + 6) x (2x + 9) – 3 ( 2x + 9) =0
36 = 6x + 12 ( 2x + 9) ( x – 3) = 0
36 – 12 = 6x 2x = -9 or x = 3
24 = 6x x = - 9/2 or 3
24 = x Since length of a rectangle can not be negative,
6 x=3
x=4
229
Dimension: 2002/2 Neco (Nov) Exercise 20.2 OM
Breadth = x and Length = x + 1.5 The area of a piece of land is 50cm2. Find the value of x if
= 3m = 3 + 1.5 = 4.5m the length and breath are given as (x + 2 )m and
1999/12b OM (x – 3)m respectively .
A rectangular lawn of length (x + 5 ) metres is A. 3m B. 5m C. 7cm D. 8cm E. 10m
( x – 2 )metres wide . If the diagonal is ( x + 6) metres,
find (i) the value of x 2002/9b NABTEB Exercise20.3 OM
(ii) the area of the lawn
Solution (2x+y)cm
From Pythagoras rule
(x + 6)2 = (x + 5)2 + ( x – 2)2
x2 + 12x + 36 = x2 + 10x + 25 + x2 - 4x + 4 (1+x-y)cm 2ycm
x2 + 12x + 36 = 2x2 + 6x + 29
2x2 + 6x + 29 – ( x2 + 12x + 36 ) = 0
2x2 + 6x + 29 – x2 – 12x – 36 = 0 (3y+5)cm
x2 – 6x – 7 = 0 The figure above is a rectangle with the length of the sides as
x2 – 7x + x – 7 = 0 given . Find
x (x –7) + 1(x – 7 ) = 0 ( i ) the value of x and y and
(x – 7) (x +1) = 0 ( ii ) its area to 3 significant figures .
x – 7 = 0 or x + 1 = 0
x = 7 or – 1 2004/10 ( Nov ) OM
Since sides of a rectangle can not be negative; x = 7m A rectangular photograph 15cm by 9cm is pasted on a
(ii) Length = x + 5 and breath = x – 2 rectangular card . If a margin of 2.5cm is left round the
=7+5 =7–2 photograph , find the perimeter of the card .
= 12m = 5m A. 58cm B . 68cm C . 98cm D. 228cm
Thus area = L x b Solution
= 12 x 5 Sketching the diagram , we have
= 60m2
1996/23 OM Card
One side of a rectangle is 8cm and the diagonal is
10cm. What is the area of the rectangle ? 15cm
A. 80cm2 B. 48cm2 C 40m2 D 36cm2 E 24cm2
Analysis and Solution Picture 9cm
Area of rectangle = L x b 2.5cm
One side is given, then we are to find the other;
2.5cm
8cm

m Length of card = 15cm + 2.5cm + 2.5cm i.e 20cm


10c Breadth of card = 9cm + 2.5cm + 2.5cm i.e 14cm
x
 Perimeter of card = 2(L + b )
= 2(20 + 14 )
= 68cm ( B )
Applying Pythagoras’ rule from the above diagram.
102 = x2 + 82 Square
102 - 82 = x2
(10 – 8) (10 + 8) = x2
2 x 18 = x2
36 = x2
x = 36 = 6cm
L
Area of rectangle = 8 x x
= 8x6
= 48cm2 (B)

2004/11 Exercise 20.1 OM Area = L x L = L2


Find the area of a rectangle of length 4cm and whose Perimeter = 4L
diagonal is 8cm. ( leave your answer in surd form.)
A. 8  3cm2 B. 12  3cm2 C. 16  2cm2 D. 16 3cm2

230
Circle
1991/18 – 19 OM
In the diagram below PXR and PYO are two semi-circles
with diameters 14cm and 7cm respectively.
r Use the diagram to answer Questions 18 and 19
(Take  = 22)
7
X

Area = r2
Perimeter (circumference) = 2r

1998/49 ( Nov ) OM 7cm


O
The area of a circular field is 154m2. What is the P R
perimeter of the field ? ( Take  = 22/7 )
A. 44m B. 49m C. 88m D. 176m E. 484m
Solution Y
Perimeter of a circle ( circumference ) = 2 r
We find r from the given area of circle 18 Find the area of the enclosed region PXROY correct to
Area of circle =  r2 the nearest whole number.
154 = 22 x r2 A. 96cm2 B. 116cm2 C 154cm2 D. 192cm2 E 385cm2
7 Analysis and Solution
154 x 7 = r2 The given shapes are combination of two semi-circles.
22 Thus, Area of Semi-circle PXR = 1/2 r2
49 = r2 Recall that half of diameter 14cm, gives us radius 7cm
49 = r =1 x 22 x 72
7 =r 2 7
Perimeter of a circle ( circumference ) = 2 r = 11 x 7 = 77 cm2
= 2 x 22 x 7
7 Similarly,
= 44m ( A ) Area of semi-circles PYO
= 1 x 22 x ( 7/2)2
1992/4b OM 2 7
The circumference of a circular track is 400m. = 1 x 22 x 7 x 7
Find its radius, correct to the nearest metre. 2 7 2x2
(Take  = 22/7) = 19. 25cm2
Solution Area of enclosed region
Circumference (perimeter) = 2r = 77cm2 + 19.25cm2
400 = 2r = 96.25
200 = r  96cm2 to the nearest whole number (A)
200 = r
 19 Find the perimeter of the region
r = 63.64m A. 2cm B. 33cm C. 40cm D. 47 cm E. 66cm
 64 to the nearest metres Analysis and Solution
The perimeter of the region is the sum of the perimeter of the
2000/47 Exercise 20.4 OM two semi-circles.
The area of a circle is 38. 5cm2 Thus, perimeter of Semi-circle P X R
Find its diameter. (Take  = 22/7 ) = ½ (2r)
A. 22cm B 14cm C. 7cm D 6cm = r
= 22 x 7 = 22cm
1999/40 ( Nov ) Exercise 20.5 OM 7
Find the diameter of a circular field whose area is Similarly,
616cm2. Perimeter of semi-circle P Y O
( Take  = 22/7 ) = r = 22 x 7
A. 98.00m B.49.00m C. 28.00m D. 24.82m 7 2
= 11cm
Hence, Perimeter of region = 22cm + 11cm
= 33cm (B)

231
Triangle Parallelogram
B

a a
h c h

C A
b
b
Area = ½ b x h i.e area = ½ base x height
Area = b x h i.e area = base x height
= ½ ab sin C
= ab sin 
or ½ ac sin B
or ½ bc sin A 2004/11 NABTEB ( Dec) OM
If all the sides are given: use Hero’s formula Calculate the area of a parallelogram whose sides are
6cm and 12cm long with included angle 600.
Area = s(s – a) ( s – b ) (s – c)
A. 36 3 cm2 B. 18 3 cm2 C. 9 3 cm2 D. 3 cm2
Where s = ½ ( a + b + c)
Solution
Area of parallelogram = ab Sin
1991/47
= 6 x 12 Sin 600
Find the area of an equilateral triangle of side 16cm.
A. 64 3cm2 B. 72 3cm2 C 96cm2 = 6 x 12 x 3
2
D 128  3cm2 E. 128cm2
Analysis and Solution = 363 cm (A).
2

Though no angle was given, applying Hero’s formula


will be slower but it is quicker to use the other formula, 2004/12 NABTEB OM
since we know that; equilateral triangles has 600 as its Find the area of the parallelogram ABCD with
angles. height BX = 11cm, |CD| = 8cm and |BC| = 13cm
x D
Area = ½ x 16 x 16 sin 600 A
= 8 x 16 x 3 (sin 60 =  3 in trig)
2 2 11cm
2 8cm
= 64 3cm ( A )

1997/20 OM
C
The diagram is a triangle XYZ whose area is 23. 5cm2 B 13cm
XY = 10 cm and YZ = 80cm. Calculate , correct to the
nearest degree. A. 72cm2 B. 88cm2 C. 104cm2 D. 143cm2.
A. 340 B. 350 C. 360 D. 370 D 400 Solution
X Area of 11gm = b x h
= AD x BX
Recall that opposite sides of a 11gm are equal
cm = 13 x 11
10 = 143cm2 ( D )

Y
Properties of Triangles & Parallelogram on the
Z
8cm same base and between the same parallels.

Solution 1. Triangle on the same base and between the same


Area = ½ /XY x /YZ/ x Sin Y parallel are equal in area.
23. 5 = ½ x 10 x 8 x sin 
23. 5 = sin 
40
0.5875 = sin
 = sin-1 (0.5875)
= 35. 980
 360 ( C )

Area of ∆ACD = Area of ∆BCD


232
2. If a triangle and a parallelogram are on the Solution
same base and between same parallels, the area of Triangles lying on the same base and between the same
the triangle is equal to half of that of the parallel are equal in Area. Thus,
parallelogram Area ∆SQR = Area ∆RPQ
= ½ base x height
= ½ x 6.4 x 3.2
= 10.24cm2 ( C )

1991/49 OM
Use the information to answer Questions 49
E F

Area of ∆ECD = ½ area of parallelogram ABCD


10cm
15cm
3. Parallelogram on the same base and between
the same parallels are equal in area.
8cm
P Q
M1
M2 U1 U2
49. If the area of triangle DCF = 24cm2, find the area
the quadrilateral ABCD.
A 24cm2 B. 48cm2 C. 80cm2 D. 96 cm2 E. 120cm2
Solution
Parallelograms on the same base between the same parallels
are equal.
And the area of the triangle = 1/2 area of //gm
Thus,
Given: Parallelograms ABCD, PQCD on the same Area of ∆DCF = 24cm2
base DC and between the same parallels AQ and DC. Area of IIgm DEFC = 2 x 24
To prove: ABCD = PQCD = 48cm2
Proof: = Area of IIgm ABCD (B)
In ∆ s APD and BQC
M1 = M2 (Corr. s)
U1 = U2 (Corr.  s) Trapezium
AD = BC (Opp. Sides of llgm)
Therefore ∆ APD = ∆ BQC (AAS) a
Thus, Quad. AQCD - ∆APD is equal quad.
AQCD - ∆BQC is equal quad.
Hence PQCD = ABCD h
1990/32 OM
In the diagram below PS//RQ, /RQ/ = 6.4cm and b
perpendicular PH = 3.2 CM. Find the area of SQR.
A. 5.12cm2 B. 9.60cm2 C. 10.24cm2 Area of trapezium = 1/2 ( a + b ) x h
2
D. 20. 48cm E. 40.96cm i.e = 1/2 (sum of two parallel sides ) x height

1998/38 OM
Calculate the perimeter of the trapezium PQRS.
A 24cm B. 36cm C. 44cm D. 72cm E. 520cm

P 16cm Q

5cm
4cm
S R
A

233
Analysis and solution A. 1.75cm2 B. 3.50cm2 C. 17.50cm2
2 2
Perimeter is the distance round a given shape: D. 35.00cm E 168.00cm
Length SR is not given: Solution
The figure is a trapezium
But SA = 52 - 42
=9 Area = ½ ( a + b) x h
= 3cm and Here a = 8cm, b = 12cm, h =?
AR = 16cm But sin 300 = h
Thus, perimeter of shape 3 1/ 2
0
= 5 + 3 + 16 + 4 +16 = 44cm ( C ) h = 7 x sin 30
2
2000/44 Exercise 20.6 OM =7x1 = 7
The lengths of the parallel sides of a trapezium are 5cm 2 2 4
and 7cm. If its area is 120cm2. Find the perpendicular Area = ½ (8 + 12) x 7 = 20 x 7 =17. 5cm2 (C)
distance between the parallel sides 4 2 4
A. 5.0cm B. 6.9cm C. 10.0cm D 20. 0cm
2002/6 Neco (Nov) Exercise 20.8 OM
2001/50 Exercise 20.7 OM D 8cm C
PQRS is a trapezium in which /PS/ = 9cm 12c
/QR/ = 15cm, /PQ/ = 2 3 PQR = 900
h
m
and  QRS = 300. Calculate the area of the trapezium 0
3
0
9cm B
S A 1
6cm
P
In the diagram above, ABCD is a trapezium, in which /AB/ is
23 parallel to /DC/ . /AB/ = 16cm, /BC/ = 12 cm.
/CD/ = 8cm and ABC = 300 Calculate the area of ABCD
300
A. 144cm2 B. 92cm2 C. 72cm2 D 36cm2 E. 27cm2
Q 15cm R
2002/42 Neco Exercise 20.9 OM
A 243cm2 B. 363cm2 C. 42 3cm2 D. 72 3cm2 In the trapezium XYZM, XY// MZ,  XYZ = 900
/PZ/ = 6cm , /MZ/ = 9 cm and /MX/ = 5cm. What is the
2001/27 Neco OM perimeter of the trapezium
The area of the trapezium given below is 48cm2 X Y
What is the value of x ?
5
cm 5cm h

x
cm M P 6cm Z
9cm

A 23cm B. 24cm C. 25cm D. 30cm E. 60cm


7
cm

A. 35cm B.13cm C. 12cm D. 8cm E 5cm Rhombus (with given diagonal)


Solution A
Area of trapezium = ½ (a + b) h
48 = ½ (5 +7 ) x x
x 1
x = 2 x 48
12
= 8cm (D) B D
y1 y2
2001/44/ Neco OM x2
What is the area of the figures below?
8
cm
C
m
3 /21c

Diagonal BD = y Where y = y1 + y2
3
0
0 and diagonal AC = x Where x = x1 + x2
1
2cm Area of rhombus = ½ x y or = ½ x1 y1 x 4
Where x and y are the two stated diagonals

234
1995/23 OM Area of rhombus = 1 x y
The diagonal AC and BD of a rhombus ABCD are 2
16cm and 12cm long respectively. Calculate the area of 24 = 1 x 8 x y
the rhombus. 2
A. 24cm2 B. 36cm2 C. 48cm2 D. 60cm2 E. 96cm2 2 x 24 = y
Solution 8
Area of Rhombus = ½ x y 6cm = y
Thus, K = 6/2 = 3cm.
A By Pythagoras’ rule
Rhombus side = 42 + 32
= 16 + 9
8cm

D 6cm 6cm = 25 i.e. 5cm (B).


B
8cm

Length of an arc
For any circle, with arc PRQ (minor) or arc PSQ (major)
R
P Q P Q
C

Where x & y are the diagonals


=1/2 x 16 x 12 Majorarc
= 96cm2 (E)
S
2001/29 Neco OM Subtending an angle  at the center of the circle,
The length of the diagonals of a rhombus are 24cm and length of the arc is =  x 2r
10cm respectively. What is the perimeter of the rhombus? 360
A 169cm B. 86cm C. 52cm D 26cm E 13cm
Analysis and Solution 1989/18 OM
By Pythagoras rule An arc of a circle of radius 7cm is 14cm long. What angle
CD2 = 122 + 55 does the arc subtend at the center of the circle?
= 144 + 25 = 169 (Take  = 22/7 )
A. 25.70 B. 440 C. 51.430 D. 980 E. 114.550
CD = 169 = 13cm Solution
But the sides of a rhombus are equal; thus
Length =  x 2r
Perimeter = 4L
360
= 4 x 13 Substituting, we have
= 52cm (C)
14 =  x 2 x 22 x 7
360 7
2004/4 (Nov) OM
14 x 360 = 
The area of a rhombus is 24cm2 and one of its diagonals
is 8cm. Find the side of the rhombus. 2 x 22
A. 4.3cm B. 5cm C. 6cm D. 10cm.  = 114. 550 ( E )
Solution
Pictorially, we have 1989/42 OM
Calculate, correct to three significant figures the length of an
arc which subtends an angle of 700 at the center of the circle
of radius 4cm
4 = Side? (Take  = 22/7 )
A 2.44 cm B 4.89cm C 9.78cm D 25.1cm E 50.3cm
4 Solution
k k Length of arc =  x 2r
4 360
k
Substituting for the various value
Length of arc = 70 x 2 x 22 x 4
360 7
For us to know the Rhombus side; = 2 x 22 x 4
We first find K i.e. half of the 2nd diagonal of rhombus. 36
= 4. 8889 cm
 4. 89cm to 3 s.f
235
1990/12 OM 1996/19 OM
An arc of length 22cm subtends an angle of  at the
center of a circle. What is the value  if the radius of the X
circle is 15cm?
(Take  = 22/7 )
O
A 700 B. 840 C. 960 D. 1560 E 1680

m
5c
Solution 60 0

3.
Length of arc =  x 2r
360 P Q
Substituting, we have
22 =  x 2 x 22 x 15 (Take  = 22/7 )
360 7 What is the length of the major arc PXQ?
22 x 360 x 7 =  A. 22cm B. 18 1/3cm C. 11cm D. 9 1/6cm E. 7 1/3cm
2 x 22 x 15 Solution
 = 840 ( B ) Length of arc =  x 2r
360
1990/18 OM Here  = 360 – 600 = 3000
0

In the diagram below O is the center of the circle with Substituting,


radius 10cm and  ABC = 300. Calculate, correct to 1 Length of arc = 300 x 2 x 22 x 3.5
d.p, the length of arc AC 360 7
(Take  = 3.14) = 30 x 2 x 22 x 7
A 36 7 2
= 660
C 36
= 110 = 55
6 3
300
= 18 1/3cm (B)
O
B 1997/19 OM
Find the radius of a circle in which an arc of length 44cm
subtends angle 2000 at the center of the circle
A 5.2cm B 10.5cm C 13.2cm D 20.6cm E 31.4cm (Take  = 22/7 )
Solution A. 3.9cm B. 12. 6cm C. 25 .2cm D 38.4cm E. 124cm
The angle subtended by arc AC is not given but Solution
applying the theorem the angle that an arc of a circle Length of arc =  x 2r
subtends at the center is twice that which it subtends at 360
any point on the remaining part of the circumference. Substituting,
We have 44 = 200 x 2 x 22 x r
 AOC = 2 x 300 360 7
= 600 () 44 x 360 x 7 = r
Length of arc =  x 2r 200 x 2 x 22
360 r = 12.6cm (B)
= 60 x 2 x 3.14 x 10
360 1998/36 Exercise 20.11 OM
= 10.467cm In the diagram, O is the center of circle PRQ.
 10.5cm to 1 d.p (B) The radius is 3.5cm and  POQ = 500
(Take  = 3.142)
1994/29 Exercise 20.10 OM R
Calculate correct to 2 significant figures, the length of
the arc of a circle of radius 3.5cm which subtends an
angle of 750 at the center of the circle. O
( Take  = 22/7 )
3.

50 0
5c
m

A 2.3cm B 4.6cm C. 8cm D.16cm E. 110 cm


P Q
Calculate, correct to one decimal place, the length of arc PQ
A. 175.0cm B. 157. 1cm C. 37.7cm D 11.0cm E 3.1cm

236
2001/45 Exercise 20.12 OM 2002/33 Neco Exercise 20.13 OM
The arc of a circle 50cm long subtends an angle of 750 What is the length of an arc (correct to 2 significant figures)
at the center of the circle. Find correct to 3 significant of a circle of diameter 7cm which subtends an angle of 750 at
figures the radius of the circle the center of the circle?
( Take  = 22/7 ) A. 2.3cm B. 4.6cm C. 8cm D. 16cm E. 110 cm
A. 8.74cm B. 38.2cm C. 61.2cm D. 76 . 4cm
2002/10 (Nov) Exercise 20.14 OM
2001/41 OM In the figure below, AOB is a sector of a circle with center O
In the diagram O is the center of the circle. and radius 3.5cm. If the reflex  AOB = 2900, calculate the
Reflex XOY = 2100 and the length of the minor arc is length of the minor arc AB.
5.5m. find correct to the nearest metre, the length of the (Take  = 22/7 )
major arc.
A
Y
B
210
0 0 cm
3.5
290
X

A. 8cm B 9cm C. 10cm D 13cm A. 17.72cm B 15.20cm C. 14.97cm D. 4.28cm E 2.14cm


Analysis and Solution
Length of major arc =  x 2r 2004/45 NABTEB ( Dec ) Exercise 20.15 OM
360 An arc of a circle of radius 14cm is 21cm long. What angle
 = 210, r = ? does it subtend at the centre of the circle?
But we can solve for r from the information given about A. 3600 B. 2700 C. 3600 D. 2200
the minor arc   7 7
Length of minor arc =  x 2r
360 Perimeter of a Sector
Here  = 360 – 210 = 150

Substituting for given values, MajorS


ector
O
5.5 = 150 x 2 x 22 x r
360 7 r r
r = 5.5 x 360 x 7 = 2. 1m Minor
150 x 2 x22 A B S
e ctor

Thus, substituting for given values,


Length of major arc = 210 x 2 x 22 x 2.1 For any circle with center O and radius r from which a sector
360 7 AOB is formed. The perimeter of the sector is the sum of the
= 7.7m two radii and the length of the arc AB since perimeter is a
 8 m to the nearest meter (A) distance round a given shape.
Perimeter of a sector = 2r + L of arc
2001/43 Neco OM Where L of arc =  x 2r
An arc of a circle subtends an angle of 750 at the center 360
of the circle. If the diameter of the circle is 7cm, what is 1988/42 OM
the length of the arc? A rope of length 18cm is used to form a sector of circle of
( Take  = 3.142 ) radius 3.5cm, on a school playing field. What is the size of
A. 4.58cm B. 8.02 cm C. 16.04cm the angle of the sector correct to the nearest degree?
D. 32.08cm E 34cm A. 330 B. 400 C. 900 D. 1800 E. 2700
Solution Solution
L of arc =  x 2r Perimeter of a sector = 2r + L of arc
360 We are to find 
Here r = diameter = 7 Substituting,
2 2 18 = 2 x 3.5 +  x 2 x 22 x 3.5
Substituting 360 7
L of arc = 75 x 2 x 3.142 x 7 18 - 2 x 3.5 =  x 2 x 22 x 3.5
360 2 360 7
= 4.58cm (A)  = 11 x 360 x 7
22 x 7
= 1800 (D)

237
1992/3a OM Solution
The angle of a sector of a circle of radius 7cm is 1080. Perimeter of sector AOB = 2r + L of arc
Calculate the perimeter of the sector. = 2r + L of arc ACB
(Take  = 22/7 ) = 2 x 5 + 100 x 2 x 3.142 x 5
Solution 360
Perimeter of a sector = 2r + L of arc = 10 + 8.728
= 2 x 7 + 108 x 2 x 22 x 7 = 18.728cm
360 7  18.73 cm to 2 d.p
= 14 + 13.2
= 27. 2cm 2002/14 Neco (Nov) Exercise 20.19 OM
Calculate the perimeter of a sector of a circle of radius 14cm,
1993/18 OM which subtends an angle of 1400 at the center.
The angle of a sector of a circle of radius 10.5cm is A 1960.4cm B. 47.3cm C. 62.2cm D 53.4cm E. 34.2cm
1200. Find the perimeter of the sector
(Take  = 22/7 ) 2004/4a NABTEB ( Dec ) Exercise 20.20 OM
A. 22cm B. 33.5cm C 43cm D. 66cm E. 115.5 cm An arc of a circle radius 8cm subtends an angle of 1350 at the
Solution centre of the circle. Calculate, to one decimal place, the
Perimeter of sector = 2r + L of arc perimeter of the sector. (Take  = 3.142)
= 2 x 10. 5 + 120 x 2 x 22 x 10. 5
360 7
= 21 + 22 Area of a sector
= 43cm (C)

1995/32 Exercise 20.16 OM MajorS


ector
The angle of a sector of a circle is 1800. If the radius of O
the circle is 31/2cm. Find the perimeter of the sector. r r
A. 6 3/5cm B. 6 4/5cm C. 7 1/10cm Minor
D. 10 2/5cm E 13 3/5cm A B S
e ctor

1996/25 Exercise 20 17 OM
The angle of sector of a circle of radius 35cm is 1080. The area of a sector AOB of a circle radius r of center O is
Find the perimeter of the sector. given as
(Take  = 22/7 ) Area of sector =  x area of circle
A. 246cm B. 211cm C 141cm D 114cm E.44cm 360
=  x r2
2001/45 Exercise 20.18 OM 360
A sector of a circle of radius 14cm subtends an
angle1350 at the center of the circle. What is the 1989/37 OM
The angle of sector of a circle of diameter 8cm is 1350.
perimeter of the sector? ( Take  = 22/7 )
Find the area of the sector
A. 47cm B. 61cm C. 88cm D 231cm
(Take  = 22/7 )
2002/9a (Nov) OM A. 9 3/7 cm2 B. 12 4/7 cm2 C. 18 6/7 cm2
D. 25 1/7 cm2 E. 31 3/7 cm2
Solution
Area of sector =  x r2
360
Here, radius = diameter = 8 = 4cm
2 2
Substituting,
Area of sector = 135 x 22 x 42
360 7
= 132 = 18 6/7 cm2 (C)
7
In the diagram, OACB is a sector of a circle centre O
and radius 5cm. Given that  AOB = 1000. Calculate, 1990/25 OM
correct to two decimal places, the Perimeter of the A sector of a circle of radius 7cm has an area of 44cm.2.
sector Calculate the angle of the sector correct to the nearest degree
(Take  3.142) (Take  = 22/7 )
A. 60 B. 260 C. 520 D. 1030 E. 2060

238
Solution 1998/39 OM
Area of sector =  x Area of circle The length of an arc of a circle of radius 5cm is 4cm.
360 Find the area of the sector.
44 =  x 22 x 7 x 7 A. 1 3/5 B. 2cm2 C. 8cm2 D. 10cm2 E. 20 cm2
360 7 Analysis and solution
44 x 360 x 7 =  Area of a sector =  x area of circle
22 x 7 x 7 360
 = 102.860 =  x r2
 1030 to the nearest degree. 360
All the unknowns are given except  But
1993/14 Exercise 20. 21 OM Length of arc =  x 2r
A sector of a circle of radius 9cm subtends angle 1200 360
at the center of the circle. Find the area of the sector to 4 =  x 2 x 3 .142 x 5
the nearest cm2 360
( Take  = 22/7 )  = 360 x 4 = 45.8
A. 75cm2 B. 84cm2 C. 85cm2 D. 862 E. 95cm2 31.42
Therefore,
1994/30 OM Area of sector = 45 .8 x 3.142 x 52
A circle has radius x cm. What is the area of a sector of 360.
the circle with angle1350 , leaving the answer in terms = 9.993 cm2
of x and  ?  10cm2 (D)
A x2 B. 3x2 C. 5x2 D. 3x E. 3x
8 8 8 8 4 1999/45 (Nov) OM
Solution P
Area of sector =  x area of circle ; Here r = x 45
0

360
= 135 x  x x2
360 O
= 3x2 R
m
8 (B) 7c
Q
1996/20 Exercise 20.22 OM
In the diagram above, PQR is a circle centre O.
|OQ| = |OR| = 7cm and QPR = 450. Calculate the area of
O the shaded portion (Take  = 22/7)
A. 11cm2 B. 22cm2 C. 38.5cm2 D. 77cm2.
cm
3.5

600 Solution
Q Area of a shaded portion = area of sector QOR
P But QOR which is the angle subtended by shaded sector is
What is the area of the minor sector POQ? 2 x 450 (angle at centre and circumference theorem).
A. 148 1/2cm2 B. 77cm2 C. 32 1/12 cm2 Area of shaded portion =  x r2
D 6 5/12cm2 E. 1 5/6 cm2 360
= 90 x 22 x 72
1998/37 Exercise 2.23 OM 360 7
= 38.5cm2 (C).

Length of a chord
O
3.5

500
mc

Q O
P
r r
Calculate correct to three significant figures,
the area of sector, OPQ A B
A 3.403cm2 B. 5. 350 cm2 C. 10. 99cm2
2
D. 157. 1 cm E. 175.0cm2 Length of chord AB = 2r sin 
2

239
On the other hand if  is not given but a shape like the 8 = 1.732 x r
one below is given; we apply Pythagoras theorem to 8 = r
solve for the length of chord. 1.732
r = 4.62cm (D)
O 2004/3a OM
AB is a chord of a circle centre O. if /AB/ = 24.2cm and
the perimeter of AOB is 52.2cm.
A C B Calculate AOB correct to the nearest degree.
Solution
1989/22 OM
In the diagram below O is the center. Calculate the Sketching the diagram, we have
length of the chord AB. If /OA/ = 5cm
/OD/ = 3cm and AOD = BOD
A.3cm B.4cm C.5cm D.8cm E.15cm
O
r r
O
O
A 24.2cm B
5
3
A B From length of chord AB = 2r Sin  ;We are to find 
D
2
Analysis and Solution
By Pythagoras rule But r is not given. It can be gotten from theperimeter of
/DB/2 = 52 - 32 AOB = 52.2cm and AB = 24.2
= 25 – 9 since OA = OB (radius of circle)
/DB/2 = 16 2r = 52.2 - 24.2
DB = √16 = 4cm 2r = 28
But AB = 2 x DB
r = 28 i.e. r = 14
= 2 x 4 = 8cm (D)
Substituting: 2
1994/26 OM 24.2 = 2 x 14 Sin 
The angle subtended at the center by a chord of a circle 2
radius 6cm is 1200. Find the length of the chord 24.2 = 28 Sin 
A 3 cm B. 6cm C. 4 2cm D. 3  3cm E. 6  3cm
2
Solution
Length of chord = 2r sin  24.2 = Sin 
2 28 2
= 2 x 6 sin 120 –1
Sin 0.8643 = 
2 2
= 12 sin 60
59.80 = 
= 12 x  3
2 2
= 6  3cm (E) = 2 x 59.80 = 119.60  1200.
2003/14 Neco Exercise 20.24 OM
1999/39 (Nov) OM In the figure below, the radius |OM| = 11cm and the
A chord of a circle 8cm long subtends an angle of 1200 perpendicular distance from the centre to the chord is 7cm,
at the centre. Find the radius of the circle. calculate |MN|.
A. 6.93 B. 5.00 C. 3.46 D. 4.62
Solution
Length of chord = 2r Sin 
O
2
8 = 2 x r Sin 1200 cm
11 7cm
2
M N
P
8 = 2 x r Sin 600
8 = 2 x 0.8660 x r A. 122cm B. 92cm C. 62cm D. 6cm E. 32cm.
240
2004/ 12b OM The perimeter of a segment
= length of arc ACB + length of chord AB
=  2r + 2r sin 
360 2
O
Area of segment of a circle
x
y

Q
k P
O
In the diagram, O is the centre of the circle radius X, r r
|PQ| = Z, |OK| = y and OKP = 900. Find the value of Z
in terms of x and y
Solution A B
|PQ| = Z is the length of a chord. observe that QK = KP.
Thus by Pythagoras’ rule. Area of segment = area of sector – area of triangle
In OPK =  x r2 – ½ r2 sin 
x2 = |KP|2 + y2 360
x2 – y 2 = |KP| = r2 (  - sin  )
But 2 x |KP| = PQ 2 180
Therefore PQ = 2 x2 – y2 1992/18 OM
The diagram below shows the shaded segment of a circle of
2003/45 NABTEB OM radius 7cm. If the area of the
Calculate the length of a chord of a circle of radius
10cm. If the chord is 6cm from the centre of the circle.
A. 14cm B. 16cm C. 18cm D. 20cm.
Solution
The sketch is as;

O
triangle OXY is 12 1/4cm2. Calculate the area of the segment.
(Take  = 22/7 )
10
cm cm
10
6cm

A. 5/12cm2 B. 7/12cm2 C. 1 1/6cm2


A C B D. 2 1/3cm2 E. 6 1/6cm2
Analysis and solution
Required length is AB Area of segment = area of sector – area of ∆
But AB = AC + BC and AC = BC Here area of ∆ is given.
In OAC: by Pythagoras rules Thus,
102 = 62 + AC2 Area of sector =  x r2
AC2 = 100 -36 360
AC2 = 64 = 30 x 22 x 72
AC = 64 360 7
= 8 cm = 154
Thus length AB = AC + BC i.e. 8 + 8 = 16cm (B) 12
Therefore area of segment = 154 - 121/4
12
Perimeter of Segment of a circle = 154 - 49
12 4
7
= 154 – 147 = /12 cm 2
(B)
O Majorsegm
ent 12
r r 1992/8a OM
Calculate the area of the shaded segment of the circle shown
Min
orsegm
ent in the diagram below:
A B (Take  = 22 )
C 7

241
2004 / 8a Exercise 20.27 OM

cm
10

630
O
10cm
5c

m
O
96 m

5c
Solution
Area of segment = Area of Sector – area of ∆ x y
Area of sector =  r2
360
In the diagram, xy is a chord of a circle of radius 5cm. The
= 63 x 22 x 102
360 7 chord subtends an angle 960 at the centre. Calculate, correct
= 55cm2 to three significant figures, the area of the minor segment cut
Area of triangle = ½ r2 Sin  off by xy. ( Take  = 22/7)
=102 sin 630
2
= 44. 55cm2
Area of segment = 55 – 44 . 55
Irregular plane – shapes
= 10. 45 cm2 ( Perimeter & area )
Irregular plane - shapes are figures that are combination of
two or more plane shapes . Methods of solving these types of
2002/9b (Nov) Exercise 20.25 OM
problems do not have a particular formula. We apply our
experience in plane shapes mensuration to solve them in
parts or as a whole.

2004/ 8b OM

O 7cm
In the diagram, OACB is a sector of a circle center O 800
and radius 5cm. Given that  AOB = 1000. Calculate,
7cm

correct to two decimal places, the area of the shaded


segment.
(Take  = 22/ 7 )
The figure shows a circle of radius 7cm inscribed in a square.
2001 /48 Neco Exercise 20 .26 OM If a portion of the circle is shaded with some portions of the
What is the area of the shaded region in the diagram square, calculate the total area of the shaded portions.
below? (Take  = 3.142)
(Take  = 22/7)
Solution
The area of the shaded portions is:
Area of square minus area of the inscribed circle’s unshaded sector
Area of square = L x L
Here L is 2 x radius of inscribed circle
i.e. = 2 x 7cm implies 14cm
Thus, Area of square = 14 x 14
= 196cm2
Area of unshaded sector =  x r2
A. 1.76cm2 B. 17.57cm2 C. 28. 09cm2 360
D. 45. 67cm2 E. 175.8cm2 = 360 - 80 x 22 x 72
360 7
= 280 x 22 x 7
360
= 119.778 cm2
242
Therefore: area of shaded portion. Diagonal |OR|2 = 162 + 122
= 196 – 119.778 |OR|2 = 256 + 144
= 76.22cm2 |OR| = 400 i.e. 20cm.
(ii) Area of shaded portion = Area of sector OQS minus
2004 / 26 (Nov.) OM area of rectangle OXRT.
=  x r2 - L x b
In the diagram, PQRO is one quarter of a circle with
360
centre O. |RQ| = |PR| = 7cm. Calculate, correct to two Here  is 900 i.e. angle of rectangle at O
decimal places, the area of the shaded portion. = 90 x 3.142 x 202 – (16 x 12)
(Take  = 22/7) 360
A. 57.70cm2 B. 38.50cm2 C. 27.00cm2 D. 19.25cm2 = 314.2 – 192 = 122cm2 to 3 sf

1986/5 GCE OM
P
The diagram shows a rectangle PQRS inscribed in a circle.
|PQ| = 6cm and |QR| = 8cm.
P Q
6cm
O

8cm
R Q
7cm
Solution
Area of the shaded portion S R
= Area of sector PQR minus Calculate, correct to three significant figures, the total area of
area of Semi – Circle PRO. the shaded parts. (Take  = 3.142)
=  x R - 1r2
2 Solution
360 2 Shaded part area = Area of Circle – Area of rectangle
Here  is 90 “one quarter of a circle R”
0 But the circle’s radius is half of rectangle’s diagonal
and R = 7, r = 7/2 Semi- Circle radius. Diagonal QS = 82 + 62
Area of the shaded portion 2 Diagonal QS = 64 + 36
= 90 x 22 x 72 – 1 x 22 x  7 
360 7 2 7 2 = 100 i.e. 10cm
= 38. 5 – 19. 25 Radius  Half diagonal = 10/2 i.e. 5cm
= 19. 25cm2 (D). Therefore shaded part area = r2 – L x b
= 3.142 x 52 – (8 x 6)
2004/11a (Nov) OM = 78.55 – 48
= 30. 55  30.6 to 3sf.
Q
2004/10a NABTEB (Dec) Exercise 20.28 OM
R ABCD is a square of side 12cm long inscribed in a circle of
X
centre O.
A

O
S
O T
In the figure, OQRS is a quadrant of a Circle centre O. B
OXRT is a rectangle 16cm by 12cm. Calculate
(i) radius of the circle. C
(ii) area of the shaded portion.
correct to 3 significant figures. Find the area of the shaded portion of the circle, correcting
your answer to two decimal places
[ Take  = 3.142 ]
(Take  = 3.142)
Solution
(i) The diagonal of rectangle QXRT is the radius of the 2003/11b NABTEB OM
circle
243
The wind screen wiper of a car sweeps through an angle Area of shaded portion = L2 - r2
of 1500. The blade of the wiper is 21cm long and the = 282 – 22 x (14)2
radius of the un-swept sector is 6cm as shown in the 7
diagram below. = 784 – 22 x 14 x14
7
= 784 – 616

cm
= 168cm.
21
2001/11 NABTEB OM
Find the perimeter of the shaded region in the diagram where
ABC and D are the centres of respective circles.
m
6c

(Leave your answer in  ).


150
0
D C

What area of the wind screen is swept clean?

2cm
(Take  = 22/7)
Solution
Area swept clean
= Area of the whole Sector, radius (6 + 21) minus
area of small sector (radius 6cm) A B
= 150 x 22 x (27)2 – 150 x 22 x 62 2cm
360 7 360 7
A. ( 2 + )cm B. 2cm C. 4cm D. 8cm.
Solution
= 15 x 22 x 27 x 27 – 15 x 22 x 36
The bigger figure is a square, since AB, = BC = 2cm
36 7 36 7
Recall that each angle of a square, is 900
Perimeter of the shaded region
= 15 x 22 (729 – 36)
= Sum of the length of each arc (4)
36 7
where  = 900 and r = 2/2 i.e.1cm
= 15 x 22 x 693 = 4  x 2r
36 7 360
= 4 x 90 x 2 x 1
= 15 x 22 x 99 360
36 = 2cm (B)
= 907.5cm2. 2002/24 Neco OM
A wire was shaped to form a square of area 81cm2. An
2002/2b NABTEB OM equilateral triangle was formed by the same wire. What is the
A circle centre O is inscribed in a square of side 28cm length of a side of the triangle formed
as shown in the figure below. Find the area of the A 3cm B. 6cm C. 9cm D 12cm E 27cm
shaded portions. (Take  = 22/7) Analysis and solution
Area of square = L2
L2 = 81
L = + √81 = + 9
But length cannot be negative,
L = 9cm
O We then find the perimeter of the square
4L = 4 x 9 =36cm
It is this perimeter that will be adjusted to form the length of
three equal sides of the new shaped equilateral triangle
Thus, Length of a side of equilateral ∆ = 36
28cm 3
= 12cm (D)
Solution 2004/50 (Nov). OM
Area of shaded portion The area of a square is equal to that of a triangle of base 9cm
= Area of square minus area of circle. and altitude 32cm.
But the Length of the square is the diameter of the circle Find the length of a side of the square.
Thus, circle’s radius = ½ square’s length A. 6cm B. 6 2cm C. 12cm D. 12 2cm.
= ½ x 28 = 14cm
244
Solution Of the three properties: curved surface area, total surface area
Area of a square = L2 and volume of a cylinder to be measured, only the total
Thus to find length of a side of the square, we take the surface area that has different formulae for the two types of
“Square root” of the area of the square cylinders.
But
Area of square = area of triangle r Open circular
= 1 base x altitude (height)
top
2
= 1 x 9 x 32
2

Height
= 144cm2
Therefore, length of square = 144
= 12cm (C).
1998/29(Nov) Exercise 20.29 OM
Find the length of a side of a square which is equal in Closed circular
area to a rectangle measuring 45cm by 5cm. base
A. 25cm B. 23cm C. 16cm D. 15cm. E. 10.6cm

r Closed circular
top
Chapter Twenty one
Solid mensuration
A solid is a three dimensional shape. It is usually
represented in two dimensions pictorially in books. Height
Examples of solid are shown below

Closed circular
base

Curved surface area & volume for one – open


and closed end (s) cylinders

Curved surface area = base circumference x height


When we walk round our homes, it is observed that it = 2r x h
has some shape which are the same on both sides – but = 2rh square unit.
on snapping a picture of the house, we can only see one
Volume = base area x height
plane view of it. That is another example of a three
= r2 x h
dimensional objects represented in two dimensions
= r2 x h cubic units
Surface areas & volumes
A painter hired to paint our house can only paint the walls
round the home (surface area) On the other hand, when
we want to sit, eat, sleep or even bring in new piece of Total Surface area for open top cylinder only
furniture, we occupy certain space in the house. We
cannot sit on the wall, or lie on the wall – thus we occupy Total surface area = area of all the faces
space (volume) in our homes. We can say that surface = Area of base + curved surface area
area in solids is equivalent of perimeter in plane shapes = r2 + 2rh
whereas volume in solids is an equivalent of area in plane = r ( r + 2h)
shapes.
Total Surface area for closed tops cylinder only
CYLINDER Total surface area = area of the faces
A cylinder is a solid with circular cross section. = Area of circular base + area of circular top + curved surface area.
Examples are bournvita tin, tomatoes tin, milk tin, and = r2 + r2 + 2rh
open drum etc. = 2r2 + 2rh
There are two types of cylinders namely: = 2r ( r + h ) square units
Open top cylinder &
Closed ends cylinder

245
1990/28 – 29 OM Total surface Area
Use the following information to answer question 28 = Area of circular base + area of circular top + curve surface area
and 29. = r2 + r2 + 2rh
(Take  = 22/7 ) A cylinder container closed at both = 2r2 + 2rh
ends, has a radius of 7cm and height 5cm = 2r ( r + h)
= 2 x 22 x 3 ( 3 + 6)
28 Find the total surface area of the container 7
A 35cm2 B. 154cm2 C. 220cm2 = 2 x 22 x 3 x 9
2 2
D 528cm E. 770cm 7
Solution = 169.7cm2
Total surface area for closed ends cylinder
= area of circular base + area of circular top + curved surface (ii) Volume of cylinder
areas
= r2h
= r2 + r2 + 2rh
= 22 x 32 x 6
= 2r2 + 2rh 7
= 2r ( r + h ) = 169. 7cm3
= 2 x 22 x 7 ( 7 + 5 ) But , the required volume is in litres,
7 169.7 litres
= 528cm2 ( D ) 1000
= 0.17litres

29. What is the volume of the container? 1995/25 – 26 OM adjusted


A 35m3 B 154cm3 C 220cm3 D 528cm3 E 770cm3 A cylindrical container, closed at both ends, has a radius of
Solution 14cm and height 10cm. Use this information to answer
Volume of cylinder = circular base area x height questions 25 and 26
= r2 x h (Take  = 22/7 )
= r2h
= 22 x 7 x 7 x 5 25. Find the total surface area of the container.
7 Solution
= 770cm3 ( E ) Total surface area of closed ends cylinder
= 2r ( r + h )
1992/15 OM = 2 x 22 x 14 ( 7 + 10 )
The curved surface area of a cylindrical tin is 704cm2 . 7
Calculate the height when the radius is 8cm
= 2 x 22 x 14 x 17
( Take  = 22/7 ) 7
A. 3.5cm B. 7cm C. 14cm D 28cm E. 32cm. = 1496cm2
Solution
Curved surface area of cylinder 26. What is the volume of the container?
= base circumference x height Volume of cylinder = r2h
= 22 x 142 x 10
= 2rh 7
704 = 2 x 22 x 8 x h = 6160cm3
7
7 x 704 = h
2 x 22 x 8 1996/22 Exercise 21.1 OM
h = 14cm (C) Find, correct to 1 decimal place the volume of a cylinder of
height 8cm and base radius 3cm
1992/8b OM (Take  = 3.142 )
A tin has radius 3cm and height 6cm. Find the : A. 503.0cm3 B. 300.0cm3 C. 250.0cm3
(i) Total surface area of the tin; D. 226.2cm 3
E. 150.9cm 3

(ii) volume in litres of liquid that will fill the tin to


capacity, correct to two decimal places; 1999/38 OM
(Take  = 22/7 ) Water flows from a tap into a cylindrical container at the rate
of 5cm3 per second. If the radius of the container is 3cm,
Solution calculate the level of water in the container at the end of 9
(i )A tin is a closed ends cylinder except stated seconds
otherwise (ie opened tin) A 2cm B. 5cm C. 8cm D 15cm
Solution
Level of water = height of cylinder
246
Thus, HOLLOWED CYLINDER, PIPES & RINGS
Total water flowing in = volume of cylinder This is a special type of cylinder, which is open at both ends
9 x 5 = r2h but has very thick layer. Examples are culverts, ring pipes
45 = 32h e.t.c Diagrammatic presentation of such shapes is shown
h = 45 = 5cm (B) below
9

2000/11a OM
A cylindrincal well of radius 1 metre is dug out to a L
depth of 8 metres. Calculate in m3 the volume of soil
dug out
(Take  = 22/7 )
Solution
Here h = 8m and r = 1m
Volume of cylinder = r2h
= 22 x 12 x 8
777
= 25.14cm3

2001/49 OM Open end - side view


A solid cylinder of radius 7cm is 10cm long.
Find its total surface area
A. 70cm2 B. 18cm2 C 210cm2 D. 238cm2
R = External radius
R
Analysis and Solution
A solid cylinder has closed ends and not opens ends r = internal radius
r
Surface area for closed ends cylinder = 2r (r + h)
= 2 x  x 7 ( 7 + 10)
= 14 (17)
= 238cm (D)
Area = R2 - r2
2002/34 OM
=  (R2 – r2 )
The volume of a cylinder of radius 14cm is 210cm2.
i.e difference in areas between the big circle with radius R
What is the curved surface area of the cylinder?
and small circle with radius r
A 15cm2 B. 30cm2 C. 616cm2 D 1262cm2
Similarly,
Solution
We need height to calculate curved surface area. Volume = R2L - r2L
Thus =  L(R2 – r2 )
Volume of cylinder = r2h
210 = 22 x 14 x 14 x h 1996/6d OM
7 Calculate the volume of the material used in making a pipe
20cm long, with an internal diameter 6cm and external
210 x 7 = h = 0.34cm
22 x 14 x14 diameter 8cm ( Take  = 22/7 )
Solution
Though the type of cylinder was not specified nor Volume = L (R2 – r2 )
implied, but only curved surface area for closed end L = 20cm. Internal radius r = diameter = 6 = 3cm
suits our answers. Thus, 2 2
Total surface area = 2r (r + h) External radius(R) = diameter = 8 = 4cm
= 2 x 22 x 14 ( 14 + 0.34) 2 2
7 Substituting,
= 1261. 92 cm2 Volume = 22 x 20 (42 – 32 )
 1262cm2 (D) 7
= 22 x 20 ( 16 – 9)
2002/36 Neco Exercise 21.2 OM 7
What is the total surface area of a cylinder with radius = 22 x 20 x 7
2cm and height 1.5cm, correct to 2 significant figure? 7
A. 440cm2 B. 220cm2 C. 88cm2 D. 44cm2 E 22cm2 = 440cm3

247
1999/7a OM Total surface area
A cylindrical pipe is 28 meter long. Its internal radius is = Area of circular base + curved surface area
3.5cm and external radius 5cm. Calculate = r2 + rL
i. The volume in cm3 of metal used in making the pipe = r ( r + L)
ii. The volume of water in litres that the pipe can hold
when full, correct to 1 decimal place ( Take  = 22/7 ) Volume = 1/3 r2h
Analysis and solution Slant height (L) , radius (r) and height (h) are related by
(i) Volume =  L (R2 – r2 ) Pythagoras’ theorem
L = 28m = 2800cm ; internal radius r = 3.5cm L2 = h2 + r2
External radius R = 5cm
Substituting, Cones formed from a sector
Volume = 22 x 2800 (52 – 3. 52 ) The diagram 1 shown below is a sector of a circle radius R
7 which subtends an angle  in the center O. then FigII is the
= 22 x 2800 ( 25 – 12.25) cone formed from the sector.
7
= 22 x 400 (12.75) O
= 112200cm3
Fig II
Fig I
O
(ii) Volume of water is the same as ordinary volume R=L
of the cylinder R h
Thus, we use r internal radius
Volume = r2h
= 22 x (3.5)2 x 2800 r
7 A B
= 107,800cm3
Converting volume in cm3 to volume in litres, we have
107,800cm3 = 107800 litres To measure the properties of a cone formed from a sector we
1000 need, L and h. This is gotten from the relationship between a
= 107. 8 litres to 1 d.p sector and the cone it formed.

2002/14 (Nov) Exercise 21.3 OM Relationship between a sector of a circle and the
The internal and external radii of a cylindrical bronze cone it forms
pipe are 1.5cm and 2cm respectively. If the pipe is
From fig. I and II
10cm long, calculate the volume of the bronze used
1. Length of arc = circumference of base of cone
( Take  = 22/7 )  x 2R = 2r
A. 51/2 cm3 B. 55cm3 C. 196 2/5cm3 D 550cm3 360
R = r
360
CONE Radius r of new cone formed
A cone is a solid with circular base and slant sides. The r = R
cone of concern to us is the right circular cone. There is 360
the ordinary right circular cone and cone formed from
a sector of a circle 2. R = L
Radius of the sector of a circle is equal to the slant height of
the cone formed.

3. Recall L2 = h2 + r2
Slant (L)
height h2 = L2 – r2
Height
2
(h)
h = L2 - R
360
r
Circular base 1988/40 OM
A cone is made from a sector of a circle of radius 14 cm and
angle of 900.What is the area of the curved surface of the
Properties to be measured cone ? (Take  = 22/7 )
Curved surface area = rL A. 22cm2 B. 88cm2 C.77cm2 D. 154cm2 E. 308cm2

248
Analysis and Solution
Curved surface area of cone = r1
Slant height (L) = circle radius (R) 1989/35 OM
L = 14cm Find the volume of a cone of radius 3. 5cm and vertical
height 12cm. (Take  = 22/7 )
Cone base radius r = R (circle radius) A. 3. 4cm3 B. 15. 5cm3 C. 21. 0cm3
360 D. 42.0cm 3
E. 154.0cm 3

r = 90 x 14 = 3.5cm Solution
360 Volume of cone = 1/3 r2h
Thus, curved surface area = r1 = 1 x 22 x ( 3.5)2 x 12
= 22 x 3.5 x 14 3 7
7 = 154cm3 (E)
= 154cm2 (D)
1989/40 OM
VTR – 11/30 NTI TCII OM A sector is cut off from a circle of radius 8.2cm to form a
A cone is 8cm high and has a base diameter of 12cm. cone. If the radius of the resulting cone is 3.5cm.Calculate
Its slant height is … ? the curved surface area of the cone (Take =  = 22 /7 )
A.6cm B.8cm C.10cm D.12cm E.20cm A. 11.25cm2 B. 12.83cm2 C. 22. 0cm2
Solution D. 67.2cm 2
E. 90. 2cm 2
Recall that the formula connecting height h, radius r and Analysis and Solution
slant height L of a cone is Curved surface area of cone = r L
L2 = h2 + r2 For a cone formed from a sector of circle;
Here L =? H = 8cm, r = 12/2 i.e.6cm Circle’s radius R = L = 8.2cm
L2 = 82 + 62 Cone’s radius r = 3.5cm
= 64 + 36 Substituting,
2
L = 100 Curved surface area of cone = 22 x 3.5 x 8. 2
L= 100 i.e. 10cm 7
Slant height =10cm (C) = 90.2cm2 (E)

1988/5 OM 1989/6b OM
A sector of a circle of radius 7cm subtending an angle If the sector POQ is used to form the curved surface of a
of 2700 at the centre of the circle, is used to form a cone cone with vertex O, calculate the base radius of the cone,
(a) Find the base radius of the cone. correct to one decimal place
(b) Calculate the area of the base of the cone correct to R
the nearest square centimeter.
(Take  = 22 / 7 )
0
42
Analysis and Solution
O
cm
3 .2
( a ) radius of cone formed from a sector
P Q
2700
7cm 7cm
h Solution
r = R
360
r
But  = 2 x 420 = 840 (angle 420 is at circumference)
r = 84 x 3.2
Sector Resulting cone 360
= 0.7467
r = R = 270 x 7 = 5.25cm  0.7cm to 1 d.p
360 360
( b ) A cone base is circular, thus 1990/6a OM
Area of cone base = r2 Find the volume of a right solid cone of base radius 4cm
and perpendicular height 6cm
= 22 x (5. 25)2
7 (Take  = 3.142)
= 86 .625cm Solution
 87cm2 Volume of cone = 1/3 r2 h
= 1/3 x 3.142 x 42 x 6
= 100. 54cm3
249
1992/14 Exercise 21.4 OM 1995/27 OM
The angle of a sector of a circle of radius 8cm is Find the total surface area of a solid circular cone with base
2400.This sector is bent to form a cone. Find the radius radius 3cm and slant height 4cm
of the base of the cone. (Take  = 22 /7 )
A. 16/3cm B. 15/3cm C. 16/5cm A 37 5/7cm2 B 66cm2 C75 3/7cm2
D. 8/3cm E. 16/10cm D. 78 2/7cm 2
E. 88cm 2

Solution
1992/16 OM Total surface area of a cone =  r (r + L)
The volume of a cone of height 9cm is 1848cm3. r =3cm, L = 4cm
Find its radius (Take  = 22 /7 ) Substituting,
A. 7cm B. 14cm C. 28cm D. 98cm E. 196cm Total surface area of a cone = 22 x 3 ( 3 + 4 )
Solution 7
Volume of cone = 1/3  r2 h = 22 x 3 x 7
1848 = 1 x 22 x r2 x 9 7
3 7 = 66cm2 ( B )
1848 x 3 x 7 = r2
22 x 9 1995/31 Exercise 21.5 OM
196 = r2 A 2100sector of a circle of radius 21cm is bent to form a
196 = r i.e 14cm (B) cone. What is the base radius of the cone ?
A. 3 ½ cm B. 7cm C. 10 ½ cm D. 14cm E. 12cm
1993/15 OM
Calculate the total surface area of a cone of height 12cm 1997/17 OM
and base radius 5cm (Take  = 22 /7 ) Calculate the total surface area of a solid cone of slant height
A.180 5/7cm2 B. 240 2/7cm2 C. 235 5/7cm2 15cm and base radius 8cm in terms of 
D. 282 6/7cm 2
E. 251 3/7cm2 A. 64cm2 B. 120 cm2 C. 184 cm2
Analysis and Solution D. 200 cm E. 320 cm2
2

Total surface area of a cone Solution


= area of circular base + curved surface area Total surface area of a cone =r(r + L)
= r2 +  rL r = 8cm, L = 15cm
= r (r + L) Total surface area of cone =  x 8 (8 + 15)
r = 5cm, L = ? =  x 8 ( 23 )
But L 2 = h2 + r2 = 184 cm2 (C)
L2 = 122 + 52
L = 144 + 25 1998/47 Exercise 21.6 OM
L2 = 169 Find the curved surface area of a cone of radius 3cm and
L = 169 = 13cm slant height 7 cm (Take  = 22 /7 )
A. 22cm2 B. 44cm2 C. 66cm2 D. 132cm2 E. 198cm2
Substituting, we have
Total surface area of cone = r (r + L) 2001/43 Exercise 21 .7 OM
= 22 x 5 (5 + 13) The height of a right circular cone is 4cm. The radius of the
7 base is 3cm. Find its curved surface area.
= 1980 = 2826/7cm (D) A. 9 cm2 B. 15cm2 C. 16cm2 D.20cm2
7
2002/27 Exercise 21 .8 OM
1993/16 OM The base diameter of a cone is 14cm, and its volume is
A cone is 14cm deep and the base radius is 4½ cm. 462cm3. Find its height. ( Take  = 22 /7 )
Calculate the volume of water that is exactly half the A. 3. 5cm B. 5cm C. 7cm D. 9cm
volume of cone. ( Take  = 22 /7 )
A. 49.5cm3 B. 99cm3 C. 148. 5cm3 2002/37 OM
D. 297cm 3
E. 445. 5cm What is the total surface area of a solid right circular cone of
Analysis and Solution base radius r cm and height r cm ?
Volume of cone = 1/3 r2h A. 2r2 cm2 B.  r2 ( 1 + 2 ) cm2 C. 7/3 r2 cm2
= 1 x 22 x (4.5)2 x 14 D. 4/3  r2 cm2
3 7 Solution
Volume of cone = 297cm3 Total surface area of cone =  r (r + L)
Half volume of cone = 297 = 148. 5cm3 (C) r = given but L, is not except h
2 Recall that L2 = h2 + r2
Since h = r cm
L2 = r2 + r2
250
L2 = 2r2 ii) Total surface area
L = 2r2 cm = Curved surface + circular surface area
= 2 r2 + r2
L = r 2 cm = 3r2
Substituting, we have
Total surface area of cone = r ( r + r 2 ) iii) Volume of hemisphere
= r2 (1 + 2 )cm2 (B) = ½ ( 4/3 r3 )
= 2/3  r3 (half the volume of sphere)
2002/37 Neco Exercise 21.9 OM
A cone was developed from the sector of a circle with 1990/26 OM
radius 10cm and an angle at the centre of 1080. What is Calculate the surface area of a sphere of radius 7 cm
the radius of the cone? ( Take  = 22 /7 )
A. 9. 0cm B. 8.5cm C. 4.3cm D. 3.0cm A. 86cm2 B. 154cm2 C. 616cm2
2 2
D. 143 cm E. 4312 cm
VTR – 13/4b NTI TCII Exercise21.10 OM Solution
A sector of a circle radius 7cm subtending an angle of Surface area of sphere = 4r2
1080 at the centre of the circle is used to form a cone = 4 x 22 x 72
(i) Find the base radius of the cone 7
(ii) Calculate the area of the base of the cone, = 616 cm2 ( C )
correct to the nearest centimetre (Take  = 22/7)
1990 / 6b OM
VTR –13/42 NTI TCII Exercise 21.11 OM A hemispherical tank of diameter 10m is filled by water
Find the volume of a cone whose base diameter is 12cm issuing from a pipe of radius 20cm at 2m per second.
and height 9cm (leave your answer in ) Calculate, correct to three significant figures, the time in
A.36  cm3 B.108 cm3 C.324 cm3 minutes, it takes to fill the tank
D.432 cm3 E. 1296cm3 ( Take  = 3. 142 )
Solution
2004/23 NABTEB Exercise 21.12 OM
A solid cone has base radius 3cm and height 4cm. 10m
Calculate the total surface area.
A.9 cm2 B.15 cm2 C.21 cm2 D.24 cm2

SPHERE

.
Volume of hemispherical tank = ½ ( 4/3 r3 )
= 2/3  r3
r
r = diameter = 5m
2
= 2/3 x 3. 142 x 53
= 261. 8m3
Volume of water per second =  r2 h / sec flowing from the pipe
(I) Surface Area = 4r2 h is 2m, r = 20cm is 0. 2m
(II) Volume = 4/3  r3 = 3. 142 x (0.2)2 x 2
= 0. 2514m 3/ s
HEMISPHERE Time taken in sec = V tank
V / sec of pipe
r = 261. 8
0. 2514
= 1041. 4 sec.
Time taken in minutes = 1041.4
60
= 17.35 minutes.
 17. 4 minutes to 3. s. f
i) Curved surface area = ½ (4r2)
(Half surface area of sphere) 1994/32 OM
= 2r2 Two solid spheres have volume 250 cm3 and 128 cm3
respectively. Find the ratio of their radii
A. 125 : 64 B. 25 : 16 C. 2 : 1 D. 4: 3 E. 5: 4
251
Solution Solution
Volume of sphere = 4/3  r3 Then To find volume, we need r
Thus, surface area of sphere = 4 r2
250 = 4/3  r3 and 128 = 4/3  r3 616 = 4 r2
250 x 3 = r3 and 128 x 3 = r3 616 = r2
4 4 4
59. 68 = r3 and 30. 55 = r3 r2 = 616 = 49. 01m
r = 3. 91 cm and r = 3. 13 cm 4 x 3. 142
Approximately to the nearest cm r = 49. 01 = 7m
r = 4 and r = 3 Hence, volume of sphere = 4/3 r3
Thus, their ratio is 4: 3 (D) = 4/3 x 3. 142 x 73
 1, 400.m3 (C)
1995/28 OM Note
A hollow sphere has a volume of Kcm3 and surface area Inconsistency in unit above was from Neco not Author.
of Kcm2. Calculate the diameter of the sphere
A. 3cm B. 6 cm C. 9 cm d. 12 cm 2001/50 Neco Exercise 21. 13 OM
E. more information is needed. The radius of a sphere is 21 cm if S and V represent the
Solution curved surface area and volume respectively,
2 x radius = diameter what is S: V ?
Volume of sphere = 4/3  r3 A1:3 B1:7 C 3: 1 D 3 : 7 E 7 : 1
K = 4/3  r3
3K = r3 ------------(1) 1998/27 (Nov) Exercise 21.14 OM
4 A sphere has a surface area of 4312cm2. Calculate the radius
Also, of the sphere correct to 1 decimal place.
Surface area of sphere = 4  r2 (Take  = 22/7)
A.18.0cm B.18.5cm C.19.0cm D.19.5cm E.58.5cm
K = 4  r2
K = r2 ------------(2)
4 RIGHT TRIANGLE PRISM
Divide eqn (1) by (2) A prism is a solid with uniform cross sectional area.
3K  K = r3
4 4 r2
3K x 4 = r
4 K ht
r = 3cm, Diameter = 2 x r  2 x 3 = 6cm (B) heig
a c
2001/13a OM
A boy blew his rubber balloon to a spherical shape. The
balloon burst when its diameter was 15 cm. Calculate b
correct to the nearest whole number. The volume of air
in the balloon at the point of bursting. Volume = base area x height = 1/2 ab x h
Solution
Relationship between a, b and c is that of Pythagoras’ rule
Volume of sphere = 4/3  r3 c 2 = a 2 + b2
Radius = diameter = 15/2 cm = 7. 5cm
2
Cross sectional area = 1/2 a b
Volume of air at the point of bursting
= Volume of the spherical shape formed by the balloon
= 4 x 22 x (7. 5)3
3 7 a c
= 1767 . 86 cm3
 1768. 00 cm3 to the nearest whole number.
b
2001/47 Neco OM
The surface area of a sphere is 616m2, what is the Note
volume of the sphere (correct to two significant figures) C is not used; since the area of a triangle
(Take  = 3. 142) = 1/2 base x height (perpendicular height)
A 14 cm3 B 210 cm3 C 1, 400 cm3 = 1/2 a x b
3 3
D 2, 100 cm E 4, 300cm

252
1997/18 OM 48. What is the volume of the prism
The cross – section of a prism is a right – angled A 28 cm3 B 42 cm3 C 70 cm3 D 84 cm3
triangle 3cm by 4 cm by 5cm. The height of the prism is Solution
8 cm. Calculate its volume Volume of prism = base area x height
A. 48cm3 B. 60 cm3 C. 96 cm3 =6 x7
3 3
D. 120 cm E. 240 cm = 42 cm3 (B)
Solution 2002/47 Neco OM
Since the cross section is that of a right – angled What is the volume of the right triangular prism below.
triangle. The hypotenuse is 5 cm – as it is longer.
Thus the sketch is

13c
m
5cm

16cm
4cm

10cm
8cm
3cm
A 1040. 00cm3 B 999.00 cm3 C 870. 00cm3
Volume of prism = base area x height D. 800.00 cm3 E. 665 . 00cm3
= cross sectional area x height Solution
Volume = base area x height
= 1/2 x 3 x 4 x 8
But; base area = 1/2 base x height
=1x3x2x8
= 48 cm3 (A) = 1/2 a x b But a is not given.
By Pythagoras rule
Note 132 = 102 + a2
5 cm is not useful here since the base area is that of a a2 = 132 - 102
triangle which is given as = 169 – 100 = 69
= 1/2 base x height a =  69 = 8. 307 cm
Base = 3 cm and height = 4cm Thus
Volume = 1/2 x 8.307 x 10 x 16
= 664. 56 cm3  665. 00 cm3 (E)
2001/47 – 48 OM
R Pyramid on a square base
7cm 5cm Pyramid is a solid shape with triangular faces meeting at a
3cm

point (vertex).The shape lies on a base, which may be square,


triangle, rectangle e t c We shall concern ourselves with
Q P square base pyramid.
Vertex
Sl
an
th
eig

The diagram shows a triangular prism of length, 7cm.


ht

The right – angled triangle PQR is a cross – section of h


the prism /PR/ = 5cm and /RQ/ = 3 cm.
Use the information to answer question 47 and 48

47. What is the area of the cross- section ? Base L


A. 4 cm 2 B. 6 cm2 C.15 cm2 D. 20 cm2 L
Solution Volume = 1/3 area of base
Area of cross – section = 1/2 ab x height
Here a is given but b is not. = 1/3 L2 x h
Thus applying Pythagoras rule Since, the area of a square = L x L
52 = 32 + QP2 = L2
/QP/2 = 52 – 32
= 25 - 9 = 16 1999/33 OM
QP = 16 = 4 i.e b = 4cm The height of a pyramid on a square base is 15 cm. If the
Hence volume is 80cm3, find the length of the side of the base
Cross – sectional area = 1/2 x 3 x 4 A. 3. 3cm B.5. 3cm C. 4. 0cm D. 8. 0cm
= 6cm2 (B)
253
Solution Volume = 1 x 42 x 2
Volume = 1/3 area of base x height 3
= 1/3 L2 x h = 32 cm = 10 2/3 cm3 (B)
3
80 = 1/3 L2 x 15
80 = L2 x 5
2 2002/39 (Nov) Exercise 21.15 OM
L = 80 = 16
A pyramid on a square base of side 10cm has a height of
5
15cm. Find its volume
L = √16 = 4cm (C)
A. 150cm3 B. 500cm3 C. 1500cm3 D. 5000cm3
2000/42 OM
1999/4b (Nov) Exercise 21.16 OM
The height of a pyramid on a square base is 15cm. If
The base of a pyramid is a 12cm by 12cm square. If its
the volume is 80cm3, find the area of the square base
height is 20cm, find the volume of the pyramid.
A. 8cm2 B. 9. 6cm2 C.16cm2 D. 25cm2
A.80cm3 B.960cm3 C.1440cm3 D.1600cm3
Solution
Volume = 1/3 area of base x height
80 = 1/3 L2 x 15 Cube
80 = 5 x L2 A cube is a solid with six faces of equal dimension
L2 = 80 = 16 A B
5
L = √16 = 4cm D
C
Base area = L2
= (4)2 = 16cm2 ( C ) P x
O
x R
Q x
2001/42 OM
A right pyramid is on a square base of side 4cm. The
slanting side of the pyramid is 2√3cm. Calculate the Surface area (s) = 6 x area of one face
volume of the pyramid. = 6x2
A. 5 1/3cm3 B. 10 2/3cm3 C. 16cm3 D.32cm3 x is the length of the side
Solution Volume = base area x height
= x2 X x
= x3
Length of diagonal QB = x2 + x2 + x2
2 3

h
= 3x2
cm

A = x√3
4c m
E
Length of side diagonal = x2 + x 2

4 cm
= 2x2
B = x√2

2001/45 Neco OM
1
Volume = /3 base area x height If the volume of a cube is (8 x 103 ) cm3.
We need to know height , h What is the surface area of the cube ?
A. 2,400cm2 B. 1, 600cm2 C. 120cm2
2
(AB)2 = 42 + 42 D. 124cm E. 16cm2
= 16 + 16 = 32 Solution
AB = √32 Surface area of cube = 6x2
1
/2 AB = EB = √32 X (length) is not given but
2 Volume = 8 x 103
Also by Pythagoras rule x3 = 8 x 103
(2√3) = h2 + (EB)2 x = (8 x 103 ) 1/3
h2 = (2√3)2 - √32 2 = 20cm
2 Thus,
= 4 x 3 – 32 Surface area = 6x2
4 = 6 (20)2
= 12 – 8 = 4 = 6 x 400
h = √4 i.e 2 = 2,400cm2 (A)

254
2002/31 Neco Exercise 21.17 OM Solution
What is the volume of a cube if the diagonal of one of Total surface area of a cuboid
the sides is √50 cm ? = 2xy + 2xz + 2zy
A. 7.07cm3 B. 50. 0cm3 C. 125 . 0cm3 x = 12cm, y = 8cm, z = 3cm
3 3
D. 353. 4cm E. 625.0cm = 2 (12 x 8) + 2 (12 x 3) + 2 (8 x 3)
= 192 + 72 + 48
2004/46 Exercise 21.18 OM = 312cm2 (A)
The sides of two cubes are in the ratio 2:5. What is the
ratio of their volumes ? VTR – 10/42 NTI TCII OM
A. 4:5 B. 8:15 C. 6:125 D. 8:125 Calculate the capacity in litres of a rectangular tin 20cm by
20cm by 10cm
2001/22 NABTEB Exercise 21.19 OM A. 1 litre B.2 litres C. 3 litres D. 4 litres E.5 litres
Find the total surface area of a cube of side 8cm. Solution
A.64cm3 B.192cm3 C.364cm3 D.384cm3 To find the capacity of any (shape) first we find its volume in
cm3 then, convert to litres.
CUBOID 1000 cm3 = 1litre
A cuboid is a solid shape with a rectangular base and Volume of cuboid = length x breadth x height
sides. = 20 x 20 x 10
= 4000cm3
Capacity is = 4000 litre
1000
= 4 litre (D)

VTR – 13/43 NTI TCII Exercise 21.20 OM


A tank of 5m by 4m by 3m is filled with water. Calculate the
amount of water it can hold in litres
It consists of 6 rectangles;. of which 2 opposite ones (Take 1m3 = 1000 litre)
are the same in area. Thus , A. 60 litres B.600 litres C.650 litres
Surface area = 6 rectangles D.6,000 litres E.60,000 litres
= 2xy + 2xz + 2yz
Volume = Base area x height
= xy x z 2003/59 Neco OM
The volume of a box of height 8cm is 320cm3. What is the
= xyz
Alternatively,
volume of a similar box of height 6cm ?
Volume = Length x breadth x Height A. 1920cm3 B.240cm3 C.210cm3 D.135cm3 E.48cm3
Solution
2002/32 Neco OM Volume of cubiod = length x breadth x height
= Base area x height
The length, breadth and height of a cuboid are 8cm, 7 ½
The two boxes have same Base area but differs in height
cm and 6 ½ cm respectively, What is its volume in m3 Thus, from the 1st box
(leave your answer in standard form)? Volume = base area x height
A. 3. 9 x 10 – 1m3 B. 3. 9 x 10– 2m3 C. 3. 9 x 10+2 m3 320 = Base area x 8
D. 3. 9 x 10 – 4m3 E. 3. 9 x 10 –5m3 320 = Base area
Solution 8
Volume of cuboid 40cm2 = Base area
= Length x breadth x height 2nd Box volume = Same Base area x 6.
L = 8cm = 8/ 100m = 40 x 6
Breath = 7 ½ cm = 15/ 200m = 240cm3 (B)
Height = 6 ½ = 13/ 200m
Volume = 8 x 15 x 13 2004/49 (Nov) OM
100 200 200 A rectangular tank, 2.25m long and 1.6m wide contains
= 8 x 15 x 13 x 10 – 6 2800 litres of water. Find, correct to the nearest cm, the depth
2 2 of water in the tank.
= 390 x 10 – 6 = 3. 9 x 10 – 4m3 (D) (Take 1000cm3 = 1 litre)
A.76cm B.78cm C.770cm D.780cm
2002/39 Neco OM Solution
What is the total surface area of a cuboid whose Depth here is the Height in the formula for volume of cuboid.
dimensions are 12cm x 8cm x 3 cm And capacity 2800 litres is volume in litres. Thus
A. 312cm2 B. 96cm2 C. 36cm2 Volume of cuboid = length x breadth x height
2 2
D. 24cm E. 12cm Capacity = 2800 litres
255
Same as = 2800 x 1000cm3 x 12
Length = 2.25m i.e 2.25 x 100cm 6x = 12 (x – 12 )
Breadth (width) 1.6m is 1.6 x 100cm 6x = 12x – 144
Height (Depth) is be solved for 144 = 12x – 6x
Substituting 144 = 6x
2800 x 1000 = 2.25 x 100 x 1.6 x 100 x H x = 144/6
Height (Depth) = 2800 x 1000 = 24cm (D)
2.25 x 100 x 1.6 x 100
= 280 2001/10b Neco Exercise 21.23 OM
3.6 = 77.78  78cm (B) A frustum of a cone is cut out with diameters 6cm and 18cm
at the top and bottom respectively and its height is 15cm.
2003/21 NABTEB Exercise 21.21 OM Calculate the height of the larger cone,
A cuboid is a prism whose uniform cross – section is a
A. square B triangle C. rectangle Volume of frustum
D. rhombus = 1/3h ( R2+ Rr + r2 ) OR = 1/3h R3 - r3
R- r
2001/23 NABTEB Exercise 21.22 OM
The dimensions of a cuboid are 3m by 2m by 5m. What 2005/9b (ii) ( Nov ) OM
is its volume ?
A.30m3 B.32m3 C.40m3 D.62m3

Frustum
This is a shape obtained by cutting off the upper portion
of a pyramid or a cone.
E.g A bucket is a frustum of a cone.

1995/29 OM

The diagram shows a frustum, which is part of a solid cone.


6cm The radii of the top and bottom are 6cm and 8cm respectively.
Calculate the:
(ii) volume of the frustum, correct to the nearest whole
number.

Solution
Volume of a frustum = 1/3h ( R2+ Rr + r2 )
Here h is10, r is 6 and R is 8. Substituting, we have
= 1 x 22 x 10 ( 82+ 8 x 6 + 62 )
The diagram above shows a cone with the dimensions 3 7
of its frustum indicated. = 1 x 22 x 10 ( 64+ 48 + 36 )
Calculate the height of the cone. 3 7
A. 12cm B. 15cm C. 18cm D. 24cm E. 30cm = 1 x 22 x 10 x 148
Solution 3 7
Applying ratio of similar triangles, we let cone height = 32560
be x as shown below 21
= 1550.476cm3
 1550 cm3 to the nearest whole number

Xcm
6cm
12cm

12cm
Thus,
x - 12 =6
256
IRREGULAR SOLIDS Solution
Irregular solids do not have any particular formula used; Vol of solid = vol of cone + vol of hemisphere
we apply the relevant solid formula as it affects a given = 1 r2h + 2 r3
shape. The use of common sense in interpreting 3 3
diagrams is crucial so as to know the applicable formula r = diameter  2 i.e. 6  2 = 3 cm
h = 10 cm – 3 cm i.e. 7cm
19989/3 OM Substituting,
Volume of solid = 1 x 22 x 32 x 7 + 2 x 22 x 33
3 7 3 7
= 22 x 3 + 2 x 22 x 32
7
= 66 + 396
7
= 462 + 396
7
= 858 = 122 4/7 cm3
7

1986/11 GCE Exercise 21.24 OM


` The diagram represents the cross section of a solid in the
form of a cone of height h, and a hemispherical bowl of
The diagram above shows a wooden structure in the radius r. The volume of the hemispherical portion is half that
form of a cone, mounted on a hemispherical base. The of the conical portion
vertical height of the cone is 24cm and the base radius
7cm, Calculate, correct to three significant figures the
surface area of the structure. (Take  = 22/7 )
Solution h
Surface area of solid
= surface area of cone + surface area of hemisphere r r
= rL + 2r2 O
But slant height (L)2 = 72 + 242
= 49 + 576
L2 = 625
L = √625 = 25cm
Hence
Surface area of solid Calculate:
= 22 x 7 x 25 + 2 x 22 x 72 (a) the ratio r : h
7 7 (b) the vertical angle of the cone correct to the nearest
= 550 + 308 degree;
= 858cm2 (c) the total volume, correct to three significant figures, of
the solid given that r = 3.5 cm.
2002/5a NABTEB OM (Take  = 3.142)
A solid consists of a cone attached to a hemisphere as
shown in the diagram below. 2004/45 Neco (Nov) Exercise 21.25 OM
A solid shape is formed from a combination of a cone and
hemisphere such that the height of the solid is 6 cm. If the
radius of the hemisphere is 11/2cm. Calculate the volume of
the solid correct your answer to 3 significant figures.
A.17.6 cm3 B.17.679 cm3 C.17.68 cm3
3 3
D.17.7cm E.17.78cm
10cm

Calculate the volume of the solid if the diameter of the


hemisphere is 6cm ( Take  = 22/7)

257
1997/9 OM (a) The volume;
(b) The total surface area of the solid
( Take  = 22/7 )
Solution
( a ) Volume of solid = base area x height
Height = 15cm, r = 7cm
Base area = Area of the major sector
= _ x  r2
360
where  = 360 – 120
= 2400
Volume of solid
The solid above is a cylinder surmounted by a = 240 x 22 x 72 x 15
hemispherical bowl. Calculate its 360 7
(a) total surface area = 22 x 7 x 5 x 2
= 1540.0cm3 to 1 dp
(b) volume (Take  = 22/7 )
Solution
( b ) Total surface area
( a )Total surface area of solid
= Total surface area of 2400 of the remaining solid
= Surface area of hemisphere + surface area of cylinder
360 cylinder
= 2r2 + 2 rh +  r2 plus that of equal rectangles ABQP and BCRQ
= 2r (r + h ) + r2 where r = 7cm; h = 10cm = 2 [ r (2r + 2h) ] + 2 (15 x 7)
= 2 x 22 x 7 (7 + 10) + 22 x 72 3
7 7 = 2 [ 22 x 7 (14 + 30) ] + 210
= 2 x 22 x 17 + 22 x 7 3 7
= 748cm2 + 154cm2 = 2 (968) + 210
= 902cm2 3
( b ) Volume of solid = 645.33 + 210 = 855.33cm2  855.3cm2 to 1d.p
= Volume of hemisphere + volume of cylinder
= 2/3  r3 +  r2 h 2002/5 Neco (Nov) Exercise 21. 26 OM
=  r2 ( 2/3 r + h) A solid is made up of a cylinder of height 20cm and diameter
= 22 x 72 x ( 2/3 x 7 + 10) 14cm with a cone of slant height 10cm on top as shown in
the diagram below. Calculate the total surface area of the
7
solid
= 22 x 7 ( 14 + 10)
3 (Take  = 22/7 )
= 22 x 7 x 44 = 6776
3 3
= 2258 2/3 cm3
m
10c
2002/9 OM

B 7cm
C
120
0

A
20cm
15cm

Q 14cm

120
0
R
P A 2816 cm2 B 2200 cm2 C 1254 cm2 D 1100 cm2
The diagram is a portion of a right circular solid E 800 cm2
cylinder of radius 7cm and height 15cm. The centre of
the base of the cylinder is Q, while that of the top is B,
where  ABC = PQR = 1200.
Calculate, correct to one decimal place;

258
Chapter Twenty two I: With center B and a suitable radius draw an arc to cut AB
at Q and BC at P
Construction A
Foundation facts on construction A
To construct any given shape say triangle, we need
A compass, protractor, sharp pencil, very neat ruler
and a good eraser. All construction lines must not be Q
thick. Do not erase any arc or guiding lines to your final Q
result. Always confirm your lines or arc measurements
with ruler or compass as the case may be B
B P C P
Construction of angle 900 at a given point C C
Draw a line AB Where this arc already exist maintain it to step II
A B II: With centers at “P” and “Q” each at a time; using the
same radius, draw arcs to intersect at point y.
With center at the point C, on line AB open your A
compass to a suitable radius, cut arc twice on line AB at
points E & F A
y Q y
A B
C
E F Q
B
Changing or adjusting our compass to a bigger radius P
wider than CF or CE. Next, make E and F our centers, B C C
P
one at a time draw an arc to cut each other at top of C as
shown below: III: Join B to Y. This is the Bisector
G
A

A
y Q
y
A B
Q
E C
F B
Join point of intersection G to line AB at C, we have P
y
GCB = 900 and GCA = 900 as shown below. B P C C
Confirm it with the use of a protractor.
G Construction of 600
Step I : Draw line AB
A B

Step II: With center at A and a suitable radius draw


an arc XY to cut AB at Y
A B
X
C
E F

To bisect any given angle ABC A Y B


The angle could be Fig I or Fig II Step III : With center at Y and same radius as before, draw
A an arc to cut XY at Z
A
Z
Fig I Fig II
X

B
B C A Y B
C

259
Step IV: Join AZ . Angle ZAB = 600 (b) With center X and radius 11 cm draw an arc
(c) With center Y and radius 6 cm draw an arc to
intersect the previous one at Z
Lastly, line ZD as instructed : with centre X and radius 4.5cm
Z draw an arc to cut XY at D. Then join D to Z
X
To construct a triangle, given two sides and
one of its angles
A Y B 2002/5 Delta
Using a ruler and a pair of compasses only,
Construction of 1200 (a) Construct ∆ PQR such that PQ = 6.5cm,
We use same steps as in 600, but for step IV
QR = 8 cm and  PQR = 450
Do not join A Z Rather,
Step IV: With same radius as before and center Z draw (b) Measure  QPR
another arc to cut XY at W. Solution Diagram on page261.
Z Steps taken are:
W (i) Draw QR = 8 cm
(ii) At “Q” construct angle Q = 450 i.e bisecting of 900 and
X measure out line QP = 6.5cm through it to P
(iii) With center Q and radius 6.5cm draw an arc to cut QP.
A Y B (iv) Join P to R
Step V: Then join AW. Angle WAB = 1200
2000/5 Delta
Z Using a ruler and a pair of compasses, construct a ∆PQR
W such that PQ = 9 cm, PQR = 600 and QR = 10 cm
Measure (i) PR (ii) QPR
X Solution Diagram on page 261
A Y B For ease in measuring  QPR and constructing
 PQR = 600 ; we let Q and P be on the floor.
Construction of triangles with given length of Steps taken:
the three sides (No included angles) (a) Draw side PQ = 9 cm
The steps we follow here are the same except for (b) At point Q, construct Q = 600
change in length based on the question. (c) Measure out QR = 10 cm
Sketching is an important part of construction; so (d) Join PR
always remember to do so before the construction
proper. 2004/1 FGC ( JSS Neco ) Exercise 22.1
Construct ∆ ABC in which AB = 9cm, BC = 12cm
1997/5a Delta
Construct a triangle AB with /AB/ = 6cm, and ABC = 600
( i ) Construct the bisector of angle A and let it
/BC/ = 8cm and /AC/ = 10 cm
Measure angle ABC meet BC at D.
( ii ) Measure DC
Diagram on page 261
Solution Construction of a triangle, given two angles
(a) Draw line AB = 6 cm and one side
(b) With center B and radius 8 cm, draw an arc
(c) With center A and radius 10 cm, draw an arc to 1997/5b Delta
intersect the previous one at C. Construct a triangle PQR with /PQ/ = 10 cm
Angle ABC = 900 using a protractor to measure it.  RPQ = 300,  PQR = 600. Measure PR.
1994/5 Delta Solution Diagram on page 261
Construct a triangle XYZ in which XY =10 cm,YZ =6 cm Our Sketch must be reasonable enough to allow us construct
and ZX = 11 cm. On XY mark a point D such that XD the two given angles P and Q on the floor.
= 4.5 cm. Draw ZD, measure the length of ZD. Steps taken are:
Solution Diagram on page 261 (a) Draw PQ = 10cm
We have something to do with XY, So we let it lie on (b) At P construct angle P = 300 i.e bisecting of 600
the floor . Steps: (c) At Q construct angle Q = 600
Draw a line XY = 10 cm (d) The point of intersection of these lines is R.

260
1994/5 Delta 1997/5a Delta C
Z

/ZD/ = 7.6cm o
ABC = 90

X D Y

A B
P
2002/5 Delta
R

o
QPR = 60
2000/6 Delta

/PR/ = 9.5cm
QPR = 65o

Q R

R
P Q

/PR/ = 8.6cm

P 1997/5b Delta (adjusted) Q

261
2004/5 Delta Exercise 22.2 For emphasis sake we will note a point P on line AK i.e
Construct ∆XYZ such that X = Z = 450 and XZ = 8 cm where the line touches the arc YZ
(b) Measure: (i) side XY and (ii) angle Y 4 : Using same radius and with centers at P and Y bisect
BAK as shown below
2004/3 Osun Exercise 22.3
Using a pair of compasses and a ruler only , Z
K
construct a triangle PQR in which /QR/ = 10cm , X P
PQR = 600 and  PRQ = 300 W
( i ) measure /QP/ ( ii ) measure /PR/
( iii ) measure QPR A Y B
( iv ) What type of triangle is PQR 5 : Join line AW and we have that  BAW = 150

SPECIAL ANGLES, AND LOCUS Z


K
Under this sub topic we shall treat construction of X P
150, 750(60+15),1050(90 +15),or 1050(60+30+15), W
1350(120+15), or 1350(90 + 45),1500(120 +30).
A Y B
Construction of 150
Note that KAW is also 150
When we bisect angle 600, we get 300.Further bisecting
Readers to bisect KAZ = 300 at step5 to give 150 each as
of 300 gives 150
shown below
Steps taken :
1: Using the steps given earlier on the construction of
angle 600, we have: Z 15o
Z K
15o
X

A Y
A Y B
Most of the special angles is based on one or two
manipulations of the above bisectings

2:Using same radius and centers Z and Y bisect Construction of 750


ZAY as shown below: Construction of 750 is basically construction of two 600
Z (1200) and bisecting of one angle 600 to give 300 then further
K
X bisecting to give 150
Steps taken:
1:We construct two angles 600 (or 1200 showing two arcs)
A Y B D A

3 : Join AK. Angle BAK is 300 .Also angle ZAK is300

Z B C
K
X
ABC = 600 and ABD is the other 600 to be bisected

2 : Using same radius we bisect angle ABD = 600 at E to


A Y B give 300. Also join EB. Note that centers here are D and A
E
We are to either bisect angle BAK or ZAK
To make the work neat is always good to use an already
made arc in the construction. The visible one here is arc D
A
YZ.If you have adjusted your compass before; ensure
that you bring it back to the said arc YZ with centre at
A on line AB.
B C

262
3 : Using same radius, we bisect angle ABE = 300 at F Construction of 1350
to give 150 . Centres here are A and the point of 1350(120 +15), or 1350(90 + 45)
intersection between BE and arc AD
F 1350(120 +15) case,
E First we construct angle 1200 then bisect the last part
X Z
C

D
A
D A B

Secondly, we bisect XAC = 300 to give 150


B C
FBC = 750
ALTERNATIVE STEP 3 X Z
From step2, using same radius we can bisect angle C
EBD = 300 to get 150(With centers at D and point of o
135
intersection between line EB and arc AD)
N
E D A B
We can choose to bisect ZAB to give 300 further
bisecting to get 150
D
A 1350( 90 + 45 )
First we construct angle 900at A

M B C
MBN = 750

Construction of 1050
Construction of two 600 (1200) and bisecting of one
angle 600 to give 300 each .The last of the 300 is
bisected to give 150
so we have 600 +300 +150 = 1050 D A B
Steps taken are as shown above in construction of 750
F Secondly, we bisect one part to get 450
E C

D
A

D A B
Y B C CAB = 1350
YBF = 1050 OR CBN = 1050 shown below:
N
E Construction of 1500
1500(120 + 30).
We construct angle 1500 by bisecting the third part in 1200
D
A C

o
1
50
M B C
D A B

263
LOCUS
Locus has plural form loci. It can be defined as a set of Secondly, with radius AB but centre at C cut an arc as ;
points satisfying or obeying a given condition (rule)
There are basically three loci
1.LOCUS OF A POINT (IN A PLANE) WHICH IS C
EQUIDISTANT FROM A GIVEN POINT.
It is a circle at that point with radius equal to the fixed
distance
2. LOCUS OF A POINT (IN A PLANE) WHICH IS
EQUIDISTANT FROM TWO GIVEN POINTS.
It is the perpendicular bisector of the line between those A B
two points
3. LOCUS OF A POINT (IN A PLANE) WHICH IS Thirdly, with radius AC but centre at B draw an arc to cut the
EQUIDISTANT FROM TWO INTERSECTING first one. Join CD; CD//AB as required
STRAIGHT LINES.
It is the bisector of the angle between them
C
In subsequent examples we shall utilize the above locus D
as directed by the given questions

PARALLEL LINES
There are basically two ways of constructing parallel
lines namely:
Case of parallel line to given line at a fixed A B
distance apart.
Firstly we draw two arcs of almost the size of semi LINE RATIO CONSTRUCTION
circle on the line. It’s radius equal the given distance Find by a suitable construction, a point B on AC such
say 3cm that /AB/ : /BC/ = 3:5 .
Steps taken
1 : Draw a line AK within a reasonable angle AC
2 : Divide AK into 8 reasonable segments
3 : At D draw parallel line to KC using the steps for a
parallel line at point outside a given line

A B
Secondly, we draw a straight line CD on the arcs as C
shown below: K
C D

A B
B
Case of parallel line to given line at a fixed point D
outside the line
First, we are given a line and a point outside the line
C

A
A B

264
CONSTRUCTION OF ORDINARY TRIANGLE I 2004/12 Neco ( Nov ) OM
1989/4 OM Using a ruler and a pair of compasses only ,
Using a ruler and a pair of compasses only. Construct a (a) Construct :
triangle ABC, given that /AB/ = 8.4cm /BC/ = 6.5cm ( i ) triangle ABC such that /BC/ = 10cm, ABC = 600
and ABC = 300. Construct the locus: and ACB = 450
( a ) l1 of points equidistant from AB and BC and (ii) the locus l1 of points equidistant from B and C .
within the angle ABC (iii) the l2of points 4cm from A .
( b ) l2 of points equidistant from B and C. (b) Find the points of intersection, X and Y, of the loci l1
Locate the point of intersection P of l1 and l2 and l2 and measure /XY/.
Measure /AP/ Analysis and solution Diagram on page 266
Analysis and solution Diagram on page 266 A  with one side and two angles 600 & 450
A  with two sides and 300 angle Locus: l1 bisector of line BC
Locus: l1 bisector of angle b/w AB and BC ; l2 a circle with centre A, radius 4cm
l2 bisector of line BC Using free – hand, we sketch the required diagram showing
Using free – hand, we sketch the required diagram all the given data. Don’t wipe out or erase your sketch.
showing all the given data. Do not wipe out or erase Steps taken
your sketch. 1: Draw a line BC = 10cm with extensions as shown in
Steps taken the diagram
1: Draw a line BC = 6.5cm with extensions as shown in
2 :At point B construct 600 and Produce a line through it to
the diagram
a reasonable extent
2 :At point B construct 300 i.e bisecting of angle 600
3:At point C construct 450 i.e bisecting of 900 and
3 : Produce B to A; a straight line measuring 8.4cm
produce a line through it touching the first line at A
passing through angle 300
4 :Construction of locus l1; which is the perpendicular
4 : Join A to C to give the needed triangle.
bisector of line BC
5 : Construction of locus l1; i.e the bisector of angle 300
5 : Construction of locus l2;i.e a circle with centre A,
6 : Construction of locus l2; which is the perpendicular
radius 4cm
bisector of line BC
1988/11 Exercise 22.4 OM
2000/4 OM
Using ruler and a pair of compasses only ,
Using a ruler and a pair of compasses only ,
( a ) Construct
(a) construct  PQR such that /PQ/ = 7cm, /PR/ = 6cm
( i )  ABC such that /AB/ = 5cm, /AC/ = 7.5cm
and PQR = 600
and CAB = 1200
(b) Locate point M ,the mid- point of PQ .
( ii ) l1 of points equidistant from A and B
(c) Measure RMQ . ( iii ) l2of points equidistant from AB and AC which
Analysis and solution Diagram on page 266 passes through triangle ABC
A  with two sides and 600 angle ( b ) Label the point P where l1 and l2 intersect
Locus: Mid point of PQ i.e bisector of line PQ ( c ) Measure /CP/
Using free – hand, we sketch the required diagram
showing all the given data. Do not wipe out or erase 1994/8 Exercise 22.5 OM
your sketch. ( a ) Using a ruler and a pair of compasses only.construct:
Steps taken ( i ) a triangle XYZ in which /YZ/ = 8cm
1: Draw a line PQ = 7cm with extensions as shown in XYZ= 600 and XZY = 750 . Measure /XY/
the diagram ( ii ) the locus l1 of points equidistant from Y and Z
2 :At point Q construct 600 ( iii) the locus l2 of points equidistant from YX and YZ
3 : Produce Q passing through the angle 600 to a ( b ) Measure QY where Q is the point of intersection
reasonable extent of l1and l2
4 : With centre at P and radius 6cm draw an arc to cut
the line at R 2006/9 (Nov) Exercise 22.6 OM
5 : Join R to P to give the needed triangle. ( a ) Using a ruler and a pair of compasses only, construct :
6 : Construction of locus ; which is the perpendicular ( i ) a triangle ABC such that /AB/ = 10cm , ABC = 750
bisector of line PQ and /BC/ = 8cm
7 : To measure RMQ draw a dotted line as shown. ( ii )locus l1 of points equidistant from points A and B ;
( iii ) locus l2 of points equidistant from AC and AB .
( b ) ( i ) Locate N, the point of intersection of l2 and BC

265
1989/4 A
L2

/AP/ = 5.15cm L1

P 2000/4

B C

M
P Q

o
RMQ = 60

X
2004/12Neco ( Nov ) L2

B C

/XY/ = 7.5cm

L1
266
( ii ) Measure /AN/ . 1997/ 9 ( Nov ) OM
( C ) Measure /BK/ , where K is the point of Using a ruler and a pair of compasses only:
intersection of l1 and l2. ( a ) Construct triangle ABC such that /AB/ = 6cm
/AC/ = 8.5 cm and  BAC = 1200
CONSTRUCTION OF TRIANGLES II ( b ) locus l1 of points equidistant from A and B
( c ) locus l2 of points equidistant from AB and AC
2001/8 Neco OM
Using a ruler and a pair of compasses only: ( d ) Find the points of intersection P1 and P2 of l1and l2
(a) ( i ) Construct triangle ABC such that /AB/ = 4.5 cm and measure P1P2.
/AC/ = 7.5 cm  BAC = 1200 Analysis and solution Diagram on page 270
( ii ) Measure /BC/ A  with two sides and one angle 1200
( b ) Find by construction, the position of the point Q Locus: l1 bisector of line AB
which is equidistant from A and C and 4.5cm from C l2 bisector of the angle between AB and AC
( c ) Join Q to B and to C respectively Steps taken
( d )Measure 1: Draw a line AC = 8.5cm with extensions as shown in
( i ) /BQ/ the diagram
( ii ) BQC 2 :At point A construct 1200 and produce a line 6cm
Analysis and solution Diagram on page 268 through it to B
1 : Draw line AC = 7.5cm long with extensions as shown 3: Join B to C to complete the triangle
2 : At A construct angle 1200 4 : Construction of locus l1; which is the perpendicular
3 : Produce a line 4.5cm long from A to B through bisector of line AB
angle 1200 5 : Construction of locus l2; which is the perpendicular
4 : Join B to C to complete the triangle bisector of BAC and another bisector at A as shown.

5 : construction of bisector of AC and circle with centre at C


radius 4.5cm. Where they meet at the top is Q. 2004/ 10 NABTEB Exercise 22.7 OM
6 : From Q draw dotted lines to B and C as shown (a)Using a ruler and a pair of compasses only, Construct
( i ) ABC such that /AB/ = 7cm, /AC/ = 9cm
2001/13 (Nov) OM and  BAC = 1200
Using ruler and a pair of compasses only ( ii ) locus l1 of points equidistant from A and B
(a) construct :
( iii ) locus l2 of points 5cm from B.
( i )a triangle PQR such that /PQ/ = 7.5cm , /PR/ = 9cm
( b ) ( i )Find the points P1 and P2 of intersections of
and RPQ = 1350
l1and l2
( ii ) locus l1 of points equidistant from P and Q , ( ii ) Measure /P1P2/.
( iii ) locus l2 of points equidistant from Q and R ,
(b) Locate the point S where l1and l2 intersect . 2006/9 NABTEB ( Dec ) Exercise 22.8 OM
(c) join SR . Using a ruler and a pair of compasses only , construct a
(d) Measure : ( i ) SR ( ii ) PQR. triangle PQR in which PQR = 300, /PQ/ = 7cm
and /PR/ = 8cm.
Analysis and solution Diagram on page 268
(a) Construct a locus L which is always 5cm from the
A  with two sides and one angle 1350
point P and which intercepts PQ and PR at M and
Locus: l1 bisector of line PQ N respectively.
l2 bisector of line QR .(b) What type of shape is MNRQ ?
Steps taken (c) Construct a line QX the shortest distance from
1: Draw a line PQ = 7.5cm with extensions as shown in Q to PR.
the diagram (d) Measure /QX/ and PQR. .
2 :At point P construct 1350 and produce a line 9cm
through it to R Construction of triangles with ratios
3: Join R to Q to complete the triangle
1995/9 OM
4 : Construction of locus l1; which is the perpendicular
(a)Using a ruler and a pair of compasses only, construct
bisector of line PQ
triangle ABC with /AB/ = 7.5cm, /BC/ = 8.1cm and
5 : Construction of locus l2; which is the perpendicular
ABC = 1050
bisector of line RQ
(b) Locate a point D on BC such that /BD/ : /DC/ is 3:2
6 : Draw a dotted line at the point of intersection S to R
(c) Through D, construct a line L perpendicular to BC .
(d) If the line L meets AC at P , measure /BP/ .

267
2001/8 Neco
B

C
A

/BQ/ = 6.2cm
BQC = 160 o

2001/13 ( Nov )

/SR/ = 10.75cm
0
PQR = 25

P Q

L1
L 2

268
Analysis and solution Diagram on page 270 2006/9(a) Exercise 22.10 OM
Steps taken ( a ) Using a ruler and a pair of compasses only :
1: Draw a line AB = 7.5cm with extensions as shown in ( i ) construct XYZ such that /XY/ = 8cm and
the diagram YXZ = ZYX = 450 ;
2 :At point B construct 1050 and produce a line 8.1cm ( ii ) locate a point P inside the triangle equidistant
through it to C from YX and YZ ;
3: Join C to A to complete the triangle ( iii ) construct a circle touching the three sides of the
4 : Draw a line CK and divide it into 5 reasonable equal triangle .
segments; join K to B ( iv ) measure the radius of the circle .
5 : Mark the 3rd segment point d on CK Construction of triangles with circumcircle
6 : At d construct a parallel line to BK using steps of a It is done by perpendicular bisector of any two sides of the
point outside a parallel line and join d to D triangle. Their point of intersection is the centre of the circle
7 : At D construct a perpendicular to meet AC at P touching all the vertices of the circle and it is equal distance
from all the vertices
1987/9(a) GCE Exercise 22.9 OM
(a) Using a ruler, a protractor and a pair of compasses, 1985/21 GCE OM
construct a triangle ABC , given that /BC/ = 8.5 cm , ( a ) Construct triangle PQR such that /QR/ = 8cm
/AB/ = 5.1 cm and BAC = 650 angle PQR = 600 and angle QRP = 450
( i ) Measure /AC/. ( b ) Construct the locus l1 of points equidistant from
(ii) Find by a suitable construction, a point D on AC PQ and PR
such that /AD/ : /DC/ = 3 : 5 . ( ii ) Locate a point S equidistant from P, Q and R
( iii ) Measure angles ABD and CBD . ( iii ) Draw the locus l2 of points whose distance
Author’s hint: Use protractor to get 650\ and not by from S equal to SP
construction ( c ) If X is a point of intersection of l1and l2, measure
( i ) QX ( ii ) angle QRX
Construction of triangles with inscribed circle Analysis and solution Diagram on page 271
It is done by bisecting any two angles of the triangle. Steps taken
Their point of intersection is the centre of the circle 1: Draw a line RQ = 8cm with extensions as shown in
touching all the sides of the circle. the diagram
2006/11 Neco OM
2 :At point Q construct 600 and produce a line
(a) Using a ruler and a pair of compasses only ,
through it to a reasonable extent
construct :
( i ) a triangle PQR such that PQ = 8.5cm , PR = 7cm 3 :At point Q construct 450 and produce a line
through it to touch the first line at P
and QPR = 750 .
( ii ) the locus, L, of points equidistant from PQ and PR 4 : Construct l1 which is the bisector of P
( iii ) an inscribed circle which lies inside the triangle 5 : For ( ii ) to be fulfilled; we must construct perpendicular
and touches PQ , PR and QR. bisector of line RP and perpendicular bisector of line PQ.
The point of their intersection is S
( b ) Measure : 6 : l2 is construction of a circle with radius SP and
( i ) QR , ( ii ) the radius of the inscribed circle . centre S
Analysis and solution Diagram on page 261 l1and l2 meet at P and X but point X will serve us here
Steps taken
1: Draw a line PR = 7cm with extensions as shown in 2004/9 Exercise 22.11 OM
the diagram Using a ruler and a pair of compasses only .
(a) Construct :
2 :At point P construct 750 and produce a line 8.5cm
( i ) PQR such that /PQ/ = 8cm , /PR/ = 7cm and
through it to Q
QPR = 1050 .
3: Join Q to R to complete the triangle
4 : Construction of locus L at P is the bisecting of angle P ( ii ) locus l1 of points equidistant from P and Q .
5 : For ( iii ) to be fulfilled; we must construct bisector ( iii ) locus l2 of points equidistant from Q and R
of angle R or Q. Here we bisect R and the intersection (b) ( i ) Label the point T where l1 and l2 intersect .
K is the centre of the circle touching all the sides ( ii ) With centre T and radius /TQ/ , construct a
circle l3
(iii). Complete quadrilateral PQSR such that
/RS/ = /QS/ and /TQ/ = /TS/ .

269
270
271
SCALED, COPIED DIAGRAM CONSTRUCTION
( b )Construct :
2002/11 ( Nov ) OM
The diagram below represents a lawn ABCD where (i) The locus l1 of points equidistant from AC and BC .
AB = 45m, BC = 30m, CD = 35m, ABC = 1000 and (ii) The locus l2 of points 60m from A.
BCD = 900. ( c ) A tree T inside the plot, is on both l1and l2. Locate T and
D find /TC/ in metres .
( d ) A flag pole , P is to be placed such that it is nearer AC
than BC and more than 60m from A . Shade the region
where P can be located .

35
m
C
A

45m 100 0 30m

B
(a) Using a scale of 1cm : 5m draw an accurate
diagram of the lawn .
(b) Construct the locus:
( i ) l1of points equidistant from AD and DC ,
( ii ) l2of points 25m from B
(c) Shade the region of all points in the lawn which are
not more than 25m from B and nearer AD than DC .
Analysis and solution Diagram on page271
Steps taken
1 : Scale conversions- AB = 9cm from 45m,
BC = 6cm from 30m, CD = 7cm from 35m
2 : Draw a line AB = 9cm with extension as shown in
the diagram

3 : At B measure out 1000 with your protractor.(Do not


construct) and draw line BC = 6cm through it
4 : At C measure out 900 with protractor and draw
line CD = 7cm through it
5 : Join D to A to complete diagram
6 : Construction of locus l1 is the bisector of
angle D with the aid of compass and ruler
7 : Construction of locus l2 is a circle with
radius 5cm( from 25m) and centre B
The region is shaded in the diagram

2005/10 Exercise 22.12 OM


The sketch shows a plot of land.
C

40 0 95 0

A 85m B
( a ) Using a scale of 1cm to 10m draw an accurate
diagram of the plot .

272
Chapter twenty three we still obtain a zero correlation to indicate a non – linear
Regression and correlation relationship.
Note that: A positive correlation indicates direct association
Regression (relationship) whereas negative correlation indicates inverse
Regression is the act of estimating or predicting the association (relationship)
value of a dependent random variable y on the basis of a
known measurement of an independent variable x Y Y
For instance: If a man is 50 years old. What is his
expected blood pressure?
The age is the independent variable x or the predictor
variable; whereas the blood pressure is the dependent
variable or response variable.
If x and y have a relationship with each other, for us to X X
predict y from x , we have to be able to find a (a ) High positive correlation (b) High negative correlation
model for the relationship. The simplest model of this
type of relationship is a straight line
called simple linear regression line. Y Y
It has only one independent - variable
y = a + bx –---- (1)
Where a & b are constants
(1) is the regression line of y on x but we sometimes
have a situation of X X
x = a + by –------ (2) (d) Non linear correlation
(c) Zero correlation
(2) is the regression line of x on y
b in both cases is called the coefficient of regression
The most commonly used measure of linear correlation
Scatter diagram between 2 variables in further mathematics is the
This is the pictorial result of plotting a graph showing the Spearman’s rank correlation method; which can be
relationship between two random variables say x and y
calculated by ranking of the variable based on descending
A line of best fit plotted on a scatter diagram is at its
order of magnitude and not as they appear. After the ranking
best to say when it passes ( x , y ) and other points we now tabulate the data and their ranking along as they
been equidistant from it on both sides. The line appear originally. The formula is
itself is called regression of y on x or regression of x on 6 D 2
y depending on whether it is y = a + bx or x = a + by rk = 1 –
N ( N 21)
Correlation Another commonly used measure of linear correlation
A correlation problem differ from regression problem in between 2 variables is called the Pearson product moment
that it is concerned with the measurement of the correlation coefficient or simply the sample correlation
relationship between two or more variables rather than coefficient with formula
predicting one variable from knowledge of the
impendent variable. We define the linear correlation
n xy   x  y 
coefficient to be the measure of a linear relationship
between two random variables x and y; noted by ρ.
r=
n x   x  n y   y  
2 2 2 2

It should be noted that correlation coefficient ρ = 0


implies a lack of linearity & not a lack of association. Interpretation of correlation coefficient
To estimate a linear correlation coefficient we need a Generally – 1  r  1
random sample of pairs of measurements (x, y). Then A value of – 1 or1 will occur when sum of squares of the
by considering a scatter diagram for the (x,y) values errors (SSE) about the regression line = 0 but this is the case
(see diagram that follows) we will be able to draw where all the points are in a straight line. Hence a perfect
certain conclusion. linear relationship exist between x and y when r = 1.
If the points follow closely a straight line of positive
slope as in (a); we have a high positive correlation Values between +0.5 to +0.9 is stated as : High positive
between the variables. On the other hand, if the points correlation Whereas
follow closely a straight line of negative slope as in (b); Values between -0.5 to -0.9 is stated as : High negative
we have a high negative correlation between the two correlation
variables. If the points follows a strictly random pattern If r is close to zero the linear relationship between x and y is
as in (C); we have a zero correlation and conclude that weak; one must be careful in interpreting r beyond what has
no relationship exists between x & y. If a strong been stated above.
quadratic relationship exists between x and y as in (d);
273
2009/15 Month x y Rx Ry D=Rx– Ry D2
The table below shows the corresponding values of two Jan 1.00 8.00 12 12 0 0
variables x and y Feb 1.50 8.20 9 10 -1 1
X 33 31 28 25 23 22 19 17 16 14 Mar 1.25 8.15 11 11 0 0
Y 4 6 4 10 12 10 14 15 18 22 Apr 1.60 8.25 8 9 -1 1
May 1.40 8.30 10 8 2 4
(a) Plot a scatter diagram to represent the data June 2.00 8.60 7 5 2 4
(b) Calculate x , the mean of x and y mean of y July 2.10 8.80 6 2 4 16
Aug 2.20 8.65 5 4 1 1
(c) Draw the line of best fit to pass through ( x , y ) Sep 2.60 8.70 2 3 -1 1
(d) From your graph in (C), determine the Oct 2.40 8.50 4 6 -2 4
( i ) relationship between x and y; Nov 2.50 8.90 3 1 2 4
( ii ) value of y when x is 24. Dec 3.00 8.40 1 7 -6 36
Graph and solution is on page 275
 D  72
2

2004/14 Neco (Dec)


The table below shows the price of a commodity in 6(72)
1999 and the price of the same commodity 2002 as N x rk = 1 –
and N y respectively. 12(12 2 1)
432
Month Jan Feb Mar Apr May Jun = 1
1716
X 1.00 1.50 1.25 1.60 1.40 2.00 = 1– 0. 25
Y 8.00 8.20 8.15 8.25 8.30 8.60 = 0.75
There is a high positive correlation between x and y.
Jul Aug Sept Oct Nov Dec
2.10 2.20 2.60 2.40 2.50 3.00 1996/13 (Nov) Exercise 23.1
8.80 8.65 8.70 8.50 8.90 8.40 The table gives the distribution of the population (in 000’s )
of a city over a period of 80 years.
(a) Plot y against x on a scatter diagram and draw on it Year (y) 10 20 30 40 50 60 70 80
a line of regression
Population (P) 9.0 10.2 12.0 13.9 15.9 17.9 20.1 22.7
(b) Find the regression equation and obtain the
coefficient of regression of y on x If the population P, is linearly related to the year y,
(c) Calculate a coefficient of rank correlation between that is, P = h + ky, where h and k are constants,
the two prices and interpret your result. (a) Plot the scatter diagram:
Graph and solution a and b is on page 276 (b) Draw an eye–fitted line of best fit
(c) Use your graph to estimate the
6 D 2 (i) values of h and k
(c) Rank correlation coefficient rk = 1 – (ii) Population in the year 90
N ( N 21)
First, we arrange the given data in descending order 1999/6 (Nov) Exercise 23.2
and rank them. The table shows the values of quatities x and y obtained in a
laboratory experiment.
X Ranking Y Ranking
x 0.0 0.5 1.0 1.5 2.0 2.5 3.0
3.00 1 8.90 1 y 0.5 0.7 1.0 1.5 1.5 1.5 2.4
2.60 2 8.80 2
(a) Plot a scatter diagram for the data
2.50 3 8.70 3
(b) Draw an eye fitted line of best fit
2.40 4 8.65 4 (c) Using your graph, estimate the value of y when x = 3.5
2.20 5 8.60 5
2.10 6 8.50 6 2004/14 (b) Exercise 23.3
2.00 7 8.40 7 The table shows the mark obtained by ten students in theory
1.60 8 8.30 8 And practical tests in biology.
1.50 9 8.25 9 Students A B C D E F G H I J
1.40 10 8.20 10 Theory 79 63 84 46 77 73 56 58 49 69
1.25 11 8.15 11 Practical 56 42 59 35 54 62 47 51 24 49
1.00 12 8.00 12 (i) Plot the scatter diagram for the data
(ii) Draw an eye - fitted line of best fit
Next, we prepare a table showing them as they occur (iii) Using your graph, estimate the mark in theory when the
in the original table and allocating the rank gotten . practical mark is 50
274
275
276
2003/14 Neco 1996/23 (Nov)
(a) Define the term correlation coefficient All the plotted points on a scatter diagram lie almost on a
(b) Interpret the following regression equations straight line. Which of the following is correct about the
(i) y = a x + b distribution?
(ii) x = ay + b A The distribution is negatively skewed
(c) States 3 types of correlation B The distribution is positively skewed
Solution C The correlation coefficient will be almost zero
(a) Correlation coefficient is the measure of D There is a linear association
linear association between two random E The distribution is normal.
variables rank x and y. Solution
(b) (i) y is the regression on x All most all the points on a scatter diagram lie on a straight
(ii) x is the regression on y line then: there is a (perfect) linear association (D)
(c) Positive correlation
Negative correlation 1993/23 (Nov) Exercise 23.4
Zero correlation The line of best fit is always drawn on
(The 4th one is : Non – linear correlation) A a scatter diagram B a histogram
C a cumulative frequency curve
2007/44 Neco D a pie chart E a frequency polygon.
The linear regression function of the relationship
between scores in physics practical (y) and performance 1994/24 (Nov) Exercise 23.5
in drama (x)is given by The scatter diagram is used to estimate the
5y + 7x = 2 A means of two distributions
Find the coefficient of regression B variance of two distributions
7 5 2 5 7 C difference between two groups
A– B C D E
5 7 5 7 5 D equality of two groups
Solution E relationship between two groups.
Making y uniform gets coefficient of regression;
5y + 7x = 2 becomes 1999/10 (Nov) Exercise 23.6
5y = 2 – 7x A scatter diagram shows a positive correlation
2 7 between variables x and y. Which of the following
y=  x statement is true?
5 5 A y tends to increase as x decreases
Thus, coefficient of regression B y tends to decrease as x increases
7 C y tends to increase as x increases
of y (physics practical) on x (drama) – (A)
5 D y remains positive for all vales of x.

1994/29 2006/25 Exercise 23.7


If r denotes a correlation coefficient, which of the The equation of the line of best fit for variables x and y
following expressions is not true? is y = 19.33 + 0.42x, where x is the independent variable.
A r = –0.75 B r = 1.25 C 0.5 r 0.7 Estimate the value of y when x =15
D – 0.5  r  0. 5 E –1r1 A 18.91 B 19.74 C 25.63 D 38.23
Solution
Correlation coefficient r 1.25 as the limits for it is
– 1 r  1 i.e between positive and negative 1.Thus 1992/25 (Nov) Exercise 23.8
every other given value of r is possible What does a correlation coefficient measure?
except for r = 1.25(B) A central tendency B Dispersion
C linear association D probability E slope
1996/26 1997/ 21 Exercise 23.9
Which of the following correlation coefficient indicates A negative correlation coefficient implies that
the most high degree of linear association? (A) the association between two variables is definitely weak.
– 0.85, – 0.26, – 0.64, 0.19, 0.37, 0 .68 (B) values of two variables are increasing together
A – 0.85 B – 0.26 C 0.19 D 0.37 E 0.68 (C) values of two variables are decreasing together
Solution (D) there exists no linear association between two variables
High degree of linear association exist when the (E) an increasing variable is associated with another
correlation coefficient r is very close to 1 decreasing one.
Thus – 0.85 indicates a higher degree of linear
association though negatively; its degree of linearity is
far above that of 0.68(A)

277
Eg RC1 Consider the table below: Since we are not restricted to either spearman’s rank or
x (Height) 12 10 14 11 12 9 Pearson’s method of calculating correlation coefficient; we
shall treat both
y (weight) 18 17 23 19 20 15
(iii) Correlation coefficient: Pearson formula
(i) Determine the linear regression line of y or x n xy   x  y 
(ii) Estimate the weight, when the height is 16 r=
(iii) Compute and interpret the correlation coefficient n x   x  n y   y  
2 2 2 2

Solution
Since we are not asked to plot a scattered diagram;the Next, prepare a table for items on r formula i.e x, y, xy, x2
formula method is and y2
x y xy x2 y2
( i ) The linear regression line of y on x is y = a + bx
Where b = n xy   x  y  ; 12 18 216 144 324
n x 2   x 2


a = y  b x and

y =
y , x =
x 10 17 170 100 289
n n 14 23 322 196 529
Next, prepare a table for items on b formula i.e x, y,
xy, and x2 11 19 209 121 361
x y xy x2
12 20 240 144 400
12 18 216 144
9 15 135 81 225
10 17 170 100
x = 68 y =112 xy =1292 x2 =786 y2 =2128
14 23 322 196

11 19 209 121 6(1292)  (68) (112)


r=
6(786)  (68) 6(2128)  (112) 
2 2

12 20 240 144
7752  7616
=
9 15 135 81 4716  4624 12768  12544
x = 68 y = 112 xy = 1292 x2 = 786 136
=
6 1292  68 112 (92) (224)
b = 136
6  786  (68) 2 =
=
7752  7616 20608
4716  4624 136
=  0.95 High positive correlation
136 143 .555
=
92
b  1.48 Alternative solution Spearman’s rank
 
Next we find a = y  b x
6 D 2
But y =
y =
112
ie 18.67
(c) Spearman’s Rank correlation coefficient rk=1–
N ( N 21)
n 6 First, we arrange the given data in descending order

and x =
x =
68
ie 11.33
and rank them.
X Ranking Y Ranking
n 6
a = 18. 67 – 1.48 × 11 .33
14 1 23 1
= 18. 67 – 16. 77 12 2.5 20 2
a = 1.9 12 2.5 19 3
Thus, y = 1.9 + 1.48x 11 4 18 4
10 5 17 5
(ii) Here we substitute for x value into y = 1.9 + 1.48x 9 6 15 6
y = 1.9 + 1.48(16) Note: ranking of similar numbers is done by average as
= 1.9 + 23.68 = 25.6 treated here for 12 that appear twice in rank 2 and 3
278
x y Rx Ry D = Rx – R y D2 1992 /5 (Nov)
12 18 2.5 4 – 1.5 2.25 The mark scored by ten students in mathematics
10 17 5 5 0 0 and physics in an examination are as follows:
14 23 1 1 0 0 Student A B C D E F G H I J
11 19 4 3 1 1
12 20 2.5 2 0.5 0.25 Mathematics 85 75 59 43 74 69 62 80 54
63
9 15 6 6 0 0 Physics 92 72 62 48 85 73 46 74 58 50
D2 = 3.5

6(3.5) Calculate the spearman’s rank correlation coefficient


[

rk = 1 – Solution
6(6 2 1)
rk = 1 –  2
2
6 D
21
= 1– N ( N 1)
210
= 1– 0 . 1 First, we arrange the given data in descending
= 0.9 High positive correlation order and rank them
Maths Rank Physics Rank
1992 /28 (Nov) 85 1 92 1
The ranks of the marks scored by 4 80 2 85 2
candidates in mathematics and chemistry
75 3 74 3
are as shown in the table below
74 4 73 4
Mathematics 4 2 1 3 69 5 72 5
63 6 62 6
Chemistry 3 1 2 4 62 7 58 7
59 8 50 8
Calculate the spearman’s rank correlation coefficient.
54 9 48 9
A0 B 0.3 C 0. 6 D 0. 7 E 0. 8
43 10 46 10
Solution

rk = 1 –  2
6 D2 Next, we prepare a table showing them as they occur
N ( N 1) in the original table and allocating the rank gotten .
First we arrange the given data in descending order and Maths (x) Physics (y) Rx Ry D = Rx -Ry D2
rank them. A 85 92 1 1 0 0
Maths Ranking Chemistry Ranking B 75 72 3 5 -2 4
4 1 4 1 C 59 62 8 6 2 4
3 2 3 2
D 43 48 10 9 1 1
2 3 2 3
E 74 85 4 2 2 4
1 4 1 4
F 69 73 5 4 1 1
Next, we prepare a table showing them as they occur G 62 46 7 10 -3 9
in the original table and allocating the rank gotten .
H 80 74 2 3 -1 1
Maths Chem Rx Ry D = Rx – Ry D 2
(x) (y) I 54 58 9 7 2 4
4 3 1 2 -1 1
J 63 50 6 8 -2 4
2 1 3 4 -1 1
1 2 4 3 1 1 D2
= 32
3 4 2 1 1 1
D = 4
2
6(32)
rk = 1–
10(10 2 1)
6(4)
rk = 1– 192
4(4 2 1) = 1–
990
24 = 1– 0.1 939
= 1–
60 = 0.8061
= 1– 0.4 = 0.6 (C)

279
1993/24 (Nov) Solution
If d is the difference between each pair of ranks
between 5 paired variables and  d2 = 4. Calculate the rk = 1 –
6 D 2

N ( N 2 1)
spearman’s rank correlation for the variables.
A
1
B
1
C
4
D
29
E1 0.8 = 1 –
6 d 2

30 5 5 30 4(4 2 1)
Solution
6 D 2 0.8 = 1 – d
6 2

Spearman’s rank correlation = 1 – 60


N ( N 2 1) Clear fraction at the RHS
64 0.8 =
60  6 d 2

= 1 60
5(5 2 1) Cross multiplying, we have
=1–
24
120
i.e 1 
1
5
0.8 × 60 = 60 – 6 d 2

48 = 60 – 6  d
2
4
= (C)

1994/30 (Nov)
5 Thus, 6 d 2
= 60 – 48 Thus d 2
= 2 (A)
The table below shows the ranks Rx and Ry of marks 1995/27 (Nov)
scored by 10 candidates in an oral and written tests The ranks of the marks scored in a test by 7 candidates in
respectively. Calculate the spearman’s rank correlation statistics and Economics are given in the table.
coefficient of the data. Subject Ranks
Rx 1 2 3 4 5 6 7 8 9 10 Statistics 4 7 5 1 2 6 3
Ry 2 3 4 1 6 5 8 7 10 9
2 5 6 3 1 7 4
1 6 49 54 61 Economics
A B C D E
55 55 55 55 55 Calculate the spearman’s rank correlation coefficient
Solution A 0.71 B 0. 29 C 0.28 D 0.17 E 0.13
6 D 2 Solution
rk = 1 –
rk = 1 –  2
N ( N 2 1) 6 D2
Since they have been ranked; we go straight to the 2nd N ( N 1)
step table for D2as shown below Since the given data have been ranked, next
Rx Ry D = Rx – Ry D2 we go straight to the 2nd step of the table as shown below;
1 2 -1 1 RS RE D = RS –RE D2
2 3 -1 1 4 2 2 4
3 4 -1 1 7 5 2 4
4 1 3 9 5 6 -1 1
5 6 -1 1 1 3 -2 4
6 5 1 1 2 1 1 1
7 8 -1 1 6 7 -1 1
8 7 1 1 3 4 -1 1
9 10 -1 1
10 9 1 1 D 2
=
D = 18
2 16
6(16)
6  18 rk = 1 –
Thus, rk = 1 – 7(7 2 1)
10(10 2 1) 96
108 = 1– = 1 – 0.286 = 0.714  0.71 (A)
= 1 336
990 1996/25 Exercise 23.10
54 441 The table below gives the marks obtained by 5 candidates in
= 1 i.e
495 495 Statistics and Economics tests. Calculate the spearman’s
1995/26 (Nov) rank correlation coefficient:
If the spearman’s rank correlation coefficient of a set of
four paired data is 0.8, find the sum of the squares of Statistics 82 45 50 69 46
the difference between the ranks of each pair. Economics 49 32 47 73 67
A2 B8 C 12 D 16 E 18
A – 0.49 B – 0.50 C 0.49 D 0.50 E 1.00
280
1996/27 (Nov) Exercise 23.11 1997/5 Exercise 23.17
The table shows the marks of 6 pupils in English and The table shows the marks obtained by ten candidate in
Mathematic test: Physic and Mathematics test
English 62 69 64 70 63 68 A B C D E F G H I J
Mathematics 61 72 58 63 66 56 Cand
.
Calculate the spearman’s rank correlation coefficient
Physi
A – 0.2 B 0.2 C 0.8 D 0.87 E 0.93 c
58 71 63 45 90 87 70 65 53 59

1997/25 Exercise 23.12 score


s
The table shows how 8 men were ranked according to
their ages and weights: Math 63 65 59 50 81 73 83 72 61 62
s
Age 6 3 8 2 7 4 1 5
score
Weight 5 7 6 1 8 4 3 2 s
Calculate, correct to 2 significant figures, the
Spearman’s rank correlation coefficient. (i) Calculate, correct to two decimal places, the spearman’s
A 0.43 B 0.48 C 0.52 D 0.57 E 0.93 rank correlation coefficient
(ii) Interprete your result
2002/15b Exercise 23.13
Participants in two separate athletic events are ranked as 2000/14b Exercise 23.18
shown in the table below: The table shows the marks obtained by ten students in
Biology and Physics tests
Participant A B C D E F G H I J
Biolog 6 53 83 61 57 72 47 65 68 58
Event 2 6 1 10 3 4 8 7 9 5 y 3
No.1 Physics 6 45 71 73 47 69 50 70 42 52
Event 3 8 2 9 4 1 7 5 10 6 7
No.2
Calculate, correct to 2 decimal places, the spearman’s rank
(i) Calculate the Spearmen’s rank correlation coefficient correlation coefficient.
(ii) Interprete your result.
2005/15a (Nov) Exercise 23.19
2000/16 Neco Exercise 23.14 In a competition two judges X and Y awarded the following
The table below shows how 10 students arranged marks to the competitors.
in alphabetical order, were ranked according to their Competitors A
achievement, in both physic and chemistry. Find the B C D E F G H I J
Marks by X 37 47 50 85 60 78 62 70 80 55
coefficient of rank correlation. Marks by Y 72 59 52 80 62 75 68 66 82 55
Physics 8 3 9 2 7 10 4 6 1 5
Calculate, correct to 2 decimal place, the spearman’s rank
Chemistry 9 5 10 1 8 7 3 4 2 6 correlation coefficient of the distribution.
A 0.8525 B 0.8554 C 0.8555 D 0.8455 E 0.8545

2008/15b Exercise 23.15 2003/14 d Neco (Dec) Exercise 23.20


The table shows the positions awarded to 7 contestants The table below shows the marks obtained by ten students in
by Judges X and Y in a competition. Mathematics and Further Mathematics.
Contestants P Q R S T U V Maths 5 25 33 55 65 38 35 53 61 44
Judge X 2 7 1 3 6 5 4 1
Judge Y 4 6 2 3 7 1 5 F/math 2 65 25 36 51 50 77 31 60 5
(i) Calculate, correct to one decimal place, the s 0
Spearman’s rank correlation coefficient. Calculate the rank correlation coefficient between the two
(ii) Interpret your answer in (b) (i) above. subjects. Hence, comment briefly on your result.

2001/38 Neco Exercise 23.16


Two boxing enthusiasts X and Y were asked to rate five
great boxers. Their ratings are as follows.
Tyson Lewis Hart Ali Hollyfield
X 2 5 3 1 4
Y 4 2 5 1 3

What is the correlation coefficient between these ratings?


A –0.11 B 0.1 C 0.47 D 0.9 E 0.89

281
Chapter twenty four (4)
Normal distribution
Normal distribution is an example of a continuous
random variable. Any given problem under normal
distribution must be in standard form Z. Since the normal -3 2 0
distribution table only has Z-values. Required area = Total area - (area b/w z = 0 and z = -3)
If a given problem has not been standardized it will come MINUS (area b/w z = 0 and z = 2)
along with mean and standard deviation or with
sufficient data to calculate the mean and standard (5)
deviation ( variance). Then we standardize it by the formula.
Z= x–µ
δ
Where: x is the given problem value -3 0 2
µ is mean By symmetrical property
δ is standard deviation. Recall that standard Pr( -3 z 2) = Pr(z = 3) + Pr(z = 2)
deviation is the square root of variance.
(6)
Basic facts to know about Normal distribution
It has a bell – shaped curve

0 0.4
Pr(z=0.4)
0
.5 0
.5
The Author used real numbers to illustrate the diagrams to
accommodate those who have phobia for algebra.
1. The curve is symmetrical about a vertical axis
1997/13
through the mean µ
The heights, in meters, of some men are assumed to be
2. The curve approaches the horizontal axis
normally distributed with mean 1.73 and standard deviation
asymptotically as we proceed in either direction
0.1. Find the proportion of men whose heights.
away from the mean
( i ) are less than 1.77;
3. The TOTAL AREA under the curve and above
( ii ) lie between 1.67 and 1.75
the horizontal axis is equal to 1
(b) If 10% of the tallest men are to be considered for a Job,
4. The mode occurs at x = µ (i.e where the curve is maximum) find correct to two significant figures, the minimum height.
Solution
(i) First, we standardize 1.77 i.e changing x to z
Z= x–µ
δ
N
eg
ativ
eax
is 0 P
ositiv
eax
is Here x = 1.77 , µ =1.73 and δ =0.1
1.77  1.73
Any given problem will take a form of the Z=  0.4
0.1
following shapes
Thus Pr ( x  1.77 ) = Pr (Z 0.4 )
Negative Positive By inequalities, we sketch the normal curve as:
Diagrammatically
(1N)
Pr(z -2) (1P)
Pr(z 2)
By symmetry

Z Z
-2.0 0 0 2 0 0.4
= 0.5 - Pr(z = 2)
Interpreting the resulting normal curve
(2P)
(2N)
Pr(z -2)
Pr(z 2) = 0.5 + Pr ( Z = 0.4) from Normal distribution table
By symmetry = 0.5 + 0.1554
Z Z = 0.6554
-2.0 0 2 (ii) Pr (1.67  x  1.75) i.e changing x to z
0
= 0.5 + Pr(z = 2) First, we standardize the given values
( x = 1. 67 ) ( x = 1.75 )
(3P)
(3N)
Pr( 2 z 3) 1.67  1.73 1.75  1.73
Pr( -3 z -2) By symmetry
Z= Z=
0 .1 0.1
Z = - 0.6 Z = 0.2
Z
-3.0
Z
-2.0 0 0 2 3 Thus, Pr (1.67  x  1.75) = Pr (-0.6  z  0.2)
= Pr(z = 3) - Pr(z = 2)

282
By inequalities, we sketch the normal curve as: close to 0.45 (result of 90% of 0.5) and not 0.9 as expected
Diagrammatically 0.4495 has a z – value of 1.64
Thus 1.64 = x – 60
15
1.64  15 = x – 60
24.6 + 60 = x
-0.6 0 0.2 84.6 = x the required lowest mark
Interpreting the resulting normal curve by
applying symmetrical properties (b) We are to find Pr (65  x  75)
= Pr`(z = 0.6) + Pr(z = 0.2) from Normal table Standardizing x to Z by applying
= 0.2258 + 0.0793 Z = x–µ
= 0.3051 δ
(b) If 10% of the tallest men are to be considered for a  65  60 75  60 
Job, it follows that the minimum height will be at = Pr  Z  
90% above the mean  15 15 
(since the range of the tallest men is above the mean) = Pr ( 0.333  Z  1.000)
To find the minimum height, we go through a reverse By inequalities, we sketch the normal curve as
process, starting from the normal distribution table
D
ia
g ra
m m
atic
ally

9
0% 00
.33
3 1
.0
1
0%
Interpreting the resulting normal curve
Bearing in mind that the normal distribution table only = Pr ( Z = 1.0) – Pr ( Z = 0.333) from Normal distribution table
has half portion values, we have to look for value of or = 0.3413 – 0.1304
close to 0.45 (result of 90% of 0.5) and not 0.9 as = 0.2109
expected (c) Percentage of students whose marks differ from the
0.4495 has a z – value of 1.64 mean by 10 or more implies both above and below the
Thus 1.64 = x – 1.73 mean difference
0.1 Above mean is 60 + 10 = 70
1.64  0.1 = x – 1.73 Below mean is 60 – 10 = 50
0.164 + 1.73 = x Next we standardize the values i.e changing x to z
1.894 = x the required minimum height ( x = 50 ) ( x = 70 )
50  60 70  60
1998/13 (Nov) Z= Z=
15 15
The marks obtained by candidates in a test are assumed
Z = – 0.667 Z = 0.667
to be normally distributed with mean 60 and standard
Bearing the question in mind the we sketch the normal curve as:
deviation 15. What is :
(a) the lowest mark for grade A if the highest 10%
of the candidates obtained grade A?
(b) the probability of obtaining marks
between 65 and 75.
(c) percentage of students whose marks differ from the -
0.6
67 0 0
.6
67
mean by 10 or more? Required Probability
Solution = Total area – (Area between Z=0 and Z = 0.667) – (Area
(a) if the highest 10% of the candidates obtained grade between Z = 0 and Z = 0.667)
A then the lowest mark for grade A will be at 90% = 1 – 0.2477 – 0.2477
above the mean = 0.5046 i.e 50.46%
(since the range highest mark is above the mean)
To find the lowest mark, we go through a reverse 1997/ 27
process, starting from the normal distribution table The masses (kg) of a group of 300 women are approximately
D
ia
g ra
m m
atic
ally normally distributed with mean and standard deviation of 55
and 10kg respectively. What is the probability of a woman
selected at random from the group having a mass more than 72kg.
A 0.9554 B 0 .5446 C. 0.4554
9
0% D 0.2580 E. 0.0446
1
0%
Solution
Bearing in mind that the normal distribution table only We are to find Pr(x  72)
has half portion values, we have to look for value of or First, we standardize 72 i.e changing x to z

283
Z= x–µ 1996/30
δ The lengths of nails produced in a factory are approximately
Here x = 72, µ =55 and δ=10 normally distributed with mean 2cm and a standard deviation
72  55 0.01cm. Find the proportion of nails that will be shorter than
Z=  1.7 1.98cm.
10
A 0.9772 B 0.5228 C 0.4772
Thus Pr (x  72) = Pr (Z  1.7 ) D 0.4207 E 0.0228
By inequalities, we sketch the normal curve as: Solution
Diagrammatically
We are to find Pr(x  1.98)
First, we standardize the given value i.e changing x to z
Z= x–µ
δ
Here x = 1.98, µ = 2 and δ = 0.01
0 1.7
1.98  2
Interpreting the resulting normal curve Z=   2.0
= 0.5 – Pr ( Z = 1.7 ) from Normal table 0.01
= 0.5 – 0.4554 Thus Pr(x  1.98) = Pr ( Z  – 2.0)
= 0.0446 (E) By inequalities, we sketch the normal curve as
1996/ 24 (Nov) Diagrammatically By symmetry Diagram

The marks scored in a test are assumed to be normally


distributed with a mean 72 and standard deviation 16.
Find the probability of a candidate scoring less than
60 marks. 0 2.0
A 0.2266 B 0.2734 C 0.7266 D 0.7734 E 0.7881 -2.0 0
Solution Interpreting the resulting normal curve by symmetry
We are to find Pr( x  60) = 0.5 – Pr( Z = 2.0 ) from Normal distribution table
First, we standardize 60 i.e changing x to z = 0.5 – 0.4772
Z= x–µ = 0.0228 (E)
δ
Here x = 60, µ =72 and δ=16 1995/28 (Nov)
60  72 The scores of some 500 candidates in an examination were
Z =   0. 75 found to be approximately normally distributed with mean
16 40 and standard deviation 5. Find the number of candidate
Pr( x  60) = Pr ( Z  – 0.75 ) who scored at least 48
By inequalities, we sketch the normal curve as: A 27 B 55 B 55 C 227 D 277 E 473
Diagrammatically By symmetry Diagram Solution
We are to find Pr ( x  48)
Standardizing the value i.e charging x to Z
Z = x–µ
δ
0 0.75
-0.75 0 Here x = 48, µ = 40 and δ = 5
Interpreting the resulting normal curve by symmetry 48  40
Z   1.6
= 0.5 – Pr ( Z = 0.75) from Normal distribution table 5
= 0.5 – 0.2734 Pr( x  48) = Pr(Z  1.6 )
= 0.2266 (A) By inequalities, we sketch the normal curve as
Diagrammatically
1996/ 23
Find the area of the shaded region shown in the normal curve

0 1.6
Interpreting the resulting normal curve
-1
.4 0 2
.1 = 0.5 – Pr(Z = 1.6) from Normal distribution table
A 0.9821 B 0.9013 C 0.0987 D 0.0808 E 0.0179 = 0.5 – 0.4452
Solution = 0.0548
Required Probability Therefore number of candidates who scored
= Total area – (Area between Z = 0 and Z = 1.4) – at least 48 is 500 × 0.0548 = 27.4
(Area between Z = 0 and Z = 2.1)  27 (A)
= 1 – 0.4192 – 0.4821
= 0.0987 (C)
284
1994/6 (Nov) (b) out of a daily production of 10,000 wires, how many will
The mean score of 200 students in an examination is 40 have their length at least 0.05mm shorter than the mean?
and the standard deviation is 10. If the score are Solution
assumed to be normally distributed, find the, (a) i. We are to find Pr (x  5.00)
(a) Proportion of students obtaining more than 46. Standardizing x to Z
(b) Number of students scoring between 32 and 48. Z = x–µ
δ
Solution Here x = 5.00, µ = 4.85 and δ = 0.24
Proportion here simply means probability 5.00  4.85
(a) We are to find Pr ( x  46 )
Z=  0. 625
0.24
Standardizing the value i.e changing x to Z Thus, Pr (x  5.00) = Pr (Z  0.625)
Z = x–µ By inequalities, we sketch the normal curve as
δ Diagrammatically
Here x = 46, µ = 40 and δ = 10
46  40
Z =  0. 6
10
Thus Pr( x  46) is Pr(Z0.6) 0 0.625
By inequalities, we sketch the normal curve as Interpreting the resulting normal curve
Diagrammatically
= 0.5 – Pr ( Z = 0.625) from Normal distribution table
= 0.5 – 0.2340 = 0.266
(a) (ii) We are to find Pr (4.75  x  4.95)
Standardizing x to Z by applying
0 0.6
Z = x–µ
Interpreting the resulting normal curve
δ
= 0.5 – Pr (Z = 0.6) from Normal distribution table
= 0.5 – 0.2258  4.75  4.85 4.95  4.85 
= Pr  Z  
= 0.2742  0.24 0.24 
(b) We find the probability between 32 and 48 then = Pr (– 0.417  Z  0.417)
multiply the result by the total of 200 students. By inequalities, we sketch the normal curve as
We are to find Pr (32 x  48 ) D
iag
ram
matically
Standardizing x to Z by applying
Z = x–µ
δ
 32  40 48  40 
Pr (32 x  48 ) = Pr  Z 
 10 10  -0
.41
7 0 0
.41
7
= Pr( –0.8  Z  0.8) Interpreting the resulting normal curve by applying
By inequalities, we sketch the normal curve as symmetry properties
Diagrammatically = Pr( Z = 0.417 ) + Pr(Z = 0.417) from Normal distrib. table
= 0.1616 + 0.1616 = 0.3232
(b) We are to find Pr (x mean by 0.05) × 10,000
First, solve for Pr(x mean i.e 4.80)
Standardizing x to Z
-0.8 0 0.8 Z = x–µ
Interpreting the resulting normal curve by applying δ
symmetry properties
Here x = 4.80, µ = 4.85 and δ = 0.24
= Pr( Z = 0.8) + Pr (Z = 0.8) from Normal distribution table
4.80  4.85
= 0.2881 + 0.2881 Z    0.208
= 0.5762 0.24
Thus, number of students = 0.5762 ×200 ie 115 Thus, Pr(x mean i.e 4.80) = Pr( Z  –0.208)
By inequalities, we sketch the normal curve as
1992/13 (Nov)
The lengths of wires manufactured by a company are Diagrammatically By symmetry diagram

assumed to be normally distributed with a mean of


4.85mm and standard deviation of 0.24mm.
(a)Find the proportion of the manufactured wires that will be
( i ) Longer than 5.00mm; 0 0.208
( ii ) having lengths between 4.75mm and 4.95mm -0.208 0
Interpreting the resulting normal curve by
285
applying symmetrical properties Solution
= 0.5 – Pr(Z = 0.208) from Normal distribution table ( i ) We are to find Pr (50  x  75)
= 0.5 – 0.0824 Standardizing x to Z by applying
= 0.4176 Z = x–µ
Therefore number of products = 0.4176 × 10,000 δ
= 4176
 50  60 75  60 
1994/27 (Nov) = Pr  Z  
The life span of electric bulbs manufactured by a  25 25 
company is normally distributed with mean 2000 hours, = Pr (– 0.4  Z  0.6)
standard deviation 250 hours. Calculate the probability By inequalities, we sketch the normal curve as
Diagrammatically
that a given bulb manufactured by the company will
have a life span of 1800 hours or more.
A 0.802 B 0.788 C 0.401 D 0.399 E 0.212
Solution
We are to find Pr (x  1800) -0.4 0 0.6
Standardizing the value i.e changing x to Z Interpreting the resulting normal curve by applying
Z = x–µ symmetry properties
δ = Pr( Z = 0.4 ) + Pr(Z = 0.6) from Normal distribution table
Here x = 1800, µ = 2000 and δ = 250 = 0.1554 + 0.2258 = 0.3812
1800  2000 Thus, the required number of pupils who scored between
Z =   0.8 50% and 75% = 0.3812 × 30,000
250
= 11,436
Thus, Pr (x  1800) = Pr (Z  – 0.8 )
( ii ) We are to find Pr ( x  80)
By inequalities, we sketch the normal curve as
Standardizing x to Z by applying
Diagrammatically By symmetry diagram Z = x–µ
δ
80  60
Z   0.8
25
-0.8 0
0 0.8 Thus, Pr ( x  80) = Pr ( Z  0.8 )
Interpreting the resulting normal curve by applying By inequalities, we sketch the normal curve as
Diagrammatically
symmetrical properties
= 0.5 + Pr( Z = 0.8) from Normal distribution table
= 0.5 + 0.2881
= 0.7881 (B)
0 0.8
1992/29 (Nov ) Interpreting the resulting normal curve
Find the area of the shaped region shown in the normal = 0.5 – Pr ( Z = 0.8) from Normal distribution table
curve below: = 0.5 – 0.2881 = 0.2119
Thus, the required number of pupils who scored greater than
or equal to 80% = 0.2119 × 30,000
= 6,357
( iii ) First, we are to find Pr( x40) since it is failure
-1
.5 0 2
.0 if it were to be pass we will find Pr( x  40)
Standardizing x to Z by applying
A 0.0456 B 0.0896 C 0.044 D 0.8664 E 0.1336
Solution Z = x–µ
Required Probability δ
= Total area – (Area between Z = 0 and Z = 1.5) – 40  60
Z    0.8
(Area between Z = 0 and Z = 2.0) 25
= 1 – 0.4332 – 0.4772 Thus, Pr ( x  40) = Pr ( Z  – 0.8 )
= 0.0896( B ) By inequalities, we sketch the normal curve as

2010/14b Neco Diagrammatically By symmetry diagram


The marks of 30,000 pupils who sat for National
Common Entrance Examination in a particular year in
Niger state is normally distributed with mean 60% and
standard deviation 25%. Find the number of pupils who
( i ) scored between 50% and 75% 0 0.8
-0.8 0
( ii ) scored greater than or equal to 80% Interpreting the resulting normal curve by
( iii ) failed the examination if the pass mark is 40%
286
applying symmetrical properties 2002/30 Neco
= 0.5 – Pr ( Z = 0.8) from Normal distribution table If a student in a class scores 56 in a mathematics examination
= 0.5 – 0.2881 in which the class mean and standard deviation scores are
= 0.2119 52 and 2.5 respectively. What is the student’s z - score?
Thus, the required number of pupils who failed or A 10 B 4 C 2.5 D 1.6 E 0.8
scored less than 40% = 0.2119 × 30,000 Solution
= 6,357 We are to change x = 56 to Z
2001/44 Neco Z = x–µ
What is the area of the shaded region shown in the δ
normal curve below to one decimal place? 56  52
 = 1.6 ( D)
2.5
1994/ 26 Exercise 24.1
The marks scored by candidates in a public examination were
known to be normally distributed with mean 30 and standard
deviation 12. If 40 was fixed as the pass mark, find the proportion
-1
.1 0 1
.1 of the candidate who failed the examination.
A 0.7 B 0.5 C 0.4 D 0.3 E 0.1 A 0.2025 B 0.2975 C 0.7975 D 0.8000 E 0.8333
Solution 2005/4 Neco ( Dec) Exercise 24.2
We are to find Pr( Z  –1.1) + Pr( Z = 1.1) Investigation carried out by a dealer of tyres indicated that the life
By symmetry property Pr( Z  –1.1) = Pr( Z  1.1) span of a particular types of tyres was normally distributed about a
Pr( Z  –1.1) + Pr( Z = 1.1) = Pr( Z  1.1) + Pr( Z = 1.1) mean of 360 days and standard deviation of 15 days. If 1000 tyres
= 0.5 – 0.3643 + 0.3643 are bought, how many will the life span be;
= 0.5 ( B ) (a) more than 365days (b) between 350 and 380 days
1993/28 (Nov )
2004/8 Neco Exercise 24.3
Find, with the usual notations, P(Z  1.810 ) from the The marks scored by some officers during a promotion exercise in
tables of normal distribution. a particular company is normally distributed with a mean of 220
A0.9649 B0.7249 C0.6525 D0.5000 E0.4764 and standard deviation of 18 marks.If 200 officers sat for the
Solution examination, how many would you expect their scores to fall
The given values here have been standardized, so we within 200 and 250 marks?
proceed to the next step
Pr (Z  1.810 ) 2003/6 Neco Exercise 24.4
By inequalities, we sketch the normal curve as A random variable (x) whose distribution is normal has a mean
Diagrammatically of 25 and standard deviation of 5. Find Pr (x  26.5 )
A 0.8821 B 0.5821 C 0.3821 D 0.3812 E 0.3281

2001/45 Neco Exercise 24.5


The scores of 1000 students in a test were found to be
0 1.81 approximately normally distributed with a mean of 100 and
Interpreting the resulting normal curve standard deviation of 25. How many students scored up to 150
= 0.5 + Pr (Z = 1.810) from Normal distribution table to 3 significant figures?
= 0.5 + 0.4649 A 977 B 725 C 523 D 477 E 228
= 0.9649 (A)
2009/41 Exercise 24.6
2003/7 Neco Find P( Z < –1.2) using normal distribution table.
Using the standard distribution table, find the value of C A 0.1151 B 0.1511 C 0.1611 D 1.1510 E 11.051
such that Pr (Z  C ) = 0.3072
A 0.967 B 0.870 C 0.868 D 0.687 E 0.565 2009/8 Neco Exercise 24.7
Solution The masses of animals in a zoo are known to be normally
Pr (Z  C ) distributed with mean 250kg and standard deviation 25kg. What is
By inequalities, we sketch the normal curve as the probability that the mass of animal selected from this zoo is; (a)
Diagrammatically greater than 275kg,(b)less than 200kg,(c) lies between 290kg & 325kg.

2003/15 Neco (Dec) Exercise 24.8


The marks scored by candidates seeking admission to study
medicine at the university of Ibadan in year 2002 are normally
0 C distributed with mean of 280 marks and standard deviation of 20
Interpreting the resulting normal curve marks. Out of 1000 candidates that sought for admission in year
Pr (Z  C ) = 0.5 + Pr (Z = C ) 2003 for medicine, how many of them are expected to have their
marks fall within 275 and 295?
0.3072 = 0.5 + Pr (Z = C )
0.3072 – 0.5 = Pr (Z = C ) 2004/50 Neco (Dec) Exercise 24.9
0.1928 = Pr (Z = C ) The area between points –1.1 and 2.5 in the NORMAL curve is
Going to the normal table 0.1928 falls within 0.500 A 0.8581 B 0.4938 C 0.3643 D 0.1295 E 0.1125

287
Chapter twenty five Eg3. Our previous example ( i.e Eg1 ) is a 3 × 2 matrix
Matrix
Matrix (matrices for plural) like every other topic in 1 2 3
 
mathematics is to be covered from the very basics then Eg4. B   5 6 7  is a 3×3 matrix
we advance to the highest point that the syllabus  9 10 0 
permits. The test on this topic shall be simple but  
technical as it is with all mathematics questions- they  1 y 4
 
are not meant to consume your time. Eg5. C   x 6 7  is also 3×3 matrix
Defn: A matrix is a simple mathematical structure that  0 0 2
holds numerical information in rectangular form usually  
enclosed by bracket Eg6. 11 25  is a 2×2 matrix
D   
Eg1. A college has its number of students in each of 1 0 
the SS class classified as follows: 1
Male female  
Eg7. M   2  is a 3×1 matrix ( column matrix )
SS1 14 8  3
 
SS11 6 12
SS111 21 18 Eg8. N  4 5 6 10 is a 1×4 matrix( row matrix)
The above information can be represented in matrix form say Matrices are usually denoted by capital letters while their
14 8 elements are in lower case.
 
Matrix A   6 12  b b  b b 
B   11 12  and not b   11 12 
 21 18 
   b21 b22   b21 b22 
Where the first row represents SS1, the second SS11 Types of matrix
and so on. Similarly, the first column is the number of
( i ) Zero matrix
male students and the second column for female
This is a type of matrix whose entry are all zero
students. Thus it is important to retain each element in
its original position when dealing on matrix. It is 0 0 0
 0 0  
customary to write a matrix in the form A2  2    or B3  3   0 0 0 
 0 0 0 0 0
 a11 a12 a13   
 
A =  a 21 a 22 a 23  three by three matrix ( ii ) Square matrix
a  This is a matrix whose number of rows equal to number of
 31 a32 a32 
columns. i.e m×n matrix but m = n
b b  1 4 5
B   11 12  two by two matrix  2 4  
 b21 b22  A2  2    or B3  3   0 7 11
6 8 13 17 19 
The horizontal components  
( a11 a12 a13 ), ( a21 a22 a23 ) and ( iii ) When the elements’ row  column we say that the matrix
is non-square matrix
( a31 a32 a33 ) are the rows while the vertical components
1 2 5 10 15 
 a11 
 
 a12 
 
 a13 
  C    D   21 3 
 a 21  or  a 22  or  a 23  3 4 6 0 1
a  a  a   
 31   32   33 
( iv ) identity matrix – it is denote by I or l
are the column of the matrix .
1 0 0
Eg2. Identify the position of the elements listed below 1 0  
I 2  2    I 3 3   0 1 0 
in 3 by 3 matrix: a22 = 12 and a31 = 24 0 1 0 0 1
Solution  
Element a22 being 12 i.e the element on the second row Students should master the above pattern of generalizing the
and 2nd column. identity matrix for 2×2 and 3 by 3.
a31 = 24 i.e element on the 3rd row and first column
Class discussion/self test Addition & subtraction of matrices
Try to identify elements in a23, a32 and a12 of the matrix 6 4  2 3
in, example 1 above. Eg. 1 If A    ; B   
12 6   4 5
Definitions find ( i ) A + B ( ii ) A – B
A matrix with m rows and n columns is called an m by
n matrix. Where m × n or m by n is called the size or
shape of the matrix.

288
Solution Multiplication of matrix by a number
 6  2 4  3 8 7 ( scalar multiplication)
i. A  B    =  
12  4 6  5 16 11 Eg.1 If A   1 2  find ( i ) 5A (ii) – A (iii) –2A
    3 4
 
 6  2 4  3   4 1 Solution
ii. A  B    =    5 1 5  2 
12  4 6  5   8 1 ( i ) 5A     5 10 
  
 5 3 5 4   15 20 
Eg. 2 Perform C + D and C – D on the matrix below: (ii)  A    1  2 
 
 3 2 4   1 2 3  3 4 
    Students/readers to find – 2A
C   1 0 2 and D   0 1 1
  3 1 6  0 2 1
     1
 0
Solution Eg2. If B    find ( i ) –B ( ii ) – 3B (iii) – ½
 2 4 7   15 10 
 3  (1) 2  2 4  3    B
 
C  D    1 0 0 1 2 1  =   1 1 3  Solution
  3  0 1  2 6 1  3 3 7
    ( i )  B   0 1  observe that 0 × (-1) = 0 no negative in zero
15  10 
 
 3  (1) 2  2 4  3   4 0 1
    Students/readers to find – 3B
C  D    1 0 0 1 2 1  =   1  1 1
  3  0 1  2 6 1    3  1 5 ( iii ) 0 1
2 
     1 2 B   
 15 2  5 

Eg.3 Part-time teachers/school matrix has the structure:


Schools Eg3. The following matrix X gives the prices of comparable
A B articles by size and type ( in N )
Teachers 1 x x A  0.40 0.10 
 
Number 2 x x X  B  0.30 0.50 
3 x x C  0.20 0.60 
and it represents the number of periods taken by a
If all prices were increased by 50%, the new prices matrix Y will be?
particular teacher in a particular school in a certain Solution
week. If Wi is the teacher/school matrix for week i and Normal prices 100%
4 0 3 1 2 0 Increment 50%
      New price = 100 + 50%
W1   0 0  ; W2   2 1  ; W3   2 4
2 3 0 = 150%
   2  2
 0  = 150 = 1.5
find the value of 100
( a ) W1 + 2 ( where W1 + 2 = W1 + W2 )  0.40 0.10 
 
( b ) W 1 + 2 + 3 ( c ) W 1 + 2 + 3 - W2 Y   1.5   0.30 0.50 
Solution  0.20 0.60 
 4  3 0  1 7 1  
     1.5  0.40 1.5  0.10   0.60 0.15 
( a ) W1  2  W1  W2   0  2 0  1  = 2 1    
 2  0 3  2 2 5    1.5  0.30 1.5  0.50    0.45 0.75 
    1.5  0.20
 1.5  0.60   0.30
 0.90 
7  2 1 0  Students/readers poser
  9 1
( b ) W1  2 3  W1  2  W3   2  2 1  4  =   If all the prices were decreased by 50%, the new price matrix Z
4 5 will be?
 2  2 5  0 4 5 
  

1999/20 (Nov)
 9  3 11  6 0
     1 2 4 5 
( c ) W1  2 3  W2   4  2 5  1  = 2 4 If K =   and L =  
 4  0 5  2 4  3 4 6  7
   3  Find a matrix such that 2K + L + M = 0
A   2  9  B C   2 9  D  3 7 
 9  4 
 6  1
     
  12  1   0 15   12 1  
Solution
First let us do the scalar multiplication of 2K

289
 1 2   2 4  2 1  2 1
2K = 2   i.e.   K 2     
3 4  6 8  3 4  3 4
Next, 2K + L + M = 0  2  2 1  3 2  1  1  4 
  
  2 4 4 5   3  2  4  3 3  1  4  4 
  +   + M = 0
 6 8 6  7  4  3 2  4   7 6
     
 6 12 3 16  18 19 
  2  4 4  5
  + M =0  7 6   2 1  1 0
  Therefore , K  K  I         
2
 6 6 8 7 
18 19   3 4   0 1 
 2 9  7  2 1 6  1 0 
  + M = 0   
12 1  18  3  0 19  4 1
 2 9   2  9 10 7 
Therefore M = –   =   A =   ( B )
12 1    12  1   21 24 
2004/22 PCE 2005/7 Exercise 25.0
1 4 0 4  Given that P =  2 1  and Q =  4  8  ,
If R    and S    find 3R + S    
 3  2
3 2  1  5 5 1
find ( 2P – Q )
A  3 16  B  3 16  C  16 3  D 16 3 

10 1 10 1    10 1  1 10 
     
  6 17    2 9 C 0  6
Solution A   B     D  0 10 
First, we resolve 3R  3 1  4 1 9  8  
9  4
1 4  3 12 
3R  3   =   2001/20 UME Exercise 25.1
3 2 9 6  3  2 4 
  then -2P is
 3  0 12  4   3 16  If
P  5 0 6
Therefore 3R  S    =   (B)  7 5  1
 9  1 6  5   10 1   
 6 4  8
 6 4  8  
2003/31 PCE A.   B.   10 0 6 
 5 0 6 
 2  1 10 5   7  1 
  14 5  1 
 
If matrices P and Q are   and    5
5 6   3 2  6 4 2   6 4 8 
C.   D.  
respectively, find the matrix 2P– Q   10  2  12    10 0  12 
A   6 3  B   6  7  C   6  7  D   6 3    14  10
 2    14  10
 2 
  7 10   3 10   7 10   7 10 
       
Solution 2002/16 UME Exercise 25.2
First, we resolve 2P
If P   2 1  and I is a 2×2 unit matrix, evaluate P2-2P+4I
2  1 =  4  2  and  Q    10  5    3 0
2P  2        
5 6  10 12   3  2
 9 4  3 0  1 0  1 4
A.   B.   C.   D.  
 4 10  2  5   6  7   12 1   0  3 0 1 4 1
2 P  Q   =   ( C )
 10  3 12  2   7 10 
2005/50 UME Exercise 25.3
If M and N two matrix defined by
2001/8
 1 3 2 1  2 3 
2 1   and
M   4 5  1

N   4 1 5 
 , evaluate 2M–3N.
Given the matrix K    the matrix k2 + k + I,
3 4  3 2 0   2  3  1
     
where I is the 2×2 identity matrix is
 1 12  5    1 12 5 
A B 10 7 C  7 2 D  6 2 A.   B.  
 21 24  12 21 13 20 
9 8
   4 13 13  4 7 13 
23         12
 22
 13 3   0  5  3
 
Solution
 1 0 5    1 12  5 
Recall that the 2×2 identity matrix I   1 0  C.   D.   4 13  17 
0 1  4 7  17   
   0
 5 3    12 13 3 
 

290
Multiplication of matrix by a matrix Students/readers’ poser
Multiplication of a row matrix by a column matrix can 3 4 1 6
be done by combining the matrix as follows: If A    and B   
 1 2   0 5 
 b1 
A  a1 a 2  and B    find ( i ) A B ( ii ) A2 = ( i.e A2 = A × A )
 b2  (iii ) B2 ( iv ) show that AB  BA i.e multiplication
A × B = A•B = a1b1 + a2 b2 of matrix is not commutative, but
3 AA = AA if they are the same
Eg.1 If A  1 2 and B    find AB
4  General discussions/observation
Solution Condition under which multiplication is possible with matrix
AB = 1 × 3 + 2 × 4 is once number of rows = no of columns. From the list
=3+8 below, identify as many as possible two matrices that fulfill
= 11 such condition
 d1 
Also if C  c1 c 2 c3  and   2
 
10 
 
A   3  B   7  , C  1 0 3 , D    , E  8 1
D   d2  , 0
d  4 8 5 
 3    
Eg.2 If the matrix X is used to hold the records of sets
1 0 1 2 3  1 0 2 1 2
A, B and C of books in a library and matrix Y to hold  
the unit cost ( N 000,000 ) each set as follows F    , G   0 4 10  , H    , J   3 4 
5 6 1 0 5  0 1 5 5 6
 2.50     
A B C  
X   10 25 5  Y   0.50  , find XY 15   7 0
K    , L  0 17 , M    , N   4 9 0

 0.20  0 2 1 
  3 9 1 
 2.50  some are CA, ED, GA, FD, LK and so on.
 
X Y  10 25 5   0.50  2004/37 Neco
 0.20 
   cos x  sin x   cos x sin x 
Given that P =   and Y=   .
= 10 ( 2.50 ) + 25( 0.50 ) + 5( 0.20 )
 sin x cos x    sin x cos x 
= 25 + 12.5 + 1.0
= N ( 38.5 ) 000 000 Find the matrix PY
= N 38 500 000  0 1 1 1
A  1 0  B  0 0  C   D  0 1  E  
This is the total cost of all the books in the library. 0 1
 
1 1
   0 1 1 0
  0 1
The principle in the example above is to be applied
Solution
always in multiplying matrices i.e. row by column
operation.  cos x  sin x
2 2
cos x sin x  cos x sin x
PY =  

 sin x cos x  sin x cos x sin 2 x  cos 2 x 
Pattern of movement
=  1 0  (A)
 
0 1
 2 3 5 7 
Eg. 3 If A    and B    find AB
 6 4  9 11  2002/19 Neco
Solution 2 1 2 
  3
 2  5  3  9 2  7  3  11 If A =  1 2  2  , What is A ?
A  B    2  2  1 
 6  5  4  9 6  7  4  11  
 10  27 14  33  37 47  A 3A B 6A C 8A D 9A E 10A
=   =  
 30  36 42  44   66 86  Solution
A3 is same as A2 × A and A2 is A × A
1 8 3 4 2 1 2  2 1 2 
Eg.4 If X    and Y    find XY    
0 2 5 6 A × A =  1 2  2  1 2  2
 2  2 1   2  2 1 
   
 1 3  8  5 1 4  8  6 
X Y   
 0  3 2  5 0  4  2  6  4 1 4 2  2  4 4  2  2
 
 3  40 4  48   43 52  =  2  2  4 1  4  4 2  4  2 
=   =    4  2  2 2  4  2 4  4 1
 0 10 0 12   10 12   
291
9 0 0  3 4
  XA =(3 4 ) .  
=  0 9 0  2 5
0 0 9 = (3  3 + 4  2 3  4+4  5)
 
Then A3 = A2 × A = (9 + 8 12 +20)
= (17 32 )
9 0 0  2 1 2 
    2002/23 PCE Exercise 25.4
=  0 9 0  1 2  2
 0 0 9   2  2 1   2 3  1 3
    If P    and Q    then the matrix PQ is
 1 3  2 1
18  0  0 9  0  0 18  0  0 
  A.  8 9
 B.  5 12 
 C.  8 7
 D.  5 9 
=  0  9  0 0  18  0 0  18  0  7
 6  5
 9  9
 6  12 5 
 
 0  0  18 0  0  18 0  0  9 
  1995/25 UME Exercise 25.5
1 2  2 1
18 9 18  If X    and Y    find XY
 
= 9 18  18  i.e. 9A (D)  0 3  4 3
18  18  9  10 7 
  A.   B.  2 7  C. 10 4  D.  4 3 
     
2000/23 PCE 12 9   4 17  4 6 10 9 
If X  x1 x2  and Y   3 0  obtain the matrix
 2009/8 Neco Exercise 25.6
 4 5 

 2 4 1 1 2 1
XY If P =   , B =   Find P.B
3 1 2
A.  3x1  4 x2  5 x2  B.  3x1  4 x2 5 x2  C. 3 0 2  4 4 3
 
 3 x1 0  D.  3 x1  4 x2 
     18  12 13   18 12 13  18 12 13
  4 x2 5 x2   0 5 x2 
A.   B.   C.  
Solution   11 14 9  11 14  9   11  14 9 
XY  x1  3  x2  (4) x1  0  x2  5 18 12 13   11 14 9 
  3x1  4 x2 0  5 x2  D.   E.  
=  3x1  4 x2 5 x2  B
 11 14 9  18 12 13

2003/29 PCE Cases of equation, multiplication, addition & subtraction


Evaluate  2 1   1 1 2005/12c adjusted
   
 3  1  0 1
1 2   3 2
A. 2 1  B. 2 3 C.   2 1 D. If A =   and B =  

0

1

3

2 

 3 0
 3 4  1 4
Find C such that : AC = B
3 1 
  Solution
2 1
We list the C a 2x2 matrix such that AC = B
Solution 1 2  a b    3 2
    
2 1   1 1  2  11  0 2  1 1  1  3 4  c d   1 4 
      
3  1  0 1  3  1  (1)  0 3  1 (1)  1  a  2c b  2d    3 2 
20 2  1     
    3a  4c 3b  4 d   1 4 
3  0 3  1 
Equating entries
2 3 ( B )
   a + 2c = –3 b + 2d = 2
 3 2 
3a + 4c = 1 3b + 4d = 4
2006/12(a) Solving simultaneously
 3 4 3a + 6c = –9 3b + 6d = 6
Given that A =   and X = (3 4)
 2 5 – (3a + 4c = 1 ) – (3b + 4d = 4)
Evaluate XA 2c = – 10 2d = 2
Solution c = –5 d=1
A first look at the question gives the impression that Substituting for c value Substituting for d value
a = –3–2(–5) i.e 7 b = 2–2(1) i.e 0
multiplication condition in matrix is met
i.e Number of rows = number of columns:
 7 0
When viewed by appearance as AX. But we were asked Thus C =  
to evaluate XA.   5 1
292
2004/2 2 5
 2  3  x   1  B x, y     3, 5
Solve the equation     =   8 7 
 1 3   y   2  2 5  x  3  2 8   x   3
A x = –1, y = – 1/3 B x = 1, y = – 1/2 C        D       
 8 7   y  5  5 7   y   5
C x = 1, y = 1/3 D x = -1, y = 1/2
Solution Solution
Since matrix multiplication is not commutative
 2  3  x   1  2x  5 y  3
    =  
 1 3   y   2 8 x + 7y = 5
Can only be represented as
 2x  3y   1   2 5   x   3
  =          (C)
 x  3y   2  8 7   y   5
Equating terms
2 x – 3y = 1
x + 3y = 2 2000/9a Neco
Solving simultaneously  2 1   3   2
Find K if     = k  
x = 2 – 3y  1 3   4    3
Substituting Solution
22  3 y   3 y  1 Going by scalar multiplication in matrix, we have
4 – 6y – 3y = 1  2 1   3   2k 
– 9y = 1 – 4     =  
 9y  1 3   4    3k 
3
9 = 9  2  (3)  1 (4) 
  = 
  2k 

y = 1
3 1 (3)  3  (4)    3k 
Substituting for y value   6  4    2k 
x  23 1  
3
  = 
  3 12    3k 


=2 –1   10    2k 
=1   =  
Thus x = 1, y = 1 ( C )   15    3k 
3 Equating elements
2002/2 –2k = –10 Thus, k = 5
 x   6  or – 3k = –15
Given that  3 0   =  , find the values of x and y Thus, k = 5
 3 1 
   y    10 
A x  2, y  4 B x = - 2, y = 2 1996/13
C x = -2, y = - 4 D x = 2, y = 4  3 1   2  y 
Solution If        , find x and y
 2 x   3  10 
 3x  0    6  A x  2, y  9 B x  6, y  9
  =  
 3x  y    10  C x  2, y  8 D x  9, y  2 E x  2, y6
3x = – 6 Thus, x = –2 Solution
and 3x + y = – 10  63   y
    
It follows that 3(– 2) + y = – 10
 4  3x  10 
– 6 + y = – 10
y = –10 + 6  9   y
y=–4     
Hence x = –2 and y = – 4 (C)  4  3x  10 
Equating elements
2002/1
9 = y
Express
Also 4 + 3x = 10
2x  5 y  3
3x = 10 – 4
8 x + 7y = 5 3x = 6 Thus, x = 2
in matrix form x = 2 and y = 9 (A)
 x  2 5  3
A       
 
y 8 7  5
293
2007/4 Neco 2005/40 Neco
 a 1  1 c  What are the values of e in the equations?
If A =   and B =    a2 ab   4 7
 2 b   d 0    =  
b  d e2  d 
 a b    1 0
Find a matrix Z =   if A B = I
c d  A 9 or 5 B 8 or 4 C  2 D  1 E  2 or  2 2
(Where I is a 2 x 2 identity matrix) Solution
 1 We are Simply to solve e2 – d = 0
 0 1
 2  0 1
A   1  B  1  C  1  But, you can not find e without knowing d, and you notice
2 0 0 that d is tied to b and b is tied to a, No short course
 2   2   2 
 a2 ab   4 7
1 0   0 2  
b  d e 2  d  = 1 0
 
D  2  E  1     
1
 1 2  2  Equating elements
Solution a2 = 4 Thus a = 2
We are told that A B = I a+b =7 Thus b = 7 – 2; b = 5
 a 1  1 c  1 0 b–d =1 Thus b – 1 = d i.e d = 5 – 1  d = 4
       e2 – d = 0, Thus e2 – 4 = 0, e2 = 4 i.e. e =  2 (C)
 2 b  d 0 0 1
2005/6 Exercise 25.7
 ad ac  0  1 0  x 0
     Given that   = 2I, when I is the 2x2 identity matrix,
  2  bd 2c  0  0 1 0 y
Equating elements calculate the value of x2  y2
d–a = 1 ac = 0 A2 B2 2 C3 2 D4
bd – 2 = 0 2c = 1 Thus c = 1/2 2009/28 (Nov) Exercise 25.8
Substitute for C = 1/2 into
5 x   2  4
ac = 0 If     =   , find (x, y).
1
a ( /2 ) = 0  4 1   y  5
a = 2 × 0 thus a = 0 A (3, - 2) B (2, 3) C (- 2, 3) D (-3, 2)
Substitute for a = 0 into
d–a=1 2006/29 Exercise 25.9
d – 0 = 1 Thus d = 1
 2 3   6   3 
Next, we substitute for d = 1 into Given that     =   , find the value of
bd – 2 = 0  1 4   p    26 
b(1) – 2 = 0 P.
b – 2 = 0 Thus b = 2 A–8 B–5 C–4 D–3
a b   0 2 Joint cases of Addition, subtraction & multiplication
Thus Z    will become  1  E
d  1 1994/11
c  2   2 1 1 1 
If P =   and Q =   , find PQ + 3Q
 0 1 3  2
2004/21 PCE
A  0 3  B  C  2  1 D  8  7  E  2  7 
Given that  1 2    1 1    3 11 , find k   
0  3
  6 
4 
   
  
1  2 
5     6 5  6
  9   12  8   6  8
k 5 2
A.-5 B.-3 C.6 D.7 Solution
Solution  2 1  1  1  1 1 
PQ + 3Q =     + 3  
 1  1 2  2 1  1 2  5 

 3 11 
     0 1  3  2  3  2
 k  11  2 k  11  5  5 2 
 2  1  1  3 2   1  1   2  3
=   +  3
  1 4 1 10 

 3 11 
     0  1  1  3 0    1  1   2   9


 6 
 k  2 k 5 5 2 
equating terms in k  2  3  2  2  3  3
=   +  
-k +2=5 or k + 5 = 2 0  3 0  2  9  6
-k = 5 - 2 or k=2-5
- k = 3 i.e k = -3 or k = -3 (B ) 5  4  3  3  8  7
=   +   =   (D)
3  2 9  6 12  8 

294
1994/18 (Nov) 3  2 0  0
 2 1 =  
If A =   and I is a 2x2 unit matrix 3  0 6  2
3 0 
1 0 
Evaluate A2 – 2A + 4I. =  
3 4
 1 4  3 0 Therefore LHS = R HS Q E D
A   B   C  9 4
  12 1 
4 1  0 3   
1997/16
D  1 0
 E  1 0   2 1   3  2
 3 1    If M =   and N =   , find MN – NM
 0 1
  1 3   1  4 
Solution
  1 1 1 0   9  17 
Recall that the 2x2 identity matrix is  1 0  A   B   C  
0
 1   0 1 0 1  0  21
A2 – 2A + 4I
 0 0 2 1
D   E  
 2 1  2 1  2 1 1 0  0 0  1 2
=     – 2   + 4  
  3 0   3 0   3 0 0 1 Solution
 2 1   3  2
MN =    
=  2  2  1 (3) 2  1  1 0  +   4  2 +  4 0
  3  2  0  (3)  3 1  0  0     
0   0 4    1 3  1  4 
   6
 43 20   0  2  2  3  1  1 2  2  1  4 
=   +   =  
  6  0  3  0 6 4    1  3  3  1  1  2  3  4 
 1 2   0  2   6  1  4  4
=   +   =  
  6  3  6 4   3  3 2  12 

1 0  5  8 
=   (E) =  
0 1  6  10 

1997/4 (b)   3  2  2 1
NM =    
1 0   1  4   1 3 
If M =   and I the unit 2x2 matrix, show that
1 2 
  3  2  2  1  3  1  2  3 
M2 = 3M – 2I =  
 1  2  4  1 1  1   4  3 
Solution
LHS M2 = M × M
  6  2  3  6  4  9 
1 0  1 0  =   =  
=      2  4 1  12   6  11
1 2  1 2 
1  0 0  0 1 0  5 8   4  9 
=   =   LHS Therefore MN – NM =   –  
1  2 0  4  3 4  6  10   6  11
Recall that 2 x 2 Identity matrix  5  4  8  9    1 1
1 0  2 0 =   =   (A)
is   and 2I =    6  6  10  11  0 1
0 1  0 2
1997/4b
3 0 1 0 
At the RHS 3M =   If M =   and I the unit 2x2 matrix,
3 6 1 2 
Show that M2 = 3M – 2I
3 0  2 0 Solution
3M–2I =   –   We are to show that L H S = R H S
3 6  0 2 1 0  1 0 
L H S i.e M2 =    
1 2  1 2 
295
1  1  0  1 1  0  0  2    3 13   13  3  13 21
=   A   B   C   D
11  2 1 1 0  2  2   9  2  2 9  15 10 
1  0 0  0  10 15  13 16 
=     E  
 21 13   9 10 
1  2 0  4 
1 0
=   1999/19 (Nov) Exercise 25.11
3 4 Given that:
1 0  1 0  cos x  sin x   cos x sin x 
R H S i.e. 3M – 2I = 3   – 2   P =   , Q =  
1 2  0 1  sin x cos x    sin x cos x 
3 0  2 0  And I is a 2x2 identity matrix, find PQ + 2I
=   +  
3 6  0  2
1 0  3 0 0 1  3 0
1 0 A   B   C   D  
=   0 1  0 3 1 0 0 1
3 4 
Thus L H S = R H S QED 2000/13 Exercise 25.12
Given that P =  1 2  and Q =   2  1  ,find PQ – QP
2007/5 Neco  1 3  
 a 2  3  5 d 5   2 1 g    1  3
If      
 1 c  2   e 7 3     3 6 h 1 0   0 0 1 0 1  1
 b 4  8   4 f  6  4
     11 i  A   B   C   D  
1 1  0 0 0 1 1 0 
a b c 
find the matrix  
d e f  2002/6 (Nov) Exercise 25.13
g h i 
 
 3 1   0  4
  7 0 13   3 0 13  Given that X =   and y =   ,
A  B 
 2  1  2 1
 1 2  7  1 2  7
  8  5  2   8  5  2 find the matrix ( xy  2 x ).
   
 3 0 13   3 0 13 
C  3 8 13  D   E    2 11   5 10 
1 2 7  1 2  7
 1 2 7 
  8  5  2

  8  5  14 
A   B  
  8  5  2
        2  7    4  6 
Solution
 a 2  3  5 d 5   2 1 g   2 15   8 9 
     
 1 c  2   e 7 3   3 6 h C   D  
 b 4  8  4
   f  6   4 11 i    4  6   6  5

a–5=–2 thus a = 3 2009/23 Exercise 25.14


–1 – e = – 3 thus e = 2  1 1
b – (– 4) = 4 thus b = 0 If P =   , find (P2 + P)
 2 1
2–d=1 thus d = 1  4 3  4 3  3 2  3 2
c–7=6 thus c = 13 A   B   C   D  
4 – f = 11 thus f = – 7  6 1  6 4  6 1  6 4
–3 – 5 = g thus g = – 8 Transpose
–2 – 3 = h thus h = – 5 t
–8 –( –6) = i thus i = –2 Defn: The transpose of a matrix A, written A is the matrix
obtained by writing rows of A in orders as columns
a b c   3 0 13  1 2  1 3
    If A    then At   
Matrix  d e f  =  1 2  7 B.
g h i    8  5  2 3 4  2 4
    6
If B  6 11 then Bt   
2000/43 Neco Exercise 25.10 11
 1 3  2 4 0 1 4 0 7 3
If X =   and Y =   Evaluate X2 + 3Y – Y2    
2 2 3 0 If C   7 10 2  then C t   1 10 5 
    3
 5 16  4
 2 16 
296
2003/29 Neco 1993/2 (Nov)
 2 4  x x3
Given that A =   obtain (A + AT )T Factorize completely
 1  6  y y3
  4 8   4 2  A xy1  xy1  xy B xy y  x  y  x 
A   B  
 2  12   8  12  C 
xyx  y  x 2  xy  y 2  D x  y x 2 2
 xy  y 2 
 4 5 
C 
  4 5
 D  
 2 1 
E   E x  y x  y x  xy  y 
2 2

 5  12    12 5   4  6 Solution
Solution
x3
= xy  x y
x 3 3
 2 4   2 1 
A =   and AT =   y y 3

 1  6   4  6 Factor out common term


 2 4   2 1  = xy( y2 – x2 )
A + AT =   +   Note that y – x2 is a difference of two squares
2

 1  6  4  6 = xy y  x  y  x  (B)
 4 5 
=   2003/18 (Nov)
 5  12 
7 2 1
 4 5  If D = , find D 2
Thus (A + AT ) T =   C. 3 6
 5  12 
A6 B4 3 C 32 D 36
Students/readers’ poser Solution
Give the transpose of the matrices below: 7 2
13 1 0  D=
 4 0   3 6
P    
 , Q  10 0 1 , K  15 4 2 
 1 3  0 1 7 = 7 × 6 – (–3 × 2)
  = 42 + 6
D = 48
Determinant of matrix 1
Thus D 2 = 48
The deteminant of any matrix say B is denoted by B
or det B is a number computed by the format shown =4 3 (B)
below; depending on the order of the matrix i.e 2×2 or
3×3.If the value of matrix determinant is zero then the 2003/1
matrix is called singular matrix, otherwise it is non- Solve the pair of equations:
singular and it is only non-singular matrix that has x y x2 y2
inverse. 8  7 xy
3 2 3 2
2×2 matrix determinant Solution
 a b  then, 2 x – (– 3y) = 8 and 2 x 2 – (– 3y2) = 7 x y
A   
c d  2 x + 3y = 8 and 2 x 2 + 3y2 = 7 x y
det A = ad – bc
2 x = 8 – 3y
Eg. ( i ) If A   3 4  find detA i.e. x = 4 – 3 y
1 2
  2
Solution Substituting into the second equation
det A = 3×2 – 1× 4 2(4 – 3 y ) 2 + 3y2 = 7(4 – 3 y) y
=6–4 2 2
=2 2[16 – 12y + 9 y ] + 3y = 7(4y – 3 y2 )
2 2
4 2
32 – 24y + 9 y  3 y  28 y 
2 2 21 y2
Eg.( ii ) If D   2 6  find D 2 2
 1 4 Rearranging
 
Solution 9 2 21 2
y  y  3 y 2  52 y  32  0
D = 2 × 4 – 6×(- 1) 2 2
= 8+6 Multiply through by 2 to clear fraction
= 14 9y2 + 21y2 + 6y2 – 104y + 64 = 0
i.e. 36y2 – 104y + 64 = 0
Reducing further

297
18y2 – 52y + 32 = 0 x4 x
 2
Factorizing x 1 x 1
18y2 – 36y – 16y + 32 = 0 ( x + 4 )( x – 1 ) – x ( x + 1 ) = 2
18y( y – 2 ) – 16( y – 2 ) = 0 x2 – x + 4x – 4 – x2 – x = 2
( y – 2 ) ( 18y – 16 ) = 0 2x – 4 = 2
y – 2 = 0 or 18y – 16 = 0 2x = 2 + 4
y = 2 or 16 2x = 6
18
2x = 6
y = 2 or 8 2 2
9
x
Next, we find the x values from 2 = 8 – 3y x=3
When y = 2, Recall that matrix Q is lQl without strokes. Thus substituting
2x = 8 – 3(2) for x
x=1  3 4 3 
Q   
When y = 8
9  3  1 3  1 
8  7 3
2 x = 8 – 3  =   (D)
9  4 2 
8
= 8–
3 2000/24 PCE
2 x = 16 Thus, x= 8
3 3 If G   2 3  and

 4
P  
1  , find
 GP
 1 4 2
2000/2    3 
The matrices P and Q are: A. – 55 B. – 50 C. 66 D. 77
1 3  3  k  Solution
P =   , Q =  , Where K is a constant. We are required to perform two operations here:
 4 1  5  2 - matrix by matrix multiplication i.e GP
(a) Find PQ (b) If PQ = 144, find the value of K - determinant of the result of GP.
Solution Firstly, GP =  2 3   4 1 

 1 4   3 2
   
3  3  k 
1
(a) PQ =     2  4  3  3 2  1 3  (2) 
4  1  5  2    
 1  4  4  3 1  1 4  (2) 
  3  15  k  6 
=    89
 
2 6 

  12  5  4k  2   4  12 1 8 
 12  k  6  =  17  4
 

=    16  7
 
  17 2  4k  17  4
12  k  6 Next, GP =
Thus PQ  16  7
 17 2  4k = [ 17× (– 7 ) – (– 4 ) ×16 ]
= 12(2 – 4k) – (–17) (– k – 6) = – 119 + 64
= 24 – 48k – 17k – 102 = –55 (A)
= – 65k – 78
2002/24 PCE
(b) If PQ = 144 8 4 3 9
Thus – 65k – 78 = 144 If  find the value of y.
4 y 5 7
– 65k = 144 + 78
– 65k = 222 A. – 5 B. – 4 C. 4 D. 5
Solution
k =  222
65 i.e  3 27 You recall that l l means determinant. Thus we are to find
65 the determinant of both sides then equate them. i.e.
2001/24 PCE ( – 8 × y ) – (– 4 × 4 ) = ( 3 × 7 ) – ( 5 × 9 )
If Q  x  4 x
 2 then the matrix Q is – 8y + 16 = 21 – 45
x 1 x 1 – 8y + 16 = –24
A   7 3  B 7 3  C  7 3  D  7 3  – 8y = – 24 – 16
 4    
 2   4 2 
 4
 2 
 
 4 2  – 8y = – 40
Solution – 8y = – 40
First, we use determinant principle to find x, then –8 –8
substitute for x value to get the required matrix y = 5 (D)
298
1998/24 PCE 1999/15 UME
If a a  5 2 then value of a is Given that Q   6 0  and Q  P   7  2 

2 3 6  4 5  6 8
a    
A. 5 or – 6 B. 4 or – 6 C. – 4 or 6 D. – 5 or 6 evaluate Q  2 P
Solution
We apply determinant rule to both sides A. 90 B.96 C. 102 D. 120
(a × a ) – (a × 2 ) = ( 5 × 6 ) – ( 2 ×3 ) Solution
a2 – 2a = 30 – 6 First, we identify P
a2 – 2a = 24 i.e. P  Q  P  Q =  7  2  –  6 0 
 6 8   4 5 
a – 2a – 24 = 0
2
   
Factorizing the quadratic expression  76 20 
  
a2 – 6a + 4a – 24 = 0  6  4 8  5 
a( a – 6 ) + 4( a – 6 ) = 0  1  2  Thus  2 4 
(a + 4 ) (a – 6) = 0 P    2P   
 2 3   4 6 
a + 4 = 0 or a – 6 = 0
a = – 4 or 6 ( C)  6 2 0  (4) 
and Q  2P   
1997/24 PCE  4 4 5 6 
If N    1  5  and M   0 4  ,  8 4 
 3 2   6 3  =  
    8 11 
find 3M  N
8 4
A. 262 B. 48 C. – 218 D. – 248 Therefore, Q  2 P 
Solution 8 11
 0 4  i.e  0 12  and = ( 8 × 11 ) – [ 8 × (– 4 ) ]
3M  3    
 6 3   18 9  = 88 + 32
 1 5  = 120 (D)
 N    2010/15 Exercise 25.15
  3  2 
3 x
 0 1 12  5  i.e  1 17  If = - 2, find the value of x.
3M  N      2 x2
 18  3 9  2   15 7 
A–8 B–4 C4 D8
Therefore, 3M  N  1 17
15 7 2004/27 Neco Exercise 25.16
= ( 1 × 7 ) – ( 15 × 17 ) Which of these matrices is non – Singular?
= 7 – 255 A  6 3  B  4 2  C  2 1  D  3 1 E  1 0 
= – 248 ( D )          
 4 2 2 1 4 2  3 1 0 1
1996/23 PCE
Given that P   1 x  and Q   2  1  , 2001/23 PCE Exercise 25.17
 4 3   3 3 
    If the determinant of a matrix is zero, the matrix is called a…..
find the value of x for which P is 2 less than Q A singular matrix B. diagonal matrix
C. Unit matrix D. Symmetric matrix
A. – 1 B. 1 C. 2 D.3
Solution
1999/22 PCE Exercise 25.18
The statement: which P is 2 less than Q
Find the value of x for which
Implies P = Q –2 OR P +2 = Q x  3 x 1
 0
Using the latter x x2
P +2 = Q A. 5 B. 4 C. 3 D.2
(1 × 3 ) – ( 4 × x ) + 2 = ( 2 × 3 ) – ( 3 × – 1 )
3 – 4x + 2 = 6 + 3 1995/24 PCE Exercise 25.19
5 – 4x = 9 Calculate the determinant of  2 4 
 
– 4x = 9 – 5  2 8 
– 4x = 4 A. 8 B. 6 C. – 6 D. – 8
– 4x = 4
–4 –4 1994/27 PCE Exercise 25.20
x = – 1 A. Evaluate the determinant of  3  2 
 
 8 5 
A. 31 B. 1 C. – 1 D. – 31

299
3×3 matrix determinant 4(12 + 2x) –3(–3–2x) +2(2–8) = 17
 b11 b12 b13  48 + 8x + 9 + 6x – 12 = 17
  14x + 45 = 17
B   b21 b22 b23  then
14x = 17 – 45
 b b b 
 31 32 33  14x = –28
b b b b b b x = – 28/14 i.e –2 (B)
B   b11 22 23  b12 21 23  b13 21 22 1997/4a
b32 b33 b31 b33 b31 b32
3 4 5
= If = 0, find x
1 x 1
 b11 b22 b33  b23 b32   b12 b21 b33  b23 b31   b13 b21 b32  b22 b31 
1 0 1
The sign attached to b11, b12 b13 on and on is given by Solution
the chess board format or
3 x 1  4 1 1  5 1 x  0


 
 1 i j
where i is row and j is column 0 1 1 1 1 0
  
b11 = ( - 1 )1 +1 = ( - 1 )2 = + 1 3 (x – 0) – 4(1+1) +5(0–x) = 0
   
 b12 = ( - 1 ) 1 + 2 = ( - 1 )3 = - 1 3x – 8 – 5x = 0
and so on 3x – 5x = 8
–2x = 8 Thus, x = – 8/2 i.e – 4
2004/19 Neco (Dec)
Determine the value of the determinant of the matrix given as:
2000/10a (Nov)
3 5 1
 4 3 1 If , find the values of x
  0 4 x  90
A =  2 0 3
x 1 6
 1 1 5 
 Solution
A. 29 B. 25 C. 10 D. – 53 E. – 45 3 4 x  5 0 x 1 0 4
 90
Solution 1 6 x 6 x 1
0 3 2 3 2 0 3[4×6-(-1) × x] -5(0 × 6 – x × x ) +1[0 × (-1) – x × 4] = 90
A 4 3 1 3(24 + x) –5(0 – x2) +1(0 – 4x) = 90
1 5 1 5 1 1
72 +3x +5x2 – 4x = 90
= 4[0×5 – 1×3] – 3[–2 × 5 – 1×3] + 1[–2 ×1 – 1×0]
Rearranging 5x2 – x – 18 = 0
= 4(0 – 3) – 3(–10 –3) + 1(–2 – 0)
5x2 – 10x + 9x – 18 = 0 Factorizing
= 4(–3) – 3(–13) + 1(–2 )
5x(x – 2) + 9(x – 2) = 0
= – 12 – 39 –2
(x – 2)(5x + 9) = 0
= – 53 (D)
x – 2 = 0 or 5x + 9 = 0
1995/8 (Nov) x = 2 or – 9
5
 3 5 4  2004/11a
 
If M =  6  3  5  , Find M 10  x  6 2
 2 2 1  Solve the  6 9 x 4 = 0

A. 5 B. 17 C. 23 D. 65 E. 191 2 4 5 x
Solution Solution
3 5 6 5 6 3 9 x 4 6 4 6 9 x
M = 3 5 4 10  x  (6) 2 0
2 1 2 1 2 2 4 5 x 2 5 x 2 4

= 3[-3×1 – 2×(-5)] – 5[6×1– (-2)×(-5)]– 4[6×2 – (-2)×(-3)] 10– x[(9-x)(5-x)-16] + 6[-6(5-x)-(-8)] + 2[24-2(9-x)] = 0
= 3(– 3 + 10) – 5(6 – 10) – 4(12 – 6) 10–x(45-14x + x2 - 16) + 6(-30 + 6x + 8) + 2(24 – 18 + 2x) = 0
= 3(7) – 5(– 4) – 4(6) 10–x(x2 - 14x + 29) + 6(6x - 22) + 2(2x + 6) = 0
= 21 + 20 – 24 10x2–140x + 290 – x3 +14x2 – 29x + 36x –132 + 4x +12 = 0
= 17 (B) 170 – 129x + 24x2 – x3 = 0
(b) f(x) = 170 – 129x + 24x2 – x3
1996/15 (Nov) f(2)  170 – 258 + 96 – 8 = 0
4 3 2 Thus x – 2 is a factor; applying long division
–x2 + 22x – 85
If  1 4 x  17 , find the value of x
x–2 –x3 + 24x2 – 129x +170
2 2 3 – (–x3 + 2x2 )
A –3 B –2 C2 D3 E6 22x2 + 129x
Solution – ( 22x2 + 44x)
4 x 1 x 1 4 –85x + 170
4 3 2  17 – ( – 85x + 170)
2 3 2 3 2 2
0 0
300
Next we factorise the quotient –x2 + 22x – 85 2002/32 UME Exercise 25.22
= –x2 + 5x + 17x – 85  3 5 4
= –x(x – 5) + 17(x – 5) If  
N   6 3 5 
find N
= (x – 5) (17 – x)  2 2
 1 
Thus 170 – 129x + 24x2 – x3 = 0 A. 65 B. 23 C. 17 D. 91
(x – 2) (x – 5) (17 – x) = 0 200218 Neco Exercise 25.23
x = 2 or 5 or –17 Evaluate the determinant of the matrix given as
1996/3 (Nov) 12 1 5
(a) Expand the determinant
 4 11 7
x2 2 1
5 8  10
f ( x)   1 x3 1
1 2 x2
A. –1088 B. – 1008 C. 1008 D. 1080 E. 1088

(b) Find the zeros of f(x) in (a) 1994/25 UME Exercise 25.24
Solution The determinant of the matrix
x3 1 1 1 1 x  3 1 2 3
  is
x2  (2)  (1) 4 5 6
2 x2 1 x  2 1  2 2 0 1
 
= x–2[(x–3)(x–2) –2] + 2[–1(x–2) –1] – 1[2 – (-1)(x–3)]
A . – 64 B. –57 C. –3 D. 3
= x–2(x2- 5x + 6 - 2) + 2(-x + 2 -1) – 1(2 + x - 3)
= x–2(x2– 5x + 4) + 2(1 – x ) – 1( x – 1)
= x3 – 7x2 + 14x – 8 + 2 – 2x – x + 1 2001/11 UME Exercise 25.25
= x3 – 7x2 + 11x – 5 1 1 1
Evaluate
(b) f(x) = x3 – 7x2 + 11x – 5 3 1 1
f(1)  1 – 7 + 11 – 5 = 0 1 2 1
Thus x – 1 is a factor; applying long division A.-4 B.-12 C.4 D.-2
x2 – 6x + 5
x–1 x3 – 7x2 + 11x – 5 1997/24 UME Exercise 25.26
– ( x3 – x2 ) Find the non- zero positive value of x which satisfies the
– 6x2 + 11x equation
– (– 6x2 + 6x) x 1 0
5x – 5 1 x 1 0
– ( 5x – 5 )
0 1 x
Next, we factorize the quotient x2 - 6x + 5 = (x-5)(x-1)
A. 2 B. 3 C. 2 D. 1
Thus, x3 – 7x2 + 11x – 5 = (x – 5)(x – 1) (x – 1)
And the zeros of f(x)  (x – 5)(x – 1) (x – 1) = 0
x – 1 = 0 twice or x – 5 = 0 1995/24 UME Exercise 25.27
x = 1 twice or x = 5 Find the value of k if
2 1 1
2003/37 Neco
Find the non-zero positive value of b which satisfies the equation 2 1 k  23
b 1 0 1 3 1
1 b 1 0
A. 1 B. 2 C. 3 D. 4
0 1 b

3 2008/11b Exercise 25.28


A4 B2 C D 2 E1
Solution x3 4 3
b
b 1
1
1 1
0
1 b
0
If 5 2 2 = – 24, find the values of x.
1 b 0 b 0 1
2 4 6 x
b(b2 – 1 ) – 1(b – 0 ) + 0 = 0
b3– b – b = 0 2001/5 Neco Exercise 25.29
b3– 2b = 0
5
0 1
b(b2 – 2) = 0 , b2 = 2 i.e b = 2
If 3 1
x = 8, what is the value of x?
2000/5 Neco Exercise 25.21
Given G as  2 4 1  , find the determinant of G 1 1 1
 1 2 0
 3 0 1 A. –9 B. –3 C. 0 D. 3 E. 9
 
A –6 B2 C4 D5 E6

301
Inverse of matrix (2×2 matrix)  9  1 
 14
The inverse of a square matrix A denoted by A–1 =
14 
is given by   13 3 
 28 28 
If A   a b  then A1  1  d  b 
 c d  det A   c a  1999/12 UME
 
exchange of ‘a’ and ‘d’ and negation of ‘b’ and ‘c’ Find the matrix T if ST = I where S    1 1 

 1 2
 
Eg1. Find the inverse of matrix A   4 5  and I is the identity matrix
 2 3 
  A.   2 1  B.   2  1  C.  1 1  D.  1 1 
       
Solution  1 1   1 1   0 1   0 1 
1  3  5  But det A = 3×4 – (5× 2 ) Solution
A1    =12 – 10 i .e 2
det A   2 4  Recall that for any matrix A and its inverse A-1 ;
1 3 5  AA-1 = I
A1   
2   2 4  Thus T is the inverse of S in this case
 32  52  1   2 1 
S 1   
   det S   1  1 
 1 2  But det S = ( -2× -1) – (1× 1 )
Readers to confirm that AA–1 = I 1 1   2 1  =2 – 1 i .e 1
S   
It is true for all matrix ie. any matrix multiplied by its 1   1  1 
inverse equals identity   2 1  B
T  S 1   
 1 1 
2005/12
1 2   3 2 1999/23 PCE
Given that A =   and B =  ,
3 4  1 4  The inverse of  3 2  is
 
 1 4 
–1 –1
(a) find: (i) A (ii) B  2 1
  2 1  3  1  2  1
(b) Use your results in (a) to find (BA) – 1 A  5 5
 B  5 1 310  C  101 2 5  D  5 1 3 5 
  5 10    10 5    10 10 
1 3
Solution  10 10 
Applying inverse formula Solution
1  4  2 Inverse of  3 2  = 1  4 2 
 
(a)i A– 1 =    1 4 
  det   1 3 
d et A  3 1 
But det = ( 4× 3 ) – (2× 1 )
1  4  2
=   = 12 – 2 i .e 10
 2   3 1 
 2 1   52  15 
1  4 2   1 3  D
A– 1 =  3  =   =
 2
1 
2
10  1 3    10 10 
2003/21 UME
A matrix P has an inverse P 1   1  3  . Find
 P
1  4  2  0 1
 
( a ) ii B – 1 =  
d et B   1  3  A   1 3  B  1 3 C  1 3
 

 D  1 3 

 
1   1  0 1 
 0  0   0 1  
1  4  2
=   Solution
 14   1  3  1
Taking the inverse of P will give P i.e from indices law
 2 1  ( P 1 ) 1  P1 i.e P
=  7 7
1 3  1 1 3
 14 14  inverse of P 1 =  
(b)By the inverse property of matrix (AB)– 1 = B– 1 A– 1 det P  1  0 1 
Thus (BA)– 1 = A– 1 B– 1
But det P–1 = 1× 1 – ( 0× -3 )
 2 1    2 1 
  7 7 = 1 – 0 i .e 1
= 3 1   1 3 
 2 2   14 14 
1  1 3
 4 1  2  3  =  
 
7 14 7 14 1  0 1 
= 
 3
  6  1 3 3  1
 14 28 14 28  =   D
 0 1 

302
2004/2 Eg. 1 Solve the simultaneous equations:
Find the values of x for which the matrix 5x + 9y = - 3
6x + 2y = 2 using matrix format
x 2 1 
B =   has no inverse Solution
 2 x  3  The matrix expression is
A 2 or 5 B 2 or 4 C 1 or 3 D 1or 4  5 9   x   3
      
E – 2 or –5  6 2   y   2 
Solution 1
x   5 9  3 
The matrix B has no inverse implies B =0 Then solution       
y   6 2   2 
x2 1 det = 2×5 – ( 9×6 )
i.e =0 1
2 x3  5 9 1  2  9 = 10-54 = -44
But     
x  2x  3  2  1  0  6 2 det   6 5 
x 2  3x  2 x  6  2  0 1  2  9
   
x 2  5x  4  0 44   6 5 
Factorizing
x 2  4x  x  4  0 x  1  2  9   3 
      
x ( x – 4) –1( x – 4) = 0 y  44   6 5   2 
( x – 4)( x –1) = 0 1   6  18 
  
x – 4 = 0 or x –1 = 0 44  18  10 
x = 4 or 1 (D)
1   24 
  
44  28 
2002/12c Exercise 25.30
 2  3
Given that A =   , find A– 1  x  24 
 5 1      44  i.e x  6 and y   7
  44
28
 y  11 11
2004/30 UME Exercise 25.31
The inverse of the matrix  2 1  is 1997/( 23 ) UME
 1 1 Determine x + y if
 
 2 3   x   1 
A  1 1  B  1 1  C  1 1  D  1  1        
1 2   1 2    1 2     1 4  y   8 
      1 2 
A. 3 B. 4 C. 7 D. 12
1999/23 PCE Exercise 25.32 Solution
1
 4 2 x   2 3    1
If the matrix A =   find the inverse of A       
 y   1 4   8
 1 3 det = 4×2 – ( -3 × -1 )
A 1   3 2  B 1  4 2   2  3
1
1  4 3
= 8-3 = 5
10  1  4  10  1 3  But     
 1 4  det  1 2 
C 1  4 2  D 1  3 2 
   
10   1 3  10   1 4  1 4 3 
  
5 1 2 
Solving simultaneous linear equations with
 x  1  4 3   1 
two unknowns using 2×2 matrix      
If the equation is of the form  y  5  1 2   8 
ax + by = c 1  4  1  3  8
  
5  1  1  2  8 
dx + ey = f
The matrix form is then
 a b  x   c  1   4  24  1  20 
          
5  15 
  
 d e  y   f  5   1  16 
Readers to verify that the LHS is gotten by matrix x   4
multiplication to get to step 1 above     
1 y   3
x  c  a b  Thus x + y = 4 + 3
Then solution       
y   f  d e  = 7(C)

303
2004/2 Exercise 25.33 5 8 2 8 2 5
Solve the equation  2 3   x   1  + – +
 1      6 9 3 9 3 6
 3   y   2 
A. x = –1, y = –1/3 B. x = 1, y = –1/2 4 7 1 7 1 4
C. x = 1, y = 1/3 D. x = –1, y = 1/2 – + –
6 9 3 9 3 6
2002/2 Exercise 25.34
3 0  x    6  4 7 1 7 1 4
       + – +
 3 1   y    10  5 8 2 8 2 5
A. x = 2, y = – 4 B. x = –2, y = 4
Cofactors of A is the resulting matrix
C. x = –2, y = –4 D. x = 2, y = 4
 (45  48)  (18  24) (12  15) 
2009/10a Neco Exercise 25.35  
If A =  3 7  =   (36  42) (9  21)  (6  12) 


 4 2  (32  35)  (8  14) (5  8) 

(i) find A– 1 (ii) Compute A.A– 1
(iii) Hence, using inverse method, (x = A– 1b)  3 6  3
 
 3 7   x    1 =  6  12 6
solve the equation     =   .
 3 6  3 
  4 2  y   1  

Solving simultaneous linear equations with Difference between cofactors & determinant
three unknowns using 3×3 matrix (3×3matrix)
Here we employ two methods, the inverse or crammer’s Examples
rule. Find : (a) the determinant and
The matrix sign attached to b11, b12 b13 on and on is (b) the matrix formed by the cofactors
given by the chess board format or of the matrices given below.
 1  3 4


 
 1 i  j where i is row and j is column  
D = 2  5 7 , E = 
  4  6  3  1 1  0
, F =  
  
b11 = ( - 1 )1 +1 = ( - 1 )2 = + 1  1 0 1   2 3  4

    0 1 1  4 3   7 3 5 
b12 = ( - 1 ) 1 + 2 = ( - 1 )3 = - 1    2  
and so on
Solution
will play a prominent role in determining cofactor and Determinant of matrix D
the concept of cofactor is needed. 5 7 2 7 2 5
The concept of cofactors for 3by3 matrix a. | D | = +1  (3) 4
1 1 0 1 0 1
1 4 7
  = +1[–5– (–7)] + 3(2–0) + 4(–2 –0)
A =  2 5 8 = +1(2) + 3(2) + 4(–2)
 3 6 9 =2+6–8
 
=0
Is the making of the matrix elements as:
b. Cofactors of matrix D
5 7 2 7 2 5
      +  
      1 1 0 1 0 1
 5 8 2 8 2 5 
 3 4 1 4 1 3
 6 9  3
 9  3 6


   
1 1 0 1 0 1
 4 7 1 7 1 4  3 4 1 4 1 3
        
      5 7 2 7 2 5
 6 9  3 9  3 6 
      (5)  (7)  (2  0)  (2  0) 
 
    (3)  (4)   (1  0)  (1  0) 
 4 7 1 7 1 4    (21)  (20)  (7  8)  (5  (6) 
      
 5 8 2 8 2 5   2  2  2
       
=  1 1 1 
     
 1 1 1 

Next we pick up the resulting elements in
determinant form using the chess board sign
304
  4  6  3 2002/6 Neco
   1 0 1
E=  1 0 1  If A =  
 4  2 1 3
 2 3   4 2 1
 
a. Determinant of matrix E (a)Find A–1
0 1 1 1 1 0 (b) Show that AA–1 = I, where I is an identity matrix.
| E | = +(–4 )  (6)  (3)
2 3 4 3 4 2 Solution
= – 4(0– 2) + 6(3– 4) – 3(2– 0) AdjA
A–1 =
= – 4(–2) + 6(–1) – 3(2) A
=8–6–6
=–4 Step I find A
b. Cofactors of matrix of E Step II Find cofactor of A
Step III Transposed the cofactor of A to get Adj A
+ 0 1 
1 1

1 0
2 3 4 3 4 2 A dj A
Step IV A–1 =
– 6 3 
4 3

4 6 A
2 3 4 3 4 2 Step I :
+ 6 3 
4 3

4 6 1 3 2 3 2 1
0 1 1 1 1 0 A  1 0 1
2 1 4 1 4 2
  (0  2)  (3  4)  (2  0) 
=  = 1[(1) – (6)] – 0[(2) – (12)] + 1[(4) – (4) ]
    18  (6)     12  (12)   (  8  (24) 
  (6  0) = 1(–5) – 0(–10) + 1(0)
  (  4  (3)   (0  (6)) 
= – 5 + 0 + 0 i.e –5
 2 1 2  Step II: cofactors of A
= 
 12 0  16  1 3 2 3 2 1
 6 1
 6    
2 1 4 1 4 2
F=  1 1  0 0 1 1 1 1 0

 2

3  4   
 7 3 5  2 1 4 1 4 2
 
a. Determinant of matrix F 0 1 1 1 1 0
  
| F | = + 1 3  4  (1)  2  4  0  2 3 1 3 2 3 2 1
3 5 7 5 7 3
= +1(15–12) + 1[– 10 – (– 28 )] + 0   (1)  (6)   (2)  (12)  (4)  (4)  
 
= +1(3) + 1(18) + 0   (0)  (2)   (1)  (4)   (2)  (0) 
= 21  (0)  (1)
  (3)  (2)  (1)  (0) 
b. Cofactors of matrix of F
3 4 2 4 2 3
    5 10 0
3 5 7 5 7 3  
1 0 1 0 1 1
 A Cofactors =  2  3  2
    1 1 1 
3 5 7 5 7 3 
1 0 1 0 1 1 Step III : Adj A i.e AT Transpose of A cofactors.
  
3 4 2 4 2 3  5 2  1
 
  (15  12)  (10  (28))  (6  21)  Adj A =  10  3  1
 
=   (5  0)  (5  0)  (3  (7))   0 2
 1 
  (4  0)  (4  0)  (3  2) 
 Step IV
 3  18 15  A dj A
=  
 5 5  4 A–1 =
4 4 1 
A

Exercises 25.35a  5 2  1
 
Find : (a) the determinant and (b) the matrix formed by =  1  10  3  1
the cofactors of the matrices given below. 5
 0 2 1 
0 1 8  3 1 1  2 1 0
G=   ,A=   , B = 
 1  04 0  2
1 2 0   1 3  1   1 5  1  
1 4 9  1 1 3   1 1 0  =  2 06 0  2
     
 0 04  0  2 

305
 0  0
(ii)Show that AA–1 = I, where I is an identity matrix. = 1  i.e  
 7   1
Solution 7     1
 7  

AA–1 = 
 1 0 1  1  0  4
 
0  2

 x = 0 ( C ) Note that: y = -1 and z = -1
 2 1 3   2 06 0  2
2003/11 Neco (Dec)
 4 2 1  0 04  0  2 
   (a) Find the co-factors of matrix
 1  1  0  (2)  1  0 1 (0.4)  0  0.6  1  0.4 1 0.2  0  0.2  1 (0.2)   5 6 1
  B=  
 2  1  1 (2)  3  0 2  (0.4)  1  0.6  3  0.4 2  0.2  1  0.2  3  (0.2)   7 8 1
 4  1  2  (2)  1  0 4  (0.4)  2  0.6  1  0.4 4  0.2  2  0.2  1 (0.2)    1  15 8 
   
1 0 0 (b) Hence, find the inverse of matrix B.
=   Solution
 0 1 0
0 0 1 Cofactors of B are
 
8 1 7 1 7 8
  
1992/12 (Nov)  15 8 1 8  1  15
1 1 2
Given that = 7, find the value of x 6 1 5 1 5 6
1 0 1   
2 2 1  15 8 1 8  1  15
That satisfies x + y – 2z = 1, x + z = –1 6 1 5 1 5 6
and 2x – 2y + z = 1   
A -7 B -1 C0 D1 E7 8 1 7 1 7 8
The alternative is using crammer rule
 (64)   15  (56)  (1)  (105)  (8)  
Solution  
=   (48)  (15)  (40)  (1)  (75)  (6)
 1 1  2 x
     1  (6)  (8)  (5)  (7)  (40)  (42) 
1 0 1   y  =   1 
2  2 1   z   1  79  57  97 
   =  
  63 41 69 
and our solution is  2 2  2 
1 
1  2   1
 x  
1
 = 1    1 adj B
 y  0 1    1 (b) B 
B
 z 2
   2 1   1 
adj A First we find B
1
By inverse rule for any matrix A; A 
A 8 1 7 1 7 8
B =5 6 1
cofactors of the main matrix are  15 8 1 8  1  15

0 1

1 1

1 0  5(64)  (15)   6(56)  (1)  + 1(105)  (8) 
2 1 2 1 22 = 395 – 342 – 97
1 2 1 2 1 1 = – 44
  
2 1 2 1 2 2 Step II = A d j B is the transposed of B cofactors
1 2 1 2 1 1  79  63  2 
   Adj B =  
0 1 1 1 1 0   57 41 2
  97 69  2 

 (0)   2  (1)  (2)  (2)  (0)  
   B 1  adj B
=   (1)  (4)  (1)  (4)   (2)  (2)  B
 (1)  (0)   (1)  (2)  (0)  (1)   79  63  2 
 = 1  
   57 41 2
2 1  2  2 3 1 44  
=  and A Transpose =     97 69  2 
3 5 4  1 5  3
1 2013/9b Neco Counter example
 3  1    2 4  1
  Show that the matrix A has no multiplicative inverse, where
Determinant is given as 7 ; Thus  2 4 8
 2 3 1   1 A=  
 x     0 10  10 
  = 1   6 2 10 
 
y  1 5 3    1 
z
7   
    2 4  1  1  Solution
We are to show that | A | = 0
 2  1  3  (1)  1  1 
  = +2 10  10  (4) 0  10  8 0 10
= 1  1  1  5  (1)  3  1 
7 2 10 6 10 6 2

  2  1  4  (1)  1  1 = +2(100 + 20) +4(0 + 60) +8(0 - 60) = 0
306
Crammer’s rule 26  4 8
Since it is the understanding and applying 1 3 5
crammer’s rule that matters here.
The Author illustrates it as follows. Let 11 1 3
X1 =
170
2x + 5y –3z = 1
6x – 4y +3z = 2 26  4 8
x + 9y +7z = 5 3 5 1 5 1 3
1 3  5  26  (4) 8
be a given simultaneous linear equations 1 3 11 3 11 1
11 1 3
with three unknowns.
Next, we arrange them in matrix form = 26[(9) – (–5)] + 4[(3) – (–55)] + 8[(1) – (33)]
= 364 + 232 – 256
 2 5  3  x   1  = 340
      Thus, X1 = 340 = 2
 6  4 3   y  =  2 170
1 9 7   z   5 
 3 26 8
Then find det of problem matrix 6 1 5
 2 11 3
X2 =
2 5 3 170
i.e det = 6  4 3 326 8
1 5 6 5 6 1
1 9 7 6 1 5  3  26 8
11 3 2 3  2 11
Next we solve for X, Y and Z by replacing each  2 11 3
of the empty columns below by answer column = 3[(3) – (–55)] – 26[(18) – (10)] + 8[(66) – (–2)]
1 = 3(58) – 26(8) + 8(68)
= 174 – 208 + 544 i.e 510
matrix : 2 and solve 510
Thus, X2 = = 3
5 170
3  4 26
5 3 2 3 2 5
6 3 1
4 3 6 3 6 4
2 1 11
9 7 1 7 1 9 X3 =
X= Y= Z= 170
det det det
3  4 26
2000/9b Neco 3 1 6 1 6 3
6 3 1 3  (4)  26
Solve the following simultaneous equations by using 1 11  2 11 2 1
Cramer’s rule  2 1 11
3x1  4x 2  8x3  26 = 3[(33) – (1)] + 4[(66) – (–2)] + 26[(6) – (–6)]
(i)
= 3(32) + 4(68) + 26(12)
(ii) 6x1  3x 2  5x3  1 = 96 + 272 + 312 i.e 680
Thus, X3 = 680 = 4
(iii) - 2 x1 + x 2 + 3 x 3 = 11 170

Solution Confirm your answer by substituting for x1, x2 and x3


StepI: Bring out the numbers to matrix format 3(2) – 4(3) + 8(4) = 26
 3 4 8  x 6(2) + 3(3) – 5(4) = 1
   1  26 
– 2(2) + (3) + 3(4) = 11
 6 3  5  . x  =  1 
2
 2 1 3   x3   11 
   1992/12 (Nov)
A  X = b 1 1 2
StepII: find A Given that 1 0 1 = 7, find the value of x
3 5 6 5 6 3 2 2 1
3  (4) 8
1 3 2 3 2 1 That satisfies x + y – 2z = 1, x + z = –1 and 2x – 2y + z = 1
= 3[(9) – (–5)] + 4[(18) – (10)] + 8[(6) – (– 6)] A -7 B -1 C0 D1 E7
= 3(14) + 4(8) + 8(12) Solution
= 3(14) + 4(8) + 8(12) We will use Crammer’s rule to find x.
= 42 + 32 + 96 From the question, determinant is given as 7
Rearranging the given items in equation form
 A = 170
307
x + y – 2z = 1 2 27 5
x + 0 + z = –1 3  3 1
2x – 2y + z = 1
4 24 2
Here x = x1, y = x2 and z = x3 Y=
1 1 93
and we replace column 1 by  1 in the given matrix 2 27 5
3 1 3 1 3 3
3 3 1  2  27 5
2 1 4 24 2
24 2 4 2 4 24
1 1 2 = 2 6   24  276   4  572   12
1 0 1 = 2(18) – 27(10) + 5(84)
1 2 1 = 36 – 270 + 420
X= = 186
7
Thus, Y = 186 i.e 2
1 1 2 93
0 1 1 1 1 0
1 0 1 1 1 2 2  1 27
2 1 1 1 1 2
1 2 1 3 2 3
= 1[(0) – (–2)] –1[(–1) – (1)] –2[(2) – (0)] 4 3 24
= 1(2) – 1(–2) – 2(2)
Z =
93
= 2 + 2 – 4 i.e 0
2 1 27
0 2 3 3 3 3 2
 X  = 0 i.e. (C) 3 2 3  2 1  27
7 3 24 4 24 4 3
4 3 24
2002/7 Neco
Using Crammer’s rule, solve the following = 2 48   9  172   12  27 9  8
simultaneous linear equations: = 2(– 39) + 1(84) + 459
2x – y + 5z = 27 = 465
3x – 2y – z = –3 465
Thus, Z = i.e 5
4x + 3y + 2z = 24 93
Solution Cross checking answer Substituting for x, y and z values
Step I: Presenting the problem in matrix format 2x - y + 5 z = 27 2(2) - 1(2) + 5( 5) =
 2 1 5   X   27  3x - 2y - z = -3 3(2) - 2(2) + (- 5) =
     
 3  2  1 .  Y     3 4x- 3y + 2z = 24 4(2) + 3(2) + 2(5) =
4 3 2   Z   24  Exercise 25.36
  
Using the inverse matrix method, solve
Step II: Find A 2x + 3y – z = 1
 2 1 3 1 3 2 3x + 5y + 2z = 8
= 2 1 5 x – 2y –3z = 1
3 2 4 2 4 3
Exercise 25.37
= 2[(– 4) – (–3)] –1[ (6) – (– 4) ] + 5[ (9) – (–8) ]
Using crammer’s rule solve
= – 2 + 10 + 85
x + 2y +3z = 1
= 93
3x + 4y + 5z = 2
Step III: x + 3y +4z = 3
The Crammer’s rule proper for X, Y and Z.
1998/10 ( Nov) Exercise 25.38
27  1 5
1 1 1
 3  2 1 Evaluate 4 2 1
24 3 2
X= 9 3 1
93 Hence or otherwise solve the equations
27  1 5 p+q+r = 4
2 1  3 1 3 2
3 2  1  27  (1) 5 4p + 2q + r = 12
3 2 24 2 24 3
24 3 2 9p – 3q + r = 32
= 27[(– 4) – (–3)] +1[(–6) – (– 24)] + 5[(–9) – (– 48) ] 2013/9a Neco Exercise 25.39
= 27(–1) + 1(18) + 5(39) Using Crammer’s rule, solve the simultaneous linear
= –27 + 18 + 195 equations:
= 186 x + 2y – 6z = 4
186 –3x – 6y + 9z = –3
Thus, X = i.e 2 y – 3z = 1
93
308
Chapter twenty six To differentiate by first principle means to work strictly by
Differentiation by first principle the ordinary definition or the derived definition and not
employing any theorem or result in calculus that will be
y Q ( x + x, y + y ) given later. Our working here shall be based on the ordinary
definition.
y IPLF 1 Examples
,y) Find the derivative of the following with respect to x using
P (x
x L the first principle, 1
y+ y (i)x ( ii ) x2 ( iii ) 2x2 + x + 2 ( iv ) 2x + 1
(v) x–1 ( vi ) xn (n is a positive integer)
Solutions
(i) let y = x
Then y + y = x + x
subtracting
O x
A B y + y – y = x +  x – x
x+ x
y = x
Consider the graph of a function y = f(x) with points y = x = 1
P and Q very close to each other on the curve, where x x
P(x, y) and Q (x + x, y + y). y being a small dy = Lim y = Lim 1 = 1
increase in y due to a small increase x in x . dx x 0 x x 0
Here, y and x are enlarged in the diagram, otherwise,
they could not be seen from the diagram, (ii) Let y = x2
QL = y and PL = x Then y + y = (x + x)2
QL y Subtracting
The slope of the chord PQ is =
PL x y + y – y = (x + x)2 – x2
As x 0, Q moves into coincidence with P and the y = x2 + 2xx + (x)2 – x2
chord PQ produced becomes a tangent at P. y = 2xx + (x)2
Hence the slope of the tangent at P is the value that y Then y = 2xx + (x)2
x
x x x
approaches as x approaches zero.
Thus , the slope of the tangent at P is y = 2x + x
x
Lim y dy
x 0 x = Lim y = Lim ( 2x + x )
This value is denoted by the symbol dx x 0 x x 0
dy
(dee y , dee x) = 2x
dx 2
It is known as the first differential coefficient of y with (iii) Let y = 2x + x + 2 , then
respect to x.. It represents the rate of change of y with y + x = 2(x + x)2 + (x + x) + 2
respect to x. If y increase as x also increases then dy /dx y + x = 2[x2 + 2xx + (x)2] + (x + x) + 2
is also positive whereas if y decrease as x increases Subtracting
then dy /dx is negative. y + x – y = 2x2 + 4xx + 2(x)2 + x + x +2 – 2x2 – x – 2
y = 4xx + 2(x)2 + x
Derived definition form of first principle Then y = 4xx + 2(x)2 + x
x x x x
Consider the function
y = f(x) ………..(1) y = 4x + 2x + 1
Deduction from the diagram above , x
Let y be a small increase in y due to a small increase x in x. dy = Lim y = 4x + 1
y + y = f(x + x)……..(2) dx x 0 x
Subtracting (2) – (1)
y + y – y = f(x + x) – f(x) (iv) Let y = 1 then
y = f(x + x) – f(x) 2x + 1
Then, y = f(x + x) – f(x) y + y = 1
x x 2(x + x) + 1
Take limits as x 0 Subtracting
y + y – y = 1 – 1
dy
= Lim y = Lim f(x + x) – f(x) 2(x + x) + 1 2x + 1
dx
x 0 x x 0 x y = 2x + 1 – 2x – 2x – 1
is known as the derived definition of dy
/dx [2(x + x) + 1] [2x + 1]
309
y = –2x 2003/10 (Nov)
[2(x + x) + 1] [2x + 1] Find from the first principles, the derivatives, with respect to
1 x, of y = 3x2
Next we multiply both sides by Solution
x
y = 3x2
y = –2
y + y = 3(x + x)2
x [2(x + x) + 1] [2x + 1]
y + y = 3[ x2 + 2xx + (x)2 ]
dy = Lim y = –2 y + y = 3x2 + 6xx + 3(x)2
dx x 0 x [2(x + 0 ) + 1] [2x + 1] Subtracting
–2 y + y – y = 3x2 + 6xx + 3(x)2 – 3x2
= (2x + 1)(2x + 1) y = 6xx + 3(x)2
–2 Then y = 6xx + 3(x)2
= (2x + 1)2 x x x
y = 6x + 3x
( v ) Let y = x – 1 then
x
y + y = (x + x) – 1
dy = Lim y = 6x
Subtracting
dx x 0 x
y + y – y = (x +x) – 1 – x–1

y = [ (x + x) -1 – x - 1] [ ( x + x) -1 + x–1 ]


2004/11a Neco (Nov)
Differentiate 1 from the first principle
( x + x) – 1 + x–1 x3
Nothing happened, we just multiplied by 1/1 Solution
The top is a surd × its conjugate; Let y = 1
y = ( x + x – 1) – (x – 1) x3
x + x – 1 + x–1 1
y + y = (x + x)3
y = x
= 1
x + x – 1 + x–1 x3 + 3x2x + 3x(x)2 + (x)3
y = x Subtracting
1 1
x ( x + x – 1 + x – 1)x
y + y – y = x3 + 3x2 x + 3x(x)2 + (x)3 – x3
y = 1
y = x3 – x3 – 3x2x – 3x(x)2 – (x)3
x + x – 1 + x–1 [x3 – 3x2x + 3x(x)2 +(x)3 ] [x3]
dy = Lim y = 1 y = – 3x2x – 3x(x)2 – (x)3
dx x 0 x x –1 + x–1 [x3 – 3x2x + 3x(x)2 +(x)3 ] [x3]
= 1 y = – 3x2 – 3x(x)– (x)2
x [x – 3x2x + 3x(x)2 +(x)3 ] [x3]
3
2( x –1 )
dy = Lim y = – 3x2
( vi ) Let y = xn Then
dx x 0 x (x3)(x3)
y + y = (x + x)n
= –3
= xn + nxn–1 (x)1 + n(n–1)xn–2 (x)2 + … + (x)n
x4
2!
1992 /2 (Nov)
Subtracting
Differentiate 5x – 1 from the first principles.
y + y – y = xn + nxn – 1 (x)1 + n(n–1)xn–2 (x)2 + ….+ (x)n – xn
2! x2
y = nxn–1(x)1 + n(n –1)xn–2 (x)2 + … Solution
2! Let y = 5x – 1
y = nxn–1 + n(n – 1)xn–2x +...+ terms in higher power of x. x2
2! y + y = 5(x + x) – 1
(x + x)2
dy = Lim y = nxn – 1
y + y = 5x + 5x – 1
dx x 0 x
x + 2xx +(x)2
2
Indeed, for any constant n(positive; negative or Subtracting
fractional), if
y + y – y = 5x + 5x – 1 – 5x + 1
dy y = xn 2
x +2xx+(x) 2
x2
Then dx = nxn – 1
General formula: x here is the independent variable but any y = 5x – 1 + 1
other variable could be used. x2 + 2xx + (x)2 x2
310
RHS is purely fractional in nature and must be General formula method
regularized through our basic knowledge in fractional For any function y = axn ( where a and n are constants )
LCM. dy
= 5x[x2 + 2xx + (x)2 ](x 2) – x2 + x2 + 2xx + (x)2 = n×a x n – 1
dx
[x2 + 2xx + (x)2 ][ x 2 ] i.e differentiating y with respect to x pronounced dee y-dee x
= [5x2x + 10x(x)2 +5(x)3](x 2) + 2xx + (x)2 Examples: Differentiate the following with respect to (wrt)
[x2 + 2xx + (x)2 ][x 2] their variable
y = 5x4x + 10x3(x)2 +5x2(x)3 + 2xx + (x)2 ( i ) y = x2 ( ii ) y = 3x5 ( iii ) y = 14x2
6 4
[x2 + 2xx + (x)2 ][ x 2 ] ( iv ) y = - 12 x ( v ) s = 3t ( vi ) y = 3x
( vii ) y = x ( viii ) y = 4x3
y = 5x4 + 10x3x + 5x2(x)2 + 2x + x Solution
x [x2 + 2xx + (x)2] [x2] ( i ) y = x2
dy = Lim y = 5x4 + 2x dy
= 2x2 – 1
dx x 0 x [x2] [x2] dx
= 5x4 + 2x = 2x1 i.e 2x
x4 ( ii ) y = 3x5
= 5 + 2 dy
x3 = 5×3×x5 – 1
dx
= 15 x4
1994/10a (Nov) Exercise 26.0
Find from the first principles, the derivative, with ( iii ) y = 14 x2
respect to x, of 1 dy
3x2 + 5 = 2×14× x2 – 1
dx
= 28x1 i.e 28x
Basic results (rules) in differentiation
achieved by first principle ( iv ) y = –12 x6
dy
= 6×( – 12 )× x 6 – 1
1. If C is a constant then dx
d
(c) = 0 = – 72 x5
dx
( v ) s = 3t4
ds
2. If a is a constant then = 4 × 3 × t4 – 1 and not dy/dx or dy/dt or dt/ds
d d dt
a f(x) = a f(x) = 12t3
dx dx
( vi ) y = 3x
3. If u, v, w,… are functions of x the sum Here the power of x is one i.e x1
d
( u + v + w +,… ) =
du
+
dv
+
dw
+…  y = 3x1
dx dx dx dx dy
= 1×3 ×x1 – 1
dx
4. Product Rule: If u and v are some functions of x, then = 3x0 since x0 = 1
d du dv = 3×1 i.e 3
( u v) = V +U
dx dx dx
( vii ) y = x
5.Quotient Rule:If u and v are some functions of x, then Here the power of x is one i.e x1
du dv  y = x1
d u V U dy
  = dx dx = 1x1-1
dx  v  V2 dx
= 1x0 since x0 = 1
6. Chain Rule: If y is a function of z and z is a function = 1×1 i.e 1
of x, then
( viii ) y = 4x3
dy dy dz
= × dy
dx dz dx = 12x2
dx
dx 1 Examples i to vii involves explanation with solution. But viii
7. If y is a function of x then, =
dy dy have solution only which is the acceptable standard we shall
dx be following except when the author wants to explain
further.
311
Further examples 3  52 3
Differentiate the following with respect to their =  x i.e  5 A
variables. 2 2x 2
( 1 ) y = 1 ( 2 ) t = 3 ( 3 ) if y = 10 find dy/dx
x2 s6 2001/38 PCE Exercise 26.1
 32
( 4 ) if y = 13x, find dy/dt If y  x then dy/dx is
Solution A. – /3x
2 ½
B. – 3/2x – ½ C.- 2/3x – 5/2 D. – 3/2x –5/2
(1) y= 1
x2
The Derivative of the sum of two ( or more ) simple
 y = x–2
functions is the sum of the separate derivatives of the
dy
= - 2x – 2 – 1 functions
dx What it implies is shown below
= - 2x – 3 =-2
x3 Eg.1 If y = 4x3 – 12x2 + 3x + 12, find dy/dx.
Solution
(2) t= 3  t = 3s – 6 dy
s6 = 12x2 – 24x + 3
dx
dt
= - 6×3×s – 6 – 1 Eg. 2 If y = 5 – 2 + 14, find dy/dx
ds
= - 18 s – 7 x2 x
= - 18 Solution
s7 y = 5x – 2 – 2x – 1 + 14
dy  10 2
( 3 ) y = 10  y = 10x0  3
 2
( the variable x was not shown because it has a power of zero ) dx x x
dy Rearranging ,
= 0 ×10 x 0 – 1
dx = 2 – 10
= 0 ( since zero times any number is zero ) x2 x3
2002/19 UME
Generally any constant differential is zero
If y = x2 – 1 , find dy/dx
( 4 ) y = 13x, but we are asked to find dy/dt x
dy A.2x + 1 B. 2x + x2 C. 2x - 1 D. 2x - x2
= 0 since 13x is a constant with respect to t x2
x2
dt Solution
2007/38 PCE y = x2 – 1  y = x2 – x – 1
x dy = 2x – ( - 1 ) x – 1 – 1
If y  x , find dy dx
dx
= 2x + x – 2
1 2 x 1 = 2x + 1 ( A )
A B C D x2
2 x x 2 x
2005/6 PCE
Solution
Find the derivative of f(x) = 2x(x2 + 3)
yx
1
y  x  2
A. 4x2 + 3 B. 2(x2 + 3) C. 6(x2 + 1) D. 6(x2 + 2)
Solution
dy 1 We can easily open up the bracket as :
 1 x 2 1
 1 x  2 i.e
1 1
B
dx 2 2 2 x f(x) = 2x(x2 + 3) becomes
1997/41 PCE f(x) = 2x3 + 6x
1 f /( x) = 6x2 + 6 i.e 6(x2 + 1) C
Find the derivative of
x x
2007/15 UME Exercise 26.2
A. – 3 B. 3 C. 3 D. – 3
dy
2x5/2 2x5/2 2x – 5/2
2x – 5/2
If y = ( 1 + x )2 , find dx
Solution A. 2x – 1 B. x – 1 C. 2 + 2x D. 1 + 2x
Applying general formula method, first, we express x in its
powers properly. i.e
1994/44 PCE Exercise 26.3
1 1 1 1 Differentiate y = 3x2 + 2x + 3 with respect to x.
is same as 1 i.e 1 = A.6x + 2 B.6x2 – 2 C.6x + 3 D.6x2 - 3
x x2
1
1 2 3
x x x x2

3
2 1995/39 PCE Exercise 26.4
Let take it as y = x Differentiate 3x2 + 2/x with respect to x
dy 3 3
A. 6x + 2 B. 6x – 2 C. 6x + 2 D. 6x – 2
=  x  2 1
dx 2 x x2 x2
312
Second derivative Function of a function ( chain rule )
Differentiation can be repeated as many times as If we have a case like:
required on a given function. Under this subheading, we Given that y = ( x + 1 )2 find dy/dx
shall only discuss For us to apply the formula
2nd differentiation. We must first expand i.e
Eg. 1 If y = 3x3 – 2x2 + 4 find d2y/dx2 y = ( x + 1 )2 will become y = ( x + 1)( x + 1)
Solution = x2 + 2x + 1
y = 3x3 – 2x2 + 4 is differentiated wrt x, i.e 2
Then we differentiate y = x + 2x + 1
dy dy
= 9x2 – 4x = 2x + 2
dx dx
d2y/dx2  differentiating the result, i.e we can afford to expand it. But if we have
d 2 y = 18x – 4 y = ( x + 1 )10 find dy/dx
dx 2 I bet you – you will exhaust the whole of your note to expand it.
2 2
d y/dx is read as ( dee squared y dee x squared) The chain rule comes into play here.
Starting
Note: d2y/dx2 is different from ( dy/dx)2 ( 1 ) Given that y = ( x + 1 )2, find dy/dx
as the first is 2nd derivative while the latter is the Solution
square Let z = x + 1 – the core or bracket content
of the result of dy/dx then our original function will be
Sometimes a function can be written as y = z2
f( x ) = 2x2 + 3x + 1
instead of y = 2x2 + 3x + 1 Then, dy  dy  dz
dx dz dx
then we have that its own form of dy/dx will be f /( x). Please note that z is a dummy variable
It can be changed to u or v etc depending on the author or
Hence f /( x) = 4x + 3 teacher
Also d2y/dx2 will be f // ( x ) Lets go gradually,
dz
f // ( x ) z = x + 1 then = 1
Hence = 4. dx
Special derivatives’ result dy
Also, y = z2, then = 2z
Functions derivatives’result dz
lnx or logex 1
/x Substituting,
e x
e x dy dy dz
  will become
Sin x cos x dx dz dx
= 2z × 1
Cos x - sin x = 2z
Tan x sec2 x Substituting for z value
= 2 (x + 1 )
Cot x - cosec2 x
if we expand.
Sec x sec x tan x = 2x + 2 as gotten earlier.
Cosec x - cosec x cot x 2. If y = ( x + 1 )10, find dy/dx
Solution
1995/38 UME dz
The derivative of cosec x is Let z = x + 1 then =1
dx
A tan x cosec x B. – cot x cosec x
dy
C. tan x secx D. – cot x sec x y = ( x + 1 )10 will be y = z10, and = 10z9
Solution dz
Derivative of cosec x = – cot x cosec x ( B ) Thus,
dy dy dz
 
dx dz dx
So – the above derivatives should be part and parcel of us
= 10z9 × 1
= 10z9 substituting for z value
= 10( x + 1 )9 solved.
Differentiation rules 3. if y = ( 4x – 3 )½ , find dy/dx
We have been discussing the general formula so far, Solution
there are certain rules to follow when we have We cannot expand, thus the use of chain rule
‘complex ‘ or complicated functions to differentiate. dz
Let z = 4x – 3, then =4
dx
dy
y = ( 4x – 3 )½ will be y = z½ and = ½ Z– ½
dz
313
Since ½ – 1 = – ½ 2002/21
dy dy dz Differentiate y = (3x2 + 5)4 with respect to x.
Thus,  
dx dz dx A. 12(3x2 + 5)3 B. 24(3x2 + 5)3 C. 12x(3x2 + 5)3
2 3
D. 24x(3x + 5)
= ½ Z– ½ × 4
Solution
Substituting for z value
y = (3x2 + 5)4
= 4 × ½ ( 4x - 3 ) – ½ Let u = 3x2 + 5, then y = u4
= 2 du
/dx = 6x dy
/du = 4u3
4x – 3 dy = dy × du
dx du dx
4. If y = sin 2x find dy/dx = 4u3 × 6x
Solution = 4(3x2 + 5)3 × 6x
We know that derivative of sin x is cos x. but this is sin 2x = 24x(3x2 + 5)3 D.
dz
Let z = 2x; then =2
dx 2000/38
dy Given that y = (2x3 + 2x2 + 1)4, find dy/dx
y = sin 2x will be y = sin z and = cos z
dz A. 4(2x3 + 2x2 + 1)3 (2x2 + 4x) B. 4(2x3 + 2x2 + 1)(6x2 + 4x)3
dy dy dz C. 4(6x2 + 4x)(2x3 + 2x2 + 1)3 D. 4(6x2 + 4x)(2x3 + 2x2)3
Thus,   Solution
dx dz dx y = (2x3 + 2x2 + 1)4
= cos z × 2 i.e 2 × cos z
Let u = (2x3 + 2x2 + 1), then y = u4
= 2 cos 2x du
/dx = 6x2 + 4x dy
/du = 4u3
dy = dy × du
The pattern follows
dx du dx
If y = sin 3x
= 4u3 × (6x2 + 4x)
dx = 3 cos 3x
dy
= 4(2x3 + 2x2 + 1)3 (6x2 + 4x)
and If y = sin 4x i.e 4(6x2 + 4x) (2x3 + 2x2 + 1)3 (C)
dx = 4 cos 4x and so on
dy

Example 5 2004/26 (Nov) Neco


If y = sin2 x , find
dz Find d2y for the function y = (x + 3)
dx dx2
Solution A. – 1 B. x + 3 C. x–3
y = sin2x  y = ( sin x)2 4 (x +3) 3
2 2
1
dz
Let z = sin x , then = cos x D –1/4 (x + 3)3 E 4 (x + 3)3
dx
dy Solution
y = sin2 x will be y = z2, and = 2z y = (x + 3)1/2
dz
dy dy dz Let u = x + 3, Then y = u1/2
Thus,   du
/dx = 1 , /du = 1/2u–1/ 2
dy
dx dz dx
= 2z × cos x dy = dy × du
= 2 sin x cos x dx du dx
= sin 2x = ½ u–1/2 × 1
From the trig. identity sin 2A = 2sinAcosA = ½ (x + 3) – 1/2
Example 6 d2y = d2y × du2
d 2x  3 dx2 du2 dx2
Evaluate (e )
dx = 1  1 ( x  3)  1
3
2

Solution 2  2 
d 2x  3
(e ) could be taken as y = e 2 x  3 , find dy/dx = – 1 A.
dx 4( x + 3 ) 3
dz
Let z = 2x + 3 then =2
dx 2003/43 Neco
dy
y = e 2 x  3 will be y = e z and = ez Evaluate
d
cos (3x2 – 2x)
dz dx
dy dy dz A. – sin(6x – 2) B. –sin(3x2 – 2x)
Thus,  
dx dz dx C. –(6x – 2) sin(3x2 – 2x) D. (6x – 2) sin (3x2 – 2x)
= ez × 2 E. (6x – 2) cos (3x2 – 2x)
= 2 e2x  3
314
Solution dy = dy × du
Let y = cos (3x2 – 2x) dx du dx
and u = 3x2 – 2x, y = Cos u = 3u2 × 4 cos(4x + 10)
du
/dx = 6x – 2 dy
/du = –sin u = 12(sin4x + 10)2 cos(4x + 10)
dy = dy × du i.e = 12sin2(4x + 10) cos(4x + 10) (B).
dx du dx
= –sin u × (6x – 2) 1994/20
= – (6x – 2) sin(3x2 – 2x) C. Find dy/dx if y = loge tan 2x
A. 2 sec22x B. 2 C. 2 sec2x D. 2 sec2(2x)
2003/6a Neco (Dec) tan 2x tan 2x tan 2x
Differentiate the function with respect to x E . 1
y = 4 tan 2x
(2x3 – 6x2 + 2x + 1)4 Solution
Solution dy 1
This is a chain rule problem and not quotient rule as it Let u = tan 2x Then y = loge u and =
du u
may seems to suggest. It is properly presented as: du dz
y = 4(2x3 – 6x2 + 2x + 1)– 4 du =  here z = 2x and u = tan z
Let u = (2x3 – 6x2 + 2x + 1) then, y = 4u– 4 dx dz dx
Then du/dx = 6x2 – 12x + 2 and dy/du = – 16u– 5 = Sec2 z × 2
dy = dy × du = 2 Sec2 2x
dx du dx dy = dy × du
= – 16u– 5 × (6x2 – 12x + 2) dx du dx
= –16(6x2 – 12x + 2) (2x3 – 6x2 + 2x + 1) – 5 1
= × 2 sec2 2x
= –16(6x2 – 12x + 2) u
(2x3 – 6x2 + 2x + 1)5 = 2Sec2 (2x) D.
tan 2x
2002/9a Neco
If y = (x3 + 1)6. Find d2y 1992/17
dx2 If y = log e ( 1x ) , find dy/dx
Solution
y = (x3 + 1)6 1
A. –1/x B. –x C. 1/x D. x E. – log
Let u = (x3 + 1); then y = u6 x2
du
/dx = 3x2 ; dy/du = 6u5 Solution
dy = dy × du 1 1 dy 1
Let u = , du = - 2 then y = loge u and =
dx du dx x dx x du u
= 6u5 × 3x2 dy = dy × du
= 18x2(x3 + 1)5 dx du dx
Second differential will be product rule and chain rule
1 1
d 2 y = udv + vdu = × - 2
u x
dx 2 = 1 × – 1
= 18 [ x2 d(x3 + 1)5 + (x3 + 1)5 d(x2) ] 1
/x x2
we apply chain rule to d(x3 + 1)5
= x × – 1
=18 [x2 × (3x2) × 5(x3 + 1)4 + (x3 + 1)5 × 2x]
x2
= 18 [15x4(x3 + 1)4 + 2x(x3 + 1)5 ]
= – /x A.
1
= 18x(x3 + 1) [15x3(x3 + 1)3 + 2 ]
1996/19
2002/34 Neco Given that V = 4r3 and A = 4r2, find dA
If y = sin3(4x + 10), what is dy/dx? dV
A. 12sin2(4x + 10) cos2(4x + 10) A. 32r2 r3 B. 2r C. 2 D. 1 E. – 2
B. 12sin2(4x + 10) cos(4x + 10) r 2r r2
C. 12sin2(4x + 10) D. 3sin2(4x + 10) cos(4x + 10) Solution
E. 3 sin2(4x + 10) Applying chain rule
Solution dA = dA × dr but dr = 1
It implies: y = [sin (4x + 10)]3 dv dr dv dv dv
Let u = sin (4x + 10) Then, y = u3 and dy/du = 3u2 dr
du dz = 8r × 1
du =  here z = 4x+10 and u = sin z
dx dz dx 4r2
= Cos z × 4 = 2
= 4cos(4x +10) r (C)
315
1997/39 UME dy
y = 3 cos ( x/3 ) will be y = 3 cos z , and = - 3sin z
d
cos (3x2 – 2x) is equal to dz
dx dy dy dz
Therefore ,  
A. – sin (6x –2) B. – sin (3x2 –2x) dx dz dx
C. (6x –2) sin (3x – 2x)
2
D. – (6x – 2) sin (3x2 – 2x) = - 3 sin z × 1/3
Solution = - 3 × 1/3 sin z
Let y = cos ( 3x2 – 2x ) then we find dy/dx = - sin ( x/3 )
dz When x = 3/2 i.e 2700
take z = 3x2 – 2x; then = 6x – 2
dx
dy - sin x/3 = - sin 3/2  - sin 270 = - sin 90
Then y = cos z; and = - sin z =-1 (D)
dz
dy dy dz
Thus   2004/21 Neco Exercise 26.5
dx dz dx Find d2y of the function y = 3 x5 with respect to x
= - sin z × (6x – 2) dx2
= - (6x – 2) sin (3x2 – 2x) D. A. 45 x B. 45 C. 45 D. 3 x5 E.15 x3
4 4 x 8 x 12
2005/1 UME
If y = ( 1 – 2x )3, find the value of dy/dx at x = - 1
A.- 6 B.57 C. – 54 D.27 2010/12 Exercise 26.6
Solution x )2, find dy
If y = 2(2x +
Applying chain rule dx
dz x )  2 1 
Let z = 1 – 2x then =-2 A2 x (2x + x ) B 4(2x +  
dx 
 2 x 
dy
y = ( 1 – 2x )3 will be y = z3, and = 3z2 C 4(2x + x )(2 + x) D 8(2x + x )(2 + x )
dz
dy dy dz
Therefore,   2009/26 (Nov) Exercise 26.7
 5 4 x  with respect to x.
dx dz dx
= 3z2 × (-2) i.e (-2)× 3z2 Differentiate
= - 6 ( 1– 2x )2 4
A - 2 B -
At x = – 1
- 6 ( 1 – 2x )2 = - 6 [ 1 – 2 (-1) ]2  5  4x  5  5  4x 
= - 6 ( 1 + 2 )2 4
C 1 D
= - 6 ( 3 )2
= - 6 × 9 i.e - 54 ( C )  5  4x   5  4x 
2002/36 PCE
Differentiate ( 1 + 2x2 )4 2010/17 Neco Exercise 26.8
A.16x ( 1 + 2x2 )4 B.16x ( 1 + 2x2 )3 
2 3
Differentiate cos( – 3x ) with respect to x
C.8x ( 1 + 2x ) D. 8x ( 1 + 2x ) 3 2
Solution   
y = ( 1 + 2x2 )4 A 3cos( – 3x ) B 3sin( – 3x ) C –3cos( – 3x )
2 2 2
Applying chain rule
 
dz D – sin( – 3x ) E – 3sin( – 3x )
Let z = 1 + 2x2 ; then = 4x 2 2
dx
dy
y = ( 1 + 2x2 )4 will be y = z4 and = 4z3 2009/4 Neco Exercise 26.9
dz 2
Differentiate (3x  1) 3
dy dy dz
Thus   1 3 3
dx dz dx
A. B. C.
= 4z3 × 4x 3x  1 3
3x  1 3x  1
= 16x ( 1 + 2x2 )3 ( B )
2004/12 UME
2 3
D. E.
If y = 3 cos ( x/3 ), find dy/dx when x = 3/2. 3
3x  1 3x  1
A.1 B.– 3 C.2 D.– 1
Solution 2005/6 UME Exercise 26.10
3 is a constant , so it is the bracketed term that makes Find the derivative of y = sin ( 2x3 + 3x – 4 )
the problem chain. A. – cos ( 2x3 + 3x – 4 ) B. – ( 6x2 + 3 ) cos ( 2x3 + 3x – 4 )
x C. cos ( 2x3 + 3x – 4 ) D. ( 6x2 + 3 ) cos ( 2x3 + 3x – 4 )
dz 1
Let z = 3 then = /3
dx
316
1998/38 UME Exercise 26.11 =  cos 900 + sin 900
If y = 243 ( 4x + 5 ) – 2, find dy/dx when x = 1 2
A. – 8/3 B.3/8 C.9/8 D. – 8/9 =  ×0+1
Product Rule 2
If y = UV where u and v are some functions of x. then = 1 ( C)
dy dv du 1992/37 UME
U V if y = x sin x, find d2y/dx2
dx dx dx
dy A. 2cos x – x sin x B. sin x + x cos x
Simply as  Udv  Vdu C. sin x – x cos x D. x sin x – 2 cos x
dx
Solution
Example 1
The function is in form uv, we find dy/dx first before d2y/dx2
If y = (3x6 – 10)(x4 + 30) , find dy/dx
y = x sin x
Solution
u v
Let y = uv
i.e y = (3x6 – 10)(x4 + 30) dy
 Udv  Vdu
dy dx
Thus,  Udv  Vdu = x cos x + sin x ×1
dx
= x cos x + sin x
= ( 3x6 – 10 ) × 4x3 + ( x4 + 30 ) × 18x5
Then, d y/dx2 will be differentiating the result. Where x cos
2
= 12x9 – 40x3 + 18x9 + 540x5
x is another product rule case.
= 30x9 + 540x5 – 40x3
2000/6 Neco 2
Find the derivative of y = (x + 1)(x + 2) Thus, d y  d ( x cos x)  d (sin x)
2
A. 2x – 3 B. 2x + 3 C. 2x + 4 D. 3x – 2 E. 3x + 2 dx dx dx
Solution Let y = x cos x
y = (x + 1) (x + 2) u v
Then y = u v d/dx ( x cos x ) dy/dx = udv + vdu
dy
/dx = udv + vdu = x ×( - sin x ) + cos x × 1
= (x + 1) × 1 + (x + 2) × 1 = - x sin x + cos x
=x+1+x+2
= 2x + 3 (B)
d2y
Thus, = - x sin x + cos x + cos x
2005/37 Neco dx 2
If y = x sin 2x, find dy/dx = - x sin x + 2 cos x
A. –2cos2x + sin2x B. –2x sin cosx + 2 sin2x = 2 cos x – x sin x ( A )
C. – 2x cos2x + sin2x D. sin2x – xcos2x
E. sin2x + 2x cos2x 2004/37 PCE
Solution If y = (x2 + 2x + 4)(x – 2), find dy/dx
y = x sin 2x A. – 3x2 B.3x2 C.3x4 D.2x4
dy
/dx = Udv + Vdu Solution
= x d(sin 2x) + sin2x × d(x) Applying product rule
d y = ( x2 + 2x + 4 ) ( x – 2 )
Recall that applying chain rule (sin 2x) = 2cos2x u v
dx
dy
Thus, dy/dx = x × 2cos2x + sin 2x × 1  Udv  Vdu
= 2x cos2x + sin2x (E) dx
Addition is commutative i.e 2 + 3 = 3 + 2 so, = (x2 + 2x + 4 ) × 1 + (x – 2 ) ( 2x + 2 )
2xcos2x + sin2x is same as sin2x + 2xcos2x = ( x2 + 2x + 4 ) + ( 2x2 + 2x – 4x – 4 )
= x2 + 2x + 4 + 2x2 – 2x – 4
2001/38 UME Collect like terms together
if y = x sin x, find dy/dx when x =  = x2 + 2x2 + 2x – 2x + 4 – 4
2 = 3x2 ( B )
A. -  B. – 1 C. 1 D.  2004/15
2 2 Find the derivative of the function y = 2x2( 2x – 1 )
Solution at the point x = - 1
Let y = uv A.18 B.16 C.- 4 D.- 6
i.e y = x sin x Solution
dy Applying product rule
Th us,  Udv  Vdu
dx y = 2x2 ( 2x – 1 )
= x cos x + sin x × 1 u v
= x cos x + sin x dy
 Udv  Vdu
When x =  means 900 dx
2 = 2x2 × 2 + ( 2x – 1 ) × 4x
317
= 4x2 + 8x2 – 4x Solution
= 12x2 - 4x y = 3x2 ( x3 + 1 ) ½
At point x = - 1 u v
12x2 – 4x = 12 (-1)2 – 4 (-1) dy
= 12 + 4 i.e 16 ( B )  Udv  Vdu
dx
2007/12UME Exercise 26.12 = 3x2dv + ( x3 + 1 )½ × 6x
If y = x cos x, find dy/dx dv is a chain rule problem; lets unchain it .
A.sinx - xcosx B. sinx + xcosx C. cosx + xsinx D. cosx – xsinx
½
y = ( x3 + 1 )
2004/13 UME Exercise 26.13 z = x3 + 1 dz/dx = 3x2
Find the derivative of (2 + 3x )(1 – x) with respect to x. ½
y = ( x3 + 1 ) becomes y = z ½ and dy/dz = ½ z – ½
A.6 B.- 3 C.1 – 6x D.6x – 1 Our dv  dy/dx = dy/dz × dz/dx
2005/6 PCE Exercise 26.14 = ½ z – ½ × 3x2
Find the derivative of f( x ) = 2x( x2 + 3 ) -½
= 3/ 2 x2 ( x 3 + 1 )
A.4x2 + 3 B.2(x2 + 3) C.6(x2 + 1) D. 6x2 + 2 Substituting for dv
= 3x2 × 3/2 x2 ( x3 + 1 ) – ½ + ( x3 + 1 )½ × 6x
dy
Case involving product and chain rules. dx
1999/36 UME = 9x4 + 6x ( x3 + 1 ) ½
What is the derivative of t2 sin ( 3t – 5 ) with respect to 2( x3 + 1 ) ½
the variable t ? Applying Lcm pattern of ½ + 2
A. 6t cos ( 3t – 5 ) B. 2t sin ( 3t – 5 ) – 3t2 cos ( 3t – 5 ) = 9x4 + 12x (x3 + 1)
C. 2t sin ( 3t – 5 ) + 3t2 cos ( 3t – 5 )
D. 2t sin ( 3t – 5 ) + t2 cos 3t 2( x3 + 1 ) ½ (D)
Solution 2002/16
Let y = t2 sin( 3t – 5 ) we are to find dy/dt Find the derivative of y = sin2 ( 5x ) with respect to x.
Since t2 is there and sin carries another function of t, A. 5 sin 5x cos 5x B. 2 sin 5x cos 5x
their differential is that of Product Rule C. 15 sin 5x cos 5x D. 10 sin 5x cos 5x
y = t2 sin ( 3t – 5 ) Solution
u v y = sin2 ( 5x )  y = ( sin 5x )2
dy i.e y = sin5x sin5x
 Udv  Vdu u v
dt
= t2dv + sin ( 3t – 5 ) × 2t dy
 Udv  Vdu
dv is a function of function problem dx
= sin 5x d ( sin 5x ) + sin 5x d ( sin 5x )
let y = sin ( 3t – 5 ) Base on previous work
z = 3t – 5; dz/dt = 3 d( sin 5x ) = 5 cos 5x
y = sin ( 3t – 5 ) will become y = sin z and dy/dz = cos z dy
= sin 5x × 5cos5x + sin 5x × 5cos5x
Thus, our dv  dy/dt = dy/dz × dz/dt dx
= cos z × 3 = 5 sin 5x cos 5x + 5 sin 5x cos 5x
 dv = 3 cos ( 3t – 5 ) = 10 sin 5x cos 5x (D)
Substituting for dv 2001/35 UME Exercise 26.14b
dy
= t2 ×3 cos ( 3t – 5 ) + sin ( 3t – 5 ) × 2t Differentiate ( 2x + 5 )2 ( x – 4 ) with respect to x
dt A. 4( 2x + 5 ) ( x – 4 ) B. 4( 2x + 5 ) ( 4x – 3 )
= 3t2 cos ( 3t – 5 ) + 2t sin ( 3t – 5 ) ( C ) C. ( 2x + 5 ) ( 2x – 13 ) D. ( 2x + 5 ) ( 6x – 11 )
since addition is commutative 2 + 3 = 3 + 2 i.e
= 2t sin ( 3t – 5 ) + 3t2 cos ( 3t – 5 )

1999/39 UME
If y = 3x2 ( x3 + 1 ) ½ , find dy/dx
A. 6x ( x3 + 1 ) + 3x2 B 12x ( x3 + 1 ) + 3x2
2( x3 + 1 )1/2 2( x3 + 1 )1/2
C. – 15x4 + 6x D. 12x ( x3 + 1 ) + 9x4
6x2 ( x3 + 1 ) ½ 2( x3 + 1 ) ½

318
Quotient Rule = cosx + x sinx
u cos2 x
If y = , where u and v are some functions of x,
v = cosx + x sinx
then cos2x cos2x
dy vdu  udv = 1 + x sinx × 1

dx v2 cosx cosx cosx
It has to do with functions in fraction quoted as: Recall that: sinx is tanx and 1 is secx
Bottom dee top minus top dee bottom , all over bottom square cosx cosx
2005/11a Neco = secx + x tanx secx (D)
Differentiate with respect to x, the function It is same as secx + x secx tanx
y = 4x3 + 8
2x2 + 3 2005/11(ii) Exercise 26.15
Solution Differentiate with respect to x:
y = u(x) i.e 4x3 + 8 y = 2x – 3
v(x) 2x2 + 3 x2 + 5
dy = vdu – udv
dx v2 2003/11b (Nov) Exercise 26.16
= (2x + 3) × 12x2 – (4x3 + 8) 4x
2 Find the derivative with respect to x of y = 2x
36x2(2x2 + 3)2 x2 + 3
1996/12 (Nov) Exercise 26.17
= 24x4 + 36x2 – 16x4 – 32x
If y = 1 + x, find dy
(2x2 + 3)2 1–x dx
= 8x + 36x2 – 32x
4
A. 1 B. 2 C. 1 + x D. x
(2x2 + 3)2 (1 – x) 2
(1 – x) 2
(1 – x)2 (1 – x)2
2004/18 Neco E. –2
Differentiate the function 4x4 + x3 – 5 with respect to x. (1 – x)2
4x2
A. 2x + 5 + 1 B. x2 + x – 5 C. 2x2 + 5 + 1 2010/19 Neco Exercise 26.18
3
2x 4 4 4x 2x 4x Differentiate 4x3 + 3x2 – 5 with respect to x
5x2
D. 2x – 5 + 1 E. x2 – x – 5
2x3 2 4 4x A 2 + 4 B 2 + 4x C 12x3 + 6x
3 3
Solution x 5 x 5 10x
y = u(x) i.e 4x4 + x3 – 5 D 12x + 6x E 4x – 1 + 3
3

v(x) 4x2 25 x4 5 x2 5
Quotient and chain rule
dy = vdu – udv
dx v2
2003/5 Neco
= 4x (16x3 + 3x2) – (4x4 + x3 – 5) × 8x
2
Find the derivative of y with respect to x if y = 1+x
(4x2) 2 1–x
= 64x5 + 12x4 – 32x5 – 8x4 + 40x Solution
16x4 Applying chain rule
1
= 32x + 4x4 + 40x
5
y = 1 + x is same as y = (1 + x) 2
16x4 16x4 16x4 1–x (1 – x) 2
1

= 2x + 1 + 5 (A) 1
Let U = 1 + x then y = U 2
4 2x3 1–x
1998/39 UME dy = dy × du
Differentiate x with respect to x. dx du dx
cosx 1
= 1 u 2  du

A. 1 + x sec x tan x B. 1 + sec2 x 2 dx
C. cos x + x tan x D. sec x + x sec x tan x
du
Solution Next we find by applying Quotient rule
y = x i.e u dx
cos x v du = (1– x) × 1 – (1 + x) × ( –1)
dy vdu  udv dx (1– x)2
 = 1– x+1+x
dx v2 (1– x)2
= cos x × 1 – x (-sin x )
( cosx )2
319
= 2 2010/3 Neco Exercise 26.22
(1– x)2 dy x3
1 Find if y =
dy 1 1 x 
2
2 dx 2x 2  3
Thus, =   ×
dx 2 1 x  (1  x) 2
1 Application of Differentiation
1 1 x  2 2 Gradient ( slope at a given point).
=   × The concept of differentiation is the same as that of gradient
2 1 x  (1  x) 2 or slope. So when we say the gradient or slope of a function
1
at a point we are only referring to the differentiation of that
(1  x) 2
1 function and substituting for the x value given or among the
= 
(1  x) (1  x) 2 points given. Other concepts required in this subtopic will be
1
2

shown as we go
1
= 3 2004/15
(1  x) 2 (1  x) 2
1

The gradient of the curve y = 3x2 + 11x + 7 at the point


2000/28 Neco P(x, y) is – 1. find the co – ordinates of P.
Differentiate with respect to x, cos x A. ( -2, +3) B. (-2, -3) C. (-1, -5/2) D. (-3, -2)
sin 2x Solution
A.– sin 3x B. – sin2 2x y = 3x2 + 11x + 7,
sin2 2x sin 3x sin2 2x dy
= 6x + 11 i.e gradient
C. – sin 2x sin x + 2 cos x cos 2x dx
sin2 2x But we are told that gradient is –1
D. sin 2x cos x + cos 2x sinx Thus, dy/dx = –1
sin2 2x sin3x sin2 2x 6x + 11 = –1
E. – [sin 2x sin x + 2 cos x cos2x ] 6x = – 12
sin22x x = –2
Solution To get y co-ordinates
Let y = cos x Substitute x = - 2 into the original equation
sin 2x y = 3(-2)2 + 11(-2) + 7
Applying quotient rule = 12 – 22 + 7
dy = sin 2x × (- sin x) – cos x × (2cos2x) = –3
dx (sin2x)2 Co-ordinates of P is (-2, -3) (B)
= - sin 2x sin x – 2cosx cos2x (E)
2002/15
sin22x Find co-ordinates of the point at which the gradient of the
curve y = x2 – x + 4 is 3
1994/10 b (Nov) Exercise 26.19 A. (1, 4) B. (2, 6) C. (1, 6) D. (3, 5)
Differentiate, with respect to x Solution
y = 3 – 2x , simplifying your answer y = x2 – x + 4
(1 + x)2 dy
= 2x – 1 same as a gradient
1993/15 (Nov) Exercise 26.20 dx
x dy But we are told that gradient is 3
If y = find Thus, dy/dx = 3
( x  1)
2 1
2
dx
2x – 1 = 3
A. 2 B. – 1 C. 1 D. –2x2 + 1 2x = 3 + 1
(x2 – 1)3 (x2 – 1)3/2 (x2 – 1)3 (x2 – 1)3/2 2x = 4
E. 2x – 1
2 x=2
(x2 – 1)3/2 To get the y co – ordinates, substitute x = 2 into the original
equation.
2001/10 Neco Exercise 26.21 y = (2)2 – 2 + 4
sin 3 x =4–2+4
The derivative of with respect to x is =6
e 2 x
3 cos 3 x  2 sin 3 x 3 sin 3x  2 cos 3x Co – ordinate of the point (2, 6)
A. B.
e 2 x
e 2 x 1995/2 (Nov)
3 cos 3x  2 sin 3x 3 cos 3 x  2 sin 3 x Find the differential coefficient of the function
C. 4 x
D.
e e 2 x y = 1 at the point (4, 1/25)
3 cos 3 x  2 sin 3 x x2 + 9
E.
e 4 x A. -8/625 B. -4/625 C. 2/625 D.4/625 E. 8/625

320
Solution Solution
Differential coefficient is same as dy/dx. Thus y = x3 + 6x2 – 15x + 1
y = 1 : i.e y = (x2 + 9)–1 dy
2 = 3x2 + 12x – 15
x +9 dx
Let z = x2 + 9 then y = z – 1 Gradient is 0  dy/dx = 0 i.e
dy = dy × dz 3x2 + 12x – 15 = 0 divide through by 3
dx dz dx x2 + 4x – 5 = 0 factorising
= – 1z–2 × 2x i.e –2x x + 5x – x – 5 = 0
2

z2 [
x(x+5)–1(x+5)=0
= –2x (x+5)(x–1 )=0
(x2 + 9)2 x + 5 = 0 or x – 1 = 0
At point (4, 1/25), we put x = 4 into our dy/dx result x = – 5 or 1
= –8 i.e – 8 (A) Use x values to find y
(25)2 625 when x = 1
y = (1)3 + 6(1)2 – 15(1) + 1
1995/40 UME = –7
Find the gradient of the curve y = 2 x – 1 at the point ( 1, - 7 )
x=1 x when x = - 5 y = ( - 5 )3 + 6 ( - 5 )2 - 15( - 5 )+ 1
= - 125 + 150 + 75 + 1
A.0 B.1 C.2 D.3 = 101
Solution ( - 5 , 101 ) ( A )
y = 2 x – 1  y = 2x ½ – x – 1 2005/3 PCE
x At what point is the gradient of the curve x2 – 6x + 3
equal to zero ?
= x– ½ + x–2
dy
A. – 2 B. 3 C. – 3 D. 2
dx
Solution
= 1 + 1 which is the gradient
f( x ) = x2 – 6x + 3
x x2
Gradient of curve implies
At point x = 1
f / ( x ) = 2x - 6
= 1 + 1 = 1+ 1 =2(C)
Gradient = 0 implies 2x – 6 = 0 solving
1 12 1 1 2x = 6
2002/17 UME x = 6/2 Thus, x = 3 ( B )
The slope of the tangent to the curve y = 3x2 – 2x + 5 2009/11 Exercise 26.23
at the point ( 1, 6 ) is The gradient of point P on the curve y = 3x2 – x + 3 is 5.
A.6 B.5 C .4 D.1 Find the coordinates of P.
Solution A. (1, 5) B (1, 7) C (1, 13) D (1, 17)
y = 3x2 – 2x + 5
2006/26 Exercise 26.24
dy
= 6x - 2 i.e slope Find the coordinates of the point on the curve
dx y = x2 + 4x – 2, where the gradient is zero.
At point ( 1, 6 ) ; x = 1 A (-2, 10) B (-2, 2) C (-2, -2) D (-2, -6)
put x = 1 into 6x – 2
6( 1 ) – 2 = 4 ( C ) 1994/15 Exercise 26.25
Find the differential coefficient of y = – 5 at the point (1, -1).
x2 + 4
2003/39 UME
Find the slope of the curve y = 2x2 + 5x – 3 at ( 1, 4 ) . A. -2 B. -1 C. -2/5 D. 2/5 E. 2
A. 4 B.6 C.7 D.9 2009/30 (Nov) Exercise 26.26
Solution The equation of a curve is given by y = 2
y = 2x2 + 5x – 3 x3
dy Find its gradient at the point 1 .
= 4x + 5 i.e slope 
 1, 
dx  2 
At point ( 1, 4 ); x = 1 1 1 1 1
Put x = 1 into 4x + 5 A- B- C D
4 8 8 4
4( 1 ) + 5 = 9 ( D ) 2004/38 PCE Exercise 26.27
The gradient of the curve y = 3x2 – 5x + 6 when x = 1 is
1995/42 PCE A.1 B.4 C.6 D.7
Find the points on the curve y = x3 + 6x2 – 15x + 1 at
which the gradient is 0. 2001/37 PCE Exercise 26.28
A.( - 5, 101 ), ( 1, - 7 ) B.( - 5, -101 ), ( 1, - 7 ) The gradient of the curve y = x2 + 4x at the point ( 2, 10 )
C. ( 5, - 101 ) , ( - 1, - 7 ) D.( 5, 101 ) , ( - 1, 7 ) A.0 B.8 C.12 D.24

321
Turning Points dy/dx = 12 cos ( 2x – 25 )0
( maximum, & minimum ) It follows that
At turning points P and Q shown below; 12 cos ( 2x – 25 ) = 0
y which means cos ( 2x – 25 ) = 0

.
P
dy
dx =0
it implies 2x– 25 = cos –1 0
2x– 25 = 90
2x = 25 + 90

.
x = 115 i.e 57 ½ ( C )
2
Q 1996/38 PCE
x A swimming pool is treated periodically to control harmful
bacterial growth. The concentration of bacterial per cm3 after
But it is the second differential ( d2y/dx2 ) that reveal
t days is given by C( t ) = 30t2 – 240t + 500
whether the turning points leads to a maximum or In how many days after a treatment will the concentration be minimal ?
minimum. A.10 B.8 C.6 D.4
y

.
2
dy
Solution
dx2 0
Maximum At minimal, dc/dt = 0
P In this case, dc/dt = 60t – 240
It follows that 60t – 240 = 0

.
Q
Minimum
2
dy 60 60
60t = 240
60t = 240

dx2 0 t = 4 days ( D )
x
1992/19 (Nov)
The maximum point of the curve y = x – 2sin x,
The value of x for maximum or minimum point 0  x < 2 occurs when x equals
In this case the question already specifics the turning A. /6 B. /3 C. 5/3 D. 2/3 E. 4/3
point to be a minimum or maximum point. Note the use Solution
of the phrase‘ value of x’ in the subsequent questions At maximum or minimum dy/dx = 0
Thus, y = x – 2sin x will become
dy
/dx = 1 – 2 cos x
1990/30 UME Hence 1 – 2 cos x = 0
At what value of x is the function x2 + x +1 minimum ? 2 cos x = 1
A.- 1 B. – 1/2 C. 1/2 D.1 cos x = 1/2
Solution cos x = 0.5
We let y = x2 + x + 1 x = cos–1 0.5
At min or max, dy/dx = 0 = 600
In this case dy/dx = 2x + 1 0
The rage is from 0 to 2 i.e 360
It follows that Next we find other multiples of 600 that will give 0.5
2x + 1 = 0 1200, cos 120 = - 0.5 not accepted
2x = – 1 1800 cos 1800 = -1 not accepted
x = – 1/ 2 (B) 2400 cos 2400 = - 0.5, not accepted
1991/27 UME 300 cos3000 = 0.5 accepted
At what value of x is the function y = x2 - 2x - 3 minimum ? Thus, x = 600 or 3000
A.1 B. –1 C. – 4 D. 4 But for maximum d2y < 0
Solution dx2
Already, y = x2 – 2x - 3 2
which is d y = 2 sin x < 0
But at min or max; dy/dx = 0 dx2
In this case dy/dx = 2x - 2 For 2sin x < 0, x = 3000 and not 600
It follows that Next convert 3000 to radians
2x – 2 = 0
i.e 300 ×  = 5/3  (C)
2x = 2
180
x = 1 (A)
1998/36 UME 2010/38 Exercise 26.29
For what value of x does 6sin ( 2x – 25 )0 attain its Find the maximum value of 2 + sin ( + 250)
maximum value in the range 00  x  1800 ? A1 B2 C3 D4
A. 12 ½ B. 32 ½ C. 57 ½ D. 14 ½
2003/46 PCE Exercise 26.30
Solution
If y = 3x2 – 2x + 1 , find the value of x when dy/dx = 0
We let y = 6 sin ( 2x – 25 )0
A. 1/3 B.1/2 C.1 D.1/4
At min or max; dy/dx = 0 3
Cubic function ( x ) case
Applying function of function rule
322
2004/22 3
x= 
For what value of x will the function f(x) = x3 – 6x2 + 5 3
have turning points?
A. 0, 4 B. 0, 2 C. 0, - 2 D. 0, - 4 6x = 6 ×  3
3
Solution
At turning point f1(x) = 0 and from f(x) = x3 – 6x2 + 5 = 2 3 > 0 min. pt
f1(x)  3x2 – 12x = 0
3x (x – 4) = 0 3
x = 0 or 4 (A) But when x =  , 6x = – 2 3 < 0 max point
1995/6 (Nov) 3
Find the value of x at which the function x3 – 7x2 + 15x 3
has the greatest value. Hence  is the value of x for which the function has a
A. – 4 B. – 3 C. -5/3 D. 5/3 E. 3 3
Solution minimum value ( C )
At greatest or least values dy/dx = 0
y = x3 – 7x2 + 15x 2007/13 UME
dy
/dx = 3x2 – 14x + 15 Find the value of x for which the function
Hence, 3x2 – 14x + 15 = 0 f(x ) = 2x3 – x2 – 4x + 4 has a maximum value.
3x2 – 9x – 5x + 15 = 0 A. 2/3 B. 1 C. - 2/3 D.-1
3x(x – 3) – 5(x – 3) = 0 Solution
(3x – 5)(x – 3) = 0 Already, f(x) = 2x3 – x2 – 4x + 4
3x – 5 = 0 or x – 3 = 0 At min or max f / (x ) = 0
x = 5/3 or 3 In this case f / (x ) = 6x2 – 2x – 4
2
But at greatest value, d y < 0 It follows that
dx2 6x2 – 2x – 4 = 0
i.e d y = 6x – 14
2 Divide through by 2
dx 2 3x2 – x – 2 = 0
For 6x – 14 to be < 0 we take x = 5/3 (D) Factorising the resultant quadratic equation
Confirmation: 6 (5/3) – 14 will be – 4 3x2 – 3x + 2x – 2 = 0
3x(x – 1) + 2(x –1) = 0
( 3x + 2) (x – 1) = 0
1999/38 UME 3x + 2 = 0 or x – 1 = 0
Find the value of x for which the function 3x = – 2 or x = 1
y = x3 – x has a minimum value. x = – 2/3 or x = 1
But at maximum d2y/dx2 < 0 i.e f // (x ) < 0
3 3 it follows that
A.  3 B.  C. D. 3
3 3 f // (x ) = 12x – 2
Solution For 12x – 2 to be less than zero, we check as :
Already, y = x3 – x When x = – 2/3
At min or max dy/dx = 0 12x – 2 = 12(– 2/3 ) – 2
In this case dy/dx = 3x2 – 1 = – 10 < 0 max point
It follows that Hence – 2/3 is the value of x for which the function has a
3x2 – 1 = 0 maximum value ( C )
3x2 = 1 Reader to try substituting x = 1 into 12x – 2
x2 = 1/3
2006/4 UME Exercise 26.31
x =  1
3
Find the value of x for which the function 3x3 – 9x2
1 1 is minimum.
=  i.e  A.3 B.5 C.0 D.2
3 3
Rationalizing the surd 2012/15 Exercise 26.31a
= 3 i.e 3 or 3 If y = x3 – x2 + 6, find the values of x at the turning points
  
3 3 3 A.1/2 , –3 B. 1/3 , –1/2 C.1, –1/3 D.1, 1/3
But at minimum d2y/dx2 > 0
it follows that
d2y
= 6x
dx 2
For 6x to be greater than zero, we check as:

323
The value of y for maximum or minimum point To get max value, we substitute x = h/6 into y = 1+ hx - 3x2
Here we are required to go a step further from finding y = 1 + h ( h/6 ) – 3 ( h/6 )2
the value of x for max or min points; by substituting the But the max value is 13
value of x into the original equation. Though we have  13 = 1 + h2 – 3h2
two types of cases here: 6 36
Quadratic functions ( x2 ) h2 - h2 = 13 – 1
Cubic functions ( x3 ) 6 12
2h2 - h2 = 12
Quadratic function’s case 12
h2 = 12
1997/13 UME 12
Find the minimum value of x2 – 3x + 2 for all real value of x. h2 = 12 × 12 = 144
A. – ¼ B. – 1/2 C. ¼ D. 1/2 h =  144
We let y = x – 3x + 2
2
= 12 ( B )
At min dy/dx = 0 1997/43 PCE
In this case , dy/dx = 2x – 3 Find the maximum value of the function y = 10 + 4x – x2
It follows that A. 10 B.14 C.18 D.24
2x – 3 = 0 Solution
2x = 3 At maximum, dy/dx = 0
x = 3/ 2 In this case, dy/dx = 4 – 2x
to get min value It follows that
we substitute x = 3/2 into y = x2 – 3x + 2 4 – 2x = 0
y = ( 3/2 )2 – 3( 3/2 ) + 2 4 = 2x
y = 9/4 – 9/2 + 2 4 = 2x
= 9 – 18 + 8 2 2
4 x = 2
= – ¼ (A) To get the maximum value, we put x = 2 into y = 10 + 4x – x2
i.e y = 10 + 4( 2 ) - ( 2 )2
2002/22 UME y = 10 + 8 – 4
Find the maximum value of y in the equation y = 1- 2x -3x2 = 14 ( B )
A.4/3 B. 5/4 C. ¾ D. 5/3 2003/4
Solution A blacksmith produced x articles at a total cost of
Already, y = 1 – 2x – 3x2 $ (200 – 48x + 3x2). If each article is sold at $ 3/5x,
At max or min dy/dx = 0 find:
In this case dy/dx = - 2 - 6x ( a ) The value of x for which the blacksmith makes a
It follows that maximum profit.
– 2 – 6x = 0 ( b )The overall maximum profit
– 2 = 6x Solution
– 2/6 = x ( a ) CP Cost price of x articles = $(200 – 48x +3x2)
x = – 1/3 SP Selling price of x articles = cost price of one article × x articles
To get maximum value = $ 3/ 5x × x
Substitute x = - 1/3 into y = 1 – 2x – 3x2 = 3/5x2
y = 1 – 2( - 1/3 ) – 3( - 1/3 )2 Profit(P) = Sp – Cp
= 1 + 2/3 - 3( 1/9 ) = 3/5x2 – (200 – 48x +3x2)
P = 3/5x2 – 200 + 48x – 3x2
= 1 + 2/ 3 – 1/ 3
= 3 + 2 – 1 = 4/3 ( A ) At maximum profit dp/dx = 0
3 dp = 6 x + 48 – 6x
dx 5
1999/40 UME At maximum profit 6x + 48 – 6x = 0
If the maximum value of 5
y = 1 + hx – 3x2 is 13, find h. 6x – 30x = –240
A.13 B.12 C.11 D.10 x = 10 articles
Solution (b) For over all maximum profit,
We know that at max or min dy/dx = 0 we put x = 10 into our P equation
In this case, dy/dx = h – 6x P overall max, = 3 (10)2 – 200 + 48(10) – 3(10)2
It follows that 5
h – 6x = 0 = 300 – 200 + 480 – 300
h = 6x 5
x = h/6 = $40

324
2000/39 PCE But at maximum, d2y < 0
30metres of fencing wire is available to make a dx2
2
rectangular enclosure. Find the maximum area possible. ie d y = 12x + 6
A.629m2 B525m2 C.225m2 D.125m2 dx2
Solution For 12x + 6 to < 0
Let the length of the rectangle = x We take x = – 4 value and not x = 3
It follows that breath = 30 – x Since d2y = 12x + 6
Area of rectangle = length × breath dx2
= x( 30 – x ) = – 48 + 6
Area ( A ) = 30x – x2 = – 42 < 0
For area to be maximum Then substitute x = – 4 into the original equation
dA/dx = 0 y = 2 (– 4)3 + 3(– 4)2 – 72(– 4) + 1
d = – 128 + 48 + 288 + 1
( 30 x  x 2 ) = 0 = 209 (B)
dx
30 – 2x = 0 1993/18 (Nov)
30 = 2x Find the minimum value of the function 2x3 – 6x2
30 = 2x A. – 8 B. – 2 C. 0 D. 2 E. 8
2 2 Solution
15 = x At maximum or minimum dy/dx = 0
To get maximum area value Let y = 2x3 – 6x2
dy
Substitute x = 15 into 30x – x2 /dx = 6x2 – 12x
Maximum area = 30 ( 15 ) – ( 15 )2 Hence 6x2 – 12x = 0
= 450 – 225 6x(x – 2) = 0
= 225m2 (C) 6x = 0 or x – 2 = 0
2009/6 (Nov) Exercise 26.32 x = 0 or 2
Find the minimum value of the function But at maximum value d2y > 0
y = 2x2 – 6x + 3. dx2
A –1.5 B –1.0 C 2.5 D 3.0 i.e d2y = 12x – 12
2
dx
2006/27 Exercise 26.33 For 12x – 12 to be > 0 we choose x = 2 and not 0
Find the least value of the function f(x) = 3x2 +18x +32 Confirmation: 12x – 12 becomes 12(2) – 12 = 12
A5 B4 C3 D2 Next, we put x = 2 into the original equation
2x3 – 6x2 = 2(2)3 – 6(2)2
2001/23 Neco Exercise 26.34 = 16 – 24
The maximum value of the function y = 2 + 3x – 4x2 is = – 8 (A)
23 5 13 41 59
A.- B. C. D. E. 1992/39 UME
16 16 16 16 16 Obtain a maximum value of the function f( x ) = x3-12x + 11
A. – 5 B. – 2 C.2 D.27
1995/41 PCE Exerecise 26.35 Solution
What is the minimum value of y = x2 + 3x + 7 ? Recall that f / ( x ) is equivalent of dy/dx
A. – 3/2 B.3 C.19/4 D.7 While f / / (x ) is equivalent of d2y/dx2
2005/5 UME Exerecise 26.36 Already,
The maximum value of the function f( x ) = x3 – 12x + 11
f( x ) = 2 + x – x2 is At max or min f / ( x ) = 0
A.3/2 B. ½ C.9/4 D.7/4 In this case, f / ( x ) = 3x2 – 12
It follows that
Cubic function’s case 3x2 – 12 = 0
2000/32 (Nov) 3x2 = 12
Find the maximum value of the function 2x3 + 3x2 – 72x + 1 x2 = 12 i.e 4
A. – 134 B. 209 C. 298 D. 465 3
Solution x =  4
Let y = 2x3 + 3x2 – 72x + 1 = 2
At maximum or minimum point dy/dx = 0 To know which of the values of + 2 and – 2 is maximum:
dy
/dx = 6x2 + 6x – 72 f / / (x ) = 6x
Hence 6x2 + 6x – 72 = 0 When x = + 2
x2 + x – 12 = 0 6x = 6( 2 )
x – 3x + 4x – 12 = 0
2
= 12 > 0 minimum point
x(x – 3) + 4(x – 3) = 0 When x = - 2
(x + 4)(x – 3) = 0 6x = 6( - 2 )
x = – 4 or 3 = - 12 < 0 maximum point
325
To get maximum value of the function, Next, When x = – 2
we substituting x = - 2 into f( x ) = x3 – 12x + 11 6x + 6 = – 12 + 6
= ( - 2 )3 – 12 ( - 2 ) + 11 = – 6 < 0 maximum point target gotten
= - 8 + 24 + 11 To get the y co-ordinate; we substitute x = – 2 into
= 27 (D ) y = x3 + 3x2 - 7
2003/39 UME i.e y = (-2)3 + 3(-2)2 – 7
Determine the maximum value of y = 3x2 – x3 = – 8 + 12 – 7 i.e – 3
A.0 B.2 C.4 D.6 P(x, y) is (-2, -3) B
Solution
Already y = 3x2 - x3 2000/38 PCE
At max or min dy/dx = 0 Find the turning points of the function y = 2x3-6x2-18x + 3
In this case, dy/dx = 6x – 3x2 A.( - 1, 13 ), ( 3, - 51 ) B.( 0, 3 ), ( 2, - 23 )
It follows that C.( - 1, - 19 ), ( 2, 23 ) D.( - 1, 19 ), ( 3, 51 )
6x – 3x2 = 0 Solution
3x ( 2 – x ) = 0 Already, y = 2x3 – 6x2– 18x + 3
i.e 3x = 0 or 2 – x = 0 At turning point dy/dx = 0
x = 0 or x = 2 In this case dy/dx = 6x2 – 12x – 18
To know which of x = 0 or x = 2 is a max point, we It follows that 6x2 – 12x – 18 = 0
d2y/dx2 = 6 – 6x Divide through by 6
When x = 0 x2 – 2x – 3 = 0
6 – 6x = 6 – 6( 0 ) factorizing
= 6 > 0 min point x2 – 3x + x – 3 = 0
When x = 2 x( x – 3 ) + 1( x – 3 ) = 0
6 – 6x = 6 – 6 ( 2 ) (x+1)(x–3)=0
= – 6 < 0 max point x + 1 = 0 or x – 3 = 0
To get max value x = – 1 or + 3
we substitute for x = 2 into y = 3x2 – x3 At x = – 1
y = 3 ( 2 ) 2 – ( 2 )3 y = 2( - 1 )3 – 6( - 1 )2 – 18 ( - 1 ) + 3
=3(4)– 8 = – 2 – 6 + 18 + 3
= 12 – 8 = 4 C. = – 8 + 21
2003/4 Exercise 26.37 = 13
Given that y = x (x + 1)2 , i.e ( x, y ) = ( - 1, 13 )
calculate, the maximum value of y.
A–2 B0 C1 D2 At x = + 3
y = 2( 3 )3 – 6 ( 3 )2 – 18 ( 3 ) + 3
= 2 ( 27 ) – 6 ( 9 ) – 54 + 3
Turning points (x,y)
[the value of (x,y) for maximum or minimum point] = 54 – 54 – 54 + 3
= – 51
2000/6 i.e ( x, y ) = ( 3, - 51 ) Option A fits in
If the point p(x, y) is the maximum point on the curve
y = x3 + 3x2 – 7 , find the co-ordinates of p. 2002/38 PCE
A. (0, -2) B. (- 2, -3) C. (0, 7) D. (-1, -5) Find the coordinates of the minimum point for the
Solution equation y = 4t2 – 40t + 300
At maximum or minimum; dy/dx = 0 A.( 5, 200 ) B.( 5, 100 ) C.( 4,300) D.( 4, 100 )
y = x3 + 3x2 – 7 Solution
Given y = 4t2 – 40t + 300
dy = 3x2 + 6x
At minimum point dy/dt = 0
dx
Note that we did not use dy/dx; since the function is in y and t.
At maximum or minimum; dy/dx = 0
In this case, dy/dt = 8t – 40
Thus, 3x2 + 6x = 0
It follows that
3x(x + 2) = 0
8t – 40 = 0
3x = 0 or x + 2 = 0
8t = 40
x = 0 or – 2
8t = 40
To know which of the values is maximum
8 8 t = 5
d2y = 6x + 6
The y value is gotten by putting t = 5 into
dx2
y = 4t2 – 40t + 300
When x = 0
y = 4 ( 5 )2 – 40 ( 5 ) + 300
6x + 6 = 6  0 minimum point not our target = 4 ×25 – 200 + 300
= 100 + 300 – 200 = 200.
i.e ( t, y ) is ( 5, 200 ) ( A ).

326
2004/39 PCE Solution
The turning point of the curve y = 5 – 2x = x2 occurs at 2x2 differentiation is direct as d (2x2) i.e 4x
A.( - 2, 5 ) B.( - 1 – 6 ) C.( - 1, 6 ) D.( 1, 6 ) dx
Solution 5y2 differentiated as a chain d (5y2)
At turning point dy/dx = 0 dx
y = 5 – 2x – x2 will become d (5y2) × dy i.e 10y dy
dy/dx = –2 – 2x dy dx dx
At turning point : dy/dx = 0 6xy is differentiated as product:
i.e –2 – 2x = 0 d(6xy) = 6x dy + 6y dx
–2x = 2 dx dx dx
– 2x = 2 = 6x dy + 6y
–2 – 2 dx
x =–1 2x2 – 5y2 = 6xy differentiated wrt x becomes
The y value is gotten by putting x = - 1 into 4x – 10y dy = 6x dy + 6y
y = 5 – 2x – x2 dx dx
y = 5 – 2( - 1 ) – ( - 1 )2 Collect like terms together
=5+2–1= 6 4x – 6y = 6x dy + 10y dy
i.e ( x, y ) = ( –1, 6 ) (C) dx dx
Factor out dy/dx in the RHS
1998/41 PCE Exercise 26.38 dy
4x – 6y = (6x + 10y)
The turning point of the curve y = 6 – 4x – x2 occurs at dx
A.( - 2, 10 ) B.( 0, 6 ) C.( 2, - 6 ) D.( 4, -26 ) Make dy/dx the subject of formula
4x – 6y = dy
1992/41 PCE Exercise 26.39 6x + 10y dx
The turning point of the curve y = 5 – 8x – 2x2 occurs at 2(2x – 3y) = dy
A.( - 2, 13 ) B.( 2, 13 ) C.( 2, - 13 ) D.( - 2, - 13 ) 2(3x + 5y) dx
2x – 3y = dy (E)
2005/36 Exercise 26.40 3x + 5y dx
Find the stationary point of the function f(x) = –3x2 + 3x + 2
A.(–11/4 , 9/4 ) B.( – 3/2 , 1/2) C. ( 1/2, 11
/4 ) D.(–1/2 , 9/4 ) 2005/11a ( i )
Differentiate with respect to x : y2 + x2 – 3xy = 4
1994/5(Nov) Exercise 26.41 Solution
250 y2 is differentiated as chain:
Determine the turning point of the curve y = x2 + d ( y2) = d ( y2) × dy i.e 2y dy
x
dx dy dx dx
A. (-5, -25) B.(5, 75) C. (5, 150)
x2 is differentiated directly as: d (x2) = 2x
D. (10, 75) E. (5, 0)
dx
3xy is differentiated as product:
2000/17 Neco Exercise 26.42
d (3xy) = 3x dy + 3y dx
Determine the turning point of the curve
dx dx dx
y = 2x2 + 8788
= 3x dy + 3y
x
dx
A. ( -3, -1014) B. ( -13, -845) C. (13, -1014)
4 is a constant, hence its differential is zero
D. (13, 845) E. ( 13, 1014)
y2 + x2 – 3xy = 4 differentiated becomes
1994/5(Nov) Exercise 26.43
250 2y dy + 2x – 3x dy + 3y = 0
Determine the turning point of the curve y = x2 + dx dx
x We introduced bracket because of minus sign before3xy
A. (-5, -25) B.(5, 75) C. (5, 150) 2y dy + 2x – 3x dy – 3y = 0
D. (10, 75) E. (5, 0) dx dx
Collect like terms together
Implicit functions 2y dy – 3x dy = 3y – 2x
1997/10 dx dx
If 2x2 – 5y2 = 6xy, find dy Factor out dy/dx in the LHS
dx dy (2y – 3x) = 3y – 2x
A. 3x + 5y B. 2x + 3y C. 2x + 3y dx
2x – 3y 3x – 5y 3x + 5y Therefore, dy = 3y – 2x
D. 2x E. 2x – 3y dx 2y – 3x
3x + 5y 3x + 5y

327
2004/2(a) Neco (Dec) Factor out dy/dx
dx dy (4x3y – 6xy) = 3y2 – 6x2y2
Find if 4x4 + y3 = 12x2y dx
dy dy = 3y2 – 6x2y2
Solution dx 4x3y – 6xy
4x4 is differentiated directly as
d ( 4x4) = 16x3 IPLF 2 Example
dx y
y3 is differentiated as chain : Find the dy/dx if ( i ) cos x = sin y ( ii ) xy = log
x
d ( y3 ) = d (y3) × dy = 3y2 dy Solution
dx dy dx dx (i) cos x = sin y
12x2 y differentiated as product: d ( cos x) = d (sin y)
d (12x2 y) = 12x2 dy + 12y d (x2) dx dx
dx dx dx – sinx = (cosy) dy
= 12x2 dy + 24xy dx
dx dy = – sinx
4x4 + y3 = 12x2 y differentiated becomes dx cosy
16x3 + 3y2 dy = 12x2 dy + 24xy y
dx dx ( ii ) xy = log
Collect like terms together x
3y2 dy – 12x2 dy = 24xy – 16x3 xy = logy – logx
dx dx x dy + y × 1 = 1 dy – 1
Factor out dy/dx in the LHS dx y dx x
dy (3y2 – 12x2) = 24xy – 16x3 x dy – 1 dy = – (y + 1/x)
dx dx y dx
dy = 24xy – 16 x3
dx 3y2 – 12 x2 (x – 1/y) dy/dx = – (y + 1/x)
2003/6(b) Neco (Dec) xy – 1 dy = – 1 + xy
dx y dx x
Find of the function 2x3y2 – 3xy2 = 4 dy = – 1+ xy × y
dy
dx x xy –1
Solution
= – y(1 + xy)
2x3y2 is differentiated as product:
d (2x3 y2) = 2x3 d ( y2) + 2y2 d (x3) x(xy – 1)
dx dx dx
Implicit functions in gradient at a point
Note that: d ( y2) is d ( y2) × dy i.e 2y dy 2000/10 (C) (Nov)
dx dy dx dx Find the gradient of the curve y2x + 3x2 y = 1,
d (2x3 y2) = 2x3 × 2y dy + 2y2 × 3x2 at the point (1, 1)
dx dx Solution
= 4x y dy + 6x2y2
3
First, we differentiate implicitly.
dx y2 × dx + x × d (y2) + 3 x2 dy + 3y × d (x2) = 0
3xy2 is differentiated as product: dx dx dx dx
d (3xy2) = 3x d (y2) + 3y2 dx 2 2
dx dx dx y + x × 2y dy + 3 x dy + 3y × 2x dx = 0
Note the result of d (y2) as gotten earlier
dx dx dx
2 2
dx y + x × 2y dy + 3 x dy + 3y × 2x = 0
= 3x × 2y dy + 3y2 dx dx
dx y2 + 2x dy + 3 x2 dy + 6xy = 0
= 6xy dy + 3y2 dx dx
dx Collect like terms together
4 is a constant, hence its differential is 0 2xy dy + 3x2 dy = – (y2 + 6xy)
2x3 y2 – 3xy2 = 4 differentiated becomes dx dx
4x y dy + 6x2 y2 – 6xy dy + 3y2 = 0
3 dy (2xy + 3x ) = – (y2 + 6xy)
2

dx dx dx
Opening up bracket we have dy = – (y2 + 6xy)
4x3y dy + 6x2y2 – 6xy dy – 3y2 = 0 dx 2xy + 3x2
dx dx At point (1, 1) , x is 1 and y is 1
Collect terms in dy/dx together dy/dx = – (1 + 6 ) = -7/5 i.e -12/5
4x3y dy – 6xy dy = 3y2 – 6x2y2 2+3
dx dx
328
1996/16 (Nov) Comparing rates of change
The gradient of the curve x2– 2xy – 2y2 – 2x = 0
If we are given a ‘ main ‘ function say y = f( x ). Then
at the point (1, – 4) is
differentiating the main function y w.r.t time t
A. -4/3 B. -4/7 C. -1/2 D. -4/9 E. -1/8 ( instead of wrt to x ) will have a connecting formula
Solution through chain rule as:
First, we differentiate implicitly.
dy dy dx Where our dy/dx here can be gotten
2x – 2x dy – 2y – 4y dy – 2 = 0  
dx dx dt dx dt from the main function which is
either given directly or mentioned
Collect terms in dy/dx together in the question as shown in the
2x – 2y – 2 = 2x dy + 4y dy examples below:
dx dx 2000/38 UME
2x – 2y – 2 = dy (2x + 4y) If the volume of a hemisphere is increasing at a steady rate
dx of 18m3 s – 1, at what rate is its radius changing when
2x – 2y – 2 = dy it is 6m ?
2x + 4y dx A. 2.50ms- 1 B. 2.00ms- 1 C.0.25ms – 1 D.0.20ms – 1
2(x – y – 1) = dy Solution
2(x + 2y) dx The main function here is the vol. of hemisphere V = 2/3r3
Thus, dy = x – y – 1 dv dv dr
dx x + 2y But   ---------***
dt dr dt
At point (1, – 4) ; x is 1and y is – 4
Here dv/dt is given as 18m3s1 and dv/dr is gotten from
dy/dx = 1 + 4 – 1
1–8 the main function
= – 4 (B) V = 2/3r3
7 dv/dr = 2r2
1997/11a At r = 6m
Given the curve 50x2 + 36xy + 5y2 = – 2 2r2 = 2 ( 6 )2
Evaluate dy/dx at the point (1, – 2) = 72
Substituting for dv/dt and dv/dr into *** ,
Solution dr
Differentiating implicitly, we have 18 = 72 ×
50 d(x2) + 36x dy + 36y dx + 5 d (y2 ) = 0 dt
dx dx dx dx 18 = dr/dt
72
100x + 36x dy + 36y + 10y dy = 0 dr/dt = ¼ = 0.25ms – 1 ( C )
dx dx The unit is so because length r in metres and time t in
Collect terms in dy/dx together seconds. dr/dt means unit metres per second or
36x dy + 10y dy = – (36y + 100x) metres/seconds or metres second – 1 ( ms- 1 )
dx dx
dy (36x + 10y) = – (36y + 100x) 2002/21 UME
dx A circle with a radius 5cm has its radius increasing at the
dy = – (36y + 100x) rate of 0.2cm s- 1 .What will be the corresponding increase
dx (36x + 10y) in the area ?
At point (1, –2) ; x is 1 and y is – 2 A.  B.2 C.4 D.5
dy/dx = – (– 72 + 100) Solution
36 – 20
The main function here is Area of a circle A = r2
= -7/4 i.e -13/4 dA dA dr
But   ---------***
dt dr dt
1993/12 (Nov) Exercise 26.44 Here dA/dt is not given.
If x2 + 3xy + 2y2 = 3, find dy/dx at the point (– 1, 2 ). dA/dr is gotten from the main function
A. ¾ B. 8/11 C. 7/11 D. 2/3 E. – 4/5 A = r2
dA/dr = 2r
IPLF 3 Exercise 26.45
When r = 5cm
If ax2 + 2hxy + by2 = 0, find dy/dx
2r = 2 ( 5 )
IPLF 4 Exercise 26.46 = 10
y4 + x2 = x + y2 , find dy/dx at the point (– 1, 1). We were given that: dr/dt = 0.2cms- 1
Substituting for dA/dr and dr/dt into ***,
dA
2010/4 Exercise 26.47 = 10 × 0.2
If 3x2 + 2y2 + xy + x – 7 = 0, dt
dy = 2 ( B )
find at the point (-2, 1).
dx
329
1998/42 PCE Curve sketching
The radius of a circle is increasing at the rate of The basic guides in curve sketching are:
0.4cm s- 1. Find the rate of increase in cm2 s- 1 of the Turning points one
area when the radius is 5cm. At turning points dy = 0
A.40 B.10 C.5 D.4 dx
Provided NO TWO ROOTS ARE EQUAL in the given function
Solution
when factorised after differentiation then it could be
The main function here is area of circle i.e A = r2 maximum or minimum
dA dA dr
But  
dt dr dt
We are to find dA/dt , dr/dt is given as 0.4cms- 1.
d 2y d 2y
But dA/dr can be gotten from the main function 2 0
2 0
dx dx
A = r2
dA/dr = 2r
When r = 5cm Next is the intercept at the y-axis and x-axis
2r = 2 ( 5 )
= 10 Turning points two
Therefore If TWO ROOTS ARE EQUAL in the given function when
factorised after differentiation then it is an inflexion
dA/dt = 10 × 0.4
= 4 ( D )
d2y
2 =0
dx
1992/40 PCE Exercise 26.48
Find the rate of increase of the area of a square when
the length of the side is 3cm and is increasing at the d2y
rate of 0.2cms- 1 Note that at the point of inflexion 2 = 0
dx
A.12cms- 1 B.2.4cms- 1 C.1.6cm- 1 D.1.2cms- 1 To know the function’s pattern before and after the point of
inflexion
2005/3 UME Exercise 26.49 we get x and y values to the left and right of the point of
The radius r of a circular disc is increasing at the rate of inflexion say x = -2 and +2 into the main equation to get
0.5cm/secs. At what rate is the area of the disc the corresponding y values. You could try more –3and+3
increasing when its radius is 6cm ?
A.3cm2/sec B.36cm2/sec C.18cm2/sec Example CSK 1
D.6cm2/sec Sketch the curve y = x2 – 1
Solution
2006/25 PCE Exercise 26.50 y = x2 – 1
The radius of an ink drop on a blotting paper expands at At turning points dy = 0
the rate of 0.02 cm/sec. Find the rate of change of the dx
area when the radius is 5 cm dy  2x = 0
A.cm2/sec B.2cm2/sec C.0.2cm2/sec dx
x = 0 (No equal roots here hence no inflexion)
D.0.5cm2/sec
To determine whether max or min
d2y
= 2  0 min point
dx 2
Intercept at the y-axis i.e x = 0 in the original equation
y = 02 – 1
= – 1 the curve cut the y-axis at – 1
Intercept at the x-axis i.e y = 0 in the original equation
0 = x2 – 1
x=1
The curve cut the x-axis at x = -1 and at x = +1 .Sketching
y

x
-1 1

-1

330
Example CSK 2
Sketch the curve y = x3 Intercept at the x-axis i.e y = 0 in the original equation
Solution 0 = x3 – x2 – 5x
y = x3 i.e x(x2 – x – 5) = 0
At turning points dy = 0 x = 0 or (x2 – x – 5) = 0
dx cannot be factorized (x – x – 5) = 0
2
dy  3x2 = 0
dx  1  1  4  1  (5)
x = 0 twice (equal roots present hence inflexion) x =
2 1
d2y  1  21
At the point of inflexion =0 =
dx 2 2
i.e 6x = 0 1  4 .6 1  4 .6
= or
x = 0 thus inflexion occurred at x = 0 2 2
To know the function’s pattern before and after the = - 1.8 or 2.8
point of inflexion take - 2 and +2 Thus the curve cut the x-axis at x = - 1.8, 0, and 2.8

Before inflexion x = –2 after inflexion x = +2 Intercept at the y-axis i.e x = 0 in the original equation
y = (-2)3 y = (+2)3 y = 03 – 02 – 5(0) i.e 0
= –8 = +8 Thus the curve cut the y-axis at y= 0 Sketching
curve going down curve going up y

Sketching
y
8 x
-1.8 -1 1.7 2.8

0
2 x
-2

-8

Example CSK 3
Sketch the curve y = x3 – x2 – 5x
Solution
y = x3 – x2 – 5x
At turning points dy = 0
dx
 3x2 – 2x – 5 = 0
3x2 + 3x – 5x – 5 = 0
3x( x + 1) – 5( x + 1) = 0
(3x – 5)( x + 1) = 0
x = – 1 or 5/3 (1.7)
(No equal roots here hence no inflexion)
To determine which of x = –1 or 5/3 (1.7)is max or min
d2y
= 6x – 2
dx 2
when x = – 1
d2y
value becomes – 8 < 0 max point
dx 2
when x = 5/3
d2y
value becomes +8 > 0 min point
dx 2

331
Chapter Twenty seven We can also relate i to iv with differentiation for better
Integration understanding; since they are opposite.
Integration is the opposite of differentiation. It is Differentiation integration
represented by elongated s i.e ∫ d
sin x   cos x
dx  cos xdx  sin x  k
x n 1
In general :  x dx 
n

n 1
d
cos x    sin x  sin xdx   cos x  k
dx
i.e increase the power by one and divide by new power.
This is the fundamental formula of integration.
d
ln x   1 1
 x dx  ln x  k
dx x

Indefinite Integrals I
d x
dx
 
e  ex e
x
dx  e x  k
Lets us check this out
y = 10x3 + 3x and y = 10x3 + 3 1994/41 UME
Now differentiating Integrate 1 – x with respect to x
dy = 30x2 + 3 while dy = 30x2 x3
dx dx A. x – x2 + k B. 4 – 3 + k C. 1 – 1 + k D.1 – 1 + k
To get back our original functions, we integrate i.e x4 x4 x3 x 2x2 3x2 2x
reverse the differentiation. Solution
dy
   1 x  1 x
 dx   30 x  3   dx    3  3  dx
2
3 
x  x x 
cross multiply the dx
1 1
 dy   30x 
 3 dx  x dx  x
2 dx
3 2

Since there seems to be nothing on the LHS, it means 1   x 3 dx   x 2 dx


and 1 implies y0, i.e the variable there. Thus,
y0  1
  30x dx   3dx = x – 3+1 – x –2+1 + k
2

0 1 –3+1 – 2+1
Applying the formula to the RHS
y = 30x3 + 3x0+1 = x – 2 – x –1 + k
3 0+1 –2 –1
y = 10x3 + 3x =–1 + 1 + k
We are back. 2x2 x
Similarly, Rearranging
dy = 1– 1 + k
 dx   30 x
2
x 2x2 (C)
Note: All integrals must be simplified as far as possible
cross multiply the dx before integrating as shown above.
 dy   30x dx
2

y = 10x3 1997/41 UME


We are back but we lost something which is the Integrate 1 + cosx with respect to x
constant + 3. Thus we always add constant “C” or “K” x
to any indefinite integrals like the one shown above. A. –1 + sinx + k B. Inx + sin x + k C. Inx - sinx + k
Hence our actual answers are: x2
y = 10x3 + 3x + k and y = 10x3 + k D. –1 - sinx + k
x2
There are some basic integrals we need to know. Solution
i.  cos xdx  sin x  k 1  1
ii.  sin xdx   cos x  k  x
  cos x 

dx   x dx   cos x dx
1 Recall their results.
iii .  x dx  ln x  k = Inx + sin x + k (B)

 e dx
iv. x
 ex  k 1996/40 PCE
If dy = x3 – 2x2 – 3x + 1 , find y
dx
Also integral of sum of functions is the sum of the A. x4 – 2x3 – 3x2 + x + c B. x4 – 2x3 – 2x2 + x + c
separable integrals i .e 4 3 2 4 3 3 2
 x 
 3x  5 dx   x 2 dx   3xdx   5dx C. 4x4 – 3x3 – 3x2 – x + c D. 4x4 – 2x3 – 3x2 - x + c
2

2 2 3 2
332
Solution Solution
dy = x3 – 2x2 – 3x +1 Applying trig identities
dx
 tan xdx =  (sec 2 x  1)dx
2

Cross multiply dx
dy = (x3 –2x2 – 3x + 1)dx =  sec 2 xdx   dx
Integrating both sides
= tanx – x + c (A)
 dy   x 
 2 x 2  3x  1 dx
3

Applying integration formula 2003/23 Neco


y = x4 – 2x3 – 3x2 + x + c (A)
4 3 2

Evaluate (e x  sec x tan x)dx
Solution
1996/39 PCE
 (e  sec x tan x)dx =  e x dx   sec x tan xdx
x

x 2  3x  2
Simplify  dx Applying result of the respective integrals
x 1 = ex – secx + c
A. x3 + 3x2 +2x + c B. x3 + 2x + c C. 2x + 3 + c 2006/5 UME Exercise 27.0
3 2 3 2 If dy = x + cosx, find y. A. x2 – sinx + c
D. x2 + 2x + c dx 2
2 B. x2 - sinx + c C. x2 + sinx + c D. x2 + sinx + c
Solution 2
First we simplify the numerator by factorization. 1992/42 PCE Exercise 27.1
x2 + 3x + 2 = (x + 2)( x + 1) will give x + 2 Integrate x2(x +2) with respect to x A.x4+ x3 + 4 + c
x+1 x+1 B.x3 + x4 + 7 + c C.x3 + x2 + c D. x4+ 2x3 + c
x 2  3x  2
dx   x  2dx
4 3 4 3
Thus, 
x 1 Indefinite Integrals II (change of variable)
2
= x + 2x + c (D) In some problems, the integral does not fit in directly, in such
2 cases, a substitution involving change of the variable will help.

1994/45 PCE Example INTER 1 Solve  cos 2d


Integrate 2x (2x2 – 3x + 4) with respect to x. Solution
A.12x2– 12x + 8 B.x4 – x3 + 8x2 + c Our target will be to change 2 and d
C.x4 – x3 +x2 + c D. x4 – 2x3 + 4x2 + c We let u = 2 
Solution Differentiating u with respect to 
We simplify the function by opening up the bracket, du = 2
 2x2x 
 3x  4 dx   4x 
 6 x 2  8x dx
2 3
d
Applying integration formula Cross multiply
= 4x4 – 6x3 + 8x2 + c du = 2d
4 3 2 Making d subject formula
= x4 – 2x3 + 4x2 + c ( D ) du = d
2005/4 PCE 2
A function whose derivative is 1 has its integral as Substituting for 2 and d. Our new integral is
x2 du
A. –1 + k B.1 + k C –1 + k D. 2 + k  cos u 2
2x x x x Factor out the constant. ½
Solution 1
We are simply asked to integrate 1/x2. =  cos u du
2
1
Thus  2 dx   x  2 dx = ½ sinu + k; Replacing u with 2 = 1sin2 + k
x
Applying the formula: 2
= x –2 + 1 + k 2002/18 UME
–2+1
k or c are just constant usually added to indefinite
Evaluate sin 3xdx 
A.–1 cos3x +c B.1cos3x +c C. 2cos3x +c D.– 2cos3x + c
integrals 3 3 3 3
= x –1 + k = –1 + k (C) Solution
–1 x Our target here will be to change 3x and dx
2005/39 Neco Let u = 3x
Integrate tan2x Differentiating u with respect to x
A tanx – x + c B tan2x – 2x +c C x – tan2x +c du = 3
D x – tanx +c E – tanx + 2x + c dx
333
cross multiply 2001/40 PCE
du = 3dx Evaluate cos 3x  sin 4 x  dx

making dx the subject formula
du = dx A. 1 sin3x + 1 cos4x + k B. 1 sin3x – 1 cos4x + k
3 3 4 3 4
3 4
Substituting for 3x and dx ; our new integral is. C. 3sinx + 4cosx + k D. .sin x + 4cos x + k
du Solution
 sin u 3 Lets break them into bits

Factoring out the constant 1/3;


1
=  sin u du  
= cos 3x dx  sin 4 x dx
3 Changing variables
= - 1 cos u + c 
For cos 3x dx
3 Let u = 3x
replacing u with 3x = - 1 cos 3x + c (A) Differentiating u with respect to x
3 du = 3
Note: The constant k is a dummy variable, hence the dx
examiner here used C; any other alphabet can be used in it’s Cross multiply
place. Though C and K are commonly used du = 3dx
Making dx subject formula.
2001/34 UME du = dx
Evaluate  22 x  3 dx
2
3
3
Substituting for 3x and dx ; our new integral is
A. 3 (2x – 3 )5/3 + k B. 6 (2x –3 )5/3 + k
du
5
C .2x – 3 + k
5
D.2(2x – 3 ) + k
 cos u 3
Solution Factor out the constant 1/3 ,
We can not expand the bracket as in the case of say 1
3
= cos u du
 2 x  3  2 x  32 x  3 dx
2
dx 
= 1 sin U + k
 4x  12x  9dx
2
= 3
Next we replace u with 3x
=  4 x dx  12xdx   9dx
2
i. e = 1 sin 3x + k
you know what follows 3
In this case we have in hand,
Similarly
= 2  2 x  3 dx
2
3

1
We have factor out the constant 2; next lets target 2x - 3 and dx  sin 4 x dx  cos 4 x  k
4
let u = 2x – 3 ;Differentiating u with respect to x
du = 2 Thus,  cox3x  sin 4 x  dx =  cos 3x dx   sin 4 x dx
dx = 1 sin3x – 1cos4x + k (B)
Cross multiply 3 4
du = 2dx No need of adding two ks
Making dx the subject formula I HOPE YOU OBSERVE A FORMAT HERE.
du = dx 1
2  cos 3x dx  3 sin 3x  k
Substituting for 2x –3 and dx, our new integral is 1
2 du
2 u 3
 sin 4 x dx   4 cos 4 x  k
2 1
Factor out the constant 1/2  cos 5 x dx  5 sin 5 x  k the pattern goes on
2
=  u 3 du While that of Sine carries a minus sign along as shown below:
2

2 1
2/3 + 1
 sin 4 x dx  cos 4 x  k
4
= U + k
2
/3 +1 1
 sin 5x dx   5 cos 5x  k
= U5/3 + k
1
 sin 6 x dx   6 cos 6 x  k the pattern goes on
5
/3
= 3U5/3 + k ; Making a replacement of U = 2x – 3
5 1994/20 Exercise 27.2
= 3 (2x -3)5/3 + k (A) Using the substitution u = cosx or otherwise, find
 cos xsin xdx
5
5
334
1996/41 PCE Exercise 27.3 1 1
Find the integral of 3cos2x – 2sinx = ln(x3 + 1)
A.6sin2x – 2 cosx + c B. – 6sinx – 2cosx + c 3 0
C. 3/2 sin2x – 2cosx + c D. 3/2 sinx + 2cosx + c Substituting for the upper& lower limits
 log e 13  1  log e (0 3  1)
1 1
1999/46 PCE 3 3
If dy = 1 (sin4x + sin2x + 1), find y. 1 1
dx 2 = log e 2  log e 1
3 3
A. cos4x + cos2x + x + k B. –2cos4x – cos2x + x + k 1 2 1
8 4 2 2 = log i.e log e 2
C. – cos4x – cos2x + x + k D. 2cos4x + cos2x + x + k 3 1 3
8 4 2 2
Solution 1996/16 Exercise 27.4
dy = 1 (sin4x + sin2x + 1) 1 2
Evaluate  dx
dx 2 0 1  2x

Cross multiply the dx 1 1


A log e 2 B  log e 3 C log e 3 D log e 3 E 0
dy = 1/2( sin4x + sin2x +1)dx 2 2
Integrating both sides INTER 3 Exercise 27.5
1
 dy   2 sin 4 x  sin 2 x 1 dx Evaluate
7x
 3( x 2  3) dx
1
  sin 4 x  sin 2 x  1 dx INTER 4 Exercise 27.6
2
Applying change of variable format to RHS x3
Evaluate  4 dx
y = 1 – 1cos4x – 1 cos2x + x + k 2 x 1
2 4 2
Opening the bracket Definite Integrals
= – cos4x – cos2x + x + k ( C ) In definite integrals, constant “k” or “c” are not added to our
8 4 2 result since the value of the constant is gotten from the data
in the question.
If you observe that: Definite integrals are of two kinds:
the numerator is a derivative of the I. The problems where values of x and y are given to
denominator enable us find the value of the constant “k” or “c”
g 1 x  II. The problems, where lower and upper limits are
i.e.  g ( x) dx = ln g(x) given, thus taking care of the “k” or “c”
where g1(x) is the derivative of g(x)
Definite Integral I
Example INTER 2
4x  4 1998/41 UME
 x 2  2 x  5 dx Find the equation of the curve which passes through the
point (2,5) and whose gradient at any point is given by 6x – 5
Solution A.6x2 – 5x + 5 B.6x2 + 5x +5 C.3x2 – 5x – 5 D.3x2 – 5x + 3
4x  4 2x  2
Observe that  x  2x  5
2
dx = 2 2
x  2x  5
dx Solution
Gradient  dy = 6x – 5
And it can be seen that the numerator is the dx
derivative of the denominator dy = (6x - 5)dx
2x  2 integrating both sides
2 dx = 2ln(x2 – 2x + 5)
x  2x  5  dy   6x  5dx
2

y   6 x dx   5dx
2003/7i
1 x2 y = 6x 1+1 – 5x0+1
Evaluate  0 x 3 1
dx
1+1 0+1
Solution y = 6x 2 – 5x1
If we differentiate the denominator x3 + 1 it results to 2 1
3x2 but the numerator has x2 only y = 3x2 – 5x + k
so we introduce 1/3 to take of the coefficient 3 imported Next we find k value
Point (2,5) means x = 2, y = 5
1 x2 1 1 3x 2
Thus  0 x 3 1
dx 
3  0 x 3 1
dx Substituting x and y values into y = 3x2 – 5x + k
5 = 3(2)2 – 5(2) + k
335
5 = 12 – 10 + k To get C value, we substitute for the first set of
5–2 = k A and r values
k=3 2 = (1)3 – 1 + c
Hence y = 3x2 – 5x + 3 (D) 2=c
Thus A = r3 – r + 2
2000/39 UME Next we are to find A,when r = 2
A function f(x) passes through the origin and its first A = (2)3 – 2 + 2
derivation is 3x + 2 . What is f(x)? A = 8 (C)
A.y = 3x2+2x B.y = 3x2+ x C.y = 3x2+x D.y =3x2+2x
2 2 2 1994/4 Exercise 27.7
dA
Solution If = 3x3–1 and A=1 when x = 0, find A when x = 1
First derivation  dy/dx = 3x + 2 dx
dy = (3x + 2)dx A–10 B –8 C8 D10
Integrating both sides
2005/37 Exercise 27.8
 dy   3x  2dx Given that
dy
= 6x2–x +2 and y = 5 when x = 1,
y = 3x2 + 2x + k dx
2 find an expression for y in terms of x
Next, we find k value A. 2y = 4x3 – x2 – 4x + 3 B. 2y = 4x3 – x2 + 4x + 3
The origin means (0,0) i.e x = 0, y = 0 C. 2y = 4x3 + x2 – 4x + 3 D. 2y = 4x3 – x2 – 4x – 3
Substituting x and y values into y = 3x2 + 2x + k
2 2003/47 Neco Exercise 27.9
0 = 3(0)2 + 2(0) + k dy
2 If = 4x–3 and y = 5 when x = 2, find y in terms of x
dx
i.e k = 0
A 2x2 – 3x + 5 B 2x2 – 3x + 3 C 2x2 – 3x
Hence y = 3x2 + 2x (A)
D 2x – 3x – 4
2
E 4x
2
2002/20 UME 2006/1 UME Exercise 27.10
If dy/dx = 2x – 3 and y = 3 when x = 0, find y in terms of x. The gradient of a curve is 2x +7 and the curve passes
A. x2 – 3x – 3 B.2x2 – 3x C. x2 – 3x + 3 D. x2 – 3x. through point (2,0). Find the equation of the curve.
Solution A. y = x2 + 14x + 11 B. y = x2 + 7x + 9
2
dy = 2x - 3 C. y = x + 7x - 18 D. y = x2 + 7x + 18
dx
dy = (2x - 3)dx
Integrating, we have Definite integral II
 dy   2x  3dx 2004/16 UME

 x 
y = 2x2 – 3x + k 3
2
1 dx .
Evaluate
2 1
y = x2 – 3x + k A. 2 B. –2 C. –62/3 D. 62/3
Next we find the value of k. 3 3
Given : x = 0, y = 3 Solution
Substituting x and y values into y = x2 - 3x + k
 x 1dx = x3 - x
3
2
3
3
3 = (0)2 – 3(0) + k 1
3 = k 1
Hence y = x – 3x + 3 (C)
2
We substitute for upper and lower limits
2002/5 (Nov) = (3)3 – (3)1 – ( 1 ) 3 – (1)
dA 3 3
If = 3r2–1 and A = 2 when r = 1, find A when r = 2
dr = 27 – 3 – [ /3 – 1 ]
1

A–10 B –8 C8 D10 3
Solution = 27 – 9 – 1 – 3
dA 3 3
= 3r2–1
dr = 18 – – 2
dA = (3r2–1)dr 3 3
Integrating both sides, we have = 18 + 2 = 20 = 62/3 (D)

 dA   3r 
2
 1 dr 3 3 3

A = 3r3 – r + c
3
= r3 – r + c
336
The general pattern is = 1 – [ –8 – 12 – 6 ]
3 3
 f x  dx  F x 
b
= F(b) – F(a)
b
a = 1 – [ –26 ]
a
3 3
Where capital F is the result of the integral and b is the = 1 + 26
upper limit while a is the lower limit. 3 3
Though we will treat the formula pictorially under = 27 = 9 (C)
application in later part of this topic, 3
2003/37 UME 1998/40 UME

 sec 

Evaluate  x  x  tan 2 x dx
3 2
2
 2 x dx Evaluate
2
2

A.4 B.2 C.4 D.1 A.  B.  - 2 C.  D.  + 2


2 3
3 3
Solution
Solution
Recall the trig identity : 1 + tan2 = sec2
 x 
3 3
 sec   1  tan 
2
 2 x dx = x3 – 2x2  2
x  tan 2 x dx 
 2
x  tan 2 x dx
2 Thus
3 2 2 2 2

= ( 3 )3 – ( 3 )2 – ( 2 )3 – ( 2 )2
3 3
=  2
1dx

= 27 – 9 – 8 – 4 
= x
3 3
2
= 0 – 8 – 12
= –2(B)
3
1997/42 UME
= – [ – 4/3 ] i.e + 4/3 (C) 1
2000/35 UME If y = x(x4 + x2 + 1), evaluate  1
y dx
 cos  1 2
Find the value of 0 sin 2 
d A. 11
12
B.11 C. 5 D.0
16 6
A.  B.  C. - D. - Solution
2 2 First, we expand the bracket

 xx   x 
1 1
Solution 4
 x 2  1 dx = 5
 x 3  x dx
 cos  1
2
  sin  2 1 1

 0 sin 2 
d   0 sin 2 
d
= x6 + x4 + x 2
1

Recall from sin2 + cos2 = 1 6 4 2 -1


sin2 = 1 – cos2 = ( 1 ) + ( 1 ) + ( 1 ) – ( -1 )6 + ( -1 )4 + ( -1 )2
6 4 2

But – sin  = cos2 – 1


2 6 4 2 6 4 2
You will observe that all the powers of –1 in the bracket are
 even; hence all their values will be positive
=   0
d
=1 + 1 + 1– 1 + 1 + 1 =0(D)
 6 4 2 6 4 2
= –
1992/41 UME
0 
= –  – [–0 ] i.e –  (D)
 2 cos 2 x dx
4
Evaluate the integral 
1999/37 UME 12
2
A. –1/2 B. –1 C. 1/2 D. 1
2 x 1 dx
1
Evaluate
Solution
 
A. –31/3 B.7 C.9 D.11
 2 cos 2 x dx  2 4 cos 2 x dx
4
Solution 
12 12
We expand the bracket
We change variables 2x and dx
 x 
2

2 x 1
1 1
dx  2
 2 x  1 dx Let u = 2x
2
Differentiating u with respect to x
= x3 – 2x2 + x 1 du = 2
3 2 -2 dx
We substitute for upper and lower limits du = 2dx
= (1)3 – (1)2 + 1 – [ (-2 )3 – ( -2 )2 + (-2) ] du = dx
3 3 2
= 1 – 1 + 1 – [ –8 – 4 –2 ] Substituting
3 3
337
 1  x dx
 1 

du
= 2  4 cos u 2 2
1 1  x 2  dx 
2
12 2 1
Factor out constant 1/2 ;
2 4 = 1x1 – x -1 2

2 12
= cos u du
1 –1 -1

/4 = x + 1 2
= Sin u

/12 x -1
substituting for u = 2 + 1 – –1 + 1

/4 2 –1
= Sin 2x 1
 = 2 /2 – ( –1 –1 )
/12
= 21/2 – (– 2 )
= sin2(/4) – sin2(/12)
= 21/2 + 2 = 41/2 (A)
= sin /2 – Sin /6
1998/43 PCE
= Sin 900 – Sin 300 

= 1– 1/2 i.e 1/2 (C)



2
Evaluate cos 2 x dx
0

2003/48 PCE A. –1 B.0 C.1 D.2

 3x 
4 Solution
Evaluate
2
 2 x  1 dx Applying Change of variable format;
2
We let u = 2x
A.42 B.64 C.52 D.46
Differentiating u with respect to x
Solution
du = 2
Applying integration formula
4 dx
= 3x3 – 2x2 + x du = 2dx
3 2 2 Making dx the subject formula
= (4)3 – (4)2 + 4 – [ (2)3 – (2)2 + (2) ] du = dx
= 64 – 16 + 4 – [ 8 – 4 + 2 ] 2
= 64 + 4 – 16 – [ 8 + 2 – 4 ] Substituting for dx and 2x
= 68 – 16 – [ 10 – 4 ]  
du
0  0 cos u 2
2 2
cos 2 x dx
= 52 – 6 i.e 46 (D).
1 
2002/40 PCE =  2 cos u du
2 0
4 1
Evaluate  0
x
dx = 1sin u /2
2 0
A. – 4 B. –2 C.2 D.4 = 1sin 2x /2
Solution 2 0
Displaying the power of x properly, we have; 
= /2 sin2 ( /2 ) – 1/2 sin 2(0)
1
4 1 4 1 = 1/2 sin  – 1/2 sin 0
 0
x
dx   0
1
x2
dx
= 0 (B)
4 2005/4 UME Exercise 27.11
=  x 2 dx
 1


2
0 Evaluate sin 2 x dx
0
integrating, we have
4 A. – 1/2 B. 1 C. –1 D. 0
1/2
= x
1
/2 2005/1 PCE Exercise 27.12
0
 3x 
3

4 Evaluate
2
 2 x dx
1
= 2x1/2
0 A.18 B.17 C.16 D.15
Substituting for upper and lower limits 2009/20 Exercise 27.13
= 2(4)1/2 – 2(0)1/2 = 2( 2 ) – 0 = 4 (D) 2  x 1
3
Evaluate    dx
1999/39 UME 2
1
 x 
2  1 
Evaluate 1 1  x 2  dx A 0.5 B 1.0 C 1.5
1994/42 UME Exercise 27.14
D 2.0

A.41/2 B.17/8 C.11/2 D –11/2


2

1 2 x  1
1
Evaluate dx
Solution
Displaying the power of x properly; A. 32/3 B.4 C.41/3 D.42/3
338
Substitution method II
In some problems, the integral does not fit in directly, in
1 u2
  21
  1
2

such cases, a substitution involving change of the 2 1


variable will help.

2005/11b
 
 u 2
1 2

1
Using the substitution U = x2 – 1 1 2
= – ( 5 – x2)
 x 
2
2
Evaluate
2
 1 2 xdx 1
1
Substituting for the upper & lower limits
Solution = – ( 1 – 2)
As directed, our substitution is = – ( – 1) i.e 1
U = x2 – 1 Next, we change dx to du
du 2004/ 11b
 2x
dx Using the Substitution U = x2 + x +4,
Since we are interested in changing dx 2 x 1
du = 2x dx find  x2  x  4
dx
Making dx subject of formula
du Solution
= dx As directed our substitution is
2x
U = x2 + x +4,
  
2
x 2  1 2 xdx =  u 2 2 x du
2 1 du
1 1 2x dx = 2x +1
2x will cancel out
2 1
Next we make dx the subject of formula
=  1
u 2 du
dx =
du
3 2x  1
u2 2
= 2 x  1 du 
2 x 1
1
 
3
2 Thus dx =  
U 2  2x  1 
1

  x2  x  4
3

2 x 2 1 2 2
  12
3 1 = u du
Substituting for the upper & lower limits values 1

 
1
u 2

 k = 2 x  x 4 2 k
2
23 20  =
3 3
2 2
1
=  2
3 3 2002/11b Further example
2 27 23 3 Using the substitution u2 = (4 – x2 )
= = i.e 2 3 1 x
3 3 Evaluate  0
4  x2
dx

2006 /11 Solution


Using the substitution U = 5 – x2 Using the substitution u2 = (4 – x2 )
Evaluate 
2 x
dx It follows that u = 4  x 2  
1
5  x2 Now u2 = 4 – x2
Solution Applying implicit function differential
U = 5 – x2 Next, we change dx to du du
du 2u   2x
dx
dx = – 2x Make dx the subject of formula
du = –2xdx 2u du
Make dx the subject of formula = dx
 2x
du
– = dx
2x 1 x 1 x  2u du 
2 x 2 x  du 
Thus,  0
4  x2
dx   2

u   2x 

Thus,  1
5  x2
dx =  1 
 
u 2  2x 
1

1
=– du
0
x will cancel out
1
1  12 =– u
2 
= – u du 0

  1
1

= – 4 x2 2

339
Substituting for the upper & lower limits values  1  x  2
5
1 

 3  2  1  x  2 
3
=–  2
 5  0
 
=2– 3
 32 1 
= 2  2   1
1998/ 9a (Nov)  5 5 
Using the Substitution u = (1 + x2), 2( 26 )
5x = i.e 102/5
find  dx 5
1  x 
3
2 2 1994/11a (Nov)
6 x3

Solution dx
Evaluate
As directed our substitution is 0
2x  4
U = 1+ x2
du Solution
dx = 2x Let U = 2x  4
du = 2xdx u2  4
Make dx the subject of formula 2
U = 2x + 4 and u – 4 = 2x then
2
=x
du 2
= dx Applying implicit differential
2x
2udu
5x 5 x du 2
Thus  1  x  2
3
2
dx   3
u 2 2x
dx
2udu = 2dx
x will cancel out 2udu
= dx
5 3
2
 u du

= 2

2 udu = dx
5 u 2
1  u2 4
  1 C 3
6 x3 6
2
2  2

 Thus  dx   udu
0
2x  4 0 u
5 2 Applying basic fraction operation to the numerator
  1 C u2  4  6
2 u2 6

2
= udu
5 0 u
 C
1  x 
1
2 2 U will cancel out
6 u2  2
Example INTER FS 1 =  0 2
du
3 x 2
 2x  dx 1 6 2
Evaluate  0
1 x
=
2 0
(u  2)du

Solution 1  u3  6
=   2u 
Let U = 1 x 2 3  0
U2 = 1 + x and x = u2 – 1
1  2 x  4 2 1 
 6
3

2 
Also differentiating U2 = 1 + x by implicit pattern =   2( 2 x  4)  0
du 2  3 
2u 1
dx substituting we have 34/3 i.e 111/3
2udu = dx 1997/2 Exercise 27.15
x 2
 2x  dx = u 2

1  2 u 1
2
 2
 Using the substitution u = 9 – x2 ,
 9  x  xdx
3 3
 0
1 x
 0 u
2udu
evaluate 
3
2
0
U will cancel out
1996/20 (Nov) Exercise 27.16
 u  2u  1  2u 
3
=
4 2 2
 2 2du Using the substitution u = x2 + 1,
0
find  x 2 1 2 xdx
3
 
 2 u  1du
3
4
1 2
  
2 2
 
2 2

3 3 3
0
A x 1 2 B. x 1 2
C. x 1 2

 u5  3
3 3 3x
 2  u 
 5  0 D
1
4x
1
x2  1 2   
E. x x 2 1  12

340
1993/ 20 (Nov) Exercise 27.17 x2
Using the substitution U = x  3x  9 , 2 Example TS I  9  x2
dx

 x  3x  9 2 x  3dx
7
find 2 Solution
x2 x2
A
1 2
 6
x  5x  9  k  B It is obvious that  9  x2
dx =  dx
6 32  x 2

7 x 2  3x  9  2 x  3  k 6
Thus, we let x = 3 sin
It simply mean in trig ratio that
C 8 x  5 x  9   k D 8 x  2 x  3  k
6 8
2 2
 5x  9 x
Sin  = with diagram as:
E
1 2
8
 8
x  3x  9  k  3

And by pythagoras rule


3 becomes 3
2007/48 Neco Exercise 27.18 x x
3dx
Evaluate  4 x  1 3
9 - x2
3 1 Yes, x = 3 sin 
A. C B. C
84 x  1 84 x  1
2 2 dx
= 3cos 
5 d
3 C dx = 3cos d
C C D. 84 x  12
84 x  1
2
Also From the trig ratio triangle above
9  x 2 can be gotten by cos
6
E C 9  x2
84 x  1
2
Thus, cos = Or 9  x 2 = 3cos
3
2010/11b Exercise 27.19 x2 9 sin 2 
Using the substitution, u = x + 2, Thus  dx =  3 cos d
x 1 2 9  x2 3 cos
evaluate  dx 1
1 ( x  2) 4  9  sin 2  d  Recall that sin2 is 1  cos 2 
2
9
1  cos 2  d
2
2005/11b(i) Neco Exercise 27.20 
Find  1  x dx
9 1 
   sin 2sin    C
2005/11b(ii) Exercise 27.21 2 2 
9 9
By differentiating x 1  x with respect to x,    sin 2  C Recall that sin2 is 2sin cos
2 4
2 x
evaluate  0
1 x
dx =
9
2
9
  2 sin  cos   C
4
Now, we can substitute for 
9 x 9 x 9 x
Trigonometric substitution  sin 1   2   C
The integration of certain algebraic expressions 2 3 4 3 3
is simplified by introducing suitable trigonometric 9 x 1
= sin  2  x 9  x 2  C
function; if the integral involved are: 2 3 2

Example TS 1I
TYPE I a2  x2 , we let x = a sin
Evaluate  4 x 2  4 x  17  2 dx
1

Solution
TYPE I x2  a2 , we let x = a sec
 4 x  dx

 12
2
 4 x  17 dx =
4 x  4 x  17
2
TYPE III a2  x2 , we let x = a tan 2 2
Now 4x + 4x + 17 = 4x + 1 + 16
= (2x + 1)2 + 16
= (2x+ 1)2 + 42

341
Since it is of the form a2 + x2 ; Changing variables
We let 2x + 1 = 4 tan 1
3x = sec ; Thus x = sec
It simply means in trig ratio that 3
2x  1
tan  = with diagram as: 3
dx
= sec tan
4 d
1
dx = sec  tan  d
2x+1 3
9 x 2 1 = tan from our trig ratio triangle
4 Substituting
And by Pythagoras rule the hypotenuse is
9x 2  1 2 sec 2  1
 2 x  1  4 2 2
 x
dx =  13 sec ( 3 sec tan  d )
 4 tan  2 16   tan 2  d 
 16 tan 2  16 Recall that 1 + tan2  = sec2 

 16 tan 2  1  =  sec
2
 1d
Since addition is commutative tan  +1 is 1+tan  2 2
 sec  d    d
2
=
4 1  tan   2
= tan –  + C

4 sec  
2 = 9 x 2  1  sec 1 3x  C

From 2x + 1 = 4 tan Example TS 1V


dx
 2 x  5
dx
2 = 4sec2 Evaluate
d 2 x  3  8 x  12
dx = 2 sec2 d Solution
dx 2 sec  2
Observe that 8x–12 is 4(2x –3)
Thus  =  4 sec
d dx dx
4 x  4 x  17  2 x  5 
2

1
2 x  3  8 x  12 2 x  5 2 x  3  42 x  3
2
 sec  d Charging variables
1 U = 2x  3 2x + 5 = 2x – 3 + 8
= ln sec   tan   C
2 U2 = 2x – 3 = U2 + 8
1 1 2x  1 2u
du
2
= ln 4 x 2 4 x  17  C dx
2 4 4 2udu = 2dx
udu = dx
Example TS III Substituting
dx
Evaluate 
9x 2  1
dx  2 x  5 2 x  3  4(2 x  3)
x
udu
Solution

We observe that 9x2–1 is (3x)2 – 12
and of the form x2 – a2 thus, we let
 2

u  8 u   4u 2
udu
3x = sec 
It simply mean in trig ratio that u (u  8  4u )
2

3x du
Sec = with diagram as
1  2
u  4u  8
And by pythagoras rule
becomes If the denominator is not factorisable then complete the square
3x 3x du du
9x2 - 1 u 2
 4u  4  4

u  22  2 2
1 1 Of the form x2 + a2 ; thus we let
U + 2 = 2 tan
du
= 2sec2
d
du = 2sec2 d
342
and (u +2)2 + 22 = (2 tan )2 + 22 Theorem C: If m is a positive odd integer or n is a positive
= 4tan2  + 4 even integer then
= 4 (tan2 +1)
= 4 sec2  Next

tan m x sec n x dx can be integrated
Example TI 3
du 2 sec 2 
 u  22  2 2   4 sec 2  d 
Evaluate tan 4 x sec 6 x dx
Solution
1 1 Set u = tanx, then du = sec 2xdx
=  d    C
2
First, change from  to u
2
 
tan 4 x sec 6 x dx = tan 4 x sec 4 x sec 2 x dx


= tan x(tan x  1) 2 sec 2 x dx
4 2
1 1  u  2 
= tan  C
2  2  =  tan 4
x(tan 4 x  2 tan 2 x  1) sec2 x dx
Next, change from U to x 9 7 5
=  (tan 8
x  2 tan 6 x  tan 4 x) du = tan x  2 tan x  tan x  C
1  2x  3  2  9 7 5
 tan 1  C
 Some trig identities to the rescue
2  2  Sin ax Cos bx = 1 [Sin(a–b)x + Sin(a+b)x ]
Trigonometric integrals 2
Theorem A : If either m or n is a positive odd integer then Sin ax Sin bx = 1 [Cos(a–b)x – Cos(a+b)x ]
2
 Sin
m
xCosn x dx Cos ax Cos bx = 1 [Cos(a–b)x + Cos(a+b)x ]
2
can be integrated
All these becomes useful when the variable ‘x’ in the above
Example TI 1 examples has been replaced by its multiple

2
Evaluate Sin 5 xCos3 x dx
Example TI 4
Solution
Cos3x = Cos2x Cosx 
Evaluate sin 7 x sin 3xdx
= (1 – Sin2x) cosx Solution
  Sin 1
2 2
Thus, Sin 5 xCos3 x dx = x(1  Sin 2 x)Cosx dx
 sin 7 x sin 3xdx =  2 cos( 7  3) x  cos( 7  3) x dx
5

 
2 12
= Sin 5 xCosxdx – Sin 5 xCosxdx 1
2
= (cos 4 x  cos 10 x)dx
5 7 5 17
= Sin 5 x  Sin 5 x  C
7 17 = 1  1 sin 4 x  1 sin 10 x   C
24 10 
Theorem B: If m and n are both even integers then
1992/11b(ii)

m n
Sin xCos x dx 

 cos 4 x sin 2 xdx


2
Evaluate
0
can be integrated
The method makes use of identities Solution
1  Cos 2 1  Cos 2 Applying trig identity
Sin2 = ; Cos2 = Sin4x Cos2x = 1 [Sin(4–2)x + Sin(4+2)x ]
2 2 2
Example TI 2 = 1 (sin2x + sin6x )
2

Evaluate Sin 2 3xCos2 3x dx 
1 2
 cos 4 x sin 2 xdx =
2 0
(sin 2 x  sin 6 x)dx
2
Thus,
Solution ( Here  = 3x) 0
1  Cos 6 x 1  Cos 6 x
 Sin 3xCos 3x dx =  2 . 2 dx
2 2
= 1   1 cos 2 x  1 cos 6 x  
2 2 6  0
1
=  (1  Cos 2 6 x)dx =  1  cos 2 x  1 cos 6 x  
4 2 3  0
1 1 1  Cos12 x Substituting for the upper and lower limits
= 1dx –  dx
4 4 2 =  1 cos 2( 2 )  1 cos 6( 2 )   1 (cos 0  1 cos 0)
x 1 x 1
=     Cos12 x dx 4  3   4 3 
4 4 2 8 =  1  cos180  1 cos 540   1 1  1 
x x 1 Sin12 x 4 3  4  3
=    C
4 8 8 12 2
=  1   1  1   1  4  =
x Sin12 x 4 3 4  3 3
=  C
8 96
343
Integration by parts Example IP 2
Each differentiation formula leads to an integration
formula. So consider 
Evaluate e 2 x cos 3 xdx

d(uv) = udv + vdu Solution


Making udv the subject formula U = e2 x dv = cos3xdx
1
udv = d(uv) – vdu du = 2e2x dx V = 3 sin 3 x
Integrating both sides
 udv  uv   vdu e
2x
cos 3 xdx = e 2 x 
sin 3 x 1
3
  sin 3 x(2e 2 x ) dx
3
The idea is that; though the LHS may be complicated
but we collect the part for u and the part for dv quite e2x 2
carefully, making the integral on the RHS to be much easier IP2 = sin 3x   e 2 x sin 3 x dx
3 3
Example: IP 1 We try integration by parts on the last integral.
U = e2 x dv = sin3xdx

Evaluate x ln x dx
1
Solution du = 2e2x dx V =  3 cos 3 x
First possibility: u = x dv = lnx
du = 1dx v = ??? e
2x
sin 3 xdx =  1 e 2 x cos 3x     1 cos 3x  2e 2 x dx
3  3 
We come to a dead end since  ln x may prove difficult 1 2
  e 2 x cos 3 x   e 2 x cos 3 x dx
Second possibility: u = ln x dv = xdx 3 3
x2 e2x 2 1 2 
1 Thus IP2  sin 3 x   e 2 x cos 3 x  IP2 
du = dx v= 3 3 3 3 
x 2
x2 x2 1 e2x 2 4
Thus  x ln x dx  ln x    dx IP2 = sin 3x  e 2 x cos 3x  IP2
2 2 x 3 9 9
2x
x 2
1 4 e 2
= ln x   xdx IP 2 + IP 2 = sin 3x  e 2 x cos 3x
2 2 9 3 9
2 2x
x 1 x2 13 e 2
= ln x    C IP 2 = sin 3 x  e 2 x cos 3 x
2 2 2 9 3 9
x 2
x2 9 e 2x
2 
= ln x  C Therefore IP 2 =  sin 3 x  e 2 x cos 3 x 
2 4 13  3 9 
The bottom line here is that; pick the side that you can e2x
= (3sin3x + 2cos3x) + C
differentiate with ease for differentiation i.e U and the 13
other side for integration i.e dv
Example IP3
 sec xdx i.e  sec x sec 2 xdx
3
1992/11 a (Nov) Evaluate

Find x 2 log x dx Solution
Solution Let U = secx dv = sec2xdx
u = logx dv = xdx du = secx tanx dx V = tan x
3
 
Thus, sec xdx = sec x tan x  tan x sec x tan xdx
3
1 x
du = dx v=
x 3  sec x tan x   tan x sec xdx
2

x3 x3 1
Thus,  x log x dx =
2
log x   3  x dx  sec x tan x   sec x  1sec xdx
2

3
 sec x tan x   sec x dx   sec xdx
3
x3 1
 log x   x 2 dx
3 3 IP 3 = sec x tan x – IP 3 +  sec x dx

x3 1 x3 Collect terms in IP 3 together


 log x    C
3 3 3 2IP 3 = sec x tan x  ln (sec x  tan x)  C
1
x3 x3
 log x   C IP 3 = sec x tan x  ln sec x  tan x  C
3 9 2
344
Chapter Twenty eight –6 + 11 = –5B
Partial fractions & integration 5 = –5B
it follows that B = –1
Partial fractions Substituting for A and B values into our initial proposition
Partial fraction is a form of algebraic fractions that 2 x  11 3 1
=  (D)
follows certain rules in resolving it instead of the usual x  2 x  3 x  2 x  3
LCM method of solving fractional problems. The given
fraction, which is single; is resolved into (parts) partial
fractions depending on the following: 2005/5 Neco (Dec)
1 x3
Express in partial fractions.
I. To every linear factor of the form
xa
in the
x  2x  4
denominator; there corresponds a partial fraction of Solution
the form
A x3 A B
Let  
xa x  2x  4 x  2 x  4

II. To every repeated linear factor of the form x  a r Multiplying through by x  2x  4 on both sides
in the denominator, there corresponds partial of the equation
fractions of the form x  3  A x  4  B x  2 ----- *
A1 A2 Ar To eliminate B, we put x = 2 into *
+ 2 + …+
x  a ( x  a) ( x  a) r 2 + 3 = A(2 + 4) + B(2 – 2)
III. To every quadratic factor of the form ax2 + bx +c 5 = A(6) + B(0)
in the denominator, there corresponds a partial 5 = 6A
Ax  B 5
fraction of the form 2 if it can not be A=
ax  bx  c 6
factorized. But if it can be factorized we treat as in Next, to eliminate A, we put x = – 4 into *
I above; as for repeated quadratic factors, treat – 4 + 3 = A (0) + B (– 6)
as in II above –1 = – 6B
1
B =
IV. If the degree of the numerator is greater than or 6
equal that of the dominator, we carry out a division Substituting for A and B values into our initial proposition
until the degree of the numerator is strictly less than
x3 5 1
that of the denominator.  
In order to determine the constants, we introduce in the x  2x  4 6x  2 6x  4
above rules, we may substitute values for x (which will
eliminate some of our propose A, B,C…). However, for Test your answer
greater speed and simplicity this may be combined with Put x = 1 on both sides.
equating coefficients of x, x2 or x3 as the case may be 4 5 1
required to prove 
 15  6 1 65
2004/ 26 Neco (Dec)
2 x  11 4 5 1
Resolve into partial fractions RTP  
x  2x  3 5 6 30
2x 11 11 1  25  1 24 4
A  B  RTP  i.e  QED
x2 x3 x2 x3 30 30 5
3 1 3 1 E 1 3
C  D  
x2 x3 x2 x3 x2 x3 1994/1
Solution 10 x  1 K 3
If   ,find the value of K
Let
2 x  11

A

B x  2x  1 x  2 x  1
x  2x  3 x  2 x  3 A–3 B1 C3 D7 E 13
Multiplying through by (x – 2) (x + 3) on both sides Solution
of the equation Multiplying through both sides by (x – 2) (x + 1)
2x + 11 = A(x +3) + B(x – 2) ------*** 10x + 1 = K(x + 1) + 3(x – 2)
Put x = 2 into *** to eliminate B Since we are to find K, eliminate 3(x – 2) by
2(2) + 11 = A(2 + 3) + B(0) Putting x = 2
15 = 5A 10(2) +1 = K(3) + 3(0)
A=3 21 = 3K
Next, to eliminate A, put x = – 3 into *** K = 7 (D)
2(–3) + 11 = A(0) + B (–3 – 2)
345
1994/4 a Equating coefficient of x from *
1 –12 = 2A + 3B ------------- (2)
Resolve into partial fractions From (1) A = 5 – B
r r  2 Substitute into (2)
Solution –12 = 2(5–B) + 3B
1 A B – 12 = 10 – 2B + 3B
Let   – 12 = 10 + B
r r  2 r r  2 – 22 = B
Multiplying through both sides by r(r + 2) Thus A = 5– (–22)
1 = A (r +2) + B r ------** = 27
To eliminate B, we put r = 0 into ** Substituting for A and B value into our initial proposition
1 = A (0 +2) + B (0)
5  12 x 27 22
1 = 2A  
A=
1  2

6 x  5 x  1 3x  1 2 x  1
2 You can confirm by putting x = 1 on both sides; you will get
To eliminate A, we put r = – 2 into ** 7 7
 
1 = A (– 2 + 2) + B (– 2 ) 12 12
1 = – 2B 2000/10b (Nov)
1
B= – 2  4x
2
Substituting for A and B value into our initial proposition
Resolve into partial fractions

x x2  x  2 
1 1 1 Solution
 
r r  2 2r 2r  2 Factorizing x 2  x  2  ( x  2) ( x  1)
2  4x 2  4x

2000/50 Neco Exercise 28.1
Find the value of the constant K, if
Thus,
 
x x  2 x  2 xx  2x  1
2

2x  3 K R 2  4x A B C
  Let   
x 2  5x  6 x  3 x  2 xx  2x  1 x x  2 x  1
A. 6 B. 9 C. 7 D. 3 E. 2 Multiplying through by x(x – 2)(x + 1)
2 – 4x = A(x – 2)(x + 1) + Bx(x + 1) + Cx(x – 2)__**
1997/ 11 Exercise 28.2 To eliminate B & C ; Put x = 0 into **
Given that 2 = – 2A
4  3x  2 x 2 p Q R Thus, A = –1
  
1  2 x 1  x 1  x  1  2 x 1  x 1  x To eliminate C & A ; Put x = 2 into **
Where P, Q and R are constants, find the value of R. 2 – 8 = 6B
Thus, B = – 1
1 1
A 1 B  C D1 E 2
2 2 To ensure we get C; Put x = –2 into **
2 + 8 = A(–4)(–1) + B(–2)(–1) + C(–2) (–4)
Cases with quadratic denominator that 10 = 4A + 2B + 8C
can be factorized But A is –1 and B is –1 Thus,
10 = –4 – 2 + 8C
2005/ 11 a (Nov) 16 = 8C
C=2
5  12 x Substituting for A, B& C value into our initial proposition
Express in partial fractions
6 x 2  5x  1 2  4x 1 1 2
   Rearranging
Solution xx  2x  1 x x  2 x 1
Factorizing the denominator
2 1 1
5  12 x 5  12 x   
 x 1 x  2 x
6 x 2  5 x  1 3x  12 x  1 2000/1 Neco
5  12 x A B 4  3x  2 x 2
 
Let
3x  12 x  1 3x  1 2 x  1
Resolve

1  2 x  1  x 2 
into partial fraction

Multiplying through by 3x  12 x  1 Solution


5 12 x  A2 x  1  B3x  1 _____* The (1– x2 ) in denominator can factorize into (1– x)(1 + x)
Equating constants 4  3x  2 x 2 4  3x  2 x 2
5 = A + B ------------ (1)
Thus
 =

1  2 x  1  x 2 1  2 x 1  x (1  x)
346
4  3x  2 x 2 A B C 2008/11a
  
Let
 
1  2 x  1  x 1  2 x 1  x 1  x
2 2 x 2  5x 1
Express 3 in partial fractions.
Multiplying through by (1–2x)(1– x)(1+ x) x  4 x 2  3x
4+3x+2x2 =A(1–x)(1+x)+B(1–2x)(1+ x)+C(1–2x)(1–x)--** Solution
To eliminate A and C ; Put x = 1 into ** Applying factorizing of polynomial to the denominator
4+3 +2 = A(0) + B(–1)(2) + C(0) We try to make f(x) = 0 by trial and error
9 = –2B f(1) = (1)3 – 4(1)2 + 3(1) i.e 0
Thus x – 1 is a factor x3 – 4x2 + 3x
Thus, B =  9
2 Next, we apply long division to get the other factors
To eliminate A and B ; Put x = –1 into ** x2 – 3x
4 – 3 + 2 = A(0) + B(0) + C(3)(2) x–1 x3 – 4x2 + 3x
3 = 6C; x3 – 1x2
1
Thus, C = 2
–3x2 + 3x
–3x2 + 3x
To get A ; Put x = 0 into ** 0 0
4=A+B+C x – 4x + 3x factorizes into (x – 1)(x2 – 3x) and x2 – 3x
3 2
1
But B is  9 and C is 2 Thus, factorizes into x(x–3).
2
Thus x3 – 4x2 + 3x factorizes into x(x – 1)(x–3).
9 1
4=A– 
2 2 2 x 2  5x 1 2 x 2  5x 1
9 1 =
4 +   A it follows that A = 8 x 3  4 x 2  3x x ( x  1)( x  3)
2 2
Substituting for A,B & C values into our initial proposition 2 x 2  5x 1 A B C
Let 3   
4  3x  2 x 2 8 9 1 x  4 x  3x
2
x x 1 x  3
  
1  2 x 1  x 1  x  1  2 x 21  x  21  x  Multiplying through by x(x – 1)(x–3)
2x2 – 5x + 1 = A(x – 1)(x–3) +Bx(x–3) + Cx(x – 1)----***
1994/15 Counter example To eliminate A and C ; Put x = 1 into ***
6x  P 4 2 2 – 5 + 1 = A(0) +B(–2) + C(0)
Given that  
2 x  7 x  15 x  5 2 x  3
2
–2 = –2B
Find the value of the constant P B=1
A 20 B 12 C 6 D –10 E –22 To eliminate A and B ; Put x = 3 into ***
Solution 2(3)2 – 5(3) + 1 = A(0) +B(0) + C(6)
We apply the same partial fraction format: i.e 4 = 6C
6x  P 4 2 C= 2
  3
2 x  7 x  15 x  5 2 x  3
2
To eliminate B and C ; Put x = 0 into ***
Then multiplying through by (x + 5)(2x – 3) 1 = A(–1)( –3) + B(0) +C(0)
6x + P = 4(2x – 3) – 2(x + 5) 1 = 3A
We equate constants to get P
A= 1
P = –12 –10 3
P = –22 ( E ) Substituting for A,B & C values into our initial proposition
2 x 2  5x 1 1 1 2
1998/18 PCE Counter example =  
If 3+x express in partial fractions is 5 - 4 find b x 3  4 x 2  3x 3x x  1 3( x  3)
(x–1)(x–b) x–b x–1
A. – 2 B. – ½ C. 2 D. 3 PPF/1 Exercise 28.3
Solution 3x 2  11x  4
We apply the same partial fractions format: i.e Express in partial fractions.
x3  4x 2  x  6
3+x = 5 – 4
(x – 1 )(x – b) x–b x–1
Then, multiplying through by (x – 1)(x – b) 2002/38 Neco Exercise 28.4
3 + x = 5(x – 1) – 4 (x – b) Resolve the rational function
Since our target is to find b; 10 x  11
into partial fractions.
we put x = 1 to eliminate 5(x – 1) x2  x  2
3 + 1 = 5(1 – 1) – 4(1 – b) 3 7 3 7
4 = 5 × 0 – 4 + 4b A  B 
x 1 x  2 x 1 x  2
4 = – 4 + 4b
4 + 4 = 4b 7 7 7 3 7 7
8 = 4b Thus b = 2 (C ) C  D  E 
x  2 x 1 x  2 x 1 x  2 x 1
347
2004/5 Neco (Dec) Exercise 28.5 Next, we substitute A value into (2)
3x  4 x  2
2 5A + 10C = –1 becomes
Resolve into partial fractions

x x 2  3x  2   21 
5   + 10C = –1
 25 
1995/20 (Nov) Exercise 28.6 10C = –1 + 21
x3 p Q 5
Given that 2  
x  9 x  18 x  6 x  3 C= 8
Find the values of the constants P and Q 25
Substituting for A,B & C values into our initial proposition
A P = 4, Q = 3 B P = 3, Q = –2
C P = –3, Q = –1 D P = – 2, Q = 3 2x 2  x  2 21 12 8
  
E P = – 4, Q = 2 ( x  2) (1  2 x)
2
25( x  2) 5( x  2) 25(1  2 x)
2

1996/11 Exercise 28.7


PPF/2
3x 2  7 7 P Q 3x 3  3x 2  3x  2
If 3    , find P + Q Resolve into partial fractions
x  2 x 2  8x 8x x  4 x  2 x 3 ( x  1)
31 41 17 51 5 Solution
A B C D E
24 24 8 12 12 3x 3  3x 2  3x  2 A B C D
Let  + 2 + 3 +
x ( x  1)
3
x x x ( x  1)
Cases with quadratic denominator that has
Multiplying through by x3(x +1)
repeated roots 3x3 +3x2 +3x +2 = Ax2(x +1) +Bx(x +1) + C(x+1) +Dx3 ---**
2000/11b To get C we put x = 0 into **
2x  x  2
2 2=C
Resolve into partial fractions To get D we put x = –1 into **
( x  2) 2 (1  2 x)
–3 + 3 –3 + 2 = –D
Solution D=1
We proceed as follows To enable us equate coefficients, let us expand **
2x 2  x  2 A B C 3x3+3x2+3x+2 = (Ax3 +Ax2) +(Bx2 +Bx) + (Cx+C) +Dx3
  
( x  2) (1  2 x) x  2 ( x  2)
2 2
(1  2 x) Equating coefficients of x3 from the expansion
Repeated root (x + 2)2 was broken down as (x + 2) & (x + 2)2 3=A+D
Multiplying through by (x + 2)2(1 – 2x ) But D is 1 3= A+1
2x2–x +2 = A(x +2)(1– 2x) + B(1–2x) + C(x +2)2---** A=2
By observation we eliminate C and A putting x = – 2 into** Equating coefficients of x2 from the expansion
2(– 2)2 – (–2) + 2 = A(0)(5) + B(5) + C(0)2 3=A+B
12 = 5B But A is 2 3=2+B
12 = B B=1
5 Substituting for A,B,C & D values into our initial proposition
To get A and C let us open up RHS of **
2x2– x +2 = A(2 –3x –2x2) + B(1– 2x) + C(x2 + 4x + 4) 3x 3  3x 2  3x  2 2 1 2 1
= + 2 + 3 +
Equating terms in x2 from the expansion x ( x  1)
3
x x x ( x  1)
2 = –2A + C ---------( 1 )
Equating constants from the expansion 2004/11 Neco Exercise 28.8
2 = 2A + B + 4C 2x  1
But B is 12 (a) Express in the form
5 x  1x  22
2 = 2A + 12  4C A B C
5
  Where A, B and C are constants
2–
12
 2 A  4C x 1 x  2 x  22
5
2
  2 A  4C
5
–2 = 10A + 20C
–1 = 5A + 10C -------(2)
Multiply ( 1 ) by 10 and subtract (2)
–20A + 10C = 20
–(5A + 10C = –1)
–25A = 21
A =  21
25
348
Cases with quadratic denominator that C is not among the constants because x(Bx + C) = Bx2 + Cx
cannot be factorized Equating coefficient of x2
2005/6 Neco 0=A+B
3 0= 3 +B
Resolve

x  1 x  5x  2
into partial fractions
2
 Thus B = –
2
3
Solution 2
You will observe that (x2 – 5x + 2) cannot be factorized Equating coefficient of x
1 = 2A + C
3 A Bx  C
  2 1 = 2( 3 ) + C
Let

x  1 x  5x  2 x  1 x  5x  2
2
 2

Multiplying through by (x – 1)(x2 – 5x + 2) 1 = 3 + C Thus C = –2


3 = A(x2 – 5x + 2) + (Bx + C)( x – 1) Care must be taken to fix B value into our given proposition
3 = A(x2 – 5x + 2) + Bx(x – 1) + C( x – 1)------** x3 3  32 x   4
 2
To get A, we put x = 1 into** 
x x2  2
=
2x 
x 2
3 = A (–2) + B(0) + C(0)
3 = –2A i.e PPF/3 Counter example

A= –
3 x 2  4x  1
Resolve into partial fractions
2 x 3  3x 2  4 x  2
To eliminate B, we put x = 0 into** Solution
3 = 2A – C A critical look at the denominator shows that
But A is – 3 3 = 2 ( –3 ) – C f(-1) = -1 + 3 - 4 + 2 i.e 0
2 2
x +1 is a factor
C =–6
x2 + 2x +2
Equating coefficient of x2 x+1 x3 + 3x2 + 4x + 2
Note that Bx(x – 1) = Bx2 – Bx at ** x3 + x2
0 = A+B 2x2 + 4x
0 = –3+ B 2x2 + 2x
2
2x + 2
B= 3
2 2x + 2
3 0 0
x6 x3 +3x2 +4x +2 factorizes into (x +1)(x2 + 2x +2)
3 3 2
  You will observe that (x2 + 2x +2) cannot be factorized
Thus
 
x  1 x 2  5 x  2 2x  1 x 2  5 x  2 x 2  4x  1 A Bx  C
3x  12 3 Let 3   2
 x  3x  4 x  2 x  1 x  2 x  2
2
=

2 x  5x  2
2

2x  1 Multiplying through by (x +1)(x2 + 2x +2)
x2 + 4x + 1 = A(x2 + 2x +2) + (Bx + C)(x +1)
2007/36 Neco x2 + 4x + 1 = A(x2 + 2x +2) + (Bx2 + Bx) + (Cx +C)---**
x3
To eliminate B and C we put x = –1 into **
Express

x x2  2 
into partial fractions
1– 4 +1 = A(1– 2 + 2) + B(0) + C(0)
3 2  3x 3 2  3x –2=A
 2  2
2 x 2x  2 2 x 2x  2 
A B To eliminate B, we put x = 0 into **
1 = A(2) + C(1)
3 2  3x 3 2  3x 1 = 2A + C
 
2 x 2x  2 2 x 2x 2  2
C 2
D 1 = 2(–2) + C Thus C = 5
Equating coefficient of x2 at **
 3 2  3x
 1=A+B
E
2x 2 x 2  2   1 = –2 + B
Solution B=3
You will observe that (x2 + 2) cannot be factorized x 2  4x  1 2 3x  5
x3 A Bx  C Thus 3 = –  2
  2 x  3x  4 x  2
2
x  1 x  2x  2
Let

x x 2 x x 2
2

Multiplying through by x(x2 + 2) 1992 /11 (Nov) Exercise 28.9
x + 3 = A(x2 + 2) + x(Bx + C) --------*** 3x  4 x
2
P Qx  R
  2
Equating constant
3
If
 
x  2 x  1 x  2 x  1
2 , find P + Q + R

3 = 2A; A = A–1 B2 C4 D5 E7
2
349
Cases with degree of numerator greater 5
Next, we resolve into partial fraction
than or equal to the denominator 4  x2
2005/10a Neco (Dec) 5 5 5
 i.e
x3  x 2  4x 4  x2 2 2  x 2 2  x 2  x 
Resolve into partial fraction
x2  x  2 5 A B
Let  
Solution 2  x 2  x  2  x 2  x
The highest degree of x in the numerator is higher than
Multiplying through by (2 – x)(2 + x)
that of the denominator; which is not allowed;
5 = A(2 + x) + B(2 – x) -----**
Applying long division
x To eliminate B; put x = 2 into**
x2 + x–2 x3 + x2 + 4x 5 = 4A + B(0)
– ( x3 + x2 –2x ) 5 = 4A;
6x A= 5
4
x3  x 2  4x 6x To eliminate A; put x = –2 into**
Thus  x 2 5 = A(0) + 4B
x  x2
2
x  x2 5 = 4B;
6x
Next, we resolve into partial fractions B= 5
x  x2
2
4

6x 5 5 5
6x
 A B Thus,  
x  x2
2
 
x  2x  1 x  2 x  1 2  x 2  x  42  x  42  x 
Multiply through by (x + 2)(x – 1) x2 1 5 5
Therefore   1 
6x = A(x – 1) + B(x + 2) ------*** 4 x 2
42  x  42  x 
To eliminate A; put x = 1 into *** 5
6 = A(0) + B(3) 1
A B 1 5 1 4 1
6 = 3B and  4 =  =  ie D 
B=2 C 5 4 4 4 5 5
To eliminate B ; put x = – 2 4
6 (–2) = –3A + B(0)
–12 = –3A
A= 4
6x 4 2
Hence = 
x  2x  1 x  2 x  1
Confirm by putting x = 2 and you get 3 = 1+2
x3  x 2  4x 4 2
Therefore = x 
x  x2
2
x  2 x 1

2007/37 Neco
x2 1 B C
If  A  Where A, B and C are
4 x 2
2 x 2 x
A B
constants, calculate
C
9 4 5 1
A. B. C. D. E.  1
5 5 16 5
Solution
The degree of x in numerator is equal to that of the
denominator; which is not allowed.
Applying long division
–1
– x2 + 4 x2 + 1
– ( x2 – 4 )
5
x2 1 5
Thus   1 
4  x2 4  x2
350
Integration by partial fraction x2 3 1
The aim of studying partial fraction includes its usage in Thus  
solving integration problems. Some of the problems we x  1x  2 2x  1 2x  1
treated were actually combined with integration problems x3  2  3 1 
The basic principle of integration involved here is that  x 2  1 dx    x  2x  1  2x  1 dx
of fraction as shown below:
1 3 dx 1 dx
i  x dx = lnx +c =  xdx  2  x  1  2  x  1
3 1
and  dx = 3 dx = 3lnx +c x2 3 1
x x =  ln x  1  ln x  1  c
1 2 2 2
ii  x  1 dx = ln(x+1) +c 1992/11b(i) Nov
and 
5
dx = 5
1 x2  2
3
x 1 x 1
dx = 5ln(x+1) +c Evaluate  dx
1 x4

2005/10a Neco (Dec) Solution


The degree of x in numerator is higher than that of the
x  x  4x
3 2
denominator; which is not allowed.
Resolve into partial fractions
x2  x  2 Applying long division
x–4
x3  x 2  4x x+4 x2 – 2
and hence integrate
x2  x  2 – ( x2 + 4x )
Solution – 4x – 2
Having solved the partial fractions aspect on page350 –( – 4x – 16)
we proceed to the integration side 14
Using the result of the partial fractions: x2  2 14
x3  x 2  4x = 4 2 Thus  x4
x  x4 x4
x  x2
2
x  2 x 1
14
is a resolved partial fraction, since it has no x in the
x 3  x 2  4x 4 2 x4
 x 2  x  2 dx   ( x  x  2  x  1)dx numerator and the denominator is in linear form
x2 3 x2  2
 1 x  4 dx = 14
3
=
2
+ 4ln(x +2) + 2ln(x – 1) +C  1
(x  4 
x4
) dx

x3  2 x2
 4 x  14 ln x  4
3
2003/7(ii) Neco Evaluate  2 dx =
x 1 2 1

9 1 
=  12  14 ln 7     4  14 ln 5
Solution
The degree of x in numerator is higher than that of the
denominator; which is not allowed. 2 2 
Applying long division
15  7 
x =   27 .2427 –   22.5321
x2 – 1 x3 + 2 2  2 
– ( x3 – x ) 7 15
x+2 =  27 .2427  22 .5321
2 2
x3  2 x2 = – 4 + 4.7106 = 0 .7106
Thus  x 2
x 1
2
x 1 1996/10 Exercise 28.10
x2 2x  1
Next, we resolve into partial fraction (i)Resolve into partial fractions
x2 1 x  1x  22
x2 x2 A B 2x  1
   1
Let
x  1 x  1x  1 x  1 x  1
2 (ii)Use your result in (i) to evaluate  0 ( x  1) ( x  2) 2
dx
Multiply through by (x – 1) (x + 1)
1997/15 Exercise 28.11
x + 2 = A(x + 1) + B(x – 1) ------***
2  2 
To eliminate B; put x = 1 into ***
3 = 2A it follows that A = 3
Evaluate  1  x  1 dx
 1 
2
To eliminate A; put x = –1 into *** 1994/17 (Nov) Exercise 28.12
1 = –2B it follows that B = – 1 x 2 1 x
2
Given that 2
x  3x  2

x  2

x 1
, find x 2
 3x  2
dx

351
Chapter Twenty nine Sketching
y
Area Under a Curve, volume & trapezium rule 4
y = 4 (line of symmetry)

Area under a Curve


y y

)
Case II x

f(x
0 2 4 6

y= A
x
-12
B
x (c) Area of bdd region =
a b Area = A - B 2 6
The diagram above shows a curve of a function f(x).   (8 x  x 2  12 )dx +  (8 x  x 2  12 )dx
0 2
The area under the curve along the x-axis is denoted by:
8x 2 x 3
f x  dx  F x 
b 2

2
= F(b) – F(a) First   (8 x  x  12 ) dx = –   12 x
b 2
a
a 0 2 3 0
Also F is the integral result of f(x). The unit of Substituting for upper & lower limits
measurement is square unit. You will observe that this
is an application of definite integrals. Case II is special = negative of 4(2 ) –
2  –12(2) – 40   0  12(0)
2
3
2
3

3  
Under this subtopic curve sketching is necessary  3 
 32 
Two types of problems exist in areas under a curve: =    = 32 Square unit
1. Case where the value of ordinates are not given  3 3
(No restriction), here the sketched diagram is our guide
6 8x 2 x 3 6
2. Case where the value of ordinates are given Secondly  (8 x  x  12 )dx =   12 x
2

(there is restriction), here the ordinate given is our guide 2 2 3 2

along with the sketched diagram Substituting for upper & lower limits

1996/11 = 4(6 ) –
6  – 12 (6) – 42   2  12(2)
2
3
2
3

3  
(a) Sketch the curve y = 8x – x2 –12  3 
(b) Draw the line of symmetry of the curve
216  8 
(c) Find the area of the finite region bounded by the = 144 –  72  16   24
curve and the x - axis 3  3 
Solution
 8
(a) y = 8x – x2 –12 = 0 –  8  
dy 3  
= 8 – 2x
dx 32
= Square unit
dy 3
At turning point =0
dx 32 32 64
Thus, Area of bdd region = + = Square unit
Thus, 8 – 2x = 0 3 3 3
2x = 8 ; x = 4 1999/10 (Nov)
Value of y at the turning point Calculate the area of the finite region bounded by the
y = 8(4) – 42 – 12 curve y = x2 – 7x + 10 and the x – axis
= 32 – 16 –12 i.e 4 Solution
Next we determine whether it is a max or min point y = x2 – 7x + 10
d2y dy
= 2x – 7
= – 2  0 Hence a maximum point dx
dx 2 dy
Intercepts ( x – axis ) here y is 0 At turning point =0
8x – x2 –12 = 0 is same as dx
x2 – 8x +12 = 0 Thus, 2x – 7 = 0
x – 6x – 2x +12 = 0
2 2x = 7 ; x = 3.5
x(x– 6) – 2(x– 6) = 0 Value of y at the turning point
(x– 6)(x– 2) = 0 y = (3.5)2 – 7(3.5) +10
x– 6 = 0 or x – 2 = 0 Thus, x = 6 or 2 = 12.25 – 24.5 +10 i.e – 2.25
Intercept (y – axis) here x is 0 Next we determine whether it is a max or min point
y = 8(0) – (0)2 – 12 i.e – 12 d2y
= 2  0 Hence a minimum point
dx 2
352
The intercept (x-axis) and here y is 0 2000/7 (Nov)
x2 – 7x + 10 = 0 Calculate, in square units, the area of the finite region
x – 5x – 2x + 10 = 0
2
bounded by the cure y = x2 + 2x –3 and the x-axis
x(x – 5) – 2(x – 5) = 0 14 20 32 35
A B. C. D.
(x – 5)(x – 2) = 0 3 3 3 3
x = 5 or 2 Solution
Intercept (y – axis) here x is 0 y = x2 + 2x – 3
y = (0)2 – 7(0) +10 dy
= +10 = 2x +2
Sketching dx
y dy
10 At turning point =0
dx
Thus, 2x +2 = 0
2x = – 2 ; x = –1
Value of y at the turning point
y = (–1)2 +2(–1) – 3
0 1 2 3 4 5
x = 1– 2 – 3 i.e – 4
-1 Next we determine whether it is a max or min point
-2
-2.25
-3
d2y
= 2  0 Hence a minimum point
Area of bdd region = dx 2
2 5 the intercept (x-axis) and here y is 0
 0
( x 2  7 x  10 )dx –  2
( x 2  7 x  10 )dx
x2 + 2x – 3 = 0
2 x3 7x2 2 x + 3x – x – 3 = 0
2

First  0
( x 2  7 x  10 )dx = 
3

2
10 x
0
x(x + 3) – 1(x + 3) = 0
(x + 3)(x – 1) = 0
Substituting for upper & lower limits x = –3 or 1
23 7 2 2
 
   03 7 0 2
 10 (2)   

 10 (0) 
  Intercept (y – axis) here x is 0
3 2 y = (0)2 +2(0) – 3
3 2  =–3
8 28 Sketching
   20 y
3 2
26
= Square unit
3
x
-3 -2 -1 0 1
5 x3 7x2 5
Secondly  ( x  7 x  10 )dx =  10 x
2
2 3 2 2 -3
Substituting for upper & lower limits -4

53 7 5 2
 
   23 7 2 2
 10 (5)   

 10 (2) 
  1
3 2 3 2 
Area of bdd region = –  3
( x 2  2 x  3) dx
125 175  8 28  1 x3 x2 1

3

2
 50     20 
3 2 
But  3
( x 2  2 x  3) dx =
3

2
 3x
3
125 8 175 Substituting for upper & lower limits
    50  20  14
3 3 2 1   33 
  3  3  3
2
250  16  525  264 = 1 3  
 3  3 
6
5 1
= square unit   2   9  9  9
6 3
Thus, Area of bdd region 32
2 5

=  ( x  7 x  10 )dx – 3
 ( x 2  7 x  10 )dx
2
0 2 1

=
26

5
Thus Area of bdd region = –  3
( x 2  2 x  3) dx
3 6 32
= square units (C)
47 3
= square unit
6

353
2001 /11 Neco (c) Area enclosed by the curve and x-axis
y
Sketch the curve y = (x +2)( x – 2)( x – 4) 16
(b) Find the maximum and the minimum value of the
function to the nearest wholes number
(c) Calculate area enclosed by the curve and the x-axis. -2 0 2 4 x

Solution
First, we expand to know the function proper.
(x +2)( x – 2)( x – 4) = (x - 4) [(x +2)(x - 2)] 2
 4( x 3  4 x 2  4 x  16)dx
2    
  2
3 2
= (x - 4)(x2 - 2x + 2x - 4) = ( x 4 x 4 x 16) dx

= (x - 4) (x2 - 4) 4
4 3 2
x 4 4x3 4x 2
= x - 4 x - 4 x + 16x
2
= x(x2 - 4) - 4(x2 - 4 )     16 x
= x3 – 4x2 – 4x + 16 4 3 2 2 4 3 2 2
i.e y = x – 4x – 4x + 16
3 2
Readers to substitute and complete
dy
Next, we sketch the curve at max or min 0 2005/11b
dx
dy ( i ) Sketch the curve y = x2 – 4
 3x 2  8 x  4  0 ( ii ) find the area of finite region enclosed by the curve
dx y = x2 – 4, the x-axis and the ordinates x = –2, x =3
By quadratic general formula Solution
x =  (8)  (8)  4  3  (4)
2
First, we sketch the curve
23 y = x2 – 4
= 8  112 = 3.1 or – 0.4 dy
At max or min 0
6 dx
Next, we determine which of the turning points is max or min. dy
 2x  0
d 2y dx
= 6x – 8 x=0
dx 2
Value of y at turning point
When x = 3.1
y = (0)2 – 4
6x – 8 = 10.6 > 0 minimum point = –4
When x = – 0.4 To determine max or min point
6x – 8 = –10.4 < 0 maximum point
d 2y
Intercept at x – axis here y = 0  2  0 min point
0 = x3 – 4x2 – 4x + 16 dx 2
i.e (x +2)( x – 2)( x – 4) = 0 Intercept at x – axis , here y is 0
x = –2 or 2 or 4 0 = x2 – 4
Thus ,the curve cuts the x–axis at -2, 2 and 4  x2 –22 = 0 thus (x–2)(x + 2) = 0 ; x = 2 or –2
Intercept at y – axis, here x = 0
y = 03 – 4(0)2 – 4(0) +16 Intercept at y – axis, here x is 0
y =16 y = (0)2 – 4
The curve cut the y – axis at y =16. =–4
Sketching y Sketching
y
16

-2 0 2 x
-2 0 2 4 x

-4
(b) Maximum value of the function
value of x is -0.4 The area of bounded region here is between – 2 and 2 but the required area
of bounded region is between – 2 and 3. Hence we extend our graph to the
Max value y = (-0.4 )3 – 4(-0.4)2 – 4(-0.4) + 16 point where x = 3 by simple substitution of values.
= –0.064 + 0.64 + 1.6 + 16 When x = 3, y = 32- 4 i.e. 5
= 17.6 – 0.704 Our new sketch is
y
= 16.896 5
≈ 17
Minimum value of the function
value of x is 3.1 -2 0 2 3 x

Min value of y = (3.1)3 – 4(3.1)2 – 4(3.1) + 16


= 29.791– 38.44 – 12.4+16 -4
= 45.791 – 50.84
 x 
3
2
 4 dx
= –5.049 Area of bounded region = –
 x  +
2
2
 4 dx 2
≈ –5 2

354
 x   x 
3 2 Example AUC 1
=
2
 4 dx – 2
 4 dx
2 2 Find the area of finite region enclosed by the curve y = 2 x
 x3 2  and the lines x = 3 and x = 0
= x  4x 3 –
3
  4x  4 3 B 2 3 16 3 D 18 3
3 2 3 2 A C
= 33  23   23  (2) 3  Solution
 4(3)    4(2)    4(2)    4(2) 
3  3   3  3  y= 2 x
= 27  12   8  8   8  8    8  8  y = 2x ½
3  3   3  3  
  dy
= x –½
= –3 + 16    16  16  = 13 square unit dx
3  3 3  At max or min point dy/dx is zero
1 ½=0
2003/25 x
Calculate, in square units, the area of the finite region x=0
bounded by the curve y = 1 + x – 2x2, the x-axis and the Value of y turning point
coordinates x = 0 and x = 1 y = 2 (0) i.e 0
A 5/ 6 B1 C 13/6 D4 we are heading to a dead end, hence we try sketching the
Solution curve using values within the given range x = 0 to x = 3
Fist, we sketch y = 1 + x – 2 x2 y= 2 x
dy When x = 0, y = 0
At max or min =0,
dx When x = 1, y = 2
dy When x =2, y = 2.8
 1  4x  0 When x = 3, y = 3.5
dx y
x = 1/4 i.e 0.25
Value of y at turning point
y = 1 + (0.25) – 2(0.25)2
= 1 + 0.25 – 0.125 = 1.375 ≈ 1.4
To determine max or min point 0 3 x
2
dy From our sketching the curve lies on the upper part of x-axis
= - 4 < 0 max point along within our given range of x = 0 to x = 3
dx 2 1
3
 2x
Intercept at x–axis, here y is 0 2
0 = 1 + x – 2x2 Thus, area of bdd region = dx
0
 1 + 2x –x –2x2 = 0 3 3
1(1 + 2x ) –x(1 + 2x) = 0 2x 2
=
(1 – x )(1 + 2x ) = 0 3 0
2
x = 1 or – ½
3
Intercept y – axis , here x is 0 4 32
= x
y = 1 + 0 – 2(0)2 3 0
=1 3
Sketching 4(3) 2
4 3
y =  (0) 2
3 3
1
4 43 3
 4 3 (A).
1
= ( 27 ) 2 =
3 3
-1 -0.5 0 0.25 0.5 1
x Example AUC 2
Find the area bounded by the curve y = x2, the
The area of bounded region here is between – 0.5 and 1
x-axis and the ordinates x = – 2 and x = 2
but the required area of bounded region is between 0
Solution
and 1.
First, we sketch the curve y = x2
Hence, we don’t need to extend graph.
dy
1 At max or min =0
Area of bounded region =  0
(1  x  2 x 2 )dx dx
dy  2x = 0
x 2 2x3 1
=x+  dx
x=0
2 3 0
Value of y at turning point
= 1 + 1/ 2 – 2/ 3
y = 2(0) i.e 0
= 5/6 square unit We are heading to a dead end, hence we try sketching the curve
using values within the given range x = -2 to x = 2
355
y = x2 Area bounded between two Curves
When x = – 2 , y = (– 2 )2 i.e. 4
A
When x = –1 , y = (– 1 )2 i.e.1 y
f2(x) Cubic function

When x = 0 y = (0)2 i.e. 0


When x = 1 y = 12 i.e. 1
When x = 2 y = 22 i.e. 4 h
x
Sketching a b
y

dx Cubic function
f1(x)

y
x B
-2 -1 0 1 2

0 2
Qu
ad
Area of bounded region =  2
x 2 dx   0
x 2 dx ratic
fun
cti
3 0 2 2 on
x x
=  f2(x)
3 2 3 0
a b
0  (2) 
2
23 0 x
=    
3  3  3 3 Linear function
f1(x)
= 8 + 8
3 3 Area between two curves can be treated under:
= 16 ≈ 5.3 square units ( i ) a case where the sketching can be done by 1st and 2nd
3
differential and solving for maximum or minimum points
2010/15 Neco Exercise 29.1 ( ii ) a case where the sketching can not be done by 1st and
Find the area bounded by the curve y = 2x2 + 5, 2nd differential since all the result will play around zero
the x-axis the ordinates x = 1 , x = 3 hence we sketch by substitution of values (simple range )
A 5 2 square unit B 27 1 square unit into the two equations.
3 3
C 28 1 square unit D 33 square unit E square unit
 f ( x)  f1 ( x)  dx
2 b
3
38
3 Area between two curves A  2
a

The order of substituting equation f1(x) and f2(x) is base on


Exercise 29.2
the uppermost function from the sketch or the presented
Find the area bounded by the curve y = 3x2 and the
(diagram)
x-axis and the ordinates x = 0 and x = 3

2007/2 Neco Exercise 29.3


Find the area bounded by the curve y = x2 – 5x , the
x-axis and the co-ordinates x = 0 and x = 8 to the
nearest square unit
A 4 B 6 C 8 D 11 E 13 2000/34 UME
y

2002/35 (Nov) Exercise 29.4 y = 16


Calculate in square units, the area of the finite region
bounded by the x - axis, y – axis and the
line 2y = x + 6
A3 B6 C 9 D 12
x
(4,0)
1995/3 (Nov) Exercise 29.5 (0,0)

Calculate in square units, the area of the finite region If the diagram above is the graph of y = x2, the shaded area is
x y A. 64square units B.128square units C.64square units D.32 square units
bounded by the lines   1 , y = 0 and x = 0 3 3
4 5
Solution
A5 B10 C 15 D 18 E 20 This is a typical question of area between two curves: Shine
your eyes and you will see y = 16 (linear equation) and the
Exercise 29.6 curve equation which was mentioned y = x2.
Find the area of the finite region bounded by the curve Of course, if you err and think it is area under a curve, the
y = x3 + x2 – 2x and the x-axis answer is there too as 64/3.
356
Here our limits were given in a ,b co-ordinate forms. (iii) Co-ordinates of their points of intersection
x = 0 and x = 4; 5x – x2 = x + 3
Recall that the two functions are to be subtracted. x – 4x + 3 = 0
2

The graph indicates f2 (x) to be y = 16 x2 – 3x– x + 3 = 0

 16  x dx
4
2 (x – 3) (x – 1) = 0 Thus x = 3 or 1
0 And they are our upper and lower limits as well
= 16x - x3 4 From the sketch the curve is uppermost, hence

 5x  x   x  3 dx
3
3 0 Area of the segment =
2
1
= 16(4) – ( 4 )3 – 16( 0 ) – ( 0 )3
 4 x  x 
3
3 3 =
2
 3 dx
1
= 64 – 64
3 4x 2 x3 3
=   3x
= 192 – 64 2 3 1
3
= 128 square units (B) 4(3) 2 (3) 3 4 1 
=   3(3)     3
3 2 3 2 3 
= 18 – 9 – 9 – ( – 11/3)
2000/11 (Nov) = 11/3square units
a ( i ) Sketch on the same axes the curve y = 5x – x2
and the line y = x + 3
( ii ) Find the co-ordinates of their points of 2001/36 UME
intersection Find the area bounded by the curves y = 4 – x2
b Find the area of the segment cut off from the and y = 2x + 1.
curve by the line. A.201/3sq. units B.20 2/3sq. units
2
Solution C.10 /3sq. units D.101/3sq. units
y = 5x – x2 Solution
dy y = 4 – x2
= 5 – 2x dy = –2x
dx dx
At turning point
dy
=0 At turning point dy = 0 i.e –2x = 0
dx dx
Thus, 5 – 2x = 0 It will lead us to a dead end
5 = 2x Let us substitute using a simple range of – 3 to 3
x = 5/2 i.e 2.5 y = 4 – x2 y = 2x + 1
When x = -3 , y = -5 x = -3 , y = -5
Value of y at the turning point x = -2 , y=0 x= 1, y=3
y = 5(2.5) – (2.5)2 x = -1 , y=3
= 12.5 – 6.25) i.e 6.25 x= 0, y=4
Next we determine whether it is a max or min point x= 1, y=3
x= 2, y= 0
d2y x= 3, y = -5
= – 2  0 Hence a maximum point Sketching
dx 2 y

Intercepts ( x – axis ) here y is 0 4


5x – x2 = 0
x(5 – x ) = 0 1
x = 0 or 5 – x = 0
x = 0 or 5
Intercept (y – axis) here x is 0 -3 -2 0 1 2
x

y = 5(0) – (02) i.e 0


Sketching the curve first then fit in the straight line -5
y y
Curve
6.25 6.25 Curve+ line Here our limits were not given; thus we equate the two
equations and solve the resulting quadratic equation.
4 – x2 = 2x +1
3 4 – x – 2x – 1 = 0
2
or 2x +1 – 4 + x2 = 0
rearranging x2 + 2x – 3 = 0
0 2.5 5
x
0 2.5 5
x 3 – 2x – x = 0
2

Incorporating line y = x + 3 multiply through by minus


We only need any two points to sketch a straight line x2 +2x – 3 = 0
Here let us take When x = 0 , y = 3  When x = 1 , y = 4 Factorising

357
x2 + 3x – x – 3 = 0 27 27
x(x + 3) –1(x + 3) = 0
= 
2 3
(x– 1)(x +3 ) = 0 81  54 27
x = 1 or –3  i.e. 4.5(B)
The smallest value is –3 and that is the lower limit 6 6
Note that from the sketch the curve is uppermost, 2002/ 9
hence A fixed point A(m, n) lies on the curve y = x2 + 1

 4  x  2 x  1dx
1
2 and the line y = x +7
Area bounded = (a) Find: ( i ) the two possible coordinates of A
3
( ii ) the distances between the two locations of A
 4  x 
1
=
2
 2 x  1 dx (b) Sketch the curve and draw the line
3
(c) Find the area of the finite region enclosed by the line and
 3  2 x  x dx
1
=
2 the curve.
3 Solution
= 3x – 2x2 - x3 1
a . i Points of intersection first
2 3 -3 x2 +1 = x + 7
x –x–6=0
2
= 3x – x2 – x3 1
x2 + 2x +3x – 6 = 0
3 -3
x(x + 2) – 3(x + 2) = 0
= 3(1) – (1)2 – ( 1 )3 - 3(-3) – (-3)2 – ( -3 )3 (x + 2)(x – 3) = 0
3 3
x = - 2 or 3
= 3 – 1 – 1 – – 9 – 9 – (–27 ) Coordinate of intersection A
3 3 When x = -2 , y = (-2)2 – (- 2) – 6 i.e. 0
= 2 – 1/3 – (–9 ) When x = 3, y = (3)2 –3 – 6 i.e. 0
= 2 – 1/3 + 9 Thus coordinates of A are (-2, 0) and (3, 0)
= 6 – 1 + 27
3 (ii) Distances between two points of A
= 32/3 i.e102/3 sq units (C)
2000/40 PCE = (3  (2)2  (0  0) 2
Find the area bounded by the curve y = x2 and 25 i.e. 5
=
the straight line y = 3x.
A.22.5 B.4.5 C. – 4.5 D. – 22.5
b. y = x2 +1
Solution
It is obvious y = x2 differential will play around zero. dy
 2x
Let us substitute using a simple range of – 3 to 3 dx
y = x2 y =3x it is obvious ,everything here is about zero .Let us work by
When x = - 3, y = 9 When x = -3, y = -9 substitution of values x = - 2 to 3 say from x = -3 to 3
x = - 2, y = 4 When x = 0, y = 0 y = x2 +1 y=x+7
x = - 1, y =1 When x = - 3, y = 10 When x = 0, y = 7
x = 0, y = 0 x = - 2, y = 5 x = 3, y = 10
x = 1, y = 1 x = - 1, y = 2
x = 2, y = 4 x = 0, y=1
x = 3, y = 9 x = 1, y=2
Sketching x = 2, y=5
y
9 x = 3, y = 10
Sketching
y

4 10

7
1

-3 -2 -1 0 1 2 3 x

Equating the two curves to get limits of integral 1


x2 = 3x -3 -2 -1 0 1 2 3 x

x – 3x = 0
2
From the sketch, uppermost function is the straight line
x(x – 3) = 0
 x  7  ( x 
3
x = 0 or 3 c. Area enclosed =
2
 1) dx
Note that from the sketch the line is uppermost, Thus 2
3 3
Area bounded =  0
(3x  x 2 ) dx =  2
(x – x2 + 6) dx

3x 2 x 3 x 2 x3
=  3
0 =   6 x 32
2 3 2 3
358
3 2 33  (2) 2 (2) 3  3x 2 x 3 3
=   6(3)     6(2) =  0
2 3  2 3  2 3
= 9  9  18   2  8  12 3(3) 2 (3) 3 27 9
2  3  =  = 9 = square units.
= 27  22 2 3 2 2
2 3
Example ABTC 1 Acid test for area between two curves
= 125/6 square units. i.e 20.83 square units 5
Find the area of the figure bounded by the curve y = x
1994 / 9 16
Given the curves, y = 4x – x2 and y = x2 – 2x and the line y = x and between x = -2 and 2
(a) Determine the coordinates of their points of Solution
5
intersection. y=x y= x
(b) Sketch, on the same axes, the two curves. 16
(c) Find the area of the finite region bounded by the two dy = 1 dy = 5 x
4

curves. dx dx 16
Solution At max or min dy/dx = 0 At max or min dy/dx = 0
(a) Their points of intersection:
It is obvious ,everything here is about zero .Let us work by
4x – x2 = x2 –2x
substitution of values x = - 2 to 2
4x – x – x2 + 2x = 0
2
5
4x – 2x2 + 2x = 0 y= x y=x
2x – x2 + x = 0 16
3x – x2 = 0 x = - 2, y=-2 x = -2, y = -2
x(3 – x) = 0 x = - 1.5, y = - 0.47 x=2 y=2
x = - 1, y = -0.0625
x = 0 or 3
x = - 0.5, y = -0.002
(b) y = 4x – x2 y = x2 – 2x x = 0, y=0
dy = 4 – 2x dy = 2x – 2 x = 0.5, y = 0.002
dx dx
x = 1, y = 0.0625
At max or min dy/dx = 0 At max or min dy/dx = 0
x = 1.5, y = 0.47
4 – 2x = 0 2x – 2 = 0
x = 2, y=2
x=2 x=1
Sketching
Values of y at turning points Values of y at turning points
y
y = 4(2) –22 y = 12 – 2(1) 2
=4 =–1
d 2y = -2 < 0 Max point d 2y = 2 > 0 Min point 1

dx 2 dx 2 x
-2 -1 0 1 2
Intercept at x-axis here y is 0 Intercept at x-axis here y is 0
4x – x2 = 0 x2 – 2x = 0 -1
x(4 – x) = 0 x(x – 2) = 0
x = 0 or 4 x = 0 or 2 -2

Intercept at y-axis here x is 0 Intercept at y-axis here x is 0 In the interval 0<x<2 the straight line y = x lies above the
y = 4(0) – (0)2 y = (0)2 – 2(0) curve y = x5/16 but in the interval -2<x<0 their roles are
=0 =0 reversed hence
Sketching Area of the figure bounded
y
0  x 
5
=    x  dx + 2  x  x  dx
5
4

3
 2 16
   0  16 

= x  x 0 x6 2
6 2
+ x2
2 
96 2 2 2 96 0
1

-1 0 1 2 3 4 x = 0 –  64  4  + 4
 
64  – 0

-1
 96 2  2 96 
= 8/3 or 22/3 square units
Our limits are the points of intersection i.e 0 and 3.
From the sketch, uppermost function is 4x – x2 Example ABTC 2 exceptional case
(c) Area of the finite region bounded by the two Find the area of the figure bounded
curves. by the curve y = x2 and the line y = 2– x

 (4 x  x 
3 and the x-axis for which x > 0, y ≥0
=
2
)  ( x 2  2 x) dx
0 Solution
3 It is obvious y = x2 differential will play around zero. Let us
=  0
(3 x  x 2 ) dx substitute using a simple range but bear in mind the
condition x ≥ 0, y ≥ 0 i.e. positive axis only. Take 0 to 2
359
y = x2 y = 2– x Area of finite region bounded by a curve & the y-axis
y
x = 0, y=0 x = 0, y=2
y = f(x)
x = 0.5, y = 0.25 x = 2, y=0
a
x = 1, y=1
x = 1.5, y = 2.25
x = 2, y=4 b
Sketching
y x

2
The diagram above shows a curve of a function f(x). The
1
area of the segment cut of by the y-axis at points
a and b is denoted by
a a
0 1 2 x
 b
f ( y )dy = F ( y )
b
= F(a) – F(b)
The curve and the line meeting points
x2 = 2 – x Here F is the integral result of f(y).
x + x –2 = 0
2

Factorization Example AII 1


x2 + 2x – x – 2 = 0 Find the area of the finite region bounded by the curve
x(x + 2) – 1(x + 2) = 0 y = 9x2 + 1 and the y-axis at y = 3 and y = 1
x + 2 = 0 or x – 1 = 0 Solution
x = – 2 or 1 a

Since we are considering x ≥ 0 and y ≥ 0, we take 1


Area =  b
f ( y )dy
Hence they meet at x = 1. First we make x the subject formula
1 2 y = 9x2 + 1 becomes
The area of  0
x 2 dx   (2  x)dx
1 y – 1 = 9x2
What happened is the region from x = 0 to x = 1 is y – 1 = x2
under the curve (x2) and from x = 1 to x = 2 that region 9
is under the straight line y 1 y 1
3 1 2 2 = x ; Thus, x =
=
x
 2x 
x 9 3
3 2
( y  1) 2
0 1
1 3
= 1 + [ 4– 4 – (2– 1 )] Area =  dy
3 2 2 1 3
= ⅓ + (2 – 1½) 1 3
 ( y  1) 2 dy
1
=
=⅓+½ 3 1

= 5/6 square unit.  1  2( y  1) 2


3
3
= 
Exercise 29.7  3 3 1
Find the area of the region bounded by the curve 2( y  1) 2 3
3

y = 3 – x2 and the line y = – x + 1 and =


between x = 0 and x = 2 9 1
3
2(2) 2
2005/2 UME Exercise 29.8 = 0
Find the area of the figure bounded by the given pair of 9
curves y = x2 – x + 3 and y = 3 2 8 4 2
A.7units(sq) B.1 units(sq) C.17units(sq) D.5 units(sq) = i.e square unit
6 6 6 6 9 9
Example AII 2
Find the area of the finite region bounded by the curve
y = 16x2 and the y-axis at y = 4 and y = 0
Solution
a
Area =  b
f ( y )dy
First we make x the subject formula
y = 16x2 becomes
y
= x2
16
1
y y y2
= x Thus, x = i.e
16 4 4
360
1
4 y2 Solution
Thus, area =  0 4
dy Rotation around y – axis, volume is given by:
b
x
2
1 4 V = dy

1
= y 2 dy a
4 0
We first of all make x the subject formula from y = x2 – 1
1 3 3 4 y + 1 = x2
=  y 2  Taking the square root of both sides
4 2 0
3
( y + 1 )1/2 = x
y2 4 substituting for x in the formula

   y 1  dy
= 3 2
6 0 V=
1
2

4
3 0
2

0 4
= = square units = y2 + y 3
6 3
2 0
VOLUMES substituting for the upper and lower limits
y

=  32 + 3 –  ( 0 )2 – 0 =  ( 9/2 + 3 )
= 15/2  cubic units
)

2 2 (B)
f(x
y=

2003/12a Exercise 29.8b


( i ) Sketch the graph of y = x2 , showing the area in the first
a x quadrant enclosed by the curve and the two lines y = 1
b
If the area below the curve above is rotated: and y = 3
( i ) completely about the x - axis or 3600 or four right ( ii ) If the area in a( i ) is rotated about the y – axis, find the
angles. Then the volume of solid it generates is volume of the solid generated

x
a b

b
y
2
With the formula V = dx
a

Note: y2 and dx
( ii ) Completely about the y – axis or 3600 or four right
angles Then the volume of solid it generates is
y

a x
b

With the formula


b
x
2
dy Note: x2 and dy
a
The solid figure generated by the two diagrams are called
solid of revolution, hence this subtopic is sometimes called
volumes of revolution

2000/37
A bowl is designed by revolving completely the area
enclosed by y = x2 – 1, y = 0 and x  0 around the
y – axis at y =0 and y = 3. What is the volume of this
bowl.?
A.7 cubic units B.15/2  cubic units C.8 cubic units
D.17/2  cubic units

361
Approximate integration(trapezium rule) 2002/12a
Some problems in definite integrals cannot resolve with Using the trapezium rule with ordinates at x = 0, 1, 2, 3, 4,
ease; they require a different approach that will be estimate, correct to two decimal places, the value of
4
considered under this subtopic. At this level it is not for
you to disturb yourself with the types of definite  0
( x 2  9) dx
integrals that will fall into this area. Any question under Solution
this subtopic will be specified as using trapezium rule First we make the required table
y y = f (x) x 0 1 2 3 4
x2 + 9 9 10 13 18 25
3 3.1623 3.6056 4.2426 5
y1
2
x 9 First ord last ord
y2
y Width here is the common difference between 0, 1, 2, 3,…
3
i.e 1 – 0 or 2 – 1 = 1
h y4 yn-1 yn 4
h
h  0
( x 2  9) dx
h
= 1 (1) x [ (3 + 5) + 2(3.1623 + 3.6056 + 4.2426)]
x 2
x1 x2 x3 x4 x n-1 xn = 0.5[8 + 2(11.0105)]
The area under the curve y = f(x) bounded at the = 0.5[8 + 22.021]
x-axis with ordinates x1 = x and x = xn is divided = 0.5[30.021]
into strips of equal widths h. Each of the strips is = 15.0105
an approximate trapezium  15.01 to 2 decimal places
Area of each trapezium = ½ h(sum of two parallel sides) Example ITR 3
xn ( i )Using the trapezium rule with ordinates at x = 0,1,2,3,4,
The area under the curve =  x1
f ( x ) dx
estimate, correct to four decimal places, the value of
1
 12 ( y1  y 2 )h  12 ( y 2  y3 )h  ...  12 ( y n1  y n )h
4
B = dx
0 1 x

 12 h( y1  2 y 2  2 y3  ...  2 y n1  y n ) ( ii ) If the correct value of B is 1.6094, calculate correct


1 to two decimal places, the percentage error
= h  y1  y n  2( y 2  y 3  y 4  ...  y n 1 ) Solution
2
( i )First we make the required table
= 1 (width of each strip)   first ordinate  last ordinate 
2  2( sum of all other ordinates) x 0 1 2 3 4
 
Note that width of each strip is the common difference 1+x 1 2 3 4 5
between the values of x 1 1 0.5 0.3333 0.25 0.2
2004/12c 1+x First ord last ord.
Use the trapezium rule with 5 ordinates at x = 0, 0.25,
0.5, 0.75, 1, to estimate, correct to 2 significant figures, Width here is the common difference between 0, 1,2,3,…
1 1 i.e 1 – 0 or 2 – 1 = 1
the value of  dx 4 1
0 1 x2
 0 1  x dx
Solution
First we make the required table = 1 (1) x [ (1 + 0.2) + 2(0.5 + 0.3333 + 0.2 )]
2
x 0 0.25 0.5 0.75 1
= 0.5[1.2 + 2(1.0333)]
1 + x2 1 1.0625 1.25 1.5625 2
1 1 0.9412 0.8 0.64 0.5 = 0.5[1.2 + 2.0666]
1 + x2 First last = 0.5[3.2666]
ord ord. = 1.6333 to 4 dp
( ii ) Percentage error = wrong value – actual value x 100
Width here is the common difference between 0, 0.25, 0.5…
actual value
i.e 0.25 – 0 or 0.5 – 0.25 = 0.25
= 1.6333 – 1.6094 x 100
1 1
 0 1 x2
dx 1.6094
= 1.4850%
= 1 (0.25) x [ (1 + 0.5) + 2(0.9412 + 0.8 + 0.64)]  1.49% to 2 decimal places
2 ITR 5 Exercise 29.9
= 0.125[1.5 + 2(2.3812)] Use the trapezium rule with 5 ordinates at x = 1, 1.2, 1.4,
= 0.125[1.5 + 4.7624] 1.6, 1.8, 2 to estimate, correct to 2 significant figures, the
= 0.125[6.2624] 2 dx
= 0.7828 value of 
1 x
 0.78 to 2 significant figures

362
Chapter Thirty There are two methods of solving the problems on resultant
Composition and Resolution of forces of any given two forces; namely:
Analytical and Graphical. We shall be concern with analytical.
Composition of forces Problems on resultant of two forces
(1) Two forces acting at a point will result in a single 2003/26 Neco
force called a Resultant force Find the resultant of the forces 7N and 11N with angle 450
We apply the parallelogram law of composition of two between them. (Correct to 2 d .p)
forces which states: A0.78N B 7.80N C 7.82N D 78.2N E 78.5N
If two forces acting at a point are represented in Solution
magnitude and direction by two adjacent sides of a Sketched diagram.
parallelogram then the resultant of the two forces, is
represented in magnitude and direction by the diagonal 7N R 7N
of the parallelogram, drawn from the point of action of
0 -45
the two forces. 45
180 450
(2) A typical diagram for two forces T and S acting at a 11N
point X with resultant R is represented by the completed R2 = 72 + 112 – 2 × 7 × 11 cos (180 – 45 )
parallelogram below: = 49 + 121 – 154 cos 1350
Z = 170 + 108.89
Y R2 = 61.11
R R= 61 .11  7.82N (C )
T
1993/39 Neco
Find the resultant of forces 8N and 10N inclined at
X W an angle 1200 to each other.
S
A. 2 61 N B. 2 51 N C. 2 31 N
XZ = XY + XW
Further representation of the above diagram is: D. 2 21 N E. 2 6 . 36 N
Y Z Solution
Sketched diagram
T R
T 8N R 8N
-
180 -12
0
180 1200
0
X 120
S W V
By Cosine rule 10N
R2 = T2 + S2 – 2 × T × S × Cos( 180 –  ) R2 = 82 + 102 – 2 × 8 × 10 cos (180 – 120 )
= 64 + 100 – 160 cos 60
We can also get γ , the inclination of T to S in the right = 164 – 80
angled triangle ZXV R2 = 84
R= 84 ie 4 21 = 2 21 N ( D )
ZV ZV
tan   
XV S  WV 2007/29 Neco
Find the magnitude of the resultant of two forces 6 N and 8N
To get WV and ZV in right – angled triangle ZWV
acting at 2400 to each other.
WV and T gives CAH. Thus
WV 19 19
Cos   A. 4 19 B. 2 19 C. 2 37 D. E.
T 4 2
WV = T Cos  Solution

(3) Similarly ZV and T gives SOH, thus,


ZV
Sin  
T
ZV = T Sin 
T sin 
Hence tan  
S  T cos  We calculate resultant R using cosine rule
R2 = 62 + 82 – 2 × 6 × 8 cos (180 – 240 )
= 36 + 64 – 96 cos (–60 )
363
= 100 – 96(– 0.5 ) 49 = 25 + 4p2 – 10p
= 100 + 48 Rearranging
R2 = 148 4p2 – 10p – 24 = 0
i.e. 2p2 – 5p – 12 = 0
R= 148 = 4  37 i.e 2 37 N ( C )
Factorizing
1996/ 36 2p2 – 8p + 3p – 12 = 0
The angle between two forces of magnitudes 8N and 5N 2p( p – 4 ) + 3( p – 4 ) = 0
is 1200. Find in N, magnitude of their resultant. ( p – 4 )( 2p + 3 ) = 0
A. 13 B. 11 C. 8 D. 7 E. 5 p – 4 = 0 or 2p + 3 = 0
3
Solution p = 4 or –
Sketched diagram 2
Acceptable p here is 4.0 N
8N R 8N 2000/34
0 Forces of magnitudes 8N and 5N act on a body. If the angle
-12
180 1200
0
120 between the forces is 1200, find, correct to the nearest
degree, the angle which the resultant makes with the force 5N.
5N
R2 = 82 + 52 – 2 × 8 × 5 cos (180 – 120 ) A 80 B 380 C 820 D 830
= 64 + 25 – 80 cos 600 Solution
A C
= 89 – 40
R2 = 49
8N R 8N
R = 49 i.e 7 N ( D)
0
0
-12
2003/11 (Nov) 120
180 1200
The resultant of two forces xN and 4N is 12N. D E
If the resultant makes an angle of 300 with the force 4N, B 5N
find the value of x. (N = Newton’s) T sin 
By direct substitution tan  
A 12.6 B 11.3 C 8.8 D 6.0 S  T cos 
Solution
A D 8 Sin 120 0
tan γ =
5  8 Cos 120 0
xN 12N
xN 6.9282
tan γ =
0 1
30
B E C γ = tan– 1 6.9282
4N = 81.790
In BDE  820 (C)
X2 = 122 + 42 – 2 × 12 × 4 Cos 300 Alternative approach
= 144 + 16 – 96 Cos 300 We get γ by Sin rule
= 160 – 83.14 8 R
X2 = 76.86 = From BCD
Sin  Sin 60
X = 76 .86 But R2 = 52 + 82 – 2×5×8Cos(180 – 120)
= 8.767N = 25 + 64 – 80 Cos60
 8.8N (C) = 89 – 40
R2 = 49
1999/25 (Nov)
The angle between forces of magnitudes 2p and R= 49 i.e 7
5 Newtons acting on a particle is 1200. If the magnitude 8 7
of their resultant is 7 Newton’s, find p. Thus =
A 1.5 B 3.0 C 4.0 D 8.0
Sin  Sin 60
Solution 8 Sin 60
Sin γ =
7
2pN Sin γ = 0.9897
7N 2pN
γ = Sin – 1 0.9897
0 = 81.770
0
120 80-12 0
1 120  820 (C)
5N
7 = 5 + (2p)2 – 2×5×2p Cos (180 – 120)
2 2

49 = 25 + 4p2 – 20p Cos 60


364
Problems on angles between two forces  567
cos  
1994/36 (Nov) 798
Calculate, correct to one decimal place, the angle Cos = – 0.7105
between two forces 20N and 30N if their resultant  = –cos –1 0.7105
is 40N. 44.72
A. 14.50 B. 55.00 C. 75.50 D. 104.50 E. 135.00 Cosine is negative in 2nd quadrant with
Solution 180 – 44 .72
We sketch as shown below:  = 135.280
Thus  = 44. 72
40N  450 to nearest degree.
20N

20N 2000/18 (Nov)


If the magnitude of the resultant of two forces of magnitudes
30N P and 3P Newtons acting at a point is P 7 N , find the
The cosine formula that can help us get  is not done angle between the forces.
directly . But we proceed as follows: A. 240 B. 300 C. 450 D. 1200
We let the angle between the two force be  Solution
 +  = 1800 (interior opp s )
 = 180 –  P P7 P
30 2  20 2  40 2
cos  
2  30  20
900  400  1600
= 3P
1200 Let angle be 
 300  +  = 1800 (interior opp s )
=
1200  = 180 – 
Cos  = – 0. 25 By cosine formula
 = – cos–1 0.25
75.50 cos =

p 2  (3 p) 2  p 7 
2

Negative cosine here is 2nd quadrant 2 p 3 p


180 – 75.5 = 104.5 = p  9p 27p
2 2 2

Since  = 180 –  6P
Thus  = 75.50 (C) 3p2
=
6 p2
cos  = 0.5
1999/16a (Nov)
The angle between two forces of magnitude 19 and 21  = cos–1 0.5
= 600
Newtons is . If the magnitude of their resultant is
37 Newtons, calculate correct to the nearest degree the
But  = 180 – 1600 = 1200 (D)
value of  . 2005/16a (Nov) Exercise 30.1
Solution Two forces of magnitudes 50N and 100N have a resultant of
The cosine formula for getting  is done indirectly. But magnitude 120N. Calculate, correct to the nearest degree the
we proceed as follows: Sketch as angle between the two forces.

2006/17b Exercise 30.2


37N
19N

Two forces of magnitudes 3N and 7N have a resultant of


19N

magnitude 5N. Calculate, correct to one decimal place the


angle between forces.
21N
2000/31 Neco Exercise 30.3
 +  = 1800 (interior opp s) Two force P and Q of 2N and 6N respectively have
 = 180 –  a resultant of 5N. Calculate the angle between them.
By cosine formula A. 128.680 B. 51.320 C. 38.680 D. 32.000 E. – 38. 680
19  21  37
2 2 2
cos   2001/13 Neco Exercise 30.4
2 19  21 Two forces A and B have magnitudes of 15N and 21N
361  441  1369 respectively. The resultant of the two forces is R. If the angle
 between A and R is 450, find
798
( i ) the angle between A and B, (ii) the magnitude of R

365
Resolution of forces Case III
This is a reverse process of the resultant of two given  inclined to negative x - axis
forces treated earlier. y
Here we resolve a given force into two parts (resolute)
i.e. along the y – axis which is vertical and along the
x – axis which is horizontal F
We are to take note of the inclination or direction of the F
y
force under consideration.
For any given Force F that acts at an inclined angle  to -x x
-Fx
the x – axis, as shown below:
Case I  Fx
= Cos  Thus Fx = – F Cos 
 inclined to positive x - axis F
y Fy
= Sin  Thus Fy = F Sin 
F
Fy F Fy  F 
tan  = Thus  = tan – 1  y 
F x   Fx 
x
Fx Example RFF 1
Find the vertical and horizontal components of the force
The Fy is the Vertical component and
F = ( 200N, 0300 ).
Fx is the Horizontal component
Solution
and by Simple trig ratios y

Fx 0N
= Cos  Thus Fx = F Cos  20
F Fy
0
Fy 30
x
= Sin  Thus Fy = F Sin  Fx
F Relating Fx to 200N by trig ratio ( Adj and Hyp )
Fy  Fy  Fx
tan  = Thus  = tan –1
  = Cos 300
Fx  Fx  200
Thus, Fx = 200 Cos300
and by vector notation, i stands for x – axis and j stands = 173.2N
for y – axis; Relating Fy to 200N by trig ratio (Opp and Hypo)
F = Fx i + Fy j Fy
= Sin 300
200
Case II Fy = 200 Sin 300
 inclined upward to y - axis = 100 N
y
Fx 2004/23 Neco (Dec)
The vertical component of a force F which makes an angle
Fy F of 250 with the horizontal is 60 N. Find the magnitude of the
force.
A 0.04 N B 25.4 N C 50.4 N D 66.2 N E 141.97 N
Fx x Solution
y
Fx
= Sin  Thus Fx = F Sin 
F F
60N
Fy 25
0

= Cos  Thus Fy = F Cos  x


F
Relating 60N to F by trig ratio ( Opp, Hyp)
Fx  Fx  60
tan  = Thus  = tan – 1   = Sin 250
F  F
Fy  y
60
F =
Sin 25 0
= 141.97 N (E)
366
Resultant of several concurrent forces Solution
Resulting angles
Here we resolve each force F into vertical component Q
fy and horizontal component fx . It is done with the aid N -x Horizontal

Q
of a table as shown in the examples that follows. P Vertical
+y

 f    f 
86N
2 2 300

R = y x 86N 24N
0
30 400
Where R is resultant force Horizontal P +
+x

fy is sum of fy and fx is sum of fx W E Vertical


+y 24N

f
400

y
and tan  =
f x
Forces Vertical Components Horizontal Comp.

 = tan –1
f y 24 N (P) 24 Cos 40 = 18.39N 24 Sin 400
= 15.43N
f x
Where  is the angle, which the resultant makes with the horizontal 86N(Q) 86 Cos 300 = 74.48N - 86 Sin 300
= - 43N
2004/48 Neco
10N 20N Total = 92.87 N = - 27.57

R = 92.872   27.57
2

30
0 Fy
= 8624.8369  760.1049
30
0 x-axis
= 9384 .9418 i.e 96.88N

30N tan  =
f y

Find the sum of the components of forces along the x – axis. f x


A 48.30N B 45.98N C 35.98N 92.87
D 12.32N E 2.32N tan  =
Solution  27.57
 = tan-1 3.3685 = – 73.470
Forces Horizontal components Resulting triangles But negative shows  is
Horizontal
180 – 73.47 i.e. = 106.530   1070
x-axis
0 3 300 2004/44 Neco
30 N 30 Cos 30 = 30  Bytrigratio
Adj andHyp
Given that f1 ( 6N, N300E ) and f2 ( 8N, 0900 ) are two forces
2 -y CAH

30N
acting at a point, what is the modulus of their resultant ?
= 15 3N A 11N B 12.17N C 16.20N D 148N E262N
Solution
N
20 N 20 Sin 300 = 20 
1 Horizontal
+x F
1
20N
2 By trig ratio

6N
+y
= 10 N 300
Opp and Hyp
SOH
0
30
0
090
10 N 0 W E
8N F2
All our dealings are positive in both axis
Total ( C ) = 35.98 N
Forces Vertical Components Horizontal Comp.

3 1
6N 6 Cos 300 = 6  6 Sin 300 = 6 
2004/12 Neco (Dec) 2 2
Two forces P and Q act at a point. The magnitude of P
is 24N and the magnitude of Q is 86N. If P acts in the =3 3N = 3N
direction N 400 E and Q acts in the direction N 300 W, = 5.20N
Find the:
(i) components of P and Q 8N 0 8 Cos 00 = 8  1
(ii) magnitude of the resultant force = 8N
(iii) angle which the resultant force makes with the Total 5.20N 11N
horizontal.
367
(b) Direction of the resultant force.
R = 5.202  112
Solution
= 148 .04 Let us stick to the guide “ 6N force act in the direction 0900 ”
4N
= 12.167 N  12.17N ( B) 8N
2003/6 Neco 65 0
Calculate the sum of horizontal component of forces in 100 0
the diagram drawn below, (express your answer in 2
decimal places). 550
Y 6N
5N 65 250
10N

130
0 4N
0 0
60 45 0
-X X 5N 65
300
0 +y
30
4N -x
4N
8N
Vertical
0
-Y 65 +y
Solution
Resulting diagrams

Vertical
-y

Force Vertical Components Horizontal Comp.

6N 6 Cos 00 = 6  1
0 = 6N
Forces Horizontal Components
8N 8 Sin 550 = 6.55N 8 Cos550 = 4.59N
10N 10Cos 450 = 7.07N
4N 4 Cos 650 = 1.69N - 4Sin 650 = - 3.63N
5N - 5Cos 600 = - 2.5 N
5N -5 Cos 650 = -2.11N -5 Sin 650 = -4.53N
4N - 4Sin 300 = - 2 N
4N -4 Sin 250 = -1.69N 4 Cos 250 = 3.63N
8N 8Cos 300 = 6.93 N
Total 8.24 – 3.8 = 4.44N 14.22 – 8.16 = 6.06N

Total = 9.5 N
R = 4.442  6.06
2

2009/17
4N = 19.71  36.72
8N
= 56 .43
1000 = 7.51N
0
55

0
250
6N
(b) tan  =
f y

f
130
x
5N 4N
4.44

DIAGRAMNOT DRAWNTOSCALE 6.06
Coplanar forces 4N, 8N, 6N, 4N and 5N act at a point tan  = 0.7327
as shown in the diagram. If the 6N force act in the  = tan – 1 0.7327
direction 0900, calculate the: = 36.230
(a) Magnitude of the resultant force

368
2000/7 N
NW Resulting angles
Forces of magnitude 2, 5 and 4 units lie in the direction N
0450, 0900 and 1350 respectively. Find; Horizontal
-x
(a) The magnitude,
(b) The direction of their resultant. 25N Vertical
0
45 25N
+y
Solution 45 0

We apply our knowledge of three digit compass bearing


W E
and sketch as: 20N
Y
2 Forces Vertical Components Horizontal
Components
450
20N 20 Sin 0 = 0 20 Cos 0 = 20N
-X 0
5 X
135
450
25N 25Cos 450 = 17.68N -25 Sin 450 =
-17.68N
4
-Y
Total = - 17.68N = 2.32N

Forces Fy (Vertical Components) Fx (Horizontal


Components) R =  17.682  2.32
2

= 312.58  5.38
2 2 Cos 450 = 1.41 2 Sin 450 = 1.41
= 317 .96
 17.83 N (B)
5 fy=0 fx=5
1995/ 32 (Nov)
Three coplanar forces 4 3 N, 5 3 N, and 7N act on a
4 - 4 Sin 450 = - 2.83 4Cos 450 = 2.83 particle in the bearings 0600, 1800 and 2700respectively. Find
the magnitude of their resultant.

Total = - 1.42N = 9.24N A2 37 N B3 3N C2 7N


D2 19 N E 82 N
R =  1.42 2
 9.24
2
Solution
We apply our knowledge of three digit compass bearing and sketch as:
= 2.02  85.38 +y
4 3N Resulting angle

= 87 .4 600
4 3N
+X
= 9.35 N
f
7N +X
Vertical
y 2700 1800 +y
tan  = 600
f x

 1.42
 5 3N
9.24
tan  = - 0.1537 Forces Vertical Components Horizontal
 = - tan – 1 0.1537 Components
= 8.70
tan  is negative in 2nd & 3rd quadrant 4 3N 4 3 Cos 600 = 2 3N 4 3 Sin 600 = 6N
 is either
180 – 8.7 or 360 – 8.7
171.30 or 351.30 5 3N -5 3N 0

2004/28
Two forces of magnitudes 20N and 25N act at a point 7N 0 -7
along the East and North – West respectively. Find, in
N, the magnitude of the resultant.
A 5.00 B 17.83 C 25.91 D 41.62 Total -3 3N -1N
Solution
Applying our knowledge of compass bearing, we sketch
369
R=  3 3  2
 ( 1) 2
2002/ 8
Four coplanar forces of magnitudes 5N, 5 3 N, 10 N and
= 27  1
10 3 N act on a body in the directions 3000, 3300, 0000 and
= 28 1200 respectively.
= 2 7(C) (a)Find the magnitude of the resultant force
(b)If the mass of the body is 6kg, Find its acceleration.
1993/ 38 (Nov) Solution
Forces P(60N, 0000), Q(20N, 0900), R(50N, 1800) and Applying our knowledge of three digit bearing sketch as:
Resulting diagram
S(30N, 2700) act at a point. Calculate their resultant.
5 3N 10NY 5 3N 10NY
A (160N, 18000) B (40N, 0450)
C (10 2 N, 045 ) D (40N, 3150) E (10 2 N, 3150)
0
5N 5N 0
30
3300
Solution 120 0
300
-X 5 X
Applying our knowledge of three digit bearing -X 5 X
30 0
P 60N 3000

10 3 N 10 3 N

S -Y -Y
Q
30N 20N

Break down diagrams

R
50N

Forces Fy (Vertical Components) Fx (Horizontal


Components)

P 60N Fy = 60N Fx = 0

Forces Vertical Components Horizontal


Q 20N Fy = 0 F x = 20 N Components

10N 10N 0
R 50N Fy = - 50 N Fx = 0

S 30N Fy = 0 F x = - 30 N 10 3N -10 3 Sin 300 = -8.66N 10 3 Cos 300 =15N


Total = 10 N = - 10 N 5N 5 Sin 300 = 2.5N -5 Cos 300 = -4.33N

5 3N 5 3 Cos 300 = 7.5N -5 3 Sin 300


10 2   10
2
R = =- 4.33N
= 100  100 Total 20 – 8.66 = 11.34N 15 – 8.66 = 6.34N
= 200
= 10 2
R = 11.342  6.34
2

tan  =
f y
= 128.60  40.2
f x
= 168 .8
10 = 12.99N  13N
=
 10
(b) F=ma
tan  = - 1
13 = 6  a
 = - tan – 1 1
13
= - 450 a = i.e 2.17m/s2
tan is negative in the 2nd and 3rd quadrant 6
Thus  is either 180 – 450 or 360 – 45 2005/31
1350 or 3150 Two forces F1 ( 8N, 0300 ) and F2( 10N, 1500 ) act on a
(10 2 N, 315 ) 0
E particle. Find the magnitude of the x – component of the
resultant of the forces.
A 1.7N B 4.5N C 9.0N D 13.0N

370
Solution 2012/17b Exercise 30.5b
Applying our knowledge of three digit bearing, we 6N
8N
sketch as;
Resulting diagram
y y
0 4N
F1 F1 30
0
60

8N
8N
0 0
30 30
1500
+X
P
+X
600 Not drawn to scale
10 10
N
Forces of magnitude 8N, 6N and 4N act at the point P, as
N
F2 F2
shown in the above diagram. Find the:
-y -y
( i ) magnitude; ( ii ) direction of the resultant force

2013/6 Neco Exercise 30.5c


Find, correct to 2 decimal places, the magnitude of the
resultant of the following forces
y
8N 5N

Force x - components 700 0


60
300 x
8N ( F1) 8 Sin 300 = 4N P
20N
0
10N (F2) 10 Cos 60 = 5N
2011/6 Neco Exercise 30.5d
Total ( C ) = 9N Three forces F1(3N, 2400), F2(2N, 3000) and F3(7N, 700)
act on a body. Find the resultant force and the angle it makes
with the positive x-axis
1996/40
Calculate, in N, the horizontal components of the forces
in the diagram
A 13.00 B 9.49 C 8.26 D 8.16 E 1.33
Solution
Y
5N

8N

0
60
0
45
X

Force fx(Horizontal Components)

5N -5 Sin 600 = - 4.33

8N 8 Cos 450 = 5.66

Total = 1.33N (E)

2003/28 (Nov) Exercise 30.5


Y

8N
5N

300 45
0

X
Using the diagram above, Calculate, in Newton, the
magnitude of the resultant of the forces 5N and 8N.
A 8.16 B 8.26 C 8.49 D 10.47

371
Chapter Thirty one (b) String X Z
Equilibrium of forces XZ 650
For a body to be in equilibrium it must not have =
Sin 135 Sin 60
tendency to rotate. The equilibrium is maintained if the
total sum of forces acting on the body is zero. 650 Sin 135
XZ =
Sin 60
Three forces in equilibrium
The indirect use of sine rule will play a role here it is 459.619
=
known as Lami’s theorem which states that: 0.8660
If three forces acting at a point are in equilibrium, then
= 530.738
each force is proportional to the sine of the angle
 530.74N to 2 dp
between the lines of action of the other two forces
2006/16a
Diagrammatically:
A body of mass 15kg is suspended at a point P by two light
T2 in extensible strings X P and Y P . The strings are
T1 inclined at 600 and 400 respectively to the downward
vertical. Find, correct to two decimal places, the tensions in
the strings. [ Take g = 10ms – 2 ]
T3 Solution
X Y
0
T1 = T2 = T3 100
Sin  Sin  Sin  P
Resulting to
120140
0
0
1400
2003/12b Neco (Dec)
A particle X of weight 65kg at rest is suspended by two
light strings XY and XZ which makes angles of 450 and
W
750 respectively with the horizontal. Find the tension in
each string correct to 2 decimal places.
Solution Where W = mg
Y Z Y = 15 × 10 i.e. 150N
Z
Since we have gotten all the angles, Lami’s theorem will
60
0 apply
0 0 Resulting to 750 450
75 45 W XP YP
Horizontal
x 0 x 0 = =
165 135 Sin 100 0 Sin 140 0 Sin 120 0

65kg W (a) String X P


XP W
Since all the angles are gotten, we apply Lami’s 0 =
Sin 140 Sin 100 0
theorem
Where W = mg 150 Sin 140
XP =
= 65×10 Sin 100
= 650N 96 .418
W XY XZ =
= = 0.9848
Sin 60 Sin 165 0 Sin 135 0 = 97.906N
(a) String X Y  97.91N to 2d p
XY 650 (b) String Y P
0
=
Sin 165 Sin 60 YP W
650 Sin 165 0 =
Sin 120 Sin 100 0
XY =
Sin 60 150 Sin 120
YP =
168.23 Sin 100
=
0.8660 129 .904
=
 194.26N to 2 dp 0.9848
= 131.909
131.91N to 2d p

372
2005/13b Neco force R. If the string is inclined at 300 to the vertical and the
P system is in equilibrium, Calculate, in Newton’s
(i) The tension T in the string;
T
600 (ii) The force R. [Take g = 10 ms – 2]
S Solution
Resulting diagram
10kg
In the diagram above, an object of mass 10kg hanging
from a light inextensible string is pulled aside by a
horizontal force S. If the string is inclined at 600 to the
vertical and the system is in equilibrium. Calculate in Newton’s:
(i) The tension T in the string,
(ii) The force S. (Take g = 10ms – 2 )
Solution
Diagram can be produced out as:
Since all the angles between the forces are known we apply Lami’s theorem.
Here W = mg
= 3 ×10  i.e. 30N
W T R
0
= 0
=
Sin 120 Sin 90 Sin 150 0
(i) Tension T
T 30
=
Sin 90 Sin 120
Since the angles between the three forces are known,
we can apply Lami’s theorem. 30 Sin 90
T=
First W = mg Sin 120
= 10 ×10 = 100N
30
T W S = = 34.64N
= = 0.8660
Sin 90 Sin 150 Sin 120 0
( ii ) Force R
( i ) Tension T
R 30
T 100 =
= Sin 150 Sin 120
Sin 90 Sin 150
30 Sin 150
100 Sin 90 R=
T= Sin 120
Sin 150
100  1
15
= = 200 N = = 17.32N
0 .5 0.8660
( ii ) Force S 1996/8
S 100 An object of mass 10kg is in equilibrium when suspended by
= two light inextensible strings of lengths 4m and 3m. If the
Sin 120 Sin 150 other ends of the strings are fixed to two points, 5m apart on
100 Sin 120 the same horizontal level, calculate the tension in each
S =
0.5 string. (Take g = 10ms– 2)
86 .60 Solution
= = 173.2 N A 5m B
0 .5
2002/16 (b)
T1 T2
P 4m 3m
0 T C
30
R

3kg 10kg
In the diagram, an object of mass 3kg hanging from a First, we assume  ACB to be 900
light inextensible string is pulled aside by a horizontal Thus AB2 = AC2 + BC2
52 = 42 + 32 (which is true)
373
we re – draw our diagram and label as Solution
A 5m B P 6.9m Q

T1 T2
4m 3m 8 8
C
C

W 100N 5kg
Where W = 10kg × g = 100N Applying Cosine rule to get γ
Next, we get  and  to enable us work freely. 6.92 = 82 + 82 – 2 × 8 × 8 Cos γ
In  ABC 8 2  8 2  6.9 2
3 Cos γ =
Sin  = i.e. 0.6 288
5 64  64  47 .61
 = Sin– 1 0.6 =
128
= 36.860
80 .39
4 =
Also Sin  = i.e. 0.8 128
5
Cos γ = 0.6280
 = Sin– 1 0.8
γ = Cos– 1 0.6280
= 53.130
= 51.100
Resulting diagram
5m Since the given triangle is isosceles
A B
180  51 .10
It follows that  =
T1 T2 2
4m 3m 12 8.9
= i.e 64.450
2
Thus we re-sketch our diagram as:
P Q

W 100N
Applying Lami’s theorem T1 T2
100 T1 T2
= =
Sin 90 Sin 143 .13 Sin 126 .86
T 1 tension in one String
T1 100 W (5x10)N
=
Sin 143 .13 Sin 90 Applying Lami’s theorem
100 Sin 143 .13 W T2 T1
T1 = = =
Sin 90 Sin 51.1 Sin 154 .45 Sin 154 .45
= 60.00N
T 2 tension in the other string We observe that T1 = T2
T2 100 T1 50
=
= Sin 154 .45 Sin 51.1
Sin 126 .86 Sin 90
100 Sin 126 .86 50 Sin 154.45
T1 =
T2 = Sin 51.1
Sin 90
= 80.01N 21 .56
=
0.7782
2005/18a (Nov) = 27.70N same for T2
A light inextensible string 8 metres long suspends a ring
of mass 5kg. The two ends of the strings are attached to
points P and Q of a ceiling such that PQ = 6.9m. If
the system is in equilibrium, calculate, the tension in the
string. (Take g = 10ms – 2 )
374
1992/40 (Nov) 98 2
A body of mass 10kg hangs from a fixed point by a = × 3 × Sin 300
inextensible string. It is pulled aside by a horizontal 3
force P and rests in equilibrium with the string inclined 98 2 1
at an angle 300 to the vertical. Taking g = 9.8ms– 2, = × 3×
find the force P
3 2
98 3
98 3 49 3 = N (B)
A 49N B N C N 3
3 3 1993/35 (Nov) Exercise 31.1
D 49 3 N E 196 3N A body of mass 40kg is suspended with an inelastic string as
shown in the diagram. If the system is in equilibrium find, in
Solution
terms of g, the tension T in Newton’s in the string.
300 600 600

T T

40g
Applying Lami’s theorem A 20g B 40g C 40 3 g D 80 3 g E 80g
P W 3 3
0 =
Sin 150 Sin 120 0 2002/28 ( Nov) Exercise 31.2
Where W = 10 × 9.8 In the diagram below, forces PN, QN and 8N are in
= 98N equilibrium. Find the value of P
A 4.00 B 4.62 C 12.93 D 13.86
98 Sin 150
P=
Sin 120
3
= 49 
2
2 98 3
= 49 × = N (B)
3 3

Alternative method
Resolving along the components 2005/18a ( Nov) Exercise 31.3
A light inextensible string 8metres long suspends a ring of
At equilibrium  f  = 0,  f y = 0 mass 5kg. The two ends of the strings are attached to points
 f   P – S Sin 300 = 0 --------(1) P and Q of a ceiling such that PQ = 6.9m. If the system is
in equilibrium, calculate, the tension in the string.
 fy  - 98 + S Cos 300 = 0------(2) [Take g = 10ms– 2]
from (2) S Cos 300 = 98
98
S=
Cos 30 0
3
= 98 
2
2
= 98 ×
3
3
= 98×2× Rationalizing
3
Substituting for s in (1)
P = S Sin 300

375
Triangle of forces 4.5 = 9 Note that  = 180 – 30 i.e 1500
If we have insufficient data such as angles needed sin  sin
between the three forces in equilibrium then 4.5 = 9
represent them in a triangle and solve depending on sin 150 sin
the resulting triangle type (right-angled, acute,obtuse)
Here we need to be careful to note the flow of arrows sin = 9 sin150
and angle between each force 4.5
Triangle of forces states that: if three forces(in the same sin = 1
plane) act at a point and are in equilibrium, then they  = sin– 1 1
can be represented in magnitude and direction by the  = 900
three sides of a triangle taken in order Thus  = 180 – 90 i.e 900 ( D)

2002/35
T1 Forces of magnitudes 3N, 5N and 7N act on a body. If the
T2
T1
forces are in equilibrium, find, correct to the nearest degree,
T3
the angle between forces 5N and 7N.
Reprsented by A 1120 B 1200 C 1420 D 1580
T3
T2 Solution
Let the unknown angle be  ; then we sketch as

1992/35
Forces 8N, 7N and 5N, acting on a particle, are in
equilibrium. Find the angle between the forces 8N and 5N
A 300 B 450 C 600 D 900 E 1200
Solution
Let the unknown angle be  ; then we sketch as

By cosine rule:
32 = 72 + 52 – 2 × 7 × 5cos 
9 = 49 + 25 – 70cos 
70cos  = 65
cos  = 65 i.e 0.9286
70
By cosine rule:
 = cos– 1 0.9286
72 = 82 + 52 – 2 × 8 × 5cos 
 = 21.80
49 = 64 + 25 – 80cos 
Thus  is 180-21.80 i.e 1580 ( D )
80cos  = 40
cos  = 40 i.e 0.5
1992/8 ( Nov)
80
 = cos– 1 0.5
 = 600
Thus  = 180 – 60 i.e 1200 ( E)

1995/36 (Nov)
Three coplanar forces of magnitude 6N, 9N and 4.5N
maintain a particle in equilibrium. If the angle between
the forces 6N and 9N is 300, using Lami’s theorem or In the diagram above, forces of magnitudes 10N, 26N and
otherwise, find the angle between the forces 4.5N and 6N PN act on a particle in the directions indicated. If the particle
A 300 B 450 C 600 D 900 E 1200 is in equilibrium, find : (a) P; (b) 
Solution Solution
Let the unknown angle be  ; then we sketch as

Condition of Sine rule are met here : (Lami’s theorem)


376
The forces here are more than three; thus we resolve along
( a) By Pythagoras rule:
the components to solve for S and T
P2 = 102 + 262
At equilibrium fx = 0 and fy = 0
P2 = 776
fx  T Sin 300 – S cos 300 –10 Cos 600 + 4 = 0----(1)
P= 776 i.e 27.86N 0.5T – 0.866S – 5 + 4 = 0
(b) Sin  = 26 fy  T Cos 300 + 8 + 5Sin 300 –10Sin 600 = 0---- (2)
27.86
From (1) 0.5T– 0.866S – 5 + 4 = 0 becomes
Sin  = 0.9332
0.5T– 0.866S = 1----------(3)
 = sin– 1 0.9332
Also from (2) 0.866T + 8 + 0.5S – 8.66 = 0
 = 68.940 0.866T + 0.5S = 0.66 = ---------(4)
Thus  = 90 – 68.940 Next, (3) × 0.866 and (4) × 0.5 Then subtract
= 21.060 0.433 T – 0.750 S = 0.866
2003/7b – (0.433 T – 0.25 S = 0.33 )
Three forces F1(10N, 0300 ), F2(PN, 1200 ) and F1(15N, 0 ), – 1S = 0.536
act on a body. If the system is in equilibrium, S = –0.536
( i ) calculate the value of P Applying the given conditions
( ii ) find 
Solution S = 0.536
Three-digit bearing is our guide in sketching  0.5 N to 1 d p
Substitute S = – 0.536 into (3)
0.5 T – 0.866 (– 0.536 ) = 1
0.5T + 0.464 = 1
0.5T = 1 – 0.464
0.536
T=
0 .5
= 1.072
1.1N to 1 d p
( i ) By Pythagoras rule:
152 = 102 + P2
P2 = 152 – 102
P = 125 i.e 11.18N
( ii ) By sine rule
10 = 15
sin  sin 90
sin  = 10 sin 90
15
sin  = 0.6667
 = sin– 1 0.6667
= 41.810
By the first diagram  in three digit bearing is
120 + 41.810 i.e 161.810

Counter Example
( where Lami’s theorem, triangle law fails)
Five force T, 4, 8, S and 10 newtons acting on a body in
the direction 0300, 0900, 0000, 3000 and 2100
respectively are in equilibrium. Calculate, correct to one
decimal places, the values of T and S
Solution

377
Moments about a point Solution
Moment is the result of a turning effect of a force about 1
0m 1
0m
a point. It is the product of the force and the P Q
1
0-x x
perpendicular distance it covers.For equilibrium to hold:
Clockwise Moment = Anti clockwise Moment
2
0N 8
0N 5
0N
Defined terms
Centre of gravity: position where the resultant Let the point be D
weight of a body tends to act. Taking moment about point D
20 10 10  x  80 (10  x )  50  x
Uniform bar or beam: here the centre of gravity is
always in the centre position 400 – 20x + 800 – 80x = 50x
For instance A uniform bar of 10m must have its centre 1200 – 100x = 50x
of gravity at 5m. 1200 = 150x
Non-uniform bar or beam: centre of gravity can 1200
=x
be located at any point along the bar 150
Mass and weight : Any given mass in Kg must be x = 8m
converted to weight in Newtons by multiplication by Thus, distance from P to D = 20 – x ( i.e 10 + 10 - x)
10m/s2 or 9.8m/s2 depending on the given value of g = 20 – 8
= 12m (C )
In taking moments about a point problem, we also apply 1995/39 (Nov)
resolving of forces along the vertical components in A non – uniform beam PQ of length 4m and weight 100N,
some cases; the diagram below indicates that. rest horizontally on supports at P and Q. If the centre of
R
1 R
2 gravity of the beam is 1.5m from P, find in N, the reaction at P
A 34.5 B 37. 5 C 56. 5 D 62. 5 E 65.5
P Q Solution
R 1 R 2

W
2(N
) W
1(K
gxg
) W
3(N
) 1.5m 2.5m
P Q
At equilibrium Σ fy = 0
R1 + R2 – ( W2 + W1 + W3 ) = 0
Thus, R1 + R2 = W2 + W1 + W3 100N
Taking moment at Q to eleminate R2
4 × R1 = 100 × 2. 5
1992/33 (Nov)
4R1 = 250
A uniform rod 12m long is pivoted at its centre of
R1 = 250 ie 62. 5 N ( D )
gravity. If a mass of 50kg is suspended from one end, 4
how far from the other end must a mass of 75kg be Alternative solution
suspended to make the rod rest horizontally? Resolving vertically
A. 2m B. 4m C. 6m D. 10m E. 12m R1 + R2 = 100
Solution But taking moment at P
g
x 6m
100 × 1.5 = 4 × R2
150 = 4R2
R2 = 37. 5N
Substituting for R2
R1 + R2 = 100
75kg 50kg R1 = 100 – R2
Taking moment at point g = 100 – 37. 5 i.e 62.5 N (D)
Sum of clock wise moment = sum anti-clockwise moment 1996/37
50 × 6 = 75 × x The diagram shows a light rod PQ of length 8m, suspended
300 = 75x in a horizontal positions with two vertical strings attached to
300 points C and D. Weights 2N, 5Nand 4N are suspended from
= x i.e x = 4m ( B ) the rod in position P, H and Q. Find the tension T2 N
75 T2
T1
1994/31 P 2m 2m H 2m 2m Q
A Uniform rod PQ, is 20m long and weights 80N, C D
has a weights 20N and 50N suspended at P and Q
2N 4N
respectively. Find the distance from P where the rod
must be supported so that it will rest horizontally? 5N

A 2m B 8m C 12m D 16m E 17m A 3. 5 B 4. 5 C 6. 5 D 7.5 E 9.0

378
Solution Even though it is a sketched diagram care must be taken to
Taking moment at C to eliminate T1 place the various mass in their respective position before and
2 × 2 + T2 × (2 + 2 ) = 5 × 2 + 4 × (2 + 2 + 2 ) after centre of gravity based on the given length as compared
4 + 4T2 = 10 + 24 to the whole length of the beam.
4T2 = 34 – 4 All masses to be multiplied by g i.e 10ms–2
4T2 = 30 Taking moment about A to eliminate R1
T2 = 30 ie 7.5N R2 × (8 – 2 – 1.5 ) + 800 × 2
4
= 300×(6 - 5.5) + 400×(3.5 - 2) + 700×(6 -3) + 500× 6
Alternative solution
Taking moment at point H 4.5R2 +1600 = 150 + 600 + 2100 + 3000
2 × (2 + 2) + T2 × 2 = T1 × 2 + 4 × ( 2 + 2 ) 4.5R2 = 5850 – 1600
8 + 2T2 = 2T1 + 16 R2 = 4250
2T2 – 2T1 = 8 ------(1) 4.5
Resolving the forces vertically = 944. 44N
T1 + T2 = 2 + 5 + 4 Taking moment about B to eliminate R2
T1 + T2 = 11 R1 × (8-2-1.5) + 500 × 1.5
T1 = 11 – T2 -------(2) = 800(8 -1.5 ) + 300(5.5-1.5) + 400(8-3.5 -1.5 ) + 700(3 -1.5)
Substitute T1 into (1)
2T2 – 2(11 – T2 ) = 8 4.5R1 + 750 = 5200 + 1200 + 1200 + 1050
2T2 – 22 + 2T2 = 8 4.5R1 = 8650 – 750
4T2 = 30 7900
R1 =
T2 = 30 i.e 7.5N 4 .5
4 = 1,755.56 N
1997/36 Alternative solution
A uniform beam PQ of length 16m and weight 30N Resolving along components (vertical)
rests horizontally on supports P and Q at its ends. If an R1 + R2 = 800 + 300 + 400 + 700 + 500
additional weight 24N is suspended at a distance 2m R1 + R2 = 2700 ------ (1)
from Q, Find, in N, the reaction at P.
Taking moment about CG
A 38 B 36 C 27 D 18 E 16
R2 × ( 8 –3.5 – 1.5 ) + 800 × 3.5 + 300[5.5-(8-3.5)]
Solution
R1 R2 = R1(3.5 – 2 ) + 700 × (4.5–3 ) + 500 × 4.5
3R2 + 2800 + 300 = 1.5R1 + 1050 + 2250
3R2 – 1.5R1 = 3300 – 3100
8m 8m 3R2 – 1.5R1 = 200 -----(2)
P Q
2m From (1) R2 = 2700 – R1 put into (2)
3(2700 – R1) – 1.5R1 = 200
8100 – 3R1 – 1.5R1 = 200
30N 24N
We are asked to find R1 8100 – 200 = 4.5R1
Then to eliminate R2, we take moment at Q  R1 = 7900
4 .5
R1 × (8 + 8) = 30 × 8 + 24 × 2
= 1755.56N
16 R1 = 240 + 48
Substituting R1 value into (1)
288
R1 = R2 = 2700 – 1755 .56
16 = 944.44N
= 18 N (D) 2000/16b (Nov)
A uniform beam of length 18m and mass 30kg rests
2000/17 b horizontally on supports P and Q, which are 1m from each
A non-uniform beam XY of mass 40kg and length 8m rests end. A mass of 2kg is placed at a point M where PM = 12m
horizontally on two supports A and B ( with its centre of
Find the magnitude of the reaction at Q (Take g = ms–2)
gravity at a point 3.5m away from X). Objects of mass 80kg
and 50kg are suspended at X and Y respectively. Two other Solution
R1 R2
objects of masses 70kg and 30kg are suspended at distance 12m
M
3m and 5.5m from Y. If XA = 2m and BY = 1.5m, find the P CG Q
reactions at A and B. (Take g = 10ms–2 ) X
1m 8m 8m 1m
Y

Solution
R1 R2
2kg
X A CG B Y 30kg
2m 1.5m Taking moment about P to eliminate R1
3.5m
40kg 3m
300 × 8 +20 × 12 = R2 × (8 + 8)
80kg 50kg
70kg 2400 + 240 = 16R2
5.5m
30kg 2640 = R2
16
R2 = 165N
379
2003/18 b (Nov) counter eg non uniform units (i) Find, in m the position of the centre of gravity
A uniform beam PQ, 80cm long and of weight 60N, (ii) Calculate, in kg, the mass of the rod.
rests on a support at point X, where PX= 30cm. A (Take g = 10ms–2)
mass of mkg is dropped at P so as to keep the system in Solution
200N 300N
equilibrium.
(a) Calculate the:
X CG Y
(i) value of M, (ii) reaction at X P
2m 8m
Q
(b) The weight at P is removed and a force of 50N is
applied vertically upwards at a point H. If the system
T
is kept in equilibrium by the forces, Find in cm,HQ W
(Take g = 10ms–2) (i) Taking moment about P
Solution W × T = 200 × 2 + 300 × ( 30 – 8 )
R1 W × T = 400 + 6600
W × T = 7000 -------- (1)
40cm Resolving along the vertical components
X
P Q 200 + 300 = W
30cm 500 = W
Substituting W value into (1)
mkg 60N 7000
T=
500
Taking moment about x = 14m
M × 10 × (30 × 0 .01) = 60 × (10 × 0.01)
3m = 6 weight ( w)
(ii) Mass of rod =
m = 6 i.e 2N g (10ms  2 )
3
converting from newtons to kg 500
= = 50kg
But mkg = 2 i.e 0.2 kg 10
10
(ii) R1 2005/18 b (Nov) Non – uniform rod
Taking moment about P A non-uniform rod BC of length 2 metres and mass 3kg has
60 × ( 40 × 0.01) = R1 × ( 30 × 0.01 ) its centre of gravity at a point G, 0.8m from B. the rod rests
24 = 0 .3R1 on two supports at M and N, which are 0.3m and 1.6m from
i.e R1 = 24 B respectively.If the system is in equilibrium, under the
0 .3 action of these forces, calculate the reactions at M and N
= 80N (Take g = 10ms–2)
(b) 50N Solution
HQ R1 R2
R1
M CG N
B C
0.3m 0.4m
X 40cm
P Q 0.8m
H
3kg
60N Taking moment about M
Taking moment about Q R2 × (1.3 ) = 30 × (0.8 – 0.3 )
50 × HQ × 0.01 + 80 × (50 × 0.01) = 60 × (40 × 0.01) 1.3R2 = 15
0.5HQ + 40 = 24 15
0.5HQ = 24 – 40 R2 =
0.5HQ = –16 1 .3
= 11.538 N
16
HQ = – Next, Taking moment about N
0 .5 R1 × ( 2 – 0.3 – 0.4 ) = 30 × ( 2 – 0.4 – 0.8 )
HQ = –32m 1.3 R1 = 24
But in cm HQ = 3200 CM 24
R1 =
1 .3
2003/18b good counter eg = 18.46N
A non-uniform rod PQ of length 30m, rests horizontally Alternatively
Resolving along vertical component
on two supports X and Y such that PX = 2m and
R1 + R2 = 30
YQ = 8m. The reactions at X and Y are 200N and R1 = 30 – 11.538
= 18.46N
300N respectively.
380
2006/18b 5200
T2 =
A uniform plank PQ of length 10m and mass m kg rests 20
on two support A and B where PA = BQ = 1m. A T2 = 260 N
load of mass 8kg is place on the plank at point C such Substitute T2 value into (1)
T1 = 600 – T1 becomes
that AC = 3.5m. If the reaction at B is 100N T1 = 600 – 260
Calculate the = 340N
(i) value of m;
2003/12a non-uniform units
(ii) reaction at A . (Take g = 10ms–2)
AB is a uniform bar of length 100cm and weight 24N. C and
Solution D are two points on the bar such that AC =30cm and BD =
R1 100N
30cm. It is supported horizontally by two vertical strings,
A C 5m B one attached at point C and the other at B. A load of weight
P Q
1m 3.5m 1m 3N is suspended from A and another load of weight 10N is
suspended from D. Find the tension in each string.
( Answer in 2 d. p )
8kg mkg Solution
All masses converted to weight just by times 10
Taking moment about A to eliminate R1
80 × 3.5 + 10m × ( 5 – 1 ) = 100 × ( 10 – 1 – 1 )
280 + 40m = 800
40m = 800 – 280
520
m=
40
m = 13kg
( ii ) Resolving along vertical components Resolving along the vertical components
R1 + 100 = 80 + 130 T1 + T2 = 3 + 24 + 10
R1 = 210 – 100 T1 + T2 = 37 ------ (1)
= 110N Taking moment about C to eliminate T1
Alternative ( ii ) Reaction at A i.e R1 Care must be taken to covert cm to M
Taking moment about B T2×[(100–30) × 0.0 1] + 3×(30 × 0.01) =
R1×(10 – 1 – 1) = 130(5 – 1) + 80[10 – (3.5 + 1 ) – 1] 24×[ (50 – 30) × 0.01] + 10×[(100 – 30 –30) × 0.01]
8R1 = 520 + 360 0.7 T2 + 0.9 = 4.8 + 4.0
8R1 = 880 0.7 T2 = 8.8 – 0.9
880 0.7 T2 = 7.9
R1 = i.e 110 N
8 7 .9
T2 =
2009/18b 0.7
A uniform beam XY of mass 10kg and length 24m is
 11.29 N to 2dp
hunged horizontally from a cross bar by two vertical
Thus, from (1) T1 = 37 – T2 becomes
inextensible strings, one attached to X and the other at a
T1 = 37 – 11.29
point M, 4m away from Y . A mass of 50kg is
suspended at a point N which is 8m from X. If the  25.71 N to 2dp
system remains in equilibrium calculate the tensions in
the strings. (Take g = 10ms–2) 2004/13a Neco Exercise 31.4
Solution A uniform bar AB of length 100cm and weight 20N is
supported horizontally by two vertical string at points
T1 T2 C and B such that AC = 30cm. If loads of 3N and 10N
are placed at points A and D such that BD = 40cm, find the
tension in each string.

Resolving along the vertical components


T1 + T2 = 500 + 100
T1 + T2 = 600 ------- (1)
Taking moment about x, to eliminate T1
T2 × ( 24 – 4 ) = 500 × 8 + 100 × 12
20 T2 = 4000 + 1200
20 T2 = 5200

381
Chapter thirty two
Rough surface
Forces acting at inclined or horizontal planes Rough surface has friction
Problems on force along rough horizontal planes
Definition of terms
Friction is simply the force that opposes motion 1992/37(Nov)
A body of weight wN rests on a rough horizontal plane. If
Rough surface : here friction F opposes motion the coefficient of friction is , find the least force acting at
Smooth surface: here motion is free from friction F an angle of  to the horizontal, that would move the body.
R or N is the normal reaction between the two surfaces A  w Sin B  w tan C w cos +  Sin
Coefficient of friction  = F D w E w
R Cos  –  Sin Cos +  Sin
Solution
When sliding is taking place or about to take place N
F=R P
Forces acting at horizontal planes may be classified in to
two F
Non-inclined force P Inclined force P
N N
P W
At the point of moving F = N
F P Resolving the force P along the components
F
fx = 0
F – Pcos  = 0
F = P cos
W W N = P cos -------(1)

Forces acting at an inclined plane have both fy = 0


horizontal and vertical components PSin + N – W = 0
They may be classified into two types : PSin + N = W -------(2)
But from (1) N = Pcos put into (2)
Body prevented from sliding has frictional force F and

applied force P in same direction
PSin + Pcos = W
N P
F 
Multiply through by  to clear fraction
 PSin + Pcos = W
Factor out P at the RHS
Wcos P( Sin + cos ) = W
in
Ws P= W (E)
Sin + cos
W 2005/7 Neco
While pulling a log of wood, a boy exerts a force of 40N at
When the “body is about to move up the plane”
an angle of 350 to the horizontal. Find the effective force
has frictional force F and applied force P in opposite
pulling the log along the ground
direction
A 2.768N B 3.277N C 32.77N D 327.7 E 428N
N P Solution
N P(40N)

F 350
Wcos
in
Ws
F

W W
P has vertical and Horizontal components but the effective
For better understanding we shall treat the forces acting force pulling along the ground
at inclined or horizontal plane under rough and smooth = Pcos
surfaces = 40 Cos 350
 32.77N ( C )

382
2000/16a ( Nov) 2000/38 Neco
A force of magnitude P Newtons applied at an angle 300 A block of mass 15 kg rests on a horizontal floor, its
to the horizontal, to a particle of mass 5kg placed on a coefficient of friction is 0.45. What minimum force is
rough horizontal plane causes it to move. Given that the required to just make the block move when pulled
coefficient of friction is 0.2, calculate P. horizontally? (Take g = 10ms–2)
(Take g = 10ms–2) A 0.675N B 6.75N C 67.5N D 675N E 6750N
Solution Solution
N N
P

300 F P
F

15g
W
P has vertical & horizontal components The block is at the point of moving when friction is limiting,
fx = 0 i.e Resolving along the horizontal component we say the block is at limiting equilibrium
F – P Cos300 = 0 P = F
But at the point of moving F = N But F = N
N – P Cos300 = 0 Thus P = N
P Cos300 = N = 0.45 × 15 × 10
P Cos300 = 0.2N = 67.5N ( C )
0.866P = 0.2N ------( 1 )
Problems on force along rough inclined planes
fy = 0 i.e Resolving along the vertical component 1994/40(Nov)
P Sin 300 + N – W = 0 ( W = 5 ×10) A body of weight W rests on a rough inclined plane whose
0.5P + N = 50 -------(2) inclination to the horizontal is . The body is prevented from
from ( 1 ) N = 0.866P sliding down the plane by a force P acting parallel to the
0.2 plane. If the frictional force is F, find in Newton, the net
N = 4.33P put into (2) force acting up the plane
0.5P + 4.33P = 50 A F+P B W Sin C F + P + Wcos
4.83P = 50 D F + P + WSin E F + P – WSin
P = 50 Solution
4.83 “Body prevented from sliding diagram has F and P in the
P = 10.35N same direction”
If the block slides down the plane, frictional force will act in
opposite direction
N P
F

2004/42 Neco ( Dec)


A force P is 46.67N. If it is inclined to the horizontal at
an angle of 500 , find its horizontal component. Wcos
A 30N B 33N C 36N D 47N E 56N in
Ws
Solution
N
P W

Net force acting up the plane


500
F P + F – Wsin ( E )

1994/8(Nov)
A particle of mass 10kg is placed on a rough plane inclined
W at Sin–1(3/5) to the horizontal. A force 80N acts on the
fx = 0 i.e Resolving along the horizontal component particle in a direction parallel to and up the plane. Calculate
F – Pcos 500 = 0 the coefficient of friction between the plane and the particle
F = Pcos 500 when it is just about to move up the plane.
= 46.6 × 0.6428 (Take g = 10ms–2)
 30.00 N ( A ) Solution
Key “just about to move up the plane” diagrammatically

383
N (P) 1995/38 (Nov) coefficient of friction
80N A body of mass 10kg placed on a rough inclined plane, at
angle 300 to the horizontal, slides down at a steady velocity.
Find the coefficient of friction between the body and the
7 plane.
g sin3 10g Cos37
10 F
37 A 3 B 1 C 3/5 D3 E 1 /2
3
Solution
10g
Coefficient of friction i.e  = tan 
When the particle is just about to move up the plane = tan 30
P – ( 10g Sin370 + F ) = 0 = 1 (B)
P = F + 10g Sin370 ---- 3
(1)
Also F = N ( friction is limiting) 2002/46 Neco coefficient of friction
=  10g cos370 A certain body with mass ‘M kg’ is placed on a plane
Substitute F into (1) inclined at 300 to the horizontal. What is the coefficient of
80 =  100 cos 370 + friction of the body as it moves on the plane?
100sin370 A 0.905 B 0.866 C 0.707 D 0.677 E 0.577
80 = 79.86 + 60.18 Solution
79.86 = 80 – 60.18 Coefficient of friction  = tan 
 = 19.82 = tan 300
79.86 = 0.577 ( E )
 0.25 1999/17 b ( Nov)
1998/8(Nov) A block of mass 10kg is placed on an inclined plane. If the
A particle of mass 5Kg, placed on a rough plane inclined angle of inclination to the horizontal is 300, calculate:
at 300 to the horizontal is acted upon by a force of 40N in ( i ) the frictional force between the block and the plane
a direction parallel to and up the plane. If the particle is ( ii ) the coefficient of friction
3 where Solution
just about to move up the plane, show that  =
5 N F
–2
 is the coefficient of friction (Take g = 10ms )
Solution
“At the instant when the particle is just about to move
0
up the plane” diagrammatically sin3 mg cos30
(P) mg 30
N 40N

mg
fx = 0
30
g sin 5g Cos30 F = mg Sin 300
5 F
= 100 × 0.5
30
= 50N
5g ( ii ) Coefficient of friction  = tan 300
= 0.577
Resolving parallel to the plane
P – F – 5g Sin 300 = 0 1999/17 c ( Nov)
P = F + 50Sin 300 ------( 1 ) If the angle of inclination of the plane in 17(b) is
But friction is limiting here too Sin–1  1  to the horizontal, calculate, correct to 2
F= N  2
=  5g Cos 300 significant figures, the acceleration of the block
= 50 Cos 300  (Take g = 10ms–2)
Substitute F value into ( 1 ) Solution
P = F + 50Sin 300 becomes F = ma
40 = 50 Cos 300  + 50Sin 300 But F = mg Sin 450
1 ( note that  is changed from 30 to 45)
40 = 50 × 3 ×  + 50 ×
5 2
mg Sin 450 = m a
40 – 25 = 25 3  cancel out m
3 10 × 0.7071 = a
 = Rationalizing 3 QED Thus a  7.1 m/s2 to 2 s.f
5 3 5
384
Smooth planes no friction and the plane by
Acceleration down a slope on inclined smooth plane. Resolving along vertical component  fy = 0;
a = g sin Ө N + 600 sin 300 –50g = 0
The body moves freely there is no friction and should N + 600 sin 300 = 50 g
be noted in diagram sketching N = 500 – 600  1
2
1994/32
An object slides down a smooth plane inclined at = 500 – 300
sin–1(1/3) to the horizontal taking g = 10ms-2 , calculate the = 200 N (A)
acceleration of the particle. 2000/16(a)
A force of 15N acts at an angle of 600 on a body of mass 3kg
A.1ms-2 B.31/3 ms-2 C.4 5 ms-2 D.10ms-2 E.30ms-2 initially at rest on a smooth horizontal plane. Find its
Solution momentum after moving through a distance of 3metres.
Acceleration down a smooth in inclined plane Solution
= g sin Ө N
15N
= 10  sin 19.47
= 3.33m/s2 (B) 600
2004/18a
A body, placed on a smooth plane inclined at an angle of
sin–1(1/4) to the horizontal, is projected up the plane with a
speed of 3ms–1. Calculate in metres, the distance covered by 3g
the body before coming to rest. [Take g = 10 ms –2] F = 15N m = 3kg u = 0 , s = 3m v = ?
Solution Momentum = mv
v = 0, u = 3ms–1 , s = ? We find v from v2 = u2 + 2as (a is not given in the question)
Using v2 = u2 – 2as But F = ma
But a = g sin Ө Here the horizontal component of F = 15 cos 600 i.e 7.5N
= 10 × 1 i.e 2.5 ms–2 7.5 = 3a
4
a = 2.5ms–2
Thus, 0 = 32 – 2 × 2.5 × S i.e 5S = 9
Next v2 = 0 + 2 × 2.5 × 3
S = 9/5 i.e 1.8m v2 = 15
2005/24 Neco v = 3.87 ms–1
A young boy slides from rest down a smooth plane Thus, Momentum = mv
inclined at an angle of 300 to the horizontal at a time = 3 × 3.87
interval of 5S. Calculate the distance covered by the boy. = 11.62kg ms–1
[Take g = 9.8m/s2] 2005/12a Neco
A 1.250m B 6.124m C 61.250m D 612.500m E 6125.000m A block of wood of mass 9kg rests on a smooth horizontal
Solution table. A force of 98.1N acts on the wood upwards at an angle
u = 0, t = 5 , a can be gotten Ө0 to the horizontal so that the wood moves with an
Using S = ut + 1/2 at2 acceleration of 2/3g m/sec2 (g = 9.81m/s2)
Acceleration down a slope on inclined smooth plane. Calculation Ө and find the magnitude of the normal reaction
a = g sin Ө of the plane on the wood.
= 9.8 Sin300 i.e 4.9 ms–2 Solution
R
Thus, S = ut + 1/2 at2 98.
1N
= 0 + 1/2 × 4.9 × 52
= 61.25m (C)
2000/33
A body of mass 50kg is pulled along a smooth horizontal 9g
plane by a force of magnitude 600N at an angle of 300 to F = ma ( Here force along the horizontal)
the horizontal. Find normal reaction between the body 98.1 Cos Ө = 9 ( 2/3 × 9.81)
and the plane. (Take g = 10ms–2)
98.1 Cos Ө = 9 ( 2/3 × 9.81)
A. 200N B. 250N C. 520N D. 800N
98.1 Cos Ө = 58.86
Solution
N Cos Ө = 58.86 i.e 0.6
N
600 98.1
Ө = cos–1 0.6 = 53.130
300 Resolving along the vertical fy = 0
R + 98.1Sin Ө – 9 = 0
R + 98.1Sin 53.13 – 9 = 0
R = 9 – 78.5
50g
R = – 69.5N
We find the normal reaction between the body Thus, Magnitude of R = 69.5N

385
1996/39 (Nov) 2000/18 (a)
A block of mass 0.5kg, placed on a smooth plane An object of mass 5kg slides down a smooth plane inclined at
inclined at angle 450 to the horizontal, is kept in angle at angle 250: If the object starts from rest, find;
equilibrium by a force PN acting on the block at 600 to (i) its velocity after traveling 3m,
the plane. Find the value of P. (Take g = 10ms–2) (ii) its momentum 3m from the starting point,
A. 5 3 B5 6 C5 D5 3 E5 (iii) the force causing it to move.
2 2
Solution
2 2
( i ) v = ? u = 0 , s = 3m
Solution
we find velocity using v2 = u2 + 2as
P(N) Acceleration down a slope on smooth inclined planeis
a = g sin 
600 = 10Sin250 i.e 4.23ms–2
450 Thus, v2 = 0 + 2 × 4.23 × 3
45 0 v2 = 25.38
v = 5.04 ms–1
0.5g
The component P up the plane is equal to the ( ii ) Momentum = mv
component down the plane of weight 0.5g = 5 × 5.04
= 25.2Kg ms–1
P cos 600 = 0.5g cos 450
1 (iii) Force causing it to move = ma
0.5  10 
2 = m g sin 
P=
1 = 5 ×10Sin250 = 21.13N
2
1
= 10 =5 2 (E) 2003/18a
2 A body of mass 20kg resting on a smooth plane inclined at
2000/17(a) angle 300 to the horizontal force of magnitude PN. Calculate
A body of mass 8kg placed on a smooth plane inclined at the;
angle 300 to the horizontal, is acted on by an upward force (i) Value of P,
of magnitude 50N at angle 450 to the line of greatest (ii) Reaction of the body on the plane [Take g = 10 ms–2]
slope. Solution
Find the: P
N
(i) Acceleration of the body
(ii) Magnitude of normal reaction [ Take g = 10 ms–2 ]
Solution
5)
N)

os4
sin30
P(50

c 20g Cos30
( 50 20g 30
Px
N 450
20g
( i ) P = mg Sin 
0 = 20 × 10 × 0.5
sin3 8g Cos30
8g = 100N
30
( ii ) N = mg cos
8g = 20 × 10 × 3
2
( i ) a can be gotten from F = ma since mass 8kg is given = 173.2N
From the diagram the resultant force = 8g sin30 – Px
= 8 × 10 sin 300 – 50 cos 450 2005/18b Exercise 32.1
= 40 – 35.36 A particle rests on a smooth plane inclined at 400 to the
= 4.64N horizontal.
Thus, F = ma ( i ) Calculate, correct to 3 significant figures, the
4.64 = 8 a acceleration with which it moves down the plane
a = 0.58ms–2 ( ii ) If the particle is now projected up the plane with a
speed of 12ms–1 , calculate the distance it will cover
( ii ) R = mg cos 300 before coming to rest and the time it takes to cover the
= 8 × 10 3 i.e 69.28N distance. [Take g = 10 ms –2]
2
386
Chapter thirty three 2010/13a Neco
Motion A car starts from a point A and accelerates uniformly for 5
Definition of terms minutes before it reaches a speed of 27.8ms–1. It then moves
Motion : any form of movement ; circular movement as with the same speed for 20minutes and then retards
in the case of the earth in its orbit, straight line motion uniformly for 4 minutes before coming to a rest at a point B.
as in cars moving over a straight short distance, free Use a velocity – time graph of the car to calculate the:
falling bodies as in the case falling mango from its tree etc. ( i ) distance between A and B;
( ii ) Average speed of the car;
Speed : it is the rate of change of distance with time ( iii ) Acceleration and retardation;
Speed = distance m/s ( iv ) time taken to cover three quarters of AB.
Time Solution
Average speed = total distance travelled m/s The unit of the velocity given should be our guide to
Time taken conform with time unit for the graph.
Velocity: this is the rate of change displacement Here velocity (m/s); so we change time from the given
with time minutes to seconds
velocity = displacement m/s
time
Difference between speed and velocity is that velocity

Velocity
is speed in a specified direction though they use the
same units
Uniform velocity: it occurs when displacement is equal
to time taken (a direct proportionality) A B
U represents initial velocity 5 25 29 Time(min)
25x60s 29x60s Time(Sec)
V represents final velocity 5x60s

Acceleration: this is the rate of change velocity (i) Distance between A and B = Total area under the graph
with time = area of trapezium
1
acceleration = velocity change m/s2 = 2 (1200 +1740)×27.8
time taken = 40,866m
Deceleration/retardation: negative acceleration .Here (ii) Average speed of the car = total distance traveled
velocity decreases at equal rate with time time taken
Uniform acceleration: it occurs when the rate of change = 40866
in velocity with time is constant 1740
Acceleration due to gravity g is force of attraction = 23.49m/s
towards the earth. It affects free falling objects (iii) Acceleration and retardation;
positively and upward thrown objects negatively Acceleration and retardation
= 27.8 = 27.8
We have two types of graphs:
5×60 4×60
Displacement – time (velocity)
= 0.09m/s2 = 0.12m/s2
and velocity – time (acceleration)
(iv) time taken to cover three quarters of AB = 29 × 60 s
Some facts about the graphs are shown below:
= 1740 s
2008/17b
Displacement

A body, moving at 20ms–1 accelerates uniformly at 2.5ms–2


Displacement

y for 4 seconds. It continues the journey at this speed for


cit
elo
v 8 seconds, before coming to rest t seconds with a uniform
rm
fo retardation. If the ratio of acceleration to retardation is 3 : 4
ni
U
(i) Sketch the velocity time graph of the journey;
time (ii) find t;
time A displacement time graph (iii) find the total distance of the journey
A displacement time graph for non - uniform velocity Solution
for uniform velocity
The unit of the velocity given should be our guide to conform with
time unit for the graph. Here velocity (m/s) and time is in seconds
Velocity

on

Re
i
rat

tar

Velocity
le

da
ce
Ac

ti o
n

time 12
Velocity - time graph 4 ts Time(sec)

387
( ii ) find t; Solution
Retardation = 20 Here we find acceleration first then covert to ms – 2
t – 12 Acceleration = 30  0
Retardation can be gotten from 50
The ratio of acceleration to retardation is 3 : 4 = 30 km / h 
Acceleration = 3 5 min 
Retardation 4 Next, we convert the units
2.5 = 3 30  1000
Retardation 4
= 3600
Retardation = 2.5 × 4 i.e 3.33ms–2
3 5  60
3.33 = 20 1000 1
= 30  
t – 12 3600 300
3.33(t – 12 ) = 20 1
= ms – 2 (B)
t = 20 + 39.96 i.e 18s 36
3.33
1996/31– 32 (Nov)
( ii ) Total distance of the journey = Total area under the graph The sketch shows the velocity – time graph of a moving particle
= area of trapezium
1
= 2 (18 + 8) × 20
= 260m
2004/46 & 47 Neco

0
2 4 6 8 10 12 t(s)

31. Find, in m, the total distance covered by the particle


A 90 B 60 C 45 D 35 D 30
0 5 10 15 Time(min) Solution
Total distance covered = total area under the graph
Study the above graph relating velocity to time, and use = Area of the shape(trapezium)
it to answer questions 46 and 47.
= 1 [ (10 – 4 ) + (12 – 0 )]  5
46 Find the total distance travelled in km 2
A5 B 15 C 40 D 150 E 300 1
= [ 6 + 12 ]  5
Solution 2
The unit of the velocity given should be our guide to = 45m2 ( C )
conform with time unit for the graph.
Here velocity (Km/h); so we change time from the 32. The acceleration, in ms– 2 , of the particle in the
given minutes to hours first 4 seconds is
A 83 B 42 C 2.5 D 1.25 E0
Solution
Acceleration in first 4 seconds = slope there
Slope of that region =   v  i.e 5  0
 t 40
 
= 5 i.e 1.25m/s2 ( D)
4
2000/7 (Nov)

Total distance traveled = Total area under the graph


= Area of trapezium
1 10 5
=  60  60   15 
60  0  30
2
1
=  605  15 
60  30
2
= 5km (A)
0 2 4 6 8 10 12 14 16 18 t(s)
47. Find the acceleration in ms – 2 The sketch gives the velocity / time graph of a moving object.
A 1 B 1 C 2 D 5 E 36 (a) Find its average velocity
108 36

388
(b) For how long was the velocity constant Average speed = total distance journeyed
(c) Calculate the total distance covered in the first 6 Time taken
seconds
(d) Calculate the retardation as the object comes to rest. Since graph units for speed and time are the same with the
Solution question’s speed km/h; we can work on without conversion
(a) Average velocity = Total Distance travelled Total distance journeyed = area under the graph
Time taken = Area of trapezium
But total distance travelled is area under the graph 1
= 2 [(2-1) + (4-0)] × 40
= 1 [( 12 – 6 ) + ( 16 – 0 )]  60
2 1
= 2 [ 1 + 4] × 40
= 1 (6 + 16)  60 = 100km
2
Average speed = 100
= 22  30 i.e 660m2
4
Thus, Average velocity = 660 = 41.25m/s = 25km/h( D)
16
(b) At constant velocity, acceleration is zero,
i.e Distance from t = 6 to t = 12 2003/29 Exercise 33.1
Time of constant velocity = 12 – 6 = 6s
Q B
(c) Total distance covered in the first 6 seconds

C
0 2 7 8 Time(sec)

0
The diagram is the speed – time graph of a body.
t(s)
Calculate, in ms – 1 and correct to 3 significant
6

The shape under this section of the graph is triangle


figures, the average speed of the body.
Thus, total distance = Area under the shape
A 12.0 B 12.2 C 15.0 D 15.2
= 1 base  height
2
= 1  6  60 = 180m2
2
(d) Retardation = slope at deceleration point

0 t(s)
12 16

= 0  60 =  60 = –15 m/s2
16  12 4
The minus sign shows retardation
2002/33
SPEED IN KM/H

40

20

0 1 2 3 4 5
TIME IN HOURS
The average speed in Km h–1 , of the journey
represented in the graph above is
A 160 B 100 C 80 D 25
389
Equations of motion 30. At what time, in s, does the body come to rest ?
An object traveling with a uniform acceleration A 2.0 B 3.2 C 3.5 D 2.5
increases its initial velocity u to a final velocity v at a Solution
time t is given by u = 25m/s, a = – 10m/s2 (deceleration) s = 31.25m, v = 0, t = ?
a = v – u ---------( a) Base on the given data and for ease of solving, we apply
t v = u + at
Thus v = u + at 0 = 25 – 10t
1 10t = 25
Similarly we have S = ut + 2 at2
t = 25 i.e 2.5 s (D)
v2 = u2 + 2as 10
These are three equations of motion; we will apply them
as we proceed 2004/28 Neco (Dec)
Straight line and ground surface motions Express an acceleration of 15ms – 2 in km per hour.
2000/20 (Nov) A 60km/h B 54 km/h C 45km/h
A body starts from rest and moves in a straight line with D 9km/h E 4 km/h
an acceleration of 3ms– 2.How far does it go in 20 seconds? Solution
A 30m B 300m C 600m D 1200m 3600
15m/s2 = 15  km/hr
Solution 1000
u = 0, a = 3m/s 2, t = 20, s=? = 54km/hr (B)
Applying s = ut + 1 gt2
2 2004/30 Neco (Dec)
= 0  20 + 1  3  202 A vehicle moves with a uniform acceleration of 4ms – 2 from
2 rest. Calculate the distance travelled after 6 seconds.
= 600m (C) A 10m B 24m C 40m D 72m E 288m
2002/15 (Nov) Solution
A car, moving with a speed of 72Km h– 1 , is brought to a = 4m/s2, u = 0, s = ? t = 6s
rest within a distance of 500m. Find, in ms– 2, its retardation. we apply s = ut + 1 at2
2
A 0.20 B 0.40 C 0.72 D 1.60
Solution = 0 + 1  4  62
Speed in 72 Km h– 1 is standardize as: 2
72 Km h – 1 = 72  1000 ms – 1 = 2  36
3600 = 72m ( D)
= 20ms – 1
Next , the question data: 2004/40 Neco (Dec)
u = 20ms– 1 , v = 0, s = 500m, a = ? A body starts with velocity 36ms – 1 and at the end of the 10th
Applying v2 = u2 + 2as second, its velocity is 96ms – 1 . What is its acceleration?
0 = 202 + 2  a  500 A 50ms – 2 B 26ms – 2 C 9.6ms – 2 D 6.0ms– 2
–2
0 = 400 + 1000a E 3.6ms
– 400 = 1000a Solution
 400 u = 36m/s, t = 10s, v = 96m/s, a=?
= a Applying v = u + at
1000
96 = 36 + a  10
 a = – 0.4 ms– 1 (B) 96 – 36 = 10 a
The minus sign shows retardation 60 = a
10
2005/29 – 30 a = 6.0 m/s2 (D)
A body passes a point with a velocity of 25ms– 1 and
start to decelerate at 10ms– 2 . 2005/ 25 Neco
29.What distance does the body cover before it comes to rest? A car travels with a constant velocity of 12.5ms – 1 for 10s.
A 28.25m B 29.25m C 30.25m D 31.25m What distance does it cover in this time?
Solution A 25m B 125m C 215m D 512m E 521m
u = 25m/s a = –10m/s (since it is deceleration) s = ? v = 0 Solution
Based on the given data, we apply At constant velocity, acceleration a = 0
v2 = u2 + 2as s = ? t = 10s, u = 12.5m/s
0 = 252 + 2 ( –10 )  S 1
We apply s = ut + at2
0 = 625 – 20s 2
20s = 625 = 12.5  10 + 0
s = 625 = 125m (B)
20
= 31.25m (D)

390
2002/41 Neco 2004/4 Neco (Dec)
How long does it take for a particle to increase from A car starts from rest and moves with a uniform acceleration
8m/s to 10m/s in 80m ? of 10m/s2 for 15 seconds on a straight road. Calculate the
A 10 seconds B 91/9 seconds distance it covers in the last two seconds of its motion.
8
C 8 /9seconds D 7 7/9 seconds E6 Solution
seconds Distance covered in the last two seconds;
Solution s at t = 13 seconds
T = ? u = 8m/s v = 10m/s, s = 80m and s at t = 15 seconds
Based on the given data, we have to find acceleration Then we subtract.
along with time. Thus we apply the two equations that u = 0, a = 10m/s2, s = ?,
has both variable of a and t t = 15
v = u + at applying s = ut + 1 at2
10 = 8 + at -----------(1) 2

and s = ut + 1 at2 s = 0 + 1  10  152


2 2
80 = 8t + 1 at2-------- (2) = 1125m
2
From (1) 10 – 8 = at Next, S at t = 13 seconds
Since we are interested in time t we eliminate a ; 1
S = 0 +  10  132
2 2
Thus a =
t = 845m
Next, Substitute a value into (2) Difference in Distance = 1125 – 845
= 280m
80 = 8t + 1  2  t2 2004/24 Neco (Dec)
2 t
Find the time taken if the speed of a train increased from
80 = 8t + t 72km/h to 90km/h for 234 km.
80 = 9t A 18s B 9s C 8s D 4s E 2s
 t = 80 i.e 8 8/9 seconds (C) Solution
9 First, we covert to SI units
2000/4 Neco u = 72km/h becomes 72  1000 m/s (20m/s)
The speed of a car increased from 40 km/hr to 3600
80km/hr within a distance of 0.8 km. calculate 1000
v = 90km/h becomes 90  m/s (25m/s)
(i) its acceleration in m/s2 3600
(ii) the time taken in seconds. s = 324km becomes 324  1000 m
Solution t=?
First, we convert the given data to SI Try any of the two equations involving time,
u = 40km/hr becomes 40  1000 m/s i.e 11.11m/s Next we need acceleration
3600
v = u + at
v = 80km/hr becomes 80  1000 i.e. 22.22m/s
3600
25 = 20 + at ---------(1)
s = 0.8km becomes 0.8  1000 m i.e. 800m 1
s = ut + at2
(i) a=? 2
Based on the given data, we apply 1
324000 = 20t + at2 ----------(2)
v2 = u2 + 2as 2
22.222 = 11.112 + 2  a  800 Since we are after t, we eliminate a
22.22 – 11.112 = 1600a Difference of two squares at LHS
2 From (1) 25 – 20 = at
(22.22 – 11.11)(22. 22 + 11.11) = 1600a 5
= a
(11.11) (33.33) = 1600a t
370 .2963 Substituting a into (2)
= a
1600 1 5 2
324000 = 20t +  t
a ≈ 0.23m/s2 2 t
( ii ) t=? 5
Applying 324000 = 20t + t
2
v = u + at
40 t  5t
22.22 = 11.11 + 0.23  t 324000 =
22.22 – 11.11 = 0.23t 2
11 .11 324000  2 = 45t
= t 324000  2
0.23 = t
t ≈ 48.30s 45
14,400s = t when converted to hours it gives t = 4hrs

391
2010/35 Neco Exercise 33.2 0 = u 2 – 600
A bus traveling on a straight road at 25m/s accelerates 600 = u2
uniformly at 5m/s2 for 2 seconds. Find its speed in km/h
A 12.6 B 35.0 C 126.0 D 250.0 E 350.0 u = 600 i.e. 6100
= 10 6 (C)
2010/37 Neco Exercise 33.3
A car traveling at 40m/s was brought to a rest with 2000/13 Neco greatest height
uniform retardation in 10seconds. Find the distance it A bullet is fired vertically upwards with a speed of 30ms – 1.
traveled in this time. Find the maximum height reached. (g = 10m/s2)
A 4m B 8m C 15m D 150m E 200m A 30m B 45m C 60m D 90m E 900m
Solution
2008/7 Exercise 33.4 u = 30m/s, v = 0 at maximum height a = –g (upward
A car traveling at a velocity of 50kmh– 1 , covers a throw), s =?
distance of 20 km. If it was accelerating at 6 kmh– 2, Based on the given data, we apply
calculate, correct to one decimal place, the time the car v2 = u2 + 2as
took to cover the distance. 0 = 302 + 2 (–10)  s
0 = 900 – 20s
20s = 900
Upward(vertical) motion ; here a = – g
s = 900 i.e 45m (B)
1996/36(Nov) 20
A ball is thrown vertically upwards with a speed of 2004/3 Neco greatest height
40ms– 1 . Find the height of the ball when it is moving at A particle is projected from the ground with a velocity of
10ms– 1 . 45m/s vertically upwards. Find the greatest height reached
A 30m B 50m C 60m D 75m E 150m (Take g = 9.8 ms – 1 )
Solution A 2.3m B 4.6m C 103.3m
u = 40m/s, s = ? v = 10m/s; a = – g (upwards throw) D 220.5m E 441m
Applying v2 = u2 + 2as Solution
102 = 402 + 2  (-10 )  S u = 45m/s, a = –g,
100 = 1600 – 20 S v = 0 (velocity at greatest height) s=?
20S = 1600 – 100 Applying 2 2
v = u + 2as
S = 1500 0 = 452 + 2  (–9.8 )  s
20
0 = 2025 – 19.6s
S = 75m (D)
19.6s = 2025
s = 2025
1995/37 (Nov) 19 .6
A stone is thrown vertically upwards with a velocity of ≈ 103.3m (C)
20ms– 2. Find, in ms–1 its velocity when it attains a
height of 10m. ( g = 10ms– 2) 2003/2 Neco greatest height
A 14.14 B 20.00 C 24.49 D 28.28 E 34.14 A particle is thrown upwards with a velocity of 50m/s. Find
Solution the time taken to reach the highest point, ( Take g = 10m/s2).
u = 20m/s, v = ?, s = 10m, a = – g (upward throw) A 1 sec. B 5 secs C 6 secs
Using v2 = u2 + 2as D 8 secs. E 9 secs.
v2 = 202 + 2  (-10)  10 Solution
= 400 – 200 a = –g (upward throw), u = 50m/s t = ?
v2 = 200 v = 0 (at highest point of upward thrown object)
v = 200 Based on the given data, we apply
v = u + at
≈ 14.14 m/s (A)
0 = 50 + (–10 )  t
1999/29 (Nov) greatest height 0 = 50 –10 t
An object is projected vertically upwards from a point 10t = 50
on the ground with a velocity of Vms – 1, where V is a t = 50 i.e 5 secs (B)
10
constant. If the maximum height reached by the object
is 30m, Find V [ Take g = 10ms – 1 ] 2003/15 Neco greatest height
A particle is projected from the ground with velocity
A 3 10 B 10 3 C 6 10 D 10 6
40ms - 1 vertically upward, what is the greatest height
Solution reached, ( Take g = 10ms – 2 )?
u = V here final velocity v = 0, s = 30m, A 800m B 160m C 100m D 80m E 40m
a = -g(upward throw) Solution
Using v2 = u2 + 2as u = 40m/s s = ? a = –g (upward throw)
0 = u2 + 2  (- 10) 30 v = 0 ( At greatest height of upward thrown object )
392
Based on the given data, we apply Solution
v2 = u2 + 2as ( i ) S= ?, v = 0(At maximum height), u = 26m/s, g = 10ms-2
0 = 402 + 2  (–10)  s The motion equation that fits in is v2 = u2 + 2as which becomes
0 = 1600 – 20s v2 = u2 –2gs ( g opposes motion)
20s = 1600 0 = 262 – 2 × 10 × S
s = 1600 S = 262
20 2 × 10
= 80m (D) S = 33.8m

2007/18 Neco greatest height ( ii ) Time of max height, t = ?, v = 0, u = 26, g =10m/s2


A body is projected vertically upwards with velocity of ( not time of flight)
9.78ms – 1 . How high does it travel before it comes to The motion equation that fits in is v = u + at but here it is
rest momentarily at the top of its motion? v = u – gt( g opposes motion)
A 4.78m B 4.89m C 47.8m 0 = 26 – 10t
D 48.9m E 97.8m t = 26 i.e 2.6secs
Solution 10
a = – g , u = 9.78m/s, v = 0 (Highest point) s = ? ( iii ) Time the body hits the ground = Time of flight
We apply v2 = u2 + 2as = 2t
0 = 9.782 + 2  (–10 )  s = 2 × 2.6
0 = 9.782 – 20s = 5.2secs
20s = 9.782
95.6484 ( iv ) Velocity of the body at the point of hitting the ground
s = v = ? u = 0 (at maximum height), g =10m/s2 ,
20 t = 2.6 secs (time from maximum height to hit the ground)
≈ 4.78 m The motion equation that fits in is
v = u + at ( g does not opposes motion)
v = 0 + 10 × 2.6
Other cases of Upward(vertical) motion = 26m/s
2000/31 2002/36
The maximum height of a ball projected vertically A body is projected vertically upward from a point on the
upward is 125m. How long does it take the ball to ground, with a speed of 40ms-1. How long will it take the
return to its level body to return to its point of projection?
of projection? [Take g = 10ms– 2] (Take g = 10ms-2)
A 2 1 sec B 10 sec C 20 Sec D 50 sec A 2 seconds B 6 seconds C 8 seconds D 16 seconds
2 Solution
Solution How long will it take the body to return to its point of projection
S = 125m, v = 0( at max height), g = 10ms-2 t = ? = Time of flight i.e 2t
The motion equation that fits in is v = u + at but here it is Here t = ?,( time of maximum height), u = 40m/s,
v = u – gt( g opposes motion) At max height v = 0, g = 10m-2
First we get u from v2 = u2 + 2as which becomes The motion equation that fits in is v = u + at but here it is
v2 = u2 – 2as v = u – gt( g opposes motion)
0 = u2 – 2 10 125 0 = 40 – 10t
2500 = u2 thus u = 50 ms– 1 t = 40 i.e 4secs
Substituting u value into v = u – gt 10
0 = 50 – 10t Thus, T = 2t
10t = 50 i.e t = 5secs = 2 × 4 i.e 8Secs
Thus, time of return to its level of projection = time of flight
= 2t 2003/17
= 2 × 5 i.e 10secs A particle P is project vertically upward from a point on the
ground with initial velocity 40ms-1
(a) Find the range of values of time t seconds, for which
2000/17 (a) the particle is more than 60m from the point of
A body is projected vertically upwards with an initial projection.
velocity of 26ms-1. Find: (b) If after 1 second, another particle Q is projected
(i) The maximum height attained by the body; vertically upward from the same point, with velocity
(ii) the time the body attained its maximum height 55ms-1, find the:
(iii) the time the body hits the ground’ ( i ) Time the two particles meet;
(iv) The velocity of the body at the point of hitting the ( ii ) Distance of the point of their meeting to the
ground. (Take g = 10ms-2) point of their projection;
(iii) Maximum height attained by Q.
(Take g = 10ms-2)

393
Solution v2 = u2 + 2as which becomes
First we find the range of the values of time for which v2 = u2 – 2gs ( g opposes motion)
the particles is at 60m from point of projection 0 = 492 – 2 × 9.8× S
S = 492
2 × 9.8
S = 122.5m
( b ) t = ? u = 49m/s v = 0, g = 9.8ms-2
A B The motion equation that fits in is v = u + at but here it is
60m
v = u – gt( g opposes motion)
0 = 49 – 9.8t
point of projection t = 49 i.e 5secs
Here u = 40m/s, t = ?, h = 60m, g = 10ms – 2 9.8
1
The motion equation that fits in is S = ut + 2 at2 i.e 2006/38
1 A particle is projected vertically upwards from a height
h = ut – 2 gt2 ( g opposes motion) 45 metres above the ground with a velocity of 40ms – 1.
1
60 = 40t – 2 × 10t2 How long does it take to hit the ground?
[Take g = 10ms– 2]
60 = 40t – 5t2
60 = 40t – 5t2
Solution
t = ?, u = 40ms – 1 s = 45m a = + g at the point of hiting the ground
t – 8t + 12 = 0
2
1
( t – 6 ) (t – 2) = 0 factorised Using S = ut + 2 at2
t = 6 Secs or 2 Secs 1
Smaller time 2secs is time at point A and 6 Secs is at B 45 = 40t + 2 × 10t2
Range of time t > 2s and t < 6s i.e 2<t<6 45 = 40t + 5t2
( b ) Initial velocity u is present for both particles 45 = 40t + 5t2
g = 10ms-2 t + 8t – 9 = 0
2

time t1 be the time of meeting ( t + 9 ) (t – 1) = 0 factorised


we apply the motion equation t = 1 Secs or – 9 Secs
1 we accept t = 1secs
For particle P h = 40t1 – 2 × 10t12
1 2003/28 Exercise 33.5
For particle Q h = 55( t1 – 1) – 2 ×10( t1 – 1)2
A body is projected vertically upwards with a speed of
Equating h
20 ms – 1. Find, in seconds, the times when the body is 15 m
40t1 – 5t12 = 55(t1 – 1) – 5( t1 – 1) 2
above its point of projection. [ Take g = 10 ms – 2]
40t1 – 5t12 = 55t1 – 55 – 5(t12 – 2t1 + 1)
A 1 and 3 B 1 and 4 C 2 and 4 D 3 and 4
40t1 – 5t12 = 55t1 – 55 – 5t12 + 10 t1 – 5
40t1 = 65t1 – 60
2009/26 Exercise 33.6
65t1 – 40t1 = 60
A stone is projected vertically upwards with a speed of
t1 = 60 i.e 2.4Secs
10ms-1 from a point 8 metres above the ground. Find the
25
height reached. [ Take g = 10 ms – 2]
1
( b ) ii. From h = 40t1 – 2 × 10t12 A 13m B 15m C 18m D 23m
= 40 × 2.4 – 5 × 2.42
= 96 – 28.8 2003/48 Neco Exercise 33.7
= 67.2m A man threw a stone vertically upwards from a point Q and
( b ) iii. Max height is S. it strikes the ground with a velocity of 8m/s.Find the highest
At max height v = 0, u = 55m/s, g = 10ms-2 point reached above the ground
Using v = u2 + 2as which becomes
2 (take g = 9.8ms-2)
v2 = u2 –2gs ( g opposes motion) A 177m B 188m C 411.6m D 412m E 420m
0 = 552 – 2 × 10 × S
S = 3025 i.e 151.25m 2005/7 Neco Exercise 33.8
2×10 A stone is projected vertically upwards from the top of a cliff
1992/7 ( Nov) 50m high, with velocity 25ms-1. Calculate the time it takes to
A bullet is fired vertically upwards with a speed of reach the ground [ Take g = 10 ms – 2]
49ms-1 from the top of a building 60m high. Taking g =
9.8ms-2 , find the : 2004/25 Neco ( Dec ) Exercise 33.9
( a ) maximum height reached above the ground; A stone is thrown vertically upwards and it returns to the
( b ) time required to reach the height in (a) ground after 10seconds, find the maximum height reached
Solution (take g = 9.8ms-2)
( a ) Maximum height v = 0 and h is S, u = 49m/s, A 490m B 245m C 127.5m D 122.5m E 98m
g = 9.8ms-2
The motion equation that fits in is
394
2004/41 Neco Exercise 33.10 1999/27 (Nov)
A stone is thrown up from a point 45.5m above the An object is dropped from a height of x metres. If it strikes
ground with an initial speed of 43ms. Find the height the ground with a velocity of 24ms – 1 , find x.
after 5 seconds above the ground. (take g = 9.8ms-2) (Take g = 10m/s 2)
A 383m B 337.5m C 138m D 92.5m E 57m A 12.0 B 23.9 C 28.8 D 57.6
Solution
2003/12a Neco ( Dec ) Exercise 33.11 S = x ? U = 0, v = 24m/s a = +g (downward throw)
A ball is thrown vertically upwards and returns to the Based on the given data we apply
ground after 8 seconds. What is the: v2 = u2 + 2as
( i ) initial velocity ? 24 2
= 0 + 2  10  x
( ii ) greatest height reached ? 576 = 20 x
576
2006/8 Exercise 33.12 =x  x = 28.8m (C)
An object is projected vertically upwards with a 20
velocity of 80ms-1. Find the; 2003/10 (Nov)
(a) Maximum height reached; A stone is dropped from the top of a building and reached
(b) Time taken to return to the point of projection. the ground after 2 seconds. Calculate the height of the
(Take g = 10ms-2) building. [ Take g = 10ms- 2 ]
A 20.0m B 19.6m C 16.0m D 14.0m
2009/36 Neco Exercise 33.13 Solution
A stone is thrown upwards with a velocity of 18ms-1. a = +g (downward throw) t = 2s; s = ? u = 0
How high does it reach? Take g = 10ms-2. Based on the available data, we apply
A. 48.6m B. 16.2m C. 12.4m D. 9m E. 4m 1 2
s = ut + at
2
1
Downward(vertical) motion; here a = +g = 0 +  10  22
2
1997/25 = 20.0m (A)
A ball is projected vertically downwards with a velocity 2000/25 Neco
of 20ms– 1 from the top of a hill of height 35m. Find, in An object is dropped from a height of 18 metres. Find the
ms– 1 its velocity just before it hits the ground. velocity with which it hits the ground.
(Take g = 10ms– 2) ( Take g = 9.8m/s2 )
A 16.18 B 21.68 C 33.17 D 37.13 E 45 A 9.4m/s B 18m/s C 18.78m/s
Solution D 36m/s E 37.74m/s
u = 20m/s, s = 35m, v = ? a = +g(downward throw) Solution
Using v2 = u2 + 2as u = 0, v = ? a = +g, s = 18m
v2 = 202 + 2  10  35 Based on the available data, we apply
= 400 + 700 v2 = u2 +2as
v2 = 1100 v2 = 0 + 2  9.8  18
v= 1100 ≈ 33.17m/s ( C ) v2 = 352.8
v = 352 .8 ≈ 18.78m/s (C)
1993/36 (Nov)
A stone is dropped from a building 160 metres high. 2000/39 Neco
Find, in seconds, the time taken to reach the ground A stone dropped from a height of 32m. How long does it
(Take g =10m/s 2) take to reach the ground? (Take g = 9.8ms – 2 )
A 16 B 4 2 C 4 D2 2 E2 A 2.56 seconds B 3.27 seconds
Solution C 9.80 seconds D 25.6 seconds E 32.7 seconds
Out the three velocity equations of Solution
v = u + at , S = ut + 1 at 2 and v2 = u2 + 2as u = 0, t = ? a = +g , s = 32m
2 Based on the given data, we apply
Our problems values are S = ut + 1 at2
S = 160m, t = ? a =10m/s2, u = 0 2
32 = 0 + 1  9.8  t2
Yes s = ut + 1 at2 will apply 2
2 32 = 4.9t2
160 = 0  t + 1  10  t2 32 = t2
2 4.9
160 = 0 + 5t2 32 ≈ 2.56 seconds
160 = t 2 t =
4 .9
5

32 = t Thus, t = 4 2 (B)
395
2000/45 Neco Application of differentiation to motion
A body falls from rest with an acceleration of 3ms – 2 . For any function of displacement S = f(t)
How far has it fallen after 5 seconds ds dv d 2s
A 15m B 25.5m C 37.5 D 75m E 150 m Velocity  and acceleration i.e acceleration 2
Solution dt dt dt
u = 0, t = 5s, a = 3m/s2, s = ? 1996/14(Nov)
Based on the given data, we apply A particle moves along a straight line such that its distance,
S = ut + 1 at2 Scm, from a fixed point on the line after time ts is given by
2 S = t3 + 2t2 + 3t + 4. Find its velocity in ms – 1 at the end
S=0 + 1  3  52 of 3 seconds.
2 A 42 B 58 C 33 E 22
= 37.5m (C) Solution
2002/40 Neco S = t3 + 2t2 + 3t + 4
A body is subjected to fall freely from the top of a
ds
building. What is the height of the building if it takes Velocity  = 3t2 + 4t + 3
5 seconds for the body to touch the ground from rest? dt
(g = 10m/s2) At the end of 3 seconds
A 150m B 135m C 128m D 125m Velocity = 3( 32 ) + 4 (3) + 3
E 75m = 27 + 12 + 3
Solution = 42m/s ( A )
a = +g, s = ? t = 5s, u = 0
1997/ 33 – 34
Based on the given data, we apply
The displacement xm, of a particle from a fixed point after
S = ut + 1 at2
2 time t s is given by
1 x = t3 – 7 t2 + 2t + 5
S = 0 +  10  52 2
2 33. Find the times, when the particle is momentarily at rest.
= 125m (D) 1 1
1
2004/37 Neco (Dec) A1, 2 B 1, 5 C 2, 3 D ,2 E ,
3 3 2 3
An orange fruit drops to the ground from the top of its Solution
tree, which is 25m above the ground. How long does it Momentarily at rest, v=0
take to reach the ground? (Take g = 9.8m/s2)
A 100s B 50s C 2.5s D 2.3s E 2.0s
dx Change in displacement
But v= i.e
Solution dt time taken
a = +g, s = 25m, t=? u=0 dx
Applying
1 2
s = ut + at Thus = 3t2 – 7t + 2
2 dt
25 = 0 + 1  9.8  t2 dx
2 Next, at momentarily rest v  =0
9 .8 2
dt
25 =
2
t  3t2 – 7t + 2 = 0
50 = t t ≈ 2.3s
Factorizing
25  2 = t2 3t – 6t – t + 2 = 0
2
9 .8 9 .8
3t( t – 2 ) –1( t – 2 ) = 0
2001/7 Neco Exercise 33.14 ( 3t – 1 )( t – 2 ) = 0
A ball is dropped from the top of a tower 1200 m high. 3t – 1 = 0 or t – 2 = 0
What is its height above the ground after 15 seconds?
t = 1 or 2 (D)
(Take g = 10m s-2 ) 3
A.75m B.112.5m C. 750m D. 975m E. 1125m 34. Find in ms– 2 , the acceleration of the particle at t = 4
A 31 B 24 C 22 D 17 E 6
2010/5 Neco Exercise 33.15 Solution
A construction worker drops a block from a building v = 3t2 – 7t + 2
60metres high. dv
( a ) Find the velocity of the block Acceleration = i.e 6t – 7
( i ) after 2 seconds ( ii ) when it touches the ground dt
( b ) calculate the time taken by the block to touch the At t = 4
ground ( Take g = 10m/s2) Acceleration = 6(4) – 7
= 17m/s 2 (D)
2009/22 Exercise 33.16
A stone is dropped from a height 39.2m. Determine the
time it takes to reach the ground. [Take g = 10ms – 2 ]
A 1.8s B 2.8s C 4.0s D 14.0s
396
2005/13 2006/16b
The distance S metres a particle covers in t seconds is The height h metres, of a ball thrown into the air is
3 2 + 20t + kt2 , after t seconds. If it takes 2 seconds for the ball
given by S = 2 t2 – 3t . Find in ms–2 the acceleration of
to reach its highest point, find:
the particles ( i ) the value of k:
A4 B3 C2 D 1 ( ii ) its highest point from the point of throw
Solution Solution
Acceleration can be gotten from distance by second The equation is h = 2 + 20t + kt2
differential of S At highest point v = 0
3
S = 2 t2 – 3t dh
 20 + 2kt = 0
ds dt
= 3t – 3 Since it takes 2 seconds to reach its highest point, t = 2
dt
20 + 2k( 2 ) = 0
d 2s 20 + 4k = 0
= 3 ms–2 (B)
dt 2
4k = – 20
k= –5
2004/12 Neco ( ii ) Here we substitute t = 2 and k = – 5 into
An object moves from a position O along a straight line h = 2 + 20t + kt2 becomes
and its distance ‘S’ metres from O after time ‘t’ = 2 + 20(2) + ( –5)(2) 2
seconds is given by : S = 2t3 – 18t2 + 30t + 60, find the: = 22m
( i ) distances of the object from O when it is
momentarily at rest: 2002/25 ( Nov) Exercise 33.17
( ii ) acceleration of the object at those instances The distances S metres, covered by a body in motion at any
( iii ) at what time is the acceleration towards O or away time t seconds is given by S = 120t – 16t2. Find, in metres,
from O ? the distance covered by the body before coming to rest
Solution A 220 B 225 C 360 D 675
We need to know the time, the particle is momentarily
at rest . Here V = 0 2001/12b Neco Exercise 33.18
V can be gotten from distance S by differentiation The distance covered by a particle in time t is given by the
S = 2t3 – 18t2 + 30t + 60 formula s = 4t3 – 3t2 + t – 5. Calculate its acceleration after 3
ds seconds.
V =0  i.e 6t2 – 36t + 30 = 0
dt
t2 – 6t + 5 = 0 2009/33 Neco Exercise 33.19
(t – 1)(t – 5) = 0 factorised The distance of a particle (s) after time (t) is given by
t = 1s or 5s 1
Distances of the object from O when it is momentarily at rest S = t3 – 4t2 + 7t2 – 3.
3
When t = 1
When is the particle momentarily at rest
S = 2(1)3 – 18(1)2 + 30(1) + 60
A. 7 secs only B. 1 sec or 7 secs C. 8 secs only
= 2 – 18 + 30 + 60
D. 1 sec or 8secs E. 7 secs or 8 secs
= 74m
When t = 5 2009/27 Exercise 33.20
S = 2(5)3 – 18(5)2 + 30(5) + 60 The velocity, v ms-1 of a particle moving in a straight line is
= 250 – 450 + 150 + 60 given by v = 3t2 – 2t + 1 at time t seconds. Find the
= 10m acceleration of the particle after 3 seconds.
( ii ) acceleration is
dv
= 12t – 36 A 26ms–2 B 18ms–2 C 17ms–2 D 16ms–2
dt
When t = 1 1996/10 Exercise 33.21
dv The displacement xm from a fixed point of a particle moving
Acceleration = 12(1) – 36
dt in a straight line after time t s is given by
= – 24 ms–2 x = 12t – 15t2 + 4t3. Find its acceleration, in ms-2, after 3s
When t = 5 A 72 B 52 C 42 D 25 E 24
dv
Acceleration = 12(5) – 36
dt
= 24 ms–2
( iii ) At time t = 1 Secs
– 24 ms–2 means deceleration which is acceleration
towards the starting point O
Whereas time t = 5 seconds
24 ms–2 means acceleration away from O
397
Application of integration to motion ( ii ) when x = 0, t = 0
ds x = 2t3 + C1t + C2 becomes
If Velocity  Then S =  dv 0 = 0 + 0 + C2
dt Thus C2 = 0
i.e displacement is gotten from velocity by integration Next, when x = 20, and t = 2
d 2s x = 2t3 + C1t + C2 becomes
and acceleration then Velocity  d 2 s 20 = 2(2)3 + C1(2) + 0
d t2 20 = 16 + 2C1
i.e velocity is gotten from acceleration by integration 4 = 2C1 Thus C1 = 2
2001/12a Neco When t = 5
An object is projected on a straight line from a point O Displacement , x = 2t3 + 2t + 0 becomes
with an initial velocity of 12ms-1. If its acceleration is = 2(5)3 + 2(5) + 0
later given by 6t2 + 4t, calculate = 260m
(i) its velocity after 4 seconds, Velocity , v = 6t2 + 2 becomes
(ii) the distance covered from O after 4 seconds. = 6(5)2 + 2 = 152ms-1
Solution
2003/7 ( Nov) joint case
dv The velocity V ms-1 of a particle moving at any time t
Acceleration  = 6t2 + 4t
dt 1
seconds, is given by V = 3t2 – t3 + 9. Find:
 
Velocity  dv  (6t 2  4t )dt 3
V = 12t3 + 4t2 + k ( a ) the values of t at which the acceleration of the particle is zero
3 2 ( b ) the distance traveled by the particle between the two
V = 4t3 + 2t2 + k instants at which the acceleration is zero
With initial velocity of 12ms-1 ; t = 0 Solution
12 = 4t3 + 2t2 + k (a) We can get acceleration from velocity by differentiation
Thus k = 12 dv
and acceleration is zero implies =0
V = 4t3 + 2t2 + 12 dt
( i ) velocity after 4 seconds, V = 3t2 – 1 t3 + 9
v = 4(43) + 2(42) + 12 3
= 256 + 32 + 12 = 300 ms-1 dv
 6t – t2 = 0
(ii) The distance covered from O after 4 seconds. dt
Velocity  ds = 4t3 + 2t2 + 12 t( 6 – t ) = 0
dt t = 0 or 6
4
( b ) the distance traveled can be gotten from velocity by integration
Distance covered  ds   0
(4t 3  2t 2  12 ) dt
4 ds 1
V = 3t2 – t3 + 9
S = 4t4 + 2t3 + 12t dt 3
4 3 0
Distance traveled between the two instants implies
2 6
S = (44 ) + 3 (43) + 12(4)
= 256 + 42.7 + 48 i.e 346.7m
S=  0
(3t 2  13t 3  9)dt
1 4
2005/16b = t3 – t + 9t 6
The acceleration, in ms-2, after time ts of a particle 12 0
moving in a straight line is 12t. 1
( i ) Find an expression for the displacement x in terms of t. = (6) 3 – (6) 4 + 9(6)
( ii ) If x = 0 when t = 0 and x = 20 when t = 2, find the 12
displacement and the velocity of the particle, when t =5 = 216 – 108 + 54 = 162m
Solution 2009/35 Neco Exercise 33.22
Displacement x can be gotten from acceleration by A particle moves in a straight line. Its velocity (v) is
integration (t2-3t-2)m/s, after t second, the distance of the particle from
First step: we integrate acceleration to get velocity a fixed point on the line is s meters after t seconds. If s = 20
dv when t = 6 seconds what is the formula for s in term of t ?
Acceleration  = 12t A. 1 t5 – 3 t2 – 2t –14 B. 1 t3 – t2 – t + 14
dt 3 2 2

 dv   (12t )dt 1 3 1 3
C. t3 + t –14 D3t3 –2t2 –2t + 14 E t3 – t2 – 2t + 14
v = 6t + C1 2 3 2 3 2
Next step: we integrate velocity to get displacement x 2002/9b Neco Exercise 33.23
dx A boat moves in a straight direction and its velocity after ‘t’
V = 6t2 + C1 seconds is (2t2- 3t + 5)m/s. The distance of the boat from a
dt
fixed point in the same direction is ‘s’ metres after ‘t’
 dx =  (6t  C1 )dt
2
seconds. Find the formula for s in terms of t if s = 14m when
3
x = 2t + C1t + C2 t = 8 secs.
398
Chapter Thirty four 1999/28 (Nov)
Interaction between forces and motion; A body of mass 20kg initially at rest moves along a straight
Collision and Projectiles line under the action of a constant net force of magnitude
100N. Find the distance covered after 4seconds.
A. 40m B. 20m C. 10m D. 5m
Interaction between forces and motion
Solution
Note on some basic terms M = 20kg, u = 0, F = 100N, s = ?, t = 4s v = ?
Change in momentum = mv – mu S =  u  v  t from distance traveled been area of trapezium
 2 
Impulse and momentum are both measured in First we get v from F = ma formula
Newtons seconds Ns and
vu
Change in momentum = Impulse (Ft) F=m  
 t 
Force = ma v0
v u 100 = 20  
F=m  4 
t
100 = 5v
We shall use them along with other equations of motion
100
 v i.e 20ms–1
1994/33 5
Calculate the net force required for a mass of 300kg to Thus, S =  u  v  t
attain from rest, a speed of 12.5ms–1 in 150 seconds.  2 
A. 8N B. 12N C. 25N D. 30N E. 42N
Solution =  0  20   4 = 40m ( A )
 2 
u = 0, v = 12.5 ms–1, t = 150s, m = 300kg, F = ?
F = m  v  u  2000/14
 t  A body of mass 6kg at rest is acted upon by a force of 30N
= 300 × 12.5 – 0 for 0.5 seconds, find the final speed of the body.
150 A. 2.4m/s B. 2.5m/s C. 2.8m/s
= 300 × 12.5 D. 5.2m/s E. 5.5m/s
150 Solution
= 25N ( C ) m = 6kg, u = 0, F = 30N, t = 0.5, v = ?
F = m  v  u 
1997/40  t 
The body of mass 5kg is at rest. Find in N, the constant
force that will give the body a velocity 6.4ms–1 in 30 = 6  v  0 
4 seconds  0 .5 
A. 30 B. 20 C. 12 D. 10 E. 8 6 30  0. 5
30 = v Thus = v i.e 2.5 m/s (B)
Solution 0 .5 6
u = 0, v = 6.4 ms–1, t = 4s, m = 5kg, F = ? 2004/8
v u The speed of a car of mass 500kg is reduced from 20ms–1 to
F=m
t 15ms–1 in 4 seconds by applying the brakes.
(a) Calculate the braking force applied
F = 5  6. 4  0  (b) If the brakes are kept on, calculate:
 4 
(i) the time taken;
= 8N ( E ) (ii) the distance covered, before the car came to rest .
1998/7b Solution.
A force of magnitude F Newton’s acts on a body of (a) m = 500kg, u = 20ms–1, v = 15ms–1, t = 4s, F = ?,
mass 4kg moving at 9ms–1 for 1.5 seconds. If the final vu
velocity of body is 13.5ms–1, find F. F = m 
Solution  t 
m = 4kg , u = 9ms–1 , t = 1.5s , v = 13.5ms–1, F = ? 20 15
= 500 ×
vu 4
F = m  = 625N
 t  (b)
Substituting for the given values ( i ) Using the equation
13 .5  9 v = u + at
F= 4×
1.5 Here final velocity is zero
=4×3 5
0 = 20 – t
= 12 N 4
399
5 (iii) v = 0.83, t = 0.5s, u = 0, S = ?
t = 20 2 2
4 Thus, v = u + 2as will fit into it
20  4 We were not told that the body fell freely(vertically);
t= Thus, t = 16s hence the need to calculate acceleration as g is not applicable
5 F = ma
( ii ) Distance covered 20 = 12a Thus, a 1.67m/s2
v2 = u2 + 2as
Substituting for the given values
 5 v2 = u2 + 2as
0 = 202 + 2 ×  S
 4  (0.83)2 = 0 + 2  1.67  S

0 = 400 –
5
S
0.832
= S
2 2  1.67
5 Hence S ≈ 0.21m to 2 d.p
S = 400
2
2003/3 Neco
S = 160m
A force of 420N pulls a block of mass 80 kg despite a
constant frictional force of 60 N acting between the block
2005/2 Neco
and the ground. What is the acceleration of the block ?
A ball of mass 50kg falls and strikes the floor at a
A 0.75 m/s2 B 4.5 m/s2 C 5.25 m/s2
speed of 20ms–1. It rebounds vertically with a reduced 2 2
D 6.25 m/s E 62.5 m/s
speed of 12ms–1. If the ball was in contact with the floor
Solution
for 0.3s, Calculate the ;
F=ma
(i) Change in momentum,
Here, the resultant force on the block is 420 – 60 i.e. 360 N
(ii) average force exerted on the ball by the floor.
Thus 360 = 80  a
(i) Change in momentum = m (v – u ) 360
= a
50 80
= (20  12 ) a = 4.5 m/s2 (B)
1000
= 0. 4 kg ms–1 2005/6 Neco (Dec)
(ii) F = ma What acceleration will be produced if a horizontal force of
But a = v – u 8N acts on 60kg mass against a resistance of 0.8N ?
t Solution
12  20 The formula that relates force to acceleration is F = ma
= But the resultant force there is 8 – 0.8 i.e 7.2 N
0.3
8 Thus 7.2 = 60  a
= ms–2 7 .2
0 .3 = a
50 8 60
F=  we converted mass in Kg to g 0.12 m/s2 = a
1000 0 .3 2003/4 (Dec) Neco
= 1.33N A force of 64N is applied to a particle of mass 16 kg. If the
particle is initially at rest on a smooth horizontal surface,
2002/2 Neco find:
A body at rest and of mass 12 kg is acted upon by a (i) its acceleration
force of 20N for 0.5 seconds. Find: (ii) its speed after 8 seconds; and
(i) The change in momentum (ii) its distance
(ii) The final velocity of the body to 2 d.p Solution
(iii) The distance covered within the time interval (i) F = ma
[g = 10 m/s2] (correct to 2 decimal places) 64 = 16  a
Solution 64
= a i.e a = 4 m/s2
(i) Change in momentum = Impulse (Ft) 16
= 20  0.5
(ii) t = 8s, v = ? u = 0, a = 4
= 10Ns
we may apply v = u + a t
(ii) Final velocity v.
From F t = mv – mu (where u = 0 at rest) v =0+48
v = 32 m/s
10 = 12 v – 12  0
10 = 12 v (iii) S = ? is added to data at (ii)
10 using v2 = u2 + 2as
=v
12 322 = 0 + 2  4  s
v ≈ 0.83 m/s to 2 d .p 1024
= S i.e 128m
2 4
400
2000/35 64
= 32 –
The speed of a particle of mass 8kg, moving along a 3
straight line is changed from 12ms–1 to 18ms–1 by a 32
force acting on it. Find in Newtons, the impulse. =  10 23 m
A 48 B 96 C 144 D 240 3
(C) Maximum velocity was at t = 4
Solution
Impulse = Change in momentum v = 4 (4) – 42
= mv – mu or m ( v – u ) =0
= 8 ( 18 – 12 ) 1999/8 (Nov )
= 48 (A) A particle 5g initially at rest, is moved by a force in a
straight line, such that its velocity vms–1 at time t seconds is
2003/12 (Nov)
The speed of a car of mass 1000 kg was reduced from given by v = 3t2 – 6t. Calculate:
18ms – 1 to 12ms – 1 by a constant retarding force 2000N. ( a )the distance traveled in the fifth second;
Calculate in s , the time taken to reduce the speed. ( b )the momentum of the particle at time 3 seconds.
A2 B3 C4 D6 Solution
(a) V 
dx
Solution  3t 2  6t
m =1000kg, u = 18ms – 1 , v = 12ms – 1, F = 2000N t = ? dt
Among the equations of motion 
x  distance traveled   dx   3t 2  6t dt 
v = u + at will help here based on the
given data. But for acceleration a ( retardation ) which 3t 3 6t 2
is not given. It can be gotten from x=  k
3 2
F=ma
x = t3 – 3t2 + k
2000 = 1000  a Since the body started from rest, x = 0, t = 0
2000 Thus k = 0
=a ; a = 2m/s2 (retardation)
1000 x = t3 – 3t2
Substituting for a value into In the fifth second
v = u – a t ( a is retardation) x = 53 – 3(52 )
12 = 18 – 2t = 125 – 75
12 – 18 = –2t = 50m
6 = t ( b ) Momentum = mass × velocity
2 At time 3 seconds
3s = t (B) v = 3t2 – 6t
= 3(32 ) – 6(3)
2005/8 = 27 – 18
A particle initially at rest, moves such a way that its = 9 ms–1
velocity vms–1 at any time t is 4t – t2 .If the particle 5
comes to rest again after t second, find Momentum = 9
1000
(a) the value of t,
45
(b) the distance covered =  0. 045 NS
(c) the maximum velocity 1000
Solution
(a) v = 4t – t2 1994/33 (Nov)
When the particle comes to rest, v is zero A particle of mass 3kg moving along a straight line a under
4t – t2 = 0 the action of a force F N covers a distance x m at time t s
t(4 – t) = 0 such that x = t2 + 3t. Find the magnitude of F at time t.
t = 0 or 4 – t = 0 A. 0 B. 2 C. 3(2t + 3) D. 6 E 6t
t = 0 or 4 s Solution
dx Magnitude of F at time t i.e F = ma
(b) v is  4t  t 2 where x is distance covered But we find a from distance x = t2 + 3t
dt
Velocity 
dx
To get x, we integrate  2t  3

 4t  t dt
4 dt
 dx 
2
d 2x
Acceleration 
0
2
4t 2 t 3 4 dt 2
x=  Thus F = ma
2 3 0
=3×2
=
 
4 42 43

 40  0 
  
= 6 (D)
2 3  2 3

401
2004/43 Neco Exercise 34.1 M1 2V + M2 (–3V) = M1 V + M2 2V
Calculate the force, which acts on a body of mass 3kg 2M1V – 3M2 V = M1V + 2M2V
moving at 2.5ms – 1 for 0.5 second, if the final velocity Collect like terms together
is 4.5 ms – 1 2M1V – M1 V = 2M2V + 3M2 V
A 4N B 7N C 12N D 24N E 36N M1 V = 5M2V
M1 = 5M2 (D)
1997/37 Exercise 34.2 2002/26 ( Nov)
A body of mass 8kg is moving on a smooth floor with Two objects of equal mass moving along the same line, but
an initial velocity of 4ms – 1 when a horizontal force of in opposite direction with speeds of 8ms-1 and 4ms-1 collide.
48N is applied to it. If the body subsequently covers a If after collision they coalesce and move in the direction of
distance of 4m, find the velocity, in ms – 1 attained by it the faster object, calculate their common speed
A. 2ms-1 B. 2.4ms-1 C. 4.8ms-1 D. 6ms-1
A8 2 B8 C4 3 D5 E4 Solution
In sketching, let us take the opposite direction into consideration
1994/31 (Nov) Exercise 34.3 Before impact After impact
Which of the following is not a type of force? 4
m
/s -
8
m/
s -
v
A Reaction B. Weight C. Thrust
m m 2 m
D. Tension E. Momentum.
By law of conservation of momentum
Momentum before impact = momentum after impact
1995/40 (Nov ) Exercise 34.4
4m – 8m = –2mv
The impulse acting on a moving object equals the
– 4m = – 2mv
changes in its.
 4m  2mv
A force B. velocity C. acceleration 
D. Kinetic energy E. momentum.  2m  2m
v = 2ms-1 (A )
1992/31 (Nov) Exercise 34.5 2004/29
A unit of measurement of momentum is Two smooth balls of masses 20g and 15g move in the same
A Kg.Sec B Kg.Sec-1 C Newton metre direction, with speeds 9ms-1 and 4ms-1 respectively. If they
D Newton Second E Kg. metre Sec collide, and the speed of the first ball reduces to 6ms-1, find
the speed of the second ball.
Collision A8ms-1 B. 7ms-1 C. 5ms-1 D. 1ms-1
The law of conservation of momentum says: ‘In any Solution
collision between two bodies, the total momentum in Before impact After impact
9m/s 4m/s 6m/s v
any direction is unchanged provided no external force
acts in that direction’ 0.02kg 0.015kg 0.02kg 0.015kg
M1U1 + M2U2 = M1V1 + M2V2 Applying the law of conservation of momentum
Total momentum before impact is equal to total momentum after impact 0.02 × 9 + 0.015 × 4 = 0.02 × 6 + 0.015 V
Where M1 and M2 are masses of the given objects and U 0.18 + 0.06 = 0.12 + 0.015V
and V are their initial and final velocities 0.24 = 0.12 + 0.015V
Coalesce after collision of two particles simply implies 0.24 – 0.12 = 0.015V
“Sticking together” 0.12 = 0. 015V
Bounce off each other after collision simply implies 0.12
“ Not sticking together” = V Thus, v = 8ms-1 ( A )
0.015
2002/40 (40) 2005/14
A body of mass M1 moving with velocity 2v collides A body P of mass 5kg collides with another body Q of mass
directly with a sphere of mass M2 moving with 3kg moving in the same direction with velocity 20ms-1. After
velocity – 3v. After the impact, their velocities are collision, the two bodies move together with a velocity
v and 2v respectively. Which of the following correctly 35ms-1. Calculate, in ms-1 the velocity of P before collision.
expresses the relationship between M1 and M2 ? A. 78 B. 62 C. 58 D. 44
Solution
1 1
A M1= 3M2 B M1 = m2 C M1 = m 2 D M1 = 5M2 Before impact After impact
5 2 2
0m
/
s 3
5
m/
s
Pv
Solution Q
In sketching the collision diagram we must take into 5
k
g 3
k
g 8
k
g
consideration plus velocity and negative Applying the law of conservation of momentum
velocity (opposite direction) 5v + 3 × 20 = 8 × 35
Before impact After impact 5v + 60 = 280
2
v-
3v v 2
v 5v = 280 – 60
m m 220
m
1 m
2 1 2 v= Thus, v = 44ms-1 ( D )
5
By law of conservation of momentum
402
2006/17 (a) Solution
A ball P moving with velocity 2u ms-1 collides with a First we find the common velocity of the engine and train after impact
similar ball Q , of different mass, which is at rest. After Engine Train
collision, Q moves with velocity u ms-1 and P with M1U1 + M2U2 = V(M1 + M2 )
velocity 1/2u ms-1 in the opposite direction. Find the Since the train was at rest, U2 = 0
ratio of the masses of P and Q. M1U1 = V(M1 + M2 )
Solution 5 × 103 × 32 = V(5 × 103 + 195×103 )
Before impact After impact V = 5 × 103 × 32
2u m/s u=0
1
2 U m/s u m/s 5 × 103 + 195×103
P Q P Q = 0.8km/h
m1 m2 m1 m2 Converting it to ms-1 gives
Applying the law of conservation of momentum = 0.8 × 1000 i.e 0.22ms-1
M12U + M2 × 0 = –M1 1/2U + M2U 3600
2M1U + 1/2M1 U = U M2 Note that v here is zero and u is 0.22ms-1, S = 2 , a = ?
5
/2M1 U = U M2 Using v2 = u2 – 2as ( negative since a is retardation)
i.e 5/2M1 = M2 0 = (0.22)2 – 2a × 2
Multiply through by 2 to clear fraction a = (0.22)2
5M1 = 2M2 2×2
Ratio terms 5 : 2 = 0.0121 ms-2
M1 : M 2 2002/1 Neco
A bullet is fired with a speed of 40m/s into a wooden block
1993/40 (Nov) of mass 0.6kg and becomes embedded in it. If the two bodies
A body P of mass 15kg moving with velocity 4ms-1 assume a common speed of 15m/s, find the mass of the
collides with another body Q of mass 20kg moving with bullet( correct to 2 significant figures)
velocity 5ms-1 in the same direction. If P is brought to Solution
rest by the collision, find the velocity of Q after the collision. First we find the common velocity as mentioned
A. – 2 3 ms-1
2 Bullet Wooden block
B. 2ms-1 C. 4.6ms-1 D. 8ms-1 E.9ms-1
M1U1 + M2U2 = V(M1 + M2 )
Solution Since the wooden block was not moving U2 = 0
Before impact After impact M1U1 = V(M1 + M2 )
4m/s 5m/s v=0
P Q v M1 × 40 = 15(M1 + 0.6)
P Q
15kg 20kg 40M1 = 15M1 + 9
15kg 20kg 40M1 – 15M1 = 9
Applying the law of conservation of momentum. M1 = 9 i.e 0.36kg
15 × 4 + 20 × 5 = 15 × 0 + 20 × v 25
60 + 100 = 0 + 20v 1996/15 (Nov)
160 = 20v A particle P is projected vertically upwards with an initial
160 velocity of 80ms-1. After an interval of time 4s and while P is
= v Thus, v = 8m-1 ( D )
20 still in the air, another particle Q is projected vertically
1994/35 upwards from the same point, with an initial velocity of 160m s -1
A bullet of mass 8kg fired from a gun hit a fixed block ( a ) How long does it take Q to hit P after Q has been
of wood with velocity 120ms-1. If after 0.05s, it projected?
emerged with a velocity of 90ms-1 Calculate the ( b ) Calculate the height above the point of projection,
resistance to the motion of the bullet. at which Q hits P ( Take g = 10ms-2 )
A 360N B 480N C 720N D 1800N E 10,800N Solution
Solution Up = 80ms-1 ,
UQ = 160ms-1 Time interval = 4s
M = 8, u = 120; t = 0.5, v = 90, Resistance F = ? -2
g = 10ms , t = ? S = ?
v  u Using S = ut – 1/2gt2 (since g opposes motion)
Using F = m   Let the time taken by Q to hit P be t
 t  Thus time taken by P to reach the point where they hit = t + 4
= 8  90 120  Let the distance covered be S
 0.5  Thus it takes Q a time t to cover s while
= - 480 N (B) The minus indicates resistance it takes P a time t + 4 to cover s
For Q, S = UQ t – 1/2gt2
2005/12b Neco S =160t – 5t2 -------( 1 )
An engine of mass 5 × 103 Kg moving with a velocity of
For P, S = UQ(t + 4) – 1/2g(t + 4)2
32km/h collides and links up with a train of mass
S = 80( t + 4) – 5(t + 4)2 -------( 2 )
195 × 103 kg, which is at rest. Immediately after impact
Equating S i.e ( 1 ) = ( 2 )
the brakes are applied uniformly so that the system
160t – 5t2 = 80( t + 4) – 5(t + 4)2
comes to rest after moving a distance of 2m. Find the
160t – 5t2 = 80t + 320 – 5(t2 + 8t + 16)
magnitude of the retardation of the engine and the train
403
32– t2 = 16t + 64 – t2 – 8t – 16 Problems on projectiles
8t = 16 1992/15 (Nov)
t = 2s A particle is projected from a point O with an initial speed of
60ms-1. It strikes the ground at horizontal distance of 300m
Substituting for t = 2 from O.
Height S =160(2) – 5(2)2 (a) Calculate the:
= 320 – 20 (i) two possible angle of projection,correct to the
= 300m nearest degree;
1996/40(Nov) Exercise 34.6 (ii) difference in the times of flight corresponding to the
Two bodies, each of mass 0.6kg, moving along the angle in (i)
same line, but in opposite direction, with speeds of (b) Show that, with the greater angle of projection
10ms-1 and 4ms-1 collide. If after collision, the faster the particle can not clear a barrier 140m high any
ball moves at 2ms-1 in the direction opposite to its where in its path of flight
former direction, find the new velocity of the other body. (Take g = 10ms-2 )
A 0.8 ms-1 B 4ms-1 C 8 ms-1 D12 ms-1 E 16ms-1 Solution
(a) i We are to find  from R = 300m(Range)
1997/31 Exercise 34.7 R = U2 Sin 2
A body of mass 8kg moving with a velocity of 8ms-1 g
collides with another body of mass 2kg, moving with 300 = 602 Sin 2
velocity 3ms-1 in the same straight line but in the 10
opposite direction. If they both coalesce and move Sin 2 = 300 × 10
together along the same straight line, find their common 60 × 60
velocity in ms-1 Sin 2 = 0.8333
A 7.0 B 6.2 C 6.0 D 5.6 E 5.0 2 = Sin-1 0.8333
2 = 56.44
2002/45 Neco Exercise 34.8  = 28.220
A bullet of mass 0.5kg is fired with a velocity of 30m/s Sine is only positive in 2nd quadrant with formula 180-
into a block of wood of mass 0.9kg. what is the Thus, the second angle is 180 – 28.22 = 151.780
common velocity of the bullet and the block when the
a ii Time of flight for  = 28.220
bullet is embedded?
T = 2Usin 
A12.5m/s B10.7m/s C6.8m/s D4.6m/s E 3.5m/s
g
= 2 × 60 Sin 28.22
Projectiles 10
Y
= 5.67 Secs

U Time of flight for  = 151.780


Uy H T = 2 × 60 Sin 151.78
10
= 5.67 Secs
Difference in time of flight is zero(5.67 - 5.67)
X
Ux
( b ) Here we find the maximum height of the particle
Range
bearing in mind that a projectile attains maximum range
when the angle of projection is 450
For any typical projectile of the form above the velocity Maximum height = U2sin2 
U has vertical and horizontal components 2g
Ux = U cos = 602 Sin2( 45)
Uy = U sin 2 ×10
Maximum height = U2sin2  = 90m NOT NEAR 140m
2g Hence cannot clear a barrier 140m high any where in its path of flight

Range( R ) = U2 Sin 2 max R is attained when  =45 1993/33 - 34


g A body is projected from a point such that the horizontal and
vertical components of its velocity are 640ms-1 and 480m-1
Time of flight = 2Usin  respectively (Take g = 10ms-1)
g 33. Calculate the greatest height attained above the point of
Time to reach maximum height t = Usin  projection
g A600m B1215m C1521m D11520m E 20480m

404
Solution
To attain maximum(greatest) height, it is the vertical H
component velocity that is required
Max height = U2 A tan- (3 4)
1

2g B
= 4802
Mountain
2 × 10
= 11520m ( D)
C
Ground level
34. If the time of flight is 96 seconds, calculate the Angle of projection  =  4
tan-1 3
horizontal range through the point of projection = 36.870
A84 400m B69000m C61440m D24000m E 640m
Solution
( a ) Maximum height = U2 Sin2 + 50m
Horizontal range has to do with horizontal component
above ground g
of velocity
= 70 Sin2(36.87) + 50m
Range = Ux × T from velocity = distance/time
2 × 10
= 640 × 96
= 1.26 + 50
= 61440m ( C )
= 51.26m
1994/40
A particle is projected with a velocity of 29.4ms-1. Find  51.3m to 3 sf
its maximum range on a horizontal plane through the ( b ) Time of flight here will be time taken to cover the
point of projection. projection from A to B then B to C
A 88.2m B 44.1m C 32.6m D 29.4m E 14.7m Time of flight A to B = 2 U Sin 
Solution g
To attain maximum range  = 450 = 2 × 70 × Sin 36.87
10
Maximum range = U2 Sin 2
= 8.4 Secs
g
= (29.4)2 = 88.2m At point B, S = ut + 1/2at2
9.8 50 = 70 × Sin 36.87 + 1/2 ×10 × t2
1994/39 (Nov) 50 = 42 + 5t2
A particle is projected with a velocity of 40ms-1 at an 8 = 5t2
elevation of 600. Calculate the vertical component of its t = 1.26S
velocity at a height of 50m. (Take g = 9.8ms-2 ) Total time of flight = 8.4 + 1.26 i.e 9.66Secs
( c ) it will strike the ground at C
A 25 3 ms-1 B 20 3 ms-1 C 2 545 ms-1 Velocity = distance
Time
D2 55 ms-1 E4 115 ms-1 70 = distance
Solution 9.66
Vy = Usin ( Vertical component of the velocity) Distance = 676.2m
= 40 × sin 60  676m to 3 sf
= 40 × 3 1995/33 (Nov)
2 A particle is projected from a point (0, 0) with a speed of
80ms-1 at an angle 300 to the horizontal. Find the height
= 20 3 ms-1 ( B ) attained in 5 seconds. Take g  10 ms 2 
1994/14 (Nov) A 240 3 m B 325m C 200 3 m D 200m E 75m
A bullet is fired from the top of a mountain, 50m above Solution
the ground with a speed of 70ms-1 at tan-1  3 4  to the Since it has angle of projection, it is a full projectile
horizontal. Calculate, correct to three significant figures the, Height here is the vertical component
(a) maximum height reached above the ground, From S = ut – 1/2at2
(b) time of flight of the bullet, Sy = 80 Sin30 × 5 – ½ × 10 × 52
(c) point where it strikes the ground = 75m( E)
(Take g = 10ms-2 )
Solution 1996/34 (Nov)
Since the projection has an angle of projection at the top If a projectile has a maximum range of 40metres,
of the mountain, it will be the normal projection with Find its speed of projection. (Take g = 10ms-2 )
additional height as : A 0.5ms-1 B 2ms -1 C 4ms-1 D 20ms-1 E 400m-1
Solution
At maximum range ( = 45)
R = U2 Sin 2
g
405
40 = U2 Sin (2 ×45) 2004/13 b Neco Exercise 34.10
10 A particle is projected with speed 60m/s at angle
400 = U2 tan -1  3 4  to the horizontal. Find:
U = 20 ms-1 ( D ) (i) the time the particle is at a vertical height of 52m;
2000/15 Neco (ii) the horizontal displacement of the particle
An object is projected from a point on the ground with at that time (take g = 10m/s2)
an initial velocity of 50m/s. If the vertical distance
moved by the object after 3 seconds is 40m, find the 2010/8 Exercise 34.11
angle  to the? An arrow is shot with a velocity of 120ms-1 at an angle of
horizontal at which the particle is projected. 600 to the horizontal. Find the:
(Take g = 9.8 m/s2 ) ( i ) maximum height attained
A 75.10 B 59.10 C 56.20 D 55.70 E 34.10 ( ii ) range on the horizontal plane
Solution ( iii ) time of flight (Take g = 10m/s2)
We list out the given items to know the equation that will apply
u = 50m/s, t = 3s, s = 40m(vertical distance),  = ? 2002/42 Neco Exercise 34.12
S = ut – 1/2gt2 ( g opposes motion) A moving body under a given velocity of projection Um/s
40 = 50 Sin × 3 – 1/2 × 9.8 × 32 attains its maximum range (R)
40 + 44.1 = 150 Sin When the angle of projection  is
A 900 B 750 C 600 D 450 E 300
Sin = 84.1
150
2002/44 Neco Exercise 34.13
 = Sin-1 0.5607
What is the time of flight of a particle which is projected
 = 34.100 ( E ) with an initial speed of 50m/s and angle of elevation of 600?
(Take g = 10m/s2)
2003/10 Neco Horizontal projection A 10.6secs B 9.8secs C 8.66sec D7.66secs E 5.40secs
A boy threw a stone horizontally with a velocity of
20m/s from the top of a cliff 40m high. How far from
2002/13 Neco Exercise 34.14
the foot of the cliff does the stone strike the ground A particle is projected with an initial speed of 25m/s at an
(Take g = 9.8m/s2)?
angle of elevation  given by sin-1 (0.8660). Find:
A 0.572m B 5.72m C 57.2m D 72.5m E 75.2m
(i) The greatest height reached by the particle
Solution
(correct to 2 decimal places)
For horizontal projection
(ii) Maximum range (R) covered by the particle.
R = u  2gh (iii) The time taken to reach the greatest height
(correct to 1 decimal place).
= 20  2 9.840 (iv) The time of flight (T) taken by the particle.
= 57.14m ( C)
g  10 m / s  (correct to 2 decimal places)
2

2003/3 Neco
A particle is project from a point making an angle of
450 to the horizontal ground. If the initial velocity is
36m/s, what is the vertical distance traveled after
3 seconds?
Solution
We list out the given items to know the equation that will apply
u = 36m/s, t = 3s, s = ?(vertical distance),  = 45
S = ut – 1/2gt2 ( g opposes motion)
= 36 Sin45 × 3 – 1/2 × 10 × 32
= 76.37 – 45
= 31.37m

2003/13 Neco (Dec) Exercise 34.9


A bullet is projected with initial speed of 30ms-1 at an
angle of elevation of  , where tan   512 from a
0

point on the level ground find, the:


(a) (i) greatest height reached, and
(ii) time take to reach it.
(b) (i) time of flight; and
(ii) range of the bullet
(c) time it is traveling, perpendicular to its
original direction.
406
Chapter thirty five ( 4 ) If two vectors have the same direction ( i.e  = 0 )
Then A . B  AB
Vectors
A vector is a quantity that has both magnitude and
direction. It is represented by a small arrowhead on top A B

of a letter showing the direction PQ ; underlining of A. B  AB cos 0


letter r , or boldly written letters t are other forms of
A . B  AB
representing vectors.
We denote the vector in the direction P to Q by PQ ( 5 ) If two vectors are opposite to each other(  = 180 )
and the vector in the direction Q to P by QP Then A . B   AB
r
Q 0
1
80

PQ P B A
P

QP A. B  AB cos180
Q
A . B   AB
The following terms are relevant to the concept of
vector
Representation of vectors in three dimensions
Scalar product A vector A has 3 components in x, y, z directions. These
Suppose we have two vectors forming an angle as directions are perpendicular to each other where i, j, and k
shown below are the unit vectors
y

A j
s
co
B

A
B i x
s
co

B
A

z k
A
B i . j = ijcos 900 i.e 0
Hence we do not multiply unlike terms (vectors) and their
( 1 ) The scalar product or dot product coefficients as we have seen that cos 900 = 0
A. B  AB cos
Vector product
The vector product or cross product of two vectors
( 2 ) Cosine of angles between two vectors
A and B which is denoted as A × B and is defined as
A and B
the product of the magnitude of the vectors A and B and
Cos = A. B the sign of their included angle 
AB A × B =AB Sin

( 3 ) If two vectors A and B are perpendicular


A B
to each other (  = 900 ) Then A  B = 0 ( i ) Sin  
A B

A ( ii ) If two vectors A and B are perpendicular(orthogonal)


to each other ( = 900 )
900 A × B =AB Sin 900 since Sin 900 = 1
B A × B =AB
( iii ) If two vectors A and B are collinear or parallel to
A. B  AB cos 900 each other ( = 00 )
A B =0 A × B =AB Sin 00 since Sin 00 = 0
A×B = 0

407
A = a1 i + a2 j + a3 k Unit vector
B = b 1 + b2 j + b 3 k 1
u  a
Then a
A + B = (a1 + b1 ) i + (a2 + b2)j + (a3 + b3)k
A – B = (a1 – b1 ) i + (a2 – b2)j + (a3 – b3)k
Resultant of vectors
Also by determinant method Resultant of vectors is the addition of the vector involved

i j k Projection of vector
Projection of vector a on b
A  B  a1 a 2 a3 same apply for 2×2
b a
b1 b2 b3 P a , Projection of vector b on a P b
b a
Basic theorem to work with
Two non-zero vectors a and b are parallel if and if Magnitude of a vector
 a scalar t such that a = t b The magnitude of the vector a = 3i + 2j –7k
i.e one is a multiple of the other
a  = 32  2 2  (7 ) 2
Addition of Vectors
If two vectors a and b represents the two adjacent
sides OA and OB respectively of a parallelogram
Sum, subtraction and scalar multiplication
OACB, then the diagonal OC which passes through in vectors
their point of intersection will be equal to their vector sum 1996/33 (Nov)
OC = a + b Given that a = 7i – 3j and b = 5i + 7j,find 3a + b
And the other diagonal BA is equal to A 26i – 2j B. 21i – 2j C. 16i – 2j
BA = a – b D.26i + 2j E. 12i + 4j
A Solution
b
3a + b = 3(7i – 3j) + (5i + 7j )
= 21i – 9j + 5i + 7j
a
C = 26i – 2j (A)
a+b
1994/32 (Nov )
O Find the resultant of the vectors
a-b

In the diagonal BA, the movement to A


a = 3i
b is -ve to B direction and so we have -b
b = –2i – j
B c = i + 4j
Triangle rule A 4i + 4j B. 2i +3j C. 6i + 5j
First, we need to follow the vector arrows as given in D. – 4i – 4j E – 2i – 3j
the diagram or a triangle with arrows showing the Solution
Resultant of the vectors = a + b + c
vectors AB , BC and AC = 3i – 2i – j + i + 4j
B = 2i + 3j (B)

2000/36
 3  2 8
Given that p   + q   =   , where p and q are
1  4  6
A C

Here AB follows BC but AC contradicts them constants, find the value of ( p + q )


AB + BC = AC deductively A –4 B 1 C 3 D 4
Solution
BC = AC – AB and AB = AC – BC One column matrix ( same as vector ) problem
3p + 2q = 8 ------ ( 1)
Other definition of terms p + 4q = 6 ------ ( 2 )
(1)  2 and subtract
Origin vectors are: 6p + 4q = 16
2x2 row vector( 0, 0 ) or 3x3 row vector( 0, 0, 0 ) –( p + 4q =6 )
 0 0
or 2x2 column vector   or 3x3 column vector   5p = 10
0
 0 0
p = 10 i.e 2
  5

408
Substitute p = 2 into (2)
p + 4q = 6 will become 2003 / 7 (b)
4q = 6 – 2   12 
4q = 4 Given that p =  4  and q =   , find a vector U such
 
q = 4 i.e 1  3   5 
4 that U is parallel to p + q and is half the size of p + q
Thus p + q = 2 + 1 Solution
= 3 (C)  4   12   8 
p + q =   +   =  
1999/33 (Nov)
3   5   2
Given that a and b are scalars, solve the equation
By the theorem:
2  4   2  a
a  + b  =   Two non-zero vectors a and b are parallel if and only if
1  2   3  scalar t, such that a = t b
A. a = - 1 ; b = - 1 B. a = - 1; b = 1 Thus, a vector parallel to p + q and half the size of p + q is simply
C. a = 1 ; b = - 1 Da= ;b=
4 7
1 1  8   4 
5 10 (p+q)=   =  
Solution 2 2  2  1
2a + 4b = – 2 --------(1)
2005 / 16(a)
a – 2b = 3---------(2)
The position vectors of points R, S and T in the X – Y plane
(2)  2
are r = 5i + 3j, s = 8i – j and t = 11i – 5j
2a – 4b = 6
respectively.
 (2a  4b   2) (i) Show that, R, S and T are collinear.
 8b  8 (ii) Find scalars k1 and k2 such that
b=-1 15i + 9j = k1 r + k2 t
Substitute b value into (1) Solution
2a + 4(-1) = -2 (i) R S =s–r
2a – 4 = - 2 = ( 8i – j ) – ( 5i + 3j )
2a = - 2 + 4 = 3i – 4j
2a = 2
a = 1 Thus a = 1 and b = -1( C) Similarly S T = t – s
1999/7 ( Nov) Exercise 35.1 = (11i – 5j ) – ( 8i – j )
Given that p = ( 5i – 3j) and q = ( 4i + 7j), find the = 3i – 4j
scalar m and n such that 2i + 3j = mp + nq  R S and S T are parallel which is possible only when R,
1996/ 34 Exercise 35.2 S and T lie on one line; hence result
If p  2i  3 j, q  3i  5 j and r = i  j , ( ii ) 15i + 9j = k1( 5i + 3j ) + k2( 11i – 5j )
 q  3r = 5k1 i + 3k1 j + 11k2 i – 5k2 j
find 2 p
15i + 9j = 5k1 i + 11k2 i + 3k1 j – 5k2 j
A 2i 15 j B. 12 i j C. 2 (3 i 4 j) Equating terms
15 = 5k 1 + 11k 2 ------( a ) and
D.2( 5 i  j) E. i 12 j 9 = 3k 1 – 5k 2 -----------( b )
a × 3 and b × 5 and subtract
2003/ 26 (Nov ) Exercise 35.3 45 = 15k1 + 33k 2
If u = – 2i + 3j and v = i – 3j, Find 5u – 3v – ( 45 = 15k1 – 25k 2 )
A – 13i + 24j B – 7i + 6j C 7i + 24j D – 13i + 6j
0 = 58 k 2  k2 =0
Problems on parallel vectors Substituting k2 value into ( a ) 15 = 5k1 + 0
2000/17 ( Nov) 15
= k1  k1 = 3
Given that the vectors 3i – 2j and 4i + Pj are parallel, 5
find the constant P 2002/49 Neco
A –6 B – 8/3 C – 3/8 D 1/ 6 What is the unit vector which is parallel to the
Solution vector 7i – 4j ?
Two vectors are parallel then, A × B = 0 Solution
i j Two vector a and b are parallel
and A × B = 3 If a = sb where S is a scalar
2
Thus unit vector here
4 p 7i  4 j
7i  4 j
 3p + 8 = 0 = =
 3p = -8 Thus p = – 8/3 ( B) 7 2  (4) 2 65
409
Problems on projection of vectors
Substituting for the given value
2004/ 45 6
The projection of the vector a = 5 i – 2 j + k on the Cos =
vector b = 2 i + 3 j . 3 4
Cos = – 0 .5
A 4 13 B 5 30 C 5 13 D 4 13 E 30  = – Cos– 1 0.5
15 13 13 15
Though it is 600 but the minus sign shows that Cosine here is
Solution
in 2nd quadrant with formula 180 – 
P = ˆ a
b.  = 180 – 60
 b = 1200 (C)
But b= b where b is 2 2  32
2004/9
1 Find the angle between i + 5j and 5i – j
=2i  3 j  A 300 B 450 C 600 D 900
13 Solution
Thus P  b. ˆ a= i  5 j . 5i  j 
Cos  =
1
2i  3 j . 5i  2 j  k 
 

12  5 2  5 2   1 
2


13 55 0
=
1
10  6
Cos  =
 26  26  i.e.
26
=0

13 Cos  = 0
1 4 13  = Cos –1 0
=  4 i.e (D) = 900 (D)
13 13
2004/40 Neco
2000/24 Exercise 35.4
Find the projection of the vector a on the vector b, if
If m  4 i  3 j and n  i  2 j what is the
i  3 j and b = – i  4 j cosine of angle between m and n ?
a=2
10 A 2 5 B 5 2 C 2 D 12 5 E 2 5
A 17 B. 10 17 C 10 17 D. E. 5 17 2 25 5
10 17 17 Solution
m . n
Problems on equal vectors Cosine of angle between m and n =
m n
m . n = 4 1  3 2
2002/50 Neco
40
What is the value for which vectors x i + 8j and 5i + j = (4 + 6)
x = 10
are equal ?
   
1 1

A +5 B +4 C –5 D –9 E –10 m  4 2  32 2
and n  12  2 2 2

Solution
Two vectors are said to be equal if their = 25 = 5
i – components are equal =5
In this case Thus, Cosine between m and n = 10
5 5
i : i
x : 5 thus x = 5 ( A) = 10 5
25
Confirm it at the j-component
40 = 2 5 (E)
= 8 thus x = 5 5
x
1996/39
Problems on angle between two vectors
If PQ = 4 i  3 j and QR = 5 i  12 j,
2000/16 (Nov )
Find the cosine of the angle between the two vectors.
Given that p = 3, q = 4 and p·q = – 6, 16 13 7 4 56
A. 65 B. 13 C. 65 D. 13 E. 65
find the angle between p and q.
A. 300 B 450 C 1200 D 1350 Solution
Solution PQ. QR
Cosine of angle between the two vectors =
p. q PQ QR
Cos =
p q
410
4i  3 j . 5i  12 j  2p – 3q = (– 6 – 6 ) i + [ 8 – (–3 )] j
=
 4  3  5  12 
2 2 2 2 = –12i + 11j
Thus, 2 P  3q = (12) 2  112
20 36
=
 
25 169  = 144  121 = 265 (C)
56 2004/21 Neco (Dec)
56
= i.e 65 (E)
5 13 Calculate the magnitude of the vector a = 4i – 3j + 12k
2004/ 7 Neco A 169 B 26 C 19 D 13 E 8
Solution
Given that u = - i + 2 j – 2 k and v = 3i + 4 j –12 k Magnitude of vector
Find (i) the dot product of u and v a = 4i – 3j + 12k
(ii) the angle between u and v
a  4 2  ( 3) 2  12 2
Solution
(i). u . v =  1  3  2  4  (2  12)
= 16  9  144 = 169 i.e 13(D)
=  3  8  24 
1995/35 (Nov )
= 29
6 1
(ii) u  v = u v cos Given that a =   and b =  
3  2 
u .v
 cos Evaluate a  b  a  b
u v

u  (12 )  2 2  (2) 2 
1
2
; 
v  3 2  4 2  (12) 2 
1
2 A.0 B 14 C. 2 5 D. 5 2 E. 10 2
Solution
= 169 
1 1
= (9 )2 2 a+b and a–b
=3 = 13 6  1  7 6  1  5
29  +   =    –  =  
Thus, cos = i.e
29  3   2   1   3   2   5 
3 13 39
Next, │a + b│= 7 2  12 and │a – b│ = 52  52
cos = 0. 7436
1
 = cos 0 .7436 = 50 and = 50
= 41.960
Thus, a  b  a  b = 50 – 50
1996/38 (Nov) Exercise 35.5
= 0 (A)
If p = 2i + j and q = 5i +3j find the cosine
of the acute angle between p and q 2002/ 31 Exercise 35.8
A. 13 170 B. 7 170 C. 7 D. 13 E. 7 If p = ( 2i + 3j ) and q = ( i – 7j ) then,
170 170 13 170 170
A q + p = 63 B p  q = 5
2003/16 Neco Exercise 35.6
C pq = 101 D p  2q = 17
If a  2 i  3 j and b  2i  4 j, find the cosine of
angle between b and b
Problems on dot product of vectors
A 2 B 4 C 8 D 8 E 4
33 65 65 20 13 1995/31 (Nov )
Evaluate; (8i – 15j )  (8i – 15j)
A. 49 B. 161 C. 189 D. 225 E. 289
2002/27 (Nov) Exercise 35.7
Solution
Evaluate the angle between a = – 4i + 2j and b = i – 3j
Dot product = (8i – 15j )  (8i – 15j)
A 450 B 600 C 1350 D 3150
= 8  8  (15 (15)
= 64 + 225
Problems on vector magnitude = 289 (E)
2000 / 32
2003/4 Neco
Given that p = –3i + 4j and q = 2i – j , evaluate 2 p  3q If r = i + 2j and q = 2i – 3j, find r  q
A -4 B -3 C 21/2 D 2 E 4
A 13 B 13 C 265 D 169 Solution
Solution r  q = (i + 2j)  (2i – 3j)
2p = 2(–3i + 4j ) 3q = 3(2i – j ) =2–6
= –6i + 8j = 6i – 3j = –4 A
411
2002/48 Neco Solution
The scalar product of vectors –2i – 3j and 4i + 5j is Two vectors are perpendicular to each other if their
A 23 B 7 C –7 D – 10 E – 23 dot product equals zero.
Solution pq=0
Scalar product of two vectors is same as dot product (6i – 9j )  ( ai + 2j ) = 0
(–2i – 3j )  ( 4i + 5j ) = (–2 × 4) + (–3 × 5 ) 6 × a + (– 9) × 2 = 0
= –8 – 15 6a – 18 = 0
= – 23 (E) 6a = 18
2005/15 a = 3 (D)
Find the magnitude and direction of the resultant of
a = 2i + 3j and b = 2i + j 2004/42 Neco
A ( 4 2 , 0300 ) B (4 2 , 0450 ) If a  3i  4 j and b   j  2 k are perpendicular, what
C ( 2 2 , 0450) D ( 2 2 , 0300 ) is the value of  ?
Solution A 4 B 0 C – 3/2 D – 8/3 E–4
Resultant of ab =a+b
= ( 2i + 3j ) + ( 2i + j )
Solution
= 4i + 4j a . b  0 Condition for perpendicularity

magnitude of ab = 42  42 3 0  4    0 2  0
= 1616 4 = 0
 = 0 (B )
= 32
=4 2 2004/32 Neco (Dec)
Direction of resultant of ab Find K such that a = 4i – 10j and b = Ki + 2j are
a.b perpendicular vectors
Cos  = Solution
a b a . b  0 Condition for perpendicularity
2i  3 j . 2i  j  (4i – 10j )  ( k i + 2j ) = 0
=
2 2
 32  2 2
 12  4k + (–20 ) = 0
4k – 20 = 0
43 4k = 20
=
 13  5  4k

20
4 4
7 k = 5
Cos  =
65
7 1994/34
 = Cos -1 If p = 4i + kj and q = 2i – 3j are perpendicular,
65
find the value of k, where k is a scalar quantity.
= 29.740 8 2 3
 300 A. 3 B. 3 C. 3 D. 2 E. – 4
Thus magnitude and direction is ( 4 2 , 0300 ) (A) Solution
Two vector p and q are perpendicular, if
1996/38 Exercise 35.9 p q = 0
(4i + kj )  (2i – 3j) = 0
If PQ = 2 i  3 j and QR = 3 i  5 j , find PQ  QR
A 21 B. 15 C. 9 D. 1 E. –9 4 2  (k   3)  0
8 – 3k = 0
8 = 3k
8
3 k (B)

1994/7 June
OP, OQ and OR are the vectors 3i + 4j, 3ki + 40j and 8i – 2j
Perpendicular condition of vectors respectively, where k is a scalar.If the resultant of OP and
OR is perpendicular to OQ, find K .
2003 / 15 Solution
Given that p = ( 6i – 9j ) and q = ai + 2j are First, we find resultant of OP and OR
perpendicular, calculate the value of the constant a. = OP + OR
A –3 B 4 C 3 D 3 = (3i + 4j ) + (8i – 2j)
3 4 = (11i + 2j)
412
Next, for two vector to be perpendicular; 4i 3 j
their dot product = 0 =
(OP +OR)  OQ = 0 4 2  (3) 2
= 4 i  3 j  (C)
(11i + 2j )  (3ki + 40j) = 0 1
33k + 80 = 0 5
33k = –80
80 2000/37
k = –
33 The position vectors of points P and Q are i – j and
2i – 5j respectively. Find a unit vector perpendicular to PQ
1996/37 (Nov)
For what value of p are the vectors pi – 3j and 3i – 2j A. 
 i  4 j  B   i  4 j 
perpendicular? 17 17
2
A. -3 B. -2 C. 3 D. 2 E. 3 4 i  j  4i j 
C.  D. 
Solution 17 17
Since the two vectors are perpendicular; then
(Pi – 3j)  (3i – 2j) = 0 Rule is AB = b a
3p + 6 = 0 PQ = (2 i – 5j – (i – j )
3p = – 6
= i – 4j
6
p=– 3 i.e –2 (B)

Problems on Unit vectors Next, Unit vector perpendicular to PQ = i – 4j


1993/31 ( Nov) 12 + ( – 4 )2
Find the unit vector in the direction of the resultant of
p = 5i + 3j and q = 7i – 8j
i  4 j
=
1 1 12 5 17
A. i– j B. i j C.
12 5 13 13
2 5 2000/47 Neco Exercise 35.10
i  j The unit vector in the direction of (3i + 4j) is
13 13
1 1 1
5 12 1 1 A (3i + 4j) B (3i + 4j) C (3i + 4j)
D. i j E. i  j 6 5 4
13 13 12 5
1 1
Solution D (3i + 4j) E (3i + 4j)
Resultant of p and q = p + q 3 2
= (5i + 3j) + (7i – 8j)
= (12i – 5j)
pq 1994/34 (Nov ) Exercise 35.11
Next, unit vector is = Find the unit vector acting in the direction of 2i – j
pq
2i  j 2i  j 2i  j
12i  5 j A i+j B i–j C D E
= 5 5 3
12 2  (5) 2
12i  5 j 1999/26 (Nov) Exercise 35.12
= Find unit vector in the direction of the
169 Resultant of P = ( 4i – j ) and q = (2i – 7j )
1
= 12i  5 j  ie 12 i  5 j ( B) 3
A. i  j
4 1 1
B. i  j C. i 
1 7
j
13 13 13 5 5 6 8 5 10
1996/32 2 1
D. i  j
Given OQ = 4 i  3 j , the unit vector in the direction 5 10

of OQ is
1
A 20 i 15 j B 4i 3j C (4i  3 j )
5
D. –
1
5

4i  3 j  E.
1
7
4i  3 j 
Solution
 OQ
Unit vector in OQ the direction of OQ =
OQ
413
Problems on Joint cases of vectors properties (c) the cosine of acute angle between A and B, leaving your
1996/14 answer in surd form
The position vectors of the points P and Q are Solution
p = – i + 3j and q = 2i + 2j respectively. Find (a) 3 A  2 B
(a) 3p – 4q (b) p  q (c) the angle between p and q First we find 3A – 2B = 3(4i – 2j ) – 2(2i + 3j)
(d) the scalars h and k such that hp + kq = 5i + 7j = 12i – 6j – 4i – 6j
Solution = 12i – 4i – 6j – 6j
(a) 3p – 4q = 3(– i + 3j) – 4(2i + 2j) = 8i – 12j
= – 3i + 9j – 8i – 8j Next, 3 A  2 B  82   122  
= – 11i + j
(b) p  q = 64  144
but p + q = – i + 3j + 2i + 2j = 208 or 14.42
= i + 5j
= 4 13
Thus, P  q  12  52

= 26 (b) 8i + 8j = A + B;
(c) Angle between p and q = (4i – 2j ) + (2i + 3j)
p.q = 4 i – 2j + 2i + 3j
cosine of angle b/w p & q = = 4 i + 2i – 2j + 3j
p q 8i + 8j = (4 +2)i + (–2 + 3)j
 i  3 j . 2i  2 j  Equating terms
=
 (1) 2
 32  2 2
 22  4 + 2 = 8 -------- (1)
–2 + 3 = 8 ---------(2)
 26 Multiply (2) by 2 and add
=
  
10 8 – 4 + 6 = 16
4 + 2 = 8
4 1 8 = 24 Thus  = 3 and  = 1/2
= i.e
4 5 5 (C) Cosine of angle b/w A and B =
A B
A B
Cosine of angle b/w p and q = 0.4472
Angle b/w p and q = cos 1 0.4472 A  B = 4 2  (2 3)
0
= 63. 43 = (8 – 6)
=2
(d) hp + kq = 5i + 7j
h(–i + 3j) + k(2i + 2j) = 5i + 7j A  4 2  (2) 2 and B  2 2  32
–hi + 3hj + 2ki + 2kj = 5i + 7j = 16 4 = 4 9
Collect like terms together at LHS
–hi + 2ki + 3hj + 2kj = 5i + 7j = 20 = 13
(–h + 2k)i + (3h + 2k )j = 5i + 7j 2 2
Equating terms Cosine of angle = =
–h + 2k = 5 ------(1) 20 13 2 5  13
– (3h + 2k = 7) -----(2) 1
– 4h = –2 =
h = 1/2 65
Also from (2) 2k = 7 – 3( 2 )
1 65
=
65
i.e
1
65
 65 
2k = 7 – (
3
2 )
11
2k = 2
1994/37 (Nov )
11
If p and q are two parallel vectors in opposite directions,
k= 4 Then which of the following statements is/are true?
1992/14 (Nov) I pq= p q
The position vector of two points A and B are
a = 4i – 2j and b = 2i + 3j respectively. Find: II p  q = – p q
(a) 3 A  2 B III p  q = 0
(b) Scalars  and  such that A II and III only B. I only C. III only
8i + 8j = A + B; D. I and III only E. II only
414
Solution pq =
By condition of parallel and opposite vector.
Thus 15 2  20 2
Let the vectors be = 225  400
p = 3i + 2i and q = – 3i – 2j
Testing = 625 i.e 25
I pq= p q
LHS: (3i +2j)  (–3i – 2j) = – 9 – 4 p = 9 2 12 2 q = (6) 2  (8) 2
= – 13
RHS  3  2 . 3
2 2 2
 22 =  13 13 = 81 144 = 36  64
= 13 LHS ≠ RHS = 225 = 100
II pq=– p q = 15 = 10
From the test at I we deduce that II is true Therefore pq – { p – q }
III p  q = 0 = 25 – { 15 – 10 }
From the test at I, we deduce that III is not true = 25 – 5
The answer is II only (E) = 20 (C)
2000/15 a (Nov)
1994/7 (Nov) If x = 4i – 3j ;
Given a = 2i + 5j and b = 4i – 3j, find y = 2i + 3j
(a) The unit vector in the direction of b. z =i +j
(b) The projection of a on b ( i ) find z · y ; ( ii ) find the angle between y and z
(c) │3a – 2b│
( iii ) show that x is not perpendicular to z
Solution
Solution
b
(a) Unit vector in the direction of b = (i) z· y = ( i + j ) · ( 2i + 3j )
b =1×2 + 1×3
4i  3 j =2+3
= =5
4 2  32 ( ii ) Angle between y and z
4i  3 j 1 y.z
= i.e 4 j  3 j  Cos =
25 5 y z
 5
2  1 
(b) Projection a on b = b  a =
b
2
 32 2
 12
=  a 5
b 5 5
1
= ( 4i – 3j ) (2i + 5j)
=
 13  2  =
26
=
5.099
5 Cos  = 0.9806
1 7  = Cos– 1 0.9806 = 11.300
= (8 –15) i.e – 5
5
(c) First is 3a – 2b = 3(2i + 5j ) –2(4i –3j ) ( iii ) If x is perpendicular to z then x . z = 0
= (6i + 15j) + (–8i + 6j )
x . z  ( 4i – 3j ) · ( i + j ) = 4  1 + (–3)  1
= –2i + 21j
= 4–3
= 1
Thus │3a – 2b│ = (2) 2  212 One is not zero so x is NOT perpendicular to z.
= 445 2000/15
= 21. 095 Given that p = 2i + 3j; q = i + j ; r = i – 2j ;
2000/22 (Nov) Find: (i) 2 p  3q  r ;
Given that p = 9i + 12j and q = – 6 i – 8j (ii) The Unit vector in the opposite direction of 2p – 3q + r;
Evaluate pq – { p - q } (iii) The angle between p and r, correct to the nearest degree.
Solution
A 10 B 15 C 20 D 30 ( i ) First we do the scalar multiplications
Solution 2p – 3q + r = 2(2i + 3j) – 3(i + j) + (i – 2j)
First vector Subtraction for p and q = 4i + 6j – 3i – 3j + i – 2j
p – q = [ 9 – (–6 ) ]i + [ 12 – (–8 ) ] j Collect like terms together
= (9 + 6)i + (12 + 8 )j = 4i – 3i + i + 6j – 3j – 2j
p – q = 15i + 20j = 2i + j
415
= 29.930
Thus, 2 p  3q  r  2 2  12
 300 to the nearest degree
= 5 Length of projection of q on p = q Cos 
( ii ) 1st we find unit vector of 2p – 3q + r
2 p  3q  r
 
 9 2  82 × 0.8666
=
2 p  3q  r = 145 × 0.8666
= 10.43 units
2i  j
= 1997/7
5 The position vectors of points p and q relative to a fixed
Thus, Unit vector in opposite direction of 2p – 3q + r point O are
 2i  j  p = 3i + 4j and q = 12i + 5j respectively.

5 Find: ( i ) 2 p  3q ( ii ) the unit vector parallel to P Q ;
 2i  j (iii) the length of the projection of p on q.
 Solution
5 (i) 2 p  3q
(iii) angle between p and r is given by
p. r First find 2p – 3q
Cos  = = 2( 3i + 4j ) – 3( 12i + 5j )
p r = 6i + 8j – 36i – 15j
2i  3 j . i  2 j  = –30i – 7j

2  32
2

12  (2) 2  Thus 2 p  3 p = (30) 2  (7) 2
2  1  3  (2) = 949

 13   5 
4 ( ii ) P Q = q – p
Cos  = = (12i + 5j) – (3i + 4j)
65
= 12i + 5j – 3i – 4j
Cos  = – 0.4961
= 9i + j
 = 60. 260  600
Unit vector parallel to PQ
Cosine is negative in 2nd & 3rd quadrant
i.e.  is either 180 – 60 or 180 + 60 9i  j 9i  j
= 1200 or 2400 = =
2002/7 9 2  12 82
The position vectors of points P and Q are;
p = ( i + 3j ) (iii) Length of the projection of p on q = p Cos 
q = (9i + 8j ) P. Q
respectively. Find: First, we find Cos  =
p q
(a) PQ
3i  4 j . 12i  5 j 
 3  4  12 
(b) correct to the nearest degree, the angle =
between p and q. 2 2 2
 52
(c) The length of the projection of q on P
Solution 36  20
=
(a) PQ =q – p (5) (13)
= (9i + 8j ) – (i + 3j ) 56
= 8i + 5j Cos  =
65
P. Q
(b) Angle between p and q  Cos  = Thus length of projection of p on q = p Cos 
 3  4   5665 
p q
2 2
1 9  3 8 =
Cos  =
1 2
 32  9 2
 82  = 25 ×
56
9  24 65
=
 10   145  =5× 56
65
= 4.31 units
33
=
1450
Cos  = 0.8666
 = cos–1 0.8666
416
1999/32 (Nov) Exercise 35.13 2003 / 27
Given that a, b and d are vector quantities and  the  3 1
angle between b and d, which of the following Given that O P =   and P Q =   ,
statements is /are true?  5  4
I. a . b = b . a where O is the origin, find O Q
II. (a . b) + d = a . b + a .d
 
4
III. b . d = b d cos A   9  B   2  C  1  D  
     2
  4  1   9
A. I and II B. II and III C. I and II D. I only Solution
1997/38 Exercise 35.14
Which of the following is /are always true of vector We have a simple rule for vectors AB = b – a
P, Q and R ?  0
I PQ = QP and origin vector 0 in this case is  
II P (Q + R ) = PQ + PR  0
III PQ = O if p is perpendicular to Q
Thus O P is  0  i  =  3    i   3  i.e P =   3
 
A I only B II only C I and II only  0  j   5  j   5
        5
D II and III only E. 1, II and III
1 i4   4
2004/10 Exercise 35.15 Also P Q .is  o  i  =      i.e Q =  
o  j 
If p and q are two parallel vectors in the same direction,    4  j   9   9
which of the following statement is/are true ?
0  4 i  4
I p.q = p q Hence, O Q is   =   i.e.   (D)
0 9  j  9
II p.q = – p q III p . q =0
1997/39
A I only B I and II only C II and III only D III only The position vector of P and Q are p = i – 3j
Problems on position vectors and q = 4i + 5j respectively. Find the unit vector
1994/36 parallel to PQ
The position vectors of points P, Q and R relative to an
origin O are A. 1 (3i  8 j ) B.  1 (3i  8 j ) C. 1 (i  3 j )
73 73 73
 8   2   3  1 1
  ,   and   respectively. Find OP-3OQ + 2OR D. (4i  5 j ) E. (5i  8 j )
 4   3   5  73 89
Solution
 8   20   20  8  6 
A.   B.   C.   D.   E.   First we find PQ = q – p
 15   16  3  3   15  = (4i + 5j ) – (i – 3j )
Solution = 4i – i + 5j + 3j
OP = P – O = 3i + 8j
8  0
 8
=  i.e   Next, we find unit vector parallel to PQ
4  0 4 Recall that two non – zero vectors b and c are
Similarly 3OQ = 3Q parallel if and only if  a scalars.
Such that b = Sc
 6 
=  Thus what we need to do is to get the unit Vector of PQ
9
and 2OR = 2R PQ
=
 
6
PQ
= 
 10  3i  8 j
=
 8   6   6  32  8 2
Hence OP – 3OQ + 2OQ =   -   +  
4  9   10  1
= ( 3i + 8j ) (A)
We absorb the minus sign as 73
 8   6   6  1997/32
=  +  +  
 4   9   10  Given that AB = 5i + 3j and AC = 2i + 5j, and find CB
 8  A. 7i + 8j B. – 3i + 2j C. 3i – 2j
=  A D. 3i + 8j E – 7i – 8j
 15 
417
Solution
Sketching the vector triangle diagram noting the = 1 49 and = 25  25
direction of the arrows 50 50
B = and =

OA = OB QED

(b) (i) OC = 2O A
A C = 2( i + 7j )
From the flow of the arrows = 2i + 14j
AC + CB = AB
( ii ) OD = O A + OB
CB = AB - AC = ( i + 7j ) + ( 5i + 5j )
= ( 5i + 3j ) – ( 2i + 5j ) = 6i + 12j
= 5i – 2i + 3j – 5j
(c) First we find BC = OC – OB
= 3i – 2j (C)
2004/7 = ( 2i + 14j ) – ( 5i + 5j )
The position vectors of points P and Q relative to the origin = – 3i + 9j
are: p = – i – 4j , q = 2i – 4j Next, angle between O D and B C
( a )Find a unit vector parallel to the resultant of p and q
( b ) Calculate 2 p  3q , leaving your answer in surd form. OD . BC
Cos  =
Solution OD BC
( a ) Resultant of p and q = p + q
 6i 12 j .  3i  9 j 
6  (3) 
= (–i – 4j ) + 2i – 4j Cos  =
= – i + 2i – 4j – 4j  12 2
2 2
 92
= i – 8j
 18  108
i 8 j
Unit vector parallel to pq =
65
=
 180  90 
1 ( i + 8j) 90
=
65
=
 2  9  10   9 10 
(b) 2 p  3q Factor out common terms in denominator
First 2p – 3q = 2(–i – 4j ) – 3( 2i – 4j ) 90
= - 2i – 8j – 6i + 12j =
= - 8i + 4j
9  9 10  10  2
90
Thus, 2 p  3q = (8) 2  4 2 =
9  10  2
= 6416 1
Cos  =
= 80 i.e 4 5 in surd form 2
2004/ 17 From triangle ratio of special angles  is 450 or proceed as
The position vectors of points A and B relative to the
 = Cos -1 1
origin O are O A = i + 7j O B = 5i + 5j 2
In calculator press 2 , press its inverse, then press Cos-1 of it
( a ) Show that O A and O B have equal magnitudes
= 450
( b ) If points C and D have position vectors given by:
(d) Unit vector in direction of D O
OC = 2 OA
Readers to complete note that: DO = – O D
OD = O A + OB 2005 / 7 June
express, in terms of i and j
Given that OC = a – b and OD = 2a + 3b,
(i) OC ( ii ) OD
where a = 2i + 3j and b = 3i – 2j , find C D
(c) Calculate the angle between O D and B C Solution
Let us sketch the diagram following the directions of vectors
(d) Find a Unit vector in the direction of D O
arrow.
Solution C

( a ) To show that OA = OB
b
a-

First O A = 12  7 2 and O B = 52  52 O D
2a + 3b

418
Solution
By Parallelogram law of vectors
( i ) If P,Q and R lie on a straight line then
OC + CD = OD
CD = OD – OC the vector PQ is same as the vector QR
= ( 2a + 3b ) – ( a – b )
= 2a + 3b – a + b PQ = q – p
= a + 4b = (5i + 5j ) – ( 7i + 2j )
= ( 2i + 3j ) + 4( 3i – 2j ) = –2i + 3j
= 2i + 3j + 12i – 8j and QR = r – q
= 14i – 5j
= ( 3i + 8j ) – (5i + 5j )
= –2i + 3j QED
2005 / 35
In a triangle ABC, A B = 3i – 4j and A C = –2i – 7j. ( ii ) p = 7i + 2j and q = 5i + 5j
then np = 7ni + 2nj mq = 5mi + 5mj
Find B C . But np + mq = 7i – 8j
A - 5i + 11j B - 5i + 3j C 5i – 3j D 5i + 11j  7ni + 2nj + 5mi + 5mj = 7i – 8j
Solution Equate terms in i to i and also do same for j
Let us sketch the diagram following the directions of 7n + 5m = 7
vectors arrow. – (2n + 5m = – 8 )
A
5n = 15
n=3
Substitute for n = 3 into 7n + 5m = 7
7(3) + 5m = 7
C
m = –14/5
B

By parallelogram law for vectors 1998/14a (Nov)


AC = A B + B C The position vectors of points P, Q and R are:
 2  4  5
– 2i – 7j = 3i – 4j + B C p =   q =   r =   , respectively
3
  1
    3 
(– 2i – 7j ) – ( 3i – 4j ) = B C Show that P, Q and R are concurrent
Solution
– 5i – 3j = B C We are to prove collinearity or parallel properties

2005 / 16 ( b ) i.e PQ = QR or PQ = t QR where t is a scalar


The position vectors of points P, Q and R in the x - y PQ = q – p
plane are O P = 3i + 4j , O Q = 5i + 6kj and O R = 7i  4  2
+ 2j respectively, where k is a scalar. If the resultant of =   –  
 1   3
O P and O R is perpendicular to O Q , find the value
 2
of k . =  
Solution  4 
First, resultant of O P and O R = O P + O R QR = r – q
= (3i + 4j ) + ( 7i + 2j )  5  4
= 10i + 6j =   –  
  3   1 
Next, resultant of OP and OR perpendicular to O Q
 1
Implies ( OP + OR) ( OQ) = 0 =  
( 10i + 6j )  ( 5i + 6kj ) = 0  2 
10 × 5 + 6 × 6k = 0
Thus PQ = 2 QR QED
50 + 36k = 0
36k = –50
k =  50 =  25
36 18

2003/16b
The position vectors of points P, Q and R are
p = 7i + 2j, q = 5i + 5j and r = 3i + 8j respectively
( i ) show that P, Q and R lie on a straight line
( ii ) find scalars n and m such that 7i – 8j = np + mq
419
Problems on vectors and plane shapes 2002/5 Neco
Three vertices A, B and C of a triangle have position vectors
1992/34 (Nov)
a, b and c respectively relative to the origin O,
If PQRS is a quadrilateral whose sides represent
where a = 3i – 2j , b = 6i + j and c = 9i + 4j
vector. PQ, is equivalent to ( i ) Show that ABC is isosceles
(ii) determine the value of cos BĈX if X is the
A RP + RQ B. RS C. PS + SR + RQ
mid point of AC ( give your answer 3 s.f)
D. PS + QS E. PR - RQ Solution
Solution Isosceles triangle has two equal sides .
Let us take a case of parallelogram Lets sketch as :
Q A
R

B C
P The three lengths are :
S
AB = b – a
The PQ, is equivalent to PR - RQ = (6i + j) – ( 3i – 2j )
By Pythagoras rule; = 3i + 3j
recall that angle at Q is 900 and PR is the hypotenuse
AC = c – a
Also PQ, is equal to SR and not RS = (9i + 4j ) – (3i– 2j)
= 6i + 6j
1993/37 (Nov) BC =c–b
ABCD is a rectangle with AB = 2a and AD = 2b
= ( 9i + 4j) – (6i + j)
Find the magnitude and direction of the
= 3i + 3j
vector AB – AD
A. 2a – 2b in the direction AC Since AB = BC ABC is isosceles
B. 2 a 2
b  in the direction DB
2

C. 2 a 2
 b  in the direction BD
2
(ii ) x = mid point
D. 2 a 2
 b  in the direction AC
2 A  a is 3i – 2j
C  c is 9i + 4j
E. 2 a  b in the direction DB 3  9 12
Solution i for AC = i.e i.e 6
C
2 2
B
 24 2
j for AC = i.e 1
2a 2 2
Midpoint vector at x is 6i  j
A D
2b
We have gotten BC as = 3i + 3j
AB – AD = 2a – 2b
Next we find x = C - x
Magnitude = 2a  2b = 9i + 4j – ( 6i + j)
= 9i + 4j – 6i – j
= 2a 2  2b2
= 3i + 3j
= 4a 2  4b 2
Thus cos BCx is the angle b/w BC and  c
= 
4 a b
2 2
 BC .  c = Bc  c cos
= 2 a 2
b 
2
3i  3 j . 3i  3 j 
(b) Direction of AB – AD
B
B
3 2
 32 3 2
 32  = cos
AB normal flow of
The resulting diagram
arrow from A to B 99
By vector’s Law of triangle = cos
-AD negative flow of
arrow from D to A
18
A
cos = 1
D
A D  = cos–1 1
Thus 2 (a 2  b 2 ) in the direction of DB (B)  = 00
420
2002/12 Neco 2005/17 (Nov)
PQRS is a parallelogram whose vertices are P(x, y ), The position vectors of points P, Q , R and S relative to the
Q(5, 7), R(-3, -8) and S(-4, -6) origin are
  2 ,  0 ,  3  and  1  respectively.
(i) Express in component form PQ, SR, QR and PS        
 1   1  2  4
(ii) Hence, find the values of x and y.
(iii) Calculate the magnitude of PR (a) Show that PQRS is a rectangle
(b) Find the perimeter of the figure
(iv) Find the unit vector parallel to PR., Solution
Solution (a) Let P, Q, R and S be sketched as:
Q( 5, 7) Q R
R(-3, -8)

P(x, y) P S
S(-4, -6) If it is a rectangle then
PQ, = q – p Q P = R S and Q R = P S
=(5–x,7–y)
QP = p – q and RS = s – r
Also SR =r–s   2 0  1  3
= [– 3 – (– 4 ), – 8 – (– 6) ] =   –   =   –  
= (1, – 2 )  1   1  4  2
QR = r – q   2   2
=   =  
= (– 3 – 5, – 8 – 7)  2   2 
= (– 8, – 15)
QP = RS QED
PS =s–p To confirm, let us try Q R = PS
= (– 4 – x , –6 – y )
QR =r–q and P S = s – p
By properties of parallelogram which says opposite
sides are equal =  3  –  0  =  1  –   2 
       
 2  1  4  1 
PQ, = SR
=  3  =  3 
5 – x = 1 and7–y=–2  
 3
 
 3
x = 4 and y=9
Verified as Q R = P S
Alternatively, we equate sides QR and PS Thus PQRS is a rectangle
– 4 – x = – 8 and – 6 – y = 15
x= 4 and y = 9 confirmed (b) Perimeter of rectangle = 2( L + b )
= 2 ( QR + RS )
(iii) Magnitude of PR = PR  3    2 
= 2     
First, we get PR = r – p But P = (4, 9)  3   2 
= (–3 – 4, –8 – 9) = 2  1 
= (–7, –17 )  
 5
=  2 
Thus, PR = (7) 2  (17) 2 10 
 

= 49  289
= 338
= 18.38
(iii) Unit vector parallel to PR
it simply means the unit vector of PR
PR
=
PR
1
=  7i 17 j 
18
421
Problems on vectors and coordinate geometry
PT =T–P
1992/39 Nov
If the position vectors of P and Q are a and b = ( i – 2j ) – ( 3i – 5j )
respectively, then the position vector of the point R on = i – 3i – 2j + 5j
PQ produced such that PR : RQ = 5: 2 is = – 2i + 3j ( A )
1 1 1
A. (5b – 2a ) B. (5a – 2b) C. (5a + 2b 2003/ 16
3 3 7 Find the coordinates of the point which divides the line
) joining the points A ( 4i – 3j ) and B ( -2i – j ) internally in
1 1 the ratio 5 : 3
D. (5b + 2a ) E. (5b – 2a)
7 10 A  1 ,  7  B (-1, 7 ) C  1 ,  11  D ( 2, -11)
   
Solution 4 4  2 4 
By section formula vector a b Solution
Ratio m n m1 x2  m2 x1
mb  na Interval division ratio is x =
P= m1  m2
mn m1 y 2  m2 y1
5b  2a y=
= m1  m2
5 2 Here x is i and y is j and m1 is 5 while m2 is 3
=
1
(5b + 2a) (D) 5  (2)  3  4
7 Thus, x=
53
1999/30 (Nov)
The position vectors of points F and G with respect to a  10  12 2 1
= = i.e.
fixed point O are (3i – 2j ) and (i + j ) respectively. 8 8 4
If P divides FG in the ration 5: 3, find the position 5  (1)  3  (3)
vector of P. y=
53
1 11 9 7 1
A. i  j B. i  j C. 7i  j 59  14 7
2 8 4 8 2 = = i.e.
7 1 8 8 4
D. i  j 7
4 8 Coordinates is  1 ,  (A)
Solution 4 4 
By section formula 2003/4 Neco
mb  na If P = 3i + 4j, Q = 4i + 3j are two position vectors.
P= Find the position vector that divides the line PQ in the ratio
mn of 3 : 1
3(i  j )  5(3i  2 j ) Solution
= M 1 = 3, M 2 = 1
53
m1 x 2  m 2 x1
3i  3 j 15i 10 j i for PQ =
= m1  m 2
8
18i  7i 3  4 1 3
= =
8 3 1
9i 7 12  3 15
=  j (B) = i.e
4 8 4 4
2003/27 (Nov) midpoint, co-ordinate vector
The position vectors and points P, Q and R are
m1 y 2  m 2 y1
j for PQ =
3i – 5j , –2i + 3j and 4i – 7j respectively. If T is the m1  m 2
mid – point of QR, Find P T
A - 2i + 3j B 3i – 5j C -2i + 5j D -4i + 6j 3  3 1 4
=
Solution 3 1
 x1  x2 y1  y 2  94 13
Co-ordinates of mid point  ,  = i.e
 2 2  4 4
Mid point of QR co-ordinates =   2  4 , 3  7 
1
 2 2  Position vector PQ that divide in 3 : 1 = 15i 13 j 
=(1,–2) 4
i.e T = i – 2j in vectors representation
422
2005/8 ( Nov)   14  – 1  2  –1
The position vectors of the points A, B, C, and H A   ms B   ms
are 3i + 2j, 2i + 3j, 3i – 2j, and 8i + 3j respectively.  15   1 
If D is the mid point of BC,
 4  –1  14  – 1
( a )find the position vector of D. C   ms D   ms
  9    9 
( b )show that AH =2 O D where O is the origin
Solution
Solution u, a, and t are given,; we are to find v
D is mid point of BC i.e b = (2i + 3j) and c = (3i – 2j) v = u + at will fit into the values
x for D x 2  x1 y 2  y1
= y for D =  5   3 
2 2 =   + 3  
23  3    4 
3 2
= =   5   9   4  –1
2 2 =   +   =   ms C
5 1  3   12   9 
= =
2 2
2003/25(Nov)
5 1 A particle starts from rest, it’s acceleration at any time t
Thus, position vector D is i + j
2 2 seconds is given by a = (2t + 1)i + 3t2j. Calculate its velocity
at t = 3
( b ) Show that AH = 2 OD A 9i – 12j B 12i + 9j C 12i + 27j D 7i + 9j
Solution
First we find AH and OD Velocity can be gotten from acceleration by integration
AH = h–a dv
Acceleration is = (2t + 1)i + 3t2j
= ( 8i + 3j ) – ( 3i + 2j ) dt
= 5i + j
and velocity is ∫d v = ∫ [ (2t + 1)i + 3t2j ] dt
and OD =d–o  2t 2  3t 3 3
5 1 V =   t  i + j
= i + j – ( 0i + 0j )  2  3 0
2 2 At rest, time t is zero
5 1 V = 12i + 27j (C)
= i + j
2 2 2002/13 (Nov)
The displacement, in metres, of a particle at any time
Next, AH = 5 2  12 t second is x = t3i – 3t2j. Find, in ms – 1 , the magnitude
of its velocity at t = 3.
= 25  1 = 26
A3 5 B3 6 C9 13 D 27 2
  5   1  
2 2
Also 2 O D  = 2      
Solution
 2 2  Velocity can be gotten from displacement by differentiation
 Displacement x = t3i – 3t2j
 25 1 
=2    Velocity is
dx
= 3t2i – 6tj
 4 4 
 dt
The velocity at t = 3
26
= 2 V = 3( 32 )i – 6( 3 )j
4 V = 27i – 18j
2 27 2  18 2
= 26 i.e 26 Magnitude v i.e. v =
2
= 729  324
Thus, AH =2 OD = 26 QED
= 1053
Problems on vectors and motion = 9  9  13
2010/37 =9 13 ms – 1 (C)
 5 
The initial velocity of an object is u =   ms – 1. 2000/8
 3  A particle moves in a plane such that its
 3  –2 displacement(s) given by s = (t2 + t) i + (3t – 2) j Find;
If the acceleration of the object is a =   ms and (a) The velocity, (b) The acceleration,
 4  (c) The speed at t = 2 seconds of the particle.
it moved for 3 seconds, find the final velocity.
423
Solution Solution
ds Change in momentum = m (v – u)
(a) v is = (2t + 1) i + 3j  2    3 
dt = 3     
dv  3   4 
(b) Acceleration  = (2)i
dt = 3  5 
 
 1
(c) The speed at t =2 seconds
ds =   (C)
 15
  3
Implies where t = 2  
dt 2003/7 a
Speed = [ 2( 2 ) + 1 ] i + 3j
= 5i + 3j
A metallic ball (A) of mass 2kg moving with velocity 8 ms-  6

1
2009/8 , collide with another ball (B) of the same material but of
The position vector of a body with respect to the origin mass 1.5kg. If ball (B) is initially at rest, but moves with
is given by r = 4ti + (12 – 3t)j at any time t seconds   3
velocity 5 ms1 after collision, find the velocity with which
(a) Find the velocity of the body ball (A) moves.
(b) Calculate the magnitude of the displacement Solution
between t = 0 and t = 5 We apply common sense sketching
Solution 6 i 3
8 m/s U=0 j ? -5 m/s
( a ) velocity can be gotten from displacement by differentiation
A B A B
r = 4ti + (12 – 3t)j
2kg 1.5kg 2kg 1.5kg
dr
velocity is = 4i + (0 – 3)j By law of conservation of momentum
dt
= 4i – 3j
2   + 1.5   = 2   + 1.5  
6
8
0
0
i
j
3
5

V = 4 2  (3) 2 = 25 = 5ms-1


 12 
 =2 j +  
i

 4.5 
 16   7.5 
( b ) Displacement r = 4ti + (12 – 3t)j
At t = 0
r = 0i + (12 – 0 )j
 12   4.5 
 –   =2 j
i

 16   7.5 
= 12j
Thus r = 12
2
i.e 12m
 7. 5 
  =2 j
i

At t = 5
 23 . 5

r = 20i + (12 – 15 )j
= 20i – 3j
1  7 .5 

  = j thus, j =   m/s
2  23.5 
i i

 3.25 
 11.75 
Thus r = 20 2  (3) 2 2000/30
= 20.22m A particle of mass 2kg at rest is acted upon by forces 5i – 3j
Magnitude of the displacement between and i – 5j . Find its velocity at time 3 seconds.
t = 0 and t = 5 is 20.22 – 12 = 8.22m A. 6i – 8j B. i – 4/3 j C. 9i – 12j D. 6i + 3j
Solution
2005/12 Neco (Dec) Exercise 35.16 u = 0, v = ?, t = 3s
Given the displacement r(t) = (t2 – 5t)i – t2j of a particle Resultant force F = (5i – 3j) +( i – 5j) i.e 6i – 8j
as a function of t, evaluate its: F= m(v–u)
(i) Velocity after time t seconds, t
(ii) Speed and direction at time t = 2s 6i – 8j = 2 ( v – 0 )
(iii) Acceleration after time t second, and 3
(iv) Magnitude and direction of acceleration at time t = 2s 3(6i – 8j ) = 2v Thus v = 9i – 12j ( C )
1999/31 (Nov) Exercise 35.17
A particle of mass 3kg, resting on a smooth horizontal
Problems on vectors and forces I surface, is acted upon by forces (2i – 5j), (2i + 3j) and
2004 / 27 (– i – 7j), find its velocity after 5 seconds
A particle of mass 3kg has an initial velocity A 3i–9j B i – 3j C 5i – 15j D 3i – j
  3 – 1  2 – 1
u =   ms and a final velocity v =   ms 2000/32 Neco Exercise 35.18
 4  3 The following vectors ( 5i + 2j), (2i+4j) and (3i – j) act on
Find, in N, the change in momentum. a particle of mass 5kg which rest on a smooth horizontal
plane. What will be the velocity of the particle after
  3  3  15   15  3 seconds?
A   B   C   D  

  3  21 
  3 21
  A 3i – 6j B 6i – 3j C 3i + 6j D 5i + 2j E 6i + 3j
424
Problems on vectors and forces II Solution
2004/ 7 For equilibrium | F1 + F2 + F3 | = 0
Express p = (5N, 0600) in the form ai + bj, F1 + F2 + F3 = – i + 5j + (p + 3)i + 2j + (4 – 3p)i – 7j
where a and b are scalars = – i + 5j + pi + 3i + 2j + 4i – 3pi – 7j
5 5 5 5 5 = ( 6i + pi – 3pi) + 0j
A 2i+2 j B 2i– 2j C 2 3 i + 52 j D5 3 i + 7j = ( 6 – 2p)i + 0j
Solution Thus, | F1 + F2 + F3 | = 0
(6  2 p) 2  0 2 = 0
( 6 – 2p) = 0
6 = 2p Thus, p = 3 ( C )

2000/12 Neco Exercise 35.19


The following four forces act on a particle T,
The force P = Px + Py
F1 = 3i + 2j – 5k; F2 = 3i – 5j + 2k
In vector notation P = Pi + Pj
F3 = 4i – 3j + 2k and F4 = –2i – 4j – 3k
Relating 5N to Px by trig ratio ( Adj and Hyp)
measured in Newtons
Cos 300 = Px
Find:
5
( i ) the resultant of the forces
Px = 5 Cos 300
( ii ) the magnitude of the resultant
5
= 2 3 ( iii ) the unit vector parallel to the resultant of the vectors
Similarly Py = 5 Sin 300 r1 = –3i + 9j – k
5 r2 = 6i –3j – k
= 2
5 5 2003/8 Neco Exercise 35.20
Thus, P = 2 3i + 2 j (C) Forces p = 4i + 2j and q = – 4i – 5j act on a body at rest.
2005/15 (a) Find : ( i ) the magnitude of the resultant force
Find the magnitude and direction of the resultant of ( ii ) the direction the body begins to move
a = 2i + 3j and b = 2i + j (b)Calculate, correct to 1 decimal place, the angle between p and q
A (4 2, 0300 ) B (4 2 , 0450 )
C B ( 2 3 , 0450 ) D ( 2 3 , 0300 )
Solution
Resultant vector = a + b
= (2i + 3j) + ( 2i + j )
= (4i + 4j)
Magnitude of resultant vector | a + b | = 42  42

=4 2
Resultant vector = (4i + 4j) is represented as:

The direction is  but  = 90 – 


From the diagram tan  = 4/4 i.e 1
 = tan-1 1
= 450
The direction is  = 90 – 450
= 450
Magnitude and direction of the resultant = ( 4 2, 0450 )
2004/8
Given that forces F1 = – i + 5j, F2 = (p + 3)i + 2j
F3 = (4 – 3p)i – 7j are in equilibrium, find the value
of the constant p
A–4 B 1/3 C 3 D4
425
Chapter Thirty Six …At most,…
Probability I and Logic The given number or less. For example; If Tosin asked his
This is a branch of mathematical statistics basically classmate to lend him at most N 200.00. The classmate can
concerned with drawing conclusions or inferences from as well give him N5, or N 10.00, N 190.00; as any of these
occurrence or experiments involving uncertainties. For amounts will solve his problem provided the money is not
any conclusion or inference made to be reasonably above N 200.00.
accurate an understanding of some probability theories
is essential. …, At least,…
What is the meaning of the statements such as “Nigeria The given number or more. If the “at most” is replaced by
will probably win the football match against Norway”. “at least” in the previous example then the mentioned
You will probably pass SSCE.There is a 30% chance amount of N 10.00 or N 190.00 or N 199.99 will not take
that this job will not be finished on time”. care of Tosin’s demand except N 200.00 or above.

In each case we are expressing the outcome of which General formula


we are not certain but because of part information or The probability of an event happening can be given a
from understanding of the structure or the experiment or numerical value x ; where
event bring about some degree of confidence in the x = number of required outcomes
validity of statement. Chances, possibly, likely, Success, number of possible outcome
failure etc are examples of words used in probability (a) When x = 1
statements. If the probability of any event is1, then it is sure or certain
to occur.
Definitions of basic terms used in probability
(b) When x = 0
Sample space If the value of x is 0 then the event is impossible. It will
A set whose element represent all possible out comes of never occur.
an experiment usually denoted by S
Eg 1 A single throw of a die (c) The range of solution to probability problem is
S = {1, 2, 3, 4, 5, 6} between 0 and 1 i.e 0  Pr (x)  1
The implication is that we do not have negative answers nor
Eg 2 The tossing of a coin once values greater than 1
S {H, T}
Eg 3 The tossing of two coins (d) For any event, the probability of success plus failure:
S = {HH, HT, HT, TT} Pr (Success) + Pr (failure) = 1
Pr (x) + Pr ( x ) = 1
Eg 4 The launching of a missile from a (e) The probability of failure or non - occurrence or
Submarine. S = {Success, failure} not happening :
Pr (x) = 1 – Pr ( x )
Event
Any Subset of the Sample space
Eg 5 Consider the experiment of tossing two coins General formula problems
S = {HH, HT, TH, TT} 1994/21
Then the event (E) that head appears on the first coin is A ball is drawn randomly from a bag containing 8 red, 4 blue
E = {HH, HT} and 2 green identical balls. Find the probability that the ball
drawn is not red
Outcomes
A. 1 B. 1 C. 3 D. 4 E. 7
It refers to all possible distinct results of a trial.
14 8 7 7 8
Considering the examples given so far under above
Solution
Examples Outcomes Pr (A ball drawn not red) = 1 – Pr(red ball)
=1– 8
1 6
14
2 2
= 1 – 4 i.e 3/7 ( C )
3 4
7
4 2
1994/24
The concept of outcomes is similar to cardinality in set The probability that Ade passes a mathematics test is p and
i.e n (A) or n(B) depending on the given set. that he passes a physic test is q. find the probability that he
passes the mathematics test and fails the physics test
Experiment A. q – pq B. p – pq C. p – q + 1
D. q – p + 1 E. pq
This is the process through which outcomes are
Solution
generated. The acts of throwing a die, tossing a coin,
Pr(Pass mathematics and fail physics) = p × (1– q)
e. t. c are examples of experiment. Experiment can also
be called trial. = p – pq (B)
426
1996/21 (Nov) Solution
If P(x) denotes the probability that event x will occur Probability (x) = Number of required outcome
and P(x1) denotes the probability that event x will not Number of possible outcomes
occur , which of the following is not true? Here; number of possible outcomes: {1, 2, 3, 4, 5, 6,} i.e 6
A. 0<P(x)<1 B. 0≤P(x) ≤1 Here; number of required outcome: {5} i.e 1
C. P(x) + P(x1) =1 D. P(x1)<0 E. 0<P(x1)<1 Pr (obtaining 5) = Number of 5
Solution Total numbers in a die
Option D is not true because it falls short of probability = 1/6 (A)
properties i.e Negative values are not part of
probability. The values of probability lies between 0 2002/9a i & ii NABTEB om
and 1 as given by option A, B and E A bag contains 9, identical balls, out of which 3 are blue,
The sum of any probability and its not is equals one option c. 2 are white and the remaining are red. If a ball is drawn
from the bag, what is the probability that it is
2004/48 Neco Dec (i) blue ? (ii) not red ?
15 balls in a bag are made up of red and blue balls, if 6 Solution
of the balls are red. How many red balls must be added Pr (blue) = Number of blue balls
to the bag so that the probability of red balls will be 2/3? Total number of balls
A. 5 B. 6 C. 8 D. 12 E. 15 = 3/9
Solution = 1 /3
Presently Pr(red ball) = 6 (ii) Pr (Not red) = 1 – Pr(red)
15 But Pr(red) = Number of red balls
The new number of balls x will affect both numerator Total number of ball
(number of red balls ) and Denominator i.e = 4 /9
total balls in the bag.
Therefore, Pr (Not red) = 1 – 4/9
6 x 2 = 5/9

15  x 3
3(6 + x) = 2(15 + x) 2004/37 NABTEB om
18 +3x = 30 + 2x What is the probability of throwing an odd number with
3x – 2x = 30 – 18 a fair die ?
x = 12 (D) A.1/6 B.1/4 C.1/3 D1/2
Solution
2007/ 25
Pr(odd) = no of odds in a die
Let P be a probability function on set S,
Total number of face in a die
Where S = (a1, a2, a3, a4). Find P(a1) if P(a2)= 1/4 ,
Here: No of faces in a die {1, 2, 3, 4, 5, 6} i.e 6 of them
P(a3) = 1/5 , P(a4) = 1/6
A1/ 60 B. 1/7 C. 1/3 D 23/60 E. 37/60 No of odds : {1, 3, 5} i.e 3 of them
Solution Therefore Pr (odd) = 3/6
Applying the axiom that the sum of all probability is one. = 1 /2 (D)
P(a1) + P(a2) + P(a3) + P(a4) = 1
2004/36 NABTEB (Dec) om
P(a1) + 1 + 1 + 1 = 1
If today is Tuesday, what is the probability that the next day
4 5 6
will be Wednesday ?
P(a1) + 15 + 12 + 10 = 1
A.0 B.1/7 C.1/6 D.1
60
P(a1) + 37 = 1 Solution
60 Pr (Wednesday ) = 1 (D)
This is a sure event, the next day after
P(a1) = 1 – 37 i.e 23 (D)
60 60 Tuesday is certainly Wednesday
1993/21 (Nov)
Find the total number of possible outcomes in a throw VTR – 11/41 NTI TCII om
The numbers 1, 2, 3, 4,… 17 are placed in a container and
of three dice
A. 3 B.18 C.36 D.64 E.216 one number is drawn at random .What is the probability
that the number is odd ?
Solution
One die: six faces; possible outcome 6 i.e 61 A.1/17 B.2/17 C.3/17 D.4/17 E.9/17
Two dice: Six faces each; possible outcome 36 i.e 62 Solution
Three dice: Six faces each; possible outcome must be 63 i.e 216(E) Pr(odd) = number of odds
Total numbers
Total no.: {1, 2, 3, 4, 5, 6, 7, 8, 9, 10, 11, 12, 13, 14, 15, 16, 17, }
2001/36 NABTEB om i.e 17 of them
If a perfect die is cast, what is the probability of Odd numbers : {1, 3, 5, 7, 9, 11, 13, 15, 17} 9 of them
obtaining a 5 ?
A.1/6 B.1/3 C.1/2 D.5/6 Thus, Pr (odd) = 9/17 (E)
427
Theoretical probability 2000/41 Neco
Theoretical probability is the probability calculated Two fair dice are tossed, what is the probability of NOT
without the experiment being performed, i.e using only getting a total of 8 ?
information that is known about the physical situation A. 8 B. 31 C. 5 D. 2 E. 1
9 36 6 3 3
EXAMPLES Solution
1. Two number are chosen at random from 1,3, 6. Outcome table is
Find the probability that the sum of the two is not odd.
Solution + 1 2 3 4 5 6
Arranging the given values in a tabular form, we have 1 2 3 4 5 6 7
2 3 4 5 6 7 8
+ 1 3 6.
3 4 5 6 7 8 9
1 2 4 7
4 5 6 7 8 9 10
3 4 6 9
5 6 7 8 9 10 11
6 7 9 12
6 7 8 9 10 11 12
Pr (not odd) = 1 – Pr (odd)
Pr(Not a total of 8) = 1 – Pr(total of 8)
=1–4
= 1 – 5
9
5 36
= /9
= 31 (B)
2. Find the probability that a number selected at random 36
from 41 to 56 is a multiple of 9.
Solution 2002/37 NABTEB om
Listing the numbers: A coin and a die are tossed up together once. What is the
41, 42, 43, 44, 45, 46, 47, 48, 49, 50, 51, 52, 53, 54,55, 56, probability of a head and a six showing up ?
Total numbers =16 i.e. (56 – 41) + 1 A.1/12 B.1/6 C1/2 D.2/3
The multiples of 9 are two i.e. 45 & 54 Solution
Hence Pr (multiple of 9) = 2/16 = 1/8 The sample space is shown below:
1992/21 (Nov) Die
Two fair dice, each of 6 faces numbered 1, 2, 3, 4, 5, 6 1 2 3 4 5 6
are tossed once. Find the probability of getting a sum H H1 H2 H3 H4 H5 H6
greater than 10 Coin T T1 T2 T3 T4 T5 T6
A. 1 B. 1 C. 1 D. 1 E. 2
18 12 6 2 3 Total sample space is 12
Solution Number of Head and a six is 1
We prepare the sample space as shown below Pr(H6) = 1/12 (A)

+ 1 2 3 4 5 6 Alternatively:
1 2 3 4 5 6 7 Apply multiplication rule
2 3 4 5 6 7 8 Coin Die
3 4 5 6 7 8 9 Pr(H) = ½ Pr (6) = 1/6
4 5 6 7 8 9 10
Thus Pr( a head and a six) = Pr(H) x Pr(6)
5 6 7 8 9 10 11
= 1/ 2 x 1/ 6
6 7 8 9 10 11 12
= 1/12 (A)
Pr (sum greater than 10) = 3 i.e 1 (B) 2004/38 NABTEB (Dec) om
36 12 Two fair coins are tossed once. The probability of getting
2001/38 NABTEB om at least one tail is
If two fair coins are tossed once, What is the probability A.0 B.1/4 C.1/2 D.3/4
that at least a head shows ? Solution
A. ¼ B.1/2 C.3/4 D1 The sample is shown below:
Solution Coin I
The sample space is demonstrated in the table below H T
Coin I H HH HT
H T Coin II T TH TT
H HH HT
T TH TT Pr ( at least one tail) = Pr ( one tail)
= HT + TH + TT
Pr( at least a head ) = Pr ( one head) Total sample space
= HH + HT + TH
=¾ (D)
Total sample space
= 3 /4 (C)
428
Empirical (experimental) probability 2003/41 Neco om
Empirical (Experimental) probability is calculated using A die is tossed 100 times and the results are recorded in
the results of an experiment that has been performed a the table below. Calculate the probability of obtaining a
number of times. prime number.
Number 1 2 3 4 5 6
In Experimental probability on die
Pr (E) = f (E)
Frequency 15 19 24 12 22 8
f
Where f (E) is the number of times the event E occurred A.4/5 B13/20 C.23/50 D.7/20 E 1/5
f is the total frequency Solution
Pr(prime ) = f (prime numbers)
EXAMPLES f
1. Some families were measured by the number of Prime numbers here are: 2, 3, 5
children they have; to give the following frequency f 19 f 24 f 22
distribution Total freq (f) : 100 times
No of 0 1 2 3 4 5 or Pr (prime ) = 19 + 24 + 22
children more 100
No of 12 28 22 8 2 2 = 65/100
families = 13/20 ( B )
Use this information to calculate the probability that
another family of this type will have
(a) 2 (b) 3 or more (c) less than 2 children 2004/56 Neco om
Solution The table below shows the number of goals scored by 32
Here, f = 74 football teams in a world cup tournament. Use the
(a) Pr (2 children) = f (2) children information to answer question 56.
f Number 1 2 3 4 5 6 7
= 22 = 11 on die
74 37 Frequency 4 1 8 7 5 4 3
(b) Pr (3 or more) = f (3) + f (4) + f (5) 56. Find the probability that if a team is chosen at random
f that it scored at MOST 4 goals
Pr (3 or more) = 8 + 2 + 2 A.7/32 B.1/3 C.4/7 D.3/8 E.5/8
74 Solution
= 12 = 6 Pr ( at most 4) = Pr ( 4)
74 37 = F (4) + f(3) + f (2) + f (1)
f
(c) Pr (less than 2 children) = f (1) + f (0) =7+8+1+4
f 32
= 28 + 12 = 20
74 32
= 5/8 (E)
= 40 = 20
74 37
2. The table below shows the amount of money, which
some students possess in their wallet at a given time.
2004/29 Neco (Nov) om
Amount in N 1 2 3 4 5 6 The ages of some pupils are given in the table below.
No of 1 3 2 5 1 4 Age 7 8 9
students
Frequency 5 9 7
If a student is chosen at random from the group, what is
What is the probability that a pupil chosen at random is
the probability that the student has
not 8 years old ?
(a) N 4 (b) N 2
A.0.238 B.0333 C.0.381 D.0.429 E.0.571
Solution
Solution
(a) Pr (N 4) = f (N 4)
Pr( Not 8 years) = 1 – Pr (8 years)
f But Pr (8 years) = f(8 years)
Here f is the total number of students f
= 5
= 9/21 i.e 3/7
16
(b) Pr (N2) = f (N 2) Thus, Pr( Not 8 years) = 1 – 3/7
f = 4/7
=3 = 0.571 (E)
16
429
Addition Rule = 6/36 + 2/36
Under this subheading addition rule we have twin = 8/36
concepts namely mutually Exclusive (M.E) and non- = 2/9 (B)
mutually Exclusive Events. The words or & either are 2003/40 Neco om
used in both cases. In a basket of fruit there are 6 grapes, 11 bananas and 13
oranges. If one fruit is chosen at random, what is the
(a) Mutually Exclusive events (M.E) probability that the fruit is either a grape or banana ?
Two events A and B are M.E if the occurrence of any
A.17/30 B.11/30 C.6/30 D.5/30 E11/150
one of them excludes the other.
Solution
Mathematically
Total fruit 6 + 11 + 13 i.e 30
Pr (A or B) = Pr (AB) recall that
Pr( a grape or banana) = Pr (grape) + Pr ( banana)
Pr (AB) = Pr (A) + Pr (B) – Pr (AB) = 6/30 + 11/30
But Pr (AB) =  Since nothing is common to both = 17/30 ( A)
Hence Pr (AB) = Pr (A) + Pr (B) M. E 2003/43 Neco om
Diagrammatically, M.E events A & B are represented A ball is drawn at random from a box containing 6 red balls,
below. 4 white balls and 5 blue balls. Determine the probability that
A B it is red or white.
A.24/15 B.2/3 C.2/5 D.4/15 E.8/75
Solution
Total balls : 6 + 4 + 5 i.e 15
Pr (red or white) = Pr ( red) + Pr ( white)
Examples of M.E Events = 6/15 + 4/15
(a) The events of going to School by foot and by car = 10/15
are M. E; This is so because no single student can go to = 2/3 (B)
school by foot and at the same time by car . Impossible! 2004/59 Neco (Nov) om
(b) The events of cooking a particular pot of soup using There are 5 red, 7 blue and 6 green biros in a packet. What is
stove and gas cooker at the same time are M. E the probability of a boy picking a green or red biro closing
his eyes?
(c) The events of the current principal of a Government A.30/324 B.5/18 C.6/18 D.7/18 E.11/18
college Ughelli to be serving and be retired at the same Solution
time are M.E Total biros: 5 + 7 + 6 i.e 18
E.g. 1. A haulage contractor has 3 type A, 2 type B and Pr ( green or red biro) = Pr ( green ) + Pr (red)
7 type C lorries available for deliveries, all of which = 6/18 + 5/18
are used frequently. What is the probability that a lorry = 11/18 (E)
delivering a load will be of type A or C? 2004/35
Solution The table below gives the marks scored by a group of
If the contractor decides to use lorry A for the supply, students in a test. Use the table to answer question 35
automatically he cannot use lorry type C; thus, M. E 35. What is the probability of selecting a student from the
Pr (A or C) = Pr (A) + Pr (C) group that scored 2 or 3 ?
= 3 + 7
12 12 Mark 0 1 2 3 4 5
= 10 = 5 Frequency 1 2 7 5 4 3
12 6
2003/37 NABTEB om A.1/11 B.5/22 C.7/22 D.6/11
Two dice are thrown at the same time. What is the Solution
probability that the sum will be 7 or 11 ? Pr( mark 2 or 3) = Pr( mark 2) + Pr( mark 3)
A.1/9 B.2/9 C.1/3 D.4/9 = f(2) + f(3)
Solution f f
The sample space is as shown below : =7 + 5
22 22
sum 1 2 3 4 5 6 = 12 =6
1 2 3 4 5 6 7 22 11 (D)
2 3 4 5 6 7 8
3 4 5 6 7 8 9 VTR – 12/ 6a NTI TCII om
4 5 6 7 8 9 10 A number is chosen at random from the set 15, 16, 17,… 32.
5 6 7 8 9 10 11 What is the probability that the chosen number is
6 7 8 9 10 11 12 (i) a multiple of 7 ?
(ii) a prime number ?
Pr( sum 7 or 11) = Pr (sum 7) + Pr(sum 11) (iii) a prime or a multiple of 7.
430
Solution Multiplication rule
Sample space : {15, 16, 17, 18, 19, 20, 21, 22, 23, 24, Under multiplication rule there is also twin concepts of
25, 26, 27, 28, 29, 30, 31 32} 18 of them independent and dependent events. The words “and” or
Multiple of 7 : { 21, 28} 2 of them “both” is used in both cases
Prime number: {17, 19, 23, 29, 31} 5 of them
Independent events
(i) Pr (multiple of 7) = 2/18 i.e 1/9 Two events A and B are said to be independent if the
(ii) Pr (prime number) = 5/18 occurrence or non – occurrence of event A does not affect
the probability of event B
(iii) Pr (prime or multiple of 7 ) = 2/18 + 5/18 Mathematically.
= 7/18
Pr (A and B) = Pr (A) x Pr (B)
(b). Non – mutually Exclusive events i.e. Pr (AB) = Pr (A) x Pr (B)
Two events A and B are non – mutually exclusive, if Examples
both events share something in common i.e. AB   1.Three balls are drawn successively from a box containing 8
Mathematically, Pr (A or B) = Pr (AB), red balls, 6 white balls and 4 black balls. Find the probability
Pr (AB) = Pr (A) + Pr (B) – Pr (AB) that they are drawn in order red, white and black if each ball
Diagrammatically, non – M. E events A and B is is:
represented below. (i) Replaced.
A Solution
B You will agree with the author that once a ball is removed
and replaced it does not affect the 2nd draw. Hence this is an
independent event.
Pr (RWB) = Pr (R) x Pr (W) x Pr (B)
8 6 4
E.g. If an event A implies obtaining prime number and 18 18 18
event B implies obtaining an odd number in a single = 8
throw of a die. Find the probability of events A or B 243
Solution VTR –12/46 NTI TCII om
S = {1,2, 3, 4, 5, 6} A box contains 2 red balls and 4 blue balls. A ball is drawn
A = {2, 3, 5} and B = {1, 3, 5}, also AB = {3, 5} at random from the box and then replaced before a second
Pr (A) = 3/6 Pr (B) = 3/6 Pr (AB) = 2/6 ball is drawn. Find the probability of drawing 2 red balls.
A1/9 B.1/4 C.1/3 D.4/9 E.2/3
Thus Pr ( A or B) = Pr (A) + Pr (B) – Pr (AB) Solution
= 3 /6 + 3 / 6 – 2 / 6 The word “replaced” after a ball is drawn shows that this
= 3+3–2 multiplication is independent
6 Thus, Pr (RR) = Pr(R) x Pr (R)
= 4 /6 = 2 / 3 = 2/6 x 2/6
= 1/9 (A)
Note VTR – 11/42 NTI TCII om
When any of the word or & either is used, student A pair of fair dice with each face numbered 1 to 6 are thrown
should not jump into solving for addition. Rather, they once. Find the probability of scoring a 2 on one die and a 5
should first identify the class of the problem (i.e. M. E on the other
or non – M. E ). A.1/36 B.1/18 C.7/36 D.1/3 E.2/3
VTR- 12/45 NTI TCII om Solution
There are twelve cards numbered 1 to 12. A card is “and” shows multiplication rule. The probability of scoring
selected at random. What is the probability that it is any number on one die is independent of scoring a number
either even or a perfect square ? on the second die. Thus
A.1/8 B.1/4 C.7/12 D.2/3 E.3/4 First die 2nd die
1
Solution Pr(2) = /6 Pr(5) = 1/6
Sample space: {1, 2, 3, 4, 5, 6. 7, 8, 9, 10, 11, 12} Pr (2 and 5) = Pr(2) x Pr(5)
12 of them
= 1/6 x 1/6
Even cards : {2, 4, 6, 8, 10, 12,} 6 of them
= 1/36 (A)
Perfect square cards: {4, 9,} 2 of them
2003/11a NABTEB om
EP = {4} 1 of them A bag contains 3 black balls and 2 red balls. A ball is picked
Pr( even or perfect square) = Pr (E) + Pr (p) – Pr (EP ) at random from the bag and then replaced. A second balls is
= 6/12 + 2/12 – 1/12 chosen at random, What is the probability that
=6+2–1 (i) they are both black ?
12 = 7/12 (C) (ii) one is black and the other is red ?
431
Solution 2002/9a (iii) NABTEB
The words “both” in (i) “and” in (ii) show A bag contains 9 identical balls, out of which 3 are blue, 2
multiplication rule. are white and the remaining are red.
(i) Pr(BB) = Pr(B) x Pr(B) (iii) If 2 balls are drawn at random one after the other, what
= 3/5 x 3/5 is the probability that both of them will be red , if there is no
= 9/25 replacement ?
(ii) Pr(B and R) = Pr(B) xPr(R) Solution
= 3/5 x 2/5 Number of red balls : 9 – 5 i.e 4
= 6/25 Pr (R1 /R2) = Pr (R1) x Pr (R2/R1)
= 4/9 x 3/8
Dependent events After the first draw, without replacement ; the next draw for
Two events A and B are said to be dependent, if the red will have 3 red balls left and the total minus one ball.
occurrence of event A affects the occurrence of B. then = 1/3 x ½
Pr(A and B) = Pr(A) x Pr(B/A) = 1/6
i.e Pr(AB) = Pr(A) x Pr(B/A)
The Pr (B / A) reads probability of B given that A has 2003/38 Neco om
occurred. Though there are cases where the candidate A packet contains 4 red, 5 blue and 6 black identical biros.
will represent the above by Pr(AB) = Pr(A) x Pr(B) Two biros are picket at random without replacement, find the
and still apply the sense of dependence in the working. probability of picking a red and a black biro.
It is acceptable at O/L. A.73/105 B.2/3 C.4/35 D.8/35 E.8/125
Solution
Examples Total biros: 4 + 5 + 6 i.e 15
a. From e g 1 under independent events lets impose a
Pr ( R and B) = Pr(R) x Pr (B/R)
second condition i.e
= 4/15 x 6/14
(ii) not replaced. The second draw is affected in the total because of the NO replacement
Solution = 4
/5 x 1/7
If each ball drawn is not replaced then the number left 4
= /35 (C)
inside the box must be affected; hence this case must be
dependent.
Pr (RWB) = Pr (R) x Pr (W/R) x Pr (B/RW) Problems on probability
1993/25 (Nov)
= 8 x 6 x 4 A bag contains 6 red and 5 green identical balls. Two ball
18 17 16 are drawn at random one after the other without replacement
what is the probability that both balls are same colour?
8+6+4 7+6+4 7 +5 + 4 A. 1 B. 2 C. 3 D. 5 E. 1
= 2 11 11 11 11 11
51 Solution
Analysis Pr(Both balls are same colour) = RR or GG
In the first draw, the total balls and the number of red Take note of “without replacement” and see how we work
balls is not affected. In the 2nd draw for white the total based on that
balls has reduced as a result of the non – replacement of
the first drawn ball but the number of white balls is not = 6 × 5 + 5 × 4
affected. 11 10 11 10
In the 3rd draw, the total is affected because of the 1st = 3 + 2
and 2nddrawn balls that are not replaced, though the 11 11
number of black balls is not affected. = 5
11 ( D )
b. A science class consisting of 2 boys and 8 girls all
participated in an end of year quiz. Find the probability 1993/29(Nov)
of choosing both slot A and B by girls from the group to The probability that a man and his wife will be alive in
represent the college at the local government level. twenty years time are 0.6 and 0.7 respectively. What is the
Solution probability that one of them will not be alive in twenty
The word “both” reminds us of multiplication rule. years time
Let G1 – Girls choose the first slot A. 0.88 B. 0.46 C. 0.42 D. 0.28 E. 0.1
Let G2 – Girls choose the 2nd slot Solution
Then Pr(m) means man alive and Pr( m ) means man not alive
Pr (G1 and G2) = Pr(G1) x Pr(G2/G1) Pr(w) means wife alive and Pr( w ) means wife not alive
= 8 X 7 = 0.62 Pr (one of them not alive) = Pr(m) Pr( w ) or Pr( m ) Pr(w)
10 9 = (0.6)(1 – 0.7) + (1 – 06)(0.7)
Once a girl has chosen the first slot, there are now = (0.6)(0.3) + (1 – 0.6)(0.7)
7 girls left and the total number of the group reduces to 19 . = 0.18 + 0.28
= 0.46(B)
432
1992/26 (Nov) Alternatively
The probabilities that Bala and Uzoamaka will pass in Pr(not total of 7 or 11) = 36 – (Total of 7 + total of 11)
an examination are given as 0.8 and 0.9 respectively. 36
Find the probability that at least one of them will pass = 36 – (6 + 2)
this examination. 36
A. 0.002 B. 0.02 C. 0.10 D. 0.72 E. 0.98 = 28 i.e. 7
Solution 36 9 (E)
Bala pass = 0.8 Then B fail = 1– 0.8 i.e 0.2 1995/22 (Nov)
Uzoamaka pass = 0.9 Then B fail = 1– 0.9 i.e 0.1 A fair coin is tossed 4 times. What is the probability that
Condition of at least one of them pass for two persons is each toss results in a head?
one person pass, the other fail + reverse + two them pass A. 1 B. 1 C.1 D. 1 E. 1
Pr (at least one pass ) = 2 4 8 16 64
Pr (B pass) . Pr(Ufail) + Pr(B fail) . Pr(U pass) Solution
+ Pr(B pass) . Pr (U pass) 1st toss 2nd toss 3rd toss 4th toss
Pr (each toss result in head) = 1 × 1 × 1 × 1
= [(0.8)(0.1) + (0.2)(0.9)] + (0.8)(0.9)
2 2 2 2
= 0.08 + 0.18 + 0.72
= 1
= 0.98 (E)
16
Alternatively
1992/6 (Nov)
Since we have either a head or a tail
Two students are chosen at random from a class of 4 0
22 boys and 18 girls. Find the probability that the two 1 1
Pr (each toss result in head) =    
students selected are of the same sex 2 2
Solution 4
Pr (two same sex) = Boy.Boy or Girl .Girl 1 1
=  1 
Total students is 22 + 18 i.e 40 2 16
= 22 × 22 + 18 × 18 1995/29 (Nov)
40 40 40 40 A fair coin is tossed three times. What is the probability that
= 11 × 11 + 9 × 9 it is all tails or heads ?
20 20 20 20 A. 1 B. 1 C. 1 D. 1 E. 1
= 121 + 81 8 4 2 16
400 400 Solution
= 202 Here the sample space consists of 23 = 8
400 and all tails or all heads can only occur once i.e
= 101 Pr (all tails or heads) = 1 + 1
200 8 8
1994/24 (Nov) = 2 i.e 1 (B)
A fair die with faces numbered 1, 2,3 ,4,5, 6 is rolled 8 4
twice. Find the probability of not getting a total of 7 or 11 Alternatively
A. 2 B. 1 C. 8 D. 107 E. 7 Elements of sample space is 23 = 8
9 3 9 108 9 Listing the sample space:
Solution {HHH, HHT, HTH, THH, TTH, THT, HTT, TTT}
We prepare the sample space as shown below: Pr (all tail or heads) = 1 + 1
The sample space is as shown below : 8 8
+ 1 2 3 4 5 6 = 2 i.e 1
1 2 3 4 5 6 7 8 4 (B)
2 3 4 5 6 7 8 1996/24
3 4 5 6 7 8 9 The probability of a foot baller being selected for a club
4 5 6 7 8 9 10 team is 0.1, for a National team 0.05, and for both teams
5 6 7 8 9 10 11 is 0.25. What is the probability of his being selected for
6 7 8 9 10 11 12 at least one of the teams?
A. 0.005 B. 0.125 C. 0.150 D.0.175 E. 0.375
Total of 7 is 6 and total of 11 is 2 Solution
Pr (not total of 7 or 11) = 1 – Pr (total of 7 or 11)
Club Both National
= 1 – 6 + 2
36 36
=1– 8 From the basic rules of set: Pr(A+B) = Pr(A) + Pr(B) – Pr(A∩B)
36 Pr (at least one of them) =Pr(club) + Pr(National) – Pr(both)
= 28 i.e 7 = 0.1 + 0.05 – 0.025
36 9 (E) = 0.125 (B)

433
1996/28 4 3 = 2
A survey was carried out on 100 families each with 4+k 3+k 5
6 children. The table below shows the distribution of Cross multiply
the number of boys per family. 4 × 3 × 5 = 2(4 + k )(3 + k)
No of Boys 6 5 4 3 2 1 0 30 = (4 + k )(3 + k)
Frequency 2 3 16 30 29 15 5 30 = 12 + 7k + k2
What is the probability that a family randomly selected i.e k2 + 7k – 18 = 0
will have at least 4 girls? Factorising k2 – 2k + 9k – 18 = 0
A. 0.39 B. 0.48 C. 0.49 D. 0.51 E. 0.63 (k + 9) (k – 2) = 0
Solution k = – 9 or 2
The examiner here is smart and kind, the actual table But k must be positive, thus k = 2
based on 6 children per family is Checking
No Boys 6 5 4 3 2 1 0 4 3 = 4 3 = 2 as earlier given
No Girl 0 1 2 3 4 5 6 4+k 3+k 6 5 5
Frequency 2 3 16 30 29 15 5 1997/26
Pr (at least 4 girls) = f(E) freq. for 4 , 5 and 6 girls A bag contains 4 green and 5 white identical marbles. If two
∑f marbles are picked at random one after the other without
= 29 + 15 + 5 replacement, what is the probability of having the first green
100 and the second white?
= 49 i.e 0.49 (C) A. 2 B. 20 C. 5 D. 5 E. 20
100 9 81 81 9 27
Solution
1996/30 Nov
Pr(GW) = 4 × 5 = 5 ( C )
In a class of 100 students, 50 are studying Geography,
9 8 18
40 are studying Mathematics while 20 are studying Without replacement, so the total number of balls is affected after the first draw
both. Find the probability that a student chosen at
random is studying neither Geography nor mathematics. 2000/26 Neco
A. 0.1 B. 0.3 C. 0.5 D. 0.8 E. 0.9 A bag contains 3 red and 4green balls. Two balls are picked
Solution at random from the bag without replacement. What is the
We are to find those students not taking any of the subjects probability that they are of different colours?
Applying two set problem diagram A. 3/7 B. 4/7 C. 12/49 D. 24/49 E. 25/49
U100
Solution
G50 M40
Pr(Diff colour) = Pr(RG) or Pr (GR)
50-20 20 40-20 Note “without replacement”
= 3 × 4 + 4 × 3
x 7 6 7 6
30 + 20 + 20 + x = 100 = 2 + 2
x = 100 – 70 7 7
= 30 = 4
Pr(Student studying neither Geo nor maths) = 30 7 (B)
100 2005/25
= 0.3 (B) Pair of fair dice, with each face number 1 to 6, are
1997/6 thrown once. Find the probability of scoring a 2 on one die
A bag contains k blue and 4 black identical balls, where and a 5 on the other
k is constant. Two balls are drawn at random from the A 1 B. 1 C. 1 D. 2
bag, one after the other, without replacement. If the 36 18 3 3
probability of drawing 2 black balls is 2/5 find the value Solution
of k ? A table showing the pairing of numbers on the two dice is:
Solution
Your total alertness in applying rules here is needed 1 2 3 4 5 6
note the word “without replacement” and “2 black balls 1 1,1 1,2 1,3 1,4 1,5 1,6
probability is 2/5” 2 2,1 2,2 2,3 2,4 2,5 2,6
Pr (two black ball drawn without replacement) is 3 3,1 3,2 3,3 3,4 3,5 3,6
4 4,1 4,2 4,3 4,4 4,5 4,6
 black balls   Black balls  1  2 5 5,1 5,2 5,3 5,4 5,5 5,6
Pr    Pr   
 total balls   Total balls  1  5 6 6,1 6,2 6,3 6,4 6,5 6,6
The simple equation is
Pr(Scoring a 2 on one die and a 5 on the other)
4 4–1 = 2 = 2 i.e 1 (B)
4 + k (4 + k ) – 1 5 36 18
434
2000/13 2000/13 (Nov)
Ojo and Alex played 6 games of Ludo. Ojo won 3, Alex A fair die with six faces numbered 1, 2 , 3 ,4 ,5 and 6 is
won 2 while1 ended in a draw. If the two of them tossed twice
decide to play another set consisting of 3 games, find (a) Obtain all the possible outcomes of the magnitude of the
the probability that: difference between the numbers showing
(a) Ojo wins the 3 games (b) What is the most likely difference?
(b) Two games end in a draw (c) What is the probability of difference of zero of one?
(c) Each player wins alternatively (d) Find the mean difference
(d) Alex wins at least 1 game (e) Find the median difference
Solution Solution
Our solution will be based on the previous records ‘Magnitude’ of difference here means only positive value of
(Empirical probability) which is given as: the difference between the two numbers as shown below:
Initial total games played is 6 1 2 3 4 5 6
Pr(Ojo wins) = 3 i.e 1 Pr(Alex wins) = 2 i.e 1 1 0 1 2 3 4 5
6 2 6 3 2 1 0 1 2 3 4
Pr(draw) = 1 3 2 1 0 1 2 3
6 4 3 2 1 0 1 2
“Problem” games played 3 5 4 3 2 1 0 1
(a ) Pr(Ojo wins 3 games) 6 5 4 3 2 1 0
= Pr(Ojo win 1st ) Pr(Ojo win 2nd ) Pr(Ojo win 3rd)
= 3 3 3 (b) Most likely difference i.e mode is 1 with the highest
6 6 6 frequency of 10.
= 1 (c) Pr(zero or one) = Pr(zero) + Pr(one) since they are M.E
8 = 6 + 10
(b) Apply your sense of permutation 1st , 2nd & 3rd 36 36
Pr(two games end in a draw) = = 16 i.e 4
Pr( 1st and 2nd ) or Pr(1st and 3rd ) or Pr (2nd and 3rd) 36 9
Deduce that no draw here will be 1– 1/6 i.e 5/6 (d) Adding along the columns from the first to the sixth
= Pr (1st and 2nd then 3rd No draw) or Pr(1st and 3rd then 2nd No draw) Mean = 15 +11 + 9 + 9 +11 +15
or Pr (2nd and 3rd then 1st no draw)
36
= 1 1 5 + 1 5 1 + 5 1 1 = 70
6 6 6 6 6 6 6 6 6 30 i.e 1.94
= 5 + 5 + 5 (e) Medium difference
216 216 216 Rearranging the values
= 15 Number frequency
216 0 6
(c) Pr(each player wins alternatively ) 1 10
= Pr(Ojo wins then Alex wins then Ojo wins or 2 8
Alex wins then Ojo wins then Alex wins) 3 6
= 3 2 3 + 2 3 2 4 4
6 6 6 6 6 6 5 2
= 1 + 1 ∑f = 36
12 18 Median = ∑f since sum of frequency is even
=3 +2 2
36 = /2 = 18th class
36

= 5 Adding frequency 6 +10 + 8 i.e 3rd row from up is where the


36 18th class lies
Thus median = 2
(d) Pr(Alex wins at least one game )
= 1 – Pr(Alex wins no game out of the three) 2002/21 Neco
A fair coin is tossed 3 times. What is the probability of
 1  1  1  having a least 2 heads?
= 1  1   1   1  
 3  3  3  A. 2 B. 3 C. 1 D. 1 E. 1
3 8 2 3 8
 2   2   2  Solution
= 1        The sample space is 23 = 8
 3   3   3 
HHH, HHT, HTH, THH
8
=1  TTT, TTH, THT, HTT
27 Pr (at least 2 heads) = 4 i.e 2 heads, 3 heads
19 8
=
27 = 1/2

435
2000/14b (Nov) the class, what is the probability that he offers both subjects?
A box contains 8 good and 2 bad oranges. Three of the A. 3 B. 9 C. 11 D. 7
oranges are selected at random, one after the other with 10 20 20 10
replacement. What is the probability that none is bad? Solution
Solution Using Venn diagram, we let x represents both subjects
With replacement implies independent event. U20
Let the oranges be A, B, C P C
Pr (None is bad) = Pr(All is good) 9 x 5
= Pr(A good) Pr(B good) Pr (c good)
8 8 8
=       9 + x +5 = 20
 10   10   10 
3
x = 20 – 14
4 64 = 6
=   i.e
5 125 Pr(both subjects) = 6 i.e 3
20 10 (A)
2000/24 (Nov)
Two boxes contain 8 cards each , numbered 2002/25
1,2,3,4,5,6,7,8. If two cards are drawn at random, one One bag contains 2 yellow and 3 red balls, another
from each box, find the probability that the product of 3 yellow and 4 red balls. One ball is drawn from each bag.
the numbers on the two cards is 12. What is the probability that both balls are yellow?
A. 15 B. 1 C. 1 D. 1 A. 2 B. 3 C. 1 D. 6 E. 12
16 2 4 16 35 35 7 35 35
Solution Solution
The table of the events is: Let the two bags be A and B; then
× 1 2 3 4 5 6 7 8 Bag A Bag B
1 1 2 3 4 5 6 7 8 2 yellow & 3 red 3 yellow and 4 red
2 2 4 6 8 10 12 14 16 Probability of both reminds us of multiplication rule .
3 3 6 9 12 15 18 21 24 Next, we check whether they independent or dependent
4 4 8 12 16 20 24 28 32 In this case they are independent. Thus ,
Pr (Both balls yellow) = Pr (yellow in Bag A) × Pr (yellow in bag B)
5 5 10 15 20 25 30 35 40 = 2 × 3
6 6 12 18 24 30 36 42 48 5 7
7 7 14 21 28 35 42 49 56 = 6
8 8 16 24 32 40 48 56 64 36 (D)
2002/11(Nov)
Pr(product of two numbers is 12) = 4 A pair of fair dice with faces numbered 1, 2, 3, 4, 5, and 6 is
64 rolled once. What is the probability of obtaining at least a
= 1 sum of six?
16 (D) A. 1 B. 11 C. 1 D. 13
2000/25 (Nov) 36 36 3 18
The table shows the distribution of marks obtained by a Solution
group of pupils in a test. The sample space is listed as:
Marks 1-9 10-18 19-27 28-36 37- 45 + 1 2 3 4 5 6
Boys 3 10 9 5 5 1 2 3 4 5 6 7
Girls 1 5 10 2 0 2 3 4 5 6 7 8
25. If a pupil is randomly selected from the class, what 3 4 5 6 7 8 9
is the probability that the pupils scored at least 28 marks? 4 5 6 7 8 9 10
A. 0.14 B. 0.24 C. 0.76 D. 0.90 5 6 7 8 9 10 11
Solution 6 7 8 9 10 11 12
Pr(At least 28 marks) = ∑f (28 – 36) + ∑f (37 – 45)
Total frequency at least a sum of six is the sum 6 , 7, 8, 9, 10, 11, and 12
= (5 + 2) + ( 5 + 0) Pr (at least sum 6 ) = 26 i.e 13
4 + 15 + 19 + 7 + 5 36 18 (D)
= 12
50 2003/14 (Nov)
= 0.24 (B) A fair dice with faces numbered 1, 2, 3, 4, 5, and 6 is thrown
2000/30 (Nov) twice. What is the probability of obtaining a prime from each
Each of the 20 boys in a class offer either physics or throw?
chemistry or both. Nine boys offer physics only and A. 1 B. 1 C. 1 D. 4
five chemistry only. If a boy is selected at random from 12 9 4 9

436
Solution Solution
Here we will list the raw numbers; recalling the We list the two sample space separately as joining them
definition of prime numbers property will be of help. seems to be an impossible task.
1 2 3 4 5 6 + 1 2 3 4 5 6
1 1, 1 1, 2 1, 3 1, 4 1, 5 1, 6 1 2 3 4 5 6 7
2 2, 1 2, 2 2, 3 2, 4 2, 5 2, 6 2 3 4 5 6 7 8
3 3, 1 3, 2 3, 3 3, 4 3, 5 3, 6 3 4 5 6 7 8 9
4 4, 1 4, 2 4, 3 4, 4 4, 5 4, 6 4 5 6 7 8 9 10
5 5, 1 5, 2 5, 3 5, 4 5, 5 5, 6 5 6 7 8 9 10 11
6 6, 1 6, 2 6, 3 6, 4 6, 5 6, 6 6 7 8 9 10 11 12
Pr(a Prime from each throw) = 9 i.e 1 Pr (sum>7) = 15
36 4 (C) 36
2003/32
The probability that a man smokes cigarette is 0.5 and Two fair coins sample space = {HH,TH, HT TT}
that he drinks alcohol is 0.7. If the probability that he Pr(one head) = 2
smokes cigarette or drinks alcohol or both is 0.8 . 4
Calculate the probability that he smokes cigarette and Therefore Pr (both) = 15 × 2 = 5
drinks alcohol. 36 4 24
A. 0.15 B. 0.12 C. 0.3 D. 0.4 2005/14 b (Nov)
Solution A bag contains 6 red, 5 blue and 4 green identical balls.
Here we have something in common i.e smoking and Three balls are drawn from the bag at random without
drinking and it is probability to find replacement. Calculate correct to 3 decimal places, the
“the probability that he smokes cigarette and drinks Probability that
alcohol or both is 0.8” implies (i) The balls are of the same colour,
Pr(ciga) + Pr(alcohol) – Pr( Both) = 0.8 (ii) a ball from each of the three colours is drawn.
0.5 + 0.7 – Pr(Both) = 0.8 Solution
1.2 – 0.8 = Pr(Both) Take note of “without replacement”
0.4 = Pr(Both) (D) (i) Pr(same colour) = Pr(RRR) or Pr(BBB) or Pr (GGG)
= 6 × 5 × 4 + 5 × 4 × 3 + 4 × 3 × 2
15 14 13 15 14 13 15 14 13
2005/14(Nov) In this case, the particular coloured ball reduces by one after
In a group of 50 students, 24 study Government and each draw, same goes to the total balls in the bag
33 Economics. If 5 of them do not study either of the = 4 + 2 + 4
subjects. What is the probability that a student chosen at 91 91 455
random from the group study Economics only? Changing to decimal numbers noting that we will correct
Solution to 3 d.p hence we give that allowance
Applying venn diagram = 0.04396 + 0.02198 + 0.008791
U50 = 0.074731
G 24 E 33 = 0.075 to 3dp
24-x x 33-x (ii) Pr(a ball each from three colours)
Applying permutation to generate the sample space 3P3 = 6
5 = RBG, RGB, BRG, BGR, GRB, GBR.
= Pr(RBG) or Pr(RGB) or Pr(BRG) or Pr(BGR) or Pr(GRB) orPr(GBR)
24 – x + x + 33 – x + 5 = 50
= 6 × 5 × 4 + 6 × 4 × 5 +
62 – x = 50
15 14 13 15 14 13
62 – 50 = x
What do you observe? Numerator is same & denominator is same
12 = x
6 5 4
 33  x  = 6     
Pr(Economics only) = Pr   15 14 13 
 50 
= 24
= 33 – 12
91
50
= 0.2637  0.264 to 3d.p
= 21
50
2003 /12 Neco
A bag contains 2 ten kobo coins and 3 five kobo coins. If
2005/5 (Nov)
they are drawn out one by one without replacement. Find the
Two fair dice with faces numbered 1– 6, and two fair
probability that the first is a five kobo coin; the second is ten
coins are thrown and tossed respectively, at the same
kobo coin and so on alternatively
time. Find the probability that the sum of the numbers
A 0.20 B. 0.10 C. 0.31 D. 0.06 E. 0.90
on the faces of the dice that show up is greater than
seven and only one coin shows a head
437
In this case we are to keep drawing out till the whole 2006/13a
coins are finished. T  Ten Kobo and F  Five Kobo A man P has 5 red , 3 blue and 2 white buses. Another man
2 3 Q has 3 red, 2 blue and 4 white buses. A bus owned by P is
Note the Phrase without replacement involved in an accident with a bus belonging to Q. Calculate
Pr(Drawing out) = 3 × 2 × 2 × 1 × 1 the probability that the two buses are not of the same colour.
5 4 3 2 1 Solution
F1st T2nd F3rd T4th F5th Pr (not same colour) = 1 – Pr(same colour)
= 1 R B W
10 P Buses 5 3 2
= 0.1 (B) Q Buses 3 2 4
2003/34 Neco Two buses are involved in an accident one from each side.
A firm is independently working on two separate Jobs. Pr(same colour ) = Pr(RR) or Pr(BB) or Pr(WW)
There is a probability of only 0.4 that either of the Jobs = 5 × 3 + 3 × 2 + 2 × 4
will be finished on time. The probability that at least 10 9 10 9 10 9
one of the jobs finished on time is = 1 + 1 + 4
A. 0.42 B. 0.49 C. 051 D. 0.64 E. 0.81 6 15 45
Solution = 15 + 6 + 8 i.e 29
Let the two jobs be A and B 90 90
Pr(Least one job is finished) Thus Pr(not same colour) = 1 – 29 = 61/90
= A + B – AB 90
= 0.4 + 0.4 – (0.4)(0.4) 2002/28 Exercise 36.1
= 0.8 + 0.16 The probabilities of two independent events occurring are P1
= 0.64 (D) and P2. Find the probability that at least one of them will occur.
A.P1 P2 – P1 – P2 B. P1 – P + P1 P2
2003/7 (Dec) Neco C.P1 P2 + P1 + P2 D. P1 + P2 – P1 P2
In a quantity of transistor radios, 60% are digital and the
remaining are mechanical type. Half of the digital and 2002/28 Exercise 36.1
30% of the mechanical are found to be defective. If a The probability of three athletes P, Q, R winning a race is
radio is selected at random , find the probability that itis a : 0.2, 0.5 and 0.3 respectively. What is the probability that the
(i) Digital type three athletes will tie for the first position in a race?
(ii) Non- defective type A. 0.02 B. 0.05 C. 0.03 D. 0.10
Solution
Digital Radio = 60 i.e. 3 and defective Digital = 1 of 3 = 3 1994/22 Exercise 36.3
100 5 2 5 10 The possible result of an experiment is called
Mechanical Radio = 100 – 60 i.e. 2 and A. a random experiment B. a sample space
100 5 C. an event D. The universe set
Defective mechanical = 30 of 2 = 3 E. an outcome
100 5 25
(i) Pr(Digital type) = 3 2004/8 Neco Exercise 36.4
5 There are 4 red , 3blue and 5 green identical ball in a bag.
(ii) Pr (Non-Defective type) = 1 – Pr (Defective type) If three balls are picked at random without replacement,
= 1 – [Pr (D) + Pr (M)] what is the probability that they are of different colours?
=1– 3 + 3
A. 1/22 B. 1/11 C. 3/22 D.1/3 E1/2
10 25
= 1 – 15 + 6
2002/26 Neco Exercise 36.5
50
A die is designed such that it has 7 faces marked 1 to 7
= 1– 21 i.e. 29 A score on each throw is the number on the face which
50 50 rests on the probability of a sum of 9 in two throws?
2004/49 Neco Dec A. 6 B. 2 C. 1 D. 2 E. 7
A die and two coins are thrown together at once. What 49 9 4 7 9
is the probability of getting a six as well as a tail and a
head? 2003/31 (Nov) Exercise 36.6
A. ¾ B. ½ C. ¼ D. 1/6 E. 1/12 The probability of obtaining oranges in local markets is ¼
Solution
and that of mangoes is 2/5 .Find the probability that a student
A die sample space = {1, 2, 3, 3, 5, 6}
will obtain oranges and not mangoes in the market.
Pr[getting a six] = 1/6 A. 3 B. 1 C. 3 D. 1
Two coins sample space {HH, HT, TH, TT) 20 10 10 5
Pr(getting tail & a head) = 2
4
1
Thus Pr (Both) = 1 × 2 = /12
6 4
438
logic Connectives
The basic problem of logic is the analysis of the The words and, or, not,… then, and if and only if are the
methods of reasoning. Reasoning is that special kind of basic words used to connect proposition. They are called
thinking called inference, in which conclusions are logical connectives
drawn from premise. As an example consider:
Use of Symbols
All dogs have four legs. The power of symbolic Logic lies in its use of
Bingo is a dog symbols to represent proposition and connectives. We shall
Therefore, Bingo has four legs. represent propositions by the lower case letters p, q, r,…
The premises of this argument are all dogs have four usually, each letter will stand for a simple propositions, and
legs and bingo is a dog. The conclusion is Bingo has compound will be formed by linking two or more letters
four legs. together by means of one or more of logical connective and
The logician seeks the answer to this question. ”If the also give the name by which the compound is known
premises are true, is one justified in saying that the Connectives Symbol Names of compound
conclusion is true?”. If so, the reasoning that led to the Not ~ Negation
conclusion is valid (logical); if not, the reasoning is And  Conjunction
invalid (illogical). The systematic formalization and Or  Disjuntion
cataloging of valid methods of reasoning is one of the If,…. then  Conditional
major task of a logician. If his work uses abstract If and only if  Biconditional
symbols, then it is called symbolic logic.

PROPOSTIONS 1. Example LOG3


To avoid ambiguities, certain technical terms will now Given that:
be defined before using them. p represent “The baby is crying”
Proposition q represents “ The boys are singing” and
A proposition is a declarative sentence, which is true or r represents” “The dog is barking”
false but not both Represent the following propositions symbolically.
(A) The dog is not barking
Truth Value: (B) The baby is crying and the birds are singing
The truth value of a proposition is true if the proposition (C) The dog is barking or the birds are singing
is true and false if the proposition is false. (D) The dog is barking and the birds are not singing
Examples LOG 1 (E) If the dog is barking and the birds are not
These sentences are propositions: singing, then the baby is crying.
(A) Abuja is a town in Nigeria. (F) The baby cries if and only if the dog barks.
(B) France is one of the united states Solution
(C) 3 + 2 = 5 (a) ~ r
(D) George is tall. (b) p  q
Assertion (a) is true since it corresponds to reality. (c) r  q
Assertion(b) is false since it does not correspond to reality (d) r  ~ q
Assertion (c) is true.
(e) (r  ~ G)  p
Some logicians do not consider a sentence such as(d) to
(f) p  r
be a proposition because there are many individuals
named George and because tall is relative term.
Here we shall consider that words such as George, I and
Example LOG4
they refer to specific persons and that arrangement on
Let s and t stand for:
standards for words such as tall, short, and old has been
s : sally is smart.
reached. If these assumptions are made, (d) is a
t : tom is tall
proposition whose truth value depends upon the
What is sentence that has the same meaning as:
particular George referred to and upon the standard of
(a) ~ s
tallness agreed.
KINDS OF PROPOSITION (b) s  t
A Proposition is simple if it does not contain any (c) (t  s)  t
other proposition as a component part. It is a compound Solution
proposition if it is compose of two or more proposition. (a) Sally is not smart
(b) Sally is smart and Tom is tall
Examples LOG2
These are compound proposition: (c) If Tom is Tall and Sally is smart then Tom is Tall
(a) 1 + 1 = 2, and 4 is less than 9.
(b) Owls are nocturnal and cats are playful.
(c) If it rain, then I will not attend the game.

439
Examples LOG5 truth values required will be 22 = 4. Our convention for
If p, q, r, stand for arranging truth values in the first column of the truth tables
p : Birds fly is TTFF, and for those in second column is TFTF. The
q : The sky is blue complete truth table for conjunction is:
r : The grass is green p q pp
1.Write the sentence that has the same meaning as: T T T
(a) ~p (f) p  ~ q ~r T F F
(b) pq (g) p  ( q) F T F
(c) pq (h) (p  q)  r F F F
(d) pq (i) p↔q Disjunction The proposition pq is defined to be false
(e) pqr if p and q are both false and true in all other cases.
2.Using the proposition p, q and r of the above problem, p q pp
write symbolic representations of: T T T
(a) The sky is blue and the grass is green. T F T
(b) Birds fly or the sky is blue. F T T
(c) Birds do not fly and sky is not blue F F F
(d) If the grass is green and the sky is not blue Conditional
then the birds do not fly. The proposition p  q is defined to be false
Solution if p is true and q is false, and true in all other cases
1. (a) Birds do not fly p q pp
(b) Birds fly and the sky is blue T T T
(c) Birds fly or the sky blue T F F
(d) Birds do not fly or the sky is blue F T T
(e) Birds fly and the sky is blue and F F T
grass is green Biconditional
(f)Birds fly and the is not blue and the The proposition p  q is defined to be true when
grass is green p and q have the same truth values and false if they have
(g)Birds do not fly or the grass is not green different values.
(h) If birds fly and the sky is blue then p q pp
grass is green. T T T
(i) Birds fly if and only if the sky is blue T F F
2. (a) q  r F T F
(b) p q F F T
(c)  p   q Constructing truth tables:
(d) ( r   q)   p The tables that have been shown so far can be used
in constructing truth tables for more complex propositions.
Truth values and truth tables.
The following examples will illustrate how this is done
In the proceeding section the five basic logical
Example LOG 6
connectives were introduce. In this section, we consider
the form of compounds and the rules for assigning truth Construct the truth table for p~q
values to each of them. Solution
We start the table with the standard four rows and two
Negation columns for compound involving the two propositions p and q. the
The negation p is defined to be true if p is proposition in question p ~q is the conjunction of p and ~q.
false and false and false if p true. It is customary to us p q ~q p~q
symbol T to denote the truth-value of a true proposition T T F F
and F to donate the truth value of a proposition that is T F T T
F T F F
false. Using these symbols, the relation between the
F F T F
truth values of p and p are shown in the following After obtaining the truth values under ~q, we then use connective
truth table. (and) to find the truth values under p~q
p ~p ExampleLOG7
T F Construct the truth for ~ (p q)
F T p q p q ~ (p q)
Conjunction T T T F
The word “and” is defined to have the same T F T F
meaning in logic as in common usage. The proposition F T T F
F F F T
pq is true if p and q are both true and false in
The values for the proposition p q are obtained and then negated to
other cases. To know how many truth value are obtain the values for ~ (p q)
required. We use the formula 2n, where n denotes the
number of propositions in value. In the case of p, q two
propositions are involved and hence the numbers of
440
ExampleLOG8 Solution
Construct the truth for the following statement forms: p q ~p ~pq pq (~pq) (pq)
(a) p  ~q T T F T T T
(b) p (qq) T F F F F T
(c) (p  q)  r. F T T T T T
Solutions F F T T T T
(a) p q ~q p ~q From the table, (~pq)  (pq) is a tautology .
T T F F Thus ~pq and p q are equivalent.
T F T T
F T F T
F F T T

(b) p q qp p (qp)


T T T T
T F T T
F T F T
F F T F

(C) In this case, we have three proposition p,q and


r so we shall have 23 = 8 truth values and write
them out as follows:

p q r pq (pq)r
T T T T T
T T F T F
T T F T T
T F T F T
F T T F T
F T F F T
F F T F T
F F F F T
Logically True propositions
A propositions is logically true if it is true for
every possible arrangement of truth values of its
component propositions, and it is false for every
possible arrangement of truth values of its parts.
Logically true proposition are often called
tautologies.

Equivalence
Two proposition p and q are equivalent if the
bi-conditional pq is logically true i.e pq is
logically false, p and q are contradictories.

Examples LOG9
(a) pv~ p is a tautology
p ~p p v ~ p
T F T
F T T

(b) p~p is a contradiction


p ~p p~p
p F T
T F F
F T F

(C) Show that ~pq is equivalent to p q

441
Chapter Thirty seven
( ii ) Corresponding sides are in constant ratio
Plane Geometry A P
Angles between straight and parallel lines, types of
triangles have been discussed under ‘elements of plane
Geometry’ (Chapter 15 ). Readers to take note.

Triangles
B CQ R
Sum of angles in a triangle is 1800 (2 right angles)
AB = BC = AC
A PQ QR PR
a2 P (iii) Angle of one triangle is equal to one angle of the other
and the corresponding sides containing these equal angles
a1 are in a constant ratio . From the diagram
b2 c b1
B X above in (ii) Â = P and AB = PQ
C BC QR
Given: any ∆ ABC
To prove: Â + B + C = 1800 Similar triangles diagrams
Construction: Produce BC to point X. Draw CP//BA A B
Proof:
a1 = a2 ( alternate angles)
b1 = b2 ( corresponding angles) E

c + a1 + b1 = 1800 ( angles on straight line)


 c + a2 + b2 = 1800 C D
 c + a2 + b2 = 1800
ACB + A + B = 1800 AB = BE = AE
 A + B + C = 1800 CD CE DE
The exterior angle of a triangle is equal to the sum of P
the two interior opposite angles.
S T
a

Q R
b c PS = PT = ST
PR PQ QR
C=a+b M

Congruent triangles
When we say that a set of two triangles is congruent
Q H
then they are equal in all respects:
Two triangles are congruent if they have:
(i) Three equal sides (SSS)
N O
(ii) Two equal sides and an equal included angle
(SAS) MQ = MH = QH
(iii) Two equal angles and an equal side (AAS) MN MO NO
(iv) Two right - angled triangle (RHS)
Pythagoras’ theorem
SIMILAR TRIANGLES It states that: “in a right – angled triangle, the square of the
hypotenuse is equal to the sum of the square of the other two sides.
Two triangles are similar if they are/ their:
A
(i) Equi-angular
A P

B C
B CQ R i.e /AB/2 = /BC/2 + /AC/2
442
Areas of similar triangles 1991/27 PCE
The ratio of the areas of similar triangles is equal to the P

ratio of the squares on their corresponding sides. m


Given two similar triangles ABC and ADE 3c
A
Y
A

m
9c

B C

A X
B C 10cm Z Q

In the diagram above, XY = 9cm ,YP = 3cm, XZ = 10cm,


and XPO = YZX. Calculate ZQ
D E
A 0.30cm B 0.80cm C 1.25cm D 3.33cm
Solution
D E
By Similar triangles ratios.
XY = XZ
Area of ABC = AC2 or BC2 or AB2
XQ XP
Area of ADE AE2 DE2 AD2 i.e. XY = XZ
ZX + ZQ XY + YP
1990/41 PCE 9 = 10
In an equilateral triangle of height 7cm, find the length 10 + ZQ 9+3
of each side.
A 14 3cm B 7 3cm C 7 3cm D 14 3cm 9 = 10
3 3 10 + ZQ 12
Solution 12 × 9 = 10(10 + ZQ)
Equilateral triangle has all its sides equal. Let each side be 2x 108 = 100 + 10ZQ
108 – 100 = 10ZQ
8 = 10ZQ
8 = ZQ i.e. 0.80cm (B)
2x
Height

2x 2x
2x
7cm
10
1988/35 UME
Q

5
x x 6
2x
S

We find x By Pythagoras rule. T


(2x)2 = x2 + 72 6

4x2 = x2 + 49
4x2 – x2 = 49
P R

3x2 = 49 In the figure above, STQ = SRP, PT = TQ = 6cm and


x2 = 49 QS = 5cm. Find SR
47
3 A /5 B5 C 32/5 D 22/5
By Similar triangles ratios.
x= 49 i.e. x = 7 TQ = QS
3 3 QS + SR TQ + PT
= 7 6 = 5
3 5 + SR 6+6
By rationalization of surd
= 7 3cm Then 2x = 14 3 cm (A) 6 = 5
3 3 5 + SR 12
6 ×12 = 5(5 + SR)
72 = 25 + 5SR
72 – 25 = 5SR
47 = 5SR
47 = SR (A)
5
443
1988/37 UME 1989/34 UME
S T
0
50

S
y

P
Q

R
In the figure above PS = RS = QS and 0
24 R
Q SR = 500. Find QPR Q
P
A 250 B. 400 C. 500 D. 560 If PST is a straight line and PQ = QS = SR in the above
Solution diagram, find y
SQR = SRQ = SRP = RPS (Base angles of isosceles s) A 240 B. 480 C. 720 D. 840
SQR = 180 – 50 Solution
2 PSQ is isosceles; Thus
= 130 i.e. 650 PSQ = 240 and  PQS = 180 – (24 + 240) i.e. 1320
2
Resulting diagram Also  SQR is isosceles; Thus From line PQR,
S  SQR = 180 – 132 (angles on straight line )
50
0 i.e.  SQR = 480
QRS = 480 (Base angles of isosceles )
 QSR = 180 – (48 + 48) i.e. 840
Therefore ; y +  QSR +  PSQ = 1800 (straight line s)
65 0
65 0 y + 84 + 24 = 1800
P
Q y = 180 – 108
y = 720 (C)
65 0
65 0 1990/34 UME
X
R
QR = RP (Base sides of same isosceles s)
In ∆ QPR 4cm
5cm
RQP = QPR
 QPR = 180 – (65 + 65)
P
2
Q
=180 – 130 = 50 i.e 250 (A) 3cm

2 2 3cm

Y
1989/32 UME
P
Z

In triangle XYZ and XQP, XP = 4cm, XQ = 5cm


4cm

K
and PQ = QY = 3cm. Find ZY
H
A. 8cm B. 6cm C.4cm D. 3cm
3cm

Solution
Q R From similar triangle ratios.
PX = XY
In the diagram above HK is parallel to QR, PH = 4cm PQ ZY
and HQ = 3 cm. What is the ratio of KR : PR? 4 = 5+3
A7:3 B. 3 : 7 C. 3 : 4 D. 4 : 3 3 ZY
Solution
From Similar triangles: 4 = 8
PH = PK = 4 3 ZY
PQ PR 7 ZY = 3 × 8
Since the two triangles are similar, KR = 3cm also. ZY = 3 × 8
Thus, 4
KR : PR ZY = 6cm (B)
3 : 7 (B)

444
1993/30 PCE In SQT
Which of the following triangles are similar SQR is a straight line. Thus
4cm
SQT = 180 – 640(straight line angles)
m 4c
m
= 1160

8c
2c
QTS = 180 – 1250 (straight line QTP)

m
6cm
3cm = 550
I II x = 180 – (55 + 1160)(as in  SQT)
x = 90 (A)
7cm
m
8c

1991/38 UME Exercise 37.1


12cm P

III
A. I and II B. I and III C. II and III D. I, II and III
Solution S

I and II are similar triangles because the sides are in


simple ratio 2 : 1 (A)
0
68
Q R T
1994/29 PCE
E In the figure above, PQ = PR = PS and SRT = 680.
F Find QPS
m A 1360 B 1240 C 1120 D 680
9c
1992/24 UME Exercise 37.2
U P Q
O
cm 12 x
15 cm
Q
N
200
In the diagram above, the value of OE is 105
0

A. 8cm B.12cm C.18cm D. 20cm


T S R
Solution
By Similar triangles ratios; In the figure above, TSP = 105 and PRQ = 20 ,
0 0

ON = OE find PQR
OF OQ A. 1300 B. 1200 C. 750 D.300
15 = OE
9 12 1993/32 UME
P
15×12 = 9 × OE
15×12 = OE
9 4cm 9cm
5 × 4 = OE i.e. OE = 20cm (D)

1994/30 PCE Q R
P 3cm S

26
0 In the diagram above, QPS = SPR , PR = 9cm,
125
0 PQ = 4cm and QS = 3cm. Find SR
T
A 6 ¾ cm B 3 3/8cm C 43/8cm D 22/3 cm
Solution
The two s have one angle equal and share a common side.
x Thus, They are similar triangles,
S Q R SR = PR
Find x in the diagram above if triangle PQR is QS PQ
right – angled at R. SR = 9
A. 90 B. 80 C. 70 D. 50 3 4
Solution SR = 9 × 3
For us to know x, Firstly , we find SQP and QTS 4
In  PQR SR = 27 = 6 ¾ cm (A)
Q + P + R = 1800 (sum of s in ) 4
Q + 260 + 900 = 1800
Q = 180 – 116 i.e. Q = 640
445
1993/33 UME 1990/24 PCE Exercise 37.5
The three side of an isosceles triangle are of length L
x + 3, 2x +3, 2x – 3 respectively. Calculate x
A. 0 B. 1 C. 3 D. 6
Solution 3cm
Picture an isosceles triangle with both sides equal, then N
by observation, the sides likely to be equal are x + 3 and M 4cm
2x–3 . Since 2x + 3 is far bigger than any of the others.

x+3
2x-3 S R

In the diagram above,  LMN =  LRS and


2x+3 LNM =  LSR. If LR = 7.5cm, LM = 3cm and MN =
x + 3 = 2x – 3 4cm. Find the length of RS
3 + 3 = 2x – x A 1.6cm B 5.6cm C 8.5cm D 10.0cm
6 = x (D)
2002/30PCE
1996/25 PCE The point where the bisectors of the three sides of a triangle
intersect is the
K A in centre B ortho – centre C Ex – centre
D Circum – centre D an equilateral triangle.
Solution
10)
(2x+

n k
(7x
-10
)

2x x
L M The point K is called in-centre ( A )
In the diagram above, find angle n
1994/32 UME
A 2000 B 1600 C 1000 D 300
Solution
To find n , we work n in terms of x, Then solve for x 6cm
5cm

n + (7x – 10)0 =3600 ( angles at a point)


h
n = 360 – (7x – 10)0
= 360 – 7x +10
n = 370 – 7x
7cm
2x + x + 7x – 10 + 2x +10 = 360 (Sum s of a quadrilateral)
12x = 360 In the diagram above, find h
12x = 360 A 12 cm B 12 6cm C 7cm D 1 51cm
12 12 7 7 12 2
x = 30 Solution
Therefore, n = 370 – 7(30) Let us re-label the diagram as:
= 370 – 210 C
= 1600 (B)
6cm
5cm

1997/26 PCE Exercise 37.4


h
F
0
K
34
x x B
A 7-x D
9cm
Apply Pythagoras rule
In  ACD
47
0 62 = h2 + (7 – x)2
G
H R h2 = 36 – (7 – x)2 ……….(1)
Similarly
From the figure above FK//GR and FH = GH,  RFK = 340 In  BCD
and  FGH = 470 . Calculate the angle marked x. h2 = 25 – x2 …………….(2)
A 420 B 520 C 640 D 720 We equate (1) to(2) Since they are in h2
446
36 – (7 – x)2 = 25 – x2 Polygons
36 – ( 49 – 14x + x2) = 25 – x2 A closed figure bounded by three or more straight sides is
36 – 49 +14x – x2 = 25 – x2 called a polygon. A polygon is regular; if it has all its sides
14x – x2 + x2 = 25 – 36 + 49 and angles equal, otherwise it is irregular.
14x = 38
14x = 38 Sum of interior and exterior angles
14 14 The sum of interior angles of an n – sided polygon is
x = 19 (2n – 4) x 900
7 A
Substitute x value into (2) F B
h2 = 25 – x2 will become
h 2 = 25 – 19 2
E C
7
D
h = 25 – 361
2

49 A + B + C + … = ( 2n – 4) x 900 and
= 1225 – 361
49 each interior angles of n – side = (2n – 4) x 900
h2 = 864 n
49
While the sum of the exterior angles of an n – sided
h = 864
polygon is 4 × 900
49
a
= 144 × 6
49 e

= 12 6 (B) b
7 d
c

1990/22 UME
The lengths of the sides of a right -angled triangle a + b + c + … = 3600 and
are x cm , (3x – 1)cm and (3x + 1)cm. Find x each exterior angle of n sides = 4 × 900
A5 B7 C8 D 12 n
Solution Quadrilaterals
By observation the biggest side is (3x +1)cm. Thus Quadrilaterals are four sided figures. Quadrilaterals whose
By Pythagoras rule diagonals bisect each other at right angle are square, kite
(3x +1)2 = (3x – 1)2 + x2 and rhombus. Below is a list of some quadrilaterals and their
9x + 6x +1 = 9x2 – 6x + 1 + x2
2
properties:
9x2 – 9x2 – x2 + 6x + 6x + 1 – 1 = 0
– x2 +12x = 0 Parallelogram
i.e. 12x – x2 = 0
x(12 – x) = 0
x = 0 or 12 – x = 0
x = 0 or 12
We take x = 12 (D)

1991/30 PCE Exercise 37.6 Opposite sides are equal and parallel.
W Opposite angles are equal and the diagonal bisects each
m other. Note: other parallelograms among the quadrilaterals
12c
are rhombus, rectangle and square; NOT kite & trapezium
k
X
Rhombus
9cm All sides are equal, opposite angles are equal and the
Z diagonals bisect each other at right angles.
Y 8cm

Calculate the value of k in the figure above.


A 13cm B 15cm C 17cm D 29cm

447
Rectangle 2004/3 UME
Opposite sides are equal and the angles are 900 . The The sum of the interior angles of a pentagon is 6x + 6y.
diagonal bisect each other and are of equal length. Find y in terms of x.
A y = 90 – x B y = 150 – x C y = 60 – x D y = 120 – x
Solution
Sum of interior angles of polygon = (n –2)180
If n = 5 pentagon. Then,
6x + 6y = (5 – 2)180
Square 6(x + y) = 3×180
All sides are equal and the angles are 900. The 6(x + y) = 3 ×180
diagonals bisect each other at right angles and are of
equal length 6 6
x + y = 90
y = 90 – x (A)
2001/26 PCE
Each interior angle of a regular polygon is 7 times
Kite the corresponding exterior angle. How many sides has
Adjacent sides are equal and the diagonals bisect each the polygon?
other at right angles A 14 B 16 C 18 D 28
Solution
Find n if
Each interior angle = 7 × exterior angle
i e ( n – 2) 180 = 7 × 360
n n
Multiply through by n
( n – 2 ) 180 = 7 × 360
Trapezium ( n –2 ) 180 = 7 × 360
One pair of opposite sides is parallel 180 180
n–2=7×2
n – 2 = 14
n = 14 + 2
n = 16 (B)
1998/8 PCE
The sum of the interior angle of the regular polygon is 25200
Polygons table How many sides has it
Name No of sides A 14 B 16 C 18 D 20
Triangle 3 Solution
Quadrilateral 4 Sides is n in the formula
Pentagon 5 Sum of interior angles of polygon = (n – 2) 180
Hexagon 6 2520 = (n – 2) 180
Heptagon 7 2520 = (n – 2) 180
Octagon 8
180 180
Nonagon 9
14 = n – 2
Decagon 10
14 + 2 = n
Thus n = 16 sides (B)
2005/7 UME
The sum of the interior angles of a regular polygon is 2001/21 UME
18000.Calculate the size of one exterior angle of the polygon Find the number of sides of a regular polygon whose interior
A 120 B 180 C 300 D 240 angle is twice the exterior angle.
Solution A 2 B3 C6 D 8
First we find the number of sides (n) of the polygon. Solution
Sum of interior angles of polygon = ( n – 2) 1800 Find n if interior angle = 2×exterior
i.e. 1800 = (n –2) 180 (n–2) 180 = 2 × 360
1800 = (n – 2) 180 n n
180 180 Multiply through by n
10 = n – 2 (n–2)180 = 2 × 360
10 + 2 = n (n–2) 180 = 2 × 360
Thus n = 12 sides. 180 180
Each exterior angle = 360 = 300 n –2 = 2 × 2
12 n–2= 4
n = 4 + 2 Thus, n = 6 sides (C)
448
2002/37 UME Multiply through by n
The sum of the interior angles of a polygon is 20 right (n–2) 180 = x × 360
angles. How many sides does the polygon have? (n–2) 180 = 360 x
A 40 B 20 C10 D12 180 180
Solution n–2=2x
We are to find n from the formula n = 2x +2
Sum of interior angles of polygon = (n – 2)180 factoring out 2 we have
20 right angles = (n – 2) 180 n = 2 (x + 1) (C)
20 × 900 = (n– 2) 180
20 × 90 = (n – 2) 180 1992/27 PCE
180 180 The interior angles of a polygon are x +10, 2x + 20, x + 30,
10 = n – 2 2x + 20, x – 20, x + 20. Find the smallest of the interior angles
10 + 2 = n A 900 B 800 C 700 D 600
Thus n = 12 sides (D) Solution
Hexagon is a six sided figure. Thus
2000/23 UME Exercise 37.7 Sum of interior angles of polygon (n–2)180
In a regular polygon, each interior angle doubles its = x +10 + 2x +20 + x +30 + 2x +20 + x - 20 + x +20
corresponding exterior angle. Find the number of sides i.e (6 – 2) 180 = 8x + 80
of the polygon 8x + 80 = 4×180
A3 B4 C6 D8 8x = 720 – 80
8x = 640
1995/25 PCE Exercise 37.8 8x = 640
Find the sum of the interior angles of a polygon with 10 sides 8 8
A 10 right angles B 12 right angles x = 800
C 14 right angles D16 right angles. The smallest angle is x–20
i.e. 80–20 = 600 ( D )
1994/ 31 PCE 1991/31 PCE
If each interior angle of a regular polygon is 1350, Two of the interior angles of a polygon are 1560and 1340 and
how many sides has the polygon each of the remaining internal angles is 1510 . How many
A 45 B9 C8 D6 sides has the polygon?
Solution A5 B 10 C 12 D 15
We are to find “n” from the formula Solution
Each interior angle of polygon = (n –2)180 We are to find n in the applied formula
n sum of interior angles = (n–2) 180
135 = (n–2 ) 180 156+134+ (n–2)151 = (n – 2) 180
n The LHS (n–2) is as a result of the first two angles given, and
135n = (n –2) 180 The remaining ones is (n–2) × 151
135n = 180n – 360 290 + 151n –302 = 180n –360
Rearranging 290 + 360 –302 = 180n–151n
360 = 180n – 135n 348 = 29n
360 = 45n 348 = 29n
360 = 45n 29 29
45 45 12 = n (C)
8 = n (C) 1990/34 PCE Exercise 37.10
Find the value of each interior angle of a
1993/26 UME Exercise 37.9 12 sided regular polygon.
A regular polygon has 1500 as the size of each interior A 300 B360 C1440 D1500
angle How many sides has the polygon
A 12 B 10 C9 D8 1991/ 34 UME Exercise 37.11
If the exterior angles of a pentagon are x0 ,(x+5)0 ,
1993/27 PCE (x +10)0 ,(x +15)0 and (x +20)0 ,find x
If the interior angle of a regular polygon is x times the A 1180 B 720 C 620 D 360
exterior angle, express the number of sides of the 1990/36 UME Exercise 37.12
polygon in terms of x. Each of the interior angles of a regular polygon is 1400
A x +1 B x +1 C 2(x + 1) D 2x. How many sides has the polygon?
180 A9 B8 C7 D5
Solution
Make n the subject of the formula from: 1989/33 UME Exercise 37.13
Interior angle = x × Exterior angle A regular polygon of (2x +1) sides has 140 as the size of
(n –2) 180 = x × 360 each interior angle. Find K
n n A4 B4 ½ C8 D8 ½
449
2 2
Chapter thirty eight 1 3
Probability II (i) Pr(x = 2 ) = C2    
4

 4 4
Binomial, Poisson and approximation of
= 6 ( 0.25)2 ( 0.75 )2
binomial to Normal distributions
= 6×0.0625 × 0.5625
The term “random variable” is a function whose value is = 0.2109
real number determined by each element in the sample space. (ii) Pr ( not more than one defective )
When a die is tossed three times the outcomes are: = Pr (0 defective) + Pr (1 defective)
HHH, HHT, HTH, THH, 0 4 1 3
1 3 1 3
TTT, TTH, THT, HTT = C0     + 4C1    
4

If we are concern with the number of heads in the  4 4  4 4


outcome; then a numerical value of = 1 × (1) (0.75)4 + 4 × (0.25)1 (0.75)3
0(no head), 1(one head), 2(two heads), 3(three heads) = 0.3164 + 0.4219
will be assigned to each sample point. The numbers 0,1,2,3 = 0.7383
are random quantity determined by the outcome of an experiment
2004/15
Discrete and continuous random variables A survey shows that 60% of families in a certain area have
The two types of random variables are discrete and television set. Find the probability that among 16 families,
continuous. randomly selected in the area,
Discrete random variables represent countable data (a) 14 (b) at least 14 (c) at most 13 (d) 13 or 14
such as number of children in a family, number of have a television set.
accidents per year etc. We can have Pr(x = 3 ) but not Solution
Pr(x > 3 ) nor Pr(x < 3 ) nor Pr(x ≥ 3 ) nor Pr(x ≤ 3 ) Pr (x = r ) = nCr p r q n – r
Note that 3 here could be any number 60 3 3 2
Continuous random variables represent measured data p= i.e ; q = 1– i.e , and n = 16
100 5 5 5
such as heights, weights etc. We can have Pr(x > 3 ) or 14 2
Pr(x < 3 ) or Pr(x ≥ 3 ) or Pr(x ≤ 3 ) or forms of their 3  2
(a) Pr (x = 14) = 16C14    
range such as Pr(3≤ x ≤ 5 ) but not Pr(x = 3 ) 5  5
Note that 3 or 5 here could be any number = 120 (0.6)14 (0.4)2
= 0.01505
Binomial probability distribution (b) Pr ( at least 14 )
Binomial is an extension of Bernoulli trials or = Pr (x = 14) + Pr (x = 15) + Pr(x = 16)
experiment where the outcomes are either “success” or =16C14(0.6)14(0.4)2+16C15(0.6)15(0.4)1+16C16 (0.6)16(0.4)0
“failure”. Here P denotes success while q, which is 1– p = 0.01505 + 0.003009 + 0.0002821
denotes failure = 0.01834
The properties of binomial experiment are: (c) Pr ( at most 13 )
– The experiment consist of n repeated trials = 1 – Pr ( at least 14 )
– Each trial results in an outcome that maybe classified = 1 – 0.01834
as a success or a failure = 0.98166
– The probability of a success denoted by P remains (d) Pr ( 13 or 14 )
constant from trial to trial = Pr (13) + Pr (14)
– The repeated trials are independent 13 3 14 2
3 3 3  2
= C13     + 16C14    
16
Binomial probability distribution (definition) 5 5 5  5
Let P be the probability of success of an event and = 560 (0.6)13 (0.4)3 + 120 (0.6)14 (0.4)2
q = 1– p be failure of the event in each trial. The = 0.04681 + 0.01505
discrete random variable x is said to have a binomial = 0.06186
exactly x success in n Bernoulli trials is given by
2003/15 (a)
Pr(x = r ) = nCr p r q n – r If x denotes the number of heads in a single toss of 5 fair
Where r = 0,1,2…n and 0< p< 1 , q = 1– p coins, of the same size, find:
(i) Prob (x = 1) (ii) Prob (x 3 )
2005/15 (b) Nov Solution
Four out of every batch of 16 manufactured articles are
Pr (x = r ) = nCr p r q n – r
known to be defective. If 4 articles are picked at random
1 1
from the batch, what is the probability that they contain: Here P = , and q = Head or tail in coin and n = 5
( i ) exactly 2 defective articles 2 2
1 4
( ii ) not more than one defective article ? 1 1
Solution (i) Pr (x = 1 ) = C1    
5

Pr (x = r ) = nCr p r q n – r  2 2
4 1 1 3 = 5(0.5)1 (0.5)4
Here p = i.e ; q = 1– i.e and n = 4 = 0.1563
16 4 4 4
450
(ii) Pr (x 3) = Pr (x = 0) + Pr (x = 1) + Pr (x = 2) Solution
0 5 1 4 2 3 Pr (x = r) = nCr p r q n – r
1 1 5 1 1 5 1 1
= 5C0     + C1     + C2     1 1 4
2 2  2 2 2  2 Here P = i.e (0.2), q = 1– = i.e (0.8) and n = 5
5 5 5
= 1×1 × (0.5)5 + 0.1563 + 10 (0.5)2 (0.5)2
= 0.03125 + 0.1563 + 0.3125 (i) Pr (x = 5) = 5C 5 (0.2)5 (0.8)0
= 0.5001 = 0.00032
Note x = 3 was not included because of the strict inequality 
(ii) Pr (x = 5 ) = 1 – Pr (x = 5)
2003/15 (b) = 1 – 0.00032
The probability that automobiles using a certain road = 0.99968
will have good types is 0.6 . If 10 cars are stopped in a (iii) Pr (x = 1) = 5C 1 (0.2)1(0.8)4
random safety check, find, correct to 4 decimal places, = 0.4096
the probability that: 2002/15 (a)
(i) exactly 2 will not have good types In a survey, 5 out of every 500 Journeys made in a
(ii) at least 8 will have good types. company’s vehicle end up in road accidents. If 10 of the
Solution company’s vehicles are chosen at random for a Journey, Find
Pr (x = r ) = nCr p r q n – r the probability that:
Here p = 0.6 ; q = 1 – 0.6 i.e 0.4 and n = 10 (i) The Journey will be accident - free
(i) Pr (x = 2 not have) = 1 – Pr (x = 2 have) (ii) 2 of the vehicles will have accidents.
= 1 – 10C2(0.6)2 (0.4)8 Solution
=1 – 45(0.36) (0.4)8 Pr (x = r) = nCr p r q n – r
Multiply starting with (0.4)8 5
= 1 – 0.01062 Here P is = 0.01, q = 1– 0.01 i.e 0.99 and n is 10
500
= 0.98938
(i) Pr (x = 0) = 10C0(0.01)0 (0.99)10
 0.9894 to 4 d.p
= 0.904
(ii) Pr ( at least 8)
= Pr (x = 8) + Pr (x = 9)+ Pr (x = 10) (ii) Pr( x = 2) = 10C2(0.01)2(0.99)8
= 10C8(0.6)8 (0.4)2 +10C9(0.6)9(0.4)1 + = 0.004
10
C10(0.6)10)(0.4)0 (iii)Pr (at most 2) = Pr(x = 0) + Pr(x = 1) + Pr(x = 2)
= 45 (0.6)8 (0.16) + 10 (0.6)9(0.4) + 1(0.6)101 (i) has Pr(x=0)and (ii) has Pr(x=2) next, we find Pr(x=1)
Multiply terms with power firstly Thus, Pr(x = 1) = 10 C 1 (0.01)1(0.99)9
= 0.1209 + 0.04031 + 0.006047 = 0.091
= 0.167257 Therefore, Pr(at most 2) = 0.904 + 0.091 + 0.004
 0.1673 to 4 d.p = 0.999
2003 / 15 (a) Nov 2000/14 (a)
If x denotes the number of heads in a single toss of 5 Out of every 100 applicants for admission into the
fair coin of the same size, find: University, 20 are successful. If 10 applicants are selected at
random, find, correct to 3 significant figures,
(i) prob (x = 0) (ii) prob (x 2)
successful applicants are:
Solution
(i) exactly 4; (ii) at most 3 (iii) at least 9
Pr(x = r ) = nCr p r q n – r
Head Tail Solution
Pr( x = r) = nCr p r q n – r
Tossing of coin, p = 0.5 and q = 0.5
20
(i) Pr x = 0) = 5C0 (0.5)0 (0.5)5 P= i.e. 0.2 and q = 1 – 0.2 i.e. 0.8
= 1×1× (0.5)5 100
= 0.03125 (i) Pr (x = 4) = 10C 4 (0.2)4(0.8)6
(ii) Pr (x 2) = Pr (x = 0) + Pr (x = 1) = 210 (0.0016) (0.8)6
= 0.08808
= 5C 0 (0.5)0(0.5)5 + 5C 1 (0.5)1(0.5)4
( ii ) Pr(at most 3) = Pr(x = 0)+Pr(x = 1)+Pr(x =2)+Pr(x = 3)
= 0.03125 + 5(0.5) (0.5)4 =10C0(0.2)0(0.8)10+10C1(0.2)1(0.8)9+10C2(0.2)2(0.8)8+10C3 (0.2)3(0.8)7
= 0.03125 + 0.15625 = 1×1(0.8)10 + 10(0.04)(0.8)8 + 120(0.008)7
= 0.1875 First process items with powers before others
Notex=2 was not included because of the strict inequality  = 0.1074 + 0.06711 + 0.2013 = 0.37581  0.376 to 3 s.f
(iii) Pr (at least 9) = Pr (x = 9) + Pr (x =10)
2003 / 15 (b) Nov
= 10C9(0.2)9 (0.8)1 + 10C10(0.2)10 (0.8)0
The probability of a student coming late to school is 1/5 .
= 10(0.2)9 (0.8) + 1(0.2)10×1
Out of 5 school days in a week, find the probability that
= 0.000004096 + 0.0000001024
he will
= 0.0000041984
(i) be late to school everyday,
(ii) not be late to school at all,  0.00000420 to 3 s.f
(iii) be late for one day only
451
1999/13 (Nov) 1993/27 (Nov)
In an examination, 30% of the candidates failed. If 10 In a box of 100 items 10 are selected at random from the
of these candidates are selected at random, find the box, what is the probability that 2 of them are defective?
probability that: A 0.729 B 0.4 C 0.04 D 0.0729 E 0.00729
(a) 6 failed (b) at least 8 passed Solution
(c) at most 3 failed (d) none passed the examination Pr (x = r) = nCr p r q n – r
Solution 10
P= i.e 0.1 and q is 1– 0.1= 0.9 ; n is 5
Pr (x = r) = nCr p r q n – r 100
30 Pr(x = 2 ) = 5C2(0.1)2(0.9)3
Here P = i.e 0.3; q is 1– 0.3 i.e 0.7 and n is 10
100 = 10 (0.01)(0.9)3
= 0.0729 (D)
(a) Pr (x = 6) = 10 C6(0.3)6 (0.7)4 1992/30 (Nov)
= 0.037 It is known that 2 out of every 5 cigarette smokers in a
(b) Pr (at least 8 passed) village have cancer of the lungs. Find the probability that,
= 1– { Pr (8 failed) + Pr (9 failed) + Pr (10 failed) out of a random sample of 8 smokers from the village 5 will
= 1–10C8(0.3)8(0.7)2 + 10C9(0.3)9(0.7)1+10C10 have cancer of the lungs
(0.3)10(0.7)0} A 0.1239 B 0.1737 C 0.2500 D 0.8263 E 0.8761
= 1 – { 45(0.3)8(0.49) + 10(0.3)9(0.7)1 + 1(0.3)10 × 1 } Solution
First process items with power Pr (x = r) = nCr p r q n – r
= 1 – {0.001447 + 0.0001378 + 0.000005905} 2
P= i.e 0.4 and q is 1– 0.4 = 0.6; n is 8
= 1 – 0.001591 5
= 0.9984 Pr (x = 5 ) = 8C5(0.4)5(0.6)3
(c) Pr (at most 3 failed) = 56(0.4)5 0.216
= Pr(x = 0) + Pr(x = 1) + Pr(x = 2) + Pr(x = 3) = 0.12386304
= 10C0(0.3)0(0.7)10 + 10C1(0.3)1(0.7) 9+10C3(0.3)3(0.7)7  0.1239 (A)
= 1×1(0.7)10 + 10(0.3)1(0.7) 9 + 120(0.3)3(0.7)7 2010/14a Neco
First process items with power Find the probability that in a family of 6 children there will
= 0.0282475 + 0.1210608 + 0.2668279 be
= 0.4161362 (i) at least 2 boys
(ii) at least 1 boy and 1 girl
(d) Pr (none passed the exam) = Pr (All failed exam)
(iii) at most 3 boys
= Pr (x = 10)
Solution
= 10C 10 (0.3)10(0.7)0 Pr (x = r ) = nCr p r q n – r
= 1(0.3)10×1 1 1
Here P = , and q = Male or Female in a family and n = 6
= 0.000005905 2 2
1996/5 (Nov) (i) Pr ( at least 2 boys ) = 1 – [Pr( at most 1 )]
A new technique of curing a contagious disease has = 1 – [Pr (0) + Pr(1)]
been discovered and investigations reveal that out of = 1 – [ 6C0(0.5)0 (0.5)6]
every 10 patients treated by the new technique, 7 are = 1 – [1 x1(0.5)6]
cured. If 12 patients are treated by this technique, what = 1 – 0.015625
is the probability, correct to two significant figures, that = 0.984375
(a) exactly 7, (b) at most 10 are cured ?
Solution (ii) Pr(at least 1 boy and 1 girl)
Pr (x = r) = nCr p r q n – r = 1– [Pr(no boy and no girl at )]
= 1 – [Pr(zero boy and zero girl )]
7
P= i.e. 0.7 and q is 1– 0.7 = 0.3; n is 12 = 1– [ 6C0(0.5)0 (0.5)6 x 6C0(0.5)0(0.5)6]
10 = 1– [1 x1(0.5)6 x 1 x1(0.5)6 ]
(a) Pr (x = 7) = 12C7(0.7)7(0.3)5 = 1– [0.015625 x 0.015625 ]
= 792 (0.7)7 (0.00243) = 1– 0.00024414
= 0.158496 = 0.999756
 0.16 to 2 s.f 2008/13 Exercise 38.1
(b) Pr (at most 10) = Pr (0) + Pr (1) + … + Pr (10) A survey indicated that 65% of the families in an area have
For ease of calculation we proceed as: cars. Find, correct to three decimal places, the probability
= 1 – {Pr (x = 11) + Pr (x = 12)} that among 7 families selected at random in the area
= 1 – [12 C11(0.7)11 (0.3)1 + 12C12 (0.7)12 (0.3)0] (a) exactly 5; (b) 3 or 4; (c) at most 2 of them have cars
= 1 – [12 (0.7)11 (0.3) + 1 (0.7)12 1]
= 1 – [0.071184 + 0.013841] 2010/14a Exercise 38.2
= 1 – 0.085025 Eight coins are tossed once. Find, correct to three decimal
= 0.914975 places, the probability of obtaining
(i)exactly 8 heads; (ii)at least 5 heads; (iii)at most 1 head
 0.91 to 2 s.f
452
2010/47 Neco Exercise 38.3 e 2 2 1
A fair coin is tossed five times. Find the probability of (i) Pr (x = 1) =
obtaining at least 4 heads. 1!
1 2 3 5 1 2
A B C D E
2 5 16 32 32 = = 0.271
e2
2004/15 Neco Exercise 38.4
( ii ) Pr (more than two) = 1 – [Pr(0) + Pr(1) + Pr(2)]
In every 5 piglets given birth to by a pig, 2 will die
before 3 months. If the pig gives birth to 9 piglets, what e  2  20 e  2  21 e  2  2 2
=1– + +
is the probability correct to 2 places of decimal that: 0! 1! 2!
(i) at most 3 will die?
(ii) exactly 3 will die? 1 2 2
= 1 – + +
2001/8 (a) Neco Exercise 38.5 e2 e2 e2
A fair coin is tossed five times. What is the probability 5
=1 – 2
of having at least 3 heads in the five tosses? e
= 0.323
2003/15 (b) Dec Neco Exercise 38.6
The probability of a woman giving birth to either male 1998/5 (Nov) Adjusted
or female child is of equal chance. If a woman gives Out of every 1,000 items produced daily in a factory, 50 of
birth to seven children, what is the probability that them are found to be defective. If 100 of them are selected at
exactly three children are girls random, what is the probability correct to 3 decimal places,
that at most 3 are defective?
2005/5 Neco Exercise 38.7 Solution
A company observes that on the average 10% of the Pr (at most 3 defective) = Pr(0) + Pr(1) + Pr(2) + Pr(3)
bolts produced by a given machine will be defective for 50
a certain specified requirement. If 5 bolts produced by Here P = i.e 0.05 and q = 0.95
the machine are selected at random, find the probability 1000
that exactly 3 will be defective. Using binomial Pr (x = r) = nCr p r q n – r
Pr (x = 0 ) = 100C0(0.05)0( 0.95 )100
Poisson distribution can be used to approximate Which makes the working untidy
binomial distribution if: Here, we will apply Poisson distribution since P is close to zero
– n is large and p is close to zero Poisson conditions: P 0.1 and  = np  5
–  = np  5 and P  0.1 Here P: (0.05)  0.1 and : 100 × 0.05 = 5
Poisson distribution e  x
Experiments yielding numerical values of a random Thus, applying Pr (x) =
variable x ; the number of success occurring during a
x!
given time interval or in a specified region are called Pr (at most 3 defective) = Pr(0) + Pr(1) + Pr(2) + Pr(3)
e 5 5 e  5 5 e 5 5 e 5 5
0 1 2 3
Poisson experiment. Its formula is:
= + + +
e  x 0! 1! 2! 3!
Pr (x) = Where x = 0,1, 2…n
x! 1 5 25 125
= 5+ 5+ +
Poisson distribution is also discrete distribution just as binomial
e e 2 e5 6 e5
Eg P1 Process as:
If 1 out of every 1000 patients treated with a new 1 25 125
malaria drug suffers a reaction. Determine the = 1+5+ +
e 5
2 6
probability that out of 2000 patients:
( i ) exactly one = 0.00673795[39.33333333]
( ii ) more than two patients suffers a reaction = 0.265026033
Solution  0.265 to 3 d.p
P = 0.001 and q is 0.999 2005 /15 (Dec) Neco
Using binomial Pr (x = r) = nCr p r q n – r Out of 5,000 seeds of cocoa planted, only 2 did not reach the
Pr (x = 1) = 2000C1(0.001)1( 0.999 )1999 age of 40 years. If 10,000 seeds are planted, find the
Which makes the working untidy probability that:
Here, we will apply Poisson distribution since P is close to zero (a) exactly 3 will not reach the age of 40 years,
e  x (b) at most 3 will not reach the age of 40 years, and reach
Thus, applying Pr (x) = (c) at least 4 will not reach the age of 40 years.
x! Solution
Poisson conditions: P 0.1 and  = np  5 Poisson conditions: P 0.1 and  = np  5
Here P: (0.001)  0.1 and : 2000 × 0.001 = 2 5 2
P= i.e 0.0004 0.1 and = 10000 × 0.0004i.e 45
5000
453
e   x The next task is changing of Binomial distribution question
Thus, applying Pr (x) = data which are discrete like Pr(x = 3) to a Normal
x! distribution question data which is continuous in nature like
e 4 4 3 43 1 Pr(2.5 x 3.5); we will learn the basics on it in the
(a) Pr ( x = 3) = or  following examples
3! 3! e 4
Eg BN1
= 0.195
The probability that a patient recovers from a delicate heart
(b) Pr (at most 3) operation is 0.5. Out of the next 100 patients operated, find
= Pr (x = 0 ) + Pr (x = 2 ) + Pr (x = 3) the probability that
e 4  4 0
e 4 4 1 e 4 4 2 e 4 4 3 (a) exactly 60 survives (b) between 48 and 53 survives
= + + + (c) less than 45 survives
0 ! 1! 2! 3! Solution
1 4 16 64 Here survival is p i.e 0.5 and q (death) is 0.5
= 4 + 4 + 4 + np = 100 × 0.5 = 50  5
e e 2e 6e 4
Next, we approximate the Binomial distribution by Normal
1 4 8 32 distribution with standardized variable given as:
= 4 + 4 + 4 +
e e e 3e 4 x  np x
Z i.e Z 
= 0.0183 + 0.0733 + 0.1465 + 0.1954 npq 
= 0.4335
(c) Pr (at least 4) = 1 – Pr (at most 3)
 = np is 50 and  = npq = 100  0.5  0.5 = 5
= 1 – 0.4335 (a) Pr( x = 60)  Pr( 59.5  x  60.5)
= 0.5665 Standardizing the given values
2000/14 Neco Exercise 38.8 x = 59.5 x = 60.5
One out of 500 people reacted to a new drug. If 2500 z = 59.5 – 50 z = 60.5 – 50
people were treated with it, find the probability that: 5 5
(i) exactly two people reacted to the drug, z = 1.9 z = 2.1
(ii) at most two people reacted to the drug, Thus Pr( 59.5  x  60.5) = Pr( 1.9  Z  2.1)
(iii) at least four people reacted to the drug. By inequalities we sketch the normal curve as:

Exercise 38.9
A HIV test carried out in city shows that 1 out of every
1,000 dwellers were positive. Find that out of 8000 dwellers Z
(i) exactly two (ii) more than 3 are HIV positive 01
.9 2
.1
Interpreting the resulting normal curve
2003/15 b Neco Exercise 38.10 = Pr(Z = 1.9) – Pr(Z = 2.1)
The probability that a person gets a reaction from a new = 0.4821 – 0.4713
drug in the market is 0.001. If 3000 people are treated = 0.0108
with this drug, use Poisson probability distribution to (b) Pr(48  x  53)  Pr( 47.5  x  53.5)
find the probability that: Standardizing the given values
(i) exactly four people will get a reaction x = 47.5 x = 53.5
(ii) more than two persons will get a reaction z = 47.5 – 50 z = 53.5 – 50
(Give your answers to 3 d.p) 5 5
z = –0.5 z = 0.7
Approximation of Binomial to Thus Pr( 47.5  x  53.5) = Pr(– 0.5  Z  0.7)
Normal distributions By inequalities we sketch the normal curve as
We were told use Poisson formula when it is clumsy for
us to use Binomial formula in probability distribution.
Precisely when n is large and p  0.1 & that  = np  5
This we have seen reason with the proposals of the idea
with examples to show for it -0.5 0 0.7
But when np  5 By symmetry Pr(– 0.5  Z  0.7) = Pr(z = 0.7 ) + Pr(z = 0.5 )
We approximate the Binomial distribution by = 0.2580 + 0.1915
Normal distribution with standardized variable = 0.4495
given as: (c) Pr( x < 45)  Pr(x  44.5)
x  np x Standardizing the given value
Z i.e Z  x = 44.5
npq 
z = 44.5 – 50 ; z = –1.1
5
454
Thus Pr(x  44.5) = Pr( Z  –1.1)
By inequalities we sketch the normal curve as:
Diagrammatically By symmetry Diagram

0 1.1
-1.1 0
Pr( Z  –1.1) = 0.5 – Pr( Z = 1.1)
= 0.5 – 0.3643
= 0.1357
2004/15 (Dec) Neco
2 out of 1000 women in Oyo State give birth to their
children through caesarian section (C.S). If 3000 women
give birth in Oyo State, what is the probability that;
(a) exactly 5 will give birth by C.S ?
(b) at most 3 will give birth by C.S ?
(c) none will give birth by C.S ?
Solution
Poisson conditions: P 0.1 and  = np  5
2
P= i.e 0.002 0.1 and  = 3000 × 0.002i.e 6  5
1000
Thus, Poisson conditions fails here,
Next, we approximate the Binomial by Normal
distribution with standardized variable given as:
x  np x
Z i.e Z 
npq 
 = np is 6 and
 = npq = 3000  0.002  0.998 = 5.988
(a) Pr( x = 5)  Pr( 4.5  x  5.5)
Standardizing, we have
4.5 – 6  z  5.5 – 6
5.988 5.988
= Pr(–0.25  z  –0.0835)
By symmetry

Z Z
-0.25 -0.0835 0 0 0.0835 0.25

Interpreting the resulting normal curve


= Pr(Z = 0.25) – Pr(Z = 0.0835)
= 0.0987 – 0.0331
= 0.0656

455
Chapter thirty nine 2002/19
Coordinate geometry and conic The distance between the point (1, 2) and (x, –1) is 5units.
Find the possible values of x
Coordinate geometry I A 3 or 5 B – 3 or – 5 C – 3 or 5 D 3 or – 5
Distance between two points Solution
Let P(x1, y1) and Q(x1, y1) be two points on a Cartesian ( x2  x1 ) 2  ( y 2  y1 ) 2
Distance between two points =
plane as in fig I. then deductively, we can complete the
diagram as in fig II The order is (1, 2) and (x, – 1)
(x1, y1) (x2, y2)
y y Q(x2, y2 ) Substituting
Q(x2, y2 )
5  ( x  1) 2  (1  2) 2
y2 - y1
Take square of both sides to clear square root
P(x1, y1 ) 25 = (x –1)2 + (–3)2
P(x1, y1 ) x2 - x1 25 = x2 – 2x + 1 + 9
x x
Solving the resulting quadratic equation
Fig I
x2 – 2x – 15 = 0
Fig II
x + 3x – 5x – 15 = 0
2
The distance between P and Q can be gotten by
x(x + 3) – 5(x + 3) = 0
Pythagoras rule as PQ2 = (x2 – x1)2 + (y2 – y1)2
(x + 3)(x – 5) = 0
PQ = ( x2  x1 ) 2  ( y2  y1 ) 2 x + 3 = 0 or x – 5 = 0
x = – 3 or 5 (C)
Distance of a point Q(x2, y2) from the origin P(0, 0)
y Q(x2, y2 ) 2003/8 (Nov)
The distance between point (2m, – 2) and (m, –5) is
3 2 units. Find the possible values of m
y2 - y1 A 3 B 4 C 6 D 7
Solution
x Distance between two points = ( x2  x1 ) 2  ( y 2  y1 ) 2
P(x1, y1 ) x2 - x 1
The order is (2m, –2) and (m, –5)
Fig III
(x1, y1) (x2, y2)
Substituting
PQ = ( x2  0) 2  ( y2  0) 2 3 2  (m  2m) 2  (5  (2)) 2
PQ = x 22  y 22 Take square of both sides to clear square root
(3 2 )2 = (–m)2 + (–3)2
2002/ 1 (Nov) 18 = m2 + 9
Find the distance between the points (–2, –5) and (–1, 3). m2 = 9
A 5 B 17 C 65 D 73 m =  3 (A)
Solution
2003/ 38 Neco
Distance between two points = ( x2  x1 ) 2  ( y 2  y1 ) 2 Find the distances between point P(5, 4) and the point
The order is (–2, –5) and (–1, 3) common to the lines 2x – y = 4 and x + y = 2
(x1, y1) (x2, y2) A 13 B 18 C 5 D 30 E7
Substituting Solution
= [1  (2)]2  [3  (5)]2 Distance between two points = ( x2  x1 ) 2  ( y2  y1 ) 2
= 1 8 2 2 = 65 (C) Our (x1, y1) is given as P (5, 4); to get (x2, y2)
we solve the simultaneous linear equations
1994/ 14 2x – y = 4
Find the distance between the two points (–2, 3) and (– 4, –2) + (x + y = 2)
A 29 B 37 C7 D 29 E 37 3x = 6
Solution x=2
The order is (–2, 3) and (– 4, –2) Substituting for x value into x + y = 2,
(x1, y1) (x2, y2) y=2–2 = 0
Distance between two points = ( x2  x1 ) 2  ( y 2  y1 ) 2 Thus (x2, y2) is (2, 0)
2 2 Distance between two points = (2  5) 2  (0  4) 2
= [4  (2)]  (2  3)
= (3) 2  (4) 2
= (2) 2  (5) 2 = 29 (A)
= 25 = 5 (C)
456
Example D7 Mid-point theorem (Equal division)
y
Show that P(2, 3), Q(–3, – 2) and R(2, – 7) are the B(x2, y2 )
vertices of an isosceles triangle
Solution C(x , y)
y2 - y

The question wants us to show that two distances out of the


three distances are equal (since two side of isosceles triangle y - y1
are equal) A(x1, y1 )
P(2, 3) Q(–3, –2) and R(2, – 7) x - x1 D x2 - x E

PQ = (3  2) 2  (2  3) 2  25  25  50
x
PR = (2  2) 2  (7  3) 2  100
A B are point(x1, y1) and (x2, y2) which C (x, y) is the
QR = (2  (3)) 2  (7  (2)) 2  25  25  50
midpoint of AB. It follows that
PQ = QR hence result AC = CB and AD = DE EF = BF
2000/40 x – x1 = x2 – x y – y1 = y2 – y
The coordinates of points P, Q and R are (0, 0), (15, 0) 2x = x1 + x2 2y = y1 + y2
and (0, 8) respectively. Calculate the perimeter ofPQR x1  x2 y y
x= y= 1 2
A 23 B 40 C 46 D 64 2 2
Mid-point coordinates is  x1  x 2  ,  y1  y 2 
Solution
Let us sketch the triangle as:  2   2 
Q(15, 0)

Example M1
Find the mid point of the line joining (4, 2) and (6, 10)
Solution
x x  y  y2 
P(0, 0) R(0, 8) Mid point coordinates are  1 2  ,  1 
 2   2 
The perimeter of PQR will be sum of distance PQ,QR & PQ
 4  6   2  10 
Distances PQ = (15  0) 2  (0  0) 2 = 15 =  ,  
 2   2 
Distances QR = (0  15) 2  (8  0) 2 = 17  10 12 
=  , 
2 2
Distances RP = (0  0) 2  (0  8) 2 = 8 = (5, 6)
Thus perimeter of PQR = 15 + 17 + 8 Example M2
= 40 (B) Find the midpoint of the segment PQ
2004/16 Exercise 39.1 where P = (2, 7) and Q (–8, –1)
Find the distance between the point (– 4, –3) and the point of Solution
intersection of the lines x – 2y –5 = 0 and 2x – y – 1= 0 Let the midpoint be R(x, y), then the coordinates of R are:
A 3units B 5 3 units C 3 5 units D 9units x1  x2 y y
2004/ 39 Neco (Dec) Exercise 39.2 x= , y= 1 2
2 2
Find the distances of the point (4, –2) from the line 28 7 1
2x + 3y – 6 = 0 x= , y=
5 3 0 4 6
2 2
A B C D – E – 6 6
13 13 13 13 13 x= , y=
2 2
x = – 3, y=3
R = (–3, 3)
2004 /12
The midpoint of point M(4, 1) and N(x, y) is P(2, – 4).
Find the coordinates of N
A (2, –3) B (2, –7) C (–1, –3) D (–10, –7)
Solution
Mid point coordinates are
x x y y
x= 1 2 , y= 1 2
2 2
Here our (x , y ) is P(3, – 4) and
l l

(x2, y2) is (x, y)


(x1, y1) is (4, 1)
Substituting
x x y y
xl = 1 2 , yl = 1 2
2 2
457
4 x 1 y internal division external division
3= –4= DF p DF p
2 2  
6=4+x –8=1+y FE q FE q
x=2 –9 = y x  x1 p x  x1 p
 
coordinates of N is (2, – 9) x2  x q x  x2 q
qx – qx1 = px2 – px qx – qx1 = px – px2
2000/7 Make x subject formula Make x subject formula
Given the points P(–5, 3) and Q(7, – 1) such that M is px + qx = px2 + px1 px – qx = px2 – px1
the mid- point of the straight line PQ, find  PM  . x(p + q) = px2 +qx2 x(p – q) = px2 –qx2
A 4 2 B 4 10 C 3 10 D 2 10 px2  qx1 px2  qx1
x= x=
Solution pq pq
The question want us to find distance PM. It follows that similarly
First, we find the coordinates of M, which is the py  qy1 py2  qy1
y= 2 y=
mid-point of PQ pq pq
Coordinates of mid point PQ are
x =  x1  x2  , y =  y1  y2  2011/ 11 Neco
 2   2  Find the coordinates of the point which divides internally the
5  7 3 1 line joining the points (8, 9) and (–7, 4) in the ratio 2 : 3
x= , y= A (1, 2) B (2, 4) C (2, 7) D (5, 1) E (7, 1)
2 2
Solution
M (1, 1)
Internal division coordinates formula
P(–5, 3) & M(1, 1) Thus,  PM  = [1  (5)]2  (1  3) 2 px2  qx1 py2  qy1
x= , y=
pq pq
= 36  4
Where p and q are the ratio p : q
= 40 = 2 10 (D) Substituting
2( 7)  3(8) 2(4)  3(9)
x= , y=
2005/15 PCE Exercise 39.3 23 23
The midpoint of p( 5, - 2 ) and Q( x, y ) is ( 4, - 1 ). 14  24 8  27
Find the coordinates of point Q. = , =
5 5
A.( - 3, 2 ) B.( - 2, 3 ) C.(3, 0 ) D.( 0, 3 ) 10 35
= , =
5 5
2005/11 UME Exercise 39.4
Find the coordinates of the midpoint of the line joining = 2, =7
( 3, – 4 ) and (– 1, 10 ). Thus the internal divisions coordinate is (2, 7) C.
A.( 2, 3 ) B. (1, 3 ) C.( 3, 2 ) D.( 1, 2 )
Example DR 3
2004/5 UME Exercise 39.5 Find the ratio which the point (4, 2) divides the line joining
Find the midpoint of the line joining P(-3, 5) and Q(5, -3) the points (2, – 4) and (8, 14)
A.( 1, 1 ) B. ( 2, 2 ) C.( 4, 4 ) D.( 4, – 4 ) Solution
Coordinates of the point of division are
px2  qx1 py2  qy1
x= y=
The coordinates of the point which divide the line pq pq
joining two given points internally and externally in a 8 p  2q 14 p  4q
4= , 2=
given ratio pq pq
(Division or section formula) From the x – coordinate
y y
B(x2, y2 ) C(x , y) 4p + 4q = 8p +2q
C(x , y) q B(x2, y2 ) q
2q = 4p
4p = 2q
p p
A(x1, y1 ) A(x1, y1 ) p 2
S T T S 
q 4
p 1
x x 
D F E D E F q 2
Internal division External division
C division AB in ratio p : q C division AB in ratio p : q Thus ratio is 1: 2
It can be confirmed by the y – coordinate.
DF : FE = AS : ST = AC : CB = p : q

DF p
Thus 
FE q

458
Solution
Collinear points have equal gradient
Appling the formula of gradient between two points
y 2  y1
Gradient 
x 2  x1
Gradient (2, 5) to (k, – 1) = Gradient (k, 1) to (–4, –7)
1  5 7  (1)

k 2 4k
6 6

Collinear points k 2 4k
y
– 6 (– 4 – k) = – 6(k – 2)
Minus 6 will cancel out
–4–k = k–2
2k = – 2
k = – 1 (C)
2004/14
x If the points (–3, 4), (k, 1) and (5, – 6) are collinear,
find the value of the constant k.
A –7 B – 5 C 1 D 9
The three lines above are collinear, have same gradient
Solution
and are parallel to each other.
Collinear points have equal gradient
Three or more point is collinear if the gradients between
Applying the formula of gradient between two points
adjacent pairs of point are equal.
y 2  y1
Or three points will be collinear if the area of the Gradient 
x 2  x1
triangle formed by them is zero (Determinant method of
solving area of triangle) Gradient (–3, 4) to (k, 1) = Gradient (k, 1) to (5, – 6)
1 4 6  1

Example CP1 k  (3) 5k
Show that A(1, 4), B(3, –2) and C(–3, 16) are collinear 3 7
points 
k 3 5k
Solution –3(5 – k) = –7(k + 3)
2  4 6 –15 + 3k = – 7k – 21
Gradient A to B = i.e    3
3 1 2 10k = – 6
16  (2) 18 2000/11a
Gradient B to C = i.e  3 The equation of the side PQ, PR and QR of a triangle PQR
33 6
The gradients are equal so the points are collinear. are:
Alternatively 2x + 7y + 3 = 0;
Area of triangle whose vertices are (x1, y1), (x2, y2), (x3, y3) 3x – 8y + 23 = 0;
1 1 1 5x – y –11 = 0 respectively
= 1 Find the:
x1 x2 x3 (i) Coordinates of the points P, Q and R,
2
y1 y2 y3 (ii) Area of  PQR, using determinant method
1 1 1 Solution
1 Solving the linear equations simultaneously
Area of ABC  1 3  3 PQ and QR will give Q coordinates,
2
4  2 16 PR and PQ will give P coordinates
PR and QR will give R coordinates
1  3 3 1 3 1 3 
= 1 1 1  PQ and QR
2   2 16 4 16 4  2 
2x + 7y + 3 = 0 -------- (1)
1 5x – y – 11 = 0 --------- (2)
= (48  6)  1(16  12)  1(2  12)
2 (2)  7 and add
1 35x – 7y = 77
= [42 – 28 – 14] + ( 2x + 7y = – 31)
2
= 0 implies required result 37x = 74
x = 74 37 i.e 2
2004/14 Adjusted Substitute x value into (2)
If the points (2, 5), (k, –1) and (–4, –7) are collinear, 5x – y = 11 becomes
find the value of the constant k 10 – y = 11
A–7 B–5 C–1 D1 y = – 1 Thus, Q coordinates is (2, – 1)
459
PR and PQ
3x –8y + 23 = 0 --- (a) Locus of points
2x + 7y + 3 = 0 --- (b) To identify a point Q(x, y) on the x – y plane we use its
(a)  2, (b)  3 and subtract relationship with the x and y coordinates
6x – 16y = – 46 2004/10
– (6x + 21y = – 9) The point D(2, 1), E(1, 5) and F(–6, –1) are the midpoints of
– 37y = – 37 the side AC, AB and BC respectively of a triangle ABC
y=1
(a) show that  DEF is right – angled;
Substitutes y value into (b)
(b) find the coordinate of B;
2x + 7y = – 3 becomes
(c) find the equation of AC
2x + 7 = – 3
Solution
x = – 10 2 i.e. – 5 (a) By Pythagoras rule, we are to show that one out of
Thus, P coordinates is (–5, 1) DE2, DF2 and EF2 is equal to sum of the other two.
DE2 = (1 – 2)2 + (5 – 1)2 i.e 17
PR and QR DF2 = (– 6 –2)2 + (–1 – 1)2 i.e 68
3x – 8y + 23 = 0 ----(c) EF2 = (– 6 –1)2 + (–1 –5)2 i.e 85
5x – y –11 = 0 ----- (d) Since EF2 = DE2 + DF2
(d)  8 and subtract Thus  DEF is right angled.
40x – 8y = 88
– (3x – 8y = – 23) 2003/ 46 Neco
37x = 111 Find the equation of locus point P(x, y) such that
PV = PW, where V = (1, 1) and W = (3, 5)
x = 111 37 i.e 3
A 2x + 2y = 9 B 2x + 3y = 8 C 2x + y = 9
Substitutes y value into (d) D x + 2y = 8 E x – 2y = 5
5x – y = 11 becomes Solution
15 – y = 11 The description given above is that of equidistant
y=4 PV 2 = PW 2
Thus R coordinates is (3, 4) (1 – x) + (1 – y)2 = (3 – x)2 + (5 – y)2
2

1 – 2x + x2 + 1 – 2y + y2 = 9 – 6x + x2 + 25 – 10y + y 2
(ii) Area of  PQR with verities P(–5, 1), Q(2, –1) and x2 – 2x + y2 – 2y + 2 = x2 – 6x + y2 – 10y + 34
R(3, 4) is 6x – 2x + 10y – 2y + 2 – 34 = 0
Area of triangle whose vertices are (x1, y1), (x2, y2), (x3, y3) 4x + 8y – 32 = 0
1 1 1 x + 2y = 8 (D)
= 1 x1 x2 x3 2003/9i Neco Adapted
2 A is the point (3, –1) and B is the point (5, 3)
y1 y2 y3 (i) Show that the locus of a point p(x, y) which moves
so that (AP) 2 + (BP) 2 = 28 is a circle
1 1 1 Solution
1 (AP) 2 + (BP) 2 = 28
= 5 2 3
2 Distance (3, –1) and (x, y) + distance (5, 3) and (x, y) = 28
1 1 4 (x – 3)2 + (y + 1)2 + (x – 5)2 + (y – 3)2 = 28
x – 6x + 9 + y2 + 2y + 1 + x2 – 10x + 25 + y2 – 6y + 9 = 28
2
1  2 3 5 3 5 2 
= 1 1 1  2x2 + 2y2 – 16x – 4y + 16 = 0
2   1 4 1 4 1  1  x2 + y2 – 8x – 2y + 8 = 0
1
= [ (8 +3) – 1(– 20 – 3) + 1( 5 – 2)] 2004/ 6i Neco (Dec)
2
1 A point moves so that its distance from the origin is twice its
= (11  23  3) distance from the point (3, 2)
2 (i) Show that the locus is a circle
37
= 2 i.e 18.5 square units Solution
Let the point be P(x, y) the origin be A(0, 0) and Next point
be B(3, 2)
Then the locus described above is
AP 2 = 2BP 2
Distant between A(0, 0) and P(x, y)
= 2[Distant between P(x, y) and B(3, 1)]
(x – 0)2 + (y – 0)2 = 2[(x – 3)2 + (y – 2)2]
x2 + y2 = 2[x2 – 6x + 9 + y 2 – 4y + 4]
x2 + y2 = 2x2 + 2y2 – 12x – 8y + 26
x + y2 – 12x – 8y + 26 = 0
2

460
1998/35 PCE
Find the gradient of the line whose equation is given
( a – b )x + ( c – d )y = e – f
A. b – a B. a – b C. b – a D. e – f
c–d c–d c-d c–d
Solution
First, we rearrange the given equation into y = mx + c
format
( c – d )y = – ( a – b )x + ( e – f )
Divide through the coefficient of y
y = – ( a – b )x + e – f
c–d c–d
Therefore,
gradient m= –(a–b)
c–d
opening up the bracket
Equation of a straight line m = – a + b or b – a
is given as y = mx + c c–d c–d (A)
2001/24 UME
1993/39 PCE
A straight line makes an angle of 300 with the positive
The intercept which the line 2x – 3y – 5 = 0 makes with the
x – axis and cuts the y – axis at y = 5. find the equation
y – axis is of length
of the straight line
A.y = 1 x + 5 B.y = x + 5 C. 3 y = -x + 5 3 A. 5/3 B. 5/2 C. – 5/3 D. – 5/2
10 Solution
D 3y = x + 5 3 First we rearrange the given equation y = mx + c
Solution We are to find C
y 2x – 3y – 5 = 0
5 2x – 5 = 3y
y = 2x – 5
3 3
30 0
The intercept at the y-axis is – 5/3 ( C )
x
2005/10 PCE
From the sketch we have a negative gradient
The equation of the straight line which makes an intercept of
y = – mx + c
–2 with the y-axis and an intercept of 1 with the x-axis is
Here tan  is tan 300 = 1 and c is 5,
A. 3x + 2y – 6 = 0 B. 3x – 2y + 6 = 0
3
C. 2x + y – 2 = 0 D. 2x – y – 2 = 0
y=– 1x + 5
Solution
3
General form of a straight line is y = mx + c ; where m is
Multiplying through by √ 3
gradient and c is the intercept on the y-axis
The given question only has c but no m. Though we can
3 y = –x + 5 3 ( C)
simulate the two points needed in the formula
m = y2 – y1
1997/34 PCE x2 – x1
What is the y – intercept and the gradient of the line
At intercept –2 with the y-axis
whose equation is x + y = 1 ?
x = 0 and y = -2 i.e ( 0, –2 )
5 3 3
At intercept 1 with the x-axis
A. 1/3 , 3
/5 B.1, – 3/5 C. –1/3 , – 3/5 D. –1, 3/5 x = 1 and y = 0 i.e ( 1, 0 )
Solution
First, we rearrange the given equation into y = mx + c Thus, m = – 2 – 0
format 0–1
To clear fractions, multiply through by the Lcm of 5 = –2 = 2
and 3 i.e 15 –1
15 × x + y × 15 = 1 × 15 The required equation is with c = – 2 given earlier and m = 2
5 3 3 i.e y = 2x – 2
3x + 5y = 5 Rearranging 2x – y – 2 = 0 ( D )
5y = – 3x + 5
y=–3x+1
5 2004/7 Neco (Dec)
y – intercept is + 1 (a) Find the gradient of the straight line 7x + 4y + 2 = 0
Gradient is – 3/5 ( B ) (b) Determine the intercepts on the axes
461
Solution Equation of a straight line through
The straight line equation form is y = mx + c x1, y1 with gradient m is
where m is gradient y – y1 = m ( x – x1 ) OR
Making the coefficient of y unity
7x + 4y + 2 = 0 becomes m = y – y1
4y = – 7x – 2 x – x1
7 1
y = – x – 1992/35 PCE
4 2
Find the equation of a straight line of gradient 2 through the
7
Thus gradient is – point ( 1, 4 )
4 A.y – x + 2 = 0 B.y – 2x + 4 = 0 C.y + 2x + 4 = 0
(b) Intercepts on the axes D.y – 2x – 2 = 0
Intercepts at y – axis, x = 0 Solution
7x + 4y + 2 = 0 becomes Applying the formula
0 + 4y + 2 = 0 m = y – y1
4y = – 2 x – x1
1 2=y– 4
y=–
2 x–1
Intercepts at x – axis, y = 0 cross multiplying
7x + 4y + 2 = 0 becomes 2( x – 1 ) = y – 4
7x + 0 + 2 = 0 2x – 2 = y – 4
7x = – 2 0 = y – 4 + 2 – 2x
2 i.e. y – 2x – 2 = 0 (D)
x=–
7
Alternatively 1996/ 5 (Nov)
x y The equation of a straight line with gradient 1/3 which
Intercept form of straight line is   1 passes through the point (1, 2) is
a b
A 3x – y – 1 = 0 B x – 3y + 5 = 0
Hence we transform 7x + 4y + 2 = 0 to it
C 2x – 3y + 1= 0 D 3x – 2y + 1 = 0 E 2x + 3y + 8 = 0
7x + 4y = – 2
Solution
Our target will be to make LHS unity
Equation of a line through Point (x1, y1) with gradient m is
Here we divide through by – 2
 4  2 y  y1
7x
  y  m
2  2 2 x  x1
7 1 y2
 x  2y  1 
2 3 x 1
7 x – 1 = 3y – 6
x – Intercept  , y – intercept = – 2 x – 3y + 5 = 0 (B)
2

2003/40 (Nov) 2011/25 Neco


Find the equation of the straight line which makes Obtain an equation of a straight line through (4, 3)
intercept of 2 and – 3 on the x and y axes respectively if the slope is 2
A 3x – 2y – 6 = 0 B 3x – 2y + 6 = 0 A y = 5x + 2 B y = 2x + 5 C y = 5x – 2
C 3x – 2y – 5 = 0 D 3x – 2y + 5 = 0 D y = 2x – 5 E y=x–5
Solution Solution
x y Equation of a line through Point (x1, y1) with gradient m is
Intercept form of straight line is   1 y  y1
a b m
where a is x – axis intercept b is y – axis intercept x  x1
x y y 3
  1 2
2 3 x4
To clear fraction multiply through by LCM 6 2x – 8 = y – 3
3x – 2y = 6 y = 2x – 5 (D)
3x – 2y – 6 = 0 (A)

462
Equation of a line passing through two points Solution
m = y – y1 Equation of a straight line using two points
x – x1 (x1, y1) and (x2, y2) has formula as
But m = y2 – y1 y  y1 y 2  y1

x2 – x1 x  x1 x2  x1
Thus y 1 4 1

x 8 38
y2 – y1 = y – y1
y 1 3
y2 – x1 x – x1 
x  8  11
–11(y – 1) = 3(x – 8)
1994/36 UME
y
–11y + 11 = 3x – 24 Thus 11y + 3x – 35 = 0 (D)
1997/ 6
(0, 4 ) Find the equation of the line passing through P(1, 3) and Q(3, 6)
A 2x + 3y + 3 = 0 B 2x – 3y – 3 = 0
C 3x – 2y + 3 = 0 D3x – 2y + 1 = 0 E 2x – 3y + 3 = 0
(3, 0 )
Solution
x
Equation of a straight line using two points
(0, 0 ) (x1, y1) and (x2, y2) has formula as
y  y1 y 2  y1

The equation of the line in the graph above is x  x1 x2  x1
A.3y = 4x + 12 B.3y = 3x + 12 C.3y = - 4x + 12 y 3 63
D.3y = - 4x + 9 
x 1 3 1
Solution
Equation of line through two points is given as y 3 3

y – y1 = y2 – y1 x 1 2
x – x1 x2 – x1 2(y – 3) = 3(x – 1)
Here ( x1, y1 ) is ( 0 ,4 ) and ( x2, y2 ) is ( 3 ,0 ) 2y – 6 = 3x – 3
3x – 2y + 3 = 0 (C)
y– 4 = 0 – 4 2000/27 Neco
x– 0 3 – 0 Find the coordinates of the point where the line 2y = 3x – 7 meet
y–4 = – 4 the line joining the points (4, –2) and (–1, 3)
x 3 A 11 1 B 11 1 C 11 2 D 11 3 E 11 2
, , , , ,
3( y – 4 ) = – 4x 5 5 5 5 5 5 5 5 5 5
3y – 12 = – 4x Solution
3y = – 4x + 12 ( C ) First, we get the equation of the straight line joining
1995/33 UME (4, –2) and (–1, 3) by
What is the equation of the line which makes intercept y  y1 y 2  y1

of 2 and 3 on the x and y axes respectively? x  x1 x2  x1
A.2x + 3y = 1 B.3x + 2y = 1 C.3x + 2y = 6 y  (2) 3  (2)

D.3x – 2y = 6 x4 1 4
Solution y2 5
Intercept at x is 2 then y = 0 i.e ( 2, 0 ) 
x 4 5
Intercept at y is 3 then x = 0 i.e ( 0, 3 ) y2 1
 
Equation of line given two points; x4 1
y – y1 = y2 – y1 y + 2 = – 1(x – 4)
x – x1 x 2 – x1 y+2 = – x+4
Here ( x1 , y1) ( 2, 0 ) and ( x2, y2 ) is ( 0 ,3 ) y+x–2=0
Next, the coordinates of their meeting point is their intersection
y–0 = 3–0 coordinates. Solving the two equation simultaneously
x–2 0–2 y + x – 2 = 0 is y + x = 2 ------ (a)
y = 3 2y = 3x – 7 is 2y – 3x = – 7------ (b)
x–2 –2 (a) × 2 and subtract
2y + 2x = 4
– 2y = 3( x – 2 ) – ( 2y – 3x = – 7 )
– 2y = 3x – 6 5x = 11
rearranged as 3x + 2y = 6 ( C ) x = 11/5
2005 /33 substitute x = 11/5 into (a)
Find the equation of the straight line joining y + x = 2 becomes
the points (8, 1) and (–3, 4) y + 11/5 = 2
A 3x – 11y – 35 = 0 B 3x – 11y + 35 = 0 y = 2 – 11/5 = – 1/5
C 11x + 3y – 35 = 0 D 3x + 11y – 35 = 0 coordinates is 11 ,  1 (B)
5 5
463
1991/26 UME
Find the gradient of the line passing through the points
(– 2, 0 ) and ( 0, – 4 )
A. 2 B. – 4 C. –2 D. 4
Solution
Gradient = y2 – y1
x 2 – x1
Gradient Here y2 = – 4, y1 = 0; x2 = 0, x1 = –2
y Substituting,
y
Gradient = – 4 – 0
0 – (– 2 )
= –4 i.e – 2 ( C )
2
x
2002/33 PCE
x
The gradient of the line joining ( 3, y ) and (–1, –2 )
Acute angle with x – axis Obtuse angle with x – axis is 3/2. Find the value of y
gradient is positive gradient is negative A. – 4 B. –3 C. 3 D. 4
Solution
There are basically two types of gradients namely: Gradient = y2 – y1
Positive and Negative gradients x2 – x1
Gradient (slope) = Change in y Here: y2 = – 2, y1 = y
Change in x x 2 = – 1, x1 = 3 and Gradient = 3/2
Substituting
Positive gradient Negative gradient 3 = – 2– y
Increase in y Decrease in y 2 – 1– 3
Increase in x Increase in x Cross multiplying,
3(– 4 ) = 2 (– 2 – y )
In physics it is called - In physics it is – 12 = – 4 – 2y
called - – 12 + 4 = – 2y
acceleration deceleration – 8 = – 2y
since y increases as since y increases as – 8 = – 2y
x increases x decreases –2 –2 4= y (D)
1994/38 PCE Exercise 39.6
y-axis starts at the down part y-axis starts at the upper part What is the gradient of the line joining the points with
and ends at the top and ends at the down part coordinates ( 5, – 1 ) and (– 3, 7 ) ?
A. 3 B. 1/3 C. – ¾ D. –1

( x2, y2 ) ( x1, y1 )

( x1, y1 ) (x 2 ,y2)

Gradient = y2 – y1
x2 – x1
From the above table note that : y2 , y1 and x1 , x2
positions differs for positive and negative gradients

Gradient of a Curve
The gradient of any point on a curve is the gradient of the
tangent to the curve at that point. The tangent must be
produced at equidistant to the point

Gradient / slope of a straight line


is given as y2 – y1
x 2 – x1

464
Angle of slope But gradient of a line joining two point (x1,y1) and (x2, y2)
y y 2  y1
T m 
x 2  x1
V
8  (3) 11
= =
64 2
From gradient = tan 
11
x = tan 
U W 2
S  = tan – 1 5.5
 is the angle of slop which the line ST makes with the = 79.70 (E)
positive x – axis
VW
Gradient m =  tan  2004/ 16 Neco
UW Find the angle of the slope of a line if the gradient is 3/5
A 30.900 B 30.960 C 59.040 D 71.570 E 78.690
2000/ 12 (Nov) Solution
A straight line passes through (–1, 2) and makes angle Gradient = tan 
450 with the negative direction of the x – axis, find its This is the formula that connect gradient with its angle
equation 3 = tan 
A x+y–1=0 B x–y–3=0 C x+y–3=0 5
D x+y+1=0  = tan – 1 0.6
Solution = 30.960 (B)
The equation of a straight line through (x1, y1) with gradient m is 1993/ 5 (Nov)
y  y1 Find the value of the acute angle between the lines
m 
x  x1 3x – y + 1 = 0 and x – 2y + 1 = 0
Our gradient m can be gotten by A 600 B 450 C 300 D 200 E 150
tan  = m Solution
 Here 450 is angle made with negative direction of m2  m1
x – axis, thus angle made with positive direction of Acute angle between two lines is tan  = where
1  m2 m1
x – axis is 180 – 45 = 1350
tan 1350 = m m2 and m1 are gradient for the two lines respectively.
–1 = m To get the gradient, we transform the given lines to intercept
Substituting for x1, y1 from y = mc + c
y  y1 3x – y + 1 = 0 becomes y = 3x = 1 and m1 = 3
m  becomes 1 1 1
x  x1 x– 2y + 1 = 0 becomes y = x + and m2 =
y2 2 2 2
–1= 1 3
x  ( 1) 2
tan  =
–1( x + 1) = y – 2 1 1
23
–x–1=y–2 5 5
y + x – 1 = 0 (A) = 
2 2

2002/6 Neco tan  = –1


The angle of slope of a given straight line is 1500. tan  = 1
What is the gradient of the line, correct to one  = tan– 1 1
decimal place? = 450 (B)
A – 1.5 B – 1.3 C – 0.8 D – 0.6 E 0.6
Solution
The formula connecting gradient and its angle is
Gradient = tan 
Gradient = tan 1500
= – 0.5774
 – 0.6 to 1d.p (D)

2002/ 5 Neco
What is the angle of slope of the line joining points
(4, –3) and (6, 8)?
A 1200 B 1100 C 850 D 830 E 79.70
Solution
The formula connecting gradient and its angle is
Gradient = tan 
465
Gradient and parallel condition Substituting for x = –1 and y = 2
2000/3 Neco y + x = k becomes
Find the equation of the line passing through the ( 2 ) + (–1) = k
point (– 2, – 3) and parallel to line y – 2x + 4 = 0
A y – 2x – 1 = 0 B y + 2x – 1= 0 C y + 2x + 1 = 0 1 = k
D 2y – x + 1 = 0 E 2y + x – 1 = 0 The line equation is y + x = 1 (E)
Solution
The equation of the line can be gotten from (x1, y1) and 2005 /10c (Nov)
gradient M Two straight lines are 3x +2y–5 = 0 and x –2y –7 = 0. Find
y  y1 equation of the line through their point of intersection and
m
x  x1 parallel to the line x + 2y – 5 = 0
Since both lines are parallel they have equal gradient Solution
From y = 2x – 4, gradient m is 2 First is the coordinate of intersection between
3x + 2y – 5 = 0 i.e 3x + 2y = 5
y  (3) and x – 2y –7 = 0 i.e + ( x – 2y = 7 )
Thus , equation of line 2 = 4x = 12
x  (2)
x=3
y3
2 = Substitutes x value into 3x + 2y = 5
x2 3(3) + 2y = 5
2(x + 2) = y + 3 2y = 5 – 9
2x + 4 = y + 3 4
y – 2x – 1 = 0 (A) y= = –2
2
Alternative method Coordinate of intersection is (3, –2)
The line equation parallel to Next, we find the gradient of line x + 2y – 5 = 0
y – 2x + 4 = 0 is by expressing it in y = mx + c formula
y – 2x + k = 0 at (– 2, – 3) 2y = – x + 5
Substituting for x = –2 and y = –3 1 5
y=  x +
y – 2x + k = 0 becomes 2 2
1
(– 3 ) – 2(–2) + k = 0 Thus gradient m is 
2
–3+ 4+ k = 0
Our require line equation is parallel to x + 2y – 5 = 0 hence
k = – 1 same gradient
The line equation is y – 2x – 1 = 0 (A) Thus with gradient m and coordinate (3, –2)
y  y1
The line equation is m 
2000/7 Neco x  x1
Find the equation of the line passing through the 1 y  (2)
point (–1, 2) and is parallel to y + x = 5  =
2 x 3
A y + 2x = 5 B y+x=4 Cy–x=2
1 y2
D y – 3x = 0 E y+x=1  =
Solution 2 x 3
The equation of the line can be gotten from (x1, y1) and gradient M – 1(x – 3) = 2(y + 2)
by – x + 3 = 2y + 4
y  y1 2y + x + 1 = 0
m
x  x1
Since both lines are parallel they have equal gradient
From y + x = 5  y = – x + 5, gradient m is – 1
y 2
Thus, equation of line – 1 =
x  (1)
y2
–1=
x 1
– 1(x + 1) = y – 2
–x–1=y–2
y+x–1=0
i.e y + x = 1 (E)
Alternative method
The line equation parallel to
y + x = 5 is
y + x = k at (– 1, 2)
466
Gradient and perpendicular condition Equation of the perpendicular bisector
2000/8 y  y1
= m or y – y1 = m(x – x1)
A straight line 2x + 5y + 12 = 0 passes through the x  x1
point (k, –2) and is perpendicular to 5x – 2y + 3 = 0. 3 1 5
Find the value of k. y– =  x  
2 13  2
A – 11 B 8 5 C 7 5 D 1
3 1 5
Solution y– =  x 
2 13 26
First we express 2x + 5y +12 = 0 which pass through 26y – 39 = – 2x + 5
the point (k, –2) in intercept from y = mx + c 26y + 2x – 44 = 0
2x + 5y +12 = 0 becomes 13y + x – 22 = 0 (E)
5y = – 2x – 12
2 12 1994/14 (Nov)
y=– x –
5 5 Find the equation of the line perpendicular to
2 2y + 3x – 4 = 0 and passes through the point (2, –5)
m= – A 2x – 3y + 11 = 0 B 3x – 2y –16 = 0
5
Thus with gradient m and coordinate (k, –2) C x – 3y – 17 = 0 D 3x + 2y + 4 = 0
y  y1 E 2x – 3y – 19 = 0
The line equation is m  Solution
x  x1
y  y1
2 y  ( 2 ) To find the equation of the line m  , we need
– = x  x1
5 xk
gradient in addition to the point (2, –5) i.e (x1, y1)
– 2(x – k) = 5(y + 2)
– 2x + 2k = 5y + 10
First, we transform 2y +3x – 4 = 0 to y = mx + c form
5y + 2x + 10 – 2k = 0
2y = – 3x + 4
Comparing with 2x + 5y + 12 = 0
3
10 – 2k = 12 y = – x + 2
– 2k = 12 – 10 2
– 2k = 2 3
Gradient m = –
k = –1 2
1992 /9 (Nov)
Find the equation of the perpendicular bisector of the The line that is perpendicular to 2y + 3x – 4 = 0 ;
line joining the points (3, 8) and (2, –5) their gradient m1 m2 = –1
A 13x – y – 31 = 0 B x + 3y – 7 = 0 C 3x – y – 6 = 0 3
D x + 13y – 37 = 0 E x + 13y – 22 = 0 – m2 = –1
2
Solution 2
The simplest items needed for getting the equation of m2 =
the bisector here is gradient m and (x1, y1) points, which 3
is 2 y  (5)
The required equation is: =
y  y1 3 x2
m 
x  x1 2 y5
=
First gradient of the line joining (3, 8) and (2, –5) 3 x2
y 2  y1 2x – 4 = 3y + 15
m=
x 2  x1 2x – 3y – 19 = 0 (E)
58 2002/ 12
= = 13 Find the equation of the straight line which makes an
23 intercept of 2 on the y – axis and is perpendicular to the line
Thus, the gradient of the perpendicular bisector 3x – y + 7 = 0
1 A x + 3y + 6 = 0 B 3x – y – 6 = 0
m= 
13 C x + 3y – 6 = 0 D 3x – y + 6 = 0
At the point of bisecting the line is midpoint Solution
Thus, midpoint of (3, 8) and (2, –5) To get the equation of the straight line here, we use
x x y  y2 y = mx + c by employing intercept and gradient data from
= 1 2, 1 perpendicularity.
2 2
3 2 85 First, we express 3x – y + 7 = 0 in y = mx + c form
= , to get gradient m.
2 2 y = 3x + 7 ; m is 3
5 3
=  , 
2 2 The line that is perpendicular to 3x – y + 7 = 0;
We have gotten the need items gradient and (x1, y1) their gradient m1 m2 = –1
3 m2 = –1
467
1 2003 /39 Neco
m2 = – Find the value of k for which lines 3y = 4x – 1 and
3
The new equation is y = mx + c ky = x + 3 are perpendicular to each other
c is given as 2 in question 4 3 2 3 4
A – B – C D E
1 3 4 3 4 3
y= – x +2 Solution
3
3y = – x + 6 To find k, we use the property of perpendicularity.
3y + x – 6 = 0 (C) If two lines are perpendicular then their gradient
m1m2 = – 1
2007/ 7 Neco Transforming the lines equation to y = mx + c form
Find the equation of the straight line which passes 3y = 4x – 1 and ky = x + 3 becomes
through the point (–2, 7) and perpendicular to the line 4 1 1 3
y = x  and y = x 
– 4y + 2x = – 3 3 3 k k
A y + 2x +3 = 0 B y + 2x – 3 = 0 4 1
C y + 2x – 4 = 0 D y – 2x – 4 = 0 E y – 2x – 3 m1 = m2 =
3 k
=0 From m1m2 = – 1
Solution 41
y  y1  =–1
To find the equation of the line m  , we need 3k 
x  x1
4 = –3k
gradient in addition to the point (–2, 7) i.e (x1, y1) 4
First, we transform – 4y + 2x = – 3 to y = mx + c form k = – (A)
– 4y = – 2x – 3 3
2004/ 7 Neco
1 3
y= x  Find the equation of the straight line passing through the point (3,
2 4 4) and perpendicular to the straight line
1 4x – 3y = 10
Gradient m = A 4y + 3x = 25 B 4y – 3x = 22 C 4y –3x = 16
2
D – 4y + 3x = 22 E 4y – 3x = – 16
The line that is perpendicular to – 4y + 2x = – 3;
Solution
their gradient m1 m2 = –1 To find the equation of the line, we need gradient m in addition to
1 the point (3, 4) i.e (x1, y1) for use in
m2 = –1
2 y  y1
m 
m2 = 2 x  x1
y  7 The gradient can be gotten from 4x – 3y = 10 by transforming it to
The required equation is – 2 =
x  (2) y = mx + c form
–2(x + 2) = y – 7 3y = 4x – 10
– 2x – 4 = y – 7 4 10
y= x 
y + 2x – 3 = 0 (B) 3 3
4
2003/6 (Nov) m=
3
The line 9x – ky + 7 = 0, is perpendicular to the line The line that is perpendicular to 4x – 3y = 10;
2x – 3y – 3 = 0. Find the value of the constant k. their gradient m1 m2 = –1
27 2 4
A – B –6 C D6 m2 = –1
2 27 3
Solution 3
To find k, we use the property of perpendicularity. m2 = –
If two lines are perpendicular then their gradient 4
m1m2 = – 1 3 y4
The required equation is  =
Transforming the lines equation to y = mx + c form 4 x 3
ky = 9x + 7 and 3y = 2x – 3 – 3(x – 3) = 4(y – 4)
9 7 2 – 3x + 9 = 4y – 16
y = x  and y = x – 1 4y + 3x = 25 A.
k k 3
9 2 1998/11aiii (Nov) Exercise 39.7
m1 = M2 = Given the point P(11, 3), Q(–1, 12) and R(1, –2)
k 3 find : (i) the equation of the line PQ
From m1m2 = – 1 (ii) the equation of the line through R perpendicular to PQ
92
  = –1 1996/ 12 Exercise 39.8
k 3 Find the equation of a straight line through the point
6= –k (–2, 1) and perpendicular to the line – 3x + 5y + 2 = 0
Thus, k = – 6 (B) A 5x + 3y = – 7 B 5x + 3y = – 13 C 3y – 5x = 13
D 5y – 3x = 11 E 3y + 5x = 13
468
y
1999/ 6 (Nov) Exercise 39.9
Find the equation of the line which passes through the
point (–4, 1) and perpendicular to the line 2y + x +7 = 0
A 2x– y + 9 = 0 B 2x + y – 13 = 0
C x + 2y – 2 = 0 D x – 2y + 6 = 0
0
60 4
Perpendicular form
x
The equation of a line ab, where the length of the
perpendicular distant P from the origin and the
perpendicular makes an angle  with the positive Equation of the line is P = x cos + y sin
x – axis is given as Here  is 900 – 60 = 300, P is 4units
P = x cos + y sin Substituting: 4 = x cos30 + y sin30
c 4 =
3
x 
1
y
where P = 2 2
a2  b2 Multiply through by 2 clear fractions
from ax + by + c = 0 general form of straight line
y 8= 3x + y

b y+ 3x=8

2011/11a Exercise 39.10


Find the equation of the line:
P (i) that is parallel to 4x + 2y = 1 and passing through (4, 2)
(ii) Whose perpendicular distance from the origin is of
a
x length 3 units and at 300 from the x – axis

P = x cos + y sin
Example 1
Find the equation of the line whose perpendicular
distant from the origin of length 4units and at 600 from Chord, tangent and normal
the positive x – axis
Circle concept of chord, tangent and normal
Solution
Sketch
y

P
A B

A B

4 AB A B

60 0
x Q
AB is a chord Normal PQ is a straight line

Equation of the line is P = x cos + y sin


A1 B 1 is a chord perpendicular to the tangent

Here  is 600, P is 4units A11 B 11 is a tangent and touches it at the point of


Tangent is a line which contact.(PQ is a Normal to
Substituting: 4 = x cos60 + y sin60 11 11
touches a curve in two occidental the tangent A B )
1 3 points. (In actual sense it touches
4= x  y
2 2 the curve at one and only one point)
Multiply through by 2 clear fractions
Curve concept of tangent and normal
8= x + 3y P

x+ 3y=8
Example 2
Find the equation of the line whose perpendicular
distant from the origin of length 4units and at 600 from A B
the positive y – axis
Solution Q

Sketch We were informed earlier that the gradient/ slope of a


function f(x) at any point on the curve is the slope of the
tangent at that point
469
dy dy
Gradient m = We are given (x1, y1) next we find m i.e.
dx dx
The equation of tangent is x2 + x – 4y – 4 = 0 becomes
y – y1 = m(x – x1) 2 xdy dx 4dy
where (x1, y1) is the point of contact. + – – 0 = 0
dx dx dx
A normal to the tangent at that point is perpendicular. 4dy
2x + 1 – = 0
Gradient and perpendicularity property in coordinate dx
geometry is that m1 m2 = – 1 4dy
Tangent Normal = 2x + 1
1
dx
Thus m2 =  dy 1
m1 = (2x + 1)
The equation of normal in relation to the tangent at the dx 4
Gradient m at (2, 3) is put x = 2 into
point of contact (x1, y1) is
y – y1 = m2(x – x1 ) 1 1
(2x + 1) = [2(2) + 1]
1 4 4
where m2 =  and m1 is the gradient of the tangent at (x1, y1)
m1 1 5
= (4 + 1) =
4 4
Problems on tangent Substitute m and (x1, y1) value into
y – y1 = m (x – x1)
2010/16 Neco
Find the equation of the tangent to the curve 5
y–3= (x – 2)
y = (2 – x)(3 + x) at the point where x = 3. 4
A y – 7x = 15 B y – 2x = 1 C 7y – x = 15 4y – 12 = 5x – 10
D y + 7x = 15 E y + 7x = 27 4y – 5x – 2 = 0 (A).
Solution
The equation of tangent to any curve is always at a 2009/ 3
The line y = mx – 3 is a tangent to the curve
point y = 1 – 3x + 2x3 at the point (1, 0). Find the value of the constant
(x1, y1) and not x1 value alone. m.
Equation of tangent is A –4 B–1 C 3 D 4
y – y1 = m (x – x1 ) Solution
dy dy
For us to proceed we need y1 value and gradient m i.e . We are given (x1, y1) next we find m i.e.
dx dx
Put x = 3 into the original equation M is the gradient at the point (1, 0)
y = (2 – x)(3 + x) becomes y = 1 – 3x + 2x3
y = (2 – 3) (3 + 3) dy
= (–1) (6) Gradient m = – 3 + 6x2
dx
= – 6 i.e y1 At (1, 0) m = – 3 + 6(1)2
From y = (2 – x)(3 + x) = 3 (C).
dy 2007/ 12 – 13
= (2 – x)  1 + (–1) (3 + x) The tangent to the curve y = 4x3 + kx2 – 6x + 4 at the
dx
= 2– x – 3 – x point P(1, m) is parallel to the x – axis, where k and m are
constant. Use the information to answer questions 12 and 13.
= – 1 – 2x
12. Find the value of K
A 3 B 2 C–2 D –3
At the point (3, – 6), Tangent gradient m = –1 – 2(3)
Solution
=–1–6
dy
= –7 Tangent at P(1, m) parallel to x – axis implies =0
Equation of tangent y – y1 = m (x – x1 ) dx
y – (–6) = – 7(x – 3) y = 4x3 + kx2 – 6x + 4 becomes
y + 6 = – 7x + 21 dy
= 12x2 + 2kx – 6
y = –7x + 15 (D). dx
dy
But = 0 at point P (1, m)
2010 /13 Neco dx
Find the equation of the tangent at (2, 3) on the 12x2 + 2kx – 6 = 0 at point P (1, m) i.e at x = 1
parabola x2 + x – 4y – 4 = 0 12(1)2 + 2k(1) – 6 = 0
A 4y – 5x – 2 = 0 B 4y – 5x – 12 = 0 12 + 2k – 6 = 0
C 4y + 5x + 2 = 0 D 4y + 5x – 2 = 0 2k = – 6
E 8y – 10x – 2 = 0 k=–3 (D)
Solution
The equation of a tangent to a curve at (x1, y1) is 13. Determine the coordinates of p
y – y1 = m (x – x1 ) A (1, 2) B (1, 1) C (1, –1) D (1, – 2)

470
Solution But we are inform that 12x2 = 108
Coordinates of p is the x and y value of the point p(1, m) x2 = 9
on the curve y = 4x3 + kx2 – 6x + 4
x=3
Substituting for our k value of – 3
y = 4x3 + kx2 – 6x + 4 becomes
Next we substitute for x value into y = 4x 3
y = 4x3 – 3x2 – 6x + 4 Where x = +3; y = 4 (3)3 i.e 108
Next, we substitute for x value from (1, m) Where x = – 3; y = 4 (– 3)3 i.e – 108
y = 4(1)3 – 3(1)2 – 6(1) + 4 P (3, 108) and Q(–3, –108)
=4–3–6+4
=–1 2006/4 June
Thus p(1, m) becomes p(1, – 1) (C) Find the equation of the tangent to the curve
2007/11b x 1 1
The equation of a curve is x(y2 + 1) – y(x2 + 1) + 4 = 0 y= , x   at the point (1, 0)
2x  1 2
Find the (i) gradient of the curve at any point (x, y) Solution
(ii) equation of the tangent to the curve at the point (–1, –3)
Solution
Equation of a tangent to a curve at point (x1, y1) is
y – y1 = m(x – x1)
dy
(i) Gradient of the curve at any point (x, y) is the dy
dx Here (x1, y1) is (1, 0), next, we find m, i.e.
Simplifying dx
x(y2 + 1) – y (x2 + 1) + 4 = 0 becomes x 1
y =
xy2 + x – x2y – y + 4 = 0 implicit function 2x  1
d(y2 ) dy dx  dy d ( x 2 )  dy
dy (2 x  1)  1  ( x  1)  2
x  y2   x 2  y   00 By quotient rule =
dx dx dx  dx dx  dx dx (2 x  1) 2
2x  1  2x  2
dy dy dy =
2 xy  y 2  1  x 2  2 xy   0 (2 x  1) 2
dx dx dx
3
Collect terms in dy together =
dx (2 x  1) 2
dy dy dy To get gradient m at (1, 0) put x = 1 into
2 xy  x2   2 xy  y 2  1
dx dx dx 3 3 1
= 2 i.e.
dy (2 x  1) 2
3 3
(2xy – x2 – 1) = 2xy – y2 –1
dx Substituting m and (x1, y1) value into
dy 2 xy  y 2  1 y – y1 = m(x – x1) becomes
= 1
dx 2 xy  x 2  1 y – 0 = (x – 1)
(ii) The equation of the tangent to the curve at the 3
point (–1, –3) is 1
y = (x – 1) or 3y – x + 1 = 0
y – y1 = m(x – x1) 3
We are given (x1, y1) but m at (–1, –3) is gotten by 2003/21 (Nov)
dy The equation of the tangent to the curve
substituting them into y = 2x3 – x2 + 1 at the point (–1, –2) is
dx
A 4x – y + 2 = 0 B 8x – y + 6 = 0
2(-1) (-3) - (-3) 2  1 C x – 8y + 15 = 0 D 28x – y + 26 = 0
m at (–1, –3) =
2(1)( 3)  (1) 2  1 Solution
6  9 1 Equation of tangent to a curve at (x1, y1) is
= i.e. –1 y – y1 = m(x – x1)
6 11 dy
Thus equation of tangent at (–1, –3) is Here (x1, y1) is (–1, –2), next, we find m i.e
y – y1 = m(x – x1) dx
y – (–3) = m[x – (–1) y = 2x3 – x2 + 1 becomes
substituting for m = –1 dy
= 6x 2 – 2x
y +3 = –1 (x +1) dx
y+3=–x–1 To get gradient m at (–1, –2), put x = – 1 into
y+x= –4 6x2 – 2x = 6(–1)2 –2(–1)
2006/10a counter example = 6 +2 = 8
The gradient of the tangent to the curve y = 4x 3 at point Substituting for m and (x1, y1) into
P and Q is 108. Find the coordinates of P and Q y – y1 = m(x – x1) becomes
Solution y – (–2) = 8[x – (–1)]
If the curve y = 4x 3 y + 2 = 8(x + 1)
dy y + 2 = 8x + 8
Then gradient = 12x2 y – 8x – 6 = 0 (B) Same as 8x – y + 6 = 0
dx
471
2002/4(Nov) counter example Solution
The tangent to the curve y = 2x 2 – 3x + 5, at point Equation of a tangent to a curve at point(x1, y1) is
m(x, y), makes angle 1350 with the positive direction of y – y1 = m(x, – x1)
the x –axis. Find the coordinate of m dy
Here (x1, y1) is (–1, 2), next, we find m i.e
A (  1 2 , 7) B (1, 4) C ( 1 2 , 4) D (1, 10) dx
Solution x3 – 2y – 8x – 3 = 0 implicit function
Coordinate of m (x, y) dx dy dx
3x 2  2  8  0  0
We first, get x, by finding gradient dx dx dx
y = 2x2 – 3x + 5 dy
3x2 – 2 –8= 0
dy dx
Gradient = 4x – 3
dx dy
Also gradient of tangent at m makes 1350 3x2 – 8 = 2
dx
Recall from coordinate geometry dy
gradient = tan  = 1 (3x2 – 8)
dx 2
= tan 1350 To get gradient m at (–1, 2), put x = – 1 into
= – tan 450 (tan is negative 2nd quad) 1 (3x2 – 8) = 1 [3 (– 1)2 – 8]
=–1 2 2
It follows that 4x – 3 = –1 = 1 (3 – 8)
4x = 2 2
x= 1 = –5
2 2
To get y value put x = 1/2 into y = 2x2 – 3x + 5 Substituting m and (x1, y1) value into
y = 2( 1 )2 – 3( 1 ) + 5 y – y1 = m (x, – x1) becomes
2 2 y – 2 = – 5 [x – (–1)]
= 1 – 3 + 5 2
2 2 2y – 4 = –5x – 5
= 4 2y + 5x + 1 = 0 (A)
Coordinate of m (x, y) is ( 1/2 , 4) C.
2001/11 Neco Counter example
The equation of the tangent to the parabola 1994/3 Counter example
y2 = 4(x +2) at the point (2, –2) is A tangent to the curve y = 2x2 – x + 1 at the point p (x, y)
A y–x–4=0 B y–x=0 C y–x+4=0 is parallel to the line y = 3x + 4. Find the:
D y–x–4=0 E y+x=0 (a) coordinates of p ; (b) equation of the tangent at p.
Solution Solution
Equation of a tangent to a curve at point (x1, y1) is Coordinates of p(x, y)
y – y1 = m(x – x1) Tangent at p is parallel to y = 3x + 4
dy that implies equal gradient
(x1, y1) is (2, – 2), next we find m i.e From y = 2x2 – x + 1
dx dy
2
y = 4x + 8 implicit function Gradient: = 4x – 1
dy dx
2y = 4 Also from y = 3x + 4
dx dy
dy 4 Gradient = 3
= dx
dx 2y
Since m1 = m2 (parallel line)
To get gradient m at (2, –2) we substitute for y = – 2 3 = 4x – 1
4 4 4 = 4x
=
2y 2(2) x=1
=–1 Coordinates of P, so we substitute x = 1 into
Substituting m and (x1, y1) value into the equation y = 2x2 – x + 1 becomes
y – y1 = m(x – x1) becomes = 2(1) 2 – 1 + 1
y – (–2) = –1(x – 2) =2
y+2=–x+2 P ( 1, 2)
y + x = 0 (E) (b) Equation of tangent at p(x1, y1) is
y – y1 = m(x – x1)
Substituting for P( 1, 2) and m = 3
2000/ 13 (Nov) y – 2 = 3( x – 1)
Find the equation of the tangent to the curve y – 2 = 3x – 3
x3 – 2y – 8x – 3 = 0 at the point (–1, 2). y – 3x + 1 = 0
A 5x + 2y +1 = 0 B 2x + y + 3 = 0
C 2x + 5y – 8 = 0 D 5x + 2y – 9 = 0
472
Thus we need x1, y1 and m2
Where the curve cuts the y – axis, x = 0 and coordinate of y there
is gotten by putting x = 0 into original equation
y = x2 – 4x – 12 becomes
y = (0)2 – 4(0) – 12
y = – 12
Thus (x1, y1) is (0, – 12)
dy
Next, Gradient (m1) i.e of the tangent at (0, – 12)
dx
y = x2 – 4x – 12
dy
= 2x – 4
dx
At (0, – 12) , m1 is gotten by putting x = 0
2x – 4 = 2(0) – 4
=–4
Equation of normal is y – (–12) = –   1  (x – 0)
 4

Problems on Normal 1
y + 12 = x
2007/3 counter e.g 4
The normal to the curve y = 2x 2 + x – 3 at the point Multiply through by 4
(2, 7) meet the x – axis at point p. Find the coordinates 4y – x + 48 = 0
of p. (b) At the point where the normal meet (cut) the x –axis,
Solution y=0
First, we get the equation of the normal at ( x1, y1) Next, we put y = 0 into normal equation
y – y1 = m2 (x – x1) 4y – x + 48 = 0 becomes
1 4(0) – x + 48 = 0
where m2 =  and
m1 x = 48 The point is (48, 0)
m1 is gradient of the tangent at (x1 , y1)
dy
2010/ 3
Gradient (m1) i.e of the tangent at (2, 7) The equation of curve is y = x(3 – x2)
dx Find the equation of its normal at the point where x = 2.
y = 2x2 + x – 3 Solution
dy Equation of normal on curve at point (x1, y1) is
= 4x + 1
dx y – y1 = m2 (x – x1)
At (2, 7), x = 2 thus m1 is 4x + 1 = 4 (2) + 1 1
where m2 =  and
=9 m1
1 m1 is gradient of tangent at that point
Thus y – 7 = (x – 2)
9 dy
9y – 63 = – x + 2 Gradient (m1) i.e of the tangent at (x1, y1)
dx
9y + x – 65 = 0 y = x(3 – x2)
Where the normal meets the x – axis, y = 0 dy
It is also true that where it meet y – axis, x = 0 = x(– 2x) + (3 – x 2) × 1 by product rule
Hence we put y = 0 into normal equation to get the dx
x – value = – 2x2 + 3 – x2
9(0) + x – 65 = 0 = 3 – 3x2
x = 65 M1 which is gradient, at point x = 2
coordinates of P (65, 0) M1 = 3 – 3(2)2
= 3 – 12 = – 9
2003 / 3 (Nov) different pattern Since we have x1, we find y1 by substituting x = 2 into
(a) Find the equation of the normal to the curve y = x (3 – x2 ) becomes
y = x2 – 4x – 12, at the point where the curve cuts y = 2(3 – 22) i.e – 2
the y- axis Thus (x1, y1) is (2, – 2) and M1 = – 9
(b) At what point does the normal meet the x– axis ? y – (–2) = –   1  (x – 2)
Solution  9
(a) Equation of normal at (x1, y1) is 1
y – y1 = m2 (x – x1) y + 2 = (x – 2)
9
where m2 = 
1
and 9y + 18 = x – 2
m1 9y – x + 20 = 0
m1 is gradient of the tangent at (x1, y1)
473
1999 / 17 (Nov) counter e.g. 1  24 
Find the equation of the normal to the curve    
y = 2x3 + x2 – 5x + 2 at the point where it cuts the 24  9 
y – axis . 4
= 
A x – 5y + 10 = 0 B 5x + y – 2 = 0 C x + 5y – 10 = 0 3
D 5x – y + 2 = 0 1 4 3
Solution But m2 = – = –( 1 ÷  ) =
m1 3 4
Equation of normal at (x1, y1) is
Substituting for m2, and (x1, y1), normal equation is
y – y1 = m2(x – x1)
3
where m2 = 
1
and y – 3 = (x – 3)
m1 4
4y – 12 = 3x – 9
M1 is the gradient of the tangent to the curve at that
4y – 3x – 3 = 0
point
y = 2x3 + x2 – 5x + 2
2004/ 2 Neco combined tangent and normal
dy
= 6x2 + 2x – 5 Find the equation of the tangent and normal to the curve
dx y = 2x2 – 5x + 3 at the point where x = 2
At the point, where it cut the y – axis Solution
x = 0  x1 = 0 Equation of tangent on a curve at point (x1, y1) is
Gradient M at x = 0 is y – y1 = m(x – x1)
6x2 + 2x – 5 = 6(0)2 + 2 (0) – 5 dy
m = – 5 i.e. m1 To proceed, we need y1 value and gradient m i.e.
dx
To get y1, substitute x = 0 into original equation Put x = 2 into y = 2x2 – 5x + 3
y = 2(0)3 + (0)2 – 5(0) + 2 y = 2(22) – 5(2) + 3
= 2 = 8 – 10 +3 i.e. 1
Thus y– 2 = –   1  (x – 0) (x1, y1) is (2, 1)
 5 Also from y = 2x2 – 5x + 3
1 dy
y–2= x = 4x – 5
5 dx
5y – 10 = x At the point (2, 1) tangent gradient m
5y – x – 10 = 0 (A) 4x – 5 = 4(2) – 5
Same as x – 5y + 10 = 0 =3
Equation of tangent y – 1 = 3(x – 2)
1992 / 3 (Nov) Good example y – 1 = 3x – 6
Obtain the equation of the normal to the curve y – 3x + 5 = 0
24 (b) Equation of normal to the curve at point (x1, y1)
y2 = 9 – at the point (3, 1) 1
6 x y – y1 = m2 (x, – x1) where m2  
Solution m1
Equation of normal at (x1, y1) is 1
y – y1 = m2(x – x1) y–1=  (x – 2)
3
where m2 = 
1
and 3y – 3 = – x + 2
m1 3y + x – 5 = 0
m1 is the gradient of the tangent at (x1, y1)
dy 2003/11b Neco Exercise 39.11
Gradient (m1) i.e of the tangent at (3, 1) What is the equation of the normal to the curve
dx
3x2 + xy + y2 – 5x = 4 at point (1, 2)?
24
y2 = 9 – is implicit and quotient rule
6 x 2003/11a (Nov) Exercise 39.12
2 ydy  (6  x)  0  24 (1)  Given that 3x2 + 2xy – 5y2 = – 5,
= 0–  
(6  x ) 2 find the (i) tangent (ii) normal at the point (– 3, 2)
dx  
2 ydy  24  2005/34 Exercise 39.13
= 0–  2
dx  (6  x )  Find the equation to the normal to the curve
y = x3 – 3x – 5 at the point (2, – 3)
dy 1  24 
 A x + 9y + 25 = 0 B x + 9y – 27 = 0

dx 2 y  (6  x) 2 
 C x – 9y – 25 = 0 D x – 9y + 27 = 0

1  24 
At (3, 1) Gradient m1 is = 
2(1)  (6  3) 2 
474
Coordinate geometry II ( Circle ) 2g = – 3 , 2f = 4 and c = – 19
Circle is the focus of points equidistant from a fixed point 3
y y g=– , f=2
P
(x, y)
P
(x, y)
2
r r y-b Substituting
C C 2
(a, b) (a, b) x-a 3 2
Radius =    2  (9)
2
 
9 101 101
x x = 4 9 = = (B)
Fig I Fig II 4 4 2
Readers to solve using completing the squares method
Fig I is a circle with centre at C(a, b) and radius r.
P(x, y) is a point on the circle 2013/27
Deductively, we can re-draw fig I to give fig II The equation of a circle is x2 + y2 – 8x + 9y + 15 = 0
and by Pythagoras rule Find its radius
CP2 = (x – a)2 + (y – b)2 1 1
r2 = (x – a)2 + (y – b)2 A 5 B 15 C 85 D 85
2 2
which is the equation of a circle
From the general form of circle equation
Solution
(x – a)2 + ( y – b)2 = r2 Coefficients of x2 and y2 are unity i.e 1
Expanding the LHS x2 + y2 – 8x + 9y + 15 = 0
x2 – 2ax + a2 + y2 – 2by + b2 = r2 Collect like terms together
x + y2 – 2ax – 2by + a2 + b2 – r2 = 0
2 x2 – 8x + y2 + 9y = – 15
If we put g = –a , f = –b and c = a2 + b2 – r Complete the squares in x and y
2 81
x2 + y2 + 2gx + 2fy + c = 0 x2 – 8x + 42 + y2 + 9y +  9  = – 15+ 16 +
Equation of a circle has coefficient of x2 and y2 unity 2 4
centre ( –g, –f) and Radius g2  f 2 c (x – 4)2 + ( y + 9 )2 = 1 
81
2 4
2000/10 (Nov) 85 1
RHS: Radius = = 85 ( C )
If the radius of the circle x2 + y2 – 6x + 4y + k = 0 4 2
is 5 units, find the constant k . Readers to try alternative method using
A – 36 B –12 C 37 D 92 Radius = g2  f 2 c
Solution
Coefficients of x2 and y2 are unity i.e 1 2012/11
x2 + y2 – 6x + 4y + k = 0 The equation of a circle is 3x2 + 3y2 + 24x –12y = 15,
Collect like terms together find its radius
x2 – 6x + y2 + 4y = – k A 2 B 3 C4 D5
Complete the squares in x and y Solution
x2 – 6x + 9 + y2 + 4y + 4 = – k + 9 + 4 3x2 + 3y2 + 24x – 12y = 15
(x – 3)2 + ( y + 2)2 =  k  13 Coefficients of x2 and y2 are NOT unity i.e 1
Divide through by 3 to make them unity
Radius =  k  13 x2 + y2 + 8x – 4y = 5
5 =  k  13 Collect like terms together
52 = – k + 13 x2 + 8x + y2 – 4y = 5
k = 13 – 25 Complete the squares in x and y
k = – 12 ( B) x2 + 8x + 16 + y2 – 4y + 4 = 5 + 16 + 4
Readers to try alternative method using (x + 4)2 + ( y – 2)2 = 25
Radius = g2  f 2 c Radius = 5 ( D )
2002/9 Neco 2002/20
What is the radius of a circle whose equation is given The coordinates of the centre of the circle
by x2 + y2 – 3x + 4y – 19 = 0 ? 2x2 + 2y2 – 4x + 12y – 7 = 0 is
A 8 B
101
C
85
D
15
E
3 3 A(–1, 3 ) B (1, –3 ) C (2, –6 ) D(–2, 6 )
2 3 4 2 Solution
Solution 2x2 + 2y2 – 4x + 12y – 7 = 0
x2 + y2 – 3x + 4y – 19 = 0 Coefficients of x2 and y2 are NOT unity i.e 1
Coefficients of x2 and y2 are unity i.e 1 Divide through by 2 to make them unity
Radius = g 2  f 2  c 7
x2 + y2 – 2x + 6y – = 0
Compare x2 + y2 – 3x + 4y – 19 = 0 2
to x2 + y2 + 2gx + 2fy + c = 0 Collect like terms together

475
7 Solution
x2 – 2x + y2 + 6y = Equation of circle with centre and radius is
2
Complete the squares in x and y (x – a)2 + ( y – b)2 = r2
7 (x – 2)2 + [ y – (–3)]2 = 22
x2 – 2x + 1 + y2 + 6y + 9 = +1+9 (x – 2)2 + ( y + 3)2 = 22
2 x – 4x + 4 + y2 + 6y + 9 = 4
2
27 x2 + y2 – 4x + 6y + 9 = 0 ( A )
(x – 1)2 + ( y + 3)2 =
2
Compare to the general form 2015/2
(x – h)2 + ( y – k)2 = r2 Points (2, 1) and (6, 7) are opposite vertices of a square
– h = – 1 and – k = + 3 which is inscribed in a circle. Find the:
h = 1 and k = –3 (a) centre of the circle (b) equation of the circle
Thus circle centre coordinates ( 1, – 3) Solution
We sketch as:
2012/11b (2 , 1 )
The equation of a circle is given by
2x2 + 2y2 – 8x + 5y – 10 = 0
Find the : (i) coordinates of the centre
(ii) radius of the circle (6 , 7 )
(iii) coordinates of P and Q, if the circle cuts The dotted line is the diameter of the circle.
the x–axis at the points P and Q Centre of the circle is the mid-point
Solution x x y1  y 2
2x2 + 2y2 – 8x + 5y – 10 = 0 = 1 2 ,
2 2
Coefficients of x2 and y2 are NOT unity i.e 1 26 1 7
Divide through by 2 to make them unity = , = ( 4, 4 )
5 2 2
x2 + y2 – 4x + y – 5 = 0 (b) To form the equation of a circle, we need centre points
2 and radius
Collect like terms together Here radius is the distance between mid-point and any of
5 the two points on the circumference
x2 – 4x + y2 + y = 5
2 Radius = (4  2) 2  (4  1) 2
Complete the squares in x and y
5 2 25 169 = 13
x2 – 4x + 22 + y2 + y +  5  = 5 + 4 + = Equation of a circle with centre and radius is
2 4 16 16
2
(x – h)2 + ( y – k)2 = r2
(x – 2)2 + ( y + 5 )2 =  13  (x – 4)2 + ( y – 4)2 = ( 13 )2
4 4 x2 – 8x + 16 + y2 – 8y + 16 = 13
Compare to the general form x2 + y2 – 8x – 8y + 19 = 0
(x – h)2 + ( y – k)2 = r2
– h = – 2 and – k = + 5 1994/6 Exercise 39.14
4 What is the radius of the circle whose equation is
h = 2 and k = – 5 x2 + y2 – 6x – 7 = 0
4
A 2 B 2 C 7 D4 E 16
(i) coordinates of centre ( 1, – 5 )
4 1994/10(Nov) Exercise 39.15
(ii) radius of the circle 13 The equation of a circle of radius 3 is
4
x2 + y2 + 10x – 8y + k = 0. Find the value of the constant k.
(iii) where the circle cuts the x–axis, y = 0
A – 50 B 18 C 32 D 41 E 50
Substituting
2x2 + 2y2 – 8x + 5y – 10 = 0 becomes 2000/18 Exercise 39.15b
2x2 + 0 – 8x + 0 – 10 = 0 Find the radius of the circle x2 + y2 + 2x – 4y – 4 = 0.
2x2 – 8x – 10 = 0 A 1unit B 2units C 3units D 5units
i.e x2 – 4x – 5 = 0 1996/8(Nov) Exercise 39.16
factorizing: x2 – 5x + x – 5 = 0 Find the coordinates of the centre of circle
x(x – 5) + 1(x – 5) = 0 2x2 + 2y2 – 16x + 14y + 5 = 0
(x + 1)(x – 5) = 0 A (8, – 7) B (– 4, 7/2) C (4, –7/2)
x = – 1 or 5 D (–4, –7/2) E(– 8, 7)
Coordinates of P and Q (– 1, 0 ) and (5, 0 )
1992/8(Nov) Exercise 39.17
2014/29 The coordinates of the centre of the circle
Find the equation of a circle with centre (2, – 3) and 3x2 + 3y2 – 12x + 24y + 17 = 0 are
radius 2 units A (6, – 12) B (– 6, 12) C (–4, 2)
A x2 + y2 – 4x + 6y + 9 = 0 B x2 + y2 + 4x – 6y – 9 = 0 D (2, –4) E(– 2, 4)
C x2 + y2 + 4x + 6y – 9 = 0 D x2 + y2 + 4x – 6y + 9 = 0
476
2002/8 Neco Exercise 39.18 Solution
Which equation shows the general equation of a circle? Let the circle equation be
x2 y2 x2 + y2 + 2gx + 2fy + c = 0
A x2 + y2 = r2 B  1
a2 b2 At (0, 0)
C x2 + y2 + 2gx + 2fy + c = 0 0+0+0+0+c=0
D xx1+ yy1 + g(x + x1) + f(y + y1) + c = 0 E y2 = 4ax. c=0
At (2, 0)
22 + 0 + 2g(2) + 0 + c = 0
Equation of circle through three points 4 + 4g + c = 0
2006/9b 4g + c = – 4 ---------(1)
The points (7, 3) , (2, 3) and (–3, 3) lie on a circle. At (3, –1)
Find the: (i) equation ; (ii) radius of the circle 32 + (-1)2 + 2g(3) + 2f(-1) + c = 0
Solution 9 + 1 + 6g – 2f + c = 0
Let the circle equation be 6g – 2f + c = –10---------(2)
x2 + y2 + 2gx + 2fy + c = 0 Substitute c = 0 into (1)
At (7, 3) 4g + c = – 4 becomes
72 + 32 + 2g(7) + 2f(3) + c = 0 4g + 0 = – 4
49 + 9 + 14g + 6f + c = 0 4g = – 4
14g + 6f + c = – 58 ---------(1) g = –1
At (2, 8) Next we substitute g and c values into (2)
22 + 82 + 2g(2) + 2f(8) + c = 0 6g – 2f + c = –10 becomes
4 + 64 + 4g + 16f + c = 0 6(–1) – 2f + 0 = –10
4g + 16f + c = – 68 ---------(2) –6 – 2f = –10
At (–3, 3) – 2f = – 4
(–3)2 + 32 + 2g(–3) + 2f(3) + c = 0 f =2
9 + 9 – 6g + 6f + c = 0 Substituting for f , g and c values into
– 6g + 6f + c = – 18 ---------(3) x2 + y2 + 2gx + 2fy + c = 0 we have
Subtract (3) from (1) x2 + y2 – 2x + 4y = 0 (D)
14g + 6f + c = – 58
– ( – 6g + 6f + c = – 18 ) Exercise 39.19
The points (0, – 6) , (3, 4) and (1, 0) lie on a circle.
20g = – 40 Find its equation
g= –2
Subtract (3) from (2) Problems based on conditions for an equation to be a circle
4g + 16f + c = – 68
– ( – 6g + 6f + c = – 18 ) 2004/13 special case
Find the possible values of the constant m , for which the curve
10g + 10f = – 50
(m+ 5)x2 + (m2 – 1 )y2 + 2x –5y + 5 = 0, is a circle
g + f = – 5 ---------(4) A 2and 3 B 2 and –3 C –2and –3 D –2 and 3
Substitute g = – 2 into (4) Solution
–2 + f = –5 One of the conditions for a curve to be a circle is that
f = –5+2 coefficient of x2 and y2 are equal
f = –3 here it is m + 5 = m2 –1
Next to get c, we substitute g and f values into (2) m2 – m – 6 = 0
4g + 16f + c = – 68 becomes factorising m2 –3m + 2m – 6 = 0
4(–2) + 16(–3) + c = – 68 m(m –3 ) + 2(m –3) = 0
– 8 – 48 + c = – 68 (m+ 2)(m – 3) = 0
c = – 68 + 56 m + 2 = 0 or m – 3 = 0
c = –12 m = – 2 or 3 (D)
Substituting for f , g and c values into 2000 /39
x2 + y2 + 2gx + 2fy + c = 0 we have If x2 + y2 – 6x + 4y – 12 = 0 can be expressed in the form
x2 + y2 – 4x – 6y – 12 = 0 (x – p)2 + (y – q)2 = r2, where p, q and r are constants, find the
(ii) to get the radius, we apply completing the squares method values of (p + q + r)
x2 + y2 – 4x – 6y – 12 = 0 A 0 B 2 C4 D6
x2 – 4x + y2 – 6y = 12 Solution
x2 – 4x + 22 + y2 – 6y + 32 = 12 +4+9 Coefficients of x2 and y2 are unity i.e 1
(x – 2)2 + ( y – 3)2 = 52 x2 + y2 – 6x + 4y – 12 = 0
Radius = 5units Collect like terms together
1993/7 (Nov) x2 – 6x + y2 + 4y = 12
Find the equation of the circle that passes through the Complete the squares in x and y
points (0, 0), (2, 0) and (3, –1) x2 – 6x + 32 + y2 + 4y + 22 = 12 + 9 + 4 i.e 25
A x2 + y2 + 2x + 4y = 0 B x2 + y2 – 2x – 4y = 0 (x – 3)2 + ( y + 2)2 = 52
C x2 – y2 + 2x – 4y = 0 D x2 + y2 – 2x + 4y = 0 Compare to (x – p)2 + (y – q)2 = r2
E x2 + y2 – 6x – 4y = 0 p + q + r = 3 + (–2) + 5 = 6 (D)
477
Equation of a circle with two(opposite) points of its The equation of a circle with centre at the origin
diameter is deducted as: (0, 0) and passes through a point P(x, y) is
y
Let the points be A(x1, y1) and B(x2, y2) as shown below
y y
P
(x, y)
r y
A(x1, y1 ) B(x2, y2 ) A(x1, y1 ) B(x2, y2 )
x
(0, 0) x

P(x , y ) P(x , y )
x x Fig III
Fig I Fig II
Assume that P(x, y) is any point on the circle, Then r2 = x2 + y2
APB being an angle in a semi-circle is a right – angle
Thus AP and PB are perpendicular 2005/8 (Dec) Neco
Hence the product of their slope equals minus one Find the equation of circle with the centre (–1, 3) which
 y  y1   y 2  y  passes through the point (3 ,5)
    = – 1 Solution
 x  x1   x 2  x  We are given circle centre, but no radius
From fig II, we got our arrangement of formula bearing r2 = (x – a)2 + (y – b)2
in mind positive and negative slopes = [ 3– (–1)]2 + (5 – 3)2
r 2 = 42 + 22
( y  y1 ) ( y 2  y )
= –1 r = 20
( x  x1 ) ( x 2  x)
Equation of circle with centre (–1, 3) and radius 20 is
Cross multiply
(y – y1)(y2 – y ) = – 1(x – x1)(x2 – x ) [ x – ( – 1 ) ]2+ ( y – 3 )2 = ( 20 )2
(x – x1)(x2 – x ) + (y – y1)(y2 – y ) = 0 (x + 1 )2 + ( y – 3 )2 = 20
x + 2x + 1 + y2 – 6y + 9 = 20
2

2000/19 x2 + y2 + 2x – 6y – 10 = 0
The end points of the diameter of a circle are
(–1, –2) and (–3, 2).Find the equation of the circle. 2000/10 Neco
A x2 + y2 – 2x + 4y = 0 B x2 + y2 – 2x – 4 = 0 What is the equation of the circle with centre (–1 , –1)and
C x2 + y2 + 4x – 1 = 0 D x2 + y2 + 4x – 21 = 0 radius 2 unit ?
Solution A x2 – y2 –2x – 2y = 0 B x2 + y2 –2x + 2y = 0
Equation of a circle with two(opposite) points of its C x + y + 2x + 2y = 4 D x2 + y2 + 2x + 2y = 0
2 2

diameter is given as: E x2 + y2 + 2x + 2y + 4 = 0


(x – x1)(x2 – x ) + (y – y1)(y2 – y ) = 0 Solution
[x – (–1)] (–3 – x) + [y – (–2)] (2 – y) = 0 Equation of circle with centre (a, b) and radius r is
(x + 1) (–3 – x) + (y + 2) (2 – y) = 0 (x – a)2 + (y – b)2 = r2
– x2 – x – 3x – 3 + y2 – 2y + 2y + 4 = 0 Here centre (–1, – 1) and radius 2 ; substituting
– x2 – y2 – 4x + 1 = 0 [ x – ( – 1 ) ]2 + [ y – (–1 )]2 = ( 2 )2
Multiply through by minus 1 (x + 1 )2 + ( y + 1 )2 = 2
x2 + y2 + 4x – 1 = 0 ( C ) x + 2x + 1 + y2 + 2y + 1 = 2
2

x2 + y2 + 2x + 2y + 2 = 2
The equation of a circle with centre at (a, b) and x2 + y2 + 2x + 2y = 0 ( D )
passes through a point P(x, y) is 1996/7
y y Find the equation of the circle with centre (1, –3)
P P
(x, y) (x, y) and radius 5
r r
A x2+y2+2x – 6y +10 = 0 B x2 + y2 +2x – 6y +5 = 0
y-b
C C
(a, b) (a, b) x-a C x2 + y2 – 2x +6y +10 = 0 D x2 + y2 – 2x + 6y + 5 = 0
E x2 + y2 – 2x + 6y + 5 = 0
Solution
x x
Fig I Fig II Equation of circle with centre (a, b) and radius r is
(x – a)2 + (y – b)2 = r2
Fig I is a circle with centre at C(a, b) and radius r. Here centre (1, – 3) and radius 5 ; substituting
P(x, y) is a point on the circle (x – 1)2 + [ y – (–3 )]2 = ( 5 )2
Deductively, we can re-draw fig I to give fig II (x – 1)2 + ( y + 3 )2 = 5
and by Pythagoras rule x – 2x + 1 + y2 + 6y + 9 = 5
2

CP2 = (x – a)2 + (y – b)2 x2 + y2 – 2x + 6y + 5 = 0 ( D )


r2 = (x – a)2 + (y – b)2
which is the equation of a circle
478
1999/ 4 (Nov) dy
Find the equation of a circle with centre (–3, 4) and (y  f ) = –(x+g)
dx
radius 4 units dy  ( x  g)
A x2 +y2 +6x – 8y –21= 0 B x2 +y2 +6x– 8y + 9 = 0 
dx y f
C x2 +y2 - 6x + 8y + 9 = 0 D x2 +y2 – 6x +8y – 21 = 0
dy  ( x1  g )
Solution At point (x1, y1) , 
Equation of circle with centre (a, b) and radius r is dx y1  f
(x – a)2 + (y – b)2 = r2 This is the gradient of the tangent at point (x1, y1)
Here centre (–3, 4) and radius 4 ; substituting LHS can be rewritten as
[x – (–3) ]2 + (y – 4 )2 = 42 y  y1

 ( x1  g )
(x + 3)2 + (y – 4 )2 = 16 x  x1 y1  f
x + 6x + 9 + y2 – 8y + 16 = 16
2
Cross multiply
x2 + y2 + 6x – 8y + 9 = 0 ( B ) y – y1(y1 + f ) = – ( x1 + g )( x – x1 )
2 2
2002/ 7 (Nov) yy1 + yf – y1 – y1 f = – (x1x – x1 – gx + gx1)
The equation of a circle with centre (–2, 5) and 2 2
yy1 + yf – y1 – y1 f = – x1x + x1 + gx – gx1
radius 3 units is
2 2
A x2 +y2 +4x – 10y +20 = 0 B x2 +y2 +4x– 10y+26 = 0 yy1 + yf – y1 – y1 f = – x x1 + x1 – gx1 + gx
C x2 +y2+4x –10y – 38 = 0 D x2 +y2 +4x –10y+39 = 0 2 2
Solution xx1 + yy1 + gx + fy = x1 + y1 + gx1 + fy1
Equation of circle with centre (a, b) and radius r is Next we add gx1 + fy1 + c to both sides
(x – a)2 + (y – b)2 = r2 xx1 + yy1 + g(x + x1) + f(y + y1) + c =
Here centre (–2, 5) and radius 3 ; substituting x12 + y12 + 2gx1 + 2fy1 + c
[x – (–2) ]2 + (y – 5)2 = 32
(x + 2)2 + (y – 5)2 = 9 Thus equation of a tangent to a circle at point (x1, y1) is
x2 + 4x + 4 + y2 – 10y + 25 = 9 xx1 + yy1 + g(x + x1) + f(y + y1) + c = 0
x2 + y2 + 4x – 10y + 20 = 0 ( A ) Example ET1
Find the equation of the tangent to circle
2003/37 (Nov) Exercise 39.20 x2 + y2 + 116x + 48y – 285 = 0 at point (2, 1) on the circle
Derive the equation of a circle centre (–1, 1) Solution
and radius 5 units. Equation of a tangent to a circle at point (x1, y1) is
A x2 +y2 +2x+2y – 23 = 0 B x2 +y2 +2x – 2y +27 = 0 xx1 + yy1 + g(x + x1) + f(y + y1) + c = 0
C x2 +y2 + 2x – 2y – 27 = 0 D x2 + y2 + 2x– 2y – 23 = 0 Comparing the given circle equation with the general form
x2 + y2 + 116x + 48y – 285 = 0
2011 /24 Neco Exercise 39.21 x2 + y2 + 2gx + 2fy + c = 0
Find the equation of a circle with the centre 2gx = 116x thus, g = 116 = 58
2
(3, –2) and radius 3
2fy = 48y thus, f = 48 = 24 and c = – 285
A x2+ y2 – 4x +6y +10 = 0 B x2 – y2 + 4x – 6y+10 = 0 2
C x2 +y2 +4x +6y + 10 = 0 D x2 + y2 – 4x –6y –10 = 0 Substituting x1= 2 , y1 =1, g = 58, f = 24 and c = – 285
E x2 + y2 – 4x – 6y – 10 = 0 into xx1 + yy1 + g(x + x1) + f(y + y1) + c = 0
2x + y + 58(x + 2) + 24(y + 1) – 285 = 0
Equation of a tangent to a circle at point (x1, y1) 60x + 25y – 145 = 0
P(x1, y1 ) 12x + 5y – 29 = 0
2002/5c
The equation of a circle is x2 + y2 – 8x + 4y + 15 = 0, find
. the equation of the tangent to the circle at point (2, –3)
Solution
Equation of a tangent to a circle at point (x1, y1) is
xx1 + yy1 + g(x + x1) + f(y + y1) + c = 0
From the general equation of a circle
Comparing the given circle equation with the general form
x2 + y2 + 2gx + 2fy + c = 0 ---------- (1) x2 + y2 – 8x + 4y + 15 = 0
Differentiating (1) implicitly x2 + y2 + 2gx + 2fy + c = 0
dy dy 2gx = – 8x thus, g = 8 = –4
2x + 2 y + 2g + 2 f + 0 = 0 
2
dx dx
2fy = 4y thus, f = 4 =2 and c = 15
Divide through by 2
2
dy dy Substituting x1= 2 , y1 = –3, g = – 4, f = 2 and c = 15 into
x + y +g+ f + 0 = 0
dx dx xx1 + yy1 + g(x + x1) + f(y + y1) + c = 0
dy dy 2x – 3y – 4(x + 2) + 2(y – 3) + 15 = 0
y + f = –(x+g) 2x – 3y – 4x – 8 + 2y – 6 + 15 = 0
dx dx
1 – 2x – y = 0
479
Example ET3 6y + 9x = 15
If 2x2 + 2y2 + x – 11y – 1 = 0 is the equation of a circle, – ( 6y – 4x = 2 )
find the equation of its tangent at point (–2, 5)
13x = 13
Solution
First we make the coefficient of x2 + y2 unity x=1
2x2 + 2y2 + x – 11y – 1 = 0 becomes Substitute x value into (2)
1 11 1 3y – 2x = 1 becomes
x2 + y2 + x  y – =0 3y – 2 = 1
2 2 2 3y = 3 thus, y = 1; P( x, y) is P( 1, 1)
Equation of a tangent to a circle at point (x1, y1) is
( ii ) To find equation of circle, we need
xx1 + yy1 + g(x + x1) + f(y + y1) + c = 0
centre points and radius
Comparing the given circle equation with the general form
Radius is distance PX
1 11 1
x2 + y2 + x  y – =0 PX = (1  1) 2  (4  1) 2
2 2 2
x2 + y2 + 2gx + 2fy + c = 0
= 4  9 = 13
1 1
2gx = x thus, g = Equation of a circle with centre (h, k)
2 4 (x – h)2 + (y – k)2 = r2
11 11 1
2fy =  y thus, f =  and c = – (x + 1)2 + (y – 4)2 = ( 13 )2
2 4 2
Substituting x1= –2, y1 = 5, g = 1/4, f = – 11/4 and c = – 1/2 into x2 + 2x + 1 + y2 – 8y + 16 = 13
xx1 + yy1 + g(x + x1) + f(y + y1) + c = 0 x2 + y2 + 2x – 8y + 4 = 0
1 11 1 2005/29 Neco
– 2x + 5y + (x – 2)  (y + 5) – = 0 Find the equation of a tangent at point (2, 2) to the circle
4 4 2 x2 + y2 = 8
Multiply through by 4 to clear fractions A x–y–4=0 B x+y–4=0 C x+y+4=0
– 8x + 20y + x – 2 – 11y – 55 – 2 = 0 D x + y2 – 4 = 0 E 2x + y2 – 8 = 0
– 7x + 9y – 59 = 0 Solution
7x – 9y + 59 = 0 Equation of a tangent to a circle at point (x1, y1) is
xx1 + yy1 + g(x + x1) + f(y + y1) + c = 0
1999/9b (Nov) Comparing the given circle equation with the general form
The equation of the tangent to a circle at P(x, y) is x2 + y2 – 8 = 0
2x–3y+1= 0. If the centre of the circle is (–1, 4), find: x2 + y2 + 2gx + 2fy + c = 0
( i ) the coordinates of P 2gx = 0 thus, g = 0
( ii ) the equation of the circle 2fy = 0 thus, f = 0
Solution c = –8
P(x, y) 2x - 3y + 1 = 0
Substituting x1= 2, y1 = 2, g = 0, f = 0 and c = – 8 into
Z
xx1 + yy1 + g(x + x1) + f(y + y1) + c = 0
2x + 2y + 0(x + 2) + 0(y + 2) – 8 = 0
2x + 2y – 8 = 0
X
(-1, 4) x + y – 4 = 0 (B)
Example ET6
Find the equation of the tangent to a circle
PX is perpendicular to PZ x2 + y2 – 3x + 5y = 0 at point (0, 0)
1 Solution
Thus gradient PZ = – Equation of a tangent to a circle at point (x1, y1) is
gradient PX
xx1 + yy1 + g(x + x1) + f(y + y1) + c = 0
From 2x – 3y + 1 = 0 Comparing the given circle equation with the general form
3y = 2x + 1 x2 + y2 – 3x + 5y = 0
2 1 2 x2 + y2 + 2gx + 2fy + c = 0
y= x + thus, gradient PZ = 2gx = – 3x thus, g =  3
3 3 3 2
3 2fy = 5y thus, f= 5 and c = 0
it follows that gradient PX = –
2 2
Next we find line PX Substituting x1= 0, y1 = 0, g =  3 , f = 5 and c = 0 into
2 2
y4 xx1 + yy1 + g(x + x1) + f(y + y1) + c = 0
But gradient PX =
x  1 3 5y=0
 x +
3 2 2
y4
– = Multiply through by 2 to clear fractions
2 x  1 5y – 3x = 0
–3(x + 1) = 2(y – 4) 2003/11 Exercise 39.22
–3x – 3 = 2y – 8 P(0, 2) and Q(4, 0) are two points in a plane and 0 is the
2y + 3x = 5 ---------(1) origin. (a) Find the equation of the :
Next we solve PX and PZ simultaneously to get P( x, y) (i) circle on PQ as diameter,
PZ : 2x– 3y + 1= 0 rearranged as 3y – 2x = 1------(2) (ii) diameter parallel to the tangent to the circle at Q
(1) × 2 and (2) × 3 then subtract (b) Show that the line 2y + x + 1 = 0 is a tangent to the circle

480
Length of a tangent to a circle Example LT3
The length of a tangent from a point P(x1, y1) outside a The length of a tangent from a point (3, 2) to a circle
circle x2 + y2 + 2gx + 2fy + c = 0 as shown below x2 + y2 – 2x – 3y + k = 0 is 9 units. Find the value of k
y Solution
T 2 2
(length of a tangent)2 = x1 + y1 + 2gx1 + 2fy1 + c
The coefficient of x2 and y2 is unity in the given equation
C P(x1, y1 ) x2 + y2 – 2x – 3y + k = 0
thus 2g = –2 , 2f = –3 and c = k
2 2
x
(length of a tangent)2 = x1 + y1 + 2gx1 + 2fy1 + c
92 = 32 + 22 + (–2)× 3 + (–3)× 2 + k
PT is perpendicular to radius TC , T is the point of 81 = 9 + 4 – 6 – 6 + k
contact of the tangent and C is the centre of the circle
81 = 1 + k
PT2 = PC2 – CT2 -----------**
Let centre of the circle C be (–g , –f ) then
80 = k
PC2 = PT2 + CT2
= (x1 + g)2 + (y1 + f)2 Exercise 39.23
Find the length of the tangent to the circle
Recall that Radius = g 2  f 2  c same as TC x2 + y2 + 2x + 4y – 4 = 0 from the point (8, 10)
Substituting for TC, PC into **
PT2 = (x1 + g)2 + (y1 + f)2 – (g2+ f2 – c)
2 2
PT2 = x1 + y1 + 2gx1 + 2fy1 + c
2 2
(length of a tangent)2 = x1 + y1 + 2gx1 + 2fy1 + c

2004/4 Neco (Dec)


Find the length of the tangent from the point (5, 7) to
the circle x2 + y2 – 4x + 6y + 4 = 0
A 18 units B 16 units C 12 units
D 10 units E 5units
Solution
2 2
(length of a tangent)2 = x1 + y1 + 2gx1 + 2fy1 + c
The coefficient of x2 and y2 in the given equation
x2 + y2 – 4x + 6y + 9 = 0 is unity
thus 2g = – 4 , 2f = 6 and c = 9
(length of a tangent)2 = 52 + 72 + (–4)× 5 + 6 × 7 + 4
= 25 + 49 + – 20 + 42 + 4
length of a tangent = 100
= 10 units (D)
Example LT2
Find the length of the tangent to a circle
3x2 + 3y2 + 4x + 2y + 6 = 0 from the point (2, –1)
Solution
2 2
(length of a tangent)2 = x1 + y1 + 2gx1 + 2fy1 + c
We make the coefficient of x2 and y2 unity in the
given equation 3x2 + 3y2 + 4x + 2y + 6 = 0
4 2
x2 + y2 + x+ y+2= 0
3 3
4 2
thus 2g = , 2f = and c = 2
3 3
4 2
(length of a tangent)2 = 22 + (–1)2 + ×2 + × (–1) + 2
3 3
8 2
=4+1+ – + 2
3 3
length of a tangent = 9
= 3 units

481
Conic Different forms of parabola
The locus of a point which moves in such a way that its y2 = 4ax , y2 = – 4ax , x2 = 4ay and x2 = – 4ay
distance from a fixed point has a constant ratio to its
distance from a fixed line is called conic y2 = 4ax
y
Terminologies used in conic
Focus is a fixed point
Directrix is a fixed line
Eccentricity is a constant ratio x
Axis is the line passing through the focus and
perpendicular to the directrix
Vertex is the point where the axis meets the conic
Vertex (0, 0) ; Focus (a, 0)
Locus
Directrix x = a ; Axis y = 0
A point that moves based on a given condition(s), its
Tangent at vertex x = 0
path is called locus
y2 = – 4ax
PARABOLA (e =1) y
A parabola is the locus of a point P which moves in
such a way that its distance from a fixed point (focus) is
equal to its distance from a fixed line (directrix) .
x
ELLIPSE (e < 1)
An ellipse is the locus of a point P which moves in such
such a way that the sum of its distance from two fixed
points ( foci) is constant. Vertex (0, 0) ; Focus (–a, 0)
Directrix x = a ; Axis y = 0
HYERBOLA (e > 1) Tangent at vertex x = 0
A hyperbola is the locus of a point P which moves in
such a way that the difference of its distance from two x2 = 4ay
fixed points (foci) is constant. y

Parabola and general concepts in conic


the common term used in conic are : x

Focus (foci plural)


Vertex (vertices plural)
Directrix Vertex (0, 0) ; Focus (0, a)
Eccentricity Directrix y = a ; Axis x = 0
Lactus rectum Tangent at vertex y = 0
From the diagram of the parabola given below, we get a
clear picture of their positions along with some
explanations. x2 = – 4ay
y y
)
,y
P( x
U
(-a , y) S x

V x
N (-a , 0) (0 ,0) f(a , 0) Vertex (0, 0) ; Focus (0, – a)
Directrix y = a ; Axis x = 0
Tangent at vertex y = 0
T
W
General equation of parabola
UP = FP and ratio of point f (a,0) is called the focus. The equation of a parabola with given focus coordinates and
The chord ST passing through focus f (a,0) is called directrix equation is
lactus rectum
ax  by  c
= ( x  x1 ) 2  ( y  y1 ) 2
2 2
a b

482
Example PAR 1 Lactus rectum
Find the equation of a parabola whose focus (4, 5) and The chord of a parabola which passes through the focus and
directix is x – 8y + 5 = 0 perpendicular to the axis of the parabola
y
Solution
ax  by  c P (a , 2a)
equation is = ( x  x1 ) 2  ( y  y1 ) 2
2 2
a b
R
x  8y  5 2 2
x
= ( x  4)  ( y  5)
12  (8) 2 T (a , -2a)

x  8y  5 PRT is lactus rectum


= ( x  4) 2  ( y  5) 2 Extremities of lactus rectum are
65 P(a, 2a) and T(a, –2a) quoted as (a, 2a) and (a, –2a)
Taking the square of both sides
Tangent and Normal to a parabola
( x  8 y  5) 2 Equation of tangent at (x1, y1) to a parabola y2 = 4ax is
= ( x – 4)2 + ( y – 5)2
65 yy1 = 2a(x + x1) or
( x – 8y + 5)2 = 65( x2 – 8x + 16 + y2 – 10y +25) y 2a
=
x2 – 8xy + 5x – 8xy + 64y2 – 40y + 5x – 40y +25 = ( x  x1 ) y1
65x2 – 520x + 1040 + 65y2 – 650y +1625 Equation of Normal at (x1, y1) to a parabola y2 = 4ax is
64x + y + 16xy – 530x – 570y + 160 = 0
2 2
y
y – y1 = – 1 (x – x1) or
2a
y  y1 y
Example PAR 2 = – 1
Find the equation of a parabola whose directix is x  x1 2a
y
2x + 4y = 3 and focus (2, 5)
ent
Solution Tan
g

ax  by  c

No
( x  x1 ) 2  ( y  y1 ) 2

rm
equation is =

al
2 2
a b x

2x  4 y  3
= ( x  2) 2  ( y  5) 2
2 2
2 4 Example TN type I
2x  4 y  3 Find the equation of the tangent and normal to the parabola
= ( x  2) 2  ( y  5) 2 y2 = 9x at the point (4, 6)
20 Solution
Taking the square of both sides y2 = 9x
(2 x  4 y  3) 2 Compare to the general form of parabola y2 = 4ax
= ( x – 2)2 + ( y – 5)2  4ax = 9x
20 9
(2x + 4y – 3)2 = 20( x2 – 4x + 4 + y2 – 10y +25) a=
4
4x2 + 8xy – 6x + 8xy + 16y2 – 12y =
Tangent at (4, 6)  yy1 = 2a(x + x1)
20x2 – 80x + 80 + 20y2 – 200y +500
16x – 16xy + 4y2 – 68x – 176y + 491 = 0
2 9
y(6) = 2 ( ) (x + 4)
491 4
4x2 + y2 – 4xy – 17x – 44y + =0 9
4 6y = (x + 4)
2
3
2y = (x + 4)
Exercise 39.24 2
Find the equation of a parabola whose directix is 4y = 3x + 12
3x – 4y + 5 = 0 and focus (5, 1) 4y – 3x – 12 = 0
y  y1 y
Normal at (4, 6)  = – 1
x  x1 2a
y6 9 9
= – 6 ÷ 2( ) = – 6 ÷
x4 4 2
y6 12 4
= – = –
x4 9 3
3(y – 6 ) = – 4(x – 4)
3y – 18 = – 4x + 16
3y + 4x – 34 = 0
483
2001/11 Neco Exercise 39.25 Hyperbola
The equation of the tangent to the parabola y2 = 4(x + 2) A hyperbola is the locus of a point moving in a plane such
at point (2, –2) is that its distance from two fixed points (foci) have a constant
A y–x–4=0 B y–x=0 C y–x+4=0 difference
D y+x–4=0 E y+x=0
Hyperbola standard form equation
Example TN type II x2 y2
 1
Find the equation of the tangent and normal at point a 2
b2
(3, 4) on the parabola x2 + 4x – 8y + 28 = 0 Where b2 = c2 – a2
Solution y
x2 + 4x – 8y + 28 = 0
Differentiating implicitly P(x , y)

dy
2x + 4 – 8 + 0=0
dx
dy 2x  4 F1(c , o)
x
= F2(-c , o) V2(a , 0) V1(a , 0)
dx 8
dy 2(3)  4 10 5
(3, 4) = = =
dx 8 8 4
Tangent at (3, 4) on parabola in slope form is The locus condition of hyperbola can be represented by the
y  y1 diagram above as
= m
x  x1 PF1 – PF2 = constant
y4 5
= Characteristics of hyperbola
x3 4 A hyperbola has a pair of asymptotes
4(y – 4) = 5(x – 3)
4y – 16 = 5x – 15
4y – 5x – 1 = 0
Tangent and Normal to a hyperbola
Equation of tangent at (x1, y1) to a hyperbola
Tangent at (3, 4) on parabola is
x2 y2
y  y1 1   1 is
= – a2 b2
x  x1 m xx1 yy1
y4 4 2
 1
= – a b2
x3 5 Equation of Normal at (x1, y1) to a hyperbola
5(y – 4) = – 4(x – 3) x2 y2
5y – 20 = – 4x + 12 2
  1 is
a b2
5y + 4x – 32 = 0
a2xy1 + b2x1y = (a2 + b2)x1y1
2013/27 Neco Exercise 39.26
Example H1
Find the equation of the normal at (1, 2/3 ) on the
Find the equation of the hyperbola with foci (3, 0) and
parabola 3x2 + x – 3y – 2 = 0
A 3y – 7x + 5 = 0 B 3y + 7x – 9 = 0 C 21y – 9x –5= 0 vertex (2, 0)
D 21y + 9x – 23 = 0 E 3y – x + 2 = 0 Solution
Apply the standard form of hyperbola
2010/13 Neco Exercise 39.27 x2 y2
2
 1
Find the equation of the tangent at (2, 3 ) on the a b2
parabola x2 + x – 4y + 4 = 0 a = 2, b = ? but c = 3
A 4y – 5x – 2 = 0 B 4y – 5x – 12 = 0 C 4y + 5x +2 = 0 From b2 = c2 – a2 (hyperbola)
D 4y + 5x – 2 = 0 E 8y – 10x – 2 = 0 b2 = 9 – 4 i.e 5
x2 y2
Substituting  1
4 5
It may be rewritten as 5x2 – 4y2 = 20

Example H2
Find the vertices and foci of the hyperbola
16x2 – 4y2 = 64
Solution
2 2
The canonical form of hyperbola is x 2  y 2  1
a b

484
Presenting the given equation in canonical form, we a2 = 2 and b2 = 1
make RHS unity (i.e 1) a= 2
16 x 2 4 y 2 64 2b 2
  Substituting: length of lactus rectum =
64 64 64 a
x 2
y 2
2 1
 1 =
4 16 2
x2 y2 2 2
Comparing it with  1 Rationalizing the surd =
a2 b2 2
a2 = 4 thus a = 2 = 2 = 1.414
b2 = 16 thus b = 4
Vertices are (a, 0) = (2, 0)
Foci are (c, 0)
From b2 = c2 – a2 Ellipse
b2 + a2 = c2 Ellipse (ellipse is a closed curve )
16 + 4 = c2 Canonical or standard form (x1, y1)
c =  20 Ellipse with centre at (x1, y1) other than the origin (0,0) has
thus, foci are (  2 5 , 0) canonical from equation as
( x  x1 ) 2 ( y  y1) 2
  1 with x - major axis
a2 b2
Hyperbola equations y

Most of the hyperbola equations are gotten by negating


b2 in corresponding ellipse equation x
0, 0

Ellipse Hyperbola
Equation x 2
y 2
x2 y2
with centre 2
 2
1 2
 1 ( x  x1 ) 2 ( y  y1) 2
a b a b2   1 with y - major axis
at zero b2 a2
y
Equation of xx1 yy1 xx1 yy1
with tangent  1  =1
a2 b2 a2 b2
at ( x1, y1)
Equation of y  y1 a 2 y1 y  y1 a2 y
normal at   2 1 x
x  x1 b 2 x1 x  x1 b x1 0, 0
point (x1, y1)
Equation ( x  x1 ) 2

( y  y1) 2
1
( x  x1 ) 2

( y  y1) 2
1
with centre a2 b2 a2 b2 Where a2 determine where the ellipse lies if a is under x2
at ( x1, y1) it lies along x-axis, if a is under y2 it lies along y- axis and a
x-major axis is differentiated from b numerically by
Eccentricity e2 = 1 − b2 e2 = 1 + b2 a>b
2
a a2
Length of 2 2b 2 We can also state that Canonical form of equation of ellipse is
2b
lactus a x2 y2
a   1 here vertices is at x-axis
rectum a2 b2
x2 y2
2005/34 Neco   1 here vertices is at y-axis
b2 a2
Find the length of lactus rectum of the hyperbola where b2 = a2 – c2
x2 − 8y2 = 2 2
A 0.035 B 0.038 C 0.32 D 0.35 E 0.70 Eccentricity of the curve is e2 = 1 – b 2
Solution a

2b 2
Hyperbola length of lactus rectum = Length of the lactus rectum of ellipse
a
x2 y2 2b 2
To get b or a first, we transform the hyperbola equation   1 is
to its standard form by making the RHS 1 a2 b2 a
Here we divide through by 2
a a
x2 8y 2 2 Directrices D1 and D2 x=– and x =
  e e
2 2 2
2 e
x x2 y2
 4 y 2  1 compare to 2  2  1 a a
2 a b Foci on y-axis y = – and y =
e e

485
The general form of an ellipse equation is – x1 = +1 and – y1 = –3
Ax2 + Bx2 + Cx + Dy + K= 0 x1 = – 1 y1 = 3
different from that of a circle equation by A  B, i.e. i.e (–1, 3)
coefficient of x2 and y2 are not equal in ellipse unlike in (iii) Vertices, here we identify a and b
circle where they are equal The rule is that a > b, thus from the canonical form
7x2 + 2xy + 7y2 + 10x – 10y + 7 = 0 a2 = 9 and b2 = 4
But where it is an ellipse and coefficient of a=3 and b = 2
x 2 and y2 are equal then coefficient of xy is not zero 2
Vertical axis vertices (along y – axis ) take ( y 23)
Characteristics of ellipse 2
General cases of ellipse as oppose to circle equation is x value out of (–1, 3) is constant i.e –1
that coefficient of x2 and y2 are not same in ellipsesay (–1, y1 + b) and (–1, y1– b) recall that b is ±2
25x2 + 4y2 – 50x –16y –59 = 0 (–1, 3+2) and (–1, 3–2)
Where the circle form of it theoretically is (–1, 5) and (–1, 1)
25x2 + 25y2 – 50x –16y – 59 = 0 2

But a cases of ellipse where the coefficient of x2 and y2 Horizontal axes vertices (along x-axis) take ( x 21)
3
are same there must be another item xy whose y1 value out of (–1, 3) is constant i.e 3
coefficient is not zero theoretically the last equation (x1+a, 3) and (x1–a, 3) recall that a is ± 3
ceases to be a circle by introduction of xy (–1+3, 3) and (–1–3, 3)
25x2 + 25y2 – 5xy – 50x –16y – 59 = 0 (2, 3) and (–4, 3)
Example EL1
The graphical sketching is shown below
If the general form of an ellipse equation is
y
4x2 + 9y2 + 8x – 54y + 49 = 0 ,then
5
( i) its canonical from is
4
(ii) its coordinates at the centre are
3
(iii) its four vertices or intercepts are 2
(iv) its two foci are 1
(v) its equation of directrices x
(vi) length of lactus rectum -5 -4 -3 -2 -1 0 1 2 3 4 5
-1
Solution
To convert the general form of ellipse equation to canonical,
we target a full quadratic equation in (iv) To find the foci for non zero origin ellipse ,
x2 and y2 by completing the square we first identify the major axis i.e. the axis it is lying on
4x2 + 8x + 9y2 – 54y + 49 = 0 This is done by sketching or by the positions of a and b in
Collect like terms in x and y together the canonical equation. The rule is that a > b
4x2 + 8x + 9y2 – 54y = – 49 ( x  1) 2 ( y  3) 2
Simplify LHS by factoring out common factor + = 1
9 4
4(x2 + 2x) + 9(y2 – 6y ) = – 49
Next, what do we add to the bracket items to make them
in this case a is along x
perfect square Since a is under x, y value is constant
x2 +2x + 1 and y2 – 6y + 9 , thus we subtract same as follows from y, in (–1, 3) i.e. 3
4(x2 +2x + 1 –1) + 9( y2 –6y +9 – 9) = 49 Foci c is gotten from c2 = a2 – b2
4(x2 +2x +1) – 4 + 9( y2 – 6y + 9) – 81= – 49 =9–4=5
Did you notice what happened; we had to c= ± 5
isolate – 1and – 9 but not without multiplying –1 by 4 The two foci are : ( 3, c + x1 ) and (3, c – x1)
and –9 by 9
4(x2 + 2x + 1) + 9( y2 – 6y + 9) – 85 = – 49 (3, 5 +3 ) and (3, 5 –3 )
4(x + 1)2 + 9(y – 3)2 = – 49 + 85
4(x + 1)2 + 9(y – 3)2 = 36 a
To achieve canonical from RHS is 1 (v) Equation of the directrices is x = ±
e
hence we divide through by 36 Since our ellipse is along the x-axis
4( x  1) 2 9( y  3) 2 36 c
+ = 5
36 36 36 But e = 
a 3
( x  1) 2 ( y  3) 2 5
+ = 1 Thus x = ± 3 
9 4 3
(ii) The coordinates of the centre of ellipse whose origin
9 9 5
is not zero are gotten by comparing our result =± =±
5 5
( x  1) 2 ( y  3) 2
+ = 1 with
9 4
2b 2 2 4 8
( x  x1 ) 2 ( y  y1) 2 (vi) The length of the lactus rectum is = =
 1 a 3 3
a2 b2

486
Example EL2 (iii)foci for zero – origin ellipse .
Write the equation of the ellipse First, we identify the major axis the ellipse is lying on
25x2 +4y2 − 50x − 16y −59 = 0 in canonical from it is gotten by sketching or by the positions of a and b in the
hence, determine: canonical equation form.
(i) the coordinates of the centre of the ellipse In this case it is along x- axis since a2 i.e. 16 is under x.
(ii) the four vertices of the ellipse; it is also confirmed from our sketched diagram .
(iii) the two foci of the ellipse Thus foci C is gotten from b2 = a2 + c2
Solution c 2 = a 2 − b2
25x2 + 4y2 − 50x − 16y −59 = 0 = 16 − 4
Rearranging the terms c = ± 12 i.e ± 2 3
25x2 − 50x + 4y2 − 16y = 59 Since it is lying on x-axis , y is zero
25(x2 − 2x ) + 4(y2 − 4y ) = 59 Thus two foci are ( 2 3 , 0) and (− 2 3 , 0)
Applying the principle of completing the square to the a
bracket (iv) Equation of the directrices is x = ±
e
25(x2 − 2x + 1 − 1) + 4( y2 − 4y + 4 − 4) = 59 Since our ellipse is along the x-axis
25(x2 − 2x + 1) + 4( y2 − 4y + 4 ) – 25 – 16 = 59 y
25(x2 − 2x + 1) + 4( y2 − 4y + 4 ) − 41 = 59

Directrix
Directrix
25 (x − 1)2 + 4(y − 2)2 = 100 x
In canonic form RHS is 1;
25( x  1) 2 4( y  2) 2 100
+ =
100 100 100
( x  1) 2 ( y  2) 2
c 2 3
+ = 1 But e = =
4 25 a 4
2 3
Thus x = ± 4 ÷
4
1995/16 (Nov) ADAPTED CASE I 8
2 2
x= ±
x y 3
Giving the ellipse equation + =1
16 4 2b 2 2 4
Find : i its coordinate at the centre (v) The length of the lactus rectum is = =2
a 4
ii. its four vertices
iii. its two foci 1995/16 (Nov) ADAPTED CASE II
iv. its equation of directrices
x2 y2
v. length of lactus rectum Giving the ellipse equation + =1
Solution 4 16
( i ) From the given canonical equation of the ellipse , Find : i its coordinate at the centre
it has its origin at zero (0,0) ii. its four vertices iii. its two foci
(ii) Vertices ,here we identify a and b iv. its equation of directrices v. length of lactus rectum
The rule is that a > b. From the giving equation Solution
a2 = 16 thus a = ± 4 ( i ) From the given canonical equation of the ellipse ,
b2 = 4 thus b = ± 2 it has its origin at zero (0,0)
(ii) Vertices ,here we identify a and b
y2
Vertices axes vertices (along y – axis) take The rule is that a > b. From the giving equation
4 a2 = 16 thus a = ± 4
since our centre is (0, 0) b2 = 4 thus b = ± 2
x value is constant (0,2) and (0, − 2) i.e. (0,+b)and (0,b)
Vertices axes vertices (along y – axis) take y
2

x2 16
Horizontal axes vertices (along x-axis ) take since our centre is (0, 0)
16
since our centre is (0,0) x value is constant (0,4) and (0, − 4) i.e. (0,+b)and (0,b)
y value is constant (4, 0) and (−4, 0) i.e.(+a, 0) and (− a, 0) Horizontal axes vertices (along x-axis ) take x2
4
the graphical sketching is shown below
since our centre is (0,0)
y y value is constant (2, 0) and (−2, 0) i.e.(+a, 0) and (− a, 0)
4 the graphical sketching is shown below
3 y
2 4
1 3
x 2
-4 -3 -2 -1 0 1 2 3 4 1
-1 x
-2 -4 -3 -2 -1 0 1 2 3 4
-3 -1
-4 -2
-3
-4

487
(iii) foci for zero – origin ellipse a2 = 169 and b2 = 144
First, we identify the major axis the ellipse is lying on 144
it is gotten by sketching or by the positions of a and b in the Thus, e2 = 1 –
canonical equation form.
169
In this case it is along y- axis since a2 i.e 16 is under y. 25
e2 =
It is also confirmed from our sketched diagram . 169
Thus foci c is gotten from a2 = b2 + c2 25 5
c 2 = a 2 − b2 e = = (B)
169 13
= 16 − 4
Example EL6
c = ± 12 i.e ± 2 3
Obtain an equation of ellipse with vertices (7 , 0) and
Since it is lying on y-axis , x is zero
foci (4 , 0 )
Thus two foci are (0, 2 3 ) and (0, − 2 3 ) Solution
(iv) Equation of the directrices is y = ± a Here the vertices are on the x-axis, by simple coordinates
e
Since our ellipse is along the y-axis arrangement(x, y)
y Hence the ellipse equation form is
2
Directrix
x2
+ y2 = 1
a2 b
From our given data
x
a = 7, b = ? but c = 4
Recall that a2 = b2 + c2
b2 = a2 − c2
= 72 − 42
Directrix = (7− 4)(7 + 4)
= 3 × 11 = 33
c Thus, ellipse equation is
But e = = 2 3
a 4
x2 y2
Thus y = ± 4 ÷ 2 3 + = 1
4 72 33
y= ± 8 x2 y2
3 + = 1
49 33
2
2b 2 4 Example EL7
(v) The length of the lactus rectum is = =2
a 4 Obtain an equation of ellipse with vertices (0, 7) and foci
2007/50 Neco (0,  4)
The equation of an ellipse is given by Solution
x2 y2 Here the vertices are on the y- axis by simple coordinates
+ = 1 . Find the eccentricity (e)
169 144 arrangement (x, y)
Here the ellipse equation form is
4 5 5 5 4 x2 2
A B C D E + y2 = 1
13 13 12 11 7 b2 a
Solution From our given data
Electricity of an ellipse a = 7, b = ? but c = 4
c Recall that a2 = b2 + c2
e= b2 = a2 − c2
a
= 72 − 42
But recall that a2 = b2 +c2 = (7− 4)(7 + 4)
Thus c2 = a2 − b2 = 3 × 11 = 33
Thus, ellipse equation is
x2 y2 x2 y2
Comparing + = 1 with 2  2  1 to get a, b x2 2
169 144 a b + y2 = 1
= 132 – 122 33 7
c = ± 25 i.e 5 x2 y2
+ = 1
5 33 49
Thus, e = ( B)
13 2011/23 Neco
Alternatively Obtain an equation of the ellipse with vertices
Electricity of an ellipse ( 8 , 0) and foci ( 5, 0 )
2
x2 y2 y2 x2 x2 y2
e2 = 1 – b 2 A + =1 B + =1 C + =1
a 64 25 64 25 64 39
x2 y2 x2 y2 y2 x2 y2 x2
Comparing + = 1 with 2  2  1 D + =1 E= + =0
169 144 a b 64 39 64 39
488
Solution 1995/16 (Nov)
Here the vertices are on the a-axis by simple Find the equation of the normal at the point
coordinates arrangement ( x, y ) x2 y2
2 2 ( 2 2 , 2 ) to the ellipse + =1
Here the ellipse equation form is x 2 + y 2 = 1 16 4
a b A 2y + x = 3 2 B 2y + x = 4 2 C y +2x = 3 2
a = 8, b = ? but c = 5 D y − 2x = − 3 2 E 2y − x = 0
Recall that a2 = b2 + c2 Solution
b2 = a2 − c2
x2 y2
= 82 − 52 The normal to the ellipse + = 1 at point (x1, y1)
= (8 − 5)(8 + 5) 16 4
= 3 × 13 = 39 y  y1 a 2 y1
is 
Thus, ellipse equation is x  x1 b 2 x1
x2 y2
+ = 1 y 2 16( 2 )
82 39 Substituting: 
x2 y2
x 2 2 4(2 2 )
+ = 1 ( C)
64 39 y 2 2

x 2 2 1
2004/17 Neco
16x2 + 25y2 = 400 y − 2 = 2x − 4 2
The equation above is that of a/an y − 2x = −3 2 ( D ).
A circle B curve C ellipse D hyperbola E parabola
Solution Example ENT2
16x2 + 25y2 = 400 Find the equation of the normal and tangent to the ellipse
(A) Test for circle is that the coefficient of 9x2 + 81y2 = 729 at the point (−2 , 3)
x2 and y2 are equal , even if we try to simplify the Solution
equation ,it fails the text for circle. The equation of a normal at (x1, y1) to an ellipse
(B) it is a closed curve by virtual of x2 and y2 , y  y1 a 2 y1
x2 y2 
so it is not just a curve   1 is
a2 b2 x  x1 b 2 x1
(C) An ellipse has coefficient of y2 as positive .
coefficient of x2 and y2 not equal Writing the given equation in conical form by
This is an ellipse with centre at zero (origin); making the RHS to be 1
provided we go further by making RHS 1 Here we divide through by 729
16 x 2 25 y 2 400 9x 2 81 y 2 729
+ = + =
400 400 400 729 729 729
x2 y2 x2 y2
+ = 1 + = 1
25 16 81 9
(D) A hyperbola is brother to ellipse it has coefficient Substituting: a2 = 81, b2 = 9, x1 = −2 , y1 = 3
of x2 and y2 not equal but coefficient y2 is negative y 3 81(3)

Here it is positives hence not a hyperbola x  (2) 9(2)
(E) A parabola does not have x2 and y2 at same time , y 3 27
it is either only x2 or y2 .Hence it is not a parabola 
x2 2
− 2(y − 3 ) = 27(x + 2)
Exercise 39.28
− 2y + 6 = 27x + 54
Obtain an equation of the ellipse whose vertices are
27x + 2y + 48 = 0
(2, − 2), (2, 4) and whose eccentricity is 1/3
(b) The equation of tangent at (x1, y1) to an ellipse
x2 y2 xx1 yy1
  1 is 2
 1
a 2
b 2
a b2
ELLIPSE and TANGENT x(2) y (3)
Substituting:   1
The equation of tangent at (x1, y1) to an ellipse 81 9
x2 y2 xx1 yy1 2 x 3y
  1 is 2
 1   1
a 2
b a 2
b2 81 9
ELLIPSE and NORMAL Multiply through by LCM 81 to clear fractions
The equation of normal at (x1, y1) to an ellipse − 2x + 27y = 81
x2 y2 y  y1 a 2 y1 2x − 27y + 81 = 0
  1 is 
a2 b2 x  x1 b 2 x1
2003/36 Neco Exercise 39.29
restated as [a2xy1 − b2x1y = (a2 − b2) x1y1] An ellipse is a conic section with eccentricity
A e=1 B e>0 C e<1 D e≤0 E e>1
489
490
491
492
1.1 3 1.54 0.018 4.0 –1, 3 ii 2/x2–2
1.2 36 1.55 7.270 4.1 –11
1.3 1 4 a 1.56 400.4 4.2 3 or 8 iii f(x) largest domain in R
1.4 5 9 4.3 –5, –1 is x ≠ ±1, g(x) ≠ – 2
2.0 2 : 1 4.4 –1
1.5 34 55 2.1 ( 13 11, 1 11 ) 4.5 ±1/3 6.27 a bijective function
1.6 0.004 4.6 –8 b i x +3/ 4–2x
2.3 x = 1 2 , y  3
1.7 1 27 2
4.7 3
/4 b ii 5–x/ 2x–2
1.8 5 5 2.4 – 1/2 or 3/4 4.8 –1/5 b iii 1/8
1.9 1 b iv –17/2
2.5 x =  c  c 2  4ab
5.1 –3
1.10 x = –1 or 1 2b
1.11 x=–5 5.2 m = 1, n = 0
q q 2  4bp 5.3 –10 7.0 1
1.12 x = 0 or 2 2.6
1.13 1 or 2 2p 5.4 3x–5 7.1 m
1.14 6p/qr2 2.7 4 5.5 1 7.2 r
1.15 3 2.8 4, 16 5.6 1 7.3 M
1.16 10 2.9 4, 36 5.7 2x2 +5x +16 7.4 1
/3
1.17 125/288 2.10 4, 144 5.8 p = –6, q = –1 7.5 25/a
1.18 –1 2.11 a = 4 5.9 –6 7.6 i.0 ii. –a/(1– 1/2a)
1.19 5
/4 2.12 x = 3, y = 0 5.10 x2 +2x +1 7.7 1, –2
1.20 –4 2.13 x = 8, y = 2 5.11 1 7.8 0
1.21 1 2.14 - 1 and 5 5.12 –10 7.9 37/160
1.22 2–n 2.15 (2, 5) and (5, 2) 5.13 –5, –2 7.10 –3c/2c+3
11
1.23
1
2.16 (x -1)(x + 2)(x +3) 5.14 /3 7.11 Both associative and commutative
23
1.24 4 2.17 1, –2, 3 7.12 1
1.25 – 3/2 2.18 (x-3)(x-2)(x-1) 6.0 7.13 i. yes commutative
1.26 3 2.19 1
/12 ii. 0 iii – a/(1+a2)
1.27 2 2.20 a. ½ b. – 3/4 7.14 ab = b – a + ba
1.28 y = 0 or 2 2.21 –3 7.15 1+ 6
2.22 ( + )[ ( +)2-3] 6.1 both 1 – 1 and onto 7.16 2
1.29 2, 1
2.23 3x2 +10x +3 = 0 6.2 constant 7.17 i. –1/2 ii. – 3
1.30 –8/3
2.24 3 6.3 {-1, 5, 13, 49} 7.18 20
2.25 x2 –x –2 = 0 6.4 {5, 2, 1} 7.19 I and III only
2.26 2x2 +13x +12 = 0 6.5 {-3, -2, -1, 0, 1, 2, 3} 7.20 i. closed ii. commutative
1.31A 9
2.27 25/4 – m 6.6 g only iii. associative b. e = 0
1.31 1
6.7 a.i R, x ≠ – 3/5
0
2.28 5/2
1.32 log 10 a 12 = 0 (b) Not onto (c) –2<x<3/2
2.29 1
1 6.8 –2
1.33 /5 2.30  11 18
6.9 8
1.34 –1
6.10 40
1.35 5 3.0 x<–5
1.36 4
6.11 46 8.1 –14
3.1 x>7
1.37 –2 6.12  1x  x 1 2 8.2 d + 3a = 0
3.2 x >12 8.3 45
1.38 1 1/ 8 3.3 x < 12 6.13 – 49
1.39 1 1/ 2 8.4 33
13 6.14 2x2 –16x +29 8.5 9:5
1
1.40 /2 6.15 2x2 + 1
1.41 0.6532 3.4 m4 8.6 37
6.16 4
1.42 2 – 3x 3.5 x>–7 1
8.7 3n + 2
1.43 – 3/2 3.6 x  89 6.17 15 (5x+2) 8.8 920
6.18 – /4 5 8.9 324
3.7 –3  x  4 8.10 7
1.45 14 6.19 4 x  11

1.46 1 3.8 x  3 or x  – 4 2x  5 8.11 5


1.47 x2 = 8(1+ y)x 3.9 –1, 0, 1 6.20 7–x 8.12 4
1.48 27 3.10 – 6  x  3 6.21
1 x
2x
8.13 114
3.11 1
8.14 a. 14 b. 4 c. 32
6.22 /2 8.15 2
-5 0 5 3 x 1
8.16 /256
3.12 - 3 < x  6 6.23 2
3
1.50 28.36 6.24 ii –1–3y/5y–2 8.17 /2
3.13
1.51 0.05839 iii x = 2/5 8.18 78
-5 -2
1.52 0.05113 3.14 y  x , y + x  4 and y 0 6.25 3 x  5
1.53 6.7 × 10-4 8.20 255
3.15 2x + 3y  12 6.26 i 2x/1–x
493
8.21 127/7 5 6
m 12.31 7 14.12 803 m
10.3
8.22 91 3 12.32 1 or 14 14.13 41m
8.23A 128 10.4 3 12.33 20 14.14 3.8m
10.5 –1/2 12.34 8
C5 14.15 253 m
5
8.23 5+ 4 10.6  12 ,  2
3
, 3
3 3
8.24 11/2 13.1 19 14.16 1003 m
10.7 2500 , 2900 13.2 1.54m
8.25 2
/5 14.17 69.3m
10.8 7.5 0 13.3 19.0years 14.18 25m to 2 sf
 n
169
10.9 /25 13.4 15.4years
8.27 Tn = -1024 14 10.10 7
/25
14.19 35.75m
9
13.5 46.81 47years 14.20 62.76m
T7 = -1024/16384 10.11 /41 13.6 12.6 years 14.21 600
8.28 5, 10, 15… Tn = 10.12 b2/a2 13.7 20 14.22 54m
5n  
14.23 18.3m
8.29 20 ( 2  6) 7 y 3x 14.24 (i ) 731m (ii) 365m
10.14 4
13.8
8.30 4 10 14.25 24.73  25m
( 6  2)
5 10.15 13.9 5.2
9.0 C3 i.e 10 4

9.1 32 10.16 
1  6 2  13.10
31 15.1 800
15.2 650
4 4
9.2 16 15.3 1000
10.17 2x / x2  1 13.11 7.4 ≈ 7
9.3 Definition given earlier 15.4 870
10.18 1
(1 – 3 ) 13.12 67.45
9.4 P∪P=Ø 4 13.13 8 15.5 930
9.5 19. Even, odd, prime 10.19 210 or 330 13.14 8 15.6 900
20. { 1, 3, 9 } 10.20 00 or 1800 13.15 3/2 15.7 22.56m
9.6 III only 10.21 y = 2cos x 13.16 –1 15.8 1400
10.22 c (i) 3.6 (ii) 90 &1020 13.17 2.64 15.9 2300
9.7 { 5, 7, 9 }
13.18 8yrs 15.10 2200
9.8 n(P∪Q)+ n(PQ) is 7+5=12 15.11 3300
9.9 -3<x<2 PQ = {-2,- 11.1 6 3 -10 13.19 14years
13.20 3 15.12 19.90
1,0,1} 11.2 240 15.13 2650
11.3 60 13.21 8
9.10 -3 < x  3 15.14 36.61km
11.4 720 13.22 720
9.11 { x : 2 < x < 6 } 13.23 3 15.15 82 km
11.5 1120/243
9.12 {1, 6} 13.24 III only 15.16 2250
9.13 (a) 13.25 59.78 15.17 8.7km
12.0a (n – 1)! /2
A={2,4,6,8,10,12,14} 13.26 N12 15.18 a.14.9m b.53.70 c. 9.37m
12.0b 24
B = {2,3,4,6,8,12} 13.27 1.67 15.19 2360
A 12.0c 120
4, B 13.28 4.7 15.20 19km
10,
2, 8 12.0d 120
(b) 14
13.29 1.2 15.21 i. 35km ii. 19.4km
6, 12
3 12.1 720
13.30 12.00 15.22 a. 1200 b.( i )15.6km
9
12.2 50400
C
13.31 2.25 b. ( ii ) 76.30
12.3 20
( c ) A C = (A∪C)1 = 
1 1
13.32 2
/3 15.23 56km
12.4 5040
9.15 QRP1 13.33 0.7 15.24 1640
12.5 9!8!
9.16 P∪P =  12.6 2880 13.34 2.00
13.35 2.8 16.1 3333km
9.17a (a) 5 (b) 20/25 12.7 7!8!
13.36 1.8 years 16.2 i. 25858.93  25900km
9.17b 7 12.8 a. 6! bi. 2×5! bii. 2×4!
9.18 35 12.9 24 13.37 Symmetrical ii.5028.13  5030km
9.19 8 13.38 Variance 16.3 i. 13800km ii. 3129km
9.20 i. 8 ii. 5 iii. 4 13.39 mean162.5 cm 16.4 b.11600km c. 15900km
12.11 28
9.21 8 12.12 i. 30 ii. 36 median 163 16.5 i. 9769.55  9770km
9.22 3 12.13 350 13.40 8 ii. 814.13  814km/hr
9.23 28 12.14 4410 13.41 3/2
9.24 a 12.15 a. 1960 b. 525+1960 c. 525 13.42 a. 47.9
M25
U50
12.16 200 b i 45 b ii 5 17.1 1340
10-x
P22 12.17 525 14.1 133 m 17.2 650
12.18 i. 150 ii. 90 14.2 103 /3m 17.3 520
8-x x 16-x
12.19 100 14.3 403 m 17.4 700
C30 12.20 1946 3 17.5 200
12.21 455 14.4 500m
b. 7 17.6 400
12.22 5 ways 14.5 33 m
9.25a. 156 b. 111 c. 38 d. 45 12.23 9 17.7 360 – 2x – 2y
14.6 553 m
U50
12.24 165 17.8 800
B
H 14.7 27m
16 14
30 12.25 11 14.8 4cm 17.9 1030
15
7
9
12.26 62 14.9 53 m 17.10 200
65 12.27 24 2 17.11 2040
M
12.28 3 14.10 163 m 17.12 550
10.0 12/17 12.29 3 17.13 PR 12.5, PT 7.7
3
10.2 3 12.30 1/3 14.11 42.5m 17.14 16cm
494
18.1 i. 40.6 ii. 19.7 21.1 226.2cm3 24.1 0.7975 25.32 1  3 2 
 
18.2 21.93 21.2 44cm2 24.2 (a) 369.6  370 10   1 4 
18.3 65.7 21.3 55 cm2 (b) 656.4  656 25.33 x = 1 , y = 1/3
18.4 54/5 21.4 16/3cm 24.3 163.78  164 25.34 x = -2 , y = -4
18.5 mean N39.30 21.5 7cm 24.4 0.3821 25.35a |G| 7, |A| 20, |B| -3
18.6 24 21.6 66cm2 24.5 G  18 A
9 2  8 2  2
18.7 25.06 21.7 15cm2 24.6    
 23  8 1   2 8 2 
18.8 22.25 21.8 9cm 24.7   16 8  1
 
 2
 2 8 
18.9 median 35.5, mode 36.5 21.9 3.0cm 24.8 372 B 1 1  4
18.10 LCB 49.5, 21.10 i. 2.1cm ii. 14cm  
24.9 0.8581  0 0 3 
MPM 60, 64.5Median 21.11 108cm3  1 2 11 

18.11 UCB 59.5, Pr = 3 21.12 24cm2 25.0 0 10  25.35 i. 1 2  7
   
5 21.13 1:7 9  4  34  4 3 
18.12 20 21.14 18.5cm  6 4 8 
21.15 500cm3
25.1   ii.  1 0 iii. 1   9 
34   1 
  10 0  12 
 0 1 
21.16 960cm3   14  10 2   
 
19.0 21.17 125.0cm3 25.36 x = 3, y = -1 , z = 2
19.1 540 21.18 8 : 125 25.2  1 0  25.37 x = -3/2, y =7/2 , z = -3/2
0 1
19.2 Option D 21.19 384cm3   25.38 p = 3, q = -1, r = 2
19.3 21.20 60,000litres 25.3   1 12  5  25.39
 
21.21 Rectangle   4 13  17  26.0  6x
19.4 21.22 30m3
  12 13
 3 

3x  5
2
 2

19.5 frequency polygon 21.23 22.5cm 25.4  8 9
 26.1 – 3/2x –5/2
19.6 frequency against 21.24 a. 1:4:14 b. 280 c. 269cm3 7 6 
 26.2 2 + 2x
the class midpoint. 21.25 17.7cm3 26.3 6x + 2
21.26 1254cm2 25.5 10 7

19.7 Lower quartile 12 9  26.4 6x – 2

19.8 Median & percentile x2
19.9 Option C 22.1 i. DC = 6.5cm 18 12 13 26.5 45
19.10 o-give 22.2 (b) i. XY = 5.5cm ii. 910 25.6   4 x
19.11 22.3 i.QP = 5.1cm ii.PR = 8.8cm  11 14 9  26.6 Option B
19.12 iii. 880 iv. Scalene triangle 25.7 2 2 26.7 Option A
19.13 22.4 6.7cm 25.8 (-2, 3 ) 26.8 Option B
19.14 22.5 XY = 10.6cm QY = 4.5cm 25.9 – 5 26.9 Option D
22.6 AN = 9.6cm BK = 5.5cm
22.7 7cm 25.10   4 13  26.10 (6x2 + 3 ) cos (2x3 + 3x – 4)
20.1 16  3 cm2  9 10  26.11 – 8/3
22.8 b. Trapezium  
20.2 8cm 26.12 cosx – xsinx
c. Bisector of Q produced to PR 25.11  3 0 
20.3 i. x = 7/ 4 and y = 17/   26.13 1 – 6x
d. QX = 8.9cm PQR = 300  0 3 26.14 6(x2 + 1)
4 22.9 ABD = 410 CBD = 410
25.12 1 0  26.14b (2x + 5)(6x –11)
ii. 35.9cm2 AC = 9.3cm  
20.4 7cm 22.10 Radius 1.6cm 1 1  26.15 (–2x2+6x+10)/(x2+5)2
20.5 28.00m  8 9 26.16 (2x2 – 4x+6)/(x2+3)2
22.11 25.13   26.17
20.6 20.0cm 2
22.12   6  5 1  x 2
20.7 24  3 cm2
25.14 4 3 26.18 Option A
20.8 72cm2  
20.9 24cm  6 4 
26.19 2 x  6 x  8
2
23.1
20.10 4.6cm 23.2 25.15 4 1  x 4
20.11 3.1cm 23.3 25.16 1 0 1
20.12 38.2cm  0 1  26.20 
20.13 4.6cm
23.4 a scatter diagram
23.5 relationship between two groups. 25.17
 
Singular matrix
x  13 / 2
2

20.14 4.28cm 23.6 y tends to increase as x increases 26.21 Option A


25.18 3
20.15 2700 23.7 25.63 26.22 (2x4 + 9x2)/(2x2+3)3/2
25.19 8
 23.8 linear association 26.23
25.20 1
20.16 13 3/5cm 23.9 inverse association 26.24
25.21 –6
20.17 246cm 23.10 0.5 26.25 2/5
25.22 17
20.18 61cm 23.11 0.2 26.26 -1/8
25.23 –1008
20.19 62.2cm 23.12 0.57 26.27 1
25.24 –9
20.20 34.9cm 23.13 (i) 0.855 26.28 8
25.25 –4
20.21 85cm2 (ii)High positive relationship 26.29 3
20.22 6 5/12cm2 23.14 0.8545/0.8546 25.26 2 26.30 1/3
20.23 5.350cm2 23.15 25.27 2 26.31 2
20.24 12 2cm 23.16 25.28 1 or 2 26.32 –1.5
20.25 9.51cm2 23.17 (i) 0.82 25.29 3 26.33 5
20.26 17.57cm2 (ii)High positive correlation b/w 25.30  117 3 
17  26.34 41/16
20.27 8.52cm 23.18 0.46 5 2  26.35 19/4
 17 17 
20.28 82.21cm 23.19 25.31 26.36 9/4
 1 1 
20.29 15cm 23.20   26.37 0
 1 2 
495
26.38 ( -2, 10 ) 6
28.10 i 1 1 3 33.23 S
2t 3

3t 2
 5t 
814 36.5 /49
  
26.39 (-2, 13 ) x x  2 ( x  2) 2 3 2 3 36.6 3
/20
26.40 (-1/2, 11/4 ) ii 2.1451
26.41 (5, 75) 28.11 0.189 34.1 12N 37.1 1360
26.42 ( 13, 1014) 28.12 2Ln(x–2) –Ln(x–1) 34.2 8m/s 37.2 1300
26.43 (5, 75) 34.3 Momentum.
26.44 – 4/5 29.1 271/3 square units 34.4 momentum 37.4 520
26.45  2(ax  hy) 29.2 27square units 34.5 Newton Second 37.5 10.0cm
hx  hy 29.3 11sq units 34.6 12 ms-1 37.6 17cm
26.46 3/2 i.e 11/2 29.4 9 34.7 6.2ms-1 37.7 6
26.47 – 4 29.5 34.8 10.7m/s 37.8 16 right angles.
26.48 1.2cms- 1 29.6 37/12 sq units 34.9 37.9 12
26.49 6cm2/sec 29.7 – 4.5 34.10 37.10 1500
26.50 0.2cm2/sec 29.8 34.11 37.11 620
29.8b 34.12 450 37.12 9
29.9 0.69 34.13 8.66sec 37.13 4
27.0 2
x + sinx + c 34.14 i 27.06m ii 54.13m
iii 2.2s iv 4.33s 38.1 a 0.298 b 0.412 c 0.944
2 30.1  = 790
38.2 i. 0.0039 iii.0.03515
27.1 x4 + 2x2 + c 30.2  141.80 1d.p. 35.1 m = 21/47 n = 21/47 38.3 3/16
4 3 30.3 128.680  j)
35.2 2( 5 i 38.4 i. 0.48 ii. 0.25
27.2 1 + c 30.4 i 104.70 ii 28.74N 38.5 ½ or 0.5
6cos6x 35.3 – 13i + 24j
35.4 38.6 0.273
27.3 3
/2 sin2x – 2cosx + c 30.5 8.26N 10 17
38.7 0.0081
17
27.4 Loge 3 30.5b 14.94N, 92.070 or 292.070 38.8 i. 0.084 ii. 0.124 iii. 0.735
30.5c 17.18N, 173.820 or 353.820 35.5 13 170
27.5 7
/6ln(x-3) 170 38.9 i. 0.01 ii. 0.97
30.5d 2.94N, 40.030 35.6 8 38.10 i. 0.168 ii. 0.4
1
27.6 /8ln(2x4 +1) 40 3 g
3 31.1 65
27.7 /4 39.1
3 35.7 1350
27.8 2y = 4x3–x2 + 4x +3 39.2
31.2 13.86N 35.8 p  q = 101
27.9 2x2 – 3x + 3 39.3
31.3 49.41N 35.9 21
27.10 y = x2 + 7x - 18 39.4
31.4 T1 = 24.29N
27.11 –1 1
35.10 (3i + 4j) 39.5
T2 = 8.71N
27.12 18 5 39.6
27.13 1.0 2i  j 39.7 (i) 12y + 9x +135 = 0
32.1
27.14 42/3 35.11 (ii) 3y – 4x + 10 = 0
i 6.43ms-2 39.8 5x + 3y = – 7
27.15 9 5
ii S =11.20m ; t = 1.87S 39.9 2x – y + 9 = 0
27.16 1 2

3

x 1 2  3 4
3 35.12 i j 39.10 (i) y = – 2x + 6
27.17 5 5
1 2
8
 8
x  3x  9  k  33.1 12.2
(ii) y + 3 x = 6
3 33.2 126 35.13 I and II 39.11
27.18 C 39.12
84 x  1
2 33.3 200m 35.14 I, II and III are true
1 33.4 0.39Hour 35.15 p .q = p q only 39.13 9y + x + 25 = 0
27.19 /2 39.14 4
2 33.5 1 and 3 35.16 i v = (3t2 –5)i – 2tj
1  x  2  c 39.15 32
3
27.20 33.6 13m ii v =7i – 4j
3 33.7 39.15b 3units
27.21 4
/3 iii a =6ti – 4j 39.16 ( 4, – 7/2)
33.8 iv |a| = 12.17ms-2
33.9 122.5m 39.17 ( 2, – 4)
28.1 9 35.17 5i – 15j 39.18 x2+ y2+2gx+2fy+c = 0
33.10 138m 35.18 6i + 3j
28.2 1/2 33.11 40m/s 80m 39.194x2+ 4y2–142x+ 47y +138 = 0
28.3 A = 3/2 B=2 C = -1/2 35.19 i 8i – 10j – 4k 39.20 x2+y2+2x–2y –23 = 0
33.12 a 320m, b 8s
3 7 33.13 16.2m ii 180 = 13.42N 39.21 x2+y2–4x+6y+10 = 0
28.4  39.22
x  2 x 1 33.14 75m iii 1
(3i  6 j  2k )
28.5 1  3  1 33.15 20m/s, 34.64m/s, 3.46s 49 39.23 216
x x2 x 1 33.16 2.8s 35.20 a i | -3j | = 3N 39.24 16x2 + 9y2 + 24xy –
28.6 P = 3, Q = –2 33.17 225 a ii The body begins 20x – 10y + 625 = 0
28.7 P = 41/92 and Q = 5/96 33.18 66m/s2 to move in negative OY 39.25
P + Q = 17/64 33.19 1 or 7 (b) 155.20 39.26
28.8 A= -1, B = 1 , C = 3 33.20 16ms–2 36.1 P1 + P2 - P1 P2 39.27
28.9 5 33.21 42 36.2 0.03 39.28
33.22 Option E 36.3 Sample space 39.29 e < 1
3
36.4 /11
496

You might also like